You are on page 1of 672

MATHS QUEST 10

NEW SOUTH WALES


Australian curriculum edition
Stages 5.1 and 5.2
MATHS QUEST 10
NEW SOUTH WALES
Australian curriculum edition
Stages 5.1 and 5.2

KYLIE BOUCHER | DOUGLAS SCOTT | LYN ELMS

CONTRIBUTING AUTHORS
ROGER BL ACKMAN | DEBBIE KEMPFF | RUTH BAKOGIANIS
IRENE KIROFF | ROBERT CAHN | CORAL CONNOR | TOBIAS COOPER
First edition published 2014 by
John Wiley & Sons Australia, Ltd
42 McDougall Street, Milton, Qld 4064

Typeset in 12/14pt Times LT Std

© John Wiley & Sons Australia, Ltd 2014

The moral rights of the authors have been asserted.

National Library of Australia


Cataloguing‐in‐Publication data

Author: Boucher, Kylie, author.


Title: Maths quest 10 for New South Wales
Australian curriculum edition stages 5.1 and
5.2/Kylie Boucher, Douglas Scott, Lyn Elms.
ISBN: 978-1-118-67317-1 (paperback)
978-1-118-67315-7 (eBook)
Notes: Contributing authors: Roger Blackman,
Debbie Kempff, Ruth Bakogianis, Irene
Kiroff, Coral Connor, Robert Cahn, Tobias
Cooper.
Includes index.
Target audience: For secondary school age.
Subjects: Mathematics — Study and teaching
(Secondary)
Other authors/contributors: Scott, Douglas, author.
Elms, Lyn, author.
Dewey Number: 510

Reproduction and communication for educational purposes


The Australian Copyright Act 1968 (the Act) allows a maximum of one
chapter or 10% of the pages of this work, whichever is the greater, to be
reproduced and/or communicated by any educational institution for its
educational purposes provided that the educational institution (or the body
that administers it) has given a remuneration notice to Copyright Agency
Limited (CAL).

Reproduction and communication for other purposes


Except as permitted under the Act (for example, a fair dealing for the
purposes of study, research, criticism or review), no part of this book may
be reproduced, stored in a retrieval system, communicated or transmitted
in any form or by any means without prior written permission. All
inquiries should be made to the publisher.

Cover image © vic&dd/Shutterstock.com

Internal design images © vic&dd/Shutterstock.com (triangle background),


© Maxx-Studio/Shutterstock.com (challenge cube)

Illustrated by diacriTech and Wiley Composition Services

Typeset in India by diacriTech

Printed in China by
1010 Printing International Ltd

10 9 8 7 6 5 4 3 2 1
CONTENTS
Introductionix NUMBER AND ALGEBRA
About eBookPLUS xii Chapter 3 Coordinate geometry 62
Acknowledgements xiii
3A Sketching linear graphs 64
Exercise 3A 69
NUMBER AND ALGEBRA 3B Determining linear equations 71
Exercise 3B 74
Chapter 1 Indices 2
3C The distance between two points 77
1A Review of index laws 4 Exercise 3C 79
Exercise 1A 6 3D The midpoint of a line segment 80
1B Negative indices 9 Exercise 3D 82
Exercise 1B 11 3E Parallel and perpendicular lines 83
1C Combining index laws 13 Exercise 3E 88
Exercise 1C 15 Chapter review 92
Chapter review 18 Communicating — Rich task 96
Communicating — Rich task 20 Code puzzle 98
Code puzzle 22 eBookPLUS activities 99
eBookPLUS activities 23 Answers100
Answers 24
NUMBER AND ALGEBRA
NUMBER AND ALGEBRA Chapter 4 Simultaneous linear
Chapter 2 Algebra and equations and inequalities 106
equations 26 4A Graphical solution of simultaneous
linear equations 108
2A Substitution 28
Exercise 4A 112
Exercise 2A 30
4B Solving simultaneous linear
2B Adding and subtracting algebraic
equations using substitution 115
fractions 33
Exercise 4B 116
Exercise 2B 35
4C Solving simultaneous linear equations
2C Multiplying and dividing algebraic
using elimination 117
fractions 36 Exercise 4C 120
Exercise 2C 38 4D Applications of simultaneous linear
2D Solving simple equations 40 equations 123
Exercise 2D 43 Exercise 4D 125
2E Solving multi‐step equations 46 4E Solving linear inequalities 127
Exercise 2E 49 Exercise 4E 130
Chapter review 51 Chapter review 133
Communicating — Rich task 54 Communicating — Rich task 138
Code puzzle 56 Code puzzle 140
eBookPLUS activities 57 eBookPLUS activities 141
Answers 58 Answers 142
MEASUREMENT AND GEOMETRY NUMBER AND ALGEBRA
Chapter 5 Trigonometry 144 Chapter 7 Quadratic
5A Pythagoras’ theorem 146 expressions 238
Exercise 5A 150 7A Expanding algebraic expressions 240
5B Pythagoras’ theorem in three Exercise 7A 243
dimensions 153 7B Factorising expressions with
Exercise 5B 155 three terms 245
5C Trigonometric ratios 158 Exercise 7B 248
Exercise 5C 161 7C Factorising expressions with two or
5D Using trigonometry to calculate four terms 249
side lengths 163 Exercise 7C 251
Exercise 5D 165 7D Mixed factorisation 253
5E Using trigonometry to calculate Exercise 7D 254
angle size 167 Chapter review 256
Exercise 5E 169 Communicating — Rich task 260
5F Angles of elevation and Code puzzle 262
depression 172 eBookPLUS activities 263
Exercise 5F 173 Answers 264
5G Bearings 176
Exercise 5G 180 NUMBER AND ALGEBRA
5H Applications 182 Chapter 8 Quadratic
Exercise 5H 184 equations 266
Chapter review 186
8A Solving quadratic equations
Communicating — Rich task 190
algebraically 268
Code puzzle 192
Exercise 8A 271
eBookPLUS activities 193 8B Solving quadratic equations graphically 273
Answers 195 Exercise 8B 275
Chapter review 278
MEASUREMENT AND GEOMETRY
Communicating — Rich task 280
Chapter 6 Surface area and Code puzzle 282
volume 198 eBookPLUS activities 283
Answers 284
6A Area 200
Exercise 6A 204 NUMBER AND ALGEBRA
6B Total surface area 209
Exercise 6B 214
Chapter 9 Non‐linear
6C Volume 217 relationships 286
Exercise 6C 223 9A Plotting parabolas 288
Chapter review 228 Exercise 9A 292
Communicating — Rich task 232 9B Sketching parabolas 294
Code puzzle 234 Exercise 9B 297
eBookPLUS activities 235 9C Exponential functions and graphs 299
Answers 236 Exercise 9C 302

vi Contents
9D The circle 305 12E Subjective probability 422
Exercise 9D 307 Exercise 12E 422
Chapter review 308 Chapter review 424
Communicating — Rich task 310 Communicating — Rich task 430
Code puzzle 312 Code puzzle 432
eBookPLUS activities 313 eBookPLUS activities 433
Answers 314 Answers 434

MEASUREMENT AND GEOMETRY STATISTICS AND PROBABILITY


Chapter 10 Deductive Chapter 13 Univariate data 440
geometry 322 13A Measures of central tendency 442
10A Angles, triangles and congruence 324 Exercise 13A 447
Exercise 10A 329 13B Measures of spread 453
10B Similar triangles 333 Exercise 13B 455
Exercise 10B 336 13C Box‐and‐whisker plots 458
10C Quadrilaterals 340 Exercise 13C 464
13D Comparing data sets 468
Exercise 10C 345
Exercise 13D 470
10D Polygons 349
Exercise 10D 350
Chapter review 476
Chapter review 353 Communicating — Rich task 480
Code puzzle 482
Communicating — Rich task 358
eBookPLUS activities 483
Code puzzle 360
Answers 484
eBookPLUS activities 361
Answers 362
STATISTICS AND PROBABILITY
Projectsplus ICT ACTIVITY
Chapter 14 Bivariate data 490
Getting the budget in order pro-0087364
14A Variables 492
PROBLEM SOLVING Exercise 14A 494
14B Graphing bivariate data 496
Chapter 11 Problem solving I 366 Exercise 14B 499
Answers 384 14C Scatterplots and linear modelling 502
Exercise 14C 507
STATISTICS AND PROBABILITY Chapter review 511
Chapter 12 Probability 388 Communicating — Rich task 516
Code puzzle 518
12A Review of probability 390 eBookPLUS activities 519
Exercise 12A 402 Answers 520
12B Tree diagrams 406
Exercise 12B 409 STATISTICS AND PROBABILITY
12C Independent and dependent
Chapter 15 Statistics in
events 412
Exercise 12C 415
the media 524
12D Conditional probability 418 15A Populations and samples 526
Exercise 12D 419 Exercise 15A 529

Contents vii
15B Primary and secondary data 531 16C Successive discounts 585
Exercise 15B 537 Exercise 16C 586
15C Evaluating inquiry methods 16D Compound interest 588
and statistical reports 540 Exercise 16D 593
Exercise 15C 548 16E Depreciation 596
15D Statistical investigations 552 Exercise 16E 597
Exercise 15D 556 16F Loan repayments 599
Chapter review 559 Exercise 16F 601
Communicating — Rich task 564 Chapter review 603
Code puzzle 566 Communicating — Rich task 606
eBookPLUS activities 567 Code puzzle 608
Answers 568 eBookPLUS activities 609
Projectsplus ICT ACTIVITY Answers 610
Climate change pro-0100572
PROBLEM SOLVING
NUMBER AND ALGEBRA Chapter 17 Problem solving II 612
Chapter 16 Financial Answers 635
mathematics 574
Glossary 641
16A Purchasing goods 576 Index 654
Exercise 16A 577
16B Buying on terms 579
Exercise 16B 581

viii Contents
INTRODUCTION
Australian Mathematics education is entering a historic phase. A new curriculum
offers new opportunities to engage future generations of students in exciting and
challenging mathematics.
The New South Wales Mathematics Syllabus for the Australian curriculum
provides students with essential mathematical skills and knowledge through
the strands of Number and algebra, Measurement and geometry, and Statistics
and probability. The syllabus requires students to work mathematically by
demonstrating proficiency in communicating, problem solving, reasoning,
understanding and fluency.
Maths Quest 10 New South Wales Australian curriculum edition Stages 5.1 and
5.2 is specifically written and designed to meet the requirements and aspirations
of the New South Wales Mathematics Syllabus for the Australian curriculum.
The suite of resources available includes a range of engaging digital products
designed to enrich the learning experience and improve learning outcomes for
all students.
The suite of resources includes:

AssessON

MATHS QUEST 10 Maths Quest 10 New South Wales


NEW SOUTH WALES Australian curriculum edition Stages 5.1
Australian curriculum edition

Stages 5.1 and 5.2 and 5.2 is a rich collection of teaching and
learning resources within one package.
BOUCHER SCOTT ELMS
Teacher resources are available on the
eGuidePLUS.

A student text and eBookPLUS

STUDENT TEXTBOOK AND eBookPLUS


Individual pathways: All exercises within the text are carefully graded so that students
get to work at their own level. At the start of each exercise a guide to Individual
pathways is provided.

Introduction ix
Individual pathways
Questions: Questions: Questions:
1a–f, 2a–f, 3c–d, 4a–c, 5a–c, 1c–i, 2a–f, 3c–d, 4, 5a–c, 6d–j, 1e–l, 2c–i, 3, 4, 5, 6d–j, 9, 10,
6a–e, 7, 8, 10 9, 10 11, 12, 13

Activity 2-A-1 Activity 2-A-2 Activity 2-A-3


Substitution Harder substitution Tricky substitution
doc-4960 doc-4961 doc-4962

In general terms, questions are grouped to cater for students working at different
ability levels — below, at or above the expected level. Individual pathways activities
are available on eBookPLUS to provide additional opportunities for students to
engage with relevant mathematical concepts at their own level.
Working mathematically: Most exercises are organised under the headings of
Fluency, Understanding and Reasoning. Problem solving questions are contained in all
chapter reviews. In addition there are two problem solving chapters designed to
encourage students to apply their mathematical skills in non‐routine situations. Each
chapter also contains a Communicating (Rich) task that provides students with the
opportunity to investigate and explore the topic in more depth.
Interactivities: Interactivities are online dynamic
animations that help students to understand
difficult concepts.
eLessons: eLessons are online videos or
animations to elucidate concepts in ways that
are more than the teacher can achieve in the
classroom. A series of videos titled The Story of
Mathematics is available to place the
development of mathematical ideas in their
historical context and to enable students to see
the application of mathematics in the modern
world.
Engagement: A number of explicitly engaging
activities are available for each chapter. Among
them are code puzzles and online word searches,
crosswords and sudoku puzzles.
ProjectsPLUS: Two projects are introduced in the
text and provided through the online ProjectsPLUS
platform, allowing students to work in groups
collaboratively and creatively and to be monitored
by the teacher.
Language: Students are encouraged to learn and
use mathematical language in its proper context. Each chapter review contains
a Language section where students are required to demonstrate not only their fluency

x Introduction
in mathematical language but their capacity to make links between concepts. A
glossary is also provided to enhance students’ mathematical literacy.
eBookPLUS: The eBookPLUS contains the entire student text in HTML format and may
be used on any device. It also contains additional resources such as WorkSHEETs,
SkillSHEETs, Individual pathway activities, Interactivities, eLessons, ProjectsPLUS,
crosswords, word searches, sudoku puzzles, weblinks and the Story of Mathematics
video series.

assessON
assessON is an online assessment tool that
assesses and tracks students’ readiness FOR
learning, their progress AS they learn and their
levels OF achievement.
Icons in the print text and links in the eBook
indicate the most appropriate opportunities to
use assessON.

assessON provides:
•• materials that support best practice assessment
•• the capacity for the student to create their own practice assessments or homework
activities
•• the capacity for teachers to create a variety of test types for formal and informal
assessments
•• a facility for teachers to connect to their students and create groups to track and
monitor student progress
•• automatic record keeping of all assessments and homework tasks
•• results that can be viewed in relation to the requirements of the NSW Mathematics
Syllabus for the Australian curriculum
•• a one‐to‐one correspondence to the text
•• worked solutions to all questions
•• a variety of question types.

eGuidePLUS FOR TEACHERS


The eGuidePLUS contains everything that is available to students in the eBookPLUS.
In addition, eGuidePLUS contains:
•• two customisable tests per chapter in Word format
•• fully worked solutions to WorkSHEETs
•• answers for all Individual pathway activities
•• a work program and other curriculum advice in editable Word format.

Introduction xi
About eBookPLUS

Next generation teaching and learning

This book features eBookPLUS:


an electronic version of the
entire textbook and supporting
multimedia resources. It is
available for you online at the
JacarandaPLUS website
( www.jacplus.com.au ).

Using the JacarandaPLUS


website
To access your eBookPLUS
resources, simply log on to
www.jacplus.com.au using your
existing JacarandaPLUS login and enter the registration Using eBookPLUS references
code. If you are new to JacarandaPLUS, follow the three eBookPLUS logos are used throughout
easy steps below. the printed books to inform you that a
Step 1. Create a user account multimedia resource is available for the
The first time you use the JacarandaPLUS system, you will content you are studying.
need to create a user account. Go to the JacarandaPLUS
home page ( www.jacplus.com.au ), click on the button Searchlight IDs (e.g. INT-0001) give
to create a new account and follow the instructions on you instant access to multimedia
screen. You can then use your nominated email address resources. Once you are logged in,
and password to log in to the JacarandaPLUS system. simply enter the searchlight ID for that
resource and it will open immediately.
Step 2. Enter your registration code
Once you have logged in, enter your unique registration Minimum requirements
code for this book, which is printed on the inside front JacarandaPLUS requires you to use a supported
cover of your textbook. The title of your textbook will internet browser and version, otherwise you will
appear in your bookshelf. Click on the link to open your not be able to access your resources or view all
eBookPLUS. features and upgrades. Please view the complete
list of JacPLUS minimum system requirements
Step 3. View or download eBookPLUS at http://jacplus.desk.com/customer/portal/
resources articles/463717.
Your eBookPLUS and supporting resources are provided
in a chapter-by-chapter format. Simply select the desired Troubleshooting
chapter from the drop-down list. Your eBookPLUS contains • Go to the JacarandaPLUS help page at
the entire textbook’s content in easy-to-use HTML. The www.jacplus.com.au/jsp/help.jsp.
student resources panel contains supporting multimedia • Contact John Wiley & Sons Australia, Ltd.
resources for each chapter. Email: support@jacplus.com.au
Phone: 1800 JAC PLUS (1800 522 7587)
Once you have created your account, you can use the same
email address and password in the future to register any
JacarandaPLUS titles you own.
ACKNOWLEDGEMENTS
The authors and publisher would like to thank the following copyright holders, organisations and
individuals for their assistance and for permission to reproduce copyright material in this book.

Images
• © 123RF.com: 431/ewastudio, 512/Ronald Hudson • © iStockphoto.com: 32/Steve O’connor, 351/
fsachs78, 503/Vladimir Popovic • © Creatas Images: 514 • © Digital Vision/Stephen Frink: 222 •
© John Wiley & Sons Australia: 338 (left)/Photo by Renee Bryon, 474/Photo by K Tapp • Microsoft
Corporation: 590, 591/Screenshots reprinted by permission from Microsoft Corporation • © NASA:
573 • © Photodisc Inc.: 126 (lower), 150, 151, 152, 156, 157, 175, 188, 213, 272 (lower), 272 (upper),
293 (lower), 293 (upper), 338 (right), 416 (upper), 510 (upper) • Shutterstock (all images used under
licence from Shutterstock.com): 2–3/Lightspring, 8/JIANG HONGYAN, 17 (upper)/hornyak, 17 (lower)/
Kakigori Studio, 20/nmedia, 21/Lightspring, 22/totallyPic.com, 26–7/EmiliaUngur, 33/Teguh
Mujiono, 40/Vectomart, 50/Liv friis-larsen, 53/Diego Cervo, 54/davidundderriese, 55 (upper)/9lives,
55 (middle)/Photobank gallery, 55 (lower)/Dennis Cox, 56/paween, 62–3/arbit, 91/Orla, 96/Orla,
98/jossnat, 106–7/TnT Designs, 122 (upper)/Africa Studio, 122 (lower)/Margaret M Stewart, 124/
Oleksiy Mark, 126 (upper)/Edward Westmacott, 125 (left, right)/© Hydromet, 127/Christos Georghiou,
132/Elena Elisseeva, 135/Eric Isselee, 136 (upper)/Tiago Hora, 136 (lower)/FuzzBones, 137 (upper)/
Iakov Kalinin, 137 (lower)/Tepikina Nastya, 138 (upper)/Morphart Creation, 138 (lower left)/
stockshoppe, 138 (lower right)/Maksim Toome, 140/Anna Yefimenko, 144–5/CarpathianPrince, 171/
Aleix Ventayol Farrés, 172/pichayasri, 182/Matthew Cole, 190/snapgalleria, 192/Andrey Rudometov,
198–9/More Images, 226/Pablo Hidalgo, 227/M. Unal Ozmen, 231/leonello calvetti, 232 (upper)/
Poprotskiy Alexey, 232 (lower)/m.bonotto, 233/S_E, 234/(boomerang)/Richard Peterson, 234/
(football)/patrimonio designs ltd, 238–9/Ivan Pavlov, 245/Ivonne Wierink, 253/Benis Arapovic, 255/
Irina Voloshina, 257/LoopAll, 258/Chrislofotos, 260/Maxi_m, 261/wizdata1, 262/Memo Angeles,
266–7/A.B.G., 277/Azuzl, 279/Diego Barbieri, 280/Micha Klootwijk, 281/nito, 282/JBOY, 286–7/
Christos Georghiou, 288 (upper left)/gui jun peng, 288 (upper right)/Amy Johansson, 288 (lower
left)/Paul D Smith, 289 (left)/Petinov Sergey Mihilovich, 289 (right)/albund, 299/Oxlock/hutterstock.
com, 307/Godruma, 310/alean, 312/wizdata1, 322–3/Neale Cousland, 333/Dan Breckwoldt, 339/
Vjom, 348/Yulia Glam, 349/Frontpage, 352/Darios, 358/Juliana Villalobos, 359/JSlavy, 360/
Verzzh, 364/Robert Cumming, 364–5/Oleg Golovnev, 365/XuRa, 366–7/Maxx-Studio, 369/Christos
Georghiou, 370 (upper)/Yayayoyo, 370 (lower)/Efired, 371/Barone Firenze, 373/Atypeek Design,
374/Haslam Photography, 376 (upper)/Memo Angeles, 376 (lower)/photokup, 378 (upper)/Erik
Lam, 378 (lower)/block23, 379 (upper)/Neale Cousland, 379 (lower)/Worakit Sirijinda, 381 (upper)/
aarrows, 381 (lower)/Steve Heap , 382/f9photos, 388–9/jordache, 401/Cheryl Ann Quigley, 405/
Vladyslav Starozhylov, 406/Degtiarova Viktoriia, 415/Tatyana Vyc, 416 (lower)/Astronoman, 417/
Alexey Repka, 419/Alexander Yakovlev, 420/ollyy, 421/Adrian Niederhaeuser, 423/Aaron Amat,
430/Yuri Gayvoronskiy, 432/maisak dzmitry, 440–1/tovovan, 448 (upper)/Omer N Raja, 448 (lower)/
Artazum and Iriana Shiyan, 451 (upper)/spotmatik, 451 (lower)/pkchai, 452 (upper)/Andresr, 452
(lower)/3d_kot, 456/Nadezda Cruzova, 463/Madlen, 465/Marques, 466 (upper)/Ariwasabi, 466
(lower)/psynovec, 467 (upper)/Kzenon, 467 (lower)/Givaga, 471/Kzenon, 475 (upper)/Yellowj,
475 (lower)/Nik Merkulov, 477/michaeljung, 479/Skalapendra, 480/imagedb.com, 482/Matthew
Cole, 490–1/Rob Hyrons, 494/steamroller_blues, 495 (upper)/Sebastian Kaulitzki, 495 (middle)/
Ermolaev Alexander, 495 (lower)/photobank.kiev.ua, 500/AVAVA, 502/Ain Mikail, 508/nata-lunata,
509/PHOTO FUN, 510/Vectomart, 511/racorn, 515/Nagy-Bagoly Arpad, 516/Aleksandr Bryliaev, 517
(upper)/Morphart Creation, 517 (middle)/Lorelyn Medina, 517 (lower)/Pushkin, 518/zzveillust, 524–5/
Oleksiy Mark, 529/Alexander Raths, 530/homydesign, 531/Maxx-Studio, 532 (upper)/Fotyma, 532
(lower)/alexmillos, 533 (upper)/mama_mia, 533 (lower)/Meder Lorant, 537/Iakov Kalinin, 538/
sextoacto, 540/John T Takai, 549/Maria Maarbes, 550/Natali Glado, 552/B. and E. Dudzinscy,
557/Nattika, 558/fuyu liu, 559/ponsuwan, 560/James Flint, 561/Alexander Chaikin, 562 (upper)/
BlueSkyImage, 562 (lower)/Artem Shatalov , 564 (upper)/rvika, 564 (lower)/Alhovik, 566 (upper)/

Acknowledgements xiii
BlueRingMedia, 566 (lower)/elmm, 572/Lysithee, 572–3/Mariia Sats, 574–5/style_TTT, 576/Milevski
Petar, 578/kavione, 580/Rashevskyi Viacheslav, 581/Omegafoto, 582/Kuznetsov Alexey, 583 (top)/
Portfolio, 583 (upper middle)/Maridav, 583 (lower middle)/Gena73, 583 (bottom)/AlexRoz, 584/
gst, 585/Anteromite, 586/HomeArt, 587 (upper)/Altin Osmanaj, 587 (lower)/Moving Moment, 594/
karamysh, 595/Natykach Nataliia, 598 (upper)/zefart, 598 (lower)/David Malik, 599/MO_SES, 605/
Zamanbeku, 606/primusoid, 608/Igor Zakowski, 612–13/89studio, 614/Harald Høiland Tjøstheim,
616/Nattika, 617 (upper)/Sergey Lavrentev, 617 (lower)/Alliance, 620/mady70, 621/yingthun, 622/
Kzenon, 625/Givaga, 626 (upper)/Rodrigo Garrido , 626 (lower)/robootb, 627 (upper)/Andreas
berheide, 627 (middle)/Ruth Black, 627 (lower)/Julian W, 628/Dmitrydesign, 629/ampFotoStudio,
630/Nicholas Piccillo, 631/Tsekhmister, 632/Cardens Design• ©Viewfinder Australia Photo
Library: 16

Text
• AFP, 555/“Single women earn more”, The Weekend Australian 4–5 Sept 2010 p. 20 • © Copyright
Agency Limited: 546–7/“The Great Aussie Dads survey”, The Sunday Mail 5 September 2010
pp. 14–15; 548/“Sponges are toxic”, The Sunday Mail 5 September 2010 p. 36; 554–5/Article “Word
limit”, by Roland Sussex taken from The Courier Mail 14–15 August 2010 p. 25/Professor Emeritus
Roland Sussex; 556/“Egg Shortage is No Yolk”, by Paddy Hintz, The Courier Mail 28–29 August
2010 p. 13; 563/“Taste Test”, The Sunday Mail 4 April 2010 p. 26
Every effort has been made to trace the ownership of copyright material. Information that will
enable the publisher to rectify any error or omission in subsequent editions will be welcome.
In such cases, please contact the Permissions Section of John Wiley & Sons Australia, Ltd.

xiv Acknowledgements
number and
NUMBER AND ALGEBRA
algebra

Chapter 1

Indices
WHY LEARN THIS?
Don’t you wish that your money could grow as quickly
as a culture of bacteria? Perhaps it can — both
financial investments and a culture of bacteria can
grow exponentially, that is, according to the laws of
indices. Indices are useful when a number is continually
multiplied by itself, becoming very large, or perhaps
very small.

WHAT DO YOU KNOW?


1 Think List what you know about indices.
Use a thinking tool such as a concept map to
show your list.
2 pair Share what you know with a partner
and then with a small group.
3 share As a class, create a large concept map
that shows your class’s knowledge of indices.

LEARNING SEQUENCE
1A Review of index laws
1B Negative indices
1C Combining index laws
Watch this video
The story of mathematics

Searchlight ID: eles-1840


number and algebra

1A  Review of index laws


•• When a number or pronumeral is repeatedly multiplied by itself, it can be written in
a shorter form called index form.
•• A number written in index form has two parts; the base and the index, and is
written as:

base ax index
(power or
exponent)

•• Another name for an index is an exponent or a power.


•• Performing operations on numbers or pronumerals written in index form requires the
application of the index laws.
First Index Law: When terms with the same base are multiplied, the indices are
added.
a m × a n = am + n
Second Index Law: When terms with the same base are divided, the indices are
subtracted.
a m ÷ a n = am − n

WORKED EXAMPLE 1
Simplify each of the following.
2x5y4
a m4n3p × m2n5p3 b 2a2b3 × 3ab4 c
10x2y3
TH I N K W R I TE
a 1 Write the expression. a m4n3p × m2n5p3
2 Multiply the terms with the same base = m4 + 2n3 + 5p1 + 3
by adding the indices. Note: p = p1 . = m6n8p4
b 1 Write the expression. b 2a2b3 × 3ab4
2 Simplify by multiplying the = 2 × 3 × a2+1 × b3+4
coefficients, then multiply the = 6a3b7
terms with the same base by adding
the indices.
5 4
c 1 Write the expression. c 2x y
10x2y3
2 Simplify by dividing both of the 1x5−2y4−3
=
coefficients by the same factor, then 5
divide terms with the same base by x3y
=
subtracting the indices. 5

4 Maths Quest 10 New South Wales Australian curriculum edition Stages 5.1 and 5.2
number and algebra

•• Third Index Law: Any term (excluding 0) with an index of 0 is equal to 1.


a0 = 1, a ≠ 0

WORKED EXAMPLE 2
Simplify each of the following.
a (2b3) 0 b −4(a2b5) 0

TH I N K W R I TE
a 1 Write the expression. a (2b3) 0
2 Apply the Third Index Law, which states =1
that any term (excluding 0) with an index
of 0 is equal to 1.
b 1 Write the expression. b −4(a2b5) 0
2 The entire term inside the brackets has an = −4 × 1
index of 0, so the bracket is equal to 1.
3 Simplify. = −4

•• Fourth Index Law: When a power (am) is raised to a power, the indices are
multiplied.
(am) n = amn
Fifth Index Law: When the base is a product, raise every part of the product to the
index outside the brackets.
(ab) m = ambm
Sixth Index Law: When the base is a fraction, multiply the indices of both the
numerator and denominator by the index outside the brackets.
a m am
a b = m
b b

WORKED EXAMPLE 3
Simplify each of the following.
2x3 4
a (2n4) 3 b (3a2b7) 3 c a b d (−4) 3
y4
TH I N K W R I TE
a 1 Write the expression. a (2n4) 3
2 Apply the Fourth Index Law and = 21 × 3 × n 4 × 3
simplify. = 23n12
= 8n12

Chapter 1 • Indices 5
number and algebra

b 1 Write the expression. b (3a2b7) 3


2 Apply the Fifth Index Law and simplify. = 31 × 3 × a 2 × 3 × b 7 × 3
= 33a6b21
3 Simplify. = 27a6b21
3 4
c 1 Write the expression. c a 2x b
y4

2 Apply the Sixth Index Law and simplify. 21 × 4 × x3 × 4


=
y4 × 4
16x12
= 16
y
d 1 Write the expression. d (–4) 3
2 Write in expanded form. = –4 × –4 × –4
3 Simplify, taking careful note of the = –64
negative sign.

Exercise 1A Review of index laws


INDIVIDUAL PATHWAYS
Questions: Questions: Questions:
1a–f, 2a–f, 3a–f, 4a–f, 6, 7a–f, 1d–i, 2d–i, 3a–f, 4e–l, 6, 7a–f, 1d–l, 2d–l, 3, 4d–o, 5, 6, 7d–i,
9, 10 9, 10, 11 8, 9, 10, 11, 12
Activity 1-A-1 Activity 1-A-2 Activity 1-A-3
Reviewing index operations Practising the index laws Applying the index laws
doc-4948 doc-4949 doc-4950

FLUENCY
Digital docs
1 WE1a, b Simplify each of the following.
SkillSHEET a ×
a3 a4 b a2 × a3 × a c b × b5 × b2
Index form
doc‐5168 d ab2 × a3b5 e m2n6 × m3n7 f a2b5c × a3b2c2
SkillSHEET 1
Using a calculator to g mnp × m5n3p4 h 2a × 3ab i 4a2b3 × 5a2b × 2b5
evaluate numbers given
1 1
in index form j 3m3 × 2mn2 × 6m4n5 k 4x2 × 2xy3 × 6x3y3 l 2x3y2 × 4x × 2x4y4
doc‐5169

2 WE1c Simplify each of the following.


a a4 ÷ a3 b a7 ÷ a2 c b6 ÷ b3
4a7 21b6 48m8
d e f
3a3 7b2 12m3
m7n3 2x4y3
g h i 6x7y ÷ 8x4
m4n2 4x4y
20m5n3p4 14x3y4z2
j 7ab5c4 ÷ ab2c4 k l
16m3n3p2 28x2y2z2

6 Maths Quest 10 New South Wales Australian curriculum edition Stages 5.1 and 5.2
number and algebra

3 WE2 Simplify each of the following.


a a0 b (2b) 0 c (3m2) 0
d 3x0 e 4b0 f –3 × (2n) 0
0
a
g 4a0 − a b h 5y0 − 12 i 5x0 − (5xy2) 0
4
4 WE3 Simplify each of the following.
m2 4
a (a2) 3 b (2a5) 4 c a b
3
2n4 2
d a b e (a2b) 3 f (3a3b2) 2
3
3m2n 3 a2 2
g (2m3n5) 4 h a b i a b
4 b3
5m3 4 7x 3 3a 4
j a b k a b l a b
n2 2y5 5b3
m (–3) 5 n (–7) 2 o (–2) 5

5  MC a 2m10n5 is the simplified form of:


6m10n4
A m5n3 × 2m4n2 B C (2m5n2) 2
3n
2m5 2
D 2n(m5) 2 × n4 E a b
n3
b The value of 4 − (5a) 0 is:
A –1 B 9 C 1
D 3 E 5
6 MC a 4a b × b × 5a b simplifies to:
3 4 2 3

A 9a5b8 B 20a5b7 C 20a5b8


D 9a5b7 E 21a5b8
15x9 × 3x6
b simplifies to:
9x10 × x4
A 5x9 B 9x C 5x29
D 9x9 E 5x
3p7 × 8q9
c simplifies to:
12p3 × 4q4
p4 q4 q4
A 2q4 B C
2 2
p4 q4 q4
D E
24 24
7a5b3 7b3a2
d ÷ simplifies to:
5a6b2 5b5a4
49a3b 25a3b
A B C a3 b
25 49
25ab3
D ab3 E
49

Chapter 1 • Indices 7
number and algebra

understanding
7 Evaluate each of the following.
a 23 × 22 × 2 b 2 × 32 × 22 c (52) 2
35 × 46 3 3
d e (23 × 5) 2 f a b
34 × 44 5
44 × 56
g h (33 × 24) 0 i 4(52 × 35) 0
43 × 55
8 Simplify each of the following.
a (xy) 3z b ab × (pq) 0 c ma × nb × (mn) 0
a2 x n3m2
d a b e f (am + n) p
b3 npmq
REASONING
9 Explain why a3 × a2 = a5 and not a6.
10 Is 2x ever the same as x2? Explain using examples.
11 Explain the difference between 3x0 and (3x) 0.
12 a In the table below, enter the values of 3a2 and 5a when a = 0, 1, 2 and 3.

a 0 1 2 3
3a2
5a
3a2 + 5a
3a2 × 5a

b Enter the values of 3a2 + 5a and 3a2 × 5a in the


table. REFLECTION
Why are these laws called
c What do you think will happen as a becomes very index laws?
large?

CHALLENGE 1.1

8 Maths Quest 10 New South Wales Australian curriculum edition Stages 5.1 and 5.2
number and algebra

1B  Negative indices


3
a
•• Consider the expression . This expression can be simplified in two ways.
a5
1. Written in expanded form a =
3 a×a×a Interactivity

a 5 a × a ×a×a×a Colour code breaker


int‐2777
1
=
a×a
1
= 2
a
a 3
2. Using the Second Index Law, = a3−5
a5
= a−2
1
So, a−2 = 2 .
a
0
1 a
•• In general, n = n (1 = a0)
a a
= a0−n (using the Second Index Law)
= a−n
1
Seventh Index Law: a−n = n
a
•• The convention is that an expression should be written using positive indices.

WORKED EXAMPLE 4
Express each of the following with positive indices.
4
a x −3 b 2m −4n2 c
a −3
TH I N K W R I TE
a 1 Write the expression. a x −3
2 Apply the Seventh Index Law. 1
=
x3
b 1 Write the expression. b 2m −4n2
2 Apply the Seventh Index Law to write the 2n2
= 4
expression with positive indices. m
c 1 Write the expression and rewrite the fraction, c 4 = 4 ÷ a −3
using a division sign. a −3
2 Apply the Seventh Index Law to write the 1
  = 4 ÷ 3
expression with positive indices. a
3 Change fraction division into multiplication a3
=4×
and simplify.    1
= 4a3

•• Part c from Worked example 4 demonstrates the converse of the Seventh


1
Index Law −n = an.
a

Chapter 1 • Indices 9
number and algebra

WORKED EXAMPLE 5
Simplify each of the following, expressing the answers with positive indices.
2x4y2 2m3 −2
a a2b −3 × a −5b b c ¢ b
3xy5 n −2
TH I N K W R I TE
a 1 Write the expression. a a2b −3 × a −5b
2 Apply the First Index Law. Multiply terms with = a2 + −5b −3 + 1
the same base by adding the indices. = a −3b −2
3 Express the answer with positive indices. 1
= 3 2
ab
4 2
b 1 Write the expression. b 2x y
3xy5
2x4 − 1y2 − 5
2 Apply the Second Index Law. Divide terms with =
the same base by subtracting the indices. 3
2x3y −3
=
3
2x 3
3 Express the answer with positive indices. = 3
3y
3 −2
c 1 Write the expression. c a 2m b
n −2
2 Apply the Sixth Index Law. Multiply the indices 2 −2m −6
=
of both the numerator and denominator by the n4
index outside the brackets.
3 Express all terms with positive indices. 1
=
22m6n4
4 Simplify. 1
=
4m6n4
•• Numbers in index form can be easily evaluated if they are expressed with positive
indices first. Consider the following example.
WORKED EXAMPLE 6
Evaluate 6 × 3 −3 without using a calculator.

TH I N K W R I TE
1 Write the multiplication. 6 × 3 −3
Apply the Seventh Index Law to write 3−3 with a 1
2 =6×
positive index. 33
3 Multiply the numerator of the fraction by the whole 6
=
number. 33
Evaluate the denominator. 6
4 =
27
5 Cancel by dividing both the numerator and 2
=9
denominator by the Highest Common Factor (3).

10 Maths Quest 10 New South Wales Australian curriculum edition Stages 5.1 and 5.2
number and algebra

Exercise 1B Negative indices


INDIVIDUAL PATHWAYS
Questions: Questions: Questions:
1a–i, 2a–i, 3a–f, 4, 5a–e, 1a–i, 2a–i, 3c–h, 4, 5a–g, 6, 7, 1, 2c–o, 3c–l, 4, 5d–j, 6, 7,
6a, b, 8a–c, 9, 11a, 12 8b–e, 9, 11a–b, 12, 13 8c–f, 9, 10, 11, 12, 13
Activity 1-B-1 Activity 1-B-2 Activity 1-B-3
Negative indices Harder negative indices Tricky negative indices
doc-4951 doc-4952 doc-4953

FLUENCY
1 WE4 Express each of the following with positive indices.
a x−5 b y−4 c 2a−9
4 −3
d 5
a e 3x2y−3 f 2−2m−3n−4
1 2
g 6a3b−1c−5 h i
a−6 3a−4
6a 7a−4 2m3n−5
j k l
3b−2 2b−3 3a−2b4
2 WE5 Simplify each of the following, expressing the answers with positive indices.
a a3b−2 × a−5b−1 b 2x−2y × 3x−4y−2 c 3m2n−5 × m−2n−3
d 4a3b2 ÷ a5b7 e 2xy6 ÷ 3x2y5 f 5x−2y3 ÷ 6xy2
6m4n 4x2y9 2m2n−4
g h i
2n3m6 x7y−3 6m5n−1
j (2a3m4) −5 k 4(p7q−4) −2 l 3(a−2b−3) 4
2p2 −3 a−4 2 6a2 −3
m a b n a b o a b
3q3 2b−3 3b−2
3 WE6 Evaluate each of the following without using a calculator.
a 2−3 b 6−2 c 3−4
d 3−2 × 23 e 4−3 × 22 f 5 × 6−2
6 4 × 3−3 1
g h i × 5−2 × 34
2−3 2−3 3
160 × 24 53 × 250 34 × 42
j k l
82 × 2−4 252 × 5−4 123 × 150
4 Write each of these numbers as a power of 2.
1 1
a 8 b c 32 d
8 64
5 Complete each statement by writing the correct index.
1 1
a 125 = 5 … b = 4… c = 7… d 216 = 6… e 0.01 = 10…
16 7
1 1 1
f 1 = 8… g 64 = 4… h = 4… i = 2… j = 8…
64 64 64

Chapter 1 • Indices 11
number and algebra

6 Evaluate the following expressions.


2 −1 5 −1 1 −1 1 −1
a a b b a b c a3 b d a b
3 4 2 5
7 Write the following expressions with positive indices.
a −1 a2 −1 a−2 −1 m3 −1
a a b b a b c a b d a b
b b3 b−3 n−2
8 Evaluate each of the following, using a calculator.
a 3−6 b 12−4 c 7−5
1 −8 3 −7
d a b e a b f (0.04) −5
2 4
9 MC a x−5 is the same as:
1 1
A −x5 B −5x C 5x D E
x5 x−5
1
b is the same as:
a−4
1
A 4a B −4a C a4 D E −a4
a4
1
c is the same as:
8
1
A 23 B 2−3 C 32 D 3−2 E
2−3
3m4
10  MC a Which of the following, when simplified, gives ?
4n2
3m−4n−2 3n−2
A B 3 × 2−2 × m4 × n−2 C
4 2−2m−4
2 −2
2n
D E 3m4 × 22n−2
3−1m−4
3
b When simplified, 3a−2b−7 ÷ a−4b6 is equal to:
4
4 9b 9a2
A B C
a6b13 4a6 4b
4a2 4a2
D E
b13 b
c When (2x6y−4) −3 is simplified, it is equal to:
2x18 x18 y12
A B C
y12 8y12 8x18
8y12 x18
D E
x18 6y12
2ax 3 8b9
d If a b is equal to , then x and y (in that order) are:
by a6
A −3 and −6 B −6 and −3 C −3 and 2
D −3 and −2 E −2 and −3

12 Maths Quest 10 New South Wales Australian curriculum edition Stages 5.1 and 5.2
number and algebra

UNDERSTANDING
11 Simplify, expressing your answer with positive indices.
m −3n −2 (m3n −2) −7 5(a3b −3) 2 (5a −2b) −1
a b c ÷
m −5n6 (m −5n3) 4 (ab −4) −1 (a −4b) 3
REASONING
12 Using numerical examples, explain the difference between −2x and 2x−2.
13 Explain why each of these statements is false. Illustrate
each answer by substituting a value for the pronumeral. REFLECTION
a 5x = 1
0
b 9x ÷ 3x = 3x
5 5 Do any of the index laws
from exercise 1A not apply Digital doc
1
d 2c−4 =
WorkSHEET 1.1
c a 5 ÷ a 7 = a2 to negative indices? doc‐13695
2c4

1C Combining index laws


•• When several steps are needed to simplify an expressions, expand the brackets first.
•• Where fractions are involved, it is usually easier to carry out all multiplications first,
leaving only one division as the final process.
•• Final answers are conventionally written with positive indices.

WORKED EXAMPLE 7
(2a) 4b4
Simplify .
6a3b2
TH I N K W R I TE
1 Write the expression. (2a) 4b4
6a3b2
2 Apply the Fourth Index Law to remove the 16a4b4
=
bracket. 6a3b2

3 Apply the Second Index Law for each number 8a4−3b4−2


=
and pronumeral to simplify. 3
4 Write the answer. 8ab2
=
3

WORKED EXAMPLE 8
Simplify each of the following.
7xy3 2m5n × 3m7n4
a (2a3b) 4 × 4a2b3 b c
(3x3y2) 2 7m3n3 × mn2
TH I N K W R I TE
a 1 Write the expression. a (2a3b) 4 × 4a2b3
2 Apply the Fourth Index Law. Multiply = 24a12b4 × 4a2b3
each index inside the brackets by the index
outside the brackets.

Chapter 1 • Indices 13
number and algebra

3 Evaluate the number. = 16a12b4 × 4a2b3


4 Multiply coefficients and multiply = 16 × 4 × a12 + 2b4 + 3
pronumerals. Apply the First Index Law = 64a14b7
to multiply terms with the same base by
adding the indices.
b 7xy3
b 1 Write the expression.
(3x3y2) 2
2 Apply the Fourth Index Law in the 7xy3
=
denominator. Multiply each index inside the 9x6y4
brackets by the index outside the brackets.
3 Apply the Second Index Law. Divide terms 7x −5y −1
with the same base by subtracting the indices. =
9
4 Use a−m = 1 to express the answer with 7
= 5
am 9x y
positive indices.
c 1 Write the expression. c 2m5n × 3m7n4
7m3n3 × mn2
2 Simplify each numerator and denominator 6m12n5
=
by multiplying coefficients and then terms 7m4n5
with the same base.
3 Apply the Second Index Law. Divide terms 6m8n0
=
with the same base by subtracting the indices. 7
4 Simplify the numerator using a0 = 1. 6m8 × 1
=
7
6m 8
=
7

WORKED EXAMPLE 9
Simplify each of the following.
( 5a2 b3) 2 a2 b5 8m3 n −4 4m −2 n −4
a × b ÷
a10 (a3 b) 7 (6mn2) 3 6m −5 n

TH I N K W R I TE
a 1 Write the expression. a (5a2 b3) 2 a2 b5
×
a10 (a3 b) 7
2 Remove the brackets in the numerator of the 25a4 b6 a2 b5
= × 21 7
first fraction and in the denominator of the a10 a b
second fraction.
3 Multiply the numerators and then multiply 25a6b11
=
the denominators of the fractions. (Simplify a31b7
across.)

14 Maths Quest 10 New South Wales Australian curriculum edition Stages 5.1 and 5.2
number and algebra

4 Divide terms with the same base by = 25a −25b4


subtracting the indices. (Simplify down.)
5 Express the answer with positive indices. 25b4
=
a25
b 1 Write the expression. b 8m3n −4 4m −2n −4
÷
(6mn2) 3 6m −5n
2 Remove the brackets. 8m3n −4 4m −2n −4
= ÷
216m3n6 6m −5n
3 Change the division to multiplication. 8m3n −4 6m −5n
= ×
216m3n6 4m −2n −4
4 Multiply the numerators and then multiply 48m −2n −3
the denominators. (Simplify across.) =
864mn2
5 Cancel common factors and divide pronumerals m −3n −5
with the same base. (Simplify down.) =
18
6 Simplify and express the answer with 1
=
positive indices. 18m3n5
Note that the whole numbers in part b of Worked example 9 could be cancelled in step 3.

Exercise 1C Combining index laws


INDIVIDUAL PATHWAYS
Questions: Questions: Questions:
1a–d, 2a–d, 3a–c, 4a–c, 5a, b, 1, 2c–f, 3c–f, 4b–d, 5c, d, 6, 7, 1, 2c–f, 3c–f, 4d–f, 5e, f, 6, 7, 8,
6, 7, 8, 9, 10, 11 8, 9, 10, 11 9, 10, 11
Activity 1-C-1 Activity 1-C-2 Activity 1-C-3
Review of indices Indices practice Tricky indices
doc-13692 doc-13693 doc-13694

FLUENCY
1 WE7, 8a, 8b Simplify each of the following.
(5a) 3b4
a b (4ab5) 2 × 3a3b6 c 2m3n−5 × (m2n−3) −6
10a2b2
d (2pq3) 2 × (5p2q4) 3 e (2a7b2) 2 × (3a3b3) 2 f 5(b2c−2) 3 × 3(bc5) −4
2 WE8b Simplify each of the following.
5a2b3 4x5y6 (3m2n3) 3
a b c
(2a3b) 3 (2xy3) 4 (2m5n5) 7
4x3y10 6 3a3b−5 3g2h5 3
d a b e f a b
2x7y4 (2a7b4) −3 2g4h
3 WE8c Simplify each of the following.
2a2b × 3a3b4 4m6n3 × 12mn5 10m6n5 × 2m2n3
a b c
4a3b5 6m7n6 12m4n × 5m2n3
6x3y2 × 4x6y (6x3y2) 4 5x2y3 × 2xy5
d e f
9xy5 × 2x3y6 9x5y2 × 4xy7 10x3y4 × x4y2

Chapter 1 • Indices 15
number and algebra

4 WE9a Simplify each of the following.


a3 b2 2a6b (2a6) 2 4ab6 (m4n3) 2 (m3n3) 3
a × b × c ×
5a4b7 a9b3 10a7b3 6a3 (m6n) 4 (2mn) 2
2m3n2 3 6m2n4 2xy2 4 x3y9 2 4x−5y−3 3x5y6
d a b × e a b × a b f ×
3mn5 4m3n10 3x3y5 2y10 (x2y2) −2 2−2x−7y
5 WE9b Simplify each of the following.
5a2b3 a9 b4 7a2b4 3ab 3
a ÷ b ÷ a b
6a7b5 3ab6 3a6b7 2a6b4
4a9 3 3a7 4 5x2y6 1 4x6y 2 3
c a b ÷ a b d ÷
b6 2b5 1 2x4y5 2 2 10xy3
x5y−3 −4 4x6y−10 3m3n4 2m4n6 −2
e a b ÷ f ÷ a −1 b
2xy5 (3x−2y2) −3 2m−6n−5 m n

UNDERSTANDING
6 Evaluate each of the following.
a (52 × 2) 0 × (5−3 × 20) 5 ÷ (56 × 2−1) −3
(26 × 39) 0
b (23 × 33) −2 ÷
26 × (3−2) −3
7 Evaluate the following for x = 8. (Hint: Simplify first.)
x 2 2x
(2x) −3 × a b ÷ 3 4
2 (2 )
8 a Simplify the following fraction.
a2y × 9by × (5ab) y
(ay) 3 × 5(3by) 2
b Find the value of y if the fraction is equal to 125.
x2y xy
9  MC The expression 2 3
÷ is equal to:
(2xy ) 16x0
2 2x2
A B C 2x2y6
x y6
2 b6
2 1
D E
xy6 128xy5

REASONING
10 In a controlled breeding program at the Melbourne Zoo,
the population (P) of koalas at t years is modelled by
P = P0 × 10kt. The initial number of koalas is 20 and the
population of koalas after 1 year is 40.
Given P0 = 20 and k = 0.3:
a calculate the number of koalas after 2 years.
b when will the population be equal to 1000?

16 Maths Quest 10 New South Wales Australian curriculum edition Stages 5.1 and 5.2
number and algebra

11 The decay of uranium is modelled by


D = D0 × 2−kt. If it takes 6 years for
the mass of uranium to halve, find the Digital doc
WorkSHEET 1.2
percentage remaining after: doc‐13697

a 2 years
b 5 years
c 10 years.

12 A friend is trying to calculate the volume of water in a reservoir amid fears there
may be a severe water shortage. She comes up with the following expression:
r4 u 2 ru × d2
W = 3 −2 ×

rd dr3u4
where r is the amount of rain, d is how dry the area is, u is the usage of water by

the townsfolk, and W is the amount of water in kL.
a Simplify the expression. REFLECTION
Do index laws need to be
b If the area is twice as dry as usual, would this
performed in a certain
mean that the water usage would double? Explain order?
your answer.

CHALLENGE 1.2

Chapter 1 • Indices 17
number and algebra

CHAPTER REVIEW
LANGUAGE

base expanded form index law power indices


constant exponent negative pronumeral
denominator expression numerator simplify
evaluate index positive substitute

int‐2826 doc‐13699
int‐2827 doc‐13700
int‐3588

FLUENCY
1 3d10e4 is the simplified form of:
6d10e5 d5 2
A d 6e2 × 3d 4e3 B C (3d 5e2) 2 D 3e (d5) 2 × e3 E 3a b
2e2 e2
2 8m3n × n4 × 2m2n3 simplifies to:
A 10m5n8 B 16m5n7 C 16m5n8 D 10m5n7 E 17m5n8
3 8x3 ÷ 4x–3 is equal to:
2
A 2 B 2x0 C 2x6 D 2x–1 E
x9
12x8 × 2x7
4 simplifies to:
6x9 × x5
A 4x5 B 8x C 4x D 8x5 E 4x29
(a2b3) 5
5 The expression is equal to:
(2a2b) 2
a6b13 a3 b6 a6b13 a3 b6
A B 2a6b13 C D E
4 2 2 4
(p2q) 4 (p5q2) 2
6 ÷ can be simplified to:
(2p5q2) 3 2pq5
1 22 1 1
A B C D E 22p16q
4p16q p16q 4p8 2p16q
7 Simplify each of the following.
3 26a4b6c5
a 5x3 × 3x5y4 × x2y6 b
5 12a3b3c3
20m5n2 3 14p7 4
c a b d a b
6 21q3

18 Maths Quest 10 New South Wales Australian curriculum edition Stages 5.1 and 5.2
number and algebra

8 Evaluate each of the following.


2a 0 (4b) 0
a 5a0 − a b + 12 b –(3b) 0 −
3 2
9 Simplify each of the following and express your answer with positive indices.
a 2a–5 b2 × 4a –6 b –4 b 4x –5y –3 ÷ 20x12 y –5 c (2m –3n2) –4
10 Evaluate each of the following without using a calculator.
1 −3 9 2 5
a a b b 2 × 1 3 2 −3 × a b c 4−3 × −5
2 2 8−2
11 Simplify each of the following.
(5a−2b) −3 × 4a6b−2 2x4y−5 4xy−2 −3
a b × a b
2a2b3 × 5−2a−3b−6 3y6x−2 3x−6y3
PROBLEM SOLVING
6a3m × 2b2m × 1 3ab 2 −m
1 If m = 2, determine the value of:
1 4b 2 m × a4
2 Answer the following and explain your reasoning
a What is the one’s digit of 6309?
b What is the one’s digit of 81007?
3 For the work shown below:
a calculate the correct answer
b identify where the student has made mistakes.
3 2 3 2
a2 b3 c a3b2c2 b3 c ab2c2
a 2 2b ×a 2 3 b = a 2b ×a 3 b
ab ab b b
3 2 2
bc ac
= a b ×a b
1 b
6
abc3
= a b
b
6
ac3
= a b
1
= a6c18
4 For the work shown below:
a calculate the correct answer
b identify where the student has made mistakes.
2
3a3b5c3 2ab 3a6b10c6 2ab
a b ÷ a b = ÷
5a2b c 10a4b2 c
3a6b10c6 c
= ×
10a4b2 2ab
3a6b10c7
=
20a5b3
3ab7c7
=
20

Chapter 1 • Indices 19
number and algebra
Communicating

Rich task

Digital world: ‘A bit of this


and a byte of that’

1 Complete the table below to show the difference in value between the binary and decimal
systems.

20 Maths Quest 10 New South Wales Australian curriculum edition Stages 5.1 and 5.2
number and algebra

2 The two numbering systems have led to some confusion, with some manufacturers of digital
products thinking of a kilobyte as 1000 bytes rather than 1024 bytes. Similar confusion arises
with megabytes, gigabytes, terabytes and so on. This means you might not be getting exactly
the amount of storage that you think.
If you bought a device quoted as having 16 GB memory, what would be the difference
in memory storage if the device had been manufactured using the decimal value of GB as
opposed to the binary system?
Many devices allow you to check the availability of storage. On one such device, the iPhone,
available storage is found by going to ‘General’ under the heading ‘Settings’.
3 How much storage is left in MB on the following iPhone?
General Usage
4 If each photo uses 3.2 MB of memory, how many photos
can be added? Storage
3.9 GB Available 9.5 GB Used

Photos & Camera 1.6 GB

Radio 1.6 GB

Maps 1.2 GB

My Movie 461 MB

Have you ever wondered about the capacity of our brain to store information and the speed at
which information is transmitted inside it?
5 Discuss how the storage and speed of our
brain compares to our current ability to
send and store information in the digital
world. The capacity of the human brain is
10–100 terabytes. On average 20 million
billion bits of information are transmitted
within the brain per second.
6 Investigate which country has the fastest
internet speed and compare this to
Australia.

Chapter 1 • Indices 21
number and
NUMBER AND algebra
ALGEBRA

Code puzzle

What is the largest


gland in the body?
Simplify the expressions to find the puzzle’s answer code.

a6 ÷ a2 = 6e 10 ÷ 3e 2 =

A B
15f 7 (16c 9) ÷ (8c 5) = 8n 6u 11
= =
5f 4 2nu 8
C D E
50t 12 ÷ 25t 11 8k 6e 2
e10 ÷ e 7 =
=
=
F 4e 2k 4
H I
(12c 20) ÷ (2c 13) (6n 7u 7) ÷ (2n 5u 6)
28f 10
=
=
K 4f 8
L =
N
9c 2t 7e 18a 21 ÷ 3a16 10e 3k12
= =
3ct 4ec
O P R
= 2k 5e 3

7c 8 ÷ 7c 7 18n 7u12 35f 21 ÷ 7f 12


=
=
S 3n 3u
T =
U
24e 11c15
= a 60 (20k 8t 6) ÷ (5k 6t 2k 2)
8e 9c 15 =
V a 20
W =
Y

22 Maths Quest 10 New South Wales Australian curriculum edition Stages 5.1 and 5.2
number and algebra

 ACTIVITIES
Go to assessON
Chapter opener Interactivity for questions to
Video • Colour code breaker (int‐2777) (page 9) test your readiness
• The story of mathematics (eles-1840) 1C Combining index laws FOR learning, your
Digital docs progress AS you learn
1A Review of index laws
Digital docs  (page 6) • Activity 1‐C‐1 (doc‐13692): Review of and your levels OF
• SkillSHEET (doc‐5168): Index form indices (page 15) achievement.
• SkillSHEET (doc‐5169): Using a calculator to • Activity 1‐C‐2 (doc‐13693): Indices www.assesson.com.au
evaluate numbers given in index form practice (page 15)
• Activity 1‐A‐1 (doc‐4948): Reviewing index • Activity 1‐C‐3 (doc‐13694): Tricky
operations indices (page 15)
• Activity 1‐A‐2 (doc‐4949): Practising the • WorkSHEET 1.2 (doc‐13697): Combining
index laws index laws (page 17)
• Activity 1‐A‐3 (doc‐4950): Applying the index Chapter review
laws Interactivities (page 18)
1B Negative indices • Word search (int‐2826)
Digital docs  • Crossword (int‐2827)
• Activity 1‐B‐1 (doc‐4951): Negative • Sudoku (int‐3588)
indices (page 11) Digital docs
• Activity 1‐B‐2 (doc‐4952): Harder negative • Chapter summary (doc‐13699)
indices (page 11) • Concept map (doc‐13700)
• Activity 1‐B‐3 (doc‐4953): Tricky negative To access eBookPLUS activities, log on to
indices (page 11) www.jacplus.com.au
• WorkSHEET 1.1 (doc‐13695): Apply your
knowledge of index laws to answer
questions. (page 13)

Chapter 1 • Indices 23
number and algebra

Answers
CHAPTER 1 Indices
Exercise 1A — Review of index laws
1 6 3 4
1    a a7 b a6 c b8 d a4b7 2    a b c d
e m5n13 f a5b7c3 g m6n4p5 h 6a2b a2b3 x6y n8 a2b5
4 9 8 7 k 12x6y6 l 4x8y6
i 10a b j 36m n 2y 5y 3 4y12
4 4
e f g h
2    a a b a5 c b3 d a 3x 6x3 m2n2 x5
3
1 2 8
e 3b4 f 4m5 g m3n h y 1 1 4q 3
2 i j k l
3 3 3 5 2 2 1 2 3m3n3 32a15m20 p14 a8b12
i 4
xy j 7b k 4
mp l 2
xy
27q9 b6 1
3    a 1 b 1 c 1 m n o
6
d 3 e 4 f −3 8p 4a8 8a6b6
g 3 h −7 i 4 3    a 1
b 1
c 1
d 8
8 36 81 9
1 8
4    a a6 b 16a20 c 81
m e 1
f 5
g 48 h 32
16 36 27
4 8 6 3 6 4
d 9
n e ab f 9a b i 27
= 2
125 j 4 k 125 l 3
25 4
27 6 3 a4
g 16m12n20 h 64
mn i 4    a 2 3
b 2 −3
c 2 5
d 2−6
b6
5    a 3 b −2 c −1 d 3
625m12 343x3 81a4
j k l e −2 f 0 g 3 h −3
n8 8y15 625b12 i −6 j −2
m −243 n 49 o −32 3 4 2
5    a D 6    a b c d 5
b D 2 5 7
6    a C b E c B d D b b3 a2 1
7    a b c d
7    a 64 b 72 c 625 a a2 b3 m3n2
d 48 e 1600 27
f 125 1 1
8    a b
g 20 h 1 i 4 729 20736
8    a x3yz b ab c manb 1
c d 256
a2x − −
16807
d e n 3 pm 2 q f amp + np
b3x 16384
e f 9  765  625
9 a3 = a × a × a 2187
9    a D b C c B
a2 = a × a
a3 × a2 = a × a × a × a × a 10    a B b D c C d E
= a5, not a6 m2 n2 25
11    a b c
Explanations will vary. n8 m a7b6
10 They are equal when x = 2. Explanations will vary. 12, 13 Answers will vary. Check with your teacher.
11 3x0 = 3 and (3x) 0 = 1. Explanations will vary.
12 a, b Exercise 1C — Combining index laws
2n13
1    a 54a10b9 b 48a5b16 c
a 0 1 2 3 m9
3a2 0 3 12 27 15b2
d 500p8q18 e 36a20b10 f
5a 0 5 10 15 c26
3a + 5a
2 0 8 22 42 5 x 27
2    a b c
3a × 5a
2 0 15 120 405 8a 7
4y 6 128m29n26
36
c 3a2 × 5a will become much larger than 3a2 + 5a. 64y 27h12
d e 24a24b7 f
x 24 8g6
Challenge 1.1
1.08 seconds 3a2 m2n4
3    a b 8n2 c
Exercise 1B — Negative indices 2 3
1 1 2 4x5 36x6 y2
1    a b c d e f
4
x5 y a9 3y8 y x4
4 3x2 1
d e f 2 3 3
4a b n9
5a 3 y3 4m3n4 4    a b c
5a4b7 15 4m9
6a3 2a4
g h a6 i 4m5 4
bc5 3 d e f 48x11y6
2 7b3 2m3a2 9n15 81x2y14
j 2ab k l
2a4 3b4n5

24 Maths Quest 10 New South Wales Australian curriculum edition Stages 5.1 and 5.2
number and algebra

5 56a11b6 1024b2 Problem solving


5    a b c 1
2a13 81 81a 1 12
25 4y36 2    a 6 b 2
d e f 6m19n19
128x23y4 27x16 3    a a2bc7
125 b The student made a mistake when multiplying the two brackets
6    a b 1
8 in line 3. Individual brackets should be expanded first.
7 1 9ab7c7
8    a 5y − 1 b y=4 4    a
50
9 A
b The student has made two mistakes when squaring the
10    a 79 koalas b During the 6th year.
left‐hand bracket in line 1 : 32 = 9, 52 = 25.
11   a 79% b 56% c 31%
d3 Communicating — Rich task
12    a b Discuss with your teacher. 1
ru
Power of 2 and Power of 10 and
Challenge 1.2
Unit Symbol value in bytes value in bytes
$30  720
Chapter review Byte B 20 = 1 100 = 1
Fluency
Kilobyte KB 210 = 1024 103 = 1000
1 D 2 C 3 C
4 C 5 A 6 A Megabyte MB 220 = 1  048  576 106 = 1  000  000
13ab3c2
7    a 9x10y10 b Gigabyte GB 230 = 1  073  741  824 109 = 1  000  000  000
6
1000 m15n6 16p28 Terabyte TB 240 = 1  099  511  627  776 1012 = 1  000  000  000  000
c d
27 81q12
2 Approximately 1.1 GB
8    a 16 b −32
3 3993.6 MB
8 y2 m12 4 1248 photos
9    a b c
a11b2 5x17 16n8 5 Discuss with your teacher.
3 6 Discuss with your teacher. The discussion will depend on the
10    a 8 b 2
c 0 latest information from the internet.
2a13 9y4 Code puzzle
11    a b
5b2 32x15 The liver produces a litre of bile a day. It helps break down fat.

Chapter 1 • Indices 25
number and
NUMBER AND ALGEBRA
algebra

Chapter 2

Algebra and
equations
WHY LEARN THIS?
Do you speak mathematics? Algebra is the language
of mathematics; it holds the key to understanding the
rules, formulas and relationships that summarise much
of our understanding of the universe. Every student of
mathematics needs a mathematical tool chest, a set of
algebraic skills to manipulate and process mathematical
information.

WHAT DO YOU KNOW?


1 Think List what you know about linear
equations. Use a thinking tool such as
a concept map to show your list.
2 pair Share what you know with a partner
and then with a small group.
3 share As a class, create a thinking tool that shows
your class’s knowledge of linear equations.

LEARNING SEQUENCE
2A Substitution
2B Adding and subtracting algebraic fractions
2C Multiplying and dividing algebraic fractions
2D Solving simple equations
2E Solving multi-step equations
Watch this video
The story of mathematics

Searchlight ID: eles-1841


number and algebra

2A  Substitution
When the numerical values of pronumerals are known, they can be substituted into an
algebraic expression and the expression can then be evaluated. It can be useful to place
any substituted values in brackets when evaluating an expression.

WORKED EXAMPLE 1
If a = 4, b = 2 and c = −7, evaluate the following expressions.
a a−b b a3 + 9b − c

TH I N K W R I TE
a 1 Write the expression. a a−b
2 Substitute a = 4 and b = 2 into the =4 − 2
expression.
3 Simplify. =2
b 1 Write the expression. b a3 + 9b − c
2 Substitute a = 4, b = 2 and c = −7 into = (4) 3 + 9(2) − (−7)
the expression.
3 Simplify. = 64 + 18 + 7
= 89

WORKED EXAMPLE 2
If c = "a2 + b2, calculate c if a = 12 and b = −5.

TH I N K W R I TE
1 Write the expression. c = "a2 + b2
2 Substitute a = 12 and b = −5 into the = "(12) 2 + (−5) 2
expression.
3 Simplify. = !144 + 25
= !169
= 13

Number laws
•• Recall from previous studies that when dealing with numbers and pronumerals,
particular rules must be obeyed. Before progressing further, let us briefly review the
Commutative, Associative, Identity and Inverse Laws.
•• Consider any three pronumerals x, y and z, where x, y and z are elements of the set of
Real numbers.

28 Maths Quest 10 New South Wales Australian curriculum edition Stages 5.1 and 5.2
number and algebra

Commutative Law
1. x+y=y+x (example: 3 + 2 = 5 and 2 + 3 = 5)
2. x−y≠y−x (example: 3 − 2 = 1 but 2 − 3 = −1)
3. x×y=y×x (example: 3 × 2 = 6 and 2 × 3 = 6)
4. x÷y≠y÷x (example: 3 ÷ 2 = 32, but 2 ÷ 3 = 23)

Therefore, the Commutative Law holds true for addition and multiplication, since the
order in which two numbers or pronumerals are added or multiplied does not affect the
result. However, the Commutative Law does not hold true for subtraction or division.
Associative Law
1. x + (y + z) = (x + y) + z [example: 2 + (3 + 4) = 2 + 7 = 9 and
(2 + 3) + 4 = 5 + 4 = 9]
2. x − (y − z) ≠ (x − y) − z [example: 2 − (3 − 4) = 2 − −1 = 3 and
(2 − 3) − 4 = −1 − 4 = −5]
3. x × (y × z) = (x × y) × z [example: 2 × (3 × 4) = 2 × 12 = 24 and
(2 × 3) × 4 = 6 × 4 = 24]
4. x ÷ (y ÷ z) ≠ (x ÷ y) ÷ z [example: 2 ÷ (3 ÷ 4) = 2 ÷ 34 = 2 × 43 = 83 but
(2 ÷ 3) ÷ 4 = 23 ÷ 4 = 23 × 14 = 12
2
= 16]

The Associative Law holds true for addition and multiplication since grouping two or
more numbers or pronumerals and calculating them in a different order does not affect
the result. However, the Associative Law does not hold true for subtraction or division.

Identity Law
The Identity Law states that in general: x+0=0+x=x
x×1=1×x=x
In both of the examples above, x has not been changed (that is, it has kept its identity)
when zero is added to it or it is multiplied by 1.

Inverse Law
The Inverse Law states that in general: x + −x = −x + x = 0
1 1
x× = ×x=1
x x
That is, when the additive inverse of a number or pronumeral is added to itself, it
equals 0. When the multiplicative inverse of a number or pronumeral is multiplied by
itself, it equals 1.

Closure Law
A law that you may not yet have encountered is the Closure Law. The Closure Law
states that, when an operation is performed on an element (or elements) of a set, the
result produced must also be an element of that set. For example, addition is closed on
natural numbers (that is, positive integers: 1, 2, 3, .  .  .) since adding a pair of natural
numbers produces a natural number. Subtraction is not closed on natural numbers. For
example, 5 and 7 are natural numbers and the result of adding them is 12, a natural
number. However, the result of subtracting 7 from 5 is −2, which is not a natural number.

Chapter 2 • Algebra and equations 29


number and algebra

WORKED EXAMPLE 3
Find the value of the following expressions, given the integer values x = 4
and y = −12. Comment on whether the Closure Law for integers holds for
each of the expressions when these values are substituted.
a x+y b x−y c x×y d x÷y

TH I N K W R I TE
a 1 Substitute each pronumeral into a x + y = 4 + −12
the expression.
2 Evaluate and write the answer. = −8
3 Determine whether the Closure The Closure Law holds for these
Law holds; that is, is the result substituted values.
an integer?
b Repeat steps 1 –3 of part a. b x − y = 4 − −12
= 16
The Closure Law holds for these
substituted values.
c Repeat steps 1 –3 of part a. c x × y = 4 × −12
= −48
The Closure Law holds for these
substituted values.
d Repeat steps 1 –3 of p a r t a. d x ÷ y = 4 ÷ −12
= −13
The Closure Law does not hold for
these substituted values since the
answer obtained is a fraction, not
an integer.

It is important to note that, although a particular set of numbers may be closed under a
given operation, for example multiplication, another set of numbers may not be closed
under that same operation. For example, in part c of Worked example 3, integers were
closed under multiplication. However, in some cases, the set of irrational numbers
is not closed under multiplication, since !3 × !3 = !9 = 3. In this example, two
irrational numbers produced a rational number under multiplication.

Exercise 2A Substitution
INDIVIDUAL PATHWAYS
Questions: Questions: Questions:
1a–f, 2a–f, 3c–d, 4a–c, 5a–c, 1c–i, 2a–f, 3c–d, 4, 5a–c, 6d–j, 1e–l, 2c–i, 3, 4, 5, 6d–j, 9, 10,
6a–e, 7, 8, 10 9, 10 11, 12, 13
Activity 2-A-1 Activity 2-A-2 Activity 2-A-3
Substitution Harder substitution Tricky substitution
doc-4960 doc-4961 doc-4962

30 Maths Quest 10 New South Wales Australian curriculum edition Stages 5.1 and 5.2
number and algebra

FLUENCY
1 WE1If a = 2, b = 3 and c = 5, evaluate the following expressions. Digital docs
SkillSHEET
a a+b b c−b c c−a−b Like terms
doc-5183
a b c
d c − (a − b) e 7a + 8b − 11c f + + SkillSHEET
2 3 5 Collecting like terms
h ab(c − b) i a2 + b2 − c2
doc-5184
g abc SkillSHEET
j c2 + a k −a × b × −c l 2.3a − 3.2b Finding the highest
common factor
2 If d = −6 and k = −5, evaluate the following. doc-5185
SkillSHEET
a d+k b d−k c k−d Order of operations
doc-5189
d kd e −d(k + 1) f d2
k−1
g k3 h i 3k − 5d
d
1 1
3 If x = and y = , evaluate the following.
3 4
a x+y b y−x
x
c xy d
y
9x
e x2y3 f
y2
4 If x = 3, find the value of the following.
a x2 b −x2 c (−x) 2
d 2x2 e −2x2 f (−2x) 2
5 If x = −3, find the value of the following.
a x2 b −x2 c (−x) 2
d 2x2 e −2x2 f (−2x) 2
6 WE2 Calculate the unknown variable in the following real-life mathematical
formulas.
a If c = "a2 + b2, calculate c if a = 8 and b = 15.
1
b If A = 2bh, determine the value of A if b = 12 and h = 5.
c The perimeter, P, of a rectangle is given by P = 2L + 2W. Calculate the
perimeter, P, of a rectangle, given L = 1.6 and W = 2.4.
C
d If T = , determine the value of T if C = 20.4 and L = 5.1.
L
n+1
e If K = , determine the value of K if n = 5.
n−1
9C
f Given F = + 32, calculate F if C = 20.
5
g If v = u + at, evaluate v if u = 16, a = 5, t = 6.
h The area, A, of a circle is given by the formula A = πr2. Calculate the area of a
circle, correct to 1 decimal place, if r = 6.
1
i If E = mv2, calculate m if E = 40, v = 4.
2
A
j Given r = , evaluate A to 1 decimal place if r = 14.1.
Åπ

Chapter 2 • Algebra and equations 31


number and algebra

7 MC a If p = −5 and q = 4, then pq is equal to:


5
A 20 B 1 C −1 D −20 E −
4
b If c2 = a2 + b2, and a = 6 and b = 8, then c is equal to:
A 28 B 100 C 10 D 14 E 44
c Given h = 6 and k = 7, then kh2 is equal to:
A 294 B 252 C 1764 D 5776 E 85

UNDERSTANDING
8 Knowing the length of two sides of a right-angled triangle, the third side can be
calculated using Pythagoras’ theorem. If the two shorter sides have lengths of
1.5 cm and 3.6 cm, calculate the length of the hypotenuse.
9 The volume of a sphere can be calculated using the formula
4 3
3
πr . What is the volume of a sphere with a radius of 2.5 cm?
Give your answer correct to 2 decimal places. 2.5 cm

10 A rectangular park is 200 m by 300 m. If Blake runs along


the diagonal of the park, how far will he run? Give your
answer to the nearest metre.

REASONING
11 WE3 Determine the value of the following expressions, given the integer values
x = 1, y = −2 and z = −1. Comment on whether the Closure Law for integers
holds true for each of the expressions when these values are substituted.
a x+y b y−z c y×z
d x÷z e z−x f x÷y
12 Find the value of the following expressions, given the natural number values x = 8,
y = 2 and z = 6. Comment on whether the Closure Law for natural numbers holds
true for each of the expressions.
a x+y b y−z c y×z
d x÷z e z−x f x÷y
13 For each of the following, complete the relationship to illustrate the stated law.
Justify your answer.
a (a + 2b) + 4c = ____________ Associative Law
b (x × 3y) × 5c = ____________ Associative Law
c 2p ÷ q ≠ ____________ Commutative Law
d 5d + q = ____________ Commutative Law
e 3z + 0 = ____________ Identity Law
REFLECTION
f 2x × = ____________ Inverse Law Why is knowledge of
g (4x ÷ 3y) ÷ 5z ≠ ____________ Associative Law the Commulative Law
useful?
h 3d − 4y ≠ ____________ Commutative Law

32 Maths Quest 10 New South Wales Australian curriculum edition Stages 5.1 and 5.2
number and algebra

CHALLENGE 2.1

2B  Adding and subtracting algebraic fractions


•• In an algebraic fraction, the denominator, the numerator or both are algebraic
x 3x + 1 1
expressions. For example, , and 2 are all algebraic fractions.
2 2x − 5 x +5
•• As with all fractions, algebraic fractions must have a common denominator if they
are to be added or subtracted, so an important step is to find the lowest common
denominator (LCD).

WORKED EXAMPLE 4
Simplify the following expressions.
2x x x+1 x+4
a − b +
3 2 6 4
TH I N K W R I TE
a 1 Write the expression. a 2x − x
3 2
2 Rewrite each fraction as an equivalent 2x 2 x 3
= × − ×
fraction using the LCD of 3 and 2, 3 2 2 3
which is 6. 4x 3x
= −
6 6
4x − 3x
3 Express as a single fraction. =
6
x
4 Simplify the numerator. =
6

Chapter 2 • Algebra and equations 33


number and algebra

b 1 Write the expression. b x+1 x+4


+
6 4

2 Rewrite each fraction as an equivalent x+1 2 x+4 3


= × + ×
fraction using the LCD of 6 and 4, 6 2 4 3
which is 12.
2(x + 1) 3(x + 4)
= +
12 12

3 Express as a single fraction. 2(x + 1) + 3(x + 4)


=
12

4 Simplify the numerator by expanding 2x + 2 + 3x + 12


=
brackets and collecting like terms. 12
5x + 14
=
12

Pronumerals in the denominator


•• If pronumerals appear in the denominator, the process involved in adding and
subtracting the fractions is to find a lowest common denominator as usual.

WORKED EXAMPLE 5
2 1
Simplify − .
3x 4x

TH I N K W R I TE

1 Write the expression. 2



1
3x 4x

2 Rewrite each fraction as an equivalent 2 4 1 3


fraction using the LCD of 3x and 4x, which is = 3x × 4 − 4x × 3
12x. (Note: 12x2 is not the LCD.)
8 3
= −
12x 12x

3 Express as a single fraction. 8−3


=
12x

4 Simplify the numerator. 5


=
12x

34 Maths Quest 10 New South Wales Australian curriculum edition Stages 5.1 and 5.2
number and algebra

Exercise 2B Adding and subtracting


algebraic fractions
INDIVIDUAL PATHWAYS
Questions: Questions: Questions:
1a–f, 2a–f, 3a–f, 4 1d–i, 2a–f, 3a–i, 4, 5 1g–i, 2e–l, 3d–i, 4, 5
Activity 2-B-1 Activity 2-B-2 Activity 2-B-3
Introducing algebraic Working with algebraic Advanced algebraic
fractions fractions fractions
doc-4963 doc-4964 doc-4965

FLUENCY
1 Simplify each of the following.
4 2 1 5 3 6
a + b + c +
7 3 8 9 5 15
4 3 3 2 1 x
d − e − f −
9 11 7 5 5 6
5x 4 3 2x 5 2
g − h − i −
9 27 8 5 x 3

2 WE4 Simplify the following expressions.

2y y y y 4x x
a − b − c −
3 4 8 5 3 4
Digital docs
8x 2x 2w w y y
d + e − f − SkillSHEET
9 3 14 28 20 4 Addition and s­ ubtraction
of fractions
doc-5186
12y y 10x 2x x+1 x+3 SkillSHEET
g + h + i + Writing equivalent
5 7 5 15 5 2 algebraic fractions with
the lowest common
denominator
x+2 x+6 2x − 1 2x + 1 3x + 1 5x + 2
j + k − l + doc-5190
4 3 5 6 2 3

3 WE5 Simplify the following.

2 1 3 1 5 1
a + b − c +
4x 8x 4x 3x 3x 7x

12 4 1 1 9 9
d + e + f −
5x 15x 6x 8x 4x 5x

2 7 1 5 4 3
g + h + i −
100x 20x 10x x 3x 2x

Chapter 2 • Algebra and equations 35


number and algebra

UNDERSTANDING
4 Simplify the following expressions by writing them as single fractions.

x x x 2x − 1 2x x + 2
a − + b   + −
3 2 4 3 5 10

3m − 5 5m 2m − 5 x+3 x+5
c + − d  x − +
2 3 6 3 5
5 Simplify the following expressions by writing them as
single fractions. REFLECTION
Why can’t we just add
2 3 3 1 the numerators and the
a − b   + denominators of fractions:
x 4 x y
For example
a c a+c
3x − 5 5y + 1 3x − 5 5y + 1 + = ?
c + d   + b d b+d
x y y x

2C Multiplying and dividing algebraic


fractions
Simplifying algebraic fractions
•• Algebraic fractions can be simplified using the index laws and by cancelling factors
common to the numerator and denominator.
•• A fraction can only be simplified if:
–– there is a common factor in the numerator and the denominator
–– the numerator and denominator are both written in factorised form, i.e. as the
product of two or more factors.
3ab 13 × 1a × b ← product of factors 3a + b 3 × a + b ← not a product of factors
= 4 =
12a 12 × 1a ← product of factors 12a 12 × a ← product of factors
b
    =   Cannot be simplified
4

Multiplying algebraic fractions


•• Multiplication of algebraic fractions follows the same rules as multiplication of
numerical fractions: multiply the numerators, then multiply the denominators.

WORKED EXAMPLE 6
Simplify each of the following.
5y 6z 2x x+1
a × b ×
3x 7y (x + 1)(2x − 3) x
TH I N K W R I TE
5y 6z
a 1 Write the expression. a ×
3x 7y

36 Maths Quest 10 New South Wales Australian curriculum edition Stages 5.1 and 5.2
number and algebra

5y 2
2 Cancel common factors in the 6z
=
numerator and denominator. The y 1 3x 7y
can be cancelled in the denominator 5 2z
and the numerator. Also 3 in the = ×
x 7
denominator can divide into the 6 in
the numerator.
3 Multiply the numerators, then 10z
=
multiply the denominators. 7x
b 1 Write the expression. b 2x x+1
×
(x + 1)(2x − 3) x
2 Cancel common factors in the 2x1 x + 11
numerator and the denominator. = ×
1
(x + 1)(2x − 3) x1
(x + 1) and the x are both common 2 1
in the numerator and the denominator = ×
2x − 3 1
and can therefore be cancelled.
2
3 Multiply the numerators, then =
multiply the denominators. 2x − 3

Dividing algebraic fractions


•• When dividing algebraic fractions, follow the same rules as for division of numerical
fractions: write the division as a multiplication and invert the second fraction. This
process is sometimes known as multiplying by the reciprocal.

WORKED EXAMPLE 7
Simplify the following expressions.
3xy 4x 4 x−7
a ÷ b ÷
2 9y (x + 1)(3x − 5) x + 1
TH I N K W R I TE
a 1 Write the expression. a 3xy 4x
÷
2 9y
3xy 9y
2 Change the division sign to a = ×
multiplication sign and write the 2 4x
second fraction as its reciprocal.
3y 9y
3 Cancel common factors in the = ×
numerator and denominator 2 4
and cancel. The pronumeral x is
common to both the numerator and
denominator and can therefore be
cancelled.
4 Multiply the numerators, then 27y2
=
multiply the denominators. 8

Chapter 2 • Algebra and equations 37


number and algebra

b 1 Write the expression. b 4 x−7


÷
(x + 1)(3x − 5) x + 1
2 Change the division sign to a 4 x+1
= ×
multiplication sign and write the (x + 1)(3x − 5) x − 7
second fraction as its reciprocal.
3 Cancel common factors in the 4 1
= ×
numerator and denominator and 3x − 5 x − 7
cancel. (x + 1) is common to both the
numerator and denominator and can
therefore be cancelled.
4
4 Multiply the numerators, then =
multiply the denominators. (3x − 5)(x − 7)

Exercise 2C Multiplying and dividing algebraic


fractions
INDIVIDUAL PATHWAYS
Questions: Questions: Questions:
1a–f, 2a–f, 3a–i, 4a–b 1d–i, 2a–f, 3a–i, 4 1g–l, 2e–j, 3d–l, 4
Activity 2-C-1 Activity 2-C-2 Activity 2-C-3
Learning operations with Operations with Advanced operations with
algebraic fractions algebraic fractions algebraic fractions
doc-4966 doc-4967 doc-4968

FLUENCY
1 WE6a Simplify each of the following.
Digital docs
SkillSHEET x 20 x 12 y 16
a × b × c ×
Multiplication of
5 y 4 y 4 x
­fractions
doc-5187
SkillSHEET x 9 x −25 3w −7
Simplification of d × e × f ×
a­ lgebraic fractions 2 2y 10 2y −14 x
doc-5191
SkillSHEET
3y 8z −y 6z x −9z
Division of fractions g × h × i ×
doc-5192 4x 7y 3x −7y 3z 2y
SkillSHEET
Order of operations
doc-5189
5y x −20y −21z y x
j × k × l ×
3x 8y 7x 5y −3w 2y
2 WE6b Simplify the following expressions.

2x x−1 5x 4x + 7
a × b ×
(x − 1)(3x − 2) x (x − 3)(4x + 7) x

9x 5x + 1 (x + 4) x+1
c × d ×
(5x + 1)(x − 6) 2x (x + 1)(x + 3) x + 4

38 Maths Quest 10 New South Wales Australian curriculum edition Stages 5.1 and 5.2
number and algebra

2x x−1 2 x(x + 1)
e × f ×
x + 1 (x + 1)(x − 1) x(2x − 3) 4

2x 3a 15c 21d
g × h ×
4(a + 3) 15x 12(d − 3) 6c

6x2 15(x − 2) 7x2 (x − 3) 3(x − 3)(x + 1)


i × j ×
20(x − 2) 2
16x4 5x(x + 1) 14(x − 3) 2 (x − 1)
3 WE7a Simplify the following expressions.

3 5 2 9 4 12
a ÷ b ÷ c ÷
x x x x x x
20 20 1 5 7 3
d ÷ e ÷ f ÷
y 3y 5w w 2x 5x

3xy 3x 2xy 5x 6y 3x
g ÷ h ÷ i ÷
7 4y 5 y 9 4xy
8wx 3w 2xy 3xy 10xy 20x
j ÷ k ÷ l ÷
5 4y 5 5 7 14y
4 WE7b Simplify the following expressions.
9 x+3
a ÷
(x − 1)(3x − 7) x − 1 Digital doc
1 x−9 WorkSHEET 2.1
b ÷ doc-13701
(x + 2)(2x − 5) 2x − 5
REFLECTION
How are multiplying
12(x − 3) 2 4(x − 3)
c ÷ and dividing algebraic
(x + 5)(x − 9) 7(x − 9) fractions different to
3(x + 1) adding and subtracting
13
d ÷ them?
6(x − 4) 2 (x − 1) 2(x − 4)(x − 1)

Chapter 2 • Algebra and equations 39


number and algebra

2D  Solving simple equations


•• Equations show the equivalence of two expressions.
•• Equations can be solved using inverse operations.
•• When solving equations, the last operation performed on the pronumeral when
building the equation is the first operation undone by applying inverse operations
to both sides of the equation. For example, the
equation 2x + 3 = 5 is built from x by multiplying
+ and − are inverse operations
x by 2 and then adding 3 to give the result of 5.
× and ÷ are inverse operations
To solve the equation, undo the adding 3 by 2 and are inverse operations
subtracting 3, then undo the multiplying by 2 by
dividing by 2.

One-step equations
•• Equations that require one step to solve them are called one-step equations.

WORKED EXAMPLE 8
Solve the following equations.
a a + 27 = 71 d
b = 314
16
4
c !e = 0.87 d f2 =
25
TH I N K W R I TE
a 1 Write the equation. a a + 27 = 71
2 27 has been added to a, resulting in 71. a + 27 − 27 = 71 − 27
The addition of 27 has to be reversed a = 44
by subtracting 27 from both sides of the
equation to obtain the solution.
b 1 Write the equation. b d
= 314
16
2 Express 31 as an improper fraction. d 13
4 =
16 4
3 The pronumeral d has been divided by 16, d 13
× 16 = × 16
resulting in 13
4
. Therefore the division has 16 4
to be reversed by multiplying both sides d = 52
of the equation by 16 to obtain d.
c 1 Write the equation. c !e = 0.87
2 The square root of e has been taken 1 !e 2 2 = 0.872
resulting in 0.87. Therefore, the square e = 0.7569
root has to be reversed by squaring both
sides of the equation to obtain e.
d 1 Write the equation. d f2 =
4
25

40 Maths Quest 10 New South Wales Australian curriculum edition Stages 5.1 and 5.2
number and algebra

2 The pronumeral f has been squared, 4


f = ± "25
4
resulting in Therefore the squaring has
.
25
to be reversed by taking the square root f = ± 25
of both sides of the equation to obtain f .
Note that there are two possible solutions,
one positive and one negative, since two
negative numbers can also be multiplied
together to produce a positive result.

Two-step equations
•• Two-step equations involve the inverse of two operations in their solutions.

WORKED EXAMPLE 9
Solve the following equations.
a 5y − 6 = 79 4x
b =5
9
TH I N K W R I TE
a 1 Write the equation. a 5y − 6 = 79
2 Step 1: Add 6 to both sides of the 5y − 6 + 6 = 79 + 6
equation. 5y = 85
3 Step 2: Divide both sides of the equation 5y 5y
=
by 5 to obtain y.
    5 5
y = 17
b 1 Write the equation. b        4x = 5
9
2 Step 1: Multiply both sides of the 4x
×9 = 5×9
equation by 9. 9
4x = 45
3 Step 2: Divide both sides of the equation 4x 45
=
by 4 to obtain x.
   4 4
45
x =
4
4 Express the improper fraction as a mixed x = 1114
   
number.

Equations where the pronumeral appears


on both sides
•• In solving equations where the pronumeral appears on both sides, subtract the
smaller pronumeral term so that it is eliminated from both sides of the equation.

Chapter 2 • Algebra and equations 41


number and algebra

WORKED EXAMPLE 10
Solve the following equations.
a 5h + 13 = 2h − 2 b 14 − 4d = 27 − d c 2(x − 3) = 5(2x + 4)

TH I N K W R I TE
a 1 Write the equation. a 5h + 13 = 2h − 2
2 Eliminate the pronumeral 3h + 13 = −2
from the right-hand side
by subtracting 2h from
both sides of the equation.
3 Subtract 13 from both sides 3h = −15
of the equation.
4 Divide both sides of the h = −15
equation by 3 and write
your answer.
b 1 Write the equation. b 14 − 4d = 27 − d
2 Add 4d to both sides of the 14 = 27 + 3d
equation.
3 Subtract 27 from both sides   −13 = 3d
of the equation.
4 Divide both sides of the 13
  − =d
equation by 3. 3
5 Express the improper   −413 = d
fraction as a mixed
number.
6 Write your answer so that d = −413
d is on the left-hand side.
c 1 Write the equation. c 2(x − 3) = 5(2x + 4)
2 Expand the brackets on 2x − 6 = 10x + 20
both sides of the
equation.
3 Subtract 2x from both sides 2x − 2x − 6 = 10x − 2x + 20
of the equation.
4 Subtract 20 from both sides −6 − 20 = 8x + 20 − 20
of the equation. −26 = 8x
5 Divide both sides of the 26
− =x
equation by 8. 8
6 Simplify and write your x = −13
4
answer with the pronumeral
on the left-hand side.

42 Maths Quest 10 New South Wales Australian curriculum edition Stages 5.1 and 5.2
number and algebra

Exercise 2D Solving simple equations


INDIVIDUAL PATHWAYS
Questions: Questions: Questions:
1a–f, 2a–e, 3a–f, 4a–b, 5a–b, 1d–i, 2d–i, 3a–f, 4, 5a–b, 6a–b, 3, 4, 5, 6, 7d–i, 8c–f, 9e–i,
6a–b, 7a–f, 8a–d, 9a–b, 10a–b, 7d–i, 8c–f, 9c–g, 10a–d, 11c–f, 10d–f, 11d–f, 12, 14, 15g–i, 16g–i,
11a–c, 12a–b, 13a–c, 15a–d, 12, 13d–i, 15c–f, 16c–f, 17c–f, 19, 17g–i, 18, 19, 20, 21, 22, 23, 24
16a–d, 17a–d, 19, 20, 21 20, 21, 23
Activity 2-D-1 Activity 2-D-2 Activity 2-D-3
Simple puzzling equations Puzzling equations Advanced puzzling equations
doc-4969 doc-4970 doc-4971

FLUENCY
1 WE8a Solve the following equations.
a a + 61 = 85 b k − 75 = 46 c g + 9.3 = 12.2
d r − 2.3 = 0.7 e h + 0.84 = 1.1 f i+5=3
1 1
g t − 12 = −7 h q+3=2 i x − 2 = −2
2 WE8b Solve the following equations.
f i
a =3 b = −6 c 6z = −42
4 10
k 5
d 9v = 63 e 6w = −32 f =
12 6
m 7 y
g 4a = 1.7 = h i = 5 38
19 8 4
3 WE8c, d Solve the following equations.
a !t = 10 b y2 = 289 c !q = 2.5
d f 2 = 1.44 e !h = 7
4
f p2 =
9
64
15 196
g !g = 22
h j2 = 961
i a2 = 2 79
4 Solve the following equations.
a !t − 3 = 2 b 5x2 = 180 c 3 !m = 12
d −2t = −18
2
e t2 + 11 = 111 f !m + 5 = 0
5 Solve the following equations.
3 3 1
a "x = 2 b x3 = −27 c "m =
2
27 3 5
d x3 = e "m = 0.2 f w3 = 15 8
64
6 Solve the following equations.
3
a x3 + 1 = 0 b 3x3 = −24 c "m + 5 = 6
3 3
d −2 × "w = 16 e "t − 13 = −8 f 2x3 − 14 = 2
7 WE9a Solve the following.
a 5a + 6 = 26 b 6b + 8 = 44 c 8i − 9 = 15
d 7f − 18 = 45 e 8q + 17 = 26 f 10r − 21 = 33
g 6s + 46 = 75 h 5t − 28 = 21 i 8a + 88 = 28

Chapter 2 • Algebra and equations 43


number and algebra

8 Solve the following.


f g r
a + 6 = 16 b +4=9 c +6=5
4 6 10
m n p
d − 12 = −10 e + 5 = 8.5 f − 1.8 = 3.4
9 8 12
9 Solve the following.
a 6(x + 8) = 56 b 7(y − 4) = 35
c 5(m − 3) = 7 d 3(2k + 5) = 24
e 5(3n − 1) = 80 f 6(2c + 7) = 58
g 2(x − 5) + 3(x − 7) = 19 h 3(x + 5) − 5(x − 1) = 12
10 WE9b Solve the following.
3k 9m 7p
a = 15 b = 18 c = −8
5 8 10
8u 11x 4v
d = −3 e =2 f = 0.8
11 4 15
11 Solve each of the following.
x−5 2m + 1 3w − 1
a =7 b = −3 c =6
3 3 4
t−5 6−x 3n − 5
d =0 e = −1 f = −6
2 3 4
p
12 MC a The solution to the equation + 2 = 7 is:
5
A p=5 B p = 25 C p = 45 D p = 10 E p=1
b If 5h + 8 = 53, then h is equal to:
1
A B 12.2 C 225 D 10 E 9
5
c The exact solution to the equation 14x = 75 is:
A x = 5.357 142 857 B x = 5.357 (to 3 decimal places)
5
C x = 5 14 D x = 5.4
E x = 5.5
13 Solve the following equations.
a −x = 5 b 10 − 3v = 7 c 9 − 6l = −3
d −3 − 2g = 1 e −5 − 4t = −17 3e
f − = 14
5
8j k 4f
g − =9 h − −3=6 i − +1=8
3 4 7
14 WE10a Solve the following equations.
a 6x + 5 = 5x + 7 b 7b + 9 = 6b + 14
c 11w + 17 = 6w + 27 d 8f − 2 = 7f + 5
e 12g − 19 = 3g − 31 f 7h + 5 = 2h − 6

44 Maths Quest 10 New South Wales Australian curriculum edition Stages 5.1 and 5.2
number and algebra

15 WE10b Solve the following equations.


a 5 − 2x = 6 − x b 10 − 3c = 8 − 2c c 3r + 13 = 9r − 3
d k − 5 = 2k − 6 e 5y + 8 = 13y + 17 f 17 − 3g = 3 − g
g 14 − 5w = w + 8 h 4m + 7 = 8 − m i 14 − 5p = 9 − 2p
16 WE10c Solve the following equations.
a 3(x + 5) = 2x b 8(y + 3) = 3y
c 6(t − 5) = 4(t + 3) d 10(u + 1) = 3(u − 3)
e 12( f − 10) = 4( f − 5) f 5(2d + 9) = 3(3d + 13)
g 5(h − 3) = 3(2h − 1) h 2(4x + 1) = 5(3 − x)
17 MC a The solution to 8 − 4k = −2 is:
1 1 1 1 2
A k = 22 B k = −2 2 C k = 12 D k = −1 2 E k= 5
6n
b The solution to − + 3 = −7 is:
5
1 1 1 1 1
A n = 33 B n = −3 3 C n= D n=8 E n = −8
3 3 3
c The solution to p − 6 = 8 − 4p is:
2 4 2 2 4
A p= B p=2 C p = 43 D p= E p=
5 5 3 5

UNDERSTANDING
18 If the side length of a cube is x cm, then its volume V is given by
V = x3. What is the side length (correct to the nearest cm) of a x
cube that has a volume of:
a 216 cm3? b 2 m3 ?
19 The surface area of a cube with side length x cm is given by
A = 6x2. Find the side length (correct to the nearest cm) of a cube that has a surface
area of:
a 37.5 cm2 b 1 m2 .
20 A pebble is dropped down a well. In time t seconds it falls a distance of d metres,
given by d = 5t2.
a How far does the pebble fall in 1 s?
b How many seconds will it take the pebble to fall 40 m? (Answer correct to
1 decimal place.)
21 The surface area of a sphere is given by the formula A = 4πr2, where r is the radius
of the sphere.
a Find the surface area of a sphere that has a radius of 5 cm.
b What is the radius of a sphere that has a surface area equal to 500 cm2? (Answer
correct to the nearest mm.)
REASONING
22 Find the radius of a circle of area 10 cm2.
REFLECTION
23 The volume of a sphere is given by the formula Describe in one sentence
V = 43πr3, where r is the radius of the sphere. If the what it means to solve
sphere can hold 1 litre of water, what is its radius equations.
correct to the nearest mm?

Chapter 2 • Algebra and equations 45


number and algebra

2E  Solving multi-step equations


Equations with multiple brackets
Interactivity Many equations need to be simplified by expanding brackets and collecting like terms
Solving equations
int-2778
before they are solved. Doing this reduces the equation to one of the basic types
covered in the previous exercise.

WORKED EXAMPLE 11
Solve each of the following linear equations.
a 6(x + 1) − 4(x − 2) = 0
b 7(5 − x) = 3(x + 1) − 10

TH I N K W R I TE

a 1 Write the equation. a 6(x + 1) − 4(x − 2) = 0

2 Expand all the brackets. (Be careful 6x + 6 − 4x + 8 = 0


with the −4.)

3 Collect like terms. 2x + 14 = 0

4 Subtract 14 from both sides of the equation. 2x = −14

5 Divide both sides of the equation by 2 x = −7


to find the value of x.

b 1 Write the equation. b 7(5 − x) = 3(x + 1) − 10

2 Expand all the brackets. 35 − 7x = 3x + 3 − 10

3 Collect like terms. 35 − 7x = 3x − 7

4 Create a single pronumeral term by 35 = 10x − 7


adding 7x to both sides of the equation.

5 Add 7 to both sides of the equation. 42 = 10x

6 Divide both sides of the equation by 10 42


=x
to solve for x and simplify. 10
21
=x
5
7 Express the improper fraction as a 415 = x
mixed number fraction.

8 Rewrite the equation so that x is on the x = 415


left-hand side.

46 Maths Quest 10 New South Wales Australian curriculum edition Stages 5.1 and 5.2
number and algebra

Equations involving algebraic fractions


•• To solve an equation containing algebraic fractions, multiply both sides of the
equation by the lowest common multiple (LCM) of the denominators. This gives an
equivalent form of the equations without fractions.

WORKED EXAMPLE 12
x−5 x+7
Solve the equation = and verify the solution.
3 4

TH I N K W R I TE
1 Write the equation. x−5 x+7
=
3 4
2 The LCM is 3 × 4 = 12. 412(x − 5) 312(x + 7)
Multiply both sides of the 13
= 14
equation by 12.
3 Simplify the fractions. 4(x − 5) = 3(x + 7)
4 Expand the brackets 4x − 20 = 3x + 21
5 Subtract 3x from both sides of x − 20 = 21
the equation.
6 Add 20 to both sides of the x = 41
equation and write the
answer.
7 To verify, check that the
answer x = 41 is true for
both the left-hand side (LHS)
and the right-hand side
(RHS) of the equation by
substitution.
41 − 5
Substitute x = 41 into the LHS. LHS =
3
36
=
3
= 12
41 + 7
Substitute x = 41 into the RHS. RHS =
4
48
=
4
= 12

8 Write your answer. Because the LHS = RHS, the solution


x = 41 is correct.

Chapter 2 • Algebra and equations 47


number and algebra

WORKED EXAMPLE 13
Solve each of the following equations.
x 3x 1 5(x + 3) 3(x − 1)
a − = b = 4 +
2 5 4 6 5

TH I N K W R I TE
a 1 Write the equation. a x 3x 1
− =
2 5 4
2 The lowest common denominator x 10 3x 4 1 5
× − × = ×
of 2, 5 and 4 is 20. Write each term 2 10 5 4 4 5
as an equivalent fraction with a 10x 12x 5
− =
denominator of 20. 20 20 20
3 Multiply both sides of the equation 10x 12x 5
by 20. This is the same as multiplying a − b × 20 = × 20
20 20 20
each term by 20, which cancels 10x 12x 5
out the 20 in the denominator and × 20 − × 20 = × 20
20 20 20
effectively removes it. 10x − 12x = 5
4 Simplify the left-hand side of the − 2x = 5
equation by collecting like terms.
5
5 Divide both sides of the equation by x=−
−2 to solve for x. 2

6 Express the improper fraction as a x = −212


mixed number fraction.
b 1 Write the equation. b 5(x + 3) 3(x − 1)
=4+
6 5
2 The lowest common denominator of 25(x + 3) 120 18(x − 1)
= +
5 and 6 is 30. Write each term as an 30 30 30
equivalent fraction with a common
denominator of 30.
3 Multiply each term by 30. This 25(x + 3) = 120 + 18(x − 1)
effectively removes the
denominator.
4 Expand the brackets and collect like 25x + 75 = 120 + 18x − 18
terms. 25x + 75 = 102 + 18x
5 Subtract 18x from both sides of the 7x + 75 = 102
equation.
6 Subtract 75 from both sides of the 7x = 27
equation.
7 Divide both sides of the equation by 27
x=
7 to solve for x. 7
8 Express the improper fraction as a x = 367
mixed number.

48 Maths Quest 10 New South Wales Australian curriculum edition Stages 5.1 and 5.2
number and algebra

Exercise 2E Solving multi-step equations


INDIVIDUAL PATHWAYS
Questions: Questions: Questions:
1, 2a–c, 3a–b, 4a–b, 5, 6 1, 2a–f, 3a–d, 4c–f, 6, 7, 10 1, 2c–h, 3c–f, 4e–h, 7, 8, 9, 10
Activity 2-E-1 Activity 2-E-2 Activity 2-E-3
Algebraic equations Harder algebraic equations Tricky algebraic equations
with fractions with fractions with fractions
doc-4972 doc-4973 doc-4974

FLUENCY
1 WE11 Solve each of the following linear equations.
a 6(4x − 3) + 7(x + 1) = 9 b 9(3 − 2x) + 2(5x + 1) = 0
c 8(5 − 3x) − 4(2 + 3x) = 3 d 9(1 + x) − 8(x + 2) = 2x
e 6(4 + 3x) = 7(x − 1) + 1 f 10(4x + 2) = 3(8 − x) + 6

2 WE12 Solve each of the following equations and verify each solution.
x+1 x+3 x−7 x−8
a = b =
2 3 5 4
x−6 x−2 8x + 3
c = d = 2x
4 2 5
2x − 1 x − 3 4x + 1 x + 2
e = f =
5 4 3 4
6 − x 2x − 1 8 − x 2x + 1
g = h =
3 5 9 3
3 WE13a Solve each of the following linear equations.
x 4x 1 x x 3
a + = b − =
3 5 3 4 5 4
x 4x −3x x 1
c − =2 d + =
4 7 5 8 4
2x x −3 5x 2x
e − = f −8=
3 6 4 8 3
4 WE13b Solve each of the following linear equations.
3(x + 1) 5(x + 1) 2(x + 1) 3(2x − 5)
a + =4 b + =0
2 3 7 8
2(4x + 3) 6(x − 2) 1 8(x + 3) 3(x + 2)
c − = d =
5 2 2 5 4
5(7 − x) 2(2x − 1) 2(6 − x) 9(x + 5) 1
e = +1 f = +
2 7 3 6 3
Digital doc
−5(x − 2) 6(2x − 1) 1 9(2x − 1) 4(x − 5) WorkSHEET 2.2
g − = h = doc-13703
3 5 3 7 3

Chapter 2 • Algebra and equations 49


number and algebra

UNDERSTANDING
5 Last week Maya broke into her money box. She spent one-quarter of the money
on a birthday present for her brother and one-third of the money on an evening out
with her friends, leaving her with $75. How much money was in her money box?
6 At work Keith spends one-fifth of his time in planning and buying merchandise. He
spends seven-twelfths of his time in customer service and one-twentieth of his time
training the staff. This leaves him ten hours to deal with the accounts. How many
hours does he work each week?
7 Last week’s school fete was a great success, raising a good deal of money. Three-
eighths of the profit came from sales of food and drink, and the market stalls
recorded one-fifth of the total. A third of the profit came from the major raffle, and
the jumping castle raised $1100. How much money was raised at the fete?
8 Michael’s dad is currently 3 times as old as his son, but in 10 years from now he
will only be twice as old. How old is Michael?
9 Lucy had half as much money as Mel, but since Grandma gave them each $20 she
now has three-fifths as much. How much money does Lucy have?
REASONING
10 a Which numbers less than 100 have exactly 3 factors (including 1 and the
number itself )?
b Which two numbers less than 100 have exactly 5 factors?
c Which number less than 100 has exactly 7 factors?
11 To raise money for a charity, a Year 10
class has decided to organise a school
luncheon. Tickets will cost $6 each.
The students have negotiated a special
deal for delivery of drinks and pizzas,
and they have budgeted $200 for drinks
and $250 for pizzas. If they raise $1000
or more, they qualify for a special
award.
a Write an equation to represent this
situation. REFLECTION
Do the rules for the order
b Solve the equation to find the number of of operations apply to
tickets they must sell to qualify for the algebraic fractions?
award. Explain your answer. Explain.

50 Maths Quest 10 New South Wales Australian curriculum edition Stages 5.1 and 5.2
number and algebra

CHAPTER REVIEW
LANGUAGE

algebra evaluate formula pronumeral


denominator expression multiple simplify
equation factor numerator substitution

int-2829 doc-13705
int-2830 doc-13706
int-3589

FLUENCY
1
1 Given E = 2mv2 where m = 0.2 and v = 0.5, the value of E is:
A 0.000 625 B 0.1 C 0.005 D 0.025 E 0.0025
2 The expression −6d + 3r − 4d − r simplifies to:
A 2d + 2r B −10d + 2r C −10d − 4r D 2d + 4r E −8dr
3 The expression 5(2f + 3) + 6(4f − 7) simplifies to:
A 34f + 2 B 34f − 4 C 34f − 27 D 34f + 14 E 116f − 14
4 The expression 7(b − 1) − (8 − b) simplifies to:
A 8b − 9 B 8b − 15 C 6b − 9 D 6b − 15 E 8b + 1
5 If 14p − 23 = 6p − 7 then p equals:
A −3 B −1 C 1 D 2 E 4
6 Simplify the following by collecting like terms.
a 3c − 5 + 4c − 8 b −3k + 12m − 4k − 9m
c −d + 3c − 8c − 4d d 6y2 + 2y + y2 − 7y
1
7 If A = 2bh, determine the value of A if b = 10 and h = 7.

8 Simplify the following.


5y y x+4 x+2
a − b +
3 2 5 2
5 1 x − 1 2x − 5
c − d +
3x 5x x+3 x+2
9 Simplify the following.
y 32 20y 35z
a × b ×
4 x 7x 16y
x+6 5(x + 1) 25 30
c × d ÷
(x + 1)(x + 3) x+6 x x
xy 10x 2x 9x + 1
e ÷ f ÷
5 y (x + 8)(x − 1) x+8

Chapter 2 • Algebra and equations 51


number and algebra

10 Solve the following equations.


a p − 20 = 68 b s − 0.56 = 2.45
r
c 3b = 48 d = −5
7
e !x = 12 f x3 + 5 = 348
y
g − 3 = 12 h a2 = 36
4
i 5 − k = −7 j
3
" x + 4 = −2
11 Solve the following.
a 42 − 7b = 14 b 12t − 11 = 4t + 5
c 2(4p − 3) = 2(3p − 5)
12 Solve each of the following linear equations.
a 5(x − 2) + 3(x + 2) = 0 b 7(5 − 2x) − 3(1 − 3x) = 1
c 5(x + 1) − 6(2x − 1) = 7(x + 2) d 8(3x − 2) + (4x − 5) = 7x
e 7(2x − 5) − 4(x + 20) = x − 5 f 3(x + 1) + 6(x + 5) = 3x + 40
13 Solve each of the following equations.
x x 3 x x
a + = b − =3
2 5 5 3 5
1 x x 2x − 1 x − 3
c −21 = − d − =5
7 6 4 6
2x − 3 x + 3
e =
2 5

PROBLEM SOLVING
1 A production is in town and many parents are taking their children. An adult ticket
costs $15 and a child’s ticket costs $8. Every child must be accompanied by an adult
and each adult can have no more than 4 children with them. It costs the company
$12 per adult and $3 per child to run the production. There is a seating limit of
300 people and all tickets are sold.
a Determine how much profit the company makes on each adult ticket and on each
child’s ticket.
b To maximise profit, the company should sell as many children’s tickets as
possible. Of the 300 available seats, determine how many should be allocated to
children if there is a maximum of 4 children per adult.
c Using your answer to part b, determine how many adults would make up the
remaining seats.
d Construct an equation to represent the profit that the company can make
depending on the number of children and adults attending the production.
e Substitute your values to calculate the maximum profit the company can make.
2 You are investigating prices for having business cards printed for your new games
store. A local printing company charges a flat rate of $250 for the materials used and
$40 per hour for labour.
a If h is the number of hours of labour required to print the cards, construct an
equation for the cost of the cards, C.

52 Maths Quest 10 New South Wales Australian curriculum edition Stages 5.1 and 5.2
number and algebra

b You have budgeted $1000 for the printing job. How many hours of labour can you
afford? Give your answer to the nearest minute.
c The printer estimates that it can print 1000 cards per hour of labour. How many
cards will be printed with your current budget?
d An alternative to printing is photocopying. The company charges 15 cents per
side for the first 10 000 cards and then 10 cents per side for the remaining cards.
Which is the cheaper option for 18 750 single-sided cards and by how much?
3 A scientist tried to use a mathematical formula to
predict people’s moods based on the number of hours
of sleep they had the previous night. One formula that
he used was what he called the ‘grumpy formula’,
g = 0.16(h − 8)2, which was valid on a ‘grumpy scale’
from 0 to 10 (least grumpy to most grumpy).
a Calculate the number of hours needed to not be
grumpy.
b Evaluate the grumpy factor for somebody who has
had:
     i 4 hours of sleep
   ii 6 hours of sleep
iii 10 hours of sleep.
c Calculate the number of hours of sleep required to be most grumpy.
4 Another scientist already had his own grumpy formula and claims that the scientist
from question 3 stole his idea and has just simplified it. The second scientist’s
grumpy formula was
0.16(h − 8) 2(8 − h) 2h
g= × ÷ .
8−h 3(h − 8) 3(h − 8) 2
a Write the second scientist’s formula in simplified form.
b Are the second scientist’s claims justified? Explain.

Chapter 2 • Algebra and equations 53


number and algebra
Communicating

Rich task

Checking for data entry errors

mu lti ple of 9,
ct nu mb er an d th e re co rd ed nu mb er is a
ee n th e co rre
If th e dif fer en ce be tw
ha s oc cu rre d.
a tra ns po sit ion er ro r

54 Maths Quest 10 New South Wales Australian curriculum edition Stages 5.1 and 5.2
number and algebra

We can use algebraic expressions to check this


rule. Let the digit in the thousands position be
represented by a, the digit in the hundreds position
by b, the digit in the tens position by c and the digit
in the ones position by d. So the real number can
be represented as 1000a + 100b + 10c + d.
1 If the digits in the ones position and the tens
position were written in the reverse order, the
number would be 1000a + 100b + 10d + c. The
difference between the correct number and
the incorrect one would then be: 1000a + 100b +
10c + d − 1 1000a + 100b + 10d + c 2 .
a Simplify this expression.
b Is the expression a multiple by 9? Explain.
2 If a transposition error had occurred in the tens
and hundreds position, the incorrect number
would be 1000a + 100c + 10b + d. Perform the
procedure shown in question 1 to determine
whether the difference between the correct
number and the incorrect one is a multiple of 9.
3 Consider, lastly, a transposition error in the
thousands and hundreds positions. Is the
difference between the two numbers a multiple
of 9?
4 Comment on the checking rule for transposition
errors.

Chapter 2 • Algebra and equations 55


number and algebra

Code puzzle

What was Sir Isaac Newton’s


most famous work?
If a = 3, b = –5 and c = –2, find the value of the letter in the following substitution
problems. The answers and the letters give the puzzle’s answer code.

I
(6 + a)2
––––––
a

N P A
3a3 + 7c 7a + bc –2(8c + 3a)

C M 7(a + 1)
T
c + ab2 c2 + –––––– –4b – c
2

H E R
6 12a 2a(ac – 2b) (3a)2 – 2b – 2c

31 95 27 67 73 27 31 27 14

18 14 22 36 24 18 14 22 27 73 14

56 Maths Quest 10 New South Wales Australian curriculum edition Stages 5.1 and 5.2
number and algebra

 ACTIVITIES
Go to assessON
Chapter opener • SkillSHEET (doc-5187): Multiplication of for questions to
Video fractions (page 38) test your readiness
• The story of mathematics (eles-1841) • SkillSHEET (doc-5191): Simplification of FOR learning, your
algebraic fractions (page 38) progress AS you learn
2A Substitution
• SkillSHEET (doc-5192): Division of and your levels OF
Digital docs
fractions (page 38) achievement.
• Activity 2-A-1 (doc-4960): Substitution
• SkillSHEET (doc-5189): Order of www.assesson.com.au
(page 30)
operations (page 38)
• Activity 2-A-2 (doc-4961): Harder substitution
• WorkSHEET 2.1 (doc-13701): Algebraic
(page 30)
fractions (page 39)
• Activity 2-A-3 (doc-4962): Tricky substitution
(page 30) 2D Solving simple equations
• SkillSHEET (doc-5183): Like terms (page 31) Digital docs  (page 43)
• SkillSHEET (doc-5184): Collecting like terms • Activity 2-D-1 (doc-4969): Simple puzzling
(page 31) equations
• SkillSHEET (doc-5185): Finding the highest • Activity 2-D-2 (doc-4970): Puzzling equations
common factor (page 31) • Activity 2-D-3 (doc-4971): Advanced puzzling
• SkillSHEET (doc-5189): Order of operations equations
(page 31)
2E Solving multi-step equations
2B Adding and subtracting algebraic Digital docs
fractions • Activity 2-E-1 (doc-4972): Algebraic
Digital docs  (page 35) equations with fractions (page 49)
• Activity 2-B-1 (doc-4963): Introducing • Activity 2-E-2 (doc-4973): Harder algebraic
algebraic fractions equations with fractions (page 49)
• Activity 2-B-2 (doc-4964): Working with • Activity 2-E-3 (doc-4974): Tricky algebraic
algebraic fractions equations with fractions (page 49)
• Activity 2-B-3 (doc-4965): Advanced • WorkSHEET 2.2 (doc-13703): Solving
algebraic fractions equations with fractions (page 49)
• SkillSHEET (doc-5186): Addition and Interactivity
subtraction of fractions • Solving equations (int-2778) (page 46)
• SkillSHEET (doc-5190): Writing equivalent
Chapter review
algebraic fractions with the lowest common
Interactivities  (page 51)
denominator
• Word search (int-2829)
2C Multiplying and dividing algebraic • Crossword (int-2830)
fractions • Sudoku (int-3589)
Digital docs Digital docs
• Activity 2-C-1 (doc-4966): Learning • Chapter summary (doc-13705)
operations with algebraic fractions • Concept map (doc-13706)
(page 38)
To access eBookPLUS activities, log on to
• Activity 2-C-2 (doc-4967): Operations with
www.jacplus.com.au
algebraic fractions (page 38)
• Activity 2-C-3 (doc-4968): Advanced
operations with algebraic fractions
(page 38)

Chapter 2 • Algebra and equations 57


number and algebra

Answers
Chapter 2 Algebra and equations
Exercise 2A — Substitution
5y 3y 13x
1    a 5 b 2 c 0 2    a b − c
d 6 e −17 f 3 12 40 12
g 30 h 12 i −12 14x 3w y
d e f −
j 27 k 30 l −5 9 28 5
2    a −11 b −1 c 1 89y 32x 7x + 17
d 30 e −24 f 36 g h i
35 15 10
g −125 h 1 i 15
7x + 30 2x − 11 19x + 7
7 1
b −12 c 1 j k l
3    a 12 12 12 30 6
d 113 e 1
f 48 5 5 38
576 3    a b c
8x 12x 21x
4    a 9 b −9 c 9
8 7 9
d 18 e −18 f 36 d e f
5    a 9 b −9 c 9 3x 24x 20x
d 18 e −18 f 36 37 51 1
g h i −
6    a 17 b 30 c 8 100x 10x 6x
d 4 e 1.5 f 68 x 29x − 16
g 46 h 113.1 i 5 4    a b
12 30
j 624.6
17m − 10 13x
7    a D b C c B c d
8 3.9 cm 6 15
9 65.45 cm3 8 − 3x 3y + x
5    a b
10 361 m 4a xy
11    a −1 — in this case, addition is closed on integers. 8xy − 5y + x 3x2 − 5x + 5y2 + y
b −1 — in this case, subtraction is closed on integers. c d
xy xy
c 2 — in this case, multiplication is closed on integers.
d −1 — in this case, division is closed on integers. Exercise 2C — Multiplying and dividing algebraic fractions
e −2 — in this case, subtraction is closed on integers. 4x 3x 4y
f −12 — in this case, division is not closed on integers. 1    a b c
y y x
12    a 10 — in this case, addition is closed on natural numbers. 9x −5x 3w
d e f
b −4 — in this case, subtraction is not closed on natural 4y 4y 2x
numbers. 6z 2z −3x
g h i
c 12 — in this case, multiplication is closed on natural numbers. 7x 7x 2y
d 43 — in this case, division is not closed on natural numbers. 5 12z −x
j k l
e −2 — in this case, subtraction is not closed on natural 24 x 6w
numbers.
2 5 9
f 4 — in this case, division is closed on natural numbers. 2    a b c
13    a (a + 2b) + 4c = a + (2b + 4c) 3x − 2 x−3 2(x − 6)
b (x × 3y) × 5c = x × (3y × 5c) 1 2x x+1
d e f
c 2p ÷ q ≠ q ÷ 2p x+3 (x + 1) 2 2(2x − 3)
d 5d + q = q + 5d a 35d 9
g h i
e 3z + 0 = 0 + 3z = 3z 10(a + 3) 8(d − 3) 32x2 (x − 2)
1 1 3x
f 2x × = × 2x = 1 j
2x 2x 10(x − 1)
g (4x ÷ 3y) ÷ 5z ≠ 4x ÷ (3y ÷ 5z)
3 2 1
h 3d − 4y ≠ 4y − 3d 3    a 5
b c 3
9
1 35
Challenge 2.1 d 3 e 25
f 6
or 556
3a3c2
4y2 2y2 8y2
g h i
Exercise 2B — Adding and subtracting algebraic fractions 7 25 9
26 5 49 32xy 2
1    a or 121 b c 1 j k 3
l y2
21 72 15
17 1 6 − 5x 4    a
9
b
1
d e f
99 35 30 (3x − 7) (x + 3) (x + 2) (x − 9)
15x − 4 15 − 16x 15 − 2x 21(x − 3) 13
g h i c d
27 40 3x x+5 9(x − 4) (x + 1)

58 Maths Quest 10 New South Wales Australian curriculum edition Stages 5.1 and 5.2
number and algebra

Challenge 2.2 20    a 5m b 2.8 s


The number is doubled and then subtracted from itself to give zero. 21    a 314 cm2 b 6.3 cm
The answer will always be the first (add 7) and last (multiply by 3) 22 1.8 cm
instructions. 23 6.2 cm
Exercise 2D — Solving simple equations Exercise 2E — Solving multi‐step equations
1    a a = 24 b k = 121 c g = 2.9
d r=3 e h = 0.26 f i = −2 1    a x = 20 b x = 358 c x = 29
1 31 36
g t=5 h q= 6
i x=0
8
d x = −7 e x = −211 f x = 10
2    a f = 12 b i = −60 c z = −7 43

d v=7 e w = −513 f k = 10 2    a x = 3 b x = 12 c x = −2

g a = 0.425 h m= 1658 i y= 2112 d x= 3


2
e x= −11
3
or x = −323

3    a t = 100 b y = ±17 c q = 6.25 2 5


f x = 13 g x=3 h x=
16 7
d f = ±1.2 e h= f p= ±38 5
49
3    a x = 17 b x = 15 c x = −629
g g = 225 h j = ±14 i a = ±1 23
484 31 d x = −10
19
e x = −112 f x = −192
4    a t = 25 b x = ±6 c m = 16
5
d t = ±3 e t = ±10 f m = 25 4    a x = 19 b x = 131
58
c x = 411
14
1
5    a x = 8 b x = −3 c m= d x = −315
8 17 e x = 520
43
f x = −110
13
3
d x= e m = 0.008 f w = 212
4 2 9
g x = 161 h x = −426
6    a x = −1 b x = −2 c m=1
d w = −512 e t = 125 f x=2 5 $180
7    a a=4 b b=6 c i=3 6 60 inches
d f=9 e q = 118 f r = 525 7 $12000
g s = 456 h t = 945 i a = −712 8 10 years old
9 $60
8    a f = 40 b g = 30 c r = −10 10    a 4, 9, 25, 49 b 16, 81 c 64
d m = 18 e n = 28 f p = 62.4 11    a 6x − 450 = 1000
9    a x = 113 b y=9 c m = 425 b 24123 tickets. This means they need to sell 242 tickets to
d k= 112 e n= 523 f c = 113 qualify, as the number of tickets must be a whole number.

g 10 h x=4 Chapter review


10    a k = 25 b m = 16 c p= −1137 Fluency
1 D 2 B 3 C 4 B 5 D
d u= −418 e x= 8
11
f v=3 6    a 7c − 13 b −7k + 3m c −5d − 5c d 7y2 − 5y
11    a x = 26 b m = −5 c w = 25 7 35
3 7y 7x + 18
d t=5 e x=9 f n = −19 8    a b
3 6 10
12    a B b E c C 22 3x2 + 2x − 17
c d
13    a x = −5 b v=1 c l=2 15x (x + 3) (x + 2)
8y 25z 5
d g = −2 e t=3 f e = –2313 9    a b c
x 4x x+3
g j = −338 h k = −36 i f = −1214 5 y2 2x
d 6 e f
14    a x = 2 b b=5 c w=2 50 (x − 1) (9x + 1)
d f=7 e g = −113 f h = −215 10    a p = 88 b s = 3.01 c b = 16
d r = −35 e x = 144 f x=7
15    a x = −1 b c=2 c r = 223
g y = 60 h a = ±6 i k = 12
d k=1 e y = −118 f g=7 j x = −2, 6
1
g w=1 h m= 5
i p = 123 11    a b = 4 b t=2 c p = −2
16    a x = −15 b y = −445 c t = 21 12    a x = 1
b x= 615
3
c x = −14
2
d u = −257 e f = 1212 f d = −6
d x=1 e x = 1229 f x = 116
g h = −12 h x=1
17    a A b D c B 13    a x = 67 b x = 2212 c x=2
18    a 6 cm b 1.26 m
d x = 1414 e x = 258
19    a 2.5 cm b 41 cm

Chapter 2 • Algebra and equations 59


number and algebra

Problem solving Communicating — Rich task


1    a $3 per adult ticket; $5 per child's ticket 1    a 9(c − d)
b 240 b Yes, this is a multiple of 9 as the number that multiplies the
c 60 brackets is 9.
d P = 3a + 5c, where a = number of adults and 2 90(b − c); 90 is a multiple of 9 so the difference between the
c = number of children correct and incorrect one is a multiple of 9.
e $1380 3 900(a − b); again 900 is a multiple of 9.
2    a C = 250 + 40h 4 If two adjacent digits are transposed, the difference between the
b 18 hours 45 minutes correct number and the transposed number is a multiple of 9.
c 18750 Code puzzle
d Printing is the cheaper option by $1375.
Principia mathematica
3    a h = 8 hours
b i g = 2.56
ii g = 0.64
iii g = 0.64
c h = 0.0943 hours or h = 15.9 hours
0.16(h − 8) 2
4    a g =
h
b No, the formula is not the same.

60 Maths Quest 10 New South Wales Australian curriculum edition Stages 5.1 and 5.2
number and
NUMBER AND ALGEBRA
algebra

Chapter 3

Coordinate
geometry
WHY LEARN THIS?
What did you weigh as a baby, and how tall were you?
Did you grow at a steady (linear) rate, or were there
periods in your life when you grew rapidly? What is the
relationship between your height and your weight? We
constantly seek to find relationships between variables,
and coordinate geometry provides a picture, a visual
clue as to what the relationships may be.

WHAT DO YOU KNOW?


1 Think List what you know about linear
graphs and their equations. Use a thinking
tool such as a concept map to show your list.
2 pair Share what you know with a partner
and then with a small group.
3 share As a class, create a large concept
map that shows your class's knowledge
of linear graphs and their equations.

LEARNING SEQUENCE
3A Sketching linear graphs
3B Determining linear equations
3C The distance between two points
3D The midpoint of a line segment
3E Parallel and perpendicular lines
Watch this video
The story of mathematics

Searchlight ID: eles-1842


number and algebra

3A  Sketching linear graphs


y
•• If a series of points (x, y) is plotted using the rule y = mx + b, 10 y = 2x + 5
then the points always lie in a straight line whose gradient Quadrant 2 Quadrant 1
5
equals m and whose y‐intercept equals b.
•• The rule y = mx + b is called the equation of a straight line
written in “gradient–intercept form.” –10 –5 0 5 10 x
Quadrant 3
•• A graph of the line y = 2x + 5 is shown in the figure at right. –5
Quadrant 4
–10
Plotting linear graphs
•• To plot a linear graph, complete a table of value to determine the points.

WORKED EXAMPLE 1
Plot the linear graph defined by the rule y = 2 x − 5 for the x‐values −3,
−2, −1, 0, 1, 2, 3.

TH I N K W R I TE
1 Create a table of values using x −3 −2 −1 0 1 2 3
the given x‐values.
y

2 Find the corresponding x −3 −2 −1 0 1 2 3


y‐values by substituting each
x‐value into the rule. y −11 −9 −7 −5 −3 −1 1

3 Plot the points on a Cartesian y


2
plane and rule a straight line 1
(3, 1)
through them. Since the
x‐values have been specified, –3 –2 –1–10 1 2 3x
(2, –1)
the line should only be drawn –2
–3 (1, –3)
between the x‐values of −3 –4
and 3. –5 (0, –5)
–6
(–1, –7) –7
–8
(–2, –9) –9
–10
(–3, –11) y = 2x – 5
–11
–12

4 Label the graph.

Sketching straight lines


•• A minimum of two points are necessary to plot a straight line.
•• Two methods can be used to plot a straight line:
–– Method 1: The x‐ and y‐intercept method.
–– Method 2: The gradient–intercept method.

64 Maths Quest 10 New South Wales Australian curriculum edition Stages 5.1 and 5.2
number and algebra

Method 1: Sketching a straight line using the x- and


y-intercepts
•• As the name implies, this method involves plotting the x‐ and y‐intercepts, then
joining them to sketch the straight line.
•• The line cuts the y‐axis where x = 0 and the x‐axis where y = 0.

WORKED EXAMPLE 2
Sketch graphs of the following linear equations by finding the x‐ and
y‐intercepts.
a 2x + y = 6 b y = −3x − 12

TH I N K W R I TE /DRAW
a 1 Write the equation. a 2x + y = 6
2 Find the x‐intercept by x-intercept: when y = 0,
substituting y = 0. 2x + 0 = 6
2x = 6
x=3
x-intercept is (3, 0).
3 Find the y‐intercept by y-intercept: when x = 0,
substituting x = 0. 2(0) + y = 6
y=6
y-intercept is (0, 6).
4 Plot both points and rule y
the line. 2x + y = 6
(0, 6)

0 (3, 0) x

5 Label the graph.


b 1 Write the equation. b y = −3x − 12
2 Find the x‐intercept by x-intercept: when y = 0,
substituting y = 0. −3x − 12 = 0
  i Add 12 to both sides of the −3x = 12
equation. x = −4
i i Divide both sides of the x-intercept is (−4, 0).
equation by −3.
3 Find the y‐intercept. The b = −12
equation is in the form y-intercept is (0, −12).
y = mx + b, so compare this
with our equation to find the
y‐intercept, b.

Chapter 3 • Coordinate geometry 65


number and algebra

4 Plot both points and rule y


the line.

(–4, 0)
0 x

(0, –12) y = –3x – 12

5 Label the graph.

Method 2: Sketching a straight line using the


gradient–intercept method
•• This method is often used if the equation is in the form y = mx + b, where
m represents the gradient (slope) of the straight line, and b represents the
y‐intercept.
•• The steps below outline how to use the gradient–intercept method to sketch a linear
graph.
Step 1: Plot a point at the y‐intercept.
rise
Step 2: Write the gradient in the form m = . (To write a whole number as a
run
fraction, place it over a denominator of 1.)
Step 3: Starting from the y‐intercept, move up the number of units suggested by the
rise (move down if the gradient is negative).
Step 4: Move to the right the number of units suggested by the run and plot the
second point.
Step 5: Rule a straight line through the two points.

WORKED EXAMPLE 3
Sketch the graph of y = 25 x − 3 using the gradient–intercept method.

TH I N K W R I TE /DRAW
1 Write the equation of the line. y = 25x − 3

2 Identify the value of b (that is, the b = −3, so y‐intercept: (0, −3).
y‐intercept) and plot this point.
2
3 Write the gradient, m, as a fraction. m = 5
rise
4 m= , note the rise and run. So, rise = 2; run = 5.
run

66 Maths Quest 10 New South Wales Australian curriculum edition Stages 5.1 and 5.2
number and algebra

5 Starting from the y‐intercept y


(0, −3), move 2 units up and 0
–1 1 2 3 4 5 6 7 8
x
5 units to the right to find the –2 (5, –1)
second point (5, −1). –3 (0, –3)
–4

6 To complete the graph, find the y = 25x − 3


x‐intercept by substituting y = 0.
0 = 25x − 3
3 = 25x
3 × 52 = x
x = 15
2
15
Q
2
, 0 R is the x-intercept.

7 Label the graph and draw a line y (152 , 0)


through all the points found. 0 x
–1 1 2 3 4 5 6 7 8
–2 (5, –1)
–3 (0, –3) y = 25 x – 3
–4

Sketching linear graphs of the form y = mx


•• Graphs given by y = mx pass through the origin (0, 0), since b = 0.
•• A second point may be determined by using the rule y = mx and substituting a value
for x to find y.

WORKED EXAMPLE 4
Sketch the graph of y = 3x.

TH I N K W R I TE /DRAW
1 Write the equation. y = 3x

2 Find the x‐ and y‐intercepts. x‐intercept: when y = 0,


Note: By recognising the form of this 0 = 3x
linear equation, y = mx you can simply x=0
state that the graph passes through the y‐intercept: (0, 0)
origin, (0, 0).
3 Find another point to plot by finding When x = 1, y = 3 × 1
the y‐value when x = 1.       = 3
Another point on the line is (1, 3).

Chapter 3 • Coordinate geometry 67


number and algebra

Plot the two points (0, 0) and (1, 3) and y


4 y = 3x
rule a straight line through them.
3 (1, 3)

(0, 0) x
1

5 Label the graph.

Sketching linear graphs of the form y = b and x = a


•• The line y = b is parallel to the x‐axis, having a gradient of zero and a
y‐intercept of b.
•• The line x = a is parallel to the y‐axis and has an undefined (infinite) gradient.

WORKED EXAMPLE 5
Sketch graphs of the following linear equations.
a y = −3 b x=4

TH I N K W R I TE /DRAW
a 1 Write the equation. a y = −3
2 The y‐intercept is −3. As x does not y-intercept = −3, (0, −3)
appear in the equation, the line is parallel
to the x‐axis, such that all points on the
line have a y‐coordinate equal to −3.
That is, this line is the set of points
(x, −3) where x is an element of the set
of real numbers.
3 Sketch a horizontal line through (0, −3). y

0 x

(0, –3) y = –3

4 Label the graph.


b 1 Write the equation. b x=4
2 The x‐intercept is 4. As y does not appear x-intercept = 4, (4, 0).
in the equation, the line is parallel to the
y‐axis, such that all points on the line
have an x‐coordinate equal to 4. That is,
this line is the set of points (4, y) where
y is an element of the set of real numbers.

68 Maths Quest 10 New South Wales Australian curriculum edition Stages 5.1 and 5.2
number and algebra

3 Sketch a vertical line through (4, 0). y x=4

0 (4, 0) x

4 Label the graph.

Exercise 3A Sketching linear graphs


INDIVIDUAL PATHWAYS
Questions: Questions: Questions:
1, 2, 3a–h, 4a–e, 5a–d, 6a–f, 1, 2, 3f–m, 4a–e, 5a–d, 6a–f, 1, 2, 3h–o, 4d–i, 5c–f, 6e–i,
7a–d, 8a–d, 9 7c–f, 8a–f, 9 7d–h, 8c–h, 9
Activity 3‐A‐1 Activity 3‐A‐2 Activity 3‐A‐3
Sketching linear graphs Graphs of linear equations More graphs of
doc‐4975 doc‐4976 linear equations
doc‐4977

FLUENCY
1 WE1 Generate a table of values and then plot the linear graphs defined by the Digital docs
following rules for the given range of x‐values. SkillSHEET
Describing the gradient
Rule x‐values of a line
doc‐5197
a y = 10x + 25 −5, −4, −3, −2, −1, 0, 1 SkillSHEET
Plotting a line using a
b y = 5x − 12 −1, 0, 1, 2, 3, 4 table of values
doc‐5198
c y = −0.5x + 10 −6, −4, −2, 0, 2, 4 SkillSHEET
Stating the y‐intercept
d y = 100x − 240 0, 1, 2, 3, 4, 5 from a graph
doc‐5199
e y = −5x + 3 −3, −2, −1, 0, 1, 2 SkillSHEET
Substitution into a
f y = 7 − 4x −3, −2, −1, 0, 1, 2 linear rule
doc‐5202
SkillSHEET
2 Plot the linear graphs defined by the following rules for the given range of x‐values. Solving linear equations
that arise when finding
x‐ and y‐intercepts
Rule x‐values doc‐5200

a y = −3x + 2
SkillSHEET
x −6 −4 −2 0 2 4 6 Transposing linear
equations to standard
y form
doc‐5203
b y = −x + 3 x −3 −2 −1 0 1 2 3
y

c y = −2x + 3 x −6 −4 −2 0 2 4 6
y

Chapter 3 • Coordinate geometry 69


number and algebra

3 WE2 Sketch graphs of the following linear equations by finding the x- and
y-intercepts.
a 5x − 3y = 10 b 5x + 3y = 10
c −5x + 3y = 10 d −5x − 3y = 10
e 2x − 8y = 20 f 4x + 4y = 40
g −x + 6y = 120 h −2x + 8y = −20
i 10x + 30y = −150 j 5x + 30y = −150
k −9x + 4y = 36 l 6x − 4y = −24
m y = 2x − 10 n y = −5x + 20
o y= −12x
−4
4 WE3 Sketch graphs of the following linear equations using the gradient–intercept
method.
a y = 4x + 1 b y = 3x − 7
c y = −2x + 3 d y = −5x − 4
1 2
e y = 2x − 2 f y = −7x + 3
g y = 0.6x + 0.5 h y = 8x
i y=x−7
5 WE4 Sketch the graphs of the following linear equations.
a y = 2x b y = 5x
1
c y = −3x d y = 2x
2 5
e y = 3x f y = −2x
6 WE5 Sketch the graphs of the following linear equations.
a y = 10 b y = −10
c x = 10 d x = −10
e y = 100 f y=0
g x=0 h x = −100
i y = −12
7 Transpose each of the equations to standard form (that is, y = mx + b). State the
x‐ and y‐intercept for each.
a 5(y + 2) = 4(x + 3) b 5(y − 2) = 4(x − 3)
c 2(y + 3) = 3(x + 2) d 10(y − 20) = 40(x − 2)
e 4(y + 2) = −4(x + 2) f 2(y − 2) = −(x + 5)
g −5(y + 1) = 4(x − 4) h 8(y − 5) = −4(x + 3)
i 5(y + 2.5) = 2(x − 3.5) j 2.5(y − 2) = −6.5(x − 1)

UNDERSTANDING
8 Find the x‐ and y‐intercepts of the following lines.
a −y = 8 − 4x b 6x − y + 3 = 0 REFLECTION
What types of straight lines
c 2y − 10x = 50
have an x- and y-intercept
9 Explain why the gradient of a horizontal line is equal of the same value?
to zero and the gradient of a vertical line is not defined.

70 Maths Quest 10 New South Wales Australian curriculum edition Stages 5.1 and 5.2
number and algebra

3B  Determining linear equations


Finding a linear equation given two points
on the line
•• The gradient of a straight line can be calculated y
from the coordinates of two points (x1, y1) and y2 B
(x2, y2)
(x2, y2) that lie on the line.
rise rise = y2 – y1
Gradient = m =
run A
y1
y2 − y1 (x1, y1)
run = x2 – x1
     =
x2 − x1 0 x1 x2 x

•• The equation of the straight line can then be


found in the form y = mx + b, where b is the
y‐intercept.

WORKED EXAMPLE 6
Find the equation of the straight line shown in the graph.
y

0 3 x

TH I N K W R I TE
1 There are two points given on the straight line: (0, 6), 1 3, 0 2
the x‐intercept (3, 0) and the y‐intercept (0, 6).
rise
2 Find the gradient of the line by applying the m=
run
rise y2 − y1
formula m = = , where (x1, y1) = (3, 0) y2 − y1
run x2 − x1
=
and 1 x2, y2 2 = (0, 6). x2 − x1
6−0
=
0−3
6
=
−3
= −2
The gradient m = −2.
3 The graph has a y‐intercept of 6, so b = 6. y = mx + b
Substitute m = −2, and b = 6 into y = mx + b to y = −2x + 6
find the equation.

Chapter 3 • Coordinate geometry 71


number and algebra

WORKED EXAMPLE 7
Find the equation of the straight line shown in the graph.
y

1 (2, 1)

0 2 x

TH I N K W R I TE
1 There are two points given on the straight line: (0, 0), (2, 1)
the x‐ and y‐intercept (0, 0) and another
point (2, 1).
rise
2 Find the gradient of the line by applying m=
run
rise y2 − y1
the formula m = = , where y − y1
run x2 − x1 = 2
x2 − x1
(x1, y1) = (0, 0) and (x2, y2) = (2, 1).
1−0
=
2−0
1
=
2
The gradient m = 12.

3 The y‐intercept is 0, so b = 0. Substitute m = 12 y = mx + b


and b = 0 into y = mx + b to determine the y = 12x + 0
equation.
y = 12x

WORKED EXAMPLE 8
Find the equation of the straight line passing through (−2, 5) and (1, −1).

TH I N K W R I TE
1 Write the general equation of a straight line. y = mx + b
y − y1
2 Write the formula for calculating the gradient of m= 2
x2 − x1
a line between two points.
−1 − 5
3 Let (x1, y1) and (x2, y2) be the two points (−2, 5) m=
and (1, −1) respectively. Substitute the values of 1 − −2
−6
the pronumerals into the formula to calculate the =
gradient. 3
= −2
4 Substitute the value of the gradient into the y = −2x + b
general rule.

72 Maths Quest 10 New South Wales Australian curriculum edition Stages 5.1 and 5.2
number and algebra

5 Select either of the two points, say (1, −1), and Point (1, −1):
substitute its coordinates into y = −2x + b. −1 = −2 × 1 + b
6 Solve for b; that is, add 2 to both sides of the −1 = −2 + b
equation. 1=b
7 State the equation by substituting the value of The equation of the line is
b into y = −2x + b. y = −2x + 1.

Finding the equation of a straight line using the


gradient and one point
•• If the gradient of a line is known, then only one point is needed to determine the
equation of the line.

WORKED EXAMPLE 9
Find the equation of the straight line with gradient of 2 and y‐intercept of −5.

TH I N K W R I TE
1 Write the known information. The point Gradient = 2, y‐intercept = −5
is the y‐intercept, which makes the
calculation of b straightforward.
2 State the values of m and b. m = 2, b = −5
3 Substitute these values into y = mx + b y = mx + b
to find the equation. y = 2x − 5

WORKED EXAMPLE 10
Find the equation of the straight line passing through the point (5, −1)
with a gradient of 3.

TH I N K W R I TE
1 Write the known information. Gradient = 3, point (5, −1).
2 State the values of m, x and y. m = 3, (x, y) = (5, −1)
3 Substitute the values m = 3, x = 5 and y = mx + b
y = −1 into y = mx + b and solve for b. −1 = 3(5) + b
−1 = 15 + b
−16 =b
4 Substitute m = 3 and b = −16 into The equation of the line is y = 3x − 16.
y = mx + b to determine the equation.

Chapter 3 • Coordinate geometry 73


number and algebra

A simple formula
•• The diagram at right shows a line of y
(x, y)
y
gradient m passing through the
point (x1, y1). (x1, y1)
y1
•• If (x, y) is any other point on the line, then:

rise
m=
run
x
y − y1 0 x
m= x1
x − x1
m(x − x1) = y − y1
y − y1 = m(x − x1)

•• The formula y − y1 = m(x − x1) can be used to write down the equation of a line,
given the gradient and the coordinates of one point.

WORKED EXAMPLE 11
Find the equation of the line with a gradient of −2 that passes
through the point (3, −4). Write the equation in general
form, that is in the form ax + by + c = 0.

TH I N K W R I TE
1 Use the formula y − y1 = m(x − x1) . m = −2, x1 = 3, y1 = −4
Write the values of x1, y1, y − y1 = m(x − x1)
and m.
y − (−4) = −2(x − 3)
2 Substitute for x1, y1 and m in the   
y + 4 = −2x + 6
equation.

3 Transpose the equation into the form y + 4 + 2x − 6 = 0


ax + by + c = 0. 2x + y − 2 = 0

Exercise 3B Determining linear equations


INDIVIDUAL PATHWAYS
Questions: Questions: Questions:
1a–d, 2, 3, 4, 5a–d 1a–f, 2, 3, 4, 5c–g 1d–h, 2, 3, 4, 5e–j
Activity 3‐B‐1 Activity 3‐B‐2 Activity 3‐B‐3
Determining linear equations Linear equations More complex
doc‐13707 doc‐13708 linear equations
doc‐4980

74 Maths Quest 10 New South Wales Australian curriculum edition Stages 5.1 and 5.2
number and algebra

FLUENCY
1 WE6 Determine the equation for each of the straight lines shown. Digital docs
a y b y c y SkillSHEET
Measuring the rise and
4 12 5 the run
doc‐5196
SkillSHEET
Finding the gradient
−2 0 x 0 x given two points
5 doc‐5204
0 4 x

d y e y f y

−16 0 x
0 4 x x
−6 0
−4
−8

g y h y

−5 0 x
0 –5 x
7

−5 −15

2 WE7 Determine the equation of each of the straight lines shown.


a y b y
(−4, 12) 12
6 (3, 6)

0 3 x
x
−4 0

c y d y

6
(−8, 6)
−4 0 x
(−4, −2) −2 0 x
−8

3 WE8 Find the equation of the straight line that passes through each pair of points.
a (1, 4) and (3, 6) c (−1, 4) and (3, 2) e (−4, 6) and (2, −6)
b (0, −1) and (3, 5) d (3, 2) and (−1, 0) f (−3, −5) and (−1, −7)
4 WE9 Find the linear equation given the information in each case below.
a Gradient = 3, y-intercept = 3 b Gradient = −3, y-intercept = 4
c Gradient = −4, y-intercept = 2 d Gradient = 4, y-intercept = 2

Chapter 3 • Coordinate geometry 75


number and algebra

e Gradient = −1, y-intercept = −4 f Gradient = 0.5, y-intercept = −4


g Gradient = 5, y-intercept = 2.5 h Gradient = −6, y-intercept = 3
i Gradient = −2.5, y-intercept = 1.5 j Gradient = 3.5, y-intercept = 6.5
5 WE10, 11 For each of the following, find the equation of the straight line with the
given gradient and passing through the given point.
a Gradient = 5, point = (5, 6) b Gradient = −5, point = (5, 6)
c Gradient = −4, point = (−2, 7) d Gradient = 4, point = (8, −2)
e Gradient = 3, point = (10, −5)
Digital doc
f Gradient = −3, point = (3, −3) REFLECTION
WorkSHEET 3.1
What problems might you
g Gradient = −2, point = (20, −10)
doc‐13709
encounter when calculating
h Gradient = 2, point = (2, −0.5) the equation of a line
whose graph is actually
i Gradient = 0.5, point = (6, −16)
parallel to one of the axes?
j Gradient = −0.5, point = (5, 3)

UNDERSTANDING
If t represents the time in hours and
6 a 
C represents cost ($), construct a table of
values for 0–3 hours for the cost of playing
ten‐pin bowling at the new alley.
b Use your table of values to plot a graph
of time versus cost. (Hint: Ensure your
time axis (horizontal axis) extends to 6 hours and your cost axis (vertical axis)
extends to $40.)
c i What is the y‐intercept?
   ii What does the y‐intercept represent in terms of the cost?
d Calculate the gradient.
e Write a linear equation to describe the relationship between cost and time.
f Use your linear equation from part e to calculate the cost of a 5-hour tournament.
g Use your graph to check your answer to part f.

REASONING y
y
7 When using the gradient to draw a line, does it matter if you (0, b)
rise before you run or run before you rise? Explain. (x, y)
8 a    Using the graph at right, write a general formula for the 0 x x
gradient m in terms of x, y and b.
b Transpose your formula to make y the subject.
What do you notice?

CHALLENGE 3.1

76 Maths Quest 10 New South Wales Australian curriculum edition Stages 5.1 and 5.2
number and algebra

3C The distance between two points


•• The distance between two points can be calculated using Pythagoras’ theorem.
•• Consider two points A (x1, y1) and B (x2, y2) on the Cartesian plane as shown.
If point C is placed as shown, then ABC is a right‐angled triangle and AB is the
hypotenuse. y
AC = x2 − x1 y2 B(x2, y2)
BC = y2 − y1
y1 A C
By Pythagoras’ theorem:
(x , y )
AB2 = AC2 + BC2 1 1

= (x2 − x1) 2 + (y2 − y1) 2 x1 x2 x

Hence AB = "(x2 − x1) 2 + (y2 − y1) 2


The distance between two points A (x1, y1) and B (x2, y2) is:
AB = "(x2 − x1) 2 + (y2 − y1) 2
•• This distance formula can be used to calculate the distance between any two points
on the Cartesian plane.

WORKED EXAMPLE 12
Find the distance between the points A and B in the figure below.
Give your answer correct to 2 decimal places.
y
4 B

A
1
−3 3 x

TH I N K W R I TE
1 From the graph locate points A and B. A (−3, 1) and B (3, 4)
2 Let A have coordinates (x1, y1). Let (x1, y1) = (−3, 1)
3 Let B have coordinates (x2, y2). Let (x2, y2) = (3, 4)
4 Find the length AB by applying the AB = "(x2 − x1) 2 + (y2 − y1) 2
formula for calculating the distance
­between two points. = "[3 − (−3) ] 2 + (4 − 1) 2
= "(6) 2 + (3) 2
= !36 + 9
= !45
= 3 !5
= 6.71 1 correct to 2 decimal places 2
Note: If the coordinates were named in the reverse order, the formula would still give
the same answer. Check this for yourself using (x1, y1) = (3, 4) and (x2, y2) = (–3, 1).

Chapter 3 • Coordinate geometry 77


number and algebra

WORKED EXAMPLE 13
Find the distance between the points P(−1, 5) and Q(3, −2).
TH I N K W R I TE
1 Let P have coordinates (x1, y1). Let (x1, y1) = (−1, 5)
2 Let Q have coordinates (x2, y2). Let (x2, y2) = (3, −2)

3 Find the length PQ by applying the PQ = "(x2 − x1) 2 + (y2 − y1) 2


formula for the distance between two = "[3 − (−1) ] 2 + (−2 − 5) 2
points.
= "(4) 2 + (−7) 2
= !16 + 49
= !65
= 8.06 1 correct to 2 decimal places 2

•• The distance formula has many geometric applications.

WORKED EXAMPLE 14
Prove that the points A (1, 1), B (3, −1) and C (−1, −3) are the vertices
of an isosceles triangle.
TH I N K W R I TE /DRAW
1 Plot the points and draw the triangle. 1
y A
For triangle ABC to be isosceles, two
x
sides must have the same magnitude. –1 1 3
B
C
–3

2 AC and BC seem to be equal. AC = "[1 − (−1) ] 2 + [1 − (−3) ] 2


Find the length AC.
A (1, 1) = (x2, y2) = "(2) 2 + (4) 2
C (−1, −3) = (x1, y1) = !20
= 2"5
3 Find the length BC. BC = "[3 − (−1) ] 2 + [−1 − (−3) ] 2
B (3, −1) = (x2, y2)
C (−1, −3) = (x1, y1) = "(4) 2 + (2) 2
= !20
= 2"5
4 Find the length AB. AB = "[3 − (1) ] 2 + [−1 − (1) ] 2
A (1, 1) = (x1, y1)
B (3, −1) = (x2, y2) = "(2) 2 + (−2) 2
= !4 + 4
= !8
= 2 !2
5 State your proof. Since AC = BC, triangle ABC is an
isosceles triangle.

78 Maths Quest 10 New South Wales Australian curriculum edition Stages 5.1 and 5.2
number and algebra

Exercise 3C The distance between two points


INDIVIDUAL PATHWAYS
Questions: Questions: Questions:
1, 2a–d, 5, 8 1, 2c–f, 5, 7, 9 1, 2e–h, 3, 4, 5, 6, 7, 9, 10
Activity 3‐C‐1 Activity 3‐C‐2 Activity 3‐C‐3
Finding the distance between Calculations of distance Applications of distance
two points on a straight line between two points between two points
doc‐4981 doc‐4982 doc‐4983

FLUENCY
1 WE12 Find the distance between each pair of points shown at right. G y O
6
K B
2 WE13 Find the distance between the following pairs of points. 5
4 P
a (2, 5), (6, 8) b (−1, 2), (4, 14) 3
C
c (−1, 3), (−7, −5) d (5, −1), (10, 4) E H
2 A
N
L 1
e (4, −5), (1, 1) f (−3, 1), (5, 13) –6 –5 –4 –3 –2 –1 0 1 2 3 4 5 6 x
g (5, 0), (−8, 0) h (1, 7), (1, −6) –1
F –2
i (a, b), (2a, −b) j (−a, 2b), (2a, −b) M –3
I –4 J D
UNDERSTANDING –5
–6
3 MC If the distance between the points (3, b) and (−5, 2) is 10 units,
then the value of b is:
A −8 B −4 C 4 D 0 E 2
4 MC A rhombus has vertices A (1, 6), B (6, 6), C (−2, 2) and D (x, y). The
coordinates of D are:
A (2, −3) B (2, 3) C (−2, 3) D (3, 2) E (3, −2)
5 The vertices of a quadrilateral are A (1, 4), B (−1, 8), C (1, 9) and D (3, 5).
a Find the lengths of the sides. b Find the lengths of the diagonals.
c What type of quadrilateral is it?

reAsoning
6 WE14 Prove that the points A (0, −3), B (−2, −1) and C (4, 3) are the vertices of
Digital doc
an isosceles triangle. Spreadsheet 021
7 The points P (2, −1), Q (−4, −1) and R (−1, 3 !3 − 1) are joined to form a Distance between two
points
triangle. Prove that triangle PQR is equilateral. doc–5206

8 Prove that the triangle with vertices D (5, 6), E (9, 3) and F (5, 3) is a right–angled
triangle.
9 A rectangle has vertices A (1, 5), B (10.6, z), C (7.6, −6.2) and D (−2, 1). Find:
a the length of CD
REFLECTION
b the length of AD How could you use the
c the length of the diagonal AC distance formula to show
d the value of z. that a series of points lay
on the circumference of a
10 Show that the triangle ABC with coordinates circle with centre C?
A (a, a), B (m, −a) and C (−a, m) is isosceles.

Chapter 3 • Coordinate geometry 79


number and algebra

3D  The midpoint of a line segment


Midpoint of a line segment
•• The midpoint of a line segment is the half‐way point.
•• The x‐ and y-coordinates of the midpoint are half‐way between those of the
coordinates of the end points.
•• The diagram shows the line interval AB joining points A (x1, y1) and B (x2, y2).
The midpoint of AB is P, so AP = PB.
Points C (x, y1) and D (x2, y) are added to the diagram and are used to make the two
right‐angled triangles ΔAPC and ΔPBD. y
The two triangles are congruent: y2
B (x2, y2)
AP = PB (given)
∠APC = ∠PBD (corresponding angles) P
∠CAP = ∠DPB (corresponding angles) y
(x, y)
D
So ΔAPC ≡ ΔPBD (ASA)
This means that AC = PD; y1 A
i.e. x − x1 = x2 − x (solve for x) (x 1, y1) C

    2x = x1 + x2
x + x2
x= 1 0 x1 x x2 x
2
In other words x is simply the average of x1 and x2.
y + y2
Similarly, y = 1 .
2 y
In general, the coordinates of the midpoint of a line
(x2, y2)
­segment joining the points (x1, y1) and (x2, y2) can be
found by averaging the x- and y-coordinates of the end M
points, respectively. x1 + x2 , y1 + y2
The coordinates of the midpoint of the line segment 2 2
(x1 , y1 )
x + x2 y1 + y2
joining (x1, y1) and (x2, y2) are: a 1 , b. 0 x
2 2

WORKED EXAMPLE 15
Find the coordinates of the midpoint of the line segment joining (−2 , 5 )
and (7, 1).

TH I N K W R I TE
1 Label the given points (x1, y1) and (x2, y2). Let (x1, y1) = (−2, 5) and
(x2, y2) = (7, 1)
x + x2
2 Find the x-coordinate of the midpoint. x= 1
2
−2 + 7
=
2
5
=2
= 212

80 Maths Quest 10 New South Wales Australian curriculum edition Stages 5.1 and 5.2
number and algebra

y1 + y2
3 Find the y-coordinates of the midpoint. y=
2
5+1
=
2
=6
2
=3

4 Give the coordinates of the midpoint. The midpoint is Q 212, 3 R .

WORKED EXAMPLE 16
The coordinates of the midpoint, M, of the line segment
AB are (7, 2). If the coordinates of A are (1, −4), find the coordinates
of B.

TH I N K W R I TE /DRAW
1 Let the start of the line segment Let (x1, y1) = (1, −4) and (x, y) = (7, 2)
be (x1, y1) and the midpoint
be (x, y).
x1 + x2
2 The average of the x-coordinates x=
is 7. Find the x-coordinate of the 2
end point. 1 + x2
7=
2
14 = 1 + x2
x2 = 13
y1 + y2
3 The average of the y‐coordinates y=
is 2. Find the y-coordinate of the 2
end point. −4 + y2
2=
2
4 = −4 + y2
y2 = 8
4 Give the coordinates of the end The coordinates of the point B
point. are (13, 8) .

5 Check that the coordinates are y


feasible by drawing a diagram. B (13, 8)
8

4
M
(7, 2)
0 4 8 12 x
–4 A (1, −4)

Chapter 3 • Coordinate geometry 81


number and algebra

Exercise 3D The midpoint of a line segment


INDIVIDUAL PATHWAYS
Questions: Questions: Questions:
1a–d, 2, 3a, 4, 9 1a–d, 2–6, 9 1a–f, 2–10
Activity 3‐D‐1 Activity 3‐D‐2 Activity 3‐D‐3
Finding the Calculations — Applications —
midpoint of a line midpoint of a segment midpoint of a segment
segment doc‐4985 doc‐4986
doc‐4984

FLUENCY
1 WE15 Use the formula method to find the coordinates of the midpoint of the line
segment joining the following pairs of points.
a (−5, 1), (−1, −8) b (4, 2), (11, −2)
c (0, 4), (−2, −2) d (3, 4), (−3, −1)
e (a, 2b), (3a, −b) f (a + 3b, b), (a − b, a − b)
2 WE16 The coordinates of the midpoint, M, of the line segment AB are (2, −3). If
the coordinates of A are (7, 4), find the coordinates of B.
UNDERSTANDING
3 A square has vertices A (0, 0), B 1 2, 4 2 , C 1 6, 2 2 and D 1 4, −2 2 . Find:
a the coordinates of the centre b the length of a side
c the length of a diagonal.
4 MC The midpoint of the line segment joining the points (−2, 1) and (8, −3) is:
A (6, −2) B (5, 2) C (6, 2) D (3, −1) E (5, −2)
5 MC If the midpoint of AB is (−1, 5) and the coordinates of B are (3, 8), then A has
coordinates:
Digital doc
Spreadsheet 075
A (1, 6.5) B (2, 13) C (−5, 2) D (4, 3) E (7, 11)
Midpoint of a segment 6 a The vertices of a triangle are A (2, 5), B (1, −3) and C (−4, 3). Find:
doc‐5207
i the coordinates of P, the midpoint of AC
ii the coordinates of Q, the midpoint of AB
iii the length of PQ
b Show that BC = 2 PQ.
7 a A quadrilateral has vertices A (6, 2), B (4, −3), C (−4, −3) and D (−2, 2). Find:
i the midpoint of the diagonal AC
ii the midpoint of the diagonal BD.
b What can you infer about the quadrilateral?
8 a The points A (−5, 3.5), B (1, 0.5) and C (−6, −6) are the vertices of a triangle. Find:
i the midpoint, P, of AB
ii the length of PC
iii the length of AC
iv the length of BC.
b Describe the triangle. What does PC represent?

82 Maths Quest 10 New South Wales Australian curriculum edition Stages 5.1 and 5.2
number and algebra

REASONING
9 Find the equation of the straight line that passes
through the midpoint of A(−2, 5) and B(−2, 3), and REFLECTION Digital doc
WorkSHEET 3.2
If the midpoint of a line
has a gradient of −3. segment is the origin, what
doc‐13711

10 Find the equation of the straight line that passes are the possible values of
through the midpoint of A(−1, −3) and B(3, −5) , and the x- and y-coordinates of
has a gradient of 23. the end points?

3E  Parallel and perpendicular lines


y
Parallel lines
15
•• Lines that have the same Interactivity
gradient are parallel lines. 10 y=x+2 Parallel and
perpendicular lines
5 int‐2779
y=x

0 x
–6 –4 –2 2 4 6
–5
y = x − 10
–10

–15

WORKED EXAMPLE 17
Show that AB is parallel to CD given that A has coordinates (−1, −5), B has
coordinates (5, 7), C has coordinates (−3, 1) and D has coordinates (4, 15).
TH I N K W R I TE
1 Find the gradient of AB by a­ pplying Let A (−1, −5) = (x1, y1) and
y − y1 B 1 5, 7 2 = (x2, y2)
the formula m = 2 .
x2 − x1 y − y1
Since m = 2
x2 − x1
7 − (−5)
mAB =
5 − (−1)
12
=
6
=2
2 Find the gradient of CD. Let C (−3, 1) = (x1, y1) and
D 1 4, 15 2 = (x2, y2)
15 − 1
mCD =
4 − (−3)
14
=
7
=2
3 Draw a conclusion. (Note: || means Since mAB = mCD = 2, then AB || CD.
‘is parallel to’.)

Chapter 3 • Coordinate geometry 83


number and algebra

Collinear points y

•• Collinear points are points that all lie on the same C


straight line.
•• If A, B and C are collinear, then mAB = mBC. B

0 x
WORKED EXAMPLE 18
Show that the points A (2, 0), B (4, 1) and C (10, 4) are collinear.

TH I N K W R I TE
1 Find the gradient of AB. Let A (2, 0) = (x1, y1)
and B (4, 1) = (x2, y2)
y2 − y1
Since m =
x2 − x1
1−0
mAB =
4−2
1
= 2

2 Find the gradient of BC. Let B (4, 1) = (x1, y1)


and C (10, 4) = (x2, y2)
4−1
mBC =
10 − 4
= 36

= 12

3 Show that A, B and C are collinear. Since mAB = mBC = 12 and B is common
to both line segments A, B and C are
­collinear.

y
Perpendicular lines y =− x +2
2 6
y = 2x − 6

•• There is a special relationship between the 4


gradients of two perpendicular lines.
•• Consider the diagram shown on the 2
following page in which the line segment
AB is perpendicular to the line segment 0 x
–6 –4 –2 2 4 6
BC, AC is parallel to the x‐axis, and BD –2
is the perpendicular height of the resulting
–4
triangle ABC.
–6

84 Maths Quest 10 New South Wales Australian curriculum edition Stages 5.1 and 5.2
number and algebra

In ΔABD, let mAB = m1


a
=
b y B
= tan (θ)
In ΔBCD, let mBC = m2
a
=− α θ
c
= −tan (α)
b a
In ΔABD, tan(α) =
a
b
So m2 = − θ α
a A C
−1
= b c
m1
D
1
Hence m2 = − 0 x
m1
or m1m2 = −1
•• Hence, if two lines are perpendicular to each other, then the product of their
­gradients is −1.
    Two lines are perpendicular if and only if:

m1m2 = −1
a −b
•• If two lines are perpendicular, then their gradients are and respectively.
b a

WORKED EXAMPLE 19
Show that the lines y = −5x + 2 and 5y − x + 15 = 0 are perpendicular to
one another.

TH I N K W R I TE
1 Find the gradient of the first line. y = −5x + 2
  
Hence m1 = −5

2 Find the gradient of the second line. 5y − x + 15 = 0


Rewrite in the form y = mx + b
5y = x − 15
    
x
y = −3
5

Hence m2 = 15
m1m2 = −5 × 15
= −1
3 Test for perpendicularity. (The two Hence, the two lines are perpendicular.
lines are perpendicular if the product
of their gradients is −1.)

Chapter 3 • Coordinate geometry 85


number and algebra

Determining the equation of a line parallel or


­perpendicular to another line
•• The gradient properties of parallel and perpendicular lines can be used to solve many
problems.

WORKED EXAMPLE 20
Find the equation of the line that passes through the point (3, −1) and is
parallel to the straight line with equation y = 2x + 1.

TH I N K W R I TE
1 Write the general equation.          y = mx + b
2 Find the gradient of the given line. y = 2x + 1 has a gradient of 2
The two lines are parallel, so they    Hence m = 2
have the same gradient.
3 Substitute for m in the general so y = 2x + b
equation.
4 Substitute the given point to (x, y) = (3, −1)
find b. ∴ −1 = 2(3) + b
= 6+b
b = −7
5 Substitute for b in the general y = 2x − 7
equation. or
2x − y − 7 = 0

WORKED EXAMPLE 21
Find the equation of the line that passes through the point (0, 3) and is
­perpendicular to a line with a gradient of 5.

TH I N K W R I TE
1 For perpendicular lines m1 × m2 = −1. Given m1 =5
Find the gradient of the perpendicular m2 = −15
line.
2 Use the equation y − y1 = m (x − x1) Since y − y1 = m(x − x1)
where m = −15 and (x1, y1) = (0, 3). and (x1, y1) = (0, 3)
then y − 3 = −15 (x − 0)
x
y−3 = −
5
5(y − 3) = −x
5y − 15 = −x
x + 5y − 15 = 0

86 Maths Quest 10 New South Wales Australian curriculum edition Stages 5.1 and 5.2
number and algebra

Horizontal and vertical lines


•• Recall the following. y x=4
–– Horizontal lines are parallel to the x‐axis, have a 4

gradient of zero, are expressed in the form y = b and 2 y=2

have no x‐intercept.
–– Vertical lines are parallel to the y‐axis, have an –2 0 2 4 6 x
undefined (infinite) gradient, are expressed in the –2
form x = a and have no y‐intercept.
–4

WORKED EXAMPLE 22
Find the equation of:
a the vertical line that passes through the point (2, −3)
b the horizontal line that passes through the point (−2, 6).

TH I N K W R I TE
a The equation of a vertical line is x = a. The a x=2
x‐coordinate of the given point is 2.
b The equation of a horizontal line is y = b. The b y=6
y‐coordinate of the given point is 6.

WORKED EXAMPLE 23
Find the equation of the perpendicular bisector of the line joining the
points (0, −4) and (6, 5). (A bisector is a line that crosses another line at
right angles and cuts it into two equal lengths.)
TH I N K W R I TE /DRAW
1 Find the gradient of the line joining the Let (0, −4) = (x1, y1).
given points by applying the formula. Let (6, 5) = (x2, y2).
y − y1 y − y1
m= 2 . m1 = 2
x2 − x1 x2 − x1
5 − (−4)
m1 =
      6−0
9
=
6
3
=
2
3
2 Find the gradient of the perpendicular m1 = 2
line.   
m1 × m2 = −1 m2 = −23

Chapter 3 • Coordinate geometry 87


number and algebra

x1 + x2 y1 + y2
3 Find the midpoint of the line joining the x= y=
given points. 2 2
x + x2 y1 + y2 0+6 −4 + 5
M=a 1 , b where = =
2 2 2 2
1
(x1, y1) = (0, –4) and (x2, y2) = (6, 5). =3 = 2

Hence 1 3, 12 2 are the coordinates of


the ­midpoint.
4 Find the equation of the line with Since y − y1 = m(x − x1),
gradient −23 that passes through 1 3, 12 2 . then y − 12 = −23 (x − 3)

5 Simplify: 3 1 y − 12 2 = −2(x − 3)
Multiply both sides by 3.
3y − 32 = −2x + 6
Multiply both sides by 2. 6y − 3 = −4x + 12
4x + 6y − 15 = 0

Exercise 3E Parallel and perpendicular lines


INDIVIDUAL PATHWAYS
Questions: Questions: Questions:
1a–d, 2, 5, 6a–c, 7, 8, 9a–c, 1a–d, 2, 3, 4, 5, 6c–d, 7, 8, 1c–f, 2, 3, 4, 5, 6e–f, 7, 8–19,
12, 13, 16a–b, 18, 20a, 21, 23, 9a–c, 12, 13, 15, 16a–b, 17a, 18, 20b, 21, 22, 24–31
26a, 27 20a, 21–3, 26–8, 30
Activity 3‐E‐1 Activity 3‐E‐2 Activity 3‐E‐3
Parallel and More difficult parallel and Complex parallel and
perpendicular lines perpendicular lines perpendicular lines
doc‐4987 doc‐4988 doc‐4989

FLUENCY
1 Find whether AB is parallel to CD given the following sets of points.
WE17
a A (4, 13), B (2, 9), C (0, −10), D (15, 0)
b A (2, 4), B (8, 1), C (−6, −2), D (2, −6)
c A (−3, −10), B (1, 2), C (1, 10), D (8, 16)
d A (1, −1), B (4, 11), C (2, 10), D (−1, −5)
e A (1, 0), B (2, 5), C (3, 15), D (7, 35)
f A (1, −6), B (−5, 0), C (0, 0), D (5, −4)
2 Which pairs of the following straight lines are parallel?
a 2x + y + 1 = 0 b y = 3x − 1 c 2y − x = 3
x
d y = 4x + 3 e y= −1 f 6x − 2y = 0
2
g 3y = x + 4 h 2y = 5 − x
3 WE18 Show that the points A (0, −2), B (5, 1) and C (−5, −5) are collinear.

88 Maths Quest 10 New South Wales Australian curriculum edition Stages 5.1 and 5.2
number and algebra

4 Show that the line that passes through the points (−4, 9) and (0, 3) also passes
through the point (6, −6).
5 WE19 Show that the lines y = 6x − 3 and x + 6y − 6 = 0 are perpendicular to
one another.
6 Determine whether AB is perpendicular to CD, given the following sets of points.
a A (1, 6), B (3, 8), C (4, −6), D (−3, 1)
b A (2, 12), B (−1, −9), C (0, 2), D (7, 1)
c A (1, 3), B (4, 18), C (−5, 4), D (5, 0)
d A (1, −5), B (0, 0), C (5, 11), D (−10, 8)
e A (−4, 9), B (2, −6), C (−5, 8), D (10, 14)
f A (4, 4), B (−8, 5), C (−6, 2), D (3, 11)
7 WE20 Find the equation of the line that passes through the point (4, −1) and is
parallel to the line with equation y = 2x − 5.
Digital docs
8 WE21 Find the equation of the line that passes through the point (−2, 7) and is Spreadsheet 085
2 Perpendicular checker
perpendicular to a line with a gradient of
3
. doc‐5209
Spreadsheet 029
9 Find the equations of the following lines. Equation of a
a Gradient 3 and passing through the point (1, 5) straight line
doc‐5210
b Gradient −4 and passing through the point (2, 1)
c Passing through the points (2, −1) and (4, 2)
d Passing through the points (1, −3) and (6, −5)
e Passing through the point (5, −2) and parallel to x + 5y + 5 = 0
f Passing through the point (1, 6) and parallel to x − 3y − 2 = 0
g Passing through the point (−1, −5) and perpendicular to 3x + y + 2 = 0
10 Find the equation of the line that passes through the point (−2, 1) and is:
a parallel to the line with equation 2x − y − 3 = 0
b perpendicular to the line with equation 2x − y − 3 = 0.
11 Find the equation of the line that contains the point (1, 1) and is:
a parallel to the line with equation 3x − 5y = 0
b perpendicular to the line with equation 3x − 5y = 0.
12 WE22 Find the equation of:
a the vertical line that passes through the point (1, −8)
b the horizontal line that passes through the point (−5, −7).
13 MC a The vertical line passing through the point (3, −4) is given by:
A y = −4 B x=3 C y = 3x − 4
D y = −4x + 3 E x = −4
b Which of the following points does the horizontal line given by the equation
y = −5 pass through?
A (−5, 4) B (4, 5) C (3, −5)
D (5, −4) E (5, 5)
c Which of the following statements is true?
A Vertical lines have a gradient of zero.
B The y‐coordinates of all points on a vertical line are the same.

Chapter 3 • Coordinate geometry 89


number and algebra

C Horizontal lines have an undefined gradient.


D The x‐coordinates of all points on a vertical line are the same.
E A horizontal line has the general equation x = a.
d Which of the following statements is false?
A Horizontal lines have a gradient of zero.
B The line joining the points (1, −1) and (−7, −1) is vertical.
C Vertical lines have an undefined gradient.
D The line joining the points (1, 1) and (−7, 1) is horizontal.
E A horizontal line has the general equation y = c.
14 The triangle ABC has vertices A (9, −2), B (3, 6) and C (1, 4).
a Find the midpoint, M, of BC. b Find the gradient of BC.
c Show that AM is the perpendicular d Describe triangle ABC.
bisector of BC.
15 WE23 Find the equation of the perpendicular bisector of the line joining the points
(1, 2) and (−5, −4).
16 Find the equation of the perpendicular bisector of the line joining the points (−2, 9)
and (4, 0).
17 ABCD is a parallelogram. The coordinates of A, B and C are (4, 1), (1, −2) and
(−2, 1) respectively. Find:
a the equation of AD b the equation of DC
c the coordinates of D.

UNDERSTANDING
18 In each of the following, show that ABCD is a parallelogram.
a A (2, 0), B (4, −3), C (2, −4), D (0, −1)
b A (2, 2), B (0, −2), C (−2, −3), D (0, 1)
c A (2.5, 3.5), B (10, −4), C (2.5, −2.5), D (−5, 5)
19 In each of the following, show that ABCD is a trapezium.
a A (0, 6), B (2, 2), C(0, −4), D (−5, −9)
b A 1 26, 32 2 , B (18, 16), C(1, −1), D(−3, 3)
c A (2, 7), B (1, −1), C (−0.6, −2.6), D (−2, 3)
20 MC The line that passes through the points (0, −6) and (7, 8) also passes
through:
A (4, 3) B (5, 4) C (−2, 10)
D (1, −8) E (1, 4)
21 MC The point (−1, 5) lies on a line parallel to 4x + y + 5 = 0. Another point on
the same line as (−1, 5) is:
A (2, 9) B (4, 2) C (4, 0)
D (−2, 3) E (3, −11)
22 Find the equation of the line given the following conditions:
a Passes through the point (−1, 3) and parallel to y = −2x + 5
b Passes through the point (4, −3) and parallel to 3y + 2x = −3

90 Maths Quest 10 New South Wales Australian curriculum edition Stages 5.1 and 5.2
number and algebra

23 Determine which pairs of the following lines are perpendicular.


a x + 3y − 5 = 0 b y = 4x − 7 c y=x d 2y = x + 1
e y = 3x + 2 f x + 4y − 9 = 0 g 2x + y = 6 h x+y=0
24 Find the equation of the straight line that cuts the x‐axis at 3 and is perpendicular to
the line with equation 3y − 6x = 12.
25 Calculate the value of m for which lines with the following pairs of equations are
perpendicular to each other.
a 2y − 5x = 7 and 4y + 12 = mx b 5x − 6y = −27 and 15 + mx = −3y
26 MC The gradient of the line perpendicular to the line with equation 3x − 6y = 2 is:
1
A 3 B −6 C 2 D 2 E −2
27 MC Triangle ABC has a right angle at B. The vertices are A (−2, 9), B (2, 8)
and C (1, z). The value of z is:
1 3
A 8 B 4 C 12 D 7 E −4
4 4
REASONING
28 The map shows the proposed course for a yacht race. y Scale: 1 unit 1 km N
Buoys have been positioned at 11
10
A (1, 5), B (8, 8), C (12, 6), and D (10, w). 9 Buoy B
a How far is it, from the start, O, to buoy A? 8
7
Buoy
b The race marshall boat, M, is situated halfway 6 A
M Buoy C
5
between buoys A and C. What are the coordinates 4
of the boat’s position? 3
2 H
E
Buoy D
c Stage 4 of the race (from C to D) is perpendicular 1
O
to stage 3 (from B to C). What is the gradient of CD? (Start) 1 2 3 4 5 6 7 8 9 10 11 12 x
d Find the linear equation that describes stage 4.
e Hence determine the exact position of buoy D.
f An emergency boat is to be placed at point E (7, 3). How far is the emergency
boat from the hospital, located at H, 2 km north of the start?
29 Show that the following sets of points form the
vertices of a right‐angled triangle. REFLECTION
a A (1, −4), B (2, −3), C (4, −7) How could you use coordinate
geometry to design a logo for
b A 1 3, 13 2 , B (1, 3), C (−4, 4)
an organisation?
c A (0, 5), B (9, 12), C (3, 14)

Chapter 3 • Coordinate geometry 91


number and algebra

CHAPTER REVIEW
LANGUAGE

axes gradient parallel segment


bisect gradient–intercept parallelogram substitute
Cartesian plane form perpendicular trapezium
collinear horizontal quadrilateral vertical
coordinates linear graph rhombus vertices
diagonal midpoint rise x-intercept
general form origin run y-intercept

int‐2832 doc‐13713
int‐2833 doc‐13714
int‐3590

FLUENCY
1 The equation of the line drawn below is:
y
3

0 2 x

A 3x + 2y = 6 B 3x − 2y = 6 C 2x + 3y = 6
D 2x − 3y = 6 E 2x − 3y = −6
2 The equation of a linear graph with gradient −3 and x‐intercept
of 4 is:
A y = −3x − 12 B y = −3x + 4 C y = −3x − 4
D y = −3x + 12 E y = 4x − 3
3 The equation of a linear graph which passes through (2, −7) and (−2, −2) is:
A 4x − 5y + 18 = 0 B 5x + 4y + 18 = 0 C 5x + 4y − 18 = 0
D 5x − 4y − 18 = 0 E 4x + 5y + 18 = 0
4 The distance between the points (1, 5) and (6, −7) is:
A !53 B !29 C 13 D !193 E 12
5 The midpoint of the line segment joining the points (−4, 3) and (2, 7) is:
A (−1, 5) B (−2, 10) C (−6, 4) D (−2, 4) E (−1, 2)
6 If the midpoint of the line segment joining the points A (3, 7) and B (x, y) has
coordinates (6, 2), then the coordinates of B are:
A (15, 3) B (0, −6) C (9, −3) D (4.5, 4.5) E (−9, 3)

92 Maths Quest 10 New South Wales Australian curriculum edition Stages 5.1 and 5.2
number and algebra

7 If the points (−6, −11), (2, 1) and (x, 4) are collinear, then the value of x is:
1 5
A 4 B 3.2 C 4
D 16
E 3
8 The gradient of the line perpendicular to 3x − 4y + 7 = 0 is:
3 4 4
A 4
B 3
C −3 D 3 E −4
9 The equation of the line perpendicular to 2x + y − 1 = 0 and passing through the
point (1, 4) is:
A 2x + y − 6 = 0 B 2x + y − 2 = 0 C x − 2y + 7 = 0
D x + 2y + 9 = 0 E x − 2y = 0
10 Produce a table of values, and sketch the graph of the equation y = −5x + 15 for
values of x between −10 and +10.
11 Sketch the graph of the following linear equations, labelling the x‐ and y‐intercepts.
a y = 3x − 2 b y = −5x + 15
2 7
c y = −3x + 1 d y = 5x − 3

12 Find the x‐ and y‐intercepts of the following straight lines.


3
a y = −7x + 6 b y = 8x − 5
4 3
c y = 7x − 4
d y = 0.5x + 2.8

13 Sketch graphs of the following linear equations by finding the x‐ and y‐intercepts.
a 2x − 3y = 6 b 3x + y = 0
c 5x + y = −3 d x+y+3=0
14 Sketch the graph of each of the following.
1
a y = 2x b y = −4x
c x = −2 d y=7
15 Sketch the graph of the equation 3(y − 5) = 6(x + 1).
16 Find the equations of the straight lines in the following graphs.

a y b y

−4 0 x
0 1 x

−4
−2

c y d y
• (2, 8)

2
0 6 x 0 x

Chapter 3 • Coordinate geometry 93


number and algebra

e f y
y

0 x
0 5 x
– –43

17 Find the linear equation given the information in each case below.
a Gradient = 3, y‐intercept = −4 b Gradient = −2, y‐intercept = −5
1
c Gradient = 2, y‐intercept = 5 d Gradient = 0, y‐intercept = 6
18 For each of the following, find the equation of the straight line with the given
gradient and passing through the given point.
a Gradient = 7, point (2, 1) b Gradient = −3, point (1, 1)
1 3
c Gradient = 2, point (−2, 5) d Gradient = 5, point (1, −3)
19 Find the distance between the points (1, 3) and (7, −2) in exact form.
20 Prove that triangle ABC is isosceles given A 1 3, 1 2 , B (−3, 7) and C (−1, 3).
21 Show that the points A (1, 1), B (2, 3) and C (8, 0) are the vertices of a right‐angled
triangle.
22 The midpoint of the line segment AB is (6, −4). If B has coordinates (12, 10), find
the coordinates of A.
23 Show that the points A (3, 1), B (5, 2) and C (11, 5) are collinear.
24 Show that the lines y = 2x − 4 and x + 2y − 10 = 0 are perpendicular to one another.
25 Find the equation of the straight line passing through the point (6, −2) and parallel
to the line x + 2y − 1 = 0.
26 Find the equation of the line perpendicular to 3x − 2y + 6 = 0 and having the same
y‐intercept.
27 Find the equation of the perpendicular bisector of the line joining the points (−2, 7)
and (4, 11).
28 Find the equation of the straight line joining the point (−2, 5) and the point of
intersection of the straight lines with equations y = 3x − 1 and y = 2x + 5.
29 Using the information given in the diagram: y
a find:
B (4, 9)
i the gradient of AD 9
ii the gradient of AB
C
iii the equation of BC 4 A
iv the equation of DC D
O
v the coordinates of C. x
4 5 9
b describe quadrilateral ABCD.
30 In triangle ABC, A is (1, 5), B is (−2, −3) and C is (8, −2).
a Find:
i the gradient of BC
ii the midpoint, P, of AB
iii the midpoint, Q, of AC.

94 Maths Quest 10 New South Wales Australian curriculum edition Stages 5.1 and 5.2
number and algebra

b Hence show that:


i PQ is parallel to BC
ii PQ is half the length of BC.

PROBLEM SOLVING
1 John has a part‐time job working as a gardener and is paid $13.50 per hour.
a Complete the following table of values relating the amount of money received to
the number of hours worked.
Number of hours 0 2 4 6 8 10
Pay ($)
b Find a linear equation relating the amount of money received to the number of
hours worked.
c Sketch the linear equation on a Cartesian plane over a suitable domain.
3
d Using algebra, calculate the pay that John will receive if he works for 64 hours.
2 A fun park charges a $12.50 entry fee and an additional $2.50 per ride.
a Complete the following table of values relating the total cost to the number of rides.

Number of rides 0 2 4 6 8 10
Cost ($)

b Find a linear equation relating total cost to the number of rides.


c Sketch the linear equation on a Cartesian plane over a suitable domain.
d Using algebra, calculate the cost for 7 rides.
3 The cost of hiring a boat is $160 plus $22.50 per hour.
a Sketch a graph showing the total cost for between 0 and 12 hours.
b State the equation relating cost to time rented.
c Predict the cost of hiring a boat for 12 hours and 15 minutes.
4 ABCD is a quadrilateral with vertices A (4, 9), B (7, 4), C (1, 2) and D (a, 10).
Given that the diagonals are perpendicular to each other, find:
a the equation of the diagonal AC
b the equation of the diagonal BD
c the value of a.
5 An architect decides to design a building with a y
14-metre-square base such that the external walls are
8m
initially vertical to a height of 50 metres, but taper C
so that their separation is 8 metres at its peak height
of 90 metres. A profile of the building is shown with
the point (0, 0) marked as a reference at the centre of
B
the base. 90 m
a Write the equation of the vertical line connecting
A and B. 50 m

b Write the coordinates of B and C.


0
c Find the length of the tapered section of wall A 14 m x
from B to C.

Chapter 3 • Coordinate geometry 95


number and algebra
Communicating

Rich task

What common computer


symbol is this?

puter
gr ap h nin e lin es to reveal a common com
u to
of this task requires yo y-values in the followi
ng ranges:
The construction part co m m od at e x- an d
of your graph to ac
symbol. Draw the scale nt re the axes on the grid lin
es provided.
≤ y ≤ 16 . Ce
−10 ≤ x ≤ 16 and −1 0

96 Maths Quest 10 New South Wales Australian curriculum edition Stages 5.1 and 5.2
number and algebra

• Line 4 is parallel to line 1, with a y-intercept of −3. Determine the equation of the line, and
then graph the line in the range −1 ≤ x ≤ 2.
• Line 5 has the same length as line 4 and is parallel to it. The point (−2, 3) is the starting point
of the line, which decreases in both x- and y-values from there.
• Line 6 commences at the same starting point as line 5, and then runs at right angles to
line 5. It has an x-intercept of 1 and is the same length as line 2.
• Line 7 commences at the same starting point as both lines 5 and 6. Its equation is
y = 6x + 15. The point (−1, 9) lies at the midpoint.
• Line 8 has the equation y = −x + 15. Its midpoint is the point (7, 8) and its extremities are the
points where the line meets line 7 and line 9.
• Line 9 has the equation 6y − x + 8 = 0. It runs from the intersection of lines 4 and 6 until it
meets line 8.

1 What common computer symbol have you drawn?


2 The top section of your figure is a familiar geometric shape. Use the coordinates on your graph,
together with the distance formula to determine the necessary lengths to calculate the area of
this figure.
3 Using any symbol of interest to you, draw your symbol on grid lines and provide instructions for
your design. Ensure that your design involves aspects of coordinate geometry that have been
used throughout this task.

Chapter 3 • Coordinate geometry 97


number and algebra

Code puzzle

1990 space
achievement
Find the gradient, y-intercept and equation of the line described in
each box on the left. Join the dot next to each box to the relevant
answers on the right. Each line will pass through a letter and a
number to answer the code puzzle.

A line passes
through the points

( 12– , 1 ) and (– 12– , 12– ).
• y = 4x + 8
T
A line passes D • y = –x – 3
S
through the origin • 11 W • m = –5
and the point (–1, 5). 2
5 13 • 4y = 10x + 1
M 1 • c = –1
A line passes B
through the points A
• m = 1–
• 2
F H
(1, 12) and (–2, 0). 17 • ( 0, –3)
I
R 18 • ( 0, 14– )
A line passes 10
• m = –3
14 12 4
through (2, –5) • 15
and (–2, –1). 3 • m=4
L
U • y = –5x
16 3
A line passes 6 • c = 4–
N 7
through ( 13– , –2 ) • C E • m = 52–
and ( – 32– , 1 ). 8
P • y = –3x – 1
9
• 4y – 2x = 3
A line passes O
through the points • m = –1
• • c=8
( 2, 5 14– ) and
(–1, –2 14– ). • ( 0, 0 )

1 2 3 2 4 5 5 6 3 1 3 6 3 7 8 9 10 3

11 12 7 10 6 12 8 3 13 14 15 1 9 9 16 5 14 1

1 9 5 3 8 9 17 3 1 2 3 18 14 16 7 1

7 10 12 8 3 1 3 6 3 7 8 9 10 3

98 Maths Quest 10 New South Wales Australian curriculum edition Stages 5.1 and 5.2
number and algebra

 Activities
Go to assessON
Chapter opener • Activity 3‐C‐2 (doc‐4982): Calculation of for questions to
video distance between two points test your readiness
• The story of mathematics (eles-1842) • Activity 3‐C‐3 (doc‐4983): Applications of
FOR learning, your
distance between two points
3A Sketching linear graphs progress AS you learn
• Spreadsheet 021 (doc‐5206): Distance
Digital docs and your levels OF
between two points
• Activity 3‐A‐1 (doc‐4975): Sketching linear achievement.
graphs (page 69) 3D The midpoint of a line segment www.assesson.com.au
• Activity 3‐A‐2 (doc‐4976): Graphs of linear Digital docs
equations (page 69) • Activity 3‐D‐1 (doc‐4984): Finding the
• Activity 3‐A‐3 (doc‐4977): More graphs of midpoint of a line segment (page 82)
linear equations (page 69) • Activity 3‐D‐2 (doc‐4985): Calculations —
• SkillSHEET (doc‐5197): Describing the midpoint of a segment (page 82)
gradient of a line (page 69) • Activity 3‐D‐3 (doc‐4986): Applications —
• SkillSHEET (doc‐5198): Plotting a line using a midpoint of a segment (page 82)
table of values (page 69) • Spreadsheet 075 (doc‐5207): Midpoint of a
• SkillSHEET (doc‐5199): Stating the y-intercept segment (page 82)
from a graph (page 69) • WorkSHEET 3.2 (doc‐13711): Midpoint of a line
• SkillSHEET (doc‐5201): Using Pythagoras’ segment (page 83)
theorem (page 69)
3E Parallel and perpendicular lines
• SkillSHEET (doc‐5202): Substitution into a
Digital docs 
linear rule (page 69)
• Activity 3‐E‐1 (doc‐4987): Parallel and
• SkillSHEET (doc‐5200): Solving linear
perpendicular lines (page 88)
equations that arise when finding x- and
• Activity 3‐E‐2 (doc‐4988): More difficult
y-intercepts (page 69) parallel and perpendicular lines (page 88)
• SkillSHEET (doc‐5203): Transposing linear
• Activity 3‐E‐3 (doc‐4989): Complex parallel
equations to standard form (page 69)
and perpendicular lines (page 88)
3B Determining linear equations • Spreadsheet 085 (doc‐5209): Perpendicular
Digital docs checker (page 89)
• Activity 3‐B‐1 (doc‐13707): Determining linear • Spreadsheet 029 (doc‐5210): Equation of a
equations (page 74) straight line (page 89)
• Activity 3‐B‐2 (doc‐13708): Linear equations Interactivity
(page 74) • Parallel and perpendicular lines (int‐2779)
• Activity 3‐B‐3 (doc‐4980): More complex (page 83)
linear equations (page 74)
Chapter review
• SkillSHEET (doc‐5196): Measuring the rise
Interactivities  (page 92)
and the run (page 75)
• Word search (int‐2832)
• SkillSHEET (doc‐5204): Finding the gradient
• Crossword (int‐2833)
given two points (page 75)
• Sudoku (int‐3590)
• WorkSHEET 3.1 (doc‐13709): Gradient
Digital docs
(page 76)
• Chapter summary (doc‐13713)
3C The distance between two points on a • Concept map (doc‐13714)
straight line
To access eBookPLUS activities, log on to
Digital docs  (page 79)
www.jacplus.com.au
• Activity 3‐C‐1 (doc‐4981): Finding the distance
between two points on a straight line

Chapter 3 • Coordinate geometry 99


number and algebra

Answers
CHAPTER 3 Coordinate geometry
Exercise 3A — Sketching linear graphs
1 a y e y
x y 35 x y 20
y = 10x + 25 30 15 y = –5x + 3
−5 −25 25 −3 18 10
20 5
−4 −15 15 −2 13
–3 –2 –1 0 1 2 x
10 –5
−3 −5 5 −1 8 –10

−2 5 –5 –4
–3
–2 –1–5 1 2 x 0 3
–10
−1 15 –15 1 −2
–20
0 25 –25 2 −7

1 35
f y
x y 20
y 15 y = 7 – 4x
b −3 19
x y y = 5x – 12
10
10 5
−1 −17 5 −2 15
–3 –2 –1 0 1 2 3 x
x –5
–2 –1–5 1 2 3 4 5 −1 11
0 −12
–10
1 −7 –15 0 7
–20
2 −2 1 3

3 3 2 −1

4 8
2 a y
x y 20
y 15 y = –3x + 2
c 14 y = –0.5x + 10 −6 20 10
x y
12 5
−6 13 10 −4 14
–10 –5 0 5 10 x
8 –5
−4 12 6 −2 8 –10
4 –15
−2 11 2 0 2 –20

0 10 –6 –4 –2 0 2 4 6 x 2 −4

2 9 4 −10

4 8 6 −16

b y
d y x y
x y 300 y = 100x – 240
6
250
0 −240 −3 6 5
200 4
150 −2 5 3
1 −140 100 y = –x + 3
2
2 −40
50 −1 4 1
0 12 3 4 5 x
–50 0 3 –3 –2 –1 0 1 2 3 x
3 60 –100
–150 1 2
4 160 –200
–250 2 1
5 260
3 0

100 Maths Quest 10 New South Wales Australian curriculum edition Stages 5.1 and 5.2
number and algebra

c y o y = – 1–2 x – 4 y
x y 20 x
15 y = –2x + 3 –8 –7 –6 –5 –4 –3 –2 –1–10
−6 15 10 –2
5 –3
−4 11
–4
–10 –5–50 5 10 x
−2 7
–10 y y = 4x + 1 y y = 3x – 7
0 3 4    a 5 (1, 5)
b
0 1 2 3 4 x
4 –1
2 −1 3 –2
2 –3
4 −5 1 –4 (1, –4)
6 −9 0 1 2 3 4 x –5
–6
–7
3 a y 5x – 3y = 10 b y
4 4
2 2 c y d y
4 0 1 2 3 4 x
0 x x –1
–2
–2 2 4 –2–20 2 4 3
2 –2
–4 –4
1 (1, 1) –3 y = –5x – 4
5x + 3y = 10
–4 y = –5x – 4
x
–2 –1–1 0 1 2 3 4 –5
c y d y
4 4 –2 –6
y = –2x + 3
2 2 –7
(1, –9)
–8
0 x x
–4 –2
–2 2 4 –4 –2–20 2 4 –9 (1, –9)
–4
–5x + 3y = 10
–5x – 3y = 10 e y f y
2
y = 1–2 x – 2 y = – 2–7 x + 3
e y f y 1 3
5 10 4x + 4y = 40 0 x 2 (7, 1)
5 –2 –1
–1 1 2 3 4 1
–10 –5–50 5 10
x (2, –1)
x –2
–5–50 5 10 –3
0 1 2 3 4 5 6 7 x
2x – 8y = 20 (1, –9)

g y h y
20 –2x + 8y = –20 g y h y
–x + 6y = 120 5 y = 0.6x + 0.5 8 (1, 8)
10 3.5 7
(5, 3.5)
–100–100 50
x –10 –5–50 5 10 x 6
1.5 5
y y 4 y = 8x
i j 0
1 2 3 4 5
x
5 10 3
x 5 2
–15–10 –5–50 5 10 1
x
–10 –30 –20–10 0 10 20
–5 0
–10 1 2 3x
10x + 30y = –150
y y=x–7
5x + 30y = –150 i
0 x
–1 1 2 3 4 5 6 7
k y l y
10 10 –2
5 5 6x – 4y = –24 –3
–4
x x
–10 –5–50 5 10 –10 –5–50 5 10 –5
–6
–9x + 4y = 36 (1, –6)
–7

y y = 2x – 10 y
m n 20 5    a y y = 2x b y
y = 5x
0 1 2 3 4 5
x y = –5x + 20 2 5
–1 18
–2 16
–3 14 0 1 x
–4 12 0 1 x
–5 10
–6 8 y
–7
c d y
6 y = 1–2 x
–8 4 1–
0 1 x 2
–9 2
–10 –3 0 x
x 1
0 2 4 6 8 10
y = –3x

Chapter 3 • Coordinate geometry 101


number and algebra

y y = 2–3 x y g y = 5x + 2.5 h y = −6x + 3


e f
2 i y = −2.5x + 1.5 j y = 3.5x + 6.5
0 1
x 5    a y = 5x − 19 b y = −5x + 31 c y = −4x − 1
0 3
x y = – –2 x
5 d y = 4x − 34 e y = 3x − 35 f y = −3x + 6
– –25 g y = −2x + 30 h y = 2x − 4.5 i y = 0.5x − 19
6    a b j y = −0.5x + 5.5
y y = 10 y
10 5 6 a
5
t 0 1 2 3
x
–10 –5 0 5 10
x –5
–10 –5–50 5 10 –10
C 2 8 14 20
y = –10
b C
y y 40
c d 36
(6, 38)
10 10 C = 6t + 2
5 5 32
28
x x
–5
–5
0 5 10 –10 –5 0 5 24
–5

Cost ($)
–10 x = 10 –10 20
x = –10 16
12
e y y = 100 f y 8
100 5 4
y=0
50 2
x
–10 –5 0 5 10 0 t
x –5 1 2 3 4 5 6
–10 –5 0 5 10 Time (hours)
–50
c i (0, 2) ii T
 he y-intercept represents the initial cost of
g y h bowling at the alley, which is the shoe rental.
x = –100 y
10 x = 0 10 d m=6
5 5
e C = 6t + 2
–5 0
5
x
–100 –50 0 50
x f $32
–5 –5
–10 –10 g Answers will vary.
7 It does not matter if you rise before you run or run before you
i y rise, as long as you take into account whether the rise or run is
0 x negative.
y−c
8    a m = b y = mx + c
x
–12 y = –12 Challenge 3.1
y = 35x + 2
7    a x-intercept: −0.5; y-intercept: 0.4
b x-intercept: 0.5; y-intercept: −0.4 Exercise 3C — The distance between two points
c x-intercept: 0; y-intercept: 0 1 AB = 5, CD = 2!10 or 6.32, EF = 3!2 or 4.24,
d x-intercept: −3; y-intercept: 12 GH = 2!5 or 4.47, IJ = 5, KL = !26 or 5.10,
e x-intercept: −4; y-intercept: −4 MN = 4!2 or 5.66, OP = !10 or 3.16
f x-intercept: −1; y-intercept: −0.5 2    a 5 b 13 c 10
g x-intercept: 2.75; y-intercept: 2.2 d 7.07 e 6.71 f 14.42
h x-intercept: 7; y-intercept: 3.5
g 13 h 13 i "a2 + 4b2
i x-intercept: 9.75; y-intercept: −3.9
j x-intercept: 23
13
≈ 1.77; y-intercept: 4.6 j 3"a2 + b2
3 B
8    a (2, 0), (0, −8) 4 D
b (−12, 0), (0, 3) 5    a AB = 4.47, BC = 2.24, CD = 4.47, DA = 2.24
c (−5, 0), (0, 25) b AC = 5, BD = 5
9 Answers will vary. c Rectangle
6, 7 and 8 Answers will vary.
Exercise 3B — Determining linear equations
9    a 12 b 5 c 13 d −2.2
1    a y = 2x + 4 b y = −3x + 12 c y = −x + 5 10   Answers will vary.
d y = 2x − 8 e y = 12x + 3 f y = −14x − 4
Exercise 3D — The midpoint of a line segment
g y = 7x − 5 h y = −3x − 15 1    a (−3, −3 12) b (7 12, 0) c (−1, 1)
2    a y = 2x b y = −3x c y = 12x d y = −34x d (0, 1 12) e (2a, 12b) f (a + b, 12a)
3    a y = x + 3 b y = 2x − 1 c y= −12x + 7
2
2 (−3, −10)
3    a (3, 1) b 4.47 c 6.32
d y = 12x + 12 e y = −2x − 2 f y = −x − 8 4 D
4    a y = 3x + 3 b y = −3x + 4 5 C
c y = −4x + 2 d y = 4x + 2 6  a  i (−1, 4) ii (1 12, 1) iii 3.9
e y = −x − 4 f y = 0.5x − 4 b BC = 7.8 = 2PQ.

102 Maths Quest 10 New South Wales Australian curriculum edition Stages 5.1 and 5.2
number and algebra

7 a i (1, −0.5) ii (1, −0.5) y


b The diagonals bisect each other, so it is a parallelogram. 80
60
8   a i (−2, 2) ii  8.94 iii  9.55 iv  9.55
40
b Isosceles. PC is the perpendicular height of the triangle. 20
9 y = −3x − 2
x
10 3y − 2x + 14 = 0 –10 –8 –6 –4 –2 0 2 4 6 8 10
–20
Exercise 3E — Parallel and perpendicular lines –40
–60
1    a No b Yes c No
–80
d No e Yes f No
2 b, f; c, e
3 Answers will vary. 11    a y b   y
y = –5x + 15
y = 3x – 2 15
4 Answers will vary. ( 2– , 0) 1 (1, 1) (0, 15)
5 Answers will vary. 3
x 10 (1, 10)
0
6    a Yes b Yes c No 1
d Yes e Yes f No –2 (0, –2)
7 y = 2x − 9 (3, 0)
8 3x + 2y − 8 = 0 0 1
x
9    a y = 3x + 2 b y = −4x + 9
c y d   y
c 3x − 2y − 8 = 0 d 5y + 2x + 13 = 0 (0, 1) 4
e x + 5y + 5 = 0 f x − 3y + 17 = 0 y = –2–3 x +1
1 ( 3– , 0)
g x − 3y − 14 = 0 2
0 x (2 1– , 0)
10    a 2x − y + 5 = 0 b x + 2y = 0 –1 3 7
(3, –1) 0 x
11    a 3x − 5y + 2 = 0 b 5x + 3y − 8 = 0 5
y = 7–5 x – 3
12    a x = 1 b y = −7
13    a B b C c D d B –3 (0, –3)
14    a (2, 5) b 1
c Answers will vary. d Isosceles triangle 12    a x-intercept = 67, y-intercept c = 6
15 y = −x − 3
16 4x − 6y + 23 = 0 b x-intercept = 40
3
(13 13), y-intercept c = −5
17    a y = −x + 5 b y=x+3 c (1, 4) 21
c x-intercept = 16 5
(1 16 ), y-intercept c = −34
18 Answers will vary.
19 Answers will vary. d x-intercept = −5.6, y-intercept c = 2.8
20 B
13    a b
21 E y y
22    a y = −2x + 1 b 3y + 2x + 1 = 0 2x – 3y = 6 3

23 a, e; b, f; c, h; d, g 0 3 x
–2
24 y = −12x + 32 –1 0 x
y = –3x
25    a m = −85 b m = 18
5
c y d y
26 E
27 B 5x + y = –3 x+y+3=0
28    a 5.10 km b (6.5, 5.5) c 2 – –53 0 x x
–3 0
d y = 2x − 18 e (10, 2) f 7.071 km
29 Answers will vary. –3 –3
Challenge 3.2
37, 68, 125, 230. To find the next number, add the three preceding 14    a y b y

numbers.
(1, 1–2 )
Chapter review 1–
2 0 1 x
Fluency 0 1 x y = –4x
1 A 2 D 3 B –4
y = 1–2 x
4 C 5 A 6 C
7 A 8 C 9 C
10     c d
y y
x = –2
x −10 −8 −6 −4 −2 0 2 4 6 8 10 7 y=7
y 65 55 45 35 25 15 5 −5 −15 −25 −35 –2 0 x
0 x

Chapter 3 • Coordinate geometry 103


number and algebra

15     y c

Total cost ($)


40
7 (0, 7)
30
20
– –7 0 x 10
2
0 2 4 6 8 10
3(y – 5) = 6(x + 1) Number of rides
d $30
16    a y = 2x − 2 b y = −x − 4
3    a b C = 22.50h + 160
c y = −13x + 2 d y = 4x 500
c Approximately $436
400
y = −34

Cost ($)
e f x=5 300
17    a y = 3x − 4 b y = −2x − 5 200
c y = 12x + 5 d y=6 100
0
18    a y = 7x − 13 b y = −3x + 4 0 2 4 6 8 10 12
Time (hours)
c y = 12x + 6 d y = 35x − 18
15 4    a 7x − 3y − 1 = 0 b 3x + 7y − 49 = 0 c −7
19 !61
5    a a = −7
20 Answers will vary.
b B (−7, 50), C (−4, 90)
21 Answers will vary.
c 40.1 metres
22 (0, −18)
23 Answers will vary. Communicating — Rich task
24 Answers will vary. 1 y
16
25 x + 2y − 2 = 0
14
26 2x + 3y − 9 = 0 12
27 3x + 2y − 21 = 0 10
28 3x − 2y + 16 = 0 8
29 a i −45 ii 54 6
4
iii 4x + 5y − 61 = 0 iv 5x − 4y − 25 = 0
2
v (9, 5)
b Square ‒8 ‒6 ‒4 ‒2 0 2 4 6 8 10 12 14 16 x
‒2
1 1
30 a i 10 ii Q− , 1R ‒4
2
1 1 ‒6
iii Q4 , 1 R
2 2 ‒8
b Answers will vary.
Problem solving The symbol is the one used to represent a speaker.
2 The shape is a trapezium.
1    a Number of hours 0 2 4 6 8 10
Area = 12 (length line 6 + length line 8) ×
Pay ($) 0 27 54 81 108 135 perpendicular distance between these lines.

b Pay = $13.50 × (number of hours worked) = 12 (4!2 + 14!2) × 7!2
c = 126 units2
50
40 3 Teacher to check
Pay ($)

30
20 Code puzzle
10 The Hubble telescope was placed into orbit to become the first space
0 1 2 3 4 telescope.
Hours worked (h)
d $91.13
2    a Number
0 2 4 6 8 10
of rides
Cost ($) 12.50 17.50 22.50 27.50 32.50 37.50
b Cost = $2.50 × number of rides + $12.50

104 Maths Quest 10 New South Wales Australian curriculum edition Stages 5.1 and 5.2
number and
NUMBER AND ALGEBRA
algebra

Chapter 4

Simultaneous
­linear equations
and inequalities
WHY LEARN THIS?
Picture this — you own a factory that produces two
different products, and you are planning to buy some
new machines. The big machines are more expensive
than the small ones, take up more floor space and need
more staff to operate, but they can produce more. You
have limited floor space and can only hire 10 staff. You
want to produce two different products to meet the
­projected demand. Which machines should you buy?
When there are many factors to consider, you need
to be able to solve simultaneous equations in order to
determine feasible solutions.

WHAT DO YOU KNOW?


1 Think List what you know about linear equations
and linear inequalities. Use a thinking tool
such as a concept map to show your list.
2 PAIR Share what you know with a partner
and then with a small group.
3 Share As a class, create a large concept
map that shows your class’s knowledge of
linear equations and linear i­nequalities.

LEARNING SEQUENCE
4A Graphical solution of simultaneous linear equations
4B Solving simultaneous ­linear ­equations
using substitution
4C Solving simultaneous ­linear ­equations
using elimination
4D Applications of simultaneous linear equations
4E Solving linear inequalities
Watch this video
The story of mathematics

Searchlight ID: eles-1843


number and algebra

4A  Graphical solution of simultaneous


linear equations
Simultaneous linear equations
•• Simultaneous means occurring at the same time. y
3 y=x+2
•• When a point belongs to more than one line, the coordinates 2
of the point satisfy all equations. The equations of the lines 1
are called simultaneous equations. –4 –3 –2 –1 0 x
–1 1 2 3 4
•• A simultaneous system of equations is a set of two or more –2 y = –x
equations with the same variables. –3
•• To solve simultaneous equations is to calculate the values of
the variables which s­ atisfy all equations in the system.
•• Any two linear graphs will meet at a point, unless they are parallel.
•• At this point, the two equations simultaneously share the same x‐ and y‐coordinates,
which are referred to as the solution.
•• Simultaneous equations can be solved graphically or ­algebraically.

Graphical solution
•• The solution to a pair of simultaneous equations can be found by graphing the two
equations and identifying the coordinates of the point of intersection.
•• An accurate solution depends on drawing an accurate graph.

WORKED EXAMPLE 1
Use the graph of the given simultaneous y
equations below to determine the point of 3 y = 2x – 3
intersection and, hence, the solution of the 2
simultaneous equations. 1 x + 2y = 4
x + 2y = 4
–1 0 1 2 3 4 5 x
y = 2x − 3 –1
–2
TH I N K W R I TE
–3
1 Write the equations and x + 2y = 4 [1]
number them. y = 2x − 3 [2]
2 Locate the point of Point of intersection (2, 1)
intersection of the Solution: x = 2 and y = 1
two lines. This gives y
the solution. 3 y = 2x – 3
2
1 (2, 1) x + 2y = 4

–1 0 1 2 3 4 5 x
–1
–2
–3

108 Maths Quest 10 New South Wales Australian curriculum edition Stages 5.1 and 5.2
number and algebra

3 Check the solution by Check equation [1]:


substituting x = 2 and LHS = x + 2y RHS = 4
y = 1 into the given = 2 + 2(1)
equations. ­Comment on = 4
the results obtained. LHS = RHS

Check equation [2]:


LHS = y RHS = 2x − 3
= 1 = 2(2) − 3
= 4−3
= 1
LHS = RHS
In both cases LHS = RHS, ­therefore the solution
(2, 1) is ­correct.

WORKED EXAMPLE 2
Check whether the given pair of coordinates, (5, −2), is a solution to the
following pair of simultaneous equations.
3x − 2y = 19
4y + x = −3
TH I N K W R I TE

1 Write the equations and number them. 3x − 2y = 19 [1]


4y + x = −3 [2]
2 Substitute x = 5 and y = −2 into Check equation [1]:
equation [1]. LHS = 3x − 2y RHS = 19
= 3(5) − 2(−2)
= 15 + 4
= 19
LHS = RHS
3 Substitute x = 5 and y = −2 into Check equation [2]:
equation [2]. LHS = 4y + x RHS = −3
= 4(−2) + 5
= −8 + 5
= −3
LHS = RHS
In both cases, LHS = RHS.
Therefore, (5, −2) is a solution to
both equations.

Chapter 4 • Simultaneous ­linear equations and inequalities 109


number and algebra

WORKED EXAMPLE 3
Solve the following pair of simultaneous equations using a graphical
­method.
x+y=6
2x + 4y = 20

TH I N K W R I TE /DRAW
1 Write the equations, one x+y=6 [1]
under the other and number 2x + 4y = 20 [2]
them.
2 Calculate the x- and y‐intercepts Equation [1]
for equation [1]. x‐intercept: when y = 0,
For the x‐intercept, substitute x+0 = 6
y = 0 into equation [1]. x =6
For the y‐intercept, substitute The x‐intercept is at (6, 0).
x = 0 into equation [1]. y‐intercept: when x = 0,
0+y = 6
y=6
The y‐intercept is at (0, 6).

3 Calculate the x‐ and y‐intercepts Equation [2]


for equation [2]. x‐intercept: when y = 0,
For the x‐intercept, substitute 2x + 0 = 20
y = 0 into equation [2]. 2x = 20
Divide both sides by 2. x = 10
For the y‐intercept, substitute The x‐intercept is at (10, 0).
x = 0 into equation [2]. y‐intercept: when x = 0,
Divide both sides by 4. 0 + 4y = 20
4y = 20
y= 5
The y‐intercept is at (0, 5)

4 Use graph paper to rule up a y


set of axes and label the x‐axis 6
5 (2, 4)
from 0 to 10 and the y‐axis from 4
3 2x + 4y = 20
0 to 6. 2
1 x
–3–2–1 0 1 2 3 4 5 6 7 8 910
–1
–2 x+y=6
–3

5 Plot the x‐ and y‐intercepts for


each equation.

6 Produce a graph of each


e­ quation by ruling a straight
line through its intercepts.

110 Maths Quest 10 New South Wales Australian curriculum edition Stages 5.1 and 5.2
number and algebra

7 Label each graph.

8 Locate the point of intersection The point of intersection is (2, 4).


of the lines.

9 Check the solution by Check [1]: LHS = x+y RHS = 6


substituting x = 2 and y = 4 into = 2+4
each equation. = 6
LHS = RHS
Check [2]: LHS = 2x + 4y RHS = 20
= 2(2) + 4(4)
= 4 + 16
= 20
LHS = RHS

10 State the solution. In both cases, LHS = RHS. Therefore, the


solution set (2, 4) is correct.
The solution is x = 2, y = 4.

Equations with multiple solutions


•• Two lines are coincident if they lie one on top of the y
other. 4
3
For example, the line in blue and line segment in red at
2 y = 2x, 0.5 < x < 1.5
right
1
are coincident. y = 2x
•• There are an infinite number of solutions to coincident –2 –1 0 1 2 3 4 x
–1
­equations. Every point where the lines coincide
satisfies both equations and hence is a solution to the
simultaneous equations.
•• Coincident equations have the same equation, although the equations may have been
transposed so they look different. For example, y = 2x + 3 and 2y − 4x = 6 are
coincident equations.

Equations with no solutions


•• If two lines do not intersect, there is no simultaneous y
12
­solution to the equations. For example, the lines at
10
right do not intersect, so there is no point that belongs y = 2x – 1
8
to both lines.
6
•• Parallel lines have the same gradient but a different y = 2x + 1
4
y‐intercept. 2
•• For straight lines, the only situation in which the lines
do not cross is if the lines are parallel and not –2 0 2 4 6 8 10 x
–2
coincident.

Chapter 4 • Simultaneous ­linear equations and inequalities 111


number and algebra

Perpendicular lines
•• Perpendicular lines meet at right angles (90°). y
2 y = 2x + 1
•• Perpendicular lines have negative reciprocal gradients:
1
−1
m1 = or m1m2 = −1 where m1 is the gradient of the
m2 –2 –1–10 1 2 3 x
first line and m2 is the gradient of the second line.
–2 y = –1 x + 1
For example, for the two lines at right, m1 = 2 2
–3
and m2 = −1 2
.

Exercise 4A Graphical solution of simultaneous


linear equations
INDIVIDUAL PATHWAYS
Questions: Questions: Questions:
1, 2a–d, 3a–d, 4a–d 1, 2c–g, 3a–d, 4a–f, 5, 7 1, 2e–j, 3, 4, 5, 6, 7, 8
Activity 4‐A‐1 Activity 4‐A‐2 Activity 4‐A‐3
Investigating graphs Graphing simultaneous Further graphing
of simultaneous equations equations of simultaneous
doc‐4990 doc‐4991 equations
doc‐4992

FLUENCY
1 WE1 Use the graphs below to find the solution of the simultaneous equations.

Digital docs a x+y=3 b x+y=2


SkillSHEET x−y=1 3x − y = 2
Substitution into a
linear rule
doc‐5212 y y
x–y=1 6
SkillSHEET 3x – y = 2
5 5
Solving linear equations 4 4
that arise when finding 3 3
x- and y-intercepts 2 2
doc‐5213 1 1 x
SkillSHEET –3 –2–1 0 1 2 3 4 5 x –0.5 –10 0.5 1.0 1.5 2.0 2.5
Transposing linear –1 –2
–2 x+y=2
equations to standard –3
form
–3 x+y=3 –4
doc‐5214
SkillSHEET
Measuring the rise and c y−x=4 d y + 2x = 3
the run
doc‐5215
3x + 2y = 8 2y + x = 0
SkillSHEET
Finding the gradient y y
6 3
given two points 3x + 2y = 8 y–x=4
doc‐5216 4 2 y + 2x = 3
SkillSHEET
Graphing linear 2 1
equations using the x- x x
and y-intercept method –4 –3 –2 –1 0 1 2 3 –1 0 1 2 3 4 5
doc‐5217 –2 –1

–4 –2

–6 –3 2y + x = 0

112 Maths Quest 10 New South Wales Australian curriculum edition Stages 5.1 and 5.2
number and algebra

e y − 3x = 2 f 2y − 4x = 5
x−y=2 4y + 2x = 5

y
6 y
y – 3x = 2 6
4
4
2y – 4x = 5
2 x–y=2
x 2
4y + 2x = 5
–3 –2 –1 0 1 2 3
–2 –1.0 –0.5 0 0.5 1.0 1.5 2.0 x
–2
–4
–4
–6
–6

2 WE2 For the following simultaneous equations, use substitution to check if the
given pair of ­coordinates is a solution.
a (7, 5) 3x + 2y = 31 b (3, 7) y−x=4
2x + 3y = 28 2y + x = 17

c (9, 1) x + 3y = 12 d (2, 5) x−y=7


5x − 2y = 43 2x + 3y = 18

e (4, −3) y = 3x − 15 f (6, −2) x − 2y = 2


4x + 7y = −5 3x + y = 16
g (4, −2) 2x + y = 6 h (5, 1) y − 5x = −24
x − 3y = 8 3y + 4x = 23

i (−2, −5) 3x − 2y = −4 j (−3, −1) y−x=2


2x − 3y = 11 2y − 3x = 7

3 WE3 Solve each of the following pairs of simultaneous equations using a graphical
method.
a x+y=5 b x + 2y = 10
2x + y = 8 3x + y = 15

c 2x + 3y = 6 d x − 3y = −8
2x − y = −10 2x + y = −2

e 6x + 5y = 12 f y + 2x = 6
5x + 3y = 10 2y + 3x = 9

g y = 3x + 10 h y=8
y = 2x + 8 3x + y = 17

i 4x − 2y = −5 j 3x + y = 11
x + 3y = 4 4x − y = 3

k 3x + 4y = 27 l 3y + 3x = 8
x + 2y = 11 3y + 2x = 6

Chapter 4 • Simultaneous ­linear equations and inequalities 113


number and algebra

UNDERSTANDING
4 Using technology, determine which of the following pairs of simultaneous equations
have no solutions. Confirm by finding the gradient of each line.
a y = 2x − 4 b 5x − 3y = 13
3y − 6x = 10 4x − 2y = 10
c x + 2y = 8 d y = 4x + 5
5x + 10y = 45 2y − 10x = 8
e 3y + 2x = 9 f y = 5 − 3x
6x + 4y = 22 3y = −9x + 18
g 4y + 3x = 7 h 2y − x = 0 ­
12y + 9x = 22 14y − 6x = 2

REASONING
5 Two straight lines intersect at the point (3, −4). One of the lines has a y‐intercept
of 8. The second line is a mirror image of the first in the line x = 3. Determine the
equation of the second line. (Hint: Draw a graph of both lines.)
6 At a well‐known beach resort it is possible to hire a jet‐ski by the hour in two
­different locations. On the northern beach the cost is $20 plus $12 per hour, while on
the southern beach the cost is $8 plus $18 per hour. The jet‐skis can be rented for up
to 5 hours.
a Write the rules relating cost to the length of rental for each location.
b On the same set of axes sketch a graph of cost (y‐axis) against length of rental
(x‐axis) for 0–5 hours.
c For what rental times, if any, is the northern beach rental cheaper than the
­southern beach rental? Use your graph to justify your answer.
d For what length of rental time are the two rental schemes identical? Use the graph
and your rules to justify your answer.
7 For each of the pairs of simultaneous equations below, determine whether they are
the same line, parallel lines or intersecting lines.
a 2x − y = − 9 b x−y=7
− 4x − 18 = − 2y x+y=7
c x+6=y d x + y = −2
x+y=6 x+y=7
8 Which of the following problems has one solution, an infinite number of solutions or
no solution? Explain your answers.
a x−y=1
2x − 3y = 2
REFLECTION
b 2x − y = 5 What do you think is the
4x − 2y = − 6 major error made when
solving simultaneous
c x − 2y = − 8
equations graphically?
4x − 8y = − 16

114 Maths Quest 10 New South Wales Australian curriculum edition Stages 5.1 and 5.2
number and algebra

4B  Solving simultaneous linear equations


­using substitution
•• There are two algebraic methods that are commonly used to solve simultaneous
equations.
•• They are the substitution method and the elimination method.

Substitution method
•• The substitution method is particularly useful when one (or both) of the equations is
in a form where one of the two variables is the subject.
•• This variable is then substituted into the other equation, producing a third equation
with only one variable.
•• This third equation can then be used to determine the value of the variable.

WORKED EXAMPLE 4
Solve the simultaneous equations y = 2x − 1 and 3x + 4y = 29 using the
substitution method.

TH I N K W R I TE
1 Write the equations, one under y = 2x − 1 [1]
the other and number them. 3x + 4y = 29 [2]
2 As y and 2x − 1 are equal, Substituting (2x − 1) into [2]:
substitute the expression 3x + 4(2x − 1) = 29
(2x − 1) for y into e­ quation [2].
3 Solve for x.
(i) Expand the brackets on the 3x + 8x − 4 = 29 [3]
left‐hand side of the
equation.
(ii) Collect like terms.      11x − 4 = 29
(iii) Add 4 to both sides of the 11x = 33
­equation.
(iv) Divide both sides by 11. x=3
4 Substitute x = 3 into either of the Substituting x = 3 into [1] :
­equations, say [1], to find the value y = 2(3) − 1
of y. = 6−1
=5
5 Write the solution. Solution: x = 3, y = 5 or (3, 5)
6 Check the solution by substituting Check: Substitute (3, 5) into 3x + 4y = 29.
(3, 5) into equation [2]. LHS = 3(3) + 4(5) RHS = 29
= 9 + 20
= 29
As LHS = RHS, the solution is correct.

Chapter 4 • Simultaneous ­linear equations and inequalities 115


number and algebra

WORKED EXAMPLE 5
Solve the pair of simultaneous equations y = 5x − 8 and y = −3x + 16
using the substitution method.

TH I N K W R I TE
1 Write the equations, one under the other and y = 5x − 8 [1]
number them. y = −3x + 16 [2]
2 Both equations are written with y as the 5x − 8 = −3x + 16
­subject, so equate them.
3 Solve for x.
(i) Add 3x to both sides of the equation. 8x − 8 = 16
(ii) Add 8 to both sides of the equation. 8x = 24
(iii) Divide both sides of the equation by 8. x = 33
4 Substitute the value of x into either of the Substituting x = 3 into [1]:
original equations, say [1], and solve for y. y = 5(3) − 8
= 15 − 8
=7
5 Write the solution. Solution: x = 3, y = 7 or (3, 7)
6 Check the solution by substituting the point Check: Substitute into
of intersection into equation [2]. y = −3x + 16.
LHS = y
=7
RHS = −3x + 16
= −3(3) + 16
=7
As LHS = RHS, the solution is
correct.

Exercise 4B Solving simultaneous ­linear


equations using substitution
INDIVIDUAL PATHWAYS
Questions: Questions: Questions:
1a–d, 2a–d, 4 1a–d, 2c–f, 5 1–5
Activity 4‐B‐1 Activity 4‐B‐2 Activity 4‐B‐3
Learning substitution Practising substitution Tricky substitution
doc‐4993 doc‐4994 doc‐4995

FLUENCY
1 WE4 Solve the following simultaneous equations using the substitution method.
Check your solutions using technology.
a x = −10 + 4y b 3x + 4y = 2 c 3x + y = 7
3x + 5y = 21 x = 7 + 5y x = −3 − 3y

116 Maths Quest 10 New South Wales Australian curriculum edition Stages 5.1 and 5.2
number and algebra

d 3x + 2y = 33 e y = 3x − 3 f 4x + y = 9
y = 41 − 5x −5x + 3y = 3 y = 11 − 5x
g x = −5 − 2y h x = −4 − 3y
5y + x = −11 −3x − 4y = 12
2 WE5 Solve the following pairs of simultaneous equations using the substitution
method. Check your solutions using technology.
a y = 2x − 11 and y = 4x + 1
b y = 3x + 8 and y = 7x − 12
c y = 2x − 10 and y = −3x
d y = x − 9 and y = −5x
e y = −4x − 3 and y = x − 8
f y = −2x − 5 and y = 10x + 1

UNDERSTANDING
3 A small farm has sheep and chickens. There are twice as many chicken as sheep,
and there are 104 legs between the sheep and the chickens. How many chickens
are there?
4 Determine the values of a and b so that the pair of equations ax + by = 17 and
2ax − by = − 11 has a unique solution of ( − 2, 3).
5 The earliest record of magic squares is from China in about 2200 BC. In magic
squares the sums of the numbers of each row, column and diagonal are all equal
to a magic number. Let z be the magic number. By creating a set of equations,
solve to find the magic number and the missing values in the
magic square.
REFLECTION
m 11 7 When would you choose
9 the substitution method
in solving simultaneous
n 5 10 equations?

4C Solving simultaneous linear equations


­using elimination
•• The elimination method is an algebraic method to solve 2x + y 5
simultaneous equations without graphing. 1
1
1 Interactivity
•• If two balanced equations contain the same variables, y
x
x
1
1 Simultaneous equations
the equations can be added or subtracted to eliminate int‐2780

one of the variables. For example, the equations


2x + y = 5 and x + y = 3 are shown at right on Subtract
balance scales.
x+y 3
If the left‐hand side of the second equation is subtracted 1
from the left‐hand side of the first equation, and the y
x
1
1

right‐hand side of the second equation is subtracted from


the right‐hand side of the first equation, the variable y is
eliminated, leaving x = 2.
x 2
1
x 1
Chapter 4 • Simultaneous ­linear equations and inequalities 117
x+y 3
1
y 1
number and algebra x 1

Another way to represent this situation is:


x 2
2x + y = 5 1
− (x + y = 3) x 1

x =2
In this example, the variable is eliminated by subtraction to
reveal the value of x. The value of y can then be calculated
by substituting x = 2 into either equation.
2(2) + y = 5 ⇒ y = 1

WORKED EXAMPLE 6
Solve the following pair of simultaneous equations using the elimination
method.
−2x − 3y = −9   2x + y = 7

TH I N K W R I TE
1 Write the equations, one under the −2x − 3y = −9 [1]
other and number them. 2x + y = 7 [2]
2 Look for an addition or subtraction [1] + [2] :
that will eliminate either x or y. −2x − 3y + (2x + y) = −9 + 7
Note: Adding equations [1] and [2] −2x − 3y + 2x + y = −2
in order will eliminate x. −2y = −2
3 Solve for y by dividing both sides of y=1
the equation by −2.
4 Substitute the value of y into Substituting y = 1 into [2]:
equation [2]. Note: y = 1 may be       2x + 1 = 7
substituted into either equation.
5 Solve for x.
(i) S
 ubtract 1 from both sides of the 2x = 6
      
equation.
(ii) D
 ivide both sides of the equation x=3
      
by 2.
6 Write the solution. Solution: x = 3, y = 1 or (3, 1)
7 Check the solution by ­substituting Check: Substitute into −2x − 3y = −9.
(3, 1) into equation [1], since
LHS = −2(3) − 3(1)
e­ quation [2] was used to find the
= −6 − 3
value of x.
= −9
RHS = −9
LHS = RHS, so the solution is c­ orrect.

•• If a variable is not eliminated when the equations are simply added or subtracted, it
may be necessary to multiply one or both equations by some number or numbers so
that when the equations are added, one of the variables is then eliminated.

118 Maths Quest 10 New South Wales Australian curriculum edition Stages 5.1 and 5.2
number and algebra

•• If two equal quantities are multiplied by the same number, the results remain equal.
3x + 1 4 6x + 2 8
1 1 1 1 1 1
x 1 x x 1 1
x 1 x x 1 1
x 1 x x 1 1
Double both sides and it
remains balanced

WORKED EXAMPLE 7
Solve the following pair of simultaneous equations using the elimination
method.
x − 5y = −17   2x + 3y = 5

TH I N K W R I TE
1 Write the equations, one under the x − 5y = −17 [1]
other and number them. 2x + 3y = 5 [2]
2 Look for a single multiplication that [1] × 2: 2x − 10y = −34 [3]
will create the same coefficient of
­either x or y. Multiply equation [1]
by 2 and call the new equation [3].
3 Subtract equation [2] from [3] in order [3] − [2] :
to eliminate x. 2x − 10y − (2x + 3y) = −34 − 5
2x − 10y − 2x − 3y = −39
−13y = −39
4 Solve for y by dividing both sides of y=3
the equation by −13.
5 Substitute the value of y into Substituting y = 3 into [2]:
equation [2]. 2x + 3(3) = 5
2x + 9 = 5
6 Solve for x.
(i) Subtract 9 from both sides of the 2x = −4
equation.
(ii) Divide both sides of the equation x = −2
by 2.
7 Write the solution. Solution: x = −2, y = 3 or (−2, 3)
8 Check the solution by substituting into Check: Substitute into x − 5y = −17.
equation [1]. LHS = (−2) − 5(3)
= −2 − 15
= −17
RHS = −17
LHS = RHS, so the solution is correct.

Note: In this example, equation [1] could have been multiplied by −2 (instead of by 2),
then the two equations added (instead of subtracted) to eliminate x.

Chapter 4 • Simultaneous ­linear equations and inequalities 119


number and algebra

WORKED EXAMPLE 8
Solve the following pair of simultaneous equations using the elimination
method. 6x + 5y = 3   5x + 4y = 2

TH I N K W R I TE
1 Write the equations, one under 6x + 5y = 3 [1]
the other and number them. 5x + 4y = 2 [2]
2 Decide which variable to eliminate, Eliminate y.
say y.
Multiply equation [1] by 4 and call
the new equation [3]. [1] × 4: 24x + 20y = 12 [3]
Multiply equation [2] by 5 and call
the new equation [4]. [2] × 5: 25x + 20y = 10 [4]
3 Subtract equation [3] from [4] in [4] − [3]:
order to eliminate y. 25x + 20y − (24x + 20y) = 10 − 12
25x + 20y − 24x − 20y = −2
x= −2
4 Substitute the value of x into Substituting x = −2 into [1]:
­equation [1]. 6(−2) + 5y = 3
−12 + 5y = 3
5 Solve for y.
(i) Add 12 to both sides of the
­equation 5y = 15
(ii) D  ivide both sides of the
­equation by 5. y=3
6 Write the solution. Solution: x = −2, y = 3 or (−2, 3)
7 Check the solution by substituting Check: Substitute into 5x + 4y = 2.
the solution into equation [2]. LHS = 5(−2) + 4(3)
= −10 + 12
=2
RHS = 2
LHS = RHS, so the solution is correct.

Note: Equation [1] could have been multiplied by −4 (instead of by 4), then the two
equations added (instead of subtracted) to eliminate y.

Exercise 4C Solving simultaneous linear


equations using elimination
INDIVIDUAL PATHWAYS
Questions: Questions: Questions:
1–4, 6 1–5, 7 1–5, 7

Activity 4‐C‐1 Activity 4‐C‐2 Activity 4‐C‐3


Elimination practice Let’s eliminate More elimination
doc‐4996 doc‐4997 doc‐4998

120 Maths Quest 10 New South Wales Australian curriculum edition Stages 5.1 and 5.2
number and algebra

FLUENCY
1 WE 6 Solve the following pairs of simultaneous equations by adding equations to
eliminate either x or y.
a x + 2y = 5 b 5x + 4y = 2 c −2x + y = 10
−x + 4y = 1 5x − 4y = −22 2x + 3y = 14
2 Solve the following pairs of equations by subtracting equations to eliminate either
x or y.
a 3x + 2y = 13 b 2x − 5y = −11 c −3x − y = 8
5x + 2y = 23 2x + y = 7 −3x + 4y = 13
3 Solve each of the following equations using the elimination method.
a x + 2y = 12 b 3x + 2y = −23
3x − 2y = 12 5x + 2y = −29
c 6x + 5y = −13 d 6x − 5y = −43
−2x + 5y = −29 6x − y = −23
e x − 4y = 27 f −4x + y = −10
3x − 4y = 17 4x − 3y = 14
4 WE7 Solve the following pairs of simultaneous equations.
a x + 2y = 4 b 3x + 2y = 19
3x − 4y = 2 6x − 5y = −7
c −2x + 3y = 3 d 6x + y = 9
5x − 6y = −3 −3x + 2y = 3
e x + 3y = 14 f 5x + y = 27
3x + y = 10 4x + 3y = 26
g −6x + 5y = −14 h 2x + 5y = 14
−2x + y = −6 3x + y = −5
i −3x + 2y = 6
x + 4y = −9

5 WE8 Solve the following pairs of simultaneous equations.


a 2x + 3y = 16 b 5x − 3y = 6
3x + 2y = 19 3x − 2y = 3
c 3x + 2y = 6 d 2x + 7y = 3
4x + 3y = 10 3x + 2y = 13
e 2x − 3y = 14 f −3x + 7y = −2
3x − 5y = 21 4x + 2y = 14

UNDERSTANDING
6 Solve the following simultaneous equations using an appropriate method. Check
your answer using technology.
a 7x + 3y = 16 b 2x + y = 8
y = 4x − 1 4x + 3y = 16

Chapter 4 • Simultaneous ­linear equations and inequalities 121


number and algebra

c −3x + 2y = 19 d −3x + 7y = 9
4x + 5y = 13 4x − 3y = 7
e −4x + 5y = −7 f y = −x
x = 23 − 3y y = −25x − 15

REASONING
7 Ann, Beth and Celine wanted to weigh
themselves on a coin weighing machine.
Digital doc
WorkSHEET 4.1
The problem was they only had enough
doc‐5220 money for one weighing. They decided to
weigh themselves in pairs, one stepping off
as another stepped on.
• Ann and Beth weighed 119 kg.
• Beth and Celine weighed 112 kg.
• Celine and Ann weighed 115 kg.
How much did each of the girls weigh?

REFLECTION
How does eliminating
one variable help to solve
simultaneous equations?

CHALLENGE 4.1

122 Maths Quest 10 New South Wales Australian curriculum edition Stages 5.1 and 5.2
number and algebra

4D  Applications of simultaneous


linear ­equations
•• There are many practical applications of simultaneous equations, same examples of
which are shown below.
•• When solving practical problems, the following steps can be useful.
•• Define the unknown quantities using appropriate pronumerals.
•• Use the information given in the problem to form two equations in terms of these
pronumerals.
•• Solve these equations using an appropriate method.
•• Write the solution in words.
•• Check the solution.

WORKED EXAMPLE 9
Ashley received better results for his Mathematics test than for his English
test. If the sum of the two marks is 164 and the difference is 22, calculate
the mark he received for each subject.

TH I N K W R I TE
1 Define the two variables. Let x = the Mathematics mark.
Let y = the English mark.
2 Formulate two equations from the x + y = 164 [1]
information given and number them. x − y = 22 [2]
The sum of the two marks is x + y.
The difference of the two marks is
x − y.
3 Use the elimination method by adding [1] + [2]: 2x = 186
equations [1] and [2] to eliminate y.
4 Solve for x by dividing both sides of x = 93
the equation by 2.
5 Substitute the value of x into Substituting x = 93 into [1]:
equation [1]. x + y = 164
93 + y = 164
6 Solve for y by subtracting 93 from y = 71
both sides of the equation.
7 Write the solution. Solution:
Mathematics mark (x) = 93
English mark (y) = 71
8 Check the solution by substituting Check: Substitute into x + y = 164.
x = 93 and y = 71 into equation [1]. LHS = 93 + 71 RHS = 164
= 164
As LHS = RHS, the solution is correct.

Chapter 4 • Simultaneous ­linear equations and inequalities 123


number and algebra

WORKED EXAMPLE 10
To finish a project, Genevieve buys a
total of 25 nuts and bolts from a ­
hardware store. If each nut costs 12 cents,
each bolt costs 25 cents and the total
purchase price is $4.30, how many
nuts and how many bolts does
­Genevieve buy?

TH I N K W R I TE
1 Define the two variables. Let x = the number of nuts.
Let y = the number of bolts.

2 Formulate two equations from x + y = 25 [1]


the ­information given and 12x + 25y = 430 [2]
number them.
Note: The total number of nuts
and bolts is 25. Each nut cost
12 cents, each bolt cost 25 cents
and the total cost is 430 cents
($4.30).

3 Solve simultaneously using


the ­substitution method since
equation [1] is easy to
rearrange.

4 Rearrange equation [1] to Rearrange equation [1]:


make x the subject by x + y = 25
subtracting y from both sides x = 25 − y
of equation [1].

5 Substitute the expression (25 − y) Substituting (25 − y) into [2]:


for x into equation [2]. 12(25 − y) + 25y = 430

6 Solve for y. 300 − 12y + 25y = 430


300 + 13y = 430
13y + 300 = 430
13y = 130
y = 10

7 Substitute the value of y into the Substituting y = 10 into x = 25 − y:


­rearranged equation x = 25 − y x = 25 − 10
from step 4. x = 15

124 Maths Quest 10 New South Wales Australian curriculum edition Stages 5.1 and 5.2
number and algebra

8 Write the solution. Solution:


The number of nuts (x) = 15.
The number of bolts (y) = 10.
9 Check the solution by substituting Check: Substitute into x + y = 25.
x = 15 and y = 10 into equation [1]. LHS = 15 + 10 RHS = 25
= 25
As LHS = RHS, the solution is ­correct.

Exercise 4D Applications of simultaneous


linear equations
INDIVIDUAL PATHWAYS
Questions: Questions: Questions:
1–3, 6, 8, 12, 14 1, 2, 4, 7, 9, 11, 14 1, 2, 5, 7, 9, 10, 13, 15
Activity 4‐D‐1 Activity 4‐D‐2 Activity 4‐D‐3
Problem solving Harder problem solving Tricky problem solving
doc‐4999 doc‐5000 doc‐5001

FLUENCY
1 WE9 Rick received better results for his Maths test than for his English test. If the
sum of his two marks is 163 and the difference is 31, find the mark for each subject.
2 WE10 Rachael buys 30 nuts and bolts to finish a project. If each nut costs 10 cents,
each bolt costs 20 cents and the total purchase price is $4.20, how many nuts and
how many bolts does she buy?
UNDERSTANDING
3 Find two numbers whose difference is 5 and whose sum is 11.
4 The difference between two numbers is 2. If three times the larger number minus
twice the smaller number is 13, find the two numbers.
5 One number is 9 less than three times a second number. If the first number plus
twice the second number is 16, find the two numbers.
6 A rectangular house has a perimeter of 40 metres and the length is 4 metres more
than the width. What are the dimensions of the house?
7 Mike has 5 lemons and 3 oranges in his shopping basket. The cost of the fruit is
$3.50. Voula, with 2 lemons and 4 oranges, pays $2.10 for her fruit. How much does
each type of fruit cost?

     
8 A surveyor measuring the dimensions of a block of land finds that the length of
the block is three times the width. If the perimeter is 160 metres, what are the
dimensions of the block?

Chapter 4 • Simultaneous ­linear equations and inequalities 125


number and algebra

9 Julie has $3.10 in change in her pocket. If she has only 50 cent and 20 cent pieces
and the total number of coins is 11, how many coins of each type does she have?
10 Mr Yang’s son has a total of twenty‐one $1 and $2 coins in his moneybox. When he
counts his money, he finds that its total value is $30. How many coins of each type
does he have?

11 If three Magnums and two Paddlepops cost $8.70 and the difference in price
­between a Magnum and a Paddlepop is 90 cents, how much does each type of ­
ice‐cream cost?
12 If one Redskin and 4 Golden roughs cost $1.65, whereas 2 Redskins and 3 Golden
roughs cost $1.55, how much does each type of sweet cost?

REASONING
13 A catering firm charges a fixed cost for overheads and a price per person. A party
for 20 people costs $557, whereas a party for 35 people costs $909.50. What is the
fixed cost and the cost per person charged by the company?

14 The difference between Sally’s PE mark and Science mark is 12, and the sum of
the marks is 154. If the PE mark is the higher mark,
what did Sally get for each subject? REFLECTION
15 Mozza’s cheese supplies sells six Mozzarella cheeses How do you decide which
and eight Swiss cheeses to Munga’s deli for $83.60, method to use when
solving problems using
Digital doc and four Mozzarella cheeses and four Swiss cheeses to
simultaneous linear
WorkSHEET 4.2 Mina’s deli for $48. How much does each type of equations?
doc‐13715
cheese cost?

126 Maths Quest 10 New South Wales Australian curriculum edition Stages 5.1 and 5.2
number and algebra

CHALLENGE 4.2
1

4E  Solving linear inequalities


Inequalities between two expressions
•• An equation is a statement of equality such as x = 2; an inequation is a statement of
inequality between two expressions such as x < 2 (x is less than 2).
•• The solution to a linear equation is a single point on a number line, but there are an
infinite number of solutions to an inequality.
•• The following table shows examples of four types of simple inequalities and their
corresponding representations on a number line.
•• Note that an open circle placed over the 2 indicates that 2 is not included; that is,
2 does not satisfy the inequality. A closed or solid circle indicates that 2 is included;
that is, it does satisfy the inequality.

Mathematical English Number line


statement statement diagram

x>2 x is greater than 2 –10 –8 –6 –4 –2 0 2 4 6 8 10 x

x≥2 x is greater than or equal to 2 –10 –8 –6 –4 –2 0 2 4 6 8 10


x

x<2 x is less than 2 –10 –8 –6 –4 –2 0 2 4 6 8 10


x

x≤2 x is less than or equal to 2 –10 –8 –6 –4 –2 0 2 4 6 8 10


x

Chapter 4 • Simultaneous ­linear equations and inequalities 127


number and algebra

Solving inequalities
•• The following things may be done to both sides of an inequality without affecting
its truth.
–– A number can be added or subtracted from both sides of the inequality.

Adding or subtracting a number:


e.g. 6 > 2 Add 3 to both +3 +3
sides:
9>5 (True) 0 1 2 3 4 5 6 7 8 9 10

6≥2 Subtract 3 from –3 –3


both sides:
3 ≥ −1 (True) –2 –1 0 1 2 3 4 5 6 7 8

Adding or subtracting moves both numbers the same distance along the number line.

–– A number can be multiplied or divided by a positive number.

Multiplying or dividing by a positive number:


e.g. 6>2 Multiply both × 12
sides by 12: × 12
3>1 (True)
–1 0 1 2 3 4 5 6 7

The distance between the numbers has changed, but their relative position has not.

–– Care must be taken when multiplying or dividing by a negative number.

Multiplying or dividing by a negative number:


e.g. 6 > 2 Multiply x –1
both
x –1
sides by
−1:
−6 > −2 (False)
–6 –4 –2 0 2 4 6

Multiplying or dividing by a negative number reflects numbers about x = 0. Their relative ­positions are
reversed.

–– When solving inequalities, if both sides are multiplied or divided by a negative


­number, then the inequality sign must be reversed.
For example, 6 > 2 implies that −6 < −2.

128 Maths Quest 10 New South Wales Australian curriculum edition Stages 5.1 and 5.2
number and algebra

WORKED EXAMPLE 11
Solve each of the following linear inequalities and show the solution on a number line.
a 4x − 1 < − 2 b 6x − 7 ≥ 3x + 5

TH I N K W R I TE
a 1 Write the inequality. a 4x − 1 < −2
2 Add 1 to both sides of the 4x − 1 + 1 < −2 + 1
i­nequality. 4x < −1
3 Obtain x by dividing both sides 4x 1
<−
of the inequality by 4. 4 4
1
x < −4
x < – 14

x
–2 –1 –1 0 1
4

b 1 Write the inequality. b 6x − 7 ≥ 3x + 5


2 Subtract 3x from both sides of 6x − 7 − 3x ≥ 3x + 5 − 3x
the inequality. 3x − 7 ≥ 5
3 Add 7 to both sides of the 3x − 7 + 7 ≥ 5 + 7
inequality. 3x ≥ 12
4 Obtain x by dividing both sides 3x 12

of the inequality by 3. 3 3
x ≥4
x≥4

x
0 2 4 6 8 10

WORKED EXAMPLE 12
Solve each of the following linear inequalities.
a −3m + 5 < −7 b 5(x − 2) ≥ 7(x + 3)

TH I N K W R I TE
a 1 Write the inequality. a − 3m + 5 < −7
2 Subtract 5 from both sides of − 3m + 5 − 5 < −7 − 5
the ­inequality. (No change to the − 3m < −12
­inequality sign.)
3 Obtain m by dividing both sides −3m −12
>
of the inequality by −3. Reverse −3 −3
the inequality sign, since you are m >4
dividing by a negative number.
b 1 Write the inequality. b 5(x − 2) ≥ 7(x + 3)
2 Expand both brackets. 5x − 10 ≥ 7x + 21

Chapter 4 • Simultaneous ­linear equations and inequalities 129


number and algebra

3 Subtract 7x from both sides of the 5x − 10 − 7x ≥ 7x + 21 − 7x


inequality. −2x − 10 ≥ 21
4 Add 10 to both sides of the −2x − 10 + 10 ≥ 21 + 10
inequality. −2x ≥ 31
5 Divide both sides of the inequality −2x
≤ 31
by −2. Since we need to divide −2 −2
by a negative number, reverse the −31
direction of the inequality sign. x ≤
2
x ≤ −1512

Exercise 4E Solving linear inequations


INDIVIDUAL PATHWAYS
Questions: Questions: Questions:
1a–f, 2, 3a–f, 4a–f, 5a–c, 6a–g, 1e–l, 2, 3d–i, 4d–i, 5a–c, 6a–l, 1j–l, 2g–l, 3g–l, 4d–l, 5d–f,
7, 8a–c, 9a–f, 10 7, 8a–c, 9a–i, 10, 12 6f–o, 8d–f, 9–12
Activity 4‐E‐1 Activity 4‐E‐2 Activity 4‐E‐3
Puzzling inequations 1 Puzzling inequations 2 Puzzling inequations 3
doc‐5002 doc‐5003 doc‐5004

FLUENCY
1 WE11a Solve each of the following inequalities.
a x+1>3 b a+2>1 c y−3≥4
d m−1 ≥ 3 e p+4<5 f x+2<9
g m − 5≤4 h a−2≤5 i x − 4 > −1
j 5+m≥7 k 6+q≥2 l 5 + a > −3
2 Solve each of the following inequalities. Check your solutions by substitution.
a 3m > 9 b 5p ≤ 10 c 2a < 8
d 4x ≥ 20 e 5p > −25 f 3x ≤ −21
m
g 2m ≥ −1 h 4b > −2 i >6
3
x a m
j <4 k ≤ −2 l ≥5
2 7 5
3 WE11b Solve each of the following inequalities.
a 2m + 3 < 12 b 3x + 4 ≥ 13 c 5p − 9 > 11
d 4n − 1 ≤ 7 e 2b − 6 < 4 f 8y − 2 > 14
g 10m + 4 ≤ −6 h 2a + 5 ≥ −5 i 3b + 2 < −11
j 6c + 7 ≤ 1 k 4p − 2 > −10 l 3a − 7 ≥ −28
4 WE12a Solve each of the following inequalities.
a 2m + 1 > m + 4 b 2a − 3 ≥ a − 1 c 5a − 3 < a − 7
d 3a + 4 ≤ a − 2 e 5x − 2 > 40 − 2x f 7x − 5 ≤ 11 − x
g 7b + 5 < 2b + 25 h 2(a + 4) > a + 13 i 3(m − 1) < m + 1
j 5(2m − 3) ≤ 3m + 6 k 3(5b + 2) ≤ −10 + 4b l 5(3m + 1) ≥ 2(m + 9)

130 Maths Quest 10 New South Wales Australian curriculum edition Stages 5.1 and 5.2
number and algebra

5 Solve each of the following inequalities.


x+1 x−2 x+7
a ≤4 b ≥ −4 c < −1
2 5 3 Digital doc
SkillSHEET
d 2x + 3 e 3x − 1 ≥ 2 f 5x + 9 < 0 Checking whether a
>6 7 given point makes
4 6 the inequation a true
statement
6 WE12b Solve each of the following inequalities. doc‐5218

a −2m > 4 b −5p ≤ 15 c −2a ≥ −10

d −p−3 ≤ 2 e 10 − y ≥ 13 f 14 − x < 7

g 1 − 6p > 1 h 2 − 10a ≤ 0 i 2(3 − x) < 12

j −4(a + 9) ≥ 8 k −15 ≤ −3(2 + b) l 2x − 3 > 5x + 6

m k + 5 < 2k − 3 n 3(x − 4) < 5(x + 5) o 7(a + 4) ≥ 4(2a − 3)


7 MC When solving the inequality −2x > −7 we need to:
A change the sign to ≥ B change the sign to < C change the sign to =
D change the sign to ≤ E keep the sign unchanged
8 Solve each of the following inequalities.
2−x 5−m
a >1 b ≥2 −3 − x
3 4 c < −4
5
3 − 8a 4 − 3m −2m + 6
d < −1 e ≤0 f ≤3
2 2 10
9 Solve each of the following inequalities.
a 3k > 6 b −a − 7 < −2 c 5 − 3m ≥ 0
d x+4>9 e 10 − y ≤ 3 f 5 + 3d < −1
7p 1−x −4 − 2m
g ≥ −2 h ≤2 i >0
3 3 5
j 5a − 2 < 4a + 7 k 6p + 2 ≤ 7p − 1 l 2(3x + 1) > 2x − 16

UNDERSTANDING
10 Write linear inequalities for the following statements, using x to represent the
­unknown. (Do not attempt to solve the equations.)
Digital doc
a The product of 5 and a certain number is greater than 10. SkillSHEET
b When three is subtracted from a certain number the result is less than or equal to 5. Writing equations from
worded statements
c The sum of seven and three times a certain number is less than 42. doc‐5219

REASONING
11 Solve the following inequations and sketch their solutions on a number line.
a 2 − 4x ≤ 18
b 3x + 4 < 5x − 12
12 Given the positive numbers a, b, c and d and the variable x, there is the following
relationship: −c < ax + b < −d.
a Find the possible range of values of x if a = 2, b = 3, c = 10 and d = 1
b Rewrite this relationship in terms of x only (x by itself between the < signs).

Chapter 4 • Simultaneous ­linear equations and inequalities 131


number and algebra

13 Two speed boats are racing along a section of Lake Quikalong. The speed limit
along this section of the lake is 50 km/h. Ross is travelling 6 km/h faster than
Steven and together they are travelling at a speed greater than 100 km/h.
a Write an inequation and solve it to describe all possible speeds that Steven could
be travelling at.
b At Steve’s lowest possible speed, is he over the speed limit?
c The water police issue a warning to Ross for ­exceeding the speed limit on the
lake. Show that the police were justified in issuing a warning to Ross.

14 A family has a swimming pool that is 15  m long and 7  m wide. They want to pave a
path around the swimming pool, but they are not sure how wide to make it.
a Using x to represent the width of the path, write an expression for the total pool
area including the path.
b The family discovers that the total area of their REFLECTION
pool (including the path) cannot come to more What is similar and
than 165  m2. different when solving
linear inequations to linear
i Write an inequation to represent this situation.
­equations?
ii What widths can the path take?

132 Maths Quest 10 New South Wales Australian curriculum edition Stages 5.1 and 5.2
number and algebra

CHAPTER REVIEW
LANGUAGE

collinear inequality parallel simultaneous


elimination intersection perpendicular solution
equation number line reverse substitution

int‐2835 doc‐13717
int‐2836 doc‐13718
int‐3591

FLUENCY
1 The inequality represented by the line shown below is:

x
–5 –4 –3 –2 –1

a −5 ≤ x ≤ −1 b −5 ≤ x < −1 c −5 < x < −1


d −5 < x ≤ −1 e −1 < x ≤ −5
2 The equation of a linear graph that passes through the origin with gradient −3 is:
A y = −3 B x = −3 C y = −3x D y = 3 − 3x E y = 3x − 3
3 An online music shop charges $5 postage for 2 CDs and $11 for 5 CDs. The
equation that best represents this, if C is the cost and n is the number of CDs, is:
a C = 5n + 11 b C = 6n + 5 c C = n + 2
d C = 5n + 1 e C = 2n + 1
4 During a walk‐a‐thon, Sarah receives $4 plus $3 per kilometre. The graph that best
represents Sarah walking up to 5 kilometres is:
A $ B $ C $
18 (5, 18) 24 (5, 24) 24
15 20 20 (5, 19)
12 16 16
9 12 12
6 8 8
3 4 4
0 d (km) 0 d (km) 0 d (km)
1 2 3 4 5 1 2 3 4 5 1 2 3 4 5

D $ E $
18 (5, 19) 24
15 20
(5, 18)
12 16
9 12
6 8
3 4
0 d (km) 0 1 2 3 4 5 d (km)
1 2 3 4 5

Chapter 4 • Simultaneous ­linear equations and inequalities 133


number and algebra

5 Which of the following pairs of coordinates is the solution to the given simultaneous
equations?
2x + 3y = 18
5x − y = 11
A (6, 2) B (3, −4) C (3, 9) D (3, 4) E (5, 11)
6 The graphical solution to the following pair of simultaneous equations is:
y = 5 − 2x
y = 3x − 10
A y B y
10 10
8 8
6 5 6
1 1 5
4 1
22 33 –3 3 2 1 4
2
2 2

–6 –5 –4 –3 –2 –1
–2
0 1 2 3 4 5x –6 –5 –4 –3 –2 –1
–2
0 1 2 3 4 5x
(3, –1) (–3, –1) –4
–4
–6 –6
–8 –8
–10 –10

C y D y
10 10
8 8
6 6
4 4
2 (3, 1) (–3, –1) 2

–6 –5 –4 –3 –2 –1
–2
0 1 2 3 4 5x –6 –5 –4 –3 –2 –1
–2
0 1 2 3 4 5x
1
–4 2
1 33 1 1
–3 3 –2 2 –4 –5
–6 –5 2 –6
–8 –8
–10 –10

E none of the above


7 Find the equation of the straight line that passes through each pair of points.
a (1, 7) and (3, 5) b (8, 0) and (6, 3) c (−1, 5) and (4, 7)
8 Use the graphs below, showing the given simultaneous equations, to write the point of
intersection of the graphs and, hence, the solution of the simultaneous equations.
a x + 3y = 6 b 3x + 2y = 12
y = 2x − 5 2y = 3x
y
y
6
4
4
2
2

–4 –2 0 2 4 6 x
–4 –2 0 2 4 6 x
–2
–2
–4
–4
–6

134 Maths Quest 10 New South Wales Australian curriculum edition Stages 5.1 and 5.2
number and algebra

9 Use substitution to check if the given pair of coordinates is a solution to the


given simultaneous equations.
a (7, 1) x − 2y = 5 b (4, 3) y=7−x
5y + 2x = 18 5y − 2x = 7
10 Solve each of the following pairs of simultaneous equations using a graphical
method.
a 4y − 2x = 8 b y = 2x − 2 c 2x + 5y = 20
x + 2y = 0 x − 4y = 8 y=2
11 Solve the following simultaneous equations using the substitution method.
a y = 3x + 1 b y = 2x + 7
x + 2y = 16 3y − 4x = 11
c 2x + 5y = 6 d y = −x
3
y= x+5 y = 8x + 21
2
e y = 3x − 11 f y = 4x − 17
y = 5x + 17 y = 6x − 22
12 Solve the following simultaneous equations using the elimination method.
a 3x + y = 17 b 4x + 3y = 1
7x − y = 33 −4x + y = 11
c 3x − 7y = −2 d 4y − 3x = 9
−2x − 7y = 13 y + 3x = 6
e 5x + 2y = 6 f x − 4y = −4
4x + 3y = 2 4x − 2y = 12
13 Solve the following simultaneous equations using an appropriate method.
a 3x + 2y = 6 b 6x − 4y = −6 c 6x + 2y = 14
3y + 5x = 9 7x + 3y = −30 x = −3 + 5y

PROBLEM SOLVING
1 Write the following as a pair of simultaneous equations and solve.
a Find two numbers whose difference is 5 and whose sum is 23.
b A rectangular house has a total perimeter of 34 metres and the width
is 5 metres less than the length. What are the dimensions of the house?
c If two Chupa Chups and three Wizz Fizzes cost $2.55, but five
Chupa Chups and seven Wizz Fizzes cost $6.10, find the price of
each type of lolly.
2 Laurie buys milk and bread for his family on the way home from
school each day, paying with a $10 note. If he buys three cartons of
milk and two loaves of bread, he receives 5 cents in change. If he
buys two cartons of milk and one loaf of bread, he receives $4.15 in
change. How much does each item cost?
3 A paddock contains some cockatoos (2‐legged) and kangaroos
(4‐­legged). The ­total number of animals is 21 and they have
68 legs in total. Using s­ imultaneous equations, determine
how many cockatoos and kangaroos there are in the
paddock.

Chapter 4 • Simultaneous ­linear equations and inequalities 135


number and algebra

4 There are two sections to a concert hall. Seats in the ‘Dress circle’ are arranged in rows
of 40 and cost $140 each. Seats in the ‘Bleachers’ are arranged in rows of 70 and cost
$60 each. There are 10 more rows in the ‘Dress circle’ than in the ­‘Bleachers’ and the
capacity of the hall is 7000.


a If d represent the number of rows in the ‘Dress circle’ and b represents the number
of rows in the ‘Bleachers’, write an equation in terms of these two variables based
on the fact that there are 10 more rows in the ‘Dress circle’ than in the ‘Bleachers’.
b Write an equation in terms of these two ­variables based on the fact that the
­capacity of the hall is 7000 seats.
c Solve the two equations from a and b simultaneously using the method of your
choice to find the number of rows in each section.
d Now that you have the number of rows in each section, ­calculate the number of
seats in each section.
e Hence, calculate the total receipts for a concert where all tickets are sold.
5 John is comparing two car rental companies, Golden Ace Rental Company and
Silver D ­ iamond Rental Company. Golden Ace Rental Company charges a flat rate of
$38 per day and $0.20 per kilometre. The Silver Diamond
Rental Company charges a flat rate of $30 per day plus
$0.32 per kilometre.
a Write an expression for the cost of renting a car for
three days from the Golden Ace Rental Company in
terms of the number of kilometres travelled, k.
b Write an expression for the cost of renting a car for
three days from the Silver Diamond Rental Company
in terms of the number of kilometres travelled, k.
c How many kilometres would John have to travel so
that the cost of hiring from each company for three
days is the same?
d For what number of kilometres will it be ­cheaper
to use Silver Diamond Rental Company for three
days’ hire?

136 Maths Quest 10 New South Wales Australian curriculum edition Stages 5.1 and 5.2
number and algebra

6 Frederika has $24 000 saved for a holiday and a new stereo. Her travel expenses are
$5400 and her daily expenses are $260.

a Write down an equation for the cost of her holiday if she stays for d days.
Upon her return from holidays Frederika wants to purchase a new stereo system
that will cost her $2500.
b How many days can she spend on her holiday if she wishes to purchase a new
stereo upon her return?
7 Mick the painter has fixed costs (e.g. insurance, equipment, etc) of $3400 per year.
His running cost to travel to jobs is based on $0.75 per kilometre. Last year Mick
had costs that were less than $16 000.

a Write an inequality to show this information and solve it to find how many
kilometres Mick travelled for the year.
b Explain the information you have found.

Chapter 4 • Simultaneous ­linear equations and inequalities 137


number and algebra
Communicating

rich task

Documenting business expenses


Comparison of car hire companies

Cost of Car Hire


400
300 Plan 1
200
Plan 2
100
0
250
Kilometres travelled

Jim works as a travelling sales representative.


He needs to plan his next business trip to Port
Hedland, which he anticipates will take him away
from the office for 3 or 4 days. Due to other work
commitments, he is not sure whether he can make
the trip by the end of this month or early next month. He plans to fly to Port Hedland and use a
hire car to travel when he arrives. Jim’s boss has asked him to supply documentation detailing
the anticipated costs for the hire car, based on the following quotes received.
A1 Rentals $35 per day plus 28c per kilometre of travel
Cut Price Rentals $28 per day plus 30c per kilometre of travel
Jim is aware that, although the Cut Price Rentals deal looks cheaper, it could work out more
expensive in the long run, because of the higher cost per kilometre of travel; he intends to
travel a considerable distance. Jim is advised by both rental companies that their daily hire
charges are due to rise by $2 per day from the first day of next month.

138 Maths Quest 10 New South Wales Australian curriculum edition Stages 5.1 and 5.2
number and algebra

Assuming that Jim is able to travel this month and his trip will last 3 days, use the information
given to answer questions 1 to 3.
1 Write equations to represent the costs of hiring a car from A1 Rentals and Cut Price Rentals. Use
the pronumeral C to represent the cost (in dollars) and d to represent the distance travelled (in
kilometres).
2 Plot the two equations from question 1 on the set of axes provided to show how the costs
compare over 1500 km.
Comparison of cost of hiring a car from A1 Rentals and Cut Price Rentals
C
600
500
Cost ($)

400
300
200
100

0 d
200 400 600 800 1000 1200 1400 1600
Distance travelled (km)
3 Use the graph to determine how many kilometres Jim would have to travel to make the hire
costs the same for both rental companies.
4 Assume Jim’s trip is extended to four days. Use an appropriate method to show how this
­changes the answer found in question 3.
For questions 5 to 7, assume that Jim has delayed his trip until next month when the hire
charges have increased.
5 Write equations to show the cost of hiring a car from both car rental companies for a trip ­lasting:
a 3 days b 4 days.
6 Plot the four equations from question 5 on the set of axes provided, to show how the costs
­compare over 1500 km.
Comparison of cost of hiring a car from A1 Rentals and Cut Price Rentals
C
600
500
400
Cost ($)

300
200
100

0 d
200 400 600 800 1000 1200 1400 1600
Distance travelled (km)

7 Comment on the results displayed in your graph.


8 Jim needs to provide his boss with documentation of the hire car costs, catering for all options.
On a separate sheet of paper, prepare a document for Jim to hand to his boss.

Chapter 4 • Simultaneous ­linear equations and inequalities 139


number and
NUMBER AND algebra
ALGEBRA

Code puzzle

Find the longest‐lived


creatures
Solve the simultaneous equations to find the puzzle’s code.

Insects:
30 years + 4 12 –12 7 14 9 3 7 –13 14 12
Humans:
120 years 4 2 1 6 –6 12 –11 7 10 2 –6
Marine mammal:
100 years 5 –10 14 –1 12 –14 8 0 2 14 7
Fish:
80 years + –1 6 5 7 –11 –13 –4 –14 –5 3 15 –6

2A + B = 13 3D + E = 3 2I + 5J = 0
2A – B = –5 5D + E = –3 –2I – 3J = 8

K – 4L = 9 7N + O = –27 5A + 2E = 4
3K + 4L = 11 –3N – O = 3 3A – 2E = –20

7E + 4G = 1 2H – 3G = 21 3A + S = 7
2E – 4G = 26 5H + 3G = –21 3A – 4S = 62

3L – 2M = 22 12P + N = 3 4R + 5S = –1
L + 2M = 34 7P – N = 16 2R – 5S = –83

2R + 4T = –6 5U – 2W = 4 3T + 6W = 9
4R + T = 44 2U + 4W = –56 T + W = –5

Amphibians:
50 years + 4 6 1 2 –9 12 11 3

–7 –10 –2 –6 –8 11 2 –1 –2 10 –2 –9 –3 3 13
Birds:
70 years + 8 –2 –9 –3 3 13 –10 –9 –7 6 –1 9 –2 –8 –14 15 11 –11

140 Maths Quest 10 New South Wales Australian curriculum edition Stages 5.1 and 5.2
number and algebra

 ACTIVITIES
Go to assessON
Chapter opener • WorkSHEET 4.1 (doc‐5220): Simultaneous for questions to
Video equations I (page 122) test your readiness
• The story of mathematics (eles-1843) Interactivity FOR learning, your
• Simultaneous equations (int‐2780) progress AS you learn
4A Graphical solution of simultaneous
(page 117) and your levels OF
linear equations
achievement.
Digital docs (page 112) 4D Applications of simultaneous linear
www.assesson.com.au
• Activity 4‐A‐1 (doc‐4990): ­Investigating equations
graphs of simultaneous equations Digital docs
• Activity 4‐A‐2 (doc‐4991): Graphing • Activity 4‐D‐1 (doc‐4999): Problem solving
­simultaneous equations (page 125)
• Activity 4‐A‐3 (doc‐4992): Further graphing • Activity 4‐D‐2 (doc‐5000): Harder problem
of simultaneous equations solving (page 125)
• SkillSHEET (doc‐5212): Substitution into a • Activity 4‐D‐3 (doc‐5001): Tricky problem
linear rule solving (page 125)
• SkillSHEET (doc‐5213): Solving linear • WorkSHEET 4.2 (doc‐13715): Simultaneous
equations that arise when finding x- and equations II (page 126)
y-intercepts 4E Solving linear inequalities
• SkillSHEET (doc‐5214): Transposing linear Digital docs
equations to standard form • Activity 4‐E‐1 (doc‐5002): Puzzling
• SkillSHEET (doc‐5215): Measuring the rise ­inequations 1 (page 130)
and the run • Activity 4‐E‐2 (doc‐5003): Puzzling
• SkillSHEET (doc‐5216): Finding the gradient ­inequations 2 (page 130)
given two points • Activity 4‐E‐3 (doc‐5004): Puzzling
• SkillSHEET (doc‐5217): Graphing linear ­inequations 3 (page 130)
equations using the x- and y-intercept • SkillSHEET (doc‐5218): Checking whether a
method given point makes the inequation a true
4B Solving simultaneous linear equations statement (page 131)
using substitution • SkillSHEET (doc‐5219): Writing equations
Digital docs (page 116) from worded statements (page 131)
• Activity 4‐B‐1 (doc‐4993): Learning Chapter review
­substitution (page 116) Interactivities (page 133)
• Activity 4‐B‐2 (doc‐4994): Practising • Word search (int‐2835)
­substitution (page 116) • Crossword (int‐2836)
• Activity 4‐B‐3 (doc‐4995): Tricky substitution • Sudoku (int‐3591)
4C Solving simultaneous linear equations Digital docs (page 133)
using elimination • Chapter ­summary (doc‐13717)
Digital docs • Concept map (doc‐13718)
• Activity 4‐C‐1 (doc‐4996): Elimination
To access eBookPLUS activities, log on to
­practice (page 120)
www.jacplus.com.au
• Activity 4‐C‐2 (doc‐4997): Let’s eliminate
(page 120)
• Activity 4‐C‐3 (doc‐4998): More elimination
(page 120)

Chapter 4 • Simultaneous ­linear equations and inequalities 141


number and algebra

Answers
CHAPTER 4 Simultaneous linear equations and inequalities
Exercise 4A — Graphical solution of simultaneous 5    a (5, 2) b (3, 3) c (−2, 6)
­l inear equations d (5, −1) e (7, 0) f (3, 1)
1    a (2, 1) b (1, 1) c (0, 4) 6    a (1, 3) b (4, 0) c (−3, 5)
d (2, −1) e (−2, −4) f (−0.5, 1.5) d (4, 3) e (8, 5) f Q 13, −13 R
2    a No b Yes c Yes d No 7 Ann 61 kg, Beth 58 kg, Celine 54 kg
e Yes f No g No h Yes
i No j Yes Challenge 4.1
3    a (3, 2) b (4, 3) c ( − 3, 4) d (−2, 2) z=6
e (2, 0) f (3, 0) g (−2, 4) h (3, 8) Exercise 4D — Applications of simultaneous linear equations
1 Maths mark = 97, English mark = 66
i (−12, 112) j (2, 5) k (5, 3) l ( 2, 23) 2 18 nuts, 12 bolts
4    a No solution b (2, −1) 3 8 and 3
c No solution d (1, 9) 4 9 and 7
5 6 and 5
e (3, 1) f No solution
6 Length = 12 m and width = 8 m
g No solution h (2, 1) 7 Lemons cost 55 cents and oranges cost 25 cents.
5 y = 4x − 16 8 Length 60 m and width 20 m
6    a Northern beach 9 Eight 20- cent coins and three 50- cent coins
C = 20 + 12t 120 10 Twelve $1 coins and nine $2 coins
Southern beach 100
D
11 Paddlepops cost $1.20 and a Magnum costs $2.10.
D = 8 + 18t C 12 Cost of the Golden rough = 35 cents and cost of the
b Northern beaches in 80
Redskin = 25 cents
Cost

red, southern beaches 60 13 Fixed costs = $87, cost per person = $23.50
in blue 14 The PE mark is 83 and the Science mark is 71.
40
c Time > 2 hours C = 20 + 12t 15 Mozzarella costs $6.20 and Swiss cheese costs $5.80.
d Time = 2 hours, 20 D = 8 + 18t
Challenge 4.2
cost = $44
0 1 2 3 4 5
1 Rollercoaster = $6, Ferris wheel = $4, Gravitron = $8
Time (hours) 2 89  246
7    a Same line b Intersecting Exercise 4E — Solving linear inequations
c Intersecting d Parallel 1    a x > 2 b a > −1 c y≥7
8    a 1 solution d m≥4 e p<1 f x<7
b No solution (parallel lines) g m≤9 h a≤7 i x>3
c No solution (parallel lines) j m≥2 k q ≥ −4 l a > −8
Exercise 4B — Solving simultaneous linear equations 2    a m > 3 b p≤2 c a<4
using substitution d x≥5 e p > −5 f x ≤ −7
g m ≥ −0.5 h b > −0.5 i m > 18
1    a (2, 3) b (2, −1) c (3, −2)
j x<8 k a ≤ −14 l m ≥ 25
d (7, 6) e (3, 6) f (2, 1)
g (−1, −2) h (−4, 0) 3    a m < 4.5 b x≥3 c p>4
d n≤2 e b<5 f y>2
2    a (−6, −23) b (5, 23) c (2, −6)
g m ≤ −1 h a ≥ −5 i b < −413
d Q 32, −15 R e (1, −7) f (−12, −4) j c ≤ −1 k p > −2 l a ≥ −7
2
4    a m > 3 b a≥2 c a < −1
3 26 chickens d a ≤ −3 e x>6 f x≤2
4 a = − 1, b = 5 g b<4 h a>5 i m<2
5 z = 24, m = 6, n = 9
j m≤3 k b ≤ −1611
l m≥1
m 11 7
5    a x ≤ 7 b x ≥ −18 c x < −10
9 8 7
n 5 10 d x> 1012 e x≥5 f x < −145
Exercise 4C — Solving simultaneous linear equations 6    a m < −2 b p ≥ −3 c a≤5
using elimination d p ≥ −5 e y ≤ −3 f x>7
1    a (3, 1) b (−2, 3) c (−2, 6) g p<0 h a ≥ 15 i x > −3
2    a (5, −1) b (2, 3) c (−3, 1) j a ≤ −11 k b≤3 l x < −3
3    a (6, 3) b (−3, −7) c (2, −5) m k>8 n x > −1812 o a ≤ 40
d (−3, 5) e (−5, −8) f (2, −2) 7 B
4    a (2, 1) b (3, 5) c (3, 3) 8    a x < −1 b m ≤ −3 c x > 17
d (1, 3) e (2, 4) f (5, 2) 5
d a> e m≥ 113 f m ≥ −12
g (4, 2) h (−3, 4) i Q −3, −112 R 8

142 Maths Quest 10 New South Wales Australian curriculum edition Stages 5.1 and 5.2
number and algebra

9    a k > 2 b a > −5 c m ≤ 123 Communicating — Rich task


1 A1 Rentals: C = $35 × 3 + 0.28d
d x>5 e y≥7 f d < −2
Cut Price Rentals: C = $28 × 3 + 0.3d
−6
g p≥ 7
h x ≥ −5 i m < −2 2 Comparison of cost of hiring a car from
A1 Rentals and Cut Price Rentals
j a<9 k p≥3 l x > −412 C
600
10    a 5x > 10 b x−3≤5 c 7 + 3x < 42 500

Cost ($)
400 C = 105 + 0.28 d
11    a x ≥ −4 b x>8 300 (A1 Rentals)
200 C = 84 + 0.30 d
100 (Cut Price Rentals)
–5 –4 –3 –2 –1 0 x 5 6 7 8 9 10 x 0
200 400 600 800 1000 1200 1400 1600 d
−c−b −d−b
12    a –6.5 < x < –2 b <x< Distance travelled (km)
a a
3 1050 km
13    a S > 47 b No 4 1400 km
c Answers will vary. 5    a A1 Rentals: C = $37 × 3 + 0.28d
14    a A = 1 15 + 2x 2 1 7 + 2x 2 Cut Price Rentals: C = $30 × 3 + 0.3d
b     i 1 15 + 2x 2 1 7 + 2x 2 < 165 b A1 Rentals: C = $37 × 4 + 0.28d
ii x < 1.23 m Cut Price Rentals: C = $30 × 4 + 0.3d
Chapter review 6 Comparison of cost of hiring a car from
A1 Rentals and Cut Price Rentals
Fluency C Cut Price Rentals (4-day hire)
600
1 D 500

Cost ($)
2 C 400
A1 Rentals A1 Rentals
3 E 300 (4-day hire) (3-day hire)
200
4 C 100 Cut Price Rentals
5 D (3-day hire)
0
6 A 200 400 600 800 1000 1200 1400 1600 d

7    a y = −x + 8 b y = −32x + 12 Distance travelled (km)


7 The extra cost of $2 per day for both rental companies has
c y = 25x + 27
5 not affected the charges they make for the distances travelled.
8    a (3, 1) b (2, 3) However, the overall costs have increased.
8 Presentation of the answers will vary. Answers will include:
9    a No b Yes
Travelling 3 days this month:
10    a (−2, 1) b (0, −2) c (5, 2) •• If Jim travels 1050 km, the cost will be the same for both
11    a (2, 7) b (−5, −3) c (−2, 2) rental companies; that is, $399.
7 7 5 •• If he travels less than 1050 km, Cut Price Rentals is cheaper.
d Q− , R e (−14, −53) f Q , −7 R
3 3 2 •• If he travels more than 1050 km, A1 Rentals is cheaper.
12    a (5, 2) b (−2, 3) c (−3, −1) Travelling 4 days this month:
d (1, 3) e (2, −2) f (4, 2) •• If Jim travels 1400 km, the cost will be the same for both
13    a (0, 3) b (−3, −3) c (2, 1) rental companies; that is, $532.
•• If he travels less than 1400 km, Cut Price Rentals is cheaper.
Problem solving •• If he travels more than 1400 km, A1 Rentals is cheaper.
1    a Numbers are 9 and 14. Travelling 3 days next month:
b Length = 11 metres, width = 6 metres •• If Jim travels 1050 km, the cost will be the same for both
c Chupa‐chups cost 45 cents and Whizz fizzes cost 55 cents. rental companies; that is, $405.
2 Milk $1.75, bread $2.35 •• If he travels less than 1050 km, Cut Price Rentals is cheaper.
3 13 kangaroos and 8 cockatoos •• If he travels more than 1050 km, A1 Rentals is cheaper.
4    a d = b + 10 Travelling 4 days next month:
b 7000 = 70b + 40d •• If Jim travels 1400 km, the cost will be the same for both
c b = 60 and d = 70 rental companies; that is, $540.
d Number of seats in ‘Bleachers’ is 4200; the number of seats in •• If he travels less than 1400 km, Cut Price Rentals is cheaper.
the ‘Dress circle’ is 2800. •• If he travels more than 1400 km, A1 Rentals is cheaper.
e $644 000
5    a CG = 114 + 0.20k Code puzzle
b CS = 90 + 0.32k Insects: Jewel beetle
c 200 km Humans: Japanese men
d k < 200 Marine mammal: Killer whale
6    a 5400 + 260d = CH Fish: Lake sturgeon
b 61 days
7    a n < 16 800 km
b Mick travelled less than 16 800 km for the year and his costs
stayed below $16 000.

Chapter 4 • Simultaneous ­linear equations and inequalities 143


measurement AND geometry

chapter 5

Trigonometry
WHY LEARN THIS?
Nearly 2000 years ago, Ptolemy of Alexandria published
the first book of trigonometric tables, which he used to
chart the heavens and plot the courses of the moon,
stars, and planets. He also created geographical charts
and provided instructions on how to create maps.
Trigonometry is the branch of mathematics that makes
the whole universe more easily understood.

WHAT DO YOU KNOW?


1 Think List what you know about trigonometry.
Use a thinking tool such as a concept map to
show your list.
2 pair Share what you know with a partner
and then with a small group.
3 share As a class, create a large concept map that
shows your class’s knowledge of trigonometry.

LEARNING SEQUENCE
5A Pythagoras’ theorem
5B Pythagoras’ theorem in three dimensions
5C Trigonometric ratios
5D Using trigonometry to calculate side lengths
5E Using trigonometry to calculate angle size
5F Angles of elevation and depression
5G Bearings
5H Applications

144 Maths Quest 10 New South Wales Australian curriculum edition Stages 5.1 and 5.2
Watch this video
The story of mathematics

Searchlight ID: eles-1844

Chapter 5 • Trigonometry 145


measurement AND geometry

5A  Pythagoras’ theorem


Similar right‐angled triangles
In the two similar right‐angled triangles shown below, the angles are the same and the
corresponding sides are in the same ratio.
D

A 6 cm 10 cm

3 cm 5 cm

B 4 cm C E 8 cm F

The corresponding sides are in the same ratio.


AB AC BC
= = .
DE DF EF
To write this using the side lengths of the triangles gives:
AB 3 1
= =
DE 6 2
AC 5 1
= =
DF 10 2
BC 4 1
= =
EF 8 2
This means that for right‐angled triangles, when the angles are fixed, the ratios of the
sides in the triangle are constant.
We can examine this idea further by completing the following activity.
Using a protractor and ruler, draw an angle of 70°, measuring horizontal distances of
3 cm, 7 cm and 10 cm as demonstrated in the diagram below.

c
b

a
70°

3 cm
7 cm
10 cm

Note: Diagram not drawn to scale.


Measure the perpendicular heights a, b and c.
a ≈ 8.24 cm b ≈ 19.23 cm c ≈ 27.47 cm

146 Maths Quest 10 New South Wales Australian curriculum edition Stages 5.1 and 5.2
measurement AND geometry

To test if the theory that for right‐angled triangles, when the angles are fixed, the
ratios of the sides in the triangle are constant is correct, calculate the ratios of the side
lengths.
a 8.24 b 19.23 c 27.47
≈ ≈ 2.75 ≈ ≈ 2.75 ≈ ≈ 2.75
3 3 7 7 10 10
The ratios are the same because the triangles are similar.
This important concept forms the basis of trigonometry.

Review of Pythagoras’ theorem


•• The hypotenuse is the longest side of a right‐angled triangle and
c
is always the side that is opposite the right angle. a
•• Pythagoras’ theorem states that in any right‐angled triangle, the
square of the hypotenuse is equal to the sum of the squares of the b
other two sides. The rule is written as c2 = a2 + b2 where a and b
are the two shorter sides and c is the hypotenuse.
•• Pythagoras’ theorem gives us a way of finding the length of the third side in a
triangle, if we know the lengths of the two other sides.

Finding the hypotenuse


•• To calculate the length of the hypotenuse when given the length of the two shorter
sides, substitute the known values into the formula for Pythagoras’ theorem,
c2 = a2 + b2.

WORKED EXAMPLE 1
For the triangle at right, calculate the length of the
hypotenuse, x, correct to 1 decimal place.

TH I N K W R I TE /D RAW
1 Copy the diagram and label the sides a, b and c.
Remember to label the hypotenuse as c. c=x
a=4

b=7

2 Write Pythagoras’ theorem. c2 = a2 + b2


3 Substitute the values of a, b and c into this rule x2 = 42 + 72
and simplify. = 16 + 49
= 65
4 Calculate x by taking the square root of both sides. x = ±!65
Round the positive answer correct to 1 decimal x ≈ 8.1
place, since x > 0.

Chapter 5 • Trigonometry 147


measurement AND geometry

Finding a shorter side


•• Sometimes a question will give you the length of the hypotenuse and ask you to
find one of the shorter sides. In such examples, we need to rearrange Pythagoras’
­formula. Given that c2 = a2 + b2, we can rewrite this as:
a2 = c2 − b2
or b2 = c2 − a2.

WORKED EXAMPLE 2
Calculate the length, correct to 1 decimal place, of
the unmarked side of the triangle at right.

TH I N K W R I TE /D RAW
1 Copy the diagram and label the sides a, b and c.
a
Remember to label the hypotenuse as c; it does
not matter which side is a and which side is b. c = 14

b=8

2 Write Pythagoras’ theorem. c2 = a2 + b2


3 Substitute the values of a, b and c into this rule 142 = a2 + 82
and solve for a. 196 = a2 + 64
a2 = 196 − 64
= 132
4 Find a by taking the square root of both sides. a = ±!132
Round to 1 decimal place (a > 0). a ≈ 11.5 cm
•• Pythagoras’ theorem can be used to solve many practical problems.
First model the problem by drawing a diagram, then use Pythagoras’ theorem to
solve the right‐angled triangle. Use the result to give a worded answer.
WORKED EXAMPLE 3
A ladder that is 4.5 m long leans up against a vertical wall. The foot of the
ladder is 1.2 m from the wall. How far up the wall does the ladder reach?
Give your answer correct to 1 decimal place.
TH I N K W R I TE /D RAW
1 Draw a diagram and label the sides a, b and c.
Remember to label the hypotenuse as c.

c = 4.5 m
a

b = 1.2 m

148 Maths Quest 10 New South Wales Australian curriculum edition Stages 5.1 and 5.2
measurement AND geometry

2 Write Pythagoras’ theorem.    c2 = a2 + b2


3 Substitute the values of a, b and c into this rule 4.52 = a2 + 1.22
and simplify. 20.25 = a2 + 1.44
a2 = 20.25 − 1.44
= 18.81
4 Find a by taking the square root of both sides. a = ±!18.81
Round to 1 decimal place (a > 0).   
≈ 4.3
5 Answer the question using words. The ladder reaches 4.3 m
up the wall.

WORKED EXAMPLE 4
Determine the unknown side lengths of the triangle
at right, correct to 2 decimal places.

TH I N K W R I TE /D RAW
1 Copy the diagram and label the
b = 3x
sides a, b and c.
c = 78 m

a = 2x

2 Write Pythagoras’ theorem.    c2 = a2 + b2


3 Substitute the values of a, b and c into this 782 = (3x) 2 + (2x) 2
rule and simplify. 6084 = 9x2 + 4x2
6084 = 13x2
4 Rearrange the equation so that the pronumeral   13x2 = 6084
is on the left‐hand side of the equation.
5 Divide both sides of the equation by 13. 13x2 6084
=
13
    2 13
x = 468
6 Find x by taking the square root of both sides. x = ±!468
x ≈ 21.6333
7 Substitute the value of x into 2x and 3x to find 2x = 2(21.6333)
the lengths of the unknown sides. ≈ 43.27 m
3x = 3(21.6333)
≈ 64.90 m

Chapter 5 • Trigonometry 149


measurement AND geometry

Exercise 5A Pythagoras’ theorem


INDIVIDUAL PATHWAYS
Questions: Questions: Questions:
1–4, 6, 12, 14, 15, 17 1–3, 5–8, 12, 15, 18, 21, 23 1, 2, 5, 7, 9–11, 13, 16, 19, 20,
22, 24
Activity 5‐A‐1 Activity 5‐A‐2 Activity 5‐A‐3
Review of Pythagoras’ Practising Pythagoras’ More of Pythagoras’
theorem theorem theorem
doc‐5011 doc‐5012 doc‐5013

FLUENCY
1 WE 1 For each of the following triangles, calculate the length of the hypotenuse,
giving answers correct to 2 decimal places.
a b 19.3 c
4.7

804
6.3 27.1
562

d e 0.9 f 152

7.4
87

10.3
2.7

2 WE2 Find the value of the pronumeral, correct to 2 decimal places.


a s b c
Digital doc
1.98 u
SkillSHEET 8.4
Rounding to a given 30.1
number of decimal 47.2
places
2.56 17.52
doc‐5224
t

0.28
d e 2870 f
v
468
1920 x
0.67 w

114

3 WE3 The diagonal of the rectangular sign at right is


34 cm. If the height of this sign is 25 cm, find the
width.
4 A right‐angled triangle has a base of 4 cm and a height
of 12 cm. Calculate the length of the hypotenuse to
2 decimal places.

150 Maths Quest 10 New South Wales Australian curriculum edition Stages 5.1 and 5.2
measurement AND geometry

5 Calculate the lengths of the diagonals (to 2 decimal places) of squares that have
side lengths of:
a 10 cm b 17 cm c 3.2 cm.
6 The diagonal of a rectangle is 120 cm. One side has a length of 70 cm. Determine:
a the length of the other side
b the perimeter of the rectangle
c the area of the rectangle.
7 WE4 Find the value of the pronumeral, correct to 2 decimal places for each of the
following.
a b c 2x
3x 3x
25
4x 6x
18 30
x

UNDERSTANDING
8 An isosceles triangle has a base of 30 cm and a height of 10 cm. Calculate the
length of the two equal sides.
9 An equilateral triangle has sides of length 20 cm. Find the height of the triangle.
10 A right‐angled triangle has a height of 17.2 cm, and a base that is half the height.
Calculate the length of the hypotenuse, correct to 2 decimal places.
11 The road sign shown below is based on an equilateral triangle. Find the height of
the sign and, hence, find its area.

76 cm

12 A flagpole, 12 m high, is supported by three wires, attached from the top of the
pole to the ground. Each wire is pegged into the ground 5 m from the pole. How
much wire is needed to support the pole?
13 Ben’s dog ‘Macca’ has wandered onto a frozen pond and is too frightened to walk
back. Ben estimates that the dog is 3.5 m from the edge of the pond. He finds a
plank, 4 m long, and thinks he can use it to rescue Macca. The pond is surrounded
by a bank that is 1 m high. Ben uses the plank to make a ramp
3.8 km
for Macca to walk up. Will the plank reach the dog?
14 Sarah goes canoeing in a large lake. She paddles 2.1 km to
the north, then 3.8 km to the west. Use the triangle at right to 2.1 km
find out how far she must then paddle to get back to her
starting point in the shortest possible way. Starting point

Chapter 5 • Trigonometry 151


measurement AND geometry

15 A baseball diamond is a square of side length 27 m. When a runner on first base


tries to steal second base, the catcher has to throw the ball from home base to
second base. How far is that throw?

Second base
27 m
First
base

Home base
Catcher

16 Penny, a carpenter, is building a roof for a new house. The roof


has a gable end in the form of an isosceles triangle, with a base
of 6 m and sloping sides of 7.5 m. She decides to put 5 evenly 7.5 m 7.5 m
spaced vertical strips of wood as decoration on the gable as
shown at right. How many metres of this decorative wood does
6m
she need?
17 A rectangle measures 35 mm by 4.2 cm. Calculate the length of its diagonal in
­millimetres to 2 decimal places.
18 A rectangular envelope has a length of 21 cm and a diagonal measuring 35 cm.
­Calculate:
a the width of the envelope b the area of the envelope.
19 A sheet of A4 paper measures 210 mm by 297 mm. Calculate the length of the
diagonal in centimetres to 2 decimal places.
20 A right‐angled triangle has a hypotenuse of 47.3 cm and one other side of 30.8 cm.
Calculate the area of the triangle.
21 A swimming pool is 50 m by 25 m. Peter is bored by his usual training routine, and
decides to swim the diagonal of the pool. How many diagonals must he swim to
complete his normal distance of 1200 m? Give your answer to 2 decimal places.
22 Sarah is making a gate that has to be 1200 mm wide. It must be braced with a
diagonal strut made of a different type of timber. She has only 2 m of this kind of
timber available. What is the maximum height of the gate that she can make?
23 A hiker walks 4.5 km west, then 3.8 km south. How far in metres is she from her
starting point? Give your answer to 2 decimal places.
24 A square has a diagonal of 10 cm. What is the length of each side?

152 Maths Quest 10 New South Wales Australian curriculum edition Stages 5.1 and 5.2
measurement AND geometry

REASONING
25 The triangles below are right‐angled triangles. Two possible measurements have been
suggested for the hypotenuse in each case. For each triangle, complete calculations to
determine which of the lengths is correct for the hypotenuse in each case.

a b 185 or 195 c 273


60 or 65
33
175 305 or 308
136
56 60
26 Four possible side length measurements are 105, 208,
230 and 233. Three of them together produce a REFLECTION
The square root of a
right‐angled triangle.
number usually gives
a Which of the measurements could not be the us both a positive and
hypotenuse of the triangle? Explain. negative answer. Why do
b Complete as few calculations as possible to we only take the positive
answer when using
calculate which combination of side lengths will
Pythagoras’ theorem?
produce a right‐angled triangle.

5B Pythagoras’ theorem in three dimensions


•• Many real‐life situations involve 3‐dimensional (3‐D) objects: objects with length,
width and height. Some common 3‐D objects used in this section include cuboids,
pyramids and right‐angled wedges.

Cuboid Pyramid Right-angled wedge

•• In diagrams of 3‐D objects, right angles may not look like right angles, so it is
important to redraw sections of the diagram in two dimensions, where the right
angles can be seen accurately.
WORKED EXAMPLE 5
Determine the length AG in this rectangular prism A B
(cuboid), correct to 2 decimal places. 6 cm
C
D
F
E
5 cm

H 10 cm G
TH I N K W R I TE /D RAW
1 Draw the diagram in three dimensions. A B
Draw the lines AG and EG.
6 cm
∠AEG is a right angle. D
C
F
E
5 cm

H 10 cm G

Chapter 5 • Trigonometry 153


measurement AND geometry

2 Draw Δ AEG, showing the right‐angle. A


Only 1 side is known, so EG must be found.
6

E G

3 Draw EFGH in two dimensions and E F


diagonal EG as x. x
5 5

H 10 G

4 Use Pythagoras’ theorem to calculate x. x2 = 52 + 102


(c2 = a2 + b2). = 25 + 100
= 125
x= !125
5 Place this information on triangle AEG. Label A
the side AG as y. y
6

E G
√125

6 Use Pythagoras’ theorem to find y. y2 = 62 + (!125) 2


(c2 = a2 + b2). = 36 + 125
= 161
y= !161
≈ 12.69

7 Answer the question in a sentence. The length of AG is


12.69 cm .

WORKED EXAMPLE 6
A piece of cheese in the shape of a right‐angled wedge sits on a table. It
has a rectangular base measuring 14 cm by 8 cm, and is 4 cm high at the
thickest point. An ant crawls diagonally across the sloping face. How far, to
the nearest millimetre, does the ant walk?

TH I N K W R I TE /D RAW
1 Draw a diagram in three dimensions and label B C
the vertices. Mark BD, the path taken by the ant, E 4 cm
F
with a dotted line. ∠BED is a right angle. A 14 cm D
8 cm

154 Maths Quest 10 New South Wales Australian curriculum edition Stages 5.1 and 5.2
measurement AND geometry

2 Draw Δ BED, showing the right‐angle. Only one B


side is known. ED must be found.
4

D
E

3 Draw EFDA in two dimensions, and label the E F


diagonal ED as x. x
8 8

A 14 D

4 Use Pythagoras’ theorem to calculate ED. x2 = 82 + 142


(c2 = a2 + b2) = 64 + 196
= 260
x= !260
5 Place this information on triangle BED. B
Label the side BD as y. y
4

D
E √ 260

6 Solve this triangle for y. y2 = 42 + 1 !260 2 2


(c2 = a2 + b2) = 16 + 260
= 276
y = !276
= 16.61 cm
7 Answer the question in a sentence. The ant walks 16.6 cm,
correct to the nearest
centimetre.

Exercise 5B Pythagoras’ theorem in


three dimensions
INDIVIDUAL PATHWAYS

Questions: Questions: Questions:


1a–b, 2, 6, 8, 10a 1, 3, 4, 6, 8, 10, 12 1, 3–5, 7, 9–13

Activity 5‐B‐1 Activity 5‐B‐2 Activity 5‐B‐3


Pythagoras in 3-dimensions Pythagoras in 3-D figures Investigating triangles in
doc‐5014 doc‐5015 3-D figures
doc‐5016

Where appropriate in this exercise, give answers correct to 2 decimal places.

Chapter 5 • Trigonometry 155


measurement AND geometry

FLUENCY
1 WE5 Calculate the length of AG in each of the following figures.
A A B
a A B b B c

C C
C 10 D D 10.4
D

10 E
E F
F
7.3
10
H 8.2 G
H 10 G
E F
5
H 5 G

2 Calculate the length of CE in the wedge at right and, hence, A B


Digital doc obtain AC. E 4
SkillSHEET F
Drawing 3-D shapes D 10 C 7
doc‐5229

3 If DC = 3.2 m, AC = 5.8 m, and CF = 4.5 m in the figure A B


at right, calculate the length of AD and BF.
F
D C

4 Calculate the length of BD and, hence, the height of the V


pyramid at right. 8
A B
8
D
8 C

5 The pyramid ABCDE has a square base. The pyramid E


is 20 cm high. Each sloping edge measures 30 cm. Calculate EM = 20 cm
the length of the sides of the base.
A
B
M
D C
6 The sloping side of a cone is 10 cm and the height is 8 cm.
What is the length of the radius of the base?
8 cm 10 cm

UNDERSTANDING
r
7 An ice‐cream cone has a diameter across the top of 6 cm
and a sloping side of 13 cm. How deep is the cone?

156 Maths Quest 10 New South Wales Australian curriculum edition Stages 5.1 and 5.2
measurement AND geometry

8 WE6 A piece of cheese in the shape of a right‐angled B C


wedge sits on a table. It has a base measuring 20 mm by E 4 mm
F
10 mm, and is 4 mm high at the thickest point, as shown in A D 10 mm
20 mm
the figure. A fly crawls diagonally across the sloping face.
How far, to the nearest millimetre, does the fly walk?
9 Jodie travels to Bolivia, taking with her a suitcase as shown in the photo. She buys
a carved walking stick 1.2 m long. Will she be able to fit it in her suitcase for the
flight home?
30
cm

65 cm

90 cm

10 A 10-m high flagpole is in the corner of a rectangular park


10 m
that measures 240 m by 150 m. 240 m A

a Calculate: 150 m
i the length of the diagonal of the park B
ii the distance from A to the top of the pole
iii the distance from B to the top of the pole.
b A bird flies from the top of the pole to the centre of the park.
How far does it fly?
11 A candlestick is in the shape of two cones, joined at the vertices
as shown. The smaller cone has a diameter and sloping side of 7 cm,
and the larger one has a diameter and sloping side of 10 cm.
How tall is the candlestick?
12 A tent is in the shape of a triangular prism, with a height
of 120 cm as shown at right. The width across the base of 120 cm
the door is 1 m and the tent is 2.3 m long. Digital doc
WorkSHEET 5.1
Calculate the length of each sloping side, in metres. doc‐5230
2.3 m
Then calculate the area of fabric used in the 1m
construction of the sloping rectangles which form
the sides. REFLECTION
The diagonal distance
across a rectangle of
REASONING dimensions x and y is
13 Stephano is renovating his apartment, which is at the "x2 + y2. What would be
end of two corridors. The corridors of the apartment the rule to find the length of
building are 2 m wide with 2 m high ceilings, and the a diagonal across a cuboid
of dimensions x by y by z?
first corridor is at right angles to the second. Show that
Use your rule to check your
he can carry lengths of timber up to 6 m long to his answers to question 1.
apartment.

Chapter 5 • Trigonometry 157


measurement AND geometry

5C  Trigonometric ratios


Naming the sides in a right‐angled triangle
•• In a right‐angled triangle, the longest side is called the hypotenuse.
•• If one of the two acute angles is named (say θ), then the other two sides can also be
given names, as shown in the diagram.

Opposite Hypotenuse

𝜃
Adjacent

Three basic definitions


•• Using the diagram above, the following three trigonometric ratios can be defined:
length of opposite side
–– the sine ratio, sine θ =
length of hypotenuse
length of adjacent side
–– the cosine ratio, cosine θ =
length of hypotenuse
length of opposite side
–– the tangent ratio, tangent θ = .
length of adjacent side
•• The names of the three ratios are usually shortened to sin θ, cos θ and tan θ.
•• The three ratios are often remembered using the mnemonic SOHCAHTOA, where
SOH means Sin θ = opposite over hypotenuse and so on.

Finding values using a calculator


•• The sine, cosine and tangent of an angle have numerical values that can be found
using a calculator.
•• Traditionally angles were measured in degrees, minutes and seconds, where
60 seconds = 1 minute and
60 minutes = 1 degree.
For example, 50°33′48″ means 50 degrees, 33 minutes and 48 seconds.

WORKED EXAMPLE 7
Calculate the value of each of the following, correct to 4 decimal places,
using a calculator. (Remember to initially work to 5 decimal places before
rounding.)
a cos 65°57′ b tan 56°45′30″
TH I N K Write
a Write your answer to the required a cos 65°57′ ≈ 0.40753
number of decimal places. = 0.4075
b Write your answer to the required b tan 56°45′30″ ≈ 1.52573
number of decimal places. ≈ 1.5257

158 Maths Quest 10 New South Wales Australian curriculum edition Stages 5.1 and 5.2
measurement AND geometry

WORKED EXAMPLE 8
Calculate the size of angle θ, correct to the nearest degree, given
sin θ = 0.6583.

TH I N K Write
1 Write the given equation. sin θ = 0.6583
2 To find the size of the angle, we need θ = sin−1 (0.6583)
sin−1.
to undo sine with its inverse, ≈ 41.1°
(Ensure your calculator is in degrees mode.)
3 Write your answer to the nearest degree. θ ≈ 41°

WORKED EXAMPLE 9
Calculate the value of θ:
a correct to the nearest minute, given that cos θ = 0.2547
b correct to the nearest second, given that tan θ = 2.364.

TH I N K Write
a 1 Write the equation. a cos θ = 0.2547
2 Write your answer, including seconds. cos−1 0.2547 = 75°14′39″
Round to the nearest minute. (Remember ≈ 75°15′
60ʺ = 1ʹ, so 14ʺ39ʹ rounds up to 15ʺ.)
b 1 Write the equation. b tan θ = 2.364
2 Write your answer, rounding to the tan−1 2.364 ≈ 67°4′16″
nearest second.

WORKED EXAMPLE 10
For the triangle shown, write the rules for the sine, cosine and tangent
ratios of the given angle.

c
a

𝜃
b

TH I N K W R I TE /D RAW
1 Label the diagram using the symbols O, A, H
with respect to the given angle (angle θ).
c=H
a=O

𝜃
b=A

Chapter 5 • Trigonometry 159


measurement AND geometry

2 From the diagram, identify the values of O = a, A = b, H = c


O (opposite side), A (adjacent side) and
H (the hypotenuse).
3 Write the rule for each of the sine, cosine O A O
sin θ = , cos θ = , tan θ =
and tangent ratios. H H A
4 Substitute the values of A, O and H into a b a
sin θ = , cos θ = , tan θ =
each rule. c c b

WORKED EXAMPLE 11
Write the equation that relates the two marked sides and the marked
angle.
a b
18
15
6 x
50°
b

TH I N K W R I TE /D RAW
a 1 Label the given sides of the triangle. a
15 = H
6=O

O
2 Write the ratio that contains O and H. sin θ =
H
3 Identify the values of the pronumerals. O = 6, H = 15
6 2
4 Substitute the values of the pronumerals sin b = =
into the ratio. (Since the given angle is 15 5
denoted with the letter b, replace θ
with b.)

b 1 Label the given sides of the triangle. b 18 = A


x=O
50°

O
2 Write the ratio that contains O and A. tan θ =
A
3 Identify the values of the pronumerals. O = x, A = 18, θ = 50°
x
4 Substitute the values of the pronumerals tan 50° =
18
into the ratio.

160 Maths Quest 10 New South Wales Australian curriculum edition Stages 5.1 and 5.2
measurement AND geometry

Exercise 5C Trigonometric ratios


INDIVIDUAL PATHWAYS
Questions: Questions: Questions:
1, 3, 6a–f, 7, 8 2, 3, 4, 6a–f, 7, 8, 9 2, 3, 4, 5, 6c–l, 7, 8, 9
Activity 5‐C‐1 Activity 5‐C‐2 Activity 5‐C‐3
Review of trigonometry Using trigonometry Applying trigonometry
doc‐5017 doc‐5018 doc‐5019

FLUENCY
1 Calculate each of the following, correct to 4 decimal places.
a sin 30° b cos 45° c tan 25°
d sin 57° e tan 83° f cos 44°
2 WE7 Calculate each of the following, correct to 4 decimal places.
a sin 40°30′ b cos 53°57′ c tan 27°34′
d tan 123°40′ e sin 92°32′ f sin 42°8′
g cos 35°42′35″ h tan 27°42′50″ i cos 143°25′23″
3 WE8 Find the size of angle θ , correct to the nearest degree.
a sin θ = 0.763 b cos θ = 0.912 c tan θ = 1.351
d cos θ = 0.321 e tan θ = 12.86 f cos θ = 0.756
4 WE9a Find the size of the angle θ, correct to the nearest minute.
a sin θ = 0.814 b sin θ = 0.110 c tan θ = 0.015
d cos θ = 0.296 e tan θ = 0.993 f sin θ = 0.450
5 WE9b Find the size of the angle θ , correct to the nearest second.
a tan θ = 0.5 b cos θ = 0.438 c sin θ = 0.9047
d tan θ = 1.1141 e cos θ = 0.8 f tan θ = 43.76
6 Find the value of each expression, correct to 3 decimal places.
a 3.8 cos 42° b 118 sin 37° c 2.5 tan 83°
2 220 2 cos 23°
d e f
sin 45° cos 14° 5 sin 18°
12.8 18.7 55.7
g h i
tan 60°32′ sin 35°25′42″ cos 89°21′
3.8 tan 1°51′44″ 2.5 sin 27°8′ 3.2 cos 34°52′
j k l
4.5 sin 25°45′ 10.4 cos 83°2′ 0.8 sin 12°48′
7 For each labelled angle in the following triangles, write an expression for:
WE10 Digital docs
SkillSHEET
i sine ii cosine iii tangent. Labeling the sides of a
right-angled triangle
a d b c doc‐5226
θ i h β SkillSHEET
k Selecting an
α j appropriate
f e g trigonometric ratio
based on the given
l information
doc‐5231

Chapter 5 • Trigonometry 161


measurement AND geometry

d o e f
a b
γ
n
β
c u
m v

γ
t

8 WE11 Write the equation that relates the two marked sides and the marked angle.
a b 25
c 5
θ
θ
15 4
12 30

d 2.7 e f 14.3
p 17
t
θ
35° 17.5
α

g 7 h θ i
20 31 9.8

α
3.1
x

15°

REASONING
9 Consider the right‐angled triangle shown at right.
α
a Label each of the sides using the letters O, A, H with
respect to the 41° angle.
b Measure the side lengths (to the nearest millimetre).
c Determine the value of each trigonometric ratio.
(Where applicable, answers should be given correct to
41°
2 decimal places.)
i sin 41°
ii cos 41°
iii tan 41°
d What is the value of the unknown angle, α?

162 Maths Quest 10 New South Wales Australian curriculum edition Stages 5.1 and 5.2
measurement AND geometry

e Determine the value of each of these trigonometric ratios, correct to 2 decimal


places.
i sin α ii cos α iii tan α
(Hint: First relabel the sides of the triangle with
respect to angle α.)
f What do you notice about the relationship between REFLECTION
sin 41° and cos α? How do we determine
g What do you notice about the relationship between which of sin, cos or tan
to use in a trigonometry
sin α and cos 41°?
question?
h Make a general statement about the two angles.

5D Using trigonometry to calculate


side lengths Interactivity
Using trigonometry
In a right‐angled triangle, if one side length and one acute angle are known, the length int‐1146
of another side can be found by applying trigonometric ratios.

WORKED EXAMPLE 12
Find the value of each pronumeral giving answers correct to 3 decimal
places.
a b
6 cm 32°
a
0.346 cm f
35°

TH I N K W R I TE /D RAW
a 1 Label the marked sides of the a
triangle. H O
6 cm
a

35°

O
2 Identify the appropriate sin θ =
trigonometric ratio to use. H
a
3 Substitute O = a, H = 6, θ = 35°. sin 35° =
6
4 Make a the subject of the equation. 6 sin 35° = a
a = 6 sin 35°
5 Calculate and round the answer, a ≈ 3.441 cm
correct to 3 decimal places.

Chapter 5 • Trigonometry 163


measurement AND geometry

b 1 Label the marked sides of the b


triangle.
32°
H A
0.346 cm f

2 Identify the appropriate A


cos θ =
trigonometric ratio to use. H
f
3 Substitute A = f, H = 0.346 and cos 32° =
θ = 32°. 0.346
4 Make f the subject of the equation. 0.346 cos 32° = f
f = 0.346 cos 32°
5 Calculate and round the answer, ≈ 0.293 cm
correct to 3 decimal places.

WORKED EXAMPLE 13
Find the value of the pronumeral in the triangle shown. Give the answer
correct to 2 decimal places.
120 m

P

TH I N K W R I TE /D RAW
1 Label the marked sides of the triangle. H O
120 m

A P

O
2 Identify the appropriate trigonometric tan θ =
ratio to use. A

120
3 Substitute O = 120, A = P and tan 5° =
θ = 5°. P

4 Make P the subject of the equation. P × tan 5° = 120


   (i) Multiply both sides of the 120
P =
equation by P. tan 5°
(ii) Divide both sides of the equation
by tan 5°.
5 Calculate and round the answer, P ≈ 1371.61 m
correct to 2 decimal places.

164 Maths Quest 10 New South Wales Australian curriculum edition Stages 5.1 and 5.2
measurement AND geometry

Exercise 5D Using trigonometry to calculate side


lengths
INDIVIDUAL PATHWAYS
Questions: Questions: Questions:
1, 2, 3a–b, 4, 5 1, 2, 3c–d, 4–7 1, 2, 3e–f, 4–7
Activity 5‐D‐1 Activity 5‐D‐2 Activity 5‐D‐3
Calculating side lengths Applying trigonometry to Practical applications
using trigonometry simple figures of trigonometry
doc‐5020 doc‐5021 doc‐5022

FLUENCY
1 WE 12 Find the length of the unknown side in each of the following, correct to
3 decimal places.
a b c
8
x
10 cm a 25°
a 31°
14

60°

2 WE 13 Find the length of the unknown side in each of the following triangles,
correct to 2 decimal places.
a 71° b 4.6 m
13°
m
n

2.3 m

c
94 mm

68°
t

3 Find the length of the unknown side in each of the following, correct to 2 decimal
places.

a b 8°5 P c 14 m
2'4
5''

11.7 m
43.95 m
t
12'
18°
40°26'
x

Chapter 5 • Trigonometry 165


measurement AND geometry

d e f 6°25'

x
x 80.9 cm
x

75.23 km

21°25'34"
2'
°4 11.2 mm
34

4 Find the value of the pronumeral in each of the following, correct to 2 decimal
places.
a x b 23.7 m
36°42'

y
43.9 cm
46°

c d
34

z p
°1
2'

12.3 m 15.3 m

13°12'

e f 0.732 km
q
p
73°5' a
63°11'
47.385 km b

UNDERSTANDING
5 Given that the angle θ is 42° and the length of the hypotenuse is 8.95 m in a
right‐angled triangle, find the length of:
a the opposite side
b the adjacent side.
Give each answer correct to 1 decimal point.
6 A ladder rests against a wall. If the angle between the
ladder and the ground is 35° and the foot of the ladder
is 1.5 m from the wall, how high up the wall does the REFLECTION
ladder reach? How does solving a
trigonometric equation
REASONING differ when we are
Under what circumstances will the opposite side and
7 a  finding the length of the
the adjacent side have the same length? hypotenuse side compared
to when finding the length
b For what values of θ° (the reference angle) will the of a shorter side?
adjacent side be larger than the opposite side?

166 Maths Quest 10 New South Wales Australian curriculum edition Stages 5.1 and 5.2
measurement AND geometry

5E  Using trigonometry to calculate


angle size
•• Just as inverse operations are used to solve equations, inverse trigonometric ratios are
used to solve trigonometric equations for the value of the angle.
–– Inverse sine (sin−1) is the inverse of sine.
–– Inverse cosine (cos−1) is the inverse of cosine.
–– Inverse tangent (tan−1) is the inverse of tangent.
For example, since sin(30°) = 0.5, then sin−1 (0.5) = 30°; this is read as ‘inverse sine
of 0.5 is 30 degrees’.

If sin(θ) = a, then sin−1 (a) = θ .


If cos(θ ) = a, then cos−1 (a) = θ .
If tan(θ) = a, then tan−1 (a) = θ .

•• A calculator can be used to calculate the values of inverse trigonometric ratios.


•• The size of any angle in a right‐angled triangle can be found if:
–– the lengths of any two sides are known
–– an appropriate trigonometric ratio is identified from the given lengths
–– a calculator is used to evaluate the inverse trigonometric ratio.

WORKED EXAMPLE 14
For each of the following, find the size of the angle, θ, correct to the
nearest degree.
a b

5m

5 cm 𝜃
3.5 cm
11 m

TH I N K W R I TE /D RAW
a 1 Label the given sides of the triangle. a

H O
5 cm
3.5 cm

O
2 Identify the appropriate trigonometric sin θ =
ratio given O and H. H

3.5
3 Substitute O = 3.5 and H = 5 and sin θ =
evaluate the expression. 5
= 0.7

Chapter 5 • Trigonometry 167


measurement AND geometry

4 Make θ the subject of the equation using θ = sin−1 (0.7)


inverse sine. = 44.427 004°
5 Evaluate θ and round the answer, correct θ = 44°
to the nearest degree.
b 1 Label the given sides of the triangle. b
O
5m

𝜃
11 m A
O
2 Identify the appropriate trigonometric tan θ =
ratio given O and A. A
5
3 Substitute O = 5 and A = 11. tan θ =
11
5
4 Make θ the subject of the equation using θ = tan −1 a b
inverse tangent. 11
= 24.443 954 78°
5 Evaluate θ and round the answer, correct θ ≈ 24°
to the nearest degree.

WORKED EXAMPLE 15
Find the size of angle θ: 3.1 m
a Correct to the nearest second. 𝜃
b Correct to the nearest minute.
7.2 m
.

TH I N K W R I TE /D RAW
a 1 Label the given sides of the triangle. a 3.1 m A
𝜃

O
7.2 m

O
2 Identify the appropriate trigonometric tan θ =
ratio to use. A
7.2
3 Substitute O = 7.2 and A = 3.1 and tan θ =
evaluate the expression. 3.1
7.2
4 Make θ the subject of the equation using θ = tan−1 a b
inverse tangent. 3.1

168 Maths Quest 10 New South Wales Australian curriculum edition Stages 5.1 and 5.2
measurement AND geometry

5 Evaluate θ and write the calculator θ = 66.705 436 75°


display.
6 Use the calculator to convert the answer = 66°42′19.572″
to degrees, minutes and seconds.
7 Round the answer to the nearest second. θ ≈ 66°42′20″
b Round the answer to the nearest minute. θ ≈ 66°42′

Exercise 5E Using trigonometry


to calculate angle size
INDIVIDUAL PATHWAYS
Questions: Questions: Questions:
1, 3–6, 8 1–6, 8 1–8
Activity 5‐E‐1 Activity 5‐E‐2 Activity 5‐E‐3
Review of angle calculations Calculation angles Applying trigonometry to
doc‐5023 using trigonometry angle calculations
doc‐5024 doc‐5025

FLUENCY
1 WE 14 Find the size of the angle, θ , in each of the following. Give your answer
correct to the nearest degree.
a b c

5.2 4.7
4.8 8

θ θ
θ
3.2
3
2 WE 15 Find the size of the angle marked with the pronumeral in each of the
following. Give your answer correct to the nearest minute.
a b β 7.2 m c
12

17
4m θ
10
θ
12
3 Find the size of the angle marked with the pronumeral in each of the following. Give
your answer correct to the nearest second.
Digital doc
a b α c SkillSHEET
5m Rounding angles to the
2.7 nearest degree
8 doc‐5232
α
3m 3.5
θ
2

Chapter 5 • Trigonometry 169


measurement AND geometry

4 Find the size of the angle marked with the pronumeral in each of the following,
giving your answer correct to the nearest degree.
a b

13.5 a 89.4
15.3
c
77.3

c 106.4 d d
43.7
18.7
92.7
b

e f
12.36
13.85 7.3 cm 12.2 cm
e

18.56

9.8 cm α

5 Find the size of each of the angles in the following, giving your answers correct to
the nearest minute.
a     b    c
x
d
a 5.7
0.798 2.3
56.3 y

0.342 e

b
27.2

UNDERSTANDING
Calculate the length of the sides r, l and h. Write
6 a  A
your answers correct to 2 decimal places. r
h l
b Calculate the area of ABC, correct to the nearest 125°
square centimetre. D
20 cm B 30 cm C
c Calculate ∠BCA.
7 In the sport of air racing, small planes have to travel between two large towers
(or pylons). The gap between a pair of pylons is smaller than the wing‐span of the
plane, so the plane has to go through on an angle with one wing ‘above’ the other.
The wing‐span of a competition airplane is 8 metres.

170 Maths Quest 10 New South Wales Australian curriculum edition Stages 5.1 and 5.2
measurement AND geometry

a Determine the angle, correct to 1 decimal place, that the plane has to tilt if the gap
between pylons is:
i 7 metres
ii 6 metres
iii 5 metres.
b Because the plane has rolled away from the horizontal as it travels between
the pylons it loses speed. If the plane’s speed is below 96 km/h it will
stall and possibly crash. For each degree of ‘tilt’ the speed of the plane
is reduced by 0.98 km/h. What is the minimum speed the plane must go
through each of the pylons in part a? Write your answer correct to 2 decimal
places.
REASONING
8 There are two important triangles commonly used in trigonometry.
Complete the following steps and answer the questions to create these
triangles.
Triangle 1
–– Sketch an equilateral triangle with side length 2 units.
–– Calculate the size of the internal angles.
–– Bisect the triangle to form two right‐angled triangles.
–– Redraw one of the triangles formed.
–– Calculate the side lengths of this right‐angled triangle as exact values.
–– Fully label your diagram showing all side lengths and angles.
Triangle 2
–– Draw a right‐angled isosceles triangle.
–– Calculate the sizes of the internal angles.
–– Let the sides of equal length be 1 unit long.
–– Calculate the length of the third side.
–– Fully label your diagram showing all side lengths and angles.

Chapter 5 • Trigonometry 171


measurement AND geometry

Use the triangles formed in Q8 to calculate exact values for sin(30°), cos(30°)
9 a 
and tan(30°).
b Use the exact values for sin(30°), cos(30°) and
sin(30°) REFLECTION
tan(30°) to show that tan(30°) = . How is finding the angle
cos(30°)
of a right‐angled triangle
o a
c Use the formulae sin(θ ) = and cos(θ) = to different to finding a side
Digital doc h h length?
WorkSHEET 5.2 sin(θ)
doc‐5233 prove that tan(θ ) = .
cos(θ )

CHALLENGE 5.1

5F Angles of elevation and depression


•• Consider the points A and B, where B is at a higher elevation than A.

eLesson B
Height of a satellite
eles‐0173

θ = angle of elevation of B
from A

A θ
Horizontal

If a horizontal line is drawn from A as shown, forming the angle θ , then θ is called
the angle of elevation of B from A.
•• If a horizontal line is drawn from B, forming the angle α, then α is called the angle of
depression of A from B.

Horizontal
B
α

α = angle of depression of A
from B
A

172 Maths Quest 10 New South Wales Australian curriculum edition Stages 5.1 and 5.2
measurement AND geometry

•• Because the horizontal lines are parallel, θ and α have the same size (alternate angles).
α B

θ=α

θ
A

WORKED EXAMPLE 16
From a point P on the ground, the angle of elevation of the top of a tree is
50°. If P is 8 metres from the tree, find the height of the tree.

TH I N K W R I TE /D RAW
1 Sketch a diagram and label the sides of the
triangle with respect to the given angle. Let
the height of the tree be h. h
O

50°
P
8m A

O
2 Identify the appropriate trigonometric ratio. tan θ =
A
h
3 Substitute O = h, A = 8 and θ = 50°. tan 50° =
8
4 Rearrange to make h the subject. h = 8 tan 50°
5 Calculate and round the answer to 2 decimal ≈ 9.53
places.
6 Give a worded answer. The height of the tree is 9.53 m.

Exercise 5F Angles of elevation and depression


INDIVIDUAL PATHWAYS
Questions: Questions: Questions:
1–3, 8 1–6, 9, 10, 14 1–7, 9, 11–14
Activity 5‐F‐1 Activity 5‐F‐2 Activity 5‐F‐3
Identifying elevation and Calculating elevation and Applications of elevation and
depression depression depression
doc‐5026 doc‐5027 doc‐5028

FLUENCY
1 WE 16 The angle of elevation from a point P on the ground to the top of a tree is
54°22′. If the tree is known to be 12.19 m high, how far is P from the tree (measured
horizontally)?
2 From the top of a cliff 112 m high, the angle of depression to a boat is 9°15′. How
far is the boat from the foot of the cliff?

Chapter 5 • Trigonometry 173


measurement AND geometry

3 A person on a ship observes a lighthouse on the cliff, which is 830 metres


away from the ship. The angle of elevation of the top of the lighthouse
is 12°.
a How far above sea level is the top of the lighthouse?
b If the height of the lighthouse is 24 m, how high is the cliff?
4 At a certain time of the day a post, 4 m tall, casts a shadow of 1.8 m. What is the
angle of elevation of the sun at that time?
5 An observer, who is standing 47 m from a building, measures the angle of elevation
of the top of the building as 17°. If the observer’s eye is 167 cm from the ground,
what is the height of the building?

UNDERSTANDING
6 A surveyor needs to determine the height of a building. She measures the angle of
elevation of the top of the building from two points, 38 m apart. The surveyor’s eye
Digital doc
SkillSHEET
level is 180 cm above the ground.
Drawing a diagram
from given directions
doc‐5228

47°12 35°5
' 0'
x 38 m 180 cm

a Find two expressions for the height of the building, h, in terms of x using the two
angles.
b Solve for x by equating the two expressions obtained in a.
c Find the height of the building.
7 The height of another building needs to be determined but cannot be found directly.
The surveyor decides to measure the angle of elevation of the top of the building
from different sites, which are 75 m apart. The surveyor’s eye level is 189 cm above
the ground.

43°35 32°1
' 8'
x 75 m 189 cm

a Find two expressions for the height of the building, h, in terms of x using the two
angles.
b Solve for x.
c Find the height of the building.

174 Maths Quest 10 New South Wales Australian curriculum edition Stages 5.1 and 5.2
measurement AND geometry

8 A lookout tower has been erected on top of a cliff. At a distance of 5.8 km from the
foot of the cliff, the angle of elevation to the base of the tower is 15.7° and to the
observation deck at the top of the tower is 16° respectively as shown in the figure
below. How high from the top of the cliff is the observation deck?

16°
15.7°
    5.8 km

9 Elena and Sonja were on a camping trip to the 1.3 km Angle of depression
Grampians, where they spent their first day 20°
1.5 km
hiking. They first walked 1.5 km along a path 10°
150 m
inclined at an angle of 10° to the horizontal. 1.4 km
Then they had to follow another path, which was
at an angle of 20° to the horizontal. They walked along this path for 1.3 km, which
brought them to the edge of the cliff. Here Elena spotted a large gum tree 1.4 km
away. If the gum tree is 150 m high, what is the angle of depression from the top of
the cliff to the top of the gum tree?
32°
10 From a point on top of a cliff, two boats are 58°
observed. If the angles of depression are 58° and
32° and the cliff is 46 m above sea level, how far 46 m

apart are the boats?


11 Joseph is asked to obtain an estimate of the
height of his house using any mathematical
technique. He decides to use an inclinometer
and basic trigonometry. Using the inclinometer,
Joseph determines the angle of elevation, θ, from
his eye level to the top of his house to be 42°. x
The point from which Joseph measures the angle θ
of elevation is 15 m away from his house and
h d
the distance from Joseph’s eyes to the ground is
1.76 m.
a Fill in the given information on the diagram
provided (substitute values for the pronumerals).
40°
b Determine the height of Joseph’s house. 62°
12 The competitors of a cross‐country run are nearing
the finish line. From a lookout 100 m above the 100 m
track, the angles of depression to the two leaders,
Nathan and Rachel, are 40° and 62° respectively.
How far apart are the two competitors?

Chapter 5 • Trigonometry 175


measurement AND geometry

13 A 2.05 m tall man, standing in front of a street


light 3.08 m high, casts a 1.5 m shadow.
Digital doc
WorkSHEET 5.3
a What is the angle of elevation from the ground
doc‐5234 to the source of light? 3.08 m 2.05 m
b How far is the man from the bottom of the light
1.5 m
pole?

REASONING
14 The angle of elevation of a hot air balloon changes from 27° at 7.00 am to 61°
at 7.03 am, according to an observer who is 300 m away from the take‐off point.
a Assuming a constant speed, calculate that speed
(in m/s and km/h) at which the balloon is rising, REFLECTION
correct to 2 decimal places. What is the difference
between an angle of
b The balloon then falls 120 metres. What is the angle
elevation and an angle of
of elevation now? Write your answer correct to depression?
1 decimal place.

5G  Bearings
•• A bearing gives the direction of travel from one point or object to another.
•• The bearing of B from A tells how to get to B from A. A compass rose would be
drawn at A.

N B

W E
A

To illustrate the bearing of A from B, a compass rose would be drawn at B.

W E
B

S
A

•• There are two ways in which bearings are commonly written. They are compass
bearings and true bearings.

176 Maths Quest 10 New South Wales Australian curriculum edition Stages 5.1 and 5.2
measurement AND geometry

Compass bearings
•• A compass bearing (for example N40°E or S72°W) has three parts.
–– The first part is either N or S (for north or south).
–– The second part is an acute angle.
–– The third part is either E or W (for east or west).
•• For example, the compass bearing S20°E means start by facing south and then turn
20° towards the east. This is the direction of travel.
N40°W means start by facing north and then turn 40° towards the west.
N N40°W N

40°

W E W E

20°

S20°E
S       S

True bearings
•• True bearings are measured from north in a clockwise direction and are expressed
in 3 digits.
•• The diagrams below show the bearings of 025° true and 250° true respectively.
(These true bearings are more commonly written as 025°T and 250°T.)
N 025°T N

25°

W E W E
250°

250°T

S       S

WORKED EXAMPLE 17
A boat travels a distance of 5 km from P to Q in a direction of 035°T.
a How far east of P is Q?
b How far north of P is Q?
c What is the true bearing of P from Q?

TH I N K W R I TE /D RAW
a 1 Draw a diagram showing the distance a Q
and bearing of Q from P. Complete a θ
right‐angled triangle, travelling x km N
due east from P and then y km due y
North to Q. 35°

P x

Chapter 5 • Trigonometry 177


measurement AND geometry

O
2 To determine how far Q is east of P, sin θ =
we need to find the value of x. We are H
given the length of the hypotenuse
(H) and need to find the length of
the opposite side (O). Write the
sine ratio.
x
3 Substitute O = x, H = 5 and θ = 35°. sin 35° =
5
4 Make x the subject of the equation. x = 5 sin 35°
5 Evaluate and round the answer, correct ≈ 2.87
to 2 decimal places.
6 Write the answer in words. Point Q is 2.87 km east of P.
A
b 1 To determine how far Q is north of P, b cos θ =
we need to find the value of y. This can H
be done in several ways, namely: using
the cosine ratio, the tangent ratio, or
Pythagoras’ theorem. Write the cosine
ratio.
y
2 Substitute A = y, H = 5 and θ = 35°. cos 35° =
5
3 Make y the subject of the equation. y = 5 cos 35°
4 Evaluate and round the answer, correct ≈ 4.10
to 2 decimal places.
5 Write the answer in words. Point B is 4.10 km north of A.
c 1 To find the bearing of P from Q, draw a c N
compass rose at Q. The true bearing is
Q θ
given by ∠θ.
35°

P x

2 The angle θ is the sum of 180° (from θ = 180° + 35°


north to south) and 35°. Write the = 215°
value of θ .
3 Write the answer in words. The bearing of P from Q is
215°T.

•• Sometimes a journey includes a change in directions.


In such cases, each section of the journey should be dealt with separately.

178 Maths Quest 10 New South Wales Australian curriculum edition Stages 5.1 and 5.2
measurement AND geometry

WORKED EXAMPLE 18
A boy walks 2 km on a true bearing of 090° and then 3 km on a true
bearing of 130°.
a How far east of the starting point is the boy at the completion of his
walk? (Answer correct to 1 decimal place.)
b How far south of the starting point is the boy at the completion of his
walk? (Answer correct to 1 decimal place.)
c To return directly to his starting point, how far must the boy walk and
at what bearing?

TH I N K W R I TE /D RAW
a 1 Draw a diagram of the boy’s a N N
journey.
2 130°
E
The first leg of the journey is due east. O P
50°
Label the easterly component x and the y 3
southerly component y.
x Q

O
2 Write the ratio to find x. sin θ =
H
x
3 Substitute O = x, H = 3 and θ = 50°. sin 50° =
3
4 Make x the subject of the equation. x = 3 sin 50°
5 Evaluate and round correct to ≈ 2.3 km
1 decimal place.
6 Add to this the 2 km east that was Total distance east = 2 + 2.3
walked in the first leg of the journey = 4.3 km
and give a worded answer. The boy is 4.3 km east of the
starting point.

b 1 To find y (see the diagram in part a) b Distance south = y km


use Pythagoras’ theorem, as we know a2 = c2 − b2
the lengths of two out of three sides y2 = 32 − 2.32
in the right‐angled triangle. Round = 9 − 5.29
the answer correct to 1 decimal = 3.71
place. y = !3.71
Note: Alternatively, the cosine ratio = 1.9 km
could have been used.
2 Write the answer in words. The boy is 1.9 km south of the
starting point.

Chapter 5 • Trigonometry 179


measurement AND geometry

c 1 Draw a diagram of the journey and c O 4.3


write in the results found in parts a
1.9
and b. Draw a compass rose at Q. N
z
α
Q
ß

2 Find z using Pythagoras’ theorem. z2 = 1.92 + 4.32


= 22.1
z = !22.1
≈ 4.70
4.3
3 Find α using trigonometry. tan α =
1.9

4.3
4 Make α the subject of the equation α = tan−1 a b
using the inverse tangent function. 1.9

5 Evaluate and round to the nearest = 66.161.259 82°


minute. = 66°9′40.535″
= 66°10′

6 The angle β gives the bearing. β = 360° − 66°10′


= 293°50′
7 Write the answer in words. The boy must travel 4.70 km on
a bearing of 293°50′T.

Exercise 5G Bearings
INDIVIDUAL PATHWAYS
Questions: Questions: Questions:
1, 2, 3a–d, 4a–b, 5, 6 1, 2, 3, 4a–c, 5–7, 8, 11 1–6, 8–12
Activity 5‐G‐1 Activity 5‐G‐2 Activity 5‐G‐3
Bearings Calculations involving Applications involving
doc‐5029 bearings bearings
doc‐5030 doc‐5031

FLUENCY
1 Change each of the following compass bearings to true bearings.
a N20°E b N20°W
c S35°W d S28°E
e N34°E f S42°W
2 Change each of the following true bearings to compass bearings.
a 049°T b 132°T
c 267°T d 330°T
e 086°T f 234°T

180 Maths Quest 10 New South Wales Australian curriculum edition Stages 5.1 and 5.2
measurement AND geometry

3 Describe the following paths using true bearings.


a 3k N b N

35°
m

W E
22°
2.5
km

c N d N

35° 2.5 km
4 km
W E
35°
m
8k

e N f N
N N
12 30
km 0m
65°
50° m 40°
7k 50°
0m
50

4 Show each of the following journeys as a diagram.


a A ship travels 040°T for 40 km and then 100°T for 30 km.
b A plane flies for 230 km in a direction 135°T and a further 140 km in a
direction 240°T.
c A bushwalker travels in a direction 260°T for 0.8 km, then changes direction to
120°T for 1.3 km, and finally travels in a direction of 32° for 2.1 km.
d A boat travels N40°W for 8 km, then changes direction to S30°W for 5 km and
then S50°E for 7 km.
e A plane travels N20°E for 320 km, N70°E or 180 km and S30°E for 220 km.
5 WE 17 a A yacht travels 20 km from A to B on a bearing of 042°T:
i how far east of A is B?
ii how far north of A is B?
iii what is the bearing of A from B?
b The yacht then sails 80 km from B to C on a bearing of 130°T.
i Show the journey using a diagram.
ii How far south of B is C?
iii How far east of B is C?
iv What is the bearing of B from C?
6 If a farmhouse is situated 220 m N35°E from a shed, what is the true bearing of the
shed from the house?

Chapter 5 • Trigonometry 181


measurement AND geometry

UNDERSTANDING
7 A pair of hikers travel 0.7 km on a true bearing of 240° and then 1.3 km on a
true bearing of 300°. How far west have they travelled from their starting
point?
8 WE 18 A boat travels 6 km on a true bearing of 120° and then 4 km on a true
bearing of 080°.
a How far east is the boat from the starting point on the completion of its
journey?
b How far south is the boat from the starting point on the completion of its
journey?
c What is the bearing of the boat from the starting point on the completion of its
journey?
9 A plane flies on a true bearing of 320° for 450 km. It then flies on a true bearing of
350° for 130 km and finally on a true bearing of 050° for 330 km. How far north of
its starting point is the plane?
REASONING
10 A bushwalker leaves her tent and walks due east for 4.12 km, then walks a further
3.31 km on a bearing N20°E. If she wishes to return directly to her tent, how
far must she walk and what bearing should she take? (Answer to the nearest
degree.)
11 A car travels due south for 3 km and then due east for 8 km. What is the bearing of
the car from its starting point? (Answer to the nearest degree.)
12 If the bearing of A from O is θ°T, then what is the
bearing of O from A: REFLECTION
What is the difference
i if 0° < θ ° < 180° between true bearings and
ii if 180° < θ° < 360°? compass directions?

CHALLENGE 5.2

5H  Applications
•• When applying trigonometry to a practical situation, it is essential to draw a good
Interactivity
Applying trigonometry mathematical diagram, using points, lines and angles.
to drafting problems •• Several diagrams may be needed to show all the necessary right‐angled
int‐2781
triangles.

182 Maths Quest 10 New South Wales Australian curriculum edition Stages 5.1 and 5.2
measurement AND geometry

WORKED EXAMPLE 19
A ladder of length 3 m makes an angle of 32° with the wall.
a How far is the foot of the ladder from the wall?
b How far up the wall does the ladder reach?
c What angle does the ladder make with the ground?

TH I N K W R I TE /D RAW
Sketch a diagram and label the sides of the (wall)
right‐angled triangle with respect to the given
angle. A
3m 32°
H y

α x
O

O
a 1 We need to find the distance of the foot of a sin θ =
the ladder from the wall (O) and are given H
the length of the ladder (H). Write the sine
ratio.
x
2 Substitute O = x, H = 3 and θ = 32°. sin 32° =
3
3 Make x the subject of the equation. x = 3 sin 32°
4 Evaluate and round the answer to 2 decimal ≈ 1.59 m
places.
5 Write the answer in words. The foot of the ladder is 1.59 m from
the wall.
A
b 1 We need to find the height the ladder b cos θ =
reaches up the wall (A) and are given the H
hypotenuse (H). Write the cosine ratio.
y
2 Substitute A = y, H = 3 and θ = 32°. cos 32° =
3
3 Make y the subject of the equation. y = 3 cos 32°
4 Evaluate and round the answer to 2 decimal y ≈ 2.54 m
places.
5 Write the answer in words. The ladder reaches 2.54 m up the wall.
c 1 To find the angle that the ladder makes c α + 90° + 32° = 180°
with the ground, we could use any of the α + 122° = 180°
trigonometric ratios, as the lengths of α = 180° − 122°
all three sides are known. However, it is α = 58°
quicker to use the angle sum of a triangle.
2 Write the answer in words. The ladder makes a 58° angle with the
ground.

Chapter 5 • Trigonometry 183


measurement AND geometry

Exercise 5H Applications
INDIVIDUAL PATHWAYS
Questions: Questions: Questions:
1–4, 8 1–5, 8 1, 3, 4, 6, 7, 9
Activity 5‐H‐1 Activity 5‐H‐2 Activity 5‐H‐3
Trigonometry applications 1 Trigonometry applications 2 Trigonometry applications 3
doc‐5032 doc‐5033 doc‐5034

FLUENCY
1 A carpenter wants to make a roof pitched at 29°30′, as shown in the diagram. How
long should he cut the beam, PR?
R

29°30'
P Q
10.6 m

2 The mast of a boat is 7.7 m high. A guy wire from the top of the mast is fixed to the
deck 4 m from the base of the mast. Determine the angle the wire makes with the
horizontal.

UNDERSTANDING
3 A steel roof truss is to be made to the following design.

20°

10 m

a How high is the truss?


b What is the total length of steel required to make the truss?
4 WE19 A ladder that is 2.7 m long is leaning against a wall at an angle of
20° as shown.
T

20°
2.7 m

W B

If the base of the ladder is moved 50 cm further away from the wall, what angle will
the ladder make with the wall?

184 Maths Quest 10 New South Wales Australian curriculum edition Stages 5.1 and 5.2
measurement AND geometry

5 A wooden framework is built as shown.


C

5m

38
A B

Bella plans to reinforce the framework by adding a strut from C to the midpoint of
AB. What will be the length of the strut?
6 Atlanta is standing due south of a 20-m flagpole at a point where the angle of
elevation of the top of the pole is 35°. Ginger is standing due east of the flagpole at
a point where the angle of elevation of the top of the pole is 27°. How far is Ginger
from Atlanta?
7 From a point at ground level, Henry measures the angle of elevation of the top of a
tall building to be 41°. After walking directly towards the building, he finds the angle
of elevation to be 75°. If the building is 220 m tall, how far did Henry walk between
measurements?
8 Sailing towards a mountain peak of height 893 m, Imogen measured the angle of
elevation to be 14°. A short time later the angle of elevation was 27°. How far had
Imogen sailed in that time?

REASONING
9 Aldo the carpenter is lost in a rainforest. He comes across a large river and he knows
that he cannot swim across it. Aldo intends to build a bridge across the river. He draws
some plans to calculate the distance across the river as shown in the diagram below.

72°
River Tree
4.5 cm

88°

a Aldo used a scale of 1 cm to represent 20 m. Find the real‐life distance


represented by 4.5 cm in Aldo’s plans.
b Use the diagram below to write an equation for h in terms of d and the two angles.

h
ϑ1 ϑ2
d–x x REFLECTION
d What are some real‐
life applications of
c Use your equation from b to find the distance across trigonometry?
the river, correct to the nearest metre.

Chapter 5 • Trigonometry 185


measurement AND geometry

CHAPTER REVIEW
LANGUAGE

adjacent dimensions pyramid


angle of depression equilateral Pythagoras’ theorem
angle of elevation horizontal ratio
bearing hypotenuse second
compass rose inverse sine
cosine isosceles tangent
cuboid minute true bearing
degree opposite wedge

int-2838 doc-13719
int-2839 doc-13720
int-3592

FLUENCY
1 The most accurate measure for the length of the third side in the
triangle at right is:
A 4.83 m
B 23.3 cm
5.6 m
C 3.94 m
2840 mm
D 2330 mm
E 4826 mm
2 What is the value of x in this figure?
A 5.4
B 7.5 x
C 10.1 5

D 10.3
2 7
E 4
3 What is the closest length A B
of AG of the cube at right?
A 10 D
C
10
B 30
E
C 20 F

D 14 10
E 17 H 10 G

186 Maths Quest 10 New South Wales Australian curriculum edition Stages 5.1 and 5.2
measurement AND geometry

4 If sin 38° = 0.6157, which of the following will also give this result?
A sin 218° B sin 322° C sin 578° D sin 682° E sin 142°
5 The angle 118°52′34″ is also equal to:
52°
A 118.5234° B 118 34 C 118.861° D 118.876° E 118.786°
6 Which trigonometric ratio for the triangle shown at right is incorrect?
b a a
A sin α = B sin α = C cos α =
c c c
a b
b a
D tan α = E tan θ = α θ
a b c
7 Which of the following statements is correct?
A sin 55° = cos 55° B sin 45° = cos 35° C cos 15° = sin 85°
D sin 30° = sin 60° E sin 42° = cos 48°
8 Which of the following can be used to find the value of x in the diagram below?

28.7

35°
x
28.7 28.7
A 28.7 sin 35° B 28.7 cos 35° C 28.7 tan 35° D E
sin 35° cos 35°
9 Which of the following expressions can be used to find the value of a in the triangle
shown?

75
35

35 75 35 75
A 35 sin 75° B sin−1 75 C sin−1 35 D cos−1 75 E cos−1 35
10 If a school is 320 m S42°W from the police station, what is the true bearing of the
police station from the school?
A 042°T B 048°T C 222°T D 228°T E 312°T
11 Calculate x, correct to 2 decimal places.
a b 117 mm

x
82 mm x
123.1 cm

48.7 cm

12 Calculate the value of the pronumeral, correct to 2 decimal places.


13.4 cm

x x

Chapter 5 • Trigonometry 187


measurement AND geometry

13 Calculate the height of this pyramid.

10 mm

8 mm
8 mm

14 A person standing 23 m away from a tree observes the top of the tree at an
angle of elevation of 35°. If the person is 1.5 m tall, what is the height of
the tree?
15 A man of height 1.8 m stands at the window of a tall building. He observes his
young daughter in the playground below. If the angle of depression from the man to
the girl is 47° and the floor on which the man stands is 27 m above the ground, how
far from the bottom of the building is the child?
16 A plane flies 780 km in a direction of 185°T. How far west has it travelled from the
starting point?
17 A hiker travels 3.2 km on a bearing of 250°T and then 1.8 km on a bearing of
320°T. How far west has she travelled from the starting point?
18 If a 4-m ladder is placed against a wall and the foot of the ladder is 2.6 m from the
wall, what angle does the ladder make with the wall?

PROBLEM SOLVING
1 A surveyor needs to determine the height of a building. She measures the angle of
elevation of the top of the building from two points, 64 m apart. The surveyor’s eye
level is 195 cm above the ground.

188 Maths Quest 10 New South Wales Australian curriculum edition Stages 5.1 and 5.2
measurement AND geometry

47°48 36°2
ʹ 4ʹ
x 64 m 195 cm

a Find the expressions for the height of the building, h, in terms of x using the two
angles.
b Solve for x by equating the two expressions obtained in part a.
c Find the height of the building.
2 The height of a right square‐based pyramid is 13 cm. If the angle the face makes
with the base is 67°, find:
a the length of the edge of the square base
b the length of the diagonal of the base
c the angle the slanted edge makes with the base.
3 A boat sails on a compass direction of E12°S for 10 km then changes direction to
S27°E for another 20 km. The boat then decides to return to its starting point.

A
12°
10 km B

27°
20 km

C
a How far, correct to 2 decimal places, is the boat from its starting point?
b On what bearing should the boat travel to return to its starting point? Write the
angle correct to the nearest degree.
4 A car is travelling northwards on an elevated expressway 6 m above ground at a
speed of 72 km/h. At noon another car passes under the expressway, at ground level,
travelling west, at a speed of 90 km/h.
a How far apart, in metres, are the two cars 40 seconds after noon?
b At this time the first car stops, while the second car keeps going. At what time
will they be 3.5 km apart? Write your answer correct to the nearest tenth of a
second.
5 Two towers face each other separated by a distance, d, of 20 metres. As seen from
the top of the first tower, the angle of depression of the second tower’s base is 59°
and that of the top is 31°. What is the height in metres, correct to 2 decimal places,
of each of the towers?

Chapter 5 • Trigonometry 189


measurement AND geometry
Communicating

Rich task

How steep is the land?

172
B
173

172.5
171.5

171

Contour
lines
Rectangular
block of land 170.5

170

Scale 1 : 500

1 A cross‐section shows a profile of the surface of the ground. Let us look at the cross‐section of
the ground between A and B. The technique used is as follows.
• Place the edge of a piece of paper on the line joining A and B.
• Mark the edge of the paper at the points where the contour lines intersect the paper.
• Transfer this paper edge to the horizontal scale of the profile and mark these points.
• Choose a vertical scale within the range of the heights of the contour lines.
• Plot the height at each point where a contour line crosses the paper.
• Join the points with a smooth curve.

190 Maths Quest 10 New South Wales Australian curriculum edition Stages 5.1 and 5.2
measurement AND geometry
MEASUREMENT GEOMETRY

The cross‐section has been started for you. Complete the profile of the line AB.
You can now see a visual picture of the profile of the soil between A and B.
Cross-section of AB
173 173

172.5 172.5

Height (metres)
Height (metres)
172 172

171.5 171.5

171 171

170.5 170.5

170 170
B A
Profile of line BA (metres)

2 We now need to determine the horizontal distance between A and B.


a Measure the map distance between A and B using a ruler. What is the map length?
b Using the scale of 1:500, calculate the actual horizontal distance AB (in metres).
3 The vertical difference in height between A and B is indicated by the contour lines. What is this
vertical distance?
4 Complete the measurements on this diagram.
B
Vertical
distance
= ........ m a
A
Horizontal distance = ........ m

5 The angle a represents the angle of the average slope of the land from A to B. Use the tangent
ratio to calculate this angle (to the nearest minute).
6 In general terms, an angle less than 5° can be considered a gradual to moderate rise. An angle
between 5° and 15° is regarded as moderate to steep while more than 15° is a steep rise. How
would you describe this block of land?
7 Imagine that you are going on a bush walk this
weekend with a group of friends. At right is a
contour map of the area. Starting at X, the plan
30
0

is to walk directly to the hut.


Hut
Draw a cross‐section profile of the walk in the space 25
0
below and calculate the average slope of the land.
How would you describe the walk?
200

150
X
Scale 1 : 20 000

Chapter 5 • Trigonometry 191


measurement AND geometry

Code Puzzle

What will Sir have to


follow the chicken?
Find the size of the angles marked to the nearest degree.
Shade in the box containing each of these angles. The remaining
boxes contain letters that spell out the customer’s reply.

T I N Y D R I P G U E S T
43° 72° 29° 52° 8° 58° 70° 26° 77° 80° 14° 69° 61°
L I O N I N T E X A S
12° 54° 45° 84° 19° 63° 66° 37° 60° 79° 24°
S P Y E J E C T 0.9 m
49° 82° 34° 47° 22° 31° 75° 16°

13.7 m
7.5 m 3m 1.6 m

28 m
6.3 m

4.3 m
8m
8.4 m
9.3 m 5.2 m
5m
21 m

15 m
26 m
4.8 m

5.4 m

9.5 m

2.35 m
5.4 m 2m
0.41 m

11 m
12 m
4.2 m 8.3 m
3.9 m

192 Maths Quest 10 New South Wales Australian curriculum edition Stages 5.1 and 5.2
MEASUREMENT AND GEOMETRY

 ACTIVITIES
Go to assessON
Chapter opener Digital docs (page 165) for questions to
video • Activity 5‐D‐1 (doc‐5020): Calculating side test your readiness
• The story of mathematics (eles-1844) lengths using trigonometry FOR learning, your
• Activity 5‐D‐2 (doc‐5021): Applying progress AS you learn
5A Pythagoras’ theorem ­trigonometry to simple figures and your levels OF
Digital docs (page 150) • Activity 5‐D‐3 (doc‐5022): Practical achievement.
• Activity 5‐A‐1 (doc‐5011): Review of ­applications of trigonometry www.assesson.com.au
­P ythagoras’ theorem (page 150)
• Activity 5‐A‐2 (doc‐5012): Practising 5E Using trigonometry to calculate
­P ythagoras’ theorem (page 150) ­a ngle size
• Activity 5‐A‐3 (doc‐5013): More of Digital docs

­P ythagoras’ theorem (page 150) • Activity 5‐E‐1 (doc‐5023): Review of angle


• SkillSHEET (doc‐5224): Rounding to a calculations (page 169)
­given number of decimal places • Activity 5‐E‐2 (doc‐5024): Calculating angles
(page 150) using trigonometry (page 169)
• Activity 5‐E‐3 (doc‐5025): Applying
5B Pythagoras’ theorem in three trigonometry to angle calculations
­dimensions (page 169)
Digital docs • SkillSHEET (doc‐5232): Rounding angles to
• Activity 5‐B‐1 (doc‐5014): Pythagoras in the nearest degree (page 169)
3‐dimensions (page 155) • WorkSHEET 5.2 (doc‐5233): Using
• Activity 5‐B‐2 (doc‐5015): Pythagoras in ­trigonometry (page 172)
3‐D figures (page 155)
• Activity 5‐B‐3 (doc‐5016): Investigating 5F Angles of elevation and depression
eLesson
­triangles in 3‐D figures (page 155)
• Height of a satellite (eles‐0173)
• SkillSHEET (doc‐5229): Drawing 3‐D
(page 172)
shapes (page 156)
Digital docs
• WorkSHEET 5.1 (doc‐5230): Pythagoras’
• Activity 5‐F‐1 (doc‐5026): Identifying
theorem (page 157)
­elevation and depression
5C Trigonometric ratios (page 173)
Digital docs • Activity 5‐F‐2 (doc‐5027): Calculating
• Activity 5‐C‐1 (doc‐5017): Review of ­elevation and depression
­trigonometry (page 161) (page 173)
• Activity 5‐C‐2 (doc‐5018): Using • Activity 5‐F‐3 (doc‐5028): Applications of
­trigonometry (page 161) ­elevation and depression
• Activity 5‐C‐3 (doc‐5019): Applying (page 173)
­trigonometry (page 161) • SkillSHEET (doc‐5228): Drawing a diagram
• SkillSHEET (doc‐5226): Labelling the sides of from given directions (page 174)
a right‐angled triangle (page 161) • WorkSHEET 5.3 (doc‐5234): Elevation and
• SkillSHEET (doc‐5231): Selecting an depression (page 176)
­appropriate trigonometric ratio based on
5G Bearings
the given information (page 161)
Digital docs (page 180)
5D Using trigonometry to calculate side • Activity 5‐G‐1 (doc‐5029): Bearings
lengths • Activity 5‐G‐2 (doc‐5030): Calculations
Interactivity involving bearings
• Using trigonometry (int‐1146) • Activity 5‐G‐3 (doc‐5031): Applications
(page 163) ­involving bearings

Chapter 5 • Trigonometry 193


measurement AND geometry

5H Applications Chapter review


Interactivity (page 182) Interactivities (page 186)
• Applying trigonometry to drafting problems • Word search (int‐2838)
(int‐2781) • Crossword (int‐2839)
Digital docs (page 184) • Sudoku (int‐3592)
• Activity 5‐H‐1 (doc‐5032): Trigonometry Digital docs
­applications 1 • Chapter summary (doc‐13719)
• Activity 5‐H‐2 (doc‐5033): Trigonometry • Chapter map (doc‐13720)
­applications 2
To access eBookPLUS activities, log on to
• Activity 5‐H‐3 (doc‐5034): Trigonometry
www.jacplus.com.au
­applications 3

194 Maths Quest 10 New South Wales Australian curriculum edition Stages 5.1 and 5.2
measurement AND geometry

Answers
CHAPTER 5 Trigonometry
Exercise 5A — Pythagoras’ theorem 6    a 2.824 b 71.014 c 20.361
1    a 7.86 b 33.27 c 980.95 d 2.828 e 226.735 f 1.192
d 12.68 e 2.85 f 175.14 g 7.232 h 32.259 i 4909.913
2    a 36.36 b 1.62 c 15.37 j 0.063 k 0.904 l 14.814
d 0.61 e 2133.19 f 453.90 e d e
7    a i sin( θ ) = ii cos( θ ) = iii tan( θ ) =
3 23.04 cm f f d
4 12.65 cm i h i
b i sin(α) = ii cos(α) = iii tan(α) =
5    a 14.14 cm b 24.04 cm c 4.53 cm g g h
6    a 97.47 cm b 334.94 cm c 6822.90 cm2 l j l
c i sin(β) = ii cos(β) = iii tan(β) =
7    a 6.06 b 4.24 c 4.74 k k j
8 18.03 cm n o n
d i sin(γ) = ii cos(γ) = iii tan(γ) =
9 17.32 cm m m o
10 19.23 cm b a b
e i sin(β) = ii cos(β) = iii tan(β) =
11 65.82 cm; 2501.16 cm2 c c a
v t v
12 39 m f i sin(γ) = ii cos(γ) = iii tan(γ)
u u t
13 Yes
8    a sin( θ ) = 12 b cos( θ ) = 25 c tan( θ ) = 45
14 4.34 km 15 30
15 38.2 m 2.7 17
d tan( θ ) = e sin(35°) = f sin(α) = 14.3
16 20.61 m p t 17.5

17 54.67 mm 7
g sin(15°) = h tan( θ ) = 20 i cos(α) = 3.1
18    a 28 cm b 588 cm2 x 31 9.8

19 36.37 cm 9    a α
H
O
20 552.86 cm2 41°
21 21.46 diagonals, so would need to complete 22 A
22 1600 mm
b O = 34 mm, A = 39 mm, H = 51 mm
23 5889.82 m
c  i sin(41°) = 0.66 ii cos(41°) = 0.75
24 7.07 cm
iii tan(41°) = 0.87
25    a 65 b 185 c 305
d α = 49°
26    a Neither 105 nor 208 can be the hypotenuse of the triangle,
e  i sin(49°) = 0.75 ii cos(49°) = 0.66
because they are the two smallest values. Two other values
iii tan(49°) = 1.15
could be the hypotenuse, because they enable the creation of a
f They are equal.
right‐angled triangle.
g They are equal.
b 105, 208 and 233.
h The sin of an angle is equal to the cos of its complement
Exercise 5B — Pythagoras’ theorem in three dimensions angle.
1    a 17.32 b 12.25 c 15.12 Exercise 5D — Using trigonometry to calculate side lengths
2 12.21, 12.85
1    a 8.660 b 7.250 c 8.412
3 4.84 m, 1.77m
2    a 0.79 b 4.72 c 101.38
4 11.31, 5.66
3    a 33.45 m b 74.89 m c 44.82 m
5 31.62 cm
d 7.76 mm e 80.82 km f 9.04 cm
6 6 cm
4    a x = 31.58 cm
7 12.65 cm
b y = 17.67 m
8 23 mm
c z = 14.87 m
9 No: the maximum possible stick length is only 115 cm.
d p = 67.00 m
10    a i 283.02 m ii 240.21 m iii 150.33 m b 141.86 m
e p = 21.38 km, q = 42.29 km
11 14.72 cm
f a = 0.70 km, b = 0.21 km
12 1.3 m, 5.98 m2
5    a 6.0 m b 6.7 m
13 Students’ own working
6 1.05 m
Exercise 5C — Trigonometric ratios 7    a An isosceles right‐angled triangle
1    a 0.5000 b 0.7071 c 0.4663 b θ < 45°
d 0.8387 e 8.1443 f 0.7193 Exercise 5E — Using trigonometry to calculate angle size
2    a 0.6494 b 0.5885 c 0.5220
1    a 67° b 47° c 69°
d −1.5013 e 0.9990 f 0.6709
2    a 54°47′ b 33°45′ c 33°33′
g 0.8120 h 0.5253 i −0.8031
3    a 75°31′21″ b 36°52′12″ c 37°38′51″
3    a 50° b 24° c 53°
4    a 41° b 30° c 49°
d 71° e 86° f 41°
d 65° e 48° f 37°
4    a 54°29′ b 6°19′ c 0°52′
5    a a = 25°47′, b = 64°13′ b d = 25°23′, e = 64°37′
d 72°47′ e 44°48′ f 26°45′
c x = 66°12′, y = 23°48′
5    a 26°33′54″ b 64°1′25″ c 64°46′59″
6    a r = 57.58, l = 34.87, h = 28.56
d 48°5′22″ e 36°52′12″ f 88°41′27″
b 428 cm2 c 29.7°

Chapter 5 • Trigonometry 195


measurement AND geometry

7    a   i 29.0° ii 41.4° iii 51.3° e N


b  i 124.42 km/h 70°180 km
  ii 136.57 km/h

22
iii 146.27 km/h

0k
N 30°

m
8 Answers will vary. 20°

km
S
9    a sin(30°) = 12

320
!3
    cos(30°) = 2
!3 5    a i 13.38 km ii 14.86 km iii 222°T
    tan(30°) = 3
b i N   ii 51.42 km
     b, c Answers will vary.     iii 61.28 km
130°
N B     iv 310°T
Challenge 5.1
147°0′; 12°15′ 42° 80

km
km
Exercise 5F — Angles of elevation and depression

20
A
1 8.74 m
2 687.7 m C
3    a 176.42 m b 152.42 m 6 215°T
4 65°46′ 7 1.732 km
5 16.04 m 8    a 9.135 km b 2.305 km c 104°11′ T
6    a h = x tan(47°12′) m; h = (x + 38) tan(35°50′) m 9 684.86 km
b x = 76.69 m c 84.62 m 10 6.10 km on 239°T
7    a h = x tan(43°35′) m; h = (x + 75) tan(32°18′) m 11 111°T
b 148.40 m c 141.1 m 12    i (180 + θ )° T ii ( θ − 180°)T
8 0.033 km or 33 m
Challenge 5.2
9 21°
10 44.88 m 3.65 km on a bearing of 108°T
11    a b 15.27 m Exercise 5H — Applications
x 1 6.09 m
42°
2 62°33′
1.76 m 15 m 3    a 1.82 m b 27.78 m
12 66 m 4 31°49′
13    a 54° 5 5.94 m
b 0.75 m 6 49 m
14    a 2.16 m/s, 7.77 km/h 7 194 m
b 54.5° 8 1.829 km
9    a 90 m
Exercise 5G — Bearings d tan θ 1
1    a 020°T b 340°T c 215°T b h= × tan θ 2
tan θ 1 + tan θ 2
d 152°T e 034°T f 222°T c 250 m
2    a N49°E b S48°E c S87°W
d N30°W e N86°E f S54°W Chapter review
3    a 3 km 325°T Fluency
b 2.5 km 112°T 1 E
c 8 km 235°T 2 D
d 4 km 090°T, then 2.5 km 035°T 3 E
e 12 km 115°T, then 7 km 050°T 4 E
f 300 m 310°T, then 500 m 220°T 5 D
6 B
4    a N b N 7 E
100° 135° 8 B
N 30 km 9 B
N
23

10 A
0
km

km

40°
11    a x = 113.06 cm b x = 83.46 mm
40

0k
m 12 9.48 cm
14 240°
13 8.25 mm
14 17.6 m
15 26.86 m
c N d N 16 67.98 km
N 17 4.16 km
120° 260°
18 40°32′
8k

N
m

0.8 km
5k

N 30°
km

40° Problem solving


32°
2.1

1.3
km 1    a h = tan(47°48′)x m
7k h = tan(36°24′) (x + 64) m
50° m b 129.10 m c 144.32 m
S 2    a 11.04 cm b 15.6 cm c 59°2′

196 Maths Quest 10 New South Wales Australian curriculum edition Stages 5.1 and 5.2
measurement AND geometry

3    a 27.42 km b N43°W or 227°T 4 B


4    a 1280.6 m b 12:02:16.3 pm Vertical
distance
5 33.29 m, 21.27 m = 3m a
A
Communicating — Rich task
Horizontal distance = 34 m
1 Cross-section of AB
5 a = 5°3′
173 173
6 Moderate to steep
7 Cross-section X to hut
172.5 172.5
300 300

Height (metres)

Height (metres)
172 172
Height (metres)

Height (metres)
250 250

171.5 171.5
200 200

171 171
150 150
X Hut
Profile of X to hut
170.5 170.5
The average slope is 14.04° — moderate to steep.
170 170 Code puzzle
B A
Profile of line BA (metres)
Indigestion, I expect.

2    a 6.8 cm b 34 m
3 3m

Chapter 5 • Trigonometry 197


measurement AND geometry

Chapter 6

Surface area
and volume
WHY LEARN THIS?
Humans must measure! How much paint or carpet will
you need to redecorate your bedroom? How many litres
of water will it take to fill the new pool? How far is it to
the end of the universe? These are just a few examples
of where measurements skills are needed. Measuring
tools have advanced significantly in their capacity
to measure extremely small and extremely large
amounts, leading to many breakthroughs in medicine,
engineering, science, architecture and astronomy.

WHAT DO YOU KNOW?


1 Think List what you know about measurement.
Use a thinking tool such as a concept map to show
your list.
2 pair Share what you know with a partner and then
with a small group.
3 share As a class, create a large concept map that
shows your class’s knowledge of measurement.

LEARNING SEQUENCE
6A Area
6B Total surface area
6C Volume

198 Maths Quest 10 New South Wales Australian curriculum edition Stages 5.1 and 5.2
Watch this video
The story of mathematics

Searchlight ID: eles-1845

Chapter 6 • Surface area and volume 199


measurement AND geometry

6A  Area
•• The area of a figure is the amount of surface covered by the figure.
•• The units used for area are mm2, cm2, m2, km2 or ha (hectares), depending upon the
size of the figure.
l ha = 10 000 (or 104) m2
•• There are many real‐life situations that require an understanding of the area concept.
Some are, ‘the area to be painted’, ‘the floor area of a room or house’, and ‘how
much land one has’, ‘how many tiles are needed for a wall’.
•• It is important that you are familiar with converting units of area.

Using area formulas


•• The area of many plane figures can be found by using a formula. The table below
shows the formula for the area of some common shapes.

Shape Formula
1. Square    A = l2
l

2. Rectangle   l A = lw

3. Triangle    A = 12bh
h

4. Trapezium   a A = 12 (a + b)h
h

5. Circle     A = πr2
r

6. Parallelogram A = bh
h

200 Maths Quest 10 New South Wales Australian curriculum edition Stages 5.1 and 5.2
measurement AND geometry

Shape Formula
7. Sector     θ
A= × πr2
360

θ˚
r

8. Kite A = 12xy, where x and y are diagonals.


(including y
a rhombus)  
x

9. Ellipse     A = πab, where a and b are the lengths


b
of the semi‐major and semi‐minor axes
a
respectively.

Note: A calculator uses a stored value for π of approximately 3.141  592  654. Before


calculators were in common usage, π was often taken to be approximately 22 7
or 3.14.
22
You are advised to use the π button on your calculator rather than 7 or 3.14.

Heron’s formula
•• If all sides of a triangle are known, its area, A, can be found b a eLesson
Heron’s formula
by using Heron’s formula: eles‐0177
c
A = !s(s − a) (s − b) (s − c) where a, b and c are the
a+b+c
lengths of the three sides and s is the semi‐perimeter where s = .
2
WORKED EXAMPLE 1
Find the areas of the following plane figures, correct to 2 decimal places.
a b c
3 cm 5 cm 2 cm
5 cm 15 cm
40°
6 cm

THINK W R I TE
a 1 Three side lengths are known, but not the height. a A = !s(s − a)(s − b)(s − c)
Apply Heron’s formula.
2 Identify the values of a, b and c. a = 3, b = 5, c = 6
3 Calculate the value of s, the semi‐perimeter of a+b+c
s =
the triangle. 2
3+5+6
=
2
14
=
2
=7

Chapter 6 • Surface area and volume 201


measurement AND geometry

4 Substitute the values of a, b, c and s into Heron’s A = !7(7 − 3)(7 − 5)(7 − 6)


formula and evaluate, correct to 2 decimal places. = !7 × 4 × 2 × 1
= !56
= 7.48 cm2
b 1 The shape shown is an ellipse. Write the b A = πab
appropriate area formula.
2 Identify the values of a and b (the semi‐major a = 5, b = 2
and semi‐minor axes).
3 Substitute the values of a and b into the A = π×5×2
formula and evaluate, correct to 2 decimal = 31.42 cm2
places.
θ
c 1 The shape shown is a sector. Write the formula c A= × πr2
for finding the area of a sector. 360°

2 Write the value of θ and r. θ = 40°, r = 15


40°
3 Substitute and evaluate the expression, correct A = × π × 152
360°
to 2 decimal places.
= 78.54 cm2

Areas of composite figures


•• A composite figure is a figure made up of a combination of simple figures.
•• The area of a composite figure can be calculated by:
–– calculating the sum of the areas of the simple figures that make up the composite
figure
–– calculating the area of a larger shape and then subtracting the extra area
involved.

WORKED EXAMPLE 2
Find the area of each of the following composite shapes.
a C b A B
AB = 8 cm
EC = 6 cm
FD = 2 cm 9 cm

D C
2 cm
F E
A B
E
5 cm
D
H 10 cm G

THINK W R I TE
a 1 ACBD is a quadrilateral that can be split into two a Area ACBD = Area ΔABC + Area ΔABD
triangles: ΔABC and ΔABD.

202 Maths Quest 10 New South Wales Australian curriculum edition Stages 5.1 and 5.2
measurement AND geometry

2 Write the formula for the area of a Atriangle = 12bh


triangle containing base and
height.

3 Identify the values of b and h for ΔABC. ΔABC: b = AB = 8, h = EC = 6


1
4 Substitute the values of the pronumerals Area of ΔABC = 2
× AB × EC
into the formula and, hence, calculate the = 1
×8×6
area of ΔABC. 2
= 24 cm2

5 Identify the values of b and h for ΔABD: b = AB = 8, h = FD = 2


ΔABD.

6 Calculate the area of ΔABD. Area of ΔABD = 12AB × FD


1
= 2
×8×2
= 8 cm2

7 Add the areas of the two triangles Area of ACBD = 24 cm2 + 8 cm2
together to find the area of the = 32 cm2
quadrilateral ACBD.

b 1 One way to find the area of the shape shown b Area = Area ABGH − Area DEFC
is to find the total area of the rectangle ABGH
and then subtract the area of the smaller
rectangle DEFC.
2 Write the formula for the area of a Arectangle = l × w
rectangle.
3 Identify the values of the pronumerals for the Rectangle ABGH: l = 9 + 2 + 9
rectangle ABGH. = 20
w = 10

4 Substitute the values of the pronumerals into Area of ABGH = 20 × 10


the formula to find the area of the rectangle = 200 cm2
ABGH.

5 Identify the values of the pronumerals for the Rectangle DEFC: l = 5, w = 2


rectangle DEFC.
6 Substitute the values of the pronumerals into Area of DEFC = 5 × 2
the formula to find the area of the rectangle = 10 cm2
DEFC.

7 Subtract the area of the rectangle DEFC from the Area = 200 − 10
area of the rectangle ABGH to find the area of the = 190 cm2
given shape.

Chapter 6 • Surface area and volume 203


measurement AND geometry

Exercise 6A Area
INDIVIDUAL PATHWAYS
Questions: Questions: Questions:
1, 3–5, 8, 9, 11, 12, 14 1–6, 8–10, 12, 14, 16 1–9, 12–17

Activity 6‐A‐1 Activity 6‐A‐2 Activity 6‐A‐3


Review of area Area problems Tricky area problems
doc‐5035 doc‐5036 doc‐5037

Where appropriate, give answers correct to 2 decimal places.


FLUENCY
Digital docs
1 Find the areas of the following shapes.
SkillSHEET a b c
Conversion of area units
doc‐5236 4 cm
SkillSHEET 4 cm
Using a formula to find
the area of a common 15 cm
shape
12 cm
doc‐5237

10 cm

d 12 cm e f

8 cm
15 cm 8 mm 13 mm

18 cm

7 mm

g h i 15 cm
7m 10 cm
6m
18 cm

2 Express the area in questions 1e and 1g in terms of π.


3 WE1a Use Heron’s formula to find the area of the following triangles.
a b
3 cm
8 cm
5 cm
16 cm

6 cm

12 cm

204 Maths Quest 10 New South Wales Australian curriculum edition Stages 5.1 and 5.2
measurement AND geometry

4 WE1b Find the area of the following ellipses. Answer correct to 1 decimal place.
a b
9 mm
12 mm
4 mm
5 mm

5 WE1c Find the area of the following shapes, i stating the answer exactly, that is in
terms of π, and ii correct to 2 decimal places.
a b c
30° 18 cm
6 mm 70°

12 cm
345°

6 MC A figure has an area of about 64 cm2. Which of the following cannot possibly
represent the figure?
A A triangle with base length 16 cm and height 8 cm
B A circle with radius 4.51 cm
C A rectangle with dimensions 16 cm and 4 cm
D A square with side length 8 cm
E A rhombus with diagonals 16 cm and 4 cm
7 MC The area of the quadrilateral shown below right is to be calculated.
Which of the following lists all the lengths
C
required to calculate the area?
A AB, BC, CD and AD
F
B AB, BE, AC and CD B
C BC, BE, AD and CD
D AC, BE and FD
E AC, CD and AB E

A D

8 WE2 Find the area of the following composite shapes.


a 20 cm b 40 m

28 m
15 cm

Chapter 6 • Surface area and volume 205


measurement AND geometry

c 8 cm d
3 cm 2 cm

4 cm 2.1 m
3.8 m

e f
28 cm
18 cm

5 cm

12 cm

9 Find the shaded area in each of the following.

a b 16 m

8m 2m 2m
3 cm
r =7 cm

c d

8m 3m
40°
5m

8m
e f 15 m

5m
2m 7.5 m

13 m 7 m

5m

206 Maths Quest 10 New South Wales Australian curriculum edition Stages 5.1 and 5.2
measurement AND geometry

UNDERSTANDING
10 A sheet of cardboard is 1.6 m by 0.8 m. The following shapes are cut from the
cardboard:
• a circular piece with radius 12 cm
• a rectangular piece 20 cm by 15 cm
• 2 triangular pieces with base 30 cm and height 10 cm
• a triangular piece with side length 12 cm, 10 cm and 8 cm.
What is the area of the remaining piece of cardboard?
11 A rectangular block of land, 12 m by 8 m, is surrounded by a concrete path
0.5 m wide. Find the area of the path.
12 Concrete slabs 1 m by 0.5 m are used to cover a footpath 20 m by 1.5 m. How
many slabs are needed?
13 A city council builds a 0.5 m wide concrete path around the garden as shown below.
12 m

5m

8m 3m

Find the cost of the job if the workman charges $40.00 per m2.
14 A tennis court used for doubles is 10.97 m wide, but a singles court is only 8.23 m
wide, as shown in the diagram.

8.23 m
6.40 m 10.97 m
11.89 m

a What is the area of the doubles tennis court?


b What is the area of the singles court?
c What percentage of the doubles court is used for singles?
15 Ron the excavator operator has 100 metres of barricade mesh and needs to enclose
an area to work in safely. He chooses to make a rectangular region with dimensions
x and y.
a Write an equation that connects x, y and the perimeter.
b Write y in terms of x.
c Write an equation for the area of the region in terms of x.
d Fill in the table for different values of x.

x 0 5 10 15 20 25 30 35 40 45 50
Area

Chapter 6 • Surface area and volume 207


measurement AND geometry

e Can x have a value more than 50? Why?


f Sketch a graph of area against x.
g Determine the value of x that makes the area a maximum.
h What is the value of y for maximum area?
i What shape encloses the maximum area?
j Calculate the maximum area.
Ron decides to choose to make a circular area with the barricade mesh.
k What is the radius of this circular region?
l What area is enclosed in this circular region?
m How much extra area does Ron now have compared to his rectangular
region?

REASONING
16 Dan has purchased a country property with layout and dimensions as shown in the
diagram.
a Show that the property has a total area of 987.5 ha. N
b Dan wants to split the property in half (in terms of
area) by building a straight‐lined fence running either 5000 m
1500 m
north–south or east–west through the property. Assuming 2000 m
the cost of the fencing is a fixed amount per linear metre,
justify where the fence should be built (that is, how 1000 m
many metres from the top left‐hand corner and in which
direction), to minimise the cost.
17 In question 15, Ron the excavator operator could choose to enclose a rectangular or
circular area with 100 m of barricade mesh. In this case, the circular region resulted
Digital doc in a larger safe work area.
WorkSHEET 6.1
doc‐5241 a Show that for 150 m of barricade mesh, a
circular region again results in a larger safe
work area as opposed to a rectangular region. REFLECTION
b Show that for n metres of barricade mesh, a How are perimeter and area
different but fundamentally
circular region will result in a larger safe work related?
area as opposed to a rectangular region.

CHALLENGE 6.1

208 Maths Quest 10 New South Wales Australian curriculum edition Stages 5.1 and 5.2
measurement AND geometry

6B Total surface area


•• The total surface area (TSA) of a solid is the sum of the areas of all the faces of Digital doc
SkillSHEET
that solid. Total surface area of
cubes and rectangular
prisms
TSA of rectangular prisms and cubes doc‐5238

•• The formula for finding the TSA of a rectangular prism


(cuboid) is: h

TSA = 2(lh + lw + wh) w


l

•• A special case of the rectangular prism is the cube where all sides are
equal (l = w = h).
TSA = 6l2

TSA of spheres and cylinders l

Sphere:
r
TSA = 4πr2

Interactivity
TSA‐sphere
Note: The mathematics required to prove the formula for the total surface area of a int‐2782
sphere is beyond the scope of Year 10.
Cylinder:
TSA = 2πr(r + h) or 2πr2 + 2πrh h
r

•• The formula for the TSA of a cylinder is found from the area of the net as shown.
TSA = πr2 + πr2 + 2πrh r
= 2πr2 + 2πrh A = πr 2
= 2πr(r + h) 2πr
A = 2πrh h

r
A = πr 2

WORKED EXAMPLE 3
Find the total surface area of the solids, correct to the nearest cm2.
a r = 7 cm b 50 cm

r
1.5 m

Chapter 6 • Surface area and volume 209


measurement AND geometry

THINK W R I TE
a 1 Write the formula for the TSA of a TSA = 4πr2
a sphere.
2 Identify the value for r. r=7
3 Substitute and evaluate. TSA = 4 × π × 72
≈ 615.8 cm2
≈ 616 cm2
b 1 Write the formula for the TSA of b TSA = 2πr(r + h)
a cylinder.
2 Identify the values for r and h. r = 50 cm, h = 1.5 m
Note that the units will need to be = 150 cm
the same.
3 Substitute and evaluate. TSA = 2 × π × 50 × (50 + 150)
≈ 62 831.9 cm2
≈ 62 832 cm2

TSA of cones
•• The total surface area of a cone can be found by considering its net, which is
comprised of a small circle and a sector of a large circle.

l r = radius of the cone


l = slant height of the cone.
l

r r

•• The sector is a fraction of the full circle of radius, l, with circumference, 2πl.
•• The sector has arc length, equivalent to the circumference of the base of the
cone, 2πr.
•• The fraction of the full circle represented by the sector can be found by writing the
2πr r
arc length as a fraction of the circumference of the full circle, = .
2πl l
Area of a sector = fraction of the circle × πr2
r
= × πl2
l
= πrl
Therefore, SA = Acircular base + Acurved surface
= πr2 + πrl
= πr(r + l)
Cone: TSA = πr(r + l ) or πr2 + πrl

210 Maths Quest 10 New South Wales Australian curriculum edition Stages 5.1 and 5.2
measurement AND geometry

WORKED EXAMPLE 4
Find the total surface area of the cone shown.
15 cm

12 cm

THINK W R I TE
1 Write the formula for the TSA = πr(r + l )
TSA of a cone.
2 State the values of r and l. r = 12, l = 15
3 Substitute and evaluate. TSA = π × 12 × (12 + 15)
= 1017.9 cm2

TSA of other solids


•• TSA can be found by summing the areas of each face.
•• The areas of each face may need to be calculated separately.
•• Check the total number of faces to ensure that none are left out.

WORKED EXAMPLE 5
Find the total surface area of the square‐based 5 cm
pyramid shown.
6 cm

THINK W R I TE /D RAW
1 There are five faces, the TSA = Area of square base + Area of four
square base and four identical triangular faces
triangles.
2 Find the area of the square Area of base = l2, where l = 6
base. Area of base = 62
= 36 cm2
3 Draw and label one triangular
face and write the formula for 5 cm
h
finding its area.
3 cm
Area of a triangular face = 12bh; b = 6
4 Find the height of the triangle a2 = c2 − b2, where a = h, b = 3, c = 5
h, using Pythagoras’ theorem. h2 = 52 − 32
h2 = 25 − 9
h2 = 16
h = 4 cm
5 Calculate the area of the Area of triangular face = 12 × 6 × 4
triangular face by substituting = 12 cm2
b = 6 and h = 4.

Chapter 6 • Surface area and volume 211


measurement AND geometry

6 Calculate the TSA by adding TSA = 36 + 4 × 12


the area of the square base = 36 + 48
and the area of four identical = 84 cm2
triangular faces together.

TSA of composite solids


•• Composite solids are formed when 2 or more simple solids are joined together.
•• The TSA of a composite solid is calculated by summing the areas of the solid’s
external faces.

WORKED EXAMPLE 6
Find the total surface area of the solid shown, correct to 6 cm
1 decimal place.

THINK W R I TE /D RAW
10 cm
1 The solid shown has 9 faces, TSA = 5 × area of a square
5 identical squares and + 4 × area of a triangle
4 identical triangles.
2 Find the area of one square Asquare = l 2, where l = 10
face with the side length A = 102
10 cm. A = 100 cm2
3 Draw a triangular face and
work out its height using 6 cm
Pythagoras’ theorem. h

5 cm
a2
= c2 − b2, where a = h, b = 5, c = 6
h2 = 62 − 52
h2 = 36 − 25
h2 = 11
h = 3.31662… cm (or with rounding, h = 3.3)
4 Find the area of one Atriangle = 12 bh, where b = 10, h = 3.32
triangular face. 1
= 2
× 10 × 3.316 62…
= 16.5831… cm2 (or, with rounding,
Atriangle = 16.6 cm2 )
5 Find the TSA of the solid by TSA = 5 × 100 + 4 × 16.5831…
adding the area of 5 squares = 500 + 66.3324…
and 4 triangles together. ≈ 566.3 cm2 (or = 566 using the
previously rounded value)
Note: Rounding is not done until the final step. If h had been rounded to 3.3 in step 3 and
this value used in steps 4 and 5, the decimal place value of the TSA would have been
lost. It is important to realise that rounding too early can affect the accuracy of results.

212 Maths Quest 10 New South Wales Australian curriculum edition Stages 5.1 and 5.2
measurement AND geometry

WORKED EXAMPLE 7
The silo shown at right is to be
built from metal. The top portion
of the silo is a cylinder of diameter
4 m and height 8 m. The bottom
part of the silo is a cone of slant 4m 8m
height 3 m. The silo has a circular
opening of radius 30 cm on the top.
a What area of metal (to the 3m
nearest m2) is required to build
the silo?
b If it costs $12.50 per m2 to cover
the surface with an anti‐rust material,
how much will it cost to cover the silo completely?
THINK W R I TE
a 1 The surface area of a TSA = area of annulus
the silo consists of + area of curved section of a cylinder
an annulus, the curved + area of curved section of a cone
section of the cylinder
and the curved section
of the cone.
2 To find the area of the Area of annulus = Alarge circle − Asmall circle
annulus, subtract the area = πr2 − πR2
4
of the small circle from where r = 2 = 2 m and R = 30 cm = 0.3 m.
the area of the larger Area of annulus = π × 22 − π × 0.32
circle. Let R = radius of = 12.28 m2
small circle.
3 The middle part of the Area of curved section of cylinder = 2πrh
silo is the curved part of where r = 2, h = 8.
a cylinder. Find its area. Area of curved section of cylinder
(Note that in the formula =2×π×2×8
TSAcylinder = 2πr + 2πrh,
2 = 100.53 m2
the curved part is
represented by 2πrh.)
4 The bottom part of the Area of curved section of cone = πrl
silo is the curved section where r = 2, l = 3.
of a cone. Find its area. Area of curved section of cone = π × 2 × 3
(Note that in the formula = 18.85 m2
TSAcone = πr2 + πrl,
the curved part is given
by πrl.)
5 Find the total surface area TSA = 12.28 + 100.53 + 18.85
of the silo by finding the = 131.66 m2
sum of the surface areas
calculated above.

Chapter 6 • Surface area and volume 213


measurement AND geometry

6 Write the answer in The area of metal required is 132 m2, correct
words. to the nearest square metre.
b To find the total cost, b Cost = 132 × $12.50
multiply the total surface = $1650.00
area of the silo by the cost of
the anti‐rust material per m2
($12.50).

Exercise 6B Total surface area


INDIVIDUAL PATHWAYS
Questions: Questions: Questions:
1–4, 6a–e, 7, 10 1–4, 6, 7, 9–12 1–8, 10–13
Activity 6‐B‐1 Activity 6‐B‐2 Activity 6‐B‐3
Introducing surface area Surface area problems Tricky surface area problems
doc‐5038 doc‐5039 doc‐5040

FLUENCY
Note: Where appropriate, give the answers correct to 1 decimal place.
1 Find the total surface areas of the solids shown.
a b

10 cm 8 cm

c 12 cm d 2m

1.5 m
15 cm

20 cm 3m

2 WE3 Find the total surface area of the solids shown below.

Digital doc a r=3m b 21 cm


SkillSHEET
Total surface area
of cubes and r 30 cm
­rectangular prisms
doc‐5238

c 0.5 m d
12 cm

2.1 m

214 Maths Quest 10 New South Wales Australian curriculum edition Stages 5.1 and 5.2
measurement AND geometry

3 WE4 Find the total surface area of the cones below.


a b 8 cm

20 cm
12 cm
14 cm

4 WE5 Find the total surface area of the solids below.


a b
12 cm 2.5 m

15 cm 1.5 m

c 9.1 cm d
m
8c 14 cm
6 cm
10 cm
5.1 cm 7.2 cm
7 cm

5 Find the surface areas of the following.


a A cube of side length 1.5 m
b A rectangular prism 6 m × 4 m × 2.1 m
c A cylinder of radius 30 cm and height 45 cm, open at one end
d A sphere of radius 28 mm
e An open cone of radius 4 cm and slant height 10 cm
f A square pyramid of base length 20 cm and slant edge 30 cm
6 WE6 Find the total surface area of the objects shown.
a 10 cm 8 cm b
5 cm
12 cm
5 cm

20 cm 20 cm

35 cm
12 cm

c d

2 cm
m
5 cm 2.5 c
3 cm

3 cm

Chapter 6 • Surface area and volume 215


measurement AND geometry

e f 5 cm

3.5 cm
20 cm

10 cm
12 cm

15 cm
7 MC A cube has a total surface area of 384 cm2. The length of the edge of the
cube is:
A 9 cm B 8 cm C 7 cm
D 6 cm E 5 cm

UNDERSTANDING
8 Open cones are made from nets cut from a large sheet of paper 1.2 m × 1.0 m.
If a cone has a radius of 6 cm and a slant height of 10 cm, how many cones can
be made from the sheet? (Assume there is 5% wastage of paper.)
9 A steel girder is to be painted. Calculate the area of the surface to be painted.

2 cm

2 cm
5 cm
20 cm
120 cm

2 cm
12 cm

10 WE7 The greenhouse shown below is to be built using shade cloth. It has a
wooden door of dimensions 1.2 m × 0.5 m.
a Find the total area of shade cloth needed to
complete the greenhouse.
b Find the cost of the shade cloth at $6.50 per m2.

2.5 m 5m

3m

11 A cylinder is joined to a hemisphere to make a cake holder, as shown below. The


surface of the cake holder is to be chromed at 5.5 cents per cm2.
a Find the total surface area to be chromed.
b Find the cost of chroming the cake holder.
10 cm
15 cm

216 Maths Quest 10 New South Wales Australian curriculum edition Stages 5.1 and 5.2
measurement AND geometry

REASONING
12 Find the total surface area of each of the following figures, correct to the
nearest cm2.
a 8 cm
12 cm b
30 cm

10 cm

c d

80 cm

60 cm

60°
40 cm

13 Find the total surface area of each of the following figures, correct to the nearest cm2.
(Note: A frustum is the part of a cone or pyramid that is left when the top is cut off.)
Digital doc
a The frustum of a cone b The frustum of a square pyramid WorkSHEET 6.2
doc‐5242

4 cm 6m
4m

3 cm REFLECTION
10 m Why is calculating the
8 cm
total surface area of a
composite solid more
difficult than for a simple
solid such as a rectangular
9 cm
prism or cylinder?

6C Volume
•• The volume of a 3‐dimensional object is the amount of space it takes up.
•• Volume is measured in units of mm3, cm3 and m3.

Volume of a prism
•• The volume of any solid with a uniform cross‐sectional area is given by the formula
V = AH, where A is the cross‐sectional (or base) area and H is the height of the
solid.
Cube Volume = AH
l
= area of a square × height Interactivity
= l2 × l Maximising the volume
= l3 of a cuboid
int‐1150

Chapter 6 • Surface area and volume 217


measurement AND geometry

Rectangular Volume = AH
prism h = area of a rectangle × height
= lwh
w
l

Cylinder r Volume = AH
= area of a circle × height
h = πr2h

Triangular Volume = AH
prism = area of a triangle × height
= 12 bh × H
H
h

WORKED EXAMPLE 8
Find the volumes of the following shapes.
a 14 cm b 5 cm

20 cm 4 cm
10 cm

THINK W R I TE
a 1 Write the formula for the volume of the a V = AH
cylinder (prism). = πr2h
2 Identify the value of the pronumerals. r = 14, h = 20
3 Substitute and evaluate. V = π × 142 × 20
≈ 12 315.04 cm3
1
b 1 Write the formula for the volume of a b V = 2 bh × H
triangular prism.
2 Identify the value of the pronumerals. b = 4, h = 5, H = 10
(Note h is the height of the triangle and H
is the depth of the prism.)

3 Substitute and evaluate. V = 12 × 4 × 5 × 10


= 100 cm3

218 Maths Quest 10 New South Wales Australian curriculum edition Stages 5.1 and 5.2
measurement AND geometry

WORKED EXAMPLE 9
a What effect will doubling each of the side lengths of a cube have on its
volume?
b What effect will halving the radius and doubling the height of a cylinder
have on its volume?

THINK W R I TE
a 1 Write the formula for the volume of a V = l3
the cube.
2 Identify the value of the pronumeral. lnew = 2l
Note: Doubling is the same as
multiplying by 2.
3 Substitute and evaluate. Vnew = (2l ) 3
4 Compare the answer obtained = 8l3
in step 3 with the volume of the
original shape.
5 Write your answer. Doubling each side length of a cube
increases the volume by a factor
of 8; that is, the new volume will
be 8 times as large as the original
volume.
b 1 Write the formula for the volume b V = πr2h
of the cylinder.
r
2 Identify the value of the rnew = , hnew = 2h
pronumerals. 2
Note: Halving is the same as
dividing by 2.
2
r
3 Substitute and evaluate. Vnew = πa b 2h
2
r2
= π× × 2h
24
πr2h
=
2
1 2
4 Compare the answer obtained = 2πr h
in step 3 with the volume of the
original shape.
5 Write your answer. Halving the radius and doubling the
height of a cylinder decreases the
volume by a factor of 2; that is, the
new volume will be half the original
volume.

Chapter 6 • Surface area and volume 219


measurement AND geometry

Volume of spheres
•• The volume of a sphere of radius r can be calculated using the
formula: V = 43 πr3. r

WORKED EXAMPLE 10
Find the volume of a sphere of radius 9 cm. Answer correct to 1 decimal
place.

THINK W R I TE

1 Write the formula for the volume of a sphere. V = 43πr3

2 Identify the value of r. r=9

3 Substitute and evaluate. V = 43 × π × 93


= 3053.6 cm3

Volume of pyramids
•• Pyramids are not prisms as their cross‐sections change from the
base upwards.

•• The volume of a pyramid is one‐third the volume of an equivalent prism with the
same base area and height.

    Volume of a pyramid = 13 AH      H

Area of base = A

Base

•• Since a cone is a pyramid with a circular cross‐section, the volume of


a cone is one‐third the volume of a cylinder with the same base area
and height.

Volume of a cone = 13AH


    
= 13πr2h h

220 Maths Quest 10 New South Wales Australian curriculum edition Stages 5.1 and 5.2
measurement AND geometry

WORKED EXAMPLE 11
Find the volume of each of the following solids.
a b

10 cm 12 cm

8 cm

8 cm

THINK W R I TE
1
a 1 Write the formula for the volume of a V = πr2h
3
a cone.
2 Identify the values of r and h. r = 8, h = 10

3 Substitute and evaluate. V = 13 × π × 82 × 10


= 670.21 cm3
1
b 1 Write the formula for the volume of b V = 3AH
a pyramid.
2 Find the area of the square base. A = l2 where l = 8
A = 82
= 64 cm2
3 Identify the value of H. H = 12

4 Substitute and evaluate. V = 13 × 64 × 12


= 256 cm3

Volume of composite solids


•• A composite solid is a combination of a number of smaller solids.
•• The volume of each smaller solid component can be calculated separately.
•• The volume of a composite solid is calculated by summing the volumes of each of
the smaller solid components.

WORKED EXAMPLE 12
Calculate the volume of the composite solid shown.

3m

1.5 m

Chapter 6 • Surface area and volume 221


measurement AND geometry

THINK W R I TE
1 The given solid is a composite figure, made up of a V = Volume of cube + Volume of pyramid
cube and a square‐based pyramid.
2 Find the volume of the cube. Vcube = l3 where l = 3
Vcube = 33
= 27 m3
3 Write the formula for finding the volume of a Vsquare-based pyramid = 13AH
square‐based pyramid.
4 Find the area of the square base. A = l2
= 32
= 9 m2
5 Identify the value of H. H = 1.5

6 Substitute and evaluate the volume of the pyramid. Vsquare-based pyramid = 13 × 9 × 1.5
= 4.5 m3
7 Find the total volume by adding the volume of the V = 27 + 4.5
cube and pyramid. = 31.5 m3

Capacity
•• Some 3‐dimensional objects are hollow and can be filled with liquid or some other
substance.
•• The amount of substance which a container can hold is called its capacity.
•• Capacity is essentially the same as volume but is usually measured in mL, L and kL
where 1 mL = 1 cm3
1 L = 1000 cm3
1k L = 1 m3.

WORKED EXAMPLE 13
Find the capacity (in litres) of a cuboidal aquarium, which
is 50 cm long, 30 cm wide and 40 cm high.

THINK W R I TE
1 Write the formula for the volume of a rectangular V = lwh
prism.
2 Identify the values of the pronumerals. l = 50, w = 30, h = 40

222 Maths Quest 10 New South Wales Australian curriculum edition Stages 5.1 and 5.2
measurement AND geometry

3 Substitute and evaluate. V = 50 × 30 × 40


= 60 000 cm3
4 State the capacity of the container in millilitres, 60 000 cm3 = 60 000 mL
using 1 cm3 = 1 mL.
5 Since 1 L = 1000 mL, to convert millilitres to litres = 60 L
divide by 1000.
6 Give a worded answer. The capacity of the fish tank is 60 L.

Exercise 6C Volume
INDIVIDUAL PATHWAYS

Questions: Questions: Questions:


1–4, 6–8, 9a, 10, 13, 14 1–8, 10–12, 14, 16 1–18
Activity 6‐C‐1 Activity 6‐C‐2 Activity 6‐C‐3
Review of volume and Volume and capacity Tricky volume and capacity
capacity problems problems
doc‐5041 doc‐5042 doc‐5043

FLUENCY
1 Find the volumes of the following prisms.
a b
Digital docs
SkillSHEET
Conversion of volume
doc‐5239
SkillSHEET
3 cm 4.2 m Volume of cubes and
rectangular prisms
doc‐5240

c 12 cm d

15 cm
4.2 cm
20 cm
7.5 cm
3 cm

2 Calculate the volume of each of these solids.


a b
18 mm
15 cm

[Base area: 25 mm2]


[Base area: 24 cm2]

Chapter 6 • Surface area and volume 223


measurement AND geometry

3 WE8 Find the volume of each of the following. Give each answer correct to
1 decimal place where appropriate.
a b

14 cm 2.7 m

12 cm 1.5 m

c d 12 mm
10 cm

7 cm
8 mm

8 cm 6 mm
e f
45 c
6.

m
5
m

35° 18 cm

7.1 m

4 WE10 Find the volume of a sphere (correct to 1 decimal place) with a radius of:
a 1.2 m b 15 cm c 7 mm d 50 cm.
5 Find the volume of each of these figures, correct to 2 decimal places.
a b

30 cm
1.4 m

c d
4.6 m

18 mm

6 WE11a Find the volume of each of the following cones, correct to 1 decimal place.
a b
22 mm
10 cm 20 mm

6 cm

224 Maths Quest 10 New South Wales Australian curriculum edition Stages 5.1 and 5.2
measurement AND geometry

7 WE11b Find the volume of each of the following pyramids.


a 12 cm b

42 cm

24 cm

10 cm
30 cm
8 WE12 Calculate the volume of each of the following composite solids correct to
2 decimal places where appropriate.
a 8 cm b
10 cm
5 cm
12 cm
5 cm

20 cm 20 cm

35 cm
12 cm

c d

2 cm
m
5 cm 2.5 c
3 cm

3 cm

e f 5 cm

3.5 cm
20 cm

10 cm
12 cm

15 cm

UNDERSTANDING
9 WE9 a What effect will tripling each of the side lengths of a cube have on its
volume?
b What effect will halving each of the side lengths of a cube have on its volume?
c What effect will doubling the radius and halving the height of a cylinder have on
its volume?
d What effect will doubling the radius and dividing the height of a cylinder by 4
have on its volume?
e What effect will doubling the length, halving the width and tripling the height of a
rectangular prism have on its volume?

Chapter 6 • Surface area and volume 225


measurement AND geometry

10 MC A hemispherical bowl has a thickness of 2 cm and an outer 2 cm


diameter of 25 cm. If the bowl is filled with water then the capacity
of the water will be closest to:
A 1.526 L B 1.308 L C 3.052 L
D 2.617 L E 2.425 L 25 cm
11 Tennis balls of diameter 8 cm are packed in a box
40 cm × 32 cm × 10 cm, as shown. How much space is
left unfilled?

12 WE13 A cylindrical water tank has a diameter of 1.5 m and a height


of 2.5 m. What is the capacity (in litres) of the tank?
REASONING
13 The volume of a cylinder is given by the formula V = πr2h.
a Transpose the formula to make h the subject.
b A given cylinder has a volume of 1600 cm3. Find its height if it has a
radius of:
i 4 cm ii 8 cm.
c Transpose the formula to make r the subject.
d What restrictions must be placed on r? Why?
e A given cylinder has a volume of 1800 cm3. Find its radius if it has a
height of:
i 10 cm ii 15 cm.
14 A toy maker has enough rubber to make one super‐ball
of radius 30 cm. How many balls of radius 3 cm can
he make from this rubber?

15 A manufacturer plans to make a cylindrical water tank to hold 2000 L of water.


a What must the height be if he uses a radius of 500 cm?
b What must the radius be if he uses a height of 500 cm?
c What will be the surface area of each of the two tanks?
16 The ancient Egyptians knew that
the volume of the frustum of a y
pyramid was given by the formula
V = 13h(x2 + xy + y2) although how they h
discovered this is unclear.

226 Maths Quest 10 New South Wales Australian curriculum edition Stages 5.1 and 5.2
measurement AND geometry

a Find the volume of the frustum shown at right.


b What would be the volume of the missing portion of the
pyramid?

4m
5m

6m

17 Archimedes is considered to be one of the three greatest mathematicians


of all time (along with Newton and Gauss). He discovered several of the
formulas used in this chapter. Inscribed on his tombstone was a diagram of
his proudest discovery. It shows a sphere inscribed (fitting exactly) into a
cylinder.
volume of the cylinder surface area of the cylinder
Show that = .
volume of the sphere surface area of the sphere
18 Marion has mixed together ingredients for a cake. The recipe requires a baking
tin that is cylindrical in shape with a diameter of 20 cm and a height of 5 cm.
Marion only has a tin as shown and a muffin tray consisting of 24 muffin cups.
Each of the muffin cups in the tray is a portion of a cone as shown in the
diagram.
Should Marion use the tin or muffin tray? Explain.
Digital doc
WorkSHEET 6.3
12 cm
doc‐6733

8 cm

REFLECTION
4 cm
Volume is measured in
4 cm 10 cm cubic units. How is this
15 cm
reflected in the volume
formula?
     8 cm

CHALLENGE 6.2

Chapter 6 • Surface area and volume 227


measurement AND geometry

CHAPTER REVIEW
LANGUAGE

area cube prism sphere


capacity cylinder pyramid square
circle ellipse rectangle surface
composite figure face rhombus trapezium
cone hemisphere sector triangle
cross‐section parallelogram semi‐perimeter volume

int‐2841 doc‐13721
int‐2842 doc‐13722
int‐3593

FLUENCY
1 If all measurements are in cm, the area of the figure below is:

7 3

A 16.49 cm2 B 39.25 cm2 C 9.81 cm2 D 23.56 cm2 E 30 cm2


2 If all measurements are in centimetres, the area of the figure at right is: 6
A 50.73 cm2 5
B 99.82 cm2
C 80.18 cm2 5
D 90 cm2
E 119.45 cm2 5

3 If all measurements are in centimetres, the shaded area of the figure below is:

30°

2
7

A 3.93 cm2 B 11.52 cm2 C 388.77 cm2 D 141.11 cm2 E 129.59 cm2

228 Maths Quest 10 New South Wales Australian curriculum edition Stages 5.1 and 5.2
measurement AND geometry

4 The total surface area of the solid below is:

28 mm

40 mm

A 8444.6 mm2 B 9221 mm2 C 14146.5 mm2


D 50271.1 mm2 E 16609.5 mm2
5 Find the areas of the following plane figures. All measurements are in cm.
a b 10 c
3
8 7
14
15
5

12

d e f
3
10
80° 10
6

12

6 Find the areas of the following figures. All measurements are in cm.
a b c 10
12
15 6
10
10 5
20
20

7 Find the shaded area in each of the following. All measurements are in cm.
a Q QO = 15 cm
b c
5
SO = 8 cm
PR = 18 cm
12.5
O

R
P S

Chapter 6 • Surface area and volume 229


measurement AND geometry

8 Find the total surface area of each of the following solids.


a 35 cm b 14 mm c

50 cm
20 mm 8 cm

d 14 cm e 10 mm f
10 mm 12 cm
14 mm 4 mm
18 cm
10 cm

[closed at both ends]


12 cm 10 cm
10 cm
9 Find the volume of each of the following.
a b c 35 cm
7 cm

40 cm
8 cm

12 cm
7 cm
d e f

12 cm
10 cm
3.7 m
30 cm
1m 10 cm
12 cm

g h i
11 cm 30 cm
12 mm

20 cm

42 cm
9 cm

PROBLEM SOLVING
1 A rectangular block of land 4 m × 25 m is surrounded by a concrete path 1 m wide.
a Calculate the area of the path.
b Calculate the cost of concreting at $45 per square metre.
2 What effect will tripling the radius and dividing the height of a cylinder by 6 have on
its volume (in comparison with the original shape)?
3 What effect will halving the length, tripling the width and doubling the height of a
rectangular prism have on its volume (in comparison with the original shape)?

230 Maths Quest 10 New South Wales Australian curriculum edition Stages 5.1 and 5.2
measurement AND geometry

4 A cylinder of radius 14 cm and height 20 cm is joined to a hemisphere of radius


14 cm to form a bread holder.
a Find the total surface area.
b Find the cost of chroming the bread holder on the outside at $0.05 per cm2.
c What is the storage volume of the bread holder?
d How much more space is in this new bread holder than the one it is replacing,
which had a quarter circle end with a radius of 18 cm and a length of 35 cm?
5 Bella Silos has two rows of silos for storing wheat. Each row has 16 silos and all
the silos are identical, with a cylindrical base (height of 5 m, diameter of 1.5 m) and
conical top (diameter of 1.5 m, height of 1.1 m).
a What is the slant height of the conical tops?
b What is the total surface area of all the silos?
c What will it cost to paint the silos if one litre of paint covers 40 m2 at a bulk order
price of $28.95 per litre?
d How much wheat can be stored altogether in these silos?
e Wheat is pumped from these silos into cartage trucks with rectangular containers
2.4 m wide, 5 m long and 2.5 m high. How many truckloads are necessary to
empty all the silos?
f If wheat is pumped out of the silos at 2.5 m3/min, how long will it take to fill one
truck?
6 The Greek mathematician Eratosthenes
developed an accurate method for calculating
the circumference of the Earth 2200 years
ago! The figure below illustrates how he did
this. In this figure, A is the town of Alexandria
and S is the town of Syene, exactly 787  km
due south. When the sun’s rays (blue lines)
were vertical at Syene, they formed an angle
of 7.2° at Alexandria (∠BVA = 7.2°), obtained
by placing a stick at A and measuring the
angle formed by the sun’s shadow with
the stick.

V
B
A
S

a Assuming that the sun’s rays are parallel, what is the angle ∠SCA?
b Given that the arc AS = 787  km, determine the radius of the Earth, SC.
c Given that the true radius is 6380  km, determine Eratosthenes’ percentage error.

Chapter 6 • Surface area and volume 231


measurement AND geometry
communicating

Rich task

So close!

Measurement errors
When we measure a quantity by using a
scale, the accuracy of our measurement
depends on the markings on the scale. For
example, the ruler shown can measure both
in centimetres and millimetres.
Measurements made with this ruler would
have ± 0.5 mm added to the measurement.
The quantity ± 0.5 is called the tolerance of
measurement or measurement error.
1
Tolerance of measurement = × size of smallest marked unit
2
For a measurement of 5.6 ± 0.5 mm, the largest possible value is 5.6 cm + 0.5mm = 5.65 cm,
and the smallest value is 5.6 cm − 0.5 mm = 5.55 cm.

232 Maths Quest 10 New South Wales Australian curriculum edition Stages 5.1 and 5.2
measurement AND geometry
MEASUREMENT GEOMETRY

1 For the thermometer scale at right:


a determine the temperature ˚c
45
b state the measurement with its tolerance
c determine the largest and smallest possible values. 40

2 Calculate the largest and smallest values for: 35


a 1 56.2 ± 0.1 2 − 1 19.07 ± 0.05 2 b 1 78.4 ± 0.25 2 × 1 34 ± 0.1 2 .
30
Significant figures in measurement
25
A significant figure is any non zero-digit, any zero appearing between two non-zero
digits, any trailing zeros in a number containing a decimal point, and any digits in the 20

decimal places. For example, the number 345.6054 has 7 significant figures, whereas 15
300 has 1 significant figure.
The number of significant figures is an expression of the accuracy of a measurement.
The greater the number of significant figures, the more accurate the measurement. For
example, a fast food chain claims it has sold 6  000  000  000 hamburgers, not 6  453  456  102. The
first measurement has only 1 significant figure and is a very rough approximation of the actual
number sold, which has 10 significant figures.
Reducing the number of significant figures is a process that is similar to rounding.
Rounding and measurement error in calculations
When you perform calculations, it is important to keep as many significant digits as practical,
and to perform any rounding as the final step. For example, calculating 5.34 × 341 by rounding
to 2 significant figures before multiplying gives 5.30 × 340 = 1802, compared with 1820 if the
rounding is carried out after the multiplication.
Calculations that involve numbers from measurements containing errors can result in answers
with even larger errors. The smaller the tolerances, the more accurate the answers will be.
3 a Calculate 45  943.450  3 × 86.765  303 by:
    i first rounding each number to 2 significant figures
ii rounding only the answer to 2 significant figures.
b Compare the two results.
Error in area and volume resulting from an error in a length measurement
The side length of a cube is measured and incorrectly recorded as 5 cm. The actual
size is 6 cm. The effect of the length measurement error used on calculations of
the surface area is shown at right. Complete the calculations for volume.
Error used in length measurement = 1 cm
Surface area calculated with incorrectly recorded value = 52 × 6 = 150 cm2
Surface area calculated with actual value = 62 × 6 = 216 cm2
216 − 150
Percentage error = × 100% ≈ 30.5%
6
4 a 
Complete a similar calculation for the volume of the cube using the incorrectly recorded length.
What conclusion can you make regarding errors when the number of dimensions increase?
b Give three examples of a practical situation where an error in measuring or recording would
have a potentially disastrous impact.

Chapter 6 • Surface area and volume 233


measurement AND geometry

Code puzzle

Australian inventions!
The answers to the measurement problems give the puzzle’s code.

20 4.5 56 30 4.5 28.27 67.5 92 120 4.5 27 50.27 4 92 30 320

13 cm 2m
8 cm 3m

10 cm 400 cm
700 cm
3
2 Volume = G m (2 d.p.) 3
Area = A cm Volume = O m
11 m
2 mm 12 m 8m
8 cm
2 mm 2
5 mm
Area = H m
3 area = 3 cm2 2
Volume = B mm Area = P cm (2 d.p.)

3m 15 mm

Volume = I cm3 6m
8m
2 5m 3
Area = C m (2 d.p.) 3 cm 4 cm Volume = R m

3m
Area = K mm2 16 mm
1m
2 cm
1m
3 2
Volume = D m Area = T mm
20 mm
Area = L cm2 5 cm

9 cm 6 cm
3m
15 cm
Area = E cm2 3
Volume = X cm3 (2 d.p.)
Volume = M m
8 cm
5 cm 12 m
10 m
6m
5m 13 m 8m

Volume = F cm3 (2 d.p.) Area = N m


2
Area = Y m2

20 4 92 28.27 1200 20 56 471.24 125.66 4 4.5 14.14 110 320

27 67.5 27 56 120 24 120 67.5 28.27 56 120 1.5 67.5 120

234 Maths Quest 10 New South Wales Australian curriculum edition Stages 5.1 and 5.2
measurement AND geometry

activities
Go to assessON
Chapter opener 6C Volume for questions to
video Digital docs test your readiness
• The story of mathematics (eles‐1845) • Activity 6‐C‐1 (doc‐5041): Review of volume FOR learning, your
and capacity (page 223) progress AS you learn
6A Area
• Activity 6‐C‐2 (doc‐5042): Volume and and your levels OF
Digital docs
capacity problem (page 223) achievement.
• Activity 6‐A‐1 (doc‐5035): Review of
• Activity 6‐C‐3 (doc‐5043): Tricky volume and www.assesson.com.au
area (page 204)
capacity problems (page 223)
• Activity 6‐A‐2 (doc‐5036): Area
• SkillSHEET (doc‐5239): Conversion of volume
problems (page 204)
units (page 223)
• Activity 6‐A‐3 (doc‐5037): Tricky area
• SkillSHEET (doc‐5240): Volume of cubes and
problems (page 204)
rectangular prisms (page 223)
• SkillSHEET (doc‐5236): Conversion of area
• WorkSHEET 6.3 (doc‐6733):
units (page 204)
Volume (page 227)
• SkillSHEET (doc‐5237): Using a
Interactivity
formula to find the area of a common
• Maximising the volume of a cuboid
shape (page 204)
(int‐1150) (page 217)
• WorkSHEET 6.1 (doc‐5241): Area (page 208)
eLesson Chapter review
• Heron’s formula (eles‐0177) (page 201) Interactivities (page 228)
• Word search (int‐2841)
6B Total surface area
• Crossword (int‐2842)
Interactivity
• Sudoku (int‐3593)
• TSA — sphere (int‐2782) (page 209)
Digital docs
Digital docs
• Chapter summary (doc‐13721)
• Activity 6‐B‐1 (doc‐5038): Introducing surface
• Concept map (doc‐13722)
area (page 214)
• Activity 6‐B‐2 (doc‐5039): Surface area To access eBookPLUS activities, log on to
problems (page 214) www.jacplus.com.au
• Activity 6‐B‐3 (doc‐5040): Tricky surface
area problems (page 214)
• SkillSHEET (doc‐5238): Total surface area of
cubes and rectangular prisms (pages 209,
214)
• WorkSHEET 6.2 (doc‐5242): Surface
area (page 217)

Chapter 6 • Surface area and volume 235


measurement AND geometry

ANSWERS
Chapter 6 Surface area and volume
Exercise 6A — Area 5    a 13.5 m2 b 90 m2 c 11 309.7 cm2
1    a 16 cm2 b 48 cm2 c 75 cm2 d 9852.0 mm2 e 125.7 cm2 f 1531.4 cm2
2 2
d 120 cm2 e 706.86 cm2 f 73.5 mm2 6    a 880 cm b 3072.8 cm c 75 cm2
g 254.47 cm2 h 21 m2 i 75 cm2 d 70.4 cm2 e 193.5 cm2 f 1547.2 cm2
2 Part e = 225π cm2; part g = 81π cm2 7 B 8 60
3    a 20.7 cm2 b 7.64 cm2 9 11216 cm2
4    a 113.1 mm2 b 188.5 mm2 10    a 70.0 m2 b $455
5    a i 12π cm 2 ii 37.70 cm2 11    a 3063.1 cm2 b $168.47
69π 12    a 297.06 cm2 b 423 cm2
b i mm2 ii 108.38 mm2 c 15 080 cm 2 d 8989 cm2
2
13    a 221 cm2 b 261 cm2
c i 261π cm2 ii 819.96 cm2
6 E Exercise 6C — Volume
7 D 1    a 27 cm3 b 74.088 m3
8    a 123.29 cm2 b 1427.88 m2 c 52 cm2 c 3600 cm3 d 94.5 cm3
d 30.4 m 2 e 78 cm 2 f 2015.5 cm2 2    a 450 mm3 b 360 cm2
9    a 125.66 cm2 b 102.87 m2 c 13.73 m2 3    a 6333.5 cm3 b 19.1 m3
2 2 c 280 cm 3 d 288 mm3
d 153.59 m e 13.86 m f 37.5 m2
10 11 707.92 cm2 11 21 m2 12 60 e 91.6 m3 f 21 470.8 cm3
4    a 7.2 m 3 b 14 137.2 cm3
13 $840
14    a 260.87 m2 b 195.71 m2 c 75% c 1436.8 mm3 d 523 598.8 cm3
15    a 50 = x + y b y = 50 − x 5    a 11 397.34 cm 3 b 1.44 m3
c Area = 50x − x2 c 12 214.51 mm3
e d 101.93
d 6    a 377.0 cm3 b 2303.8 mm3
x 0 5 10 15 20 25 30 35 40 45 50 7    a 400 cm3 b 10 080 cm3
2) 0 225 400 525 600 625 600 525 400 225 0 8    a 1400 cm3 b 10 379.20 cm3 c 41.31 cm3
Area (m d 48.17 cm 3
e 218.08 cm 3
f 3691.37 cm3
e No, impossible to make a rectangle. 9    a Vnew = 27l3, the volume will be 27 times as large as the
f original volume.
600
500 b Vnew = 18l2, the volume will be 18 of the original volume.
400 c Vnew = 2πr2h, the volume will be twice as large as the original
Area

300
200
volume.
100 d Vnew = πr2h, the volume will remain the same.
0 10 20 30 40 50 60 70 x e Vnew = 3lwh, the volume will be 3 times as large as the
original value.
g x = 25 h y = 25 10 E
i Square j 625 m2 11 7438.35 cm3
k r = 15.915 m l 795.77 m2 12 4417.9 L
m 170.77 m2
V
16    a Students’ work 13    a h =
b 2020.83 m; horizontal πr2
17    a Circular area, 1790.49 m2; rectangular area, 1406.25 m2 b i 31.8 cm ii 8.0 cm
1 c
b Circular area, a n2 b m2; rectangular (square) area, V
4π Å πh
4 1
1 1 n2 2 m2. Circular area is always or 1.27 a ÷ 1 b d r ≥ 0, since r is a length
16 π 4π 16
times larger. e i 7.6 cm ii 6.2 cm
14 1000
Challenge 6.1 15    a 0.02546 m b 0.3568 m
29 c Aa = 157.88 m2, Ab = 12.01 m2
50 16    a 126.67 m3
Exercise 6B — Total surface area b 180 m3
1    a 600 cm2 b 384 cm2 c 1440 cm2 17 Answers will vary.
d 27 m2 18 Required volume = 1570.80 cm3; tin volume = 1500 cm3; muffin
2    a 113.1 m2 b 6729.3 cm2 c 8.2 m2 tray volume = 2814.72 cm3. Marion should use the tin with
d 452.4 cm2 approximately 70 cm3 mixture left over.
3    a 1495.4 cm2 b 502.7 cm2 Challenge 6.2
4    a 506.0 cm2 b 9.4 m2 c 340.4 cm2 18 scoops
d 224.1 cm 2

236 Maths Quest 10 New South Wales Australian curriculum edition Stages 5.1 and 5.2
measurement AND geometry

Chapter review Communicating — Rich task


Fluency 1    a The temperature reading is 26.5°C.
1 D 2 C 3 E 4 A b The smallest unit mark is 1°C, so the tolerance is 0.5.
5    a 84 cm2 b 100 cm2 c 6.5 cm2 c Largest possible value = 27°C, smallest possible value = 26°C
d 56.52 cm2 e 60 cm2 f 244.35 cm2 2    a Largest value = 37.28, smallest value = 36.98
6    a 300 cm2 b 224.52 cm2 c 160 cm2 b Largest value = 2681.965, smallest value = 2649.285
7    a 499.86 cm2 b 44.59 cm2 c 128.76 cm2 3    a     i 4  002  000
8    a 18 692.48 cm2 b 1495.40 cm2 c 804.25 cm2 ii 4  000  000
d 871.79 cm2 e 873.36 mm2 f 760 cm2 b The result for   i has 4 significant figures, whereas ii has only
9    a 343 cm3 b 672 cm3 c 153 938.04 cm3 1 significant figure after rounding. However,   ii is closer to the
d 1.45 m3 e 1800 cm3 f 1256.64 cm3 actual value (3  986  297.386  144  940  9).
g 297 cm3 h 8400 cm3 i 7238.23 mm3 4    a Volume using the incorrectly recorded value = 125 cm3
Problem solving Volume using the actual value = 216 cm3
1    a 62 m2 b $7290 The percentage error is 42.1%, which shows that the error
compounds as the number of dimensions increases.
2 V = 32πr2h, the volume will be 1.5 times as large as the original
b Check with your teacher.
volume. Code puzzle
3 V = 3lwh, the volume will be 3 times as large as (or triple) the Bionic ear implant
original volume. Black box flight memory recorder
4    a 3605.55 cm2 b $180.33
c 18062.1 cm3 d 9155.65 cm3
5    a 1.33 m b 910.81 m2 c $655.85
d 303.48 m3 e 11 trucks f 12 minutes
6    a 7.2° b 6263  km c 1.8% error

Chapter 6 • Surface area and volume 237


number and
NUMBER AND ALGEBRA
algebra

Chapter 7

Quadratic
expressions
WHY LEARN THIS?
How is your algebraic tool kit? Is there some room
to expand your skills? As expressions become more
complex, more power will be needed to manipulate
them and to carry out basic skills such as adding,
multiplying and factorising. Dealing with quadratic
expressions is the first step to higher‐level skills.

WHAT DO YOU KNOW?


1 Think List what you know about quadratic
expressions. Use a thinking tool such as
a concept map to show your list.
2 pair Share what you know with a partner
and then with a small group.
3 share As a class, create a large
concept map that shows your class’s
knowledge of quadratic expressions.

LEARNING SEQUENCE
7A Expanding algebraic expressions
7B Factorising expressions with three terms
7C Factorising expressions with two or four terms
7D Mixed factorisation
Watch this video
The story of mathematics

Searchlight ID: eles-1846


number and algebra

7A  Expanding algebraic expressions


Binomial expansion
•• Consider the rectangle shown below. Its length is a + b and its width is c + d. Its
area is equal to (a + b)(c + d ).
a b
c ac bc

d ad bd

The diagram shows that 1 a + b 2 1 c + d 2 = ac + ad + bc + bd.

v
u
factorised form expanded form

•• Expansion of the binomial expression (x + 3)(x + 2) can be shown by this area


model.
x 3

3×x
x x × x = x2
= 3x

3×2
2 2 × x = 2x
=6

Expressed mathematically this is:


1 x + 3 2 1 x + 2 2 = x2 + 2x + 3x + 6
= x2 + 5x + 6
u

factorised form expanded form


•• There are several methods that can be used to expand two binomial factors.

FOIL method
•• The word FOIL provides us with an acronym for the expansion of a binomial
product.
F
•• First: multiply the first terms in each bracket.
(x + a)(x − b)
O
•• Outer: multiply the two outer terms.
(x + a)(x − b)
I
•• Inner: multiply the two inner terms.
(x + a)(x − b)
L
•• Last: multiply the last terms in each bracket.
(x + a)(x − b)

240 Maths Quest 10 New South Wales Australian curriculum edition Stages 5.1 and 5.2
number and algebra

WORKED EXAMPLE 1
Expand each of the following.
a (x + 3)(x + 2) b (x − 7 )(6 − x)

TH I N K W R I TE
a 1 Write the expression. a (x + 3)(x + 2)

2 Multiply the first terms in each bracket, = x2 + 2x + 3x + 6


then the outer terms, then the inner terms
and finally the last terms.
3 Collect like terms. = x2 + 5x + 6
b 1 Write the expression. b (x − 7)(6 − x)
2 Multiply the first terms in each bracket, = 6x − x2 − 42 + 7x
then the outer terms, then the inner terms
and finally the last terms.
3 Collect like terms. = −x2 + 13x − 42

•• If there is a term outside the pair of brackets, expand the brackets and then multiply
each term of the expansion by that term.

WORKED EXAMPLE 2
Expand 3(x + 8)(x + 2).

TH I N K W R I TE

1 Write the expression. 3(x + 8)(x + 2)

2 Use FOIL to expand the pair of brackets. = 3(x2 + 2x + 8x + 16)


3 Collect like terms within the brackets. = 3(x2 + 10x + 16)
4 Multiply each of the terms inside the brackets = 3x2 + 30x + 48
by the term outside the brackets.

The square of a binomial


•• The expansion of (a + b) 2 can be represented by this area model.
a b

a×b
a a × a = a2 = ab

a × b = ab b×b
b
= b2

Chapter 7 • Quadratic expressions 241


number and algebra

(a + b) 2 = a2 + ab + ab + b2
= a2 + 2ab + b2
•• Similarly (a − b) 2 = a2 − 2ab + b2.
•• This expansion is often memorised. To find the square of a binomial:
–– square the first term
–– multiply the two terms together and then double them
–– square the last term.

WORKED EXAMPLE 3
Expand and simplify each of the following.
a (2x − 5) 2 b − 3(2x + 7) 2

TH I N K W R I TE
a 1 Write the expression. a (2x − 5) 2

2 Expand using the rule = (2x) 2 − 2 × 2x × 5 + (5) 2


(a − b) 2 = a2 − 2ab + b2. = 4x2 − 20x + 25
b 1 Write the expression. b − 3(2x + 7) 2
2 Expand the brackets using the rule = −3[(2x) 2 + 2 × 2x × 7 + (7) 2]
(a + b) 2 = a2 + 2ab + b2. = −3(4x2 + 28x + 49)
3 Multiply every term inside the = −12x2 − 84x − 147
brackets by the term outside the
brackets.

The difference of two squares


•• When a + b is multiplied by a − b (or vice versa)
(a + b)(a − b) = a2 − ab + ab − b2
= a2 − b2
the expression is called the difference of two squares and is often referred to as DOTS.
•• The result can be memorised as a short cut.
WORKED EXAMPLE 4
Expand and simplify each of the following.
a (3x + 1)(3x − 1) b 4(2x − 7)(2x + 7)

TH I N K W R I TE
a 1 Write the expression. a (3x + 1)(3x − 1)
2 Expand using the rule (a + b)(a − b) = a2 − b2. = (3x) 2 − (1) 2
= 9x2 − 1
b 1 Write the expression. b 4(2x − 7)(2x + 7)
2 Expand using the difference of two squares rule. = 4[(2x) 2 − (7) 2]
= 4(4x2 − 49)
3 Multiply by 4. = 16x2 − 196

242 Maths Quest 10 New South Wales Australian curriculum edition Stages 5.1 and 5.2
number and algebra

Exercise 7A Expanding algebraic expressions


INDIVIDUAL PATHWAYS
Questions: Questions: Questions:
1a–f, 2a–h, 3a–c, 5a, c, 6a, 1d–l, 2d–j, 3a–d, 4a–c, 5a–c, 1d–l, 2f–l, 3b–f, 4, 5, 6, 7, 8c–i,
8a–e, 9a,b, 10a–f, 11–14 6, 7, 8c–h, 9c,d, 10–17 9e, f, 10–18
Activity 7‐A‐1 Activity 7‐A‐2 Activity 7‐A‐3
Review of expansion Expanding algebraic Expanding more complex
doc‐5044 expressions algebraic expressions
doc‐5045 doc‐5046

FLUENCY
1 Expand each of the following.
a 2(x + 3) b 4(x − 5) c 3(7 − x)
d −(x + 3) e x(x + 2) f 2x(x − 4) Digital docs
g 3x(5x − 2) h 5x(2 − 3x) i 2x(4x + 1) SkillSHEET
Expanding brackets
j 2x (2x − 3)
2 k 3x (2x − 1)
2 l 5x2 (3x + 4) doc‐5244
SkillSHEET
2 WE1 Expand each of the following. Expanding a pair of
a (x + 3)(x − 4) b (x + 1)(x − 3) c (x − 7)(x + 2) brackets
doc‐5245
d (x − 1)(x − 5) e (2 − x)(x + 3) f (x − 4)(x − 2)
g (2x − 3)(x − 7) h (x − 1)(3x + 2) i (3x − 1)(2x − 5)
j (3 − 2x)(7 − x) k (5 − 2x)(3 + 4x) l (11 − 3x)(10 + 7x)
3 WE2 Expand each of the following.
a 2(x + 1)(x − 3) b 4(2x + 1)(x − 4) c −2(x + 1)(x − 7)
d 2x(x − 1)(x + 1) e 3x(x − 5)(x + 5) f 6x(x − 3)(x + 3)
4 Expand each of the following.
a (x − 1)(x + 1)(x + 2) b (x − 3)(x − 1)(x + 2) c (x − 5)(x + 1)(x − 1)
d (x − 1)(x − 2)(x − 3) e (2x − 1)(x + 1)(x − 4) f (3x + 1)(2x − 1)(x − 1)
5 Expand each of the following and simplify.
a (x + 2)(x − 1) − 2x b 3x − (2x − 5)(x + 2)
c (2x − 3)(x + 1) + (3x + 1)(x − 2) d (x − 2)(x − 5) − (x − 1)(x − 4)
6 MC a (3x − 1)(2x + 4) expands to:
A 6x2 + 10x − 4 B 5x2 − 24x + 3 C 3x2 + 2x − 4
D 6x2 − 10x − 4 E 6x2 − 4
b −2x(x − 1)(x + 3) expands to:
A x2 + 2x − 3 B −2x2 − 4x + 6 C −2x3 − 4x2 + 6x
D −2x3 + 4x2 − 6x E −2x3 − 3
7 MC The expression (x − 1)(x − 3)(x + 2) is not the same as:
A (x − 3)(x − 1)(x + 2) B (x + 3)(x − 1)(x − 2) C (x − 1)(x + 2)(x − 3)
D (x + 2)(x − 1)(x − 3) E (x − 3)(x + 2)(x − 1)
8 WE3a Expand and simplify each of the following.
a (x − 1) 2 b (x + 2) 2 c (x + 5) 2
d (4 + x) 2 e (7 − x) 2 f (12 − x) 2
g (3x − 1) 2 h (12x − 3) 2 i (5x + 2) 2

Chapter 7 • Quadratic expressions 243


number and algebra

9 WE3b Expand and simplify each of the following.


a 2(x − 3) 2 b 4(x − 7) 2 c 3(x + 1) 2
d −(2x + 3) 2 e −(7x − 1) 2 f 2(2x − 3) 2
10 WE4 Expand and simplify each of the following.
a (x + 7)(x − 7) b (x + 9)(x − 9) c (x − 5)(x + 5)
d (x − 1)(x + 1) e (2x − 3)(2x + 3) f (3x − 1)(3x + 1)
g (7 − x)(7 + x) h (8 + x)(8 − x) i (3 − 2x)(3 + 2x)

UNDERSTANDING
11 The length of the side of a rectangle is (x + 1) cm and the width is (x − 3) cm.
a Find an expression for the area of the rectangle.
b Simplify the expression by expanding.
c If x = 5 cm, find the dimensions of the rectangle and, hence, its area.
12 Chickens are kept in a square enclosure with sides measuring x m. The number of
chickens is increasing and so the size of the enclosure is to have 1 metre added to
one side and 2 metres to the adjacent side.
a Draw a diagram of the original enclosure.
b Add to the first diagram or draw another one to show the new enclosure. Mark
the lengths on each side on your diagram.
c Write an expression for the area of the new enclosure, in factorised form.
d Expand and simplify the expression.
e If the original enclosure had sides of 2 metres, find the area of the original
square and then the area of the new enclosure.
13 Shown below are three students’ attempts at expanding (3x + 4)(2x + 5).
STUDENT A STUDENT B

STUDENT C

a Which student’s work was correct?


b Copy each of the incorrect answers into your workbook and correct the mistakes
in each as though you were the teacher of these students.

244 Maths Quest 10 New South Wales Australian curriculum edition Stages 5.1 and 5.2
number and algebra

14 If a = 5 and b = 3, show that (a − b)(a + b) = a2 − b2 by evaluating both


expressions.
15 If a = 5 and b = 3, show that (a + b) 2 = a2 + 2ab + b2 by evaluating both
expressions.
16 Write an expression in factorised and expanded form that is:
a a quadratic trinomial b the square of a binomial
c the difference of two squares d both a and b.

REASONING REFLECTION
17 Explain the difference between ‘the square of a binomial’ and ‘the Why does the Difference of
Two Squares rule have that
difference between two squares’.
name?
18 Show that (a + b)(c + d ) = (c + d )(a + b).

7B  Factorising expressions with three terms


Factorising monic quadratic trinomials
•• A monic quadratic expression is ax2 + bx + c where a = 1.
–– The area model of binomial expansion can be used to find a pattern for factorising
a general quadratic expression. For example,
(x + f )(x + h) = x2 + fx + hx + f h (x + 4)(x + 3) = x2 + 4x + 3x + 12
= x2 + ( f + h)x + f h = x2 + 7x + 12
x + f x + 4

x x2 fx x x2 4x

+ +
h hx fh 3 3x 12

Chapter 7 • Quadratic expressions 245


number and algebra

–– To factorise a general quadratic, look for factors of c that sum to b.


x2 + bx + c = (x + f ) (x + h)

Factors of c that sum to b


3+4=7
For example, x2 + 7x + 12 = (x + 3)(x + 4) .
3 × 4 = 12

WORKED EXAMPLE 5
Factorise the following quadratic expressions.
a x2 + 5x + 6 b x2 + 10x + 24

TH I N K W R I TE
a 1 Write the expression and: a x2 + 5x + 6
i check for a common factor. There is
no common factor.
ii check for a DOTS pattern. The
expression is not in the form a2 − b2.
iii check for a perfect squares pattern.
6 is not a perfect square.
This must be a general quadratic
expression.
2 • The general quadratic Factors of 6 Sum of factors
expression has the pattern
x2 + 5x + 6 = (x + f )(x + h). 1 and 6 7
f and h are a factor pair of 6 which 2 and 3 5
add to 5.
• Calculate the sums of factor pairs
of 6. The factors of 6 that add to 5
are 2 and 3, as shown in blue.
3 Substitute the values of f and h into x2 + 5x + 6 = (x + 2)(x + 3)
the expression in its factorised form.
b 1 Check for patterns of common factors, b x2 + 10x + 24
DOTS and perfect squares patterns.
None of these apply, so the expression
is a general quadratic.
2 • The general quadratic Factors of 24 Sum of factors
expression has the pattern
x2 + 10x + 24 = (x + f)(x + h), 1 and 24 25
where f and h are a factor pair of 24 2 and 12 14
which add to 10. 3 and 8 11
• Calculate the sums of factor pairs
4 and 6 10
of 24. The factors of 24 that add to
10 are 4 and 6, as shown in blue.
3 Substitute the values of f and h into x2 + 10x + 24 = (x + 4)(x + 6)
the expression in its factorised form.

246 Maths Quest 10 New South Wales Australian curriculum edition Stages 5.1 and 5.2
number and algebra

•• When minus signs are present, negative factors of the constant term c need to be
considered.

WORKED EXAMPLE 6
Factorise the following simple trinomials.
a x2 − 9x + 18
b x2 + 6x − 16

TH I N K W R I TE
a 1 Check for patterns of common a x2 − 9x + 18
factors, DOTS and perfect squares
patterns. None of these apply,
so the expression is a general
quadratic.
2 • The general quadratic Factors of 18 Sum of factors
expression has the pattern
x2 − 9x + 18 = (x + f )(x + h), 1 and 18 19
−1 and −18 −19
where f and h are a factor pair
of 18 which add to −9. 2 and 9 11
• Calculate the sums of factor −2 and −9 −11
pairs of 18. As shown in blue,

3 and −6 are factors of 18 that 3 and 6 9

add to −9. 3 and −6 −
9

3 Substitute the values of f and h x2 − 9x + 18 = (x − 3)(x − 6)


into the expression in its
factorised form.
b 1 Check for patterns of common b x2 + 6x − 16
factors, DOTS and perfect squares
patterns. None of these apply,
so the expression is a general
quadratic.
2 • The general quadratic Factors of −16 Sum of factors
expression has the pattern
x2 + 6x − 16 = (x + f )(x + h), 1 and −16 −15

−1 and 16 15
where f and h are a factor pair
of −16 which add to 6. 2 and −8 −6
• Calculate the sums of factor −2 and 8
6
pairs of −16. As shown in blue,
− 2 and 8 are factors of −16 that

add to 6.
3 Substitute the values of f and h x2 + 6x − 16 = (x − 2)(x + 8)
into the expression in its
factorised form.

Chapter 7 • Quadratic expressions 247


number and algebra

Exercise 7B Factorising expressions with three terms


INDIVIDUAL PATHWAYS
Questions: Questions: Questions:
1–3, 5, 6 1–7 1–8
Activity 7‐B‐1 Activity 7‐B‐2 Activity 7‐B‐3
Introducing quadratic Practising quadratic Tricky quadratic
factorisation factorisation factorisation
doc‐5047 doc‐5048 doc‐5049

FLUENCY
Digital docs
1 WE5 Factorise each of the following.
SkillSHEET a x2+ 3x + 2 b x2 + 4x + 3 c x2 + 10x + 16
Factorising by ­taking
out the highest d x2 + 8x + 16 e x2 − 2x − 3 f x2 − 3x − 4
­common factor
doc‐5246
g x2 − 11x − 12 h x2 − 4x − 12 i x2 + 3x − 4
SkillSHEET j x2 + 4x − 5 k x2 + 6x − 7 l x2 + 3x − 10
Factorising by taking
out a common binomial m x2 − 4x + 3 n x2 − 9x + 20 o x2 + 9x − 70
factor
doc‐5247 2 Factorise each of the following.
SkillSHEET a −x2 − 11x − 10 b −x2 − 7x − 10
Finding a factor
pair that adds to c −x2 − 13x − 12 d −x2 − 7x − 12
a given number
doc‐5250
3 WE6 Factorise each of the following.
a a2 − 6a − 7 b t2 − 6t + 8 c b2 + 5b + 4 d m2 + 2m − 15
e p − 13p − 48
2 f c + 13c − 48
2 g k2 + 22k + 57 h s2 − 16s − 57
i g2 − g − 72 j v2 − 28v + 75 k x2 + 14x − 32 l x2 − 19x + 60

UNDERSTANDING
4 Consider the expression (x − 1) 2 + 5(x − 1) − 6.
a Substitute w = x − 1 in this expression.
b Factorise the resulting quadratic.
c Replace w with x − 1 and simplify each factor. This is the factorised form of the
original expression.
5 Use the method outlined in question 9 to factorise each of the following
expressions.
a (x + 1) 2 + 3(x + 1) − 4
b (x + 2) 2 + (x + 2) − 6
c (x − 3) 2 + 4(x − 3) + 4
d (x + 3) 2 + 8(x + 3) + 12
e (x − 7) 2 − 7(x − 7) − 8
f (x − 5) 2 − 3(x − 5) − 10
6 Factorise x2 + x − 0.75.
7 Students decide to make Valentine’s Day cards. The total area of
each card is equal to (x2 − 4x − 5) cm2. Happy
a Factorise the expression to find the dimensions of the cards in Valentine's
terms of x. Day

248 Maths Quest 10 New South Wales Australian curriculum edition Stages 5.1 and 5.2
number and algebra

b Write down the length of the shorter side in terms of x.


c If the shorter sides of a card are 10 cm in length and the longer sides are 16 cm in
length, find the value of x.
d Find the area of the card proposed in part c.
Digital doc
e If the students want to make 3000 Valentine’s Day cards, how much cardboard WorkSHEET 7.1
will be required? Give your answer in terms of x. doc‐5251

REASONING
REFLECTION
8 Each factorisation below contains an error. Identify the error in each statement. In your own
a x2 − 7x + 12 = (x + 3)(x − 4) b x 2 − x − 1 2 = (x − 3 )(x + 4 ) words,
describe how
c x2 − x + 2 = (x − 1)(x + 1) d x2 − 4x − 21 = (x − 3)(x − 7)
you would
e x2 + 4x − 21 = (x + 3)(x − 7) f x2 − x − 30 = (x − 5)(x + 6) factorise a
g x2 + 7x − 8 = (x + 1)(x − 8) h x2 − 11x + 30 = (x − 5)(x + 6) quadratic
trinomial.

7C Factorising expressions with two


or four terms
Factorising expressions with two terms
•• If the terms in an expanded expression have a common factor, the highest common
factor is written at the front of the brackets and the remaining factor for each term in
the expression is written in the brackets. For example, 4x2 − 36 = 4(x2 − 9).
•• A Difference of Two Squares (DOTS) expression in expanded form has two squared
terms separated by a subtraction symbol.

a2 − b2 = (a − b)(a + b)

Expanded form      
Factorised form

WORKED EXAMPLE 7
Factorise the following.
a 12k2 + 18 b 16a2 − 25b4

TH I N K W R I TE
a 1 Write the expression and look for common a 12k2 + 18 = 6(2k2 + 3)
factors. The terms have a highest common
factor of 6. Write the 6 in front of a set of
brackets, then determine what must go inside
the brackets. 12k2 = 6 × 2k2, 18 = 6 × 3
2 Look for patterns in the expression inside the
brackets to factorise further. The expression
inside the brackets cannot be factorised further.

Chapter 7 • Quadratic expressions 249


number and algebra

b 1 Write the expression and look for common b 16a2 − 25b4


factors. The expression has no common factor.
2 Look for the DOTS pattern in the expression. = 42a2 − 52 (b2) 2
Write the equation showing squares. = (4a) 2 − (5b2) 2
3 Use the pattern for DOTS to write the factors. = (4a + 5b2) (4a − 5b2)
a2 − b2 = (a + b)(a − b)

Factorising expressions with four terms


•• If an expression has four terms, it may require grouping to factorise it.
•• In the process known as grouping ‘two and two’, the terms of the expression are
grouped into two pairs, then a common factor is removed from each pair.
•• When selecting terms to place as pairs, each pair after factorising
should result in a common binomial factor. For example:
2a − 6b + 3ac − 9bc = 2(a − 3b) + 3c(a − 3b)
•• Now we will look at grouping a different combination, known as grouping ‘three
and one’.

WORKED EXAMPLE 8
Factorise each of the following.
a x − 4y + mx − 4my b x2 + 3x − y2 + 3y

TH I N K W R I TE
a 1 Write the expression and look for a a x − 4y + mx − 4my
common factor. (There isn’t one.)
2 Group the terms so that those with = (x − 4y) + (mx − 4my)
common factors are next to each
other.
3 Take out a common factor from = 1(x − 4y) + m(x − 4y)
each group (it may be 1).
4 Factorise by taking out a common = (x − 4y)(1 + m)
binomial factor. The factor (x − 4y)
is common to both groups.
b 1 Write the expression and look b x2 + 3x − y2 + 3y
for a common factor.
2 Group the terms so that those with = (x2 − y2) + (3x + 3y)
common factors are next to each
other.
3 Factorise each group. = (x + y)(x − y) + 3(x + y)
4 Factorise by taking out a common = (x + y)(x − y + 3)
binomial factor. The factor (x + y)
is common to both groups.

250 Maths Quest 10 New South Wales Australian curriculum edition Stages 5.1 and 5.2
number and algebra

WORKED EXAMPLE 9
Factorise the following expression: x2 + 12x + 36 − y2.

TH I N K W R I TE
1 Write the expression and look for a x2 + 12x + 36 − y2
common factor.
2 Group the terms so that those that can = (x2 + 12x + 36) − y2
be factorised are next to each other.
3 Factorise the quadratic trinomial. This = (x + 6)(x + 6) − y2
is the form of a perfect square. = (x + 6) 2 − y2
4 Factorise the expression using = (x + 6 + y)(x + 6 − y)
a2 − b2 = (a + b)(a − b).

Exercise 7C Factorising expressions with


two or four terms
INDIVIDUAL PATHWAYS
Questions: Questions: Questions:
1–4, 8a–h, 9a–d, 10a–c, 14 1–4, 5a, 7a–c, 8a–h, 9a–d, 1–7, 8e–l, 9–15
10a–d, 11–13, 15
Activity 7‐C‐1 Activity 7‐C‐2 Activity 7‐C‐3
Factorising expressions More factorising Advanced factorising
with two or four terms expressions with two expressions with two
doc‐5050 or four terms or four terms
doc‐5051 doc‐5052

FLUENCY
1 Factorise each of the following by taking out a common factor.
a x2 + 3x b x2 − 4x c 3x2 − 6x
d 4x2 + 16x e 9x2 − 3x f 8x − 8x2
g 12x − 3x2 h 8x − 12x2 i 8x2 − 11x
2 Factorise each of the following by taking out a common binomial factor.
a 3x(x − 2) + 2(x − 2) b 5(x + 3) − 2x(x + 3)
c (x − 1) 2 + 6(x − 1) d (x + 1) 2 − 2(x + 1)
e (x + 4)(x − 4) + 2(x + 4) f 7(x − 3) − (x + 3)(x − 3)
3 WE7a Factorise each of the following.
a x2 − 1 b x2 − 9 c x2 − 25
d x2 − 100 e y2 − k2 f 4x2 − 9y2
g 16a2 − 49 h 25p2 − 36q2 i 1 − 100d2

Chapter 7 • Quadratic expressions 251


number and algebra

4 WE7b Factorise each of the following.


a 4x2 − 4 b 5x2 − 80 c ax2 − 9a
d 2b2 − 8d2 e 100x2 − 1600 f 3ax2 − 147a
g 4px2 − 256p h 36x2 − 16 i 108 − 3x2
5 MC a If the factorised expression is (x + 7)(x − 7), then the expanded expression
must be:
Digital docs
A x2 − 7 B x2 + 7 C x2 − 49
SkillSHEET
Factorising by ­taking D x2 + 49 E x2 − 14x + 49
out the highest
­common factor x 3 x 3
doc‐5246 b If the factorised expression is a − b a + b then the expanded expression
SkillSHEET must be: 4 5 4 5
Factorising by ­taking
x2 3 x2 9 x2 1 !3 2 2 x2 9 x2 1 !3 2 2
out a common
­binomial factor
A − B − C − D − E −
doc‐5247
4 5 16 25 4 1 !5 2 2 4 25 16 1 !5 2 2
c The factorised form of 64x2 − 9y2 is:
A (64x + 9y)(64x − 9y) B (8x + 3y)(8x − 3y) C (8x − 3y)(8x − 3y)
D (8x + 3y)(8x + 3y) E (16x + 3y)(16x − 3y)

6 MC Which of the following expressions would be factorised by grouping ‘two


and two’?
A x2 − a2 + 12a − 36 B x2 − 7x − 10
C 2x2 − 6x − xy + 3y D (s − 5) 2–25(s + 3) 2
E (r + 5) − (r + 3)(r + 5)
7 Factorise each of the following expressions.
a (x − 1) 2 − 4 b (x + 1) 2 − 25 c (x − 2) 2 − 9
d (x + 3) 2 − 16 e 49 − (x + 1) 2 f 36 − (x − 4) 2
8 WE8a Factorise each of the following.
a x − 2y + ax − 2ay b 2x + ax + 2y + ay
c ax − ay + bx − by d 4x + 4y + xz + yz
e ef − 2e + 3f − 6 f mn − 7m + n − 7
g 6rt − 3st + 6ru − 3su h 7mn − 21n + 35m − 105
i 64 − 8j + 16k − 2jk j 3a2 − a2b + 3ac − abc
k 5x2 + 10x + x2y + 2xy l 2m2 − m2n + 2mn − mn2
9 WE8b Factorise each of the following.
a xy + 7x − 2y − 14 b mn + 2n − 3m − 6
c pq + 5p − 3q − 15 d s2 + 3s − 4st − 12t
e a2b − cd − bc + a2d f xy − z − 5z2 + 5xyz
10 Factorise each of the following.
a a2 − b2 + 4a − 4b b p2 − q2 − 3p + 3q
c m2 − n2 + lm + ln d 7x + 7y + x2 − y2
e 5p − 10pq + 1 − 4q2 f 49g2 − 36h2 − 28g − 24h
11 WE9 Factorise each of the following.
a x2 + 14x + 49 − y2 b x2 + 20x + 100 − y2
c a2 − 22a + 121 − b2 d 9a2 + 12a + 4 − b2

252 Maths Quest 10 New South Wales Australian curriculum edition Stages 5.1 and 5.2
number and algebra

12 MC In the expression 3(x − 2) + 4y(x − 2), the common binomial factor is:
A 3 + 4y B 3 − 4y
C x D −x + 2
E x−2
b Which of the following terms is a perfect square?
A 9 B (x + 1)(x − 1)
C 3x2 D 5(a + b) 2
E 25x
c Which of the following expressions can be factorised using grouping?
A x2 − y2 B 1 + 4y − 2xy + 4x2
C 3a2 + 8a + 4 D x2 + x + y − y2
E 2a + 4b − 6ab + 18
13 MC When factorised, 6(a + b) − x(a + b) equals:
A 6 − x(a + b) B (6 − x)(a + b)
C 6(a + b − x) D (6 + x)(a − b)
E (6 + x)(a + b)

UNDERSTANDING
14 The area of a rectangle is (x2 − 25) cm2.
a Factorise the expression.
b Find the length of the rectangle if the width is x + 5 cm.
c If x = 7 cm, find the dimensions of the rectangle.
d Hence, find the area of the rectangle.
e If x = 13 cm, how much bigger would the area of this
rectangle be?
15 A roll of material is (x + 2) metres wide. Annie buys (x + 3)
metres and Bronwyn buys 5 metres.
a Write an expression, in terms of x, for the area of
each piece of material.
b If Annie has bought more material than Bronwyn, write an
expression for how much more she has than Bronwyn.
c Factorise and simplify this expression.
d Find the width of the material if Annie has 5 m2
more than Bronwyn. REFLECTION
e How much material does each person have? Explain What do you always check
your answer. for first when factorising?

7D  Mixed factorisation


•• The following exercise will help you to practise recognising the appropriate method
of factorising needed for a given expression. Apply what has been covered, in this
chapter to the following exercise.
–– Factorising monic trinomials
–– Factorising by grouping
–– Difference of two squares

Chapter 7 • Quadratic expressions 253


number and algebra

Exercise 7D Mixed factorisation


INDIVIDUAL PATHWAYS
Questions: Questions: Questions:
1, 2, 4–6, 8, 9, 13–15, 18, 20, 26, 1, 2, 4–6, 8, 9, 13–15, 18, 20, 26, 5, 6, 8, 9, 13–15, 18, 20, 26,
34, 35a–c 29, 30, 34, 36a–c 34–37
Activity 7‐D‐1 Activity 7‐D‐2 Activity 7‐D‐3
Mixed factorisation Harder mixed factorisation Advanced mixed factorisation
doc‐13723 doc‐13724 doc‐13725

Digital docs FLUENCY


SkillSHEET
Factorising by grouping
Factorise each of the following expressions in questions 1 –45.
three and one 1 3x + 9 2 x2 + 4x + 4 − 9y2 3 x2 − 36
doc‐5252
SkillSHEET 4 x2 − 49 5 15x − 20y 6 5c + de + dc + 5e
Simplifying algebraic
fractions 7 5x − 80
2 8 −x − 6x − 5
2 9 x2 + x − 12
doc‐5248
10 mn + 1 + m + n 11 x2 − 7 12 16x2 − 4x
13 18 + 9x − 6y − 3xy 14 x2 − 8x + 16 − y2 15 4x2 + 8
16 fg + 2h + 2g + fh 17 x2 − 5 18 10mn − 5n + 10m − 5
19 x + 6x + 5
2 20 x − 10x − 11
2 21 x2 − 4
22 −5a + bc + ac − 5b 23 xy − 1 + x − y 24 7x2 − 28
25 2p − rs + pr − 2s 26 3x2 − 27 27 −3u + tv + ut − 3v
28 x2 − 11 29 (x − 1) 2 − 4 30 (x + 2) 2 − 16
31 (2x + 3) 2 − 25 32 3(x + 5) 2 − 27 33 25 − (x − 2) 2
34 4(3 − x) 2 − 16y2

UNDERSTANDING
35 Consider the following product of algebraic fractions.

x2 + 3x − 10 x2 + 4x + 4
× 2
x2 − 4 x − 2x − 8
a Factorise the expression in each numerator and denominator.
b Cancel factors common to both the numerator and the denominator.
c Simplify the expression as a single fraction.
36 Use the procedure in question 35 to factorise and simplify each of the following.
x2 − 4x + 3 x2 + 5x + 6
a ×
x2 − 4x − 12 x2 − 9
6x − 12 3x + 6
b ×
x −4
2 x(x − 5)

x2 + 4x − 5 x2 + 10x + 25
c ÷ 2
x2 + x − 2 x + 4x + 4

254 Maths Quest 10 New South Wales Australian curriculum edition Stages 5.1 and 5.2
number and algebra

x2 − 7x + 6 x2 − x − 12
d ÷
x2 + x − 2 x2 − 2x − 8
4ab + 8a 5ac + 5a
e ÷ 2
(c − 3) c − 2c − 3 REFLECTION
When an expression is fully
Digital doc
p2 − 7p p2 + p − 6 factorised, what should it
f ÷ look like?
WorkSHEET 7.2
p2 − 49 p2 + 14p + 49 doc‐13726

Challenge 7.2

b
d
e f

Chapter 7 • Quadratic expressions 255


number and algebra

CHAPTER REVIEW
LANGUAGE

binomial factorise monic


common factor FOIL non‐monic
difference of two squares grouping three and one perfect squares
expand grouping two and two quadratic trinomial

int‐2844 doc‐13728
int‐2845 doc‐13729
int‐3594

FLUENCY
1 When expanded, −3x(x + 4)(5 − x) becomes:
A −3x3 − 3x2–27x B −3x3 + 3x2–27x C 3x3 + 3x2 − 60x
D −3x3 + 3x2–60x E 3x3 − 3x2 − 60x
2 When expanded, (3x + 7) 2 becomes:
A 9x2 + 49 C 3x2 + 21x + 49 E 9x2 + 21x + 49
B 3x2 + 49 D 9x2 + 42x + 49
3 The factorised form of −3d2 − 9d + 30 is:
A −3(d − 5)(d − 2) C −(3d + 5)(d − 2) E −3(d + 5)(d − 2)
B −3(d + 5)(d − 6) D −(3d + 5)(d − 6)
4 If the factorised expression is 1 2x– 5 2 (2x + 5), then the original expression must
have been:
A 2x2 − 5 B 4x2 − 5 C 4x2 − 25
D 4x2 − 20x + 25 E 2x2 + 25
5 To factorise −5x2 − 45x + 100, the first step is to:
A find factors of 5 and 100 that sum to −45
B take out 5 as a common factor
C take out −5 as a common factor
D find factors of 5 and −45 that will add to make 100
E take out −5x as a common factor
6 In the expanded form of (x − 3)(x + 5), which of the following is incorrect?
A The value of the constant is −15.
B The coefficient of the x term is 2.
C The coefficient of the x term is −8.
D The coefficient of the x2 term is 1.
E The expansion shows this to be a trinomial expression.

256 Maths Quest 10 New South Wales Australian curriculum edition Stages 5.1 and 5.2
number and algebra

7 Expand each of the following and simplify where necessary.


a 3x(x − 4) b −7x(3x + 1)
c (x − 7)(x + 1) d (2x − 5)(x − 3)
e (4x − 1)(3x − 5) f 3(x − 4)(2x + 7)
g (2x − 5)(x + 3)(x + 7) h (x + 5)(x + 7) + (2x − 5)(x − 6)
i (x + 3)(5x − 1) − 2x
8 Expand and simplify each of the following.
a (x − 7) 2 b (2 − x) 2 c (3x + 1) 2
d −2(3x − 2) 2 e −7(2x + 5) 2 f −10(4x − 5) 2
g (x + 9)(x − 9) h (3x − 1)(3x + 1) i (5 + 2x)(5 − 2x)
9 Factorise each of the following.
a 2x2 − 8x b −4x2 + 12x
c 3ax − 2ax2 d (x + 1) 2 + (x + 1)
10 Factorise each of the following.
a x2 − 16 b x2 − 25 c 2x2 − 72
d 3x2 − 27y2 e 4ax2 − 16ay2 f (x − 4) 2 − 9
11 Factorise each of the following by grouping.
a ax − ay + bx − by b 7x + ay + ax + 7y
c xy + 2y + 5x + 10 d mn − q − 2q2 + 2mnq
e pq − 5r2 − r + 5pqr f uv − u + 9v − 9
g a − b + 5a − 5b
2 2 h d2 − 4c2 − 3d + 6c
i 2 + 2m + 1 − m2
12 Factorise each of the following by grouping.
a 4x2 + 12x + 9 − y2 b 49a2 − 28a + 4 − 4b2
c 64s2 − 16s + 1 − 3t
13 Factorise each of the following.
a x2 + 10x + 9 b x2 − 11x + 18 c x2 − 4x − 21
d x2 + 3x − 28 e −x2 + 6x − 9 f 3x2 + 33x − 78
g −2x2 + 8x + 10 h −3x2 + 24x − 36
14 Factorise each of the following using the most appropriate method.
a 3x2 − 12x b 4x2 − 25 c 2ax + 4x + 3a + 6
15 First factorise then simplify each of the following.
x+4 2x − 12 3x + 6 7x − 42 x2 − 4 x2 + 4x − 5
a × b × c ×
5x − 30 x+1 4x − 24 6x + 12 x2 + 5x x2 − 2x − 8

PROBLEM SOLVING
1 A large storage box has a square base with sides
measuring (x + 2) cm and is 32 cm high.
a Write an expression for the area of the base
of the box.
b Write an expression for the volume of the box
(V = area of base × height).

Chapter 7 • Quadratic expressions 257


number and algebra

c Simplify the expression by expanding the brackets.


d If x = 30 cm, find the volume of the box in cm3.
2 A section of garden is to have a
circular pond of radius 2r with a
2 m path around its edge.
a State the diameter of the pond.
b State the radius of the pond
and path.
c State the area of the pond.
d State the area of the pond and
path.
e Write an expression to find the
area of the path only and write
it in factorised form.
f If the radius of the pond is 3 metres, find the area of the path.
3 In order to make the most of the space available for headlines and stories, the front
page of a newspaper is given an area of x2 − 5x − 14 cm2.
a Factorise the expression to find the dimensions of the paper in terms of x.
b Write down the length of the shorter side in terms of x.
c If the shorter side of the front page is 28 cm, find the value of x.
d Find the area of this particular paper.
4 Here is a well‐known puzzle.
Let a = b = 1.
Step 1: Write a = b. a=b
Step 2: Multiply both sides by a. a2 = ab
Step 3: Subtract b2 from both sides. a2 − b2 = ab − b2
Step 4: Factorise. (a + b)(a − b) = b(a − b)
Step 5: Simplify by dividing by (a − b). (a + b) = b
Step 6: Substitute a = b = 1. 1+1=1
Where is the error?
Show your thinking.
Investigate the patterns in addition and subtraction symbols in general quadratic
5 a 
expressions by expanding the following binomial factors.
a i (x + 2)(x + 3) ii (x − 2)(x + 3) iii (x + 2)(x − 3) iv (x − 2)(x − 3)
b i (x + 5)(x + 1) ii (x − 5)(x + 1) iii (x + 5)(x − 1) iv (x − 5)(x − 1)
c i (x + 10)(x + 7) ii (x − 10)(x + 7) iii (x + 10)(x − 7) iv (x − 10)(x − 7)
d i (x + 8)(x + 6) ii (x − 8)(x + 6) iii (x + 8)(x − 6) iv (x − 8)(x − 6)
b Describe how the addition and subtraction symbols in an expanded quadratic
equation can help identify the binomial factors.

258 Maths Quest 10 New South Wales Australian curriculum edition Stages 5.1 and 5.2
number and algebra

c Copy and complete the flow chart below using your findings.

Is c positive?

Is b positive?
Either f is positive and h is negative
OR
Yes

6 A proposed new flag for Australian schools will have the Australian flag in the top
left-hand corner. The dimensions given are in metres.
x

x−2
x

2x

a Write an expression, in factorised form, for the area of:


i the Australian flag
ii the proposed flag.
b Use the answers to part a to write the area of the Australian flag as a fraction of
the school flag and then simplify this fraction.
c Use the fraction from part b to express the area of the Australian flag as a
percentage of the proposed school flag.
d Using the formula for the percentage of area taken up by the Australian flag, find
the percentages for the following school flag widths.
i 4  m
ii 4.5  m
iii 4.8  m
e If the percentage of the school flag taken up by the Australian flag measures the
importance a school places on Australia, what can be said for the three flags?

Chapter 7 • Quadratic expressions 259


number and algebra
Communicating

Rich task

Celebrity squares and doubles

260 Maths Quest 10 New South Wales Australian curriculum edition Stages 5.1 and 5.2
number and algebra

2 Beginning the game


• There is to be no communication between players at this time.
• Your teacher will randomly allocate a headband to each player by placing a headband on
their head without the player seeing the number on their headband.
3 Playing the game
The object of the game
• The object of the game is to use the process of Hmm… now, Lizzy is 16 and
elimination for you to find your pair. A possible Jo is 8 so they are a match.
Nick is 10. I can’t see a
train of thought illustrated at right. 25…maybe he is my match.
Starting the game
• Once all headbands have been allocated, stand in 25
a circle or walk around freely.
• Without speaking, determine who is a match; then
by a process of elimination, determine who might
be your match.
Making a match
• When you think you have find your match,
­approach that person and say ‘I think I am your
match’.
• The other player should now check to see if you have a match elsewhere and can reply
by saying one of two things: ‘Yes, I think I am your match’ or ‘I know your match is still out
there’.
• If a match is agreed upon, the players should sit out for the remainder of the game. If a
match is not agreed upon, players should continue looking.
Ending the game
• The class should continue until everyone is in a pair, at which time the class can check
their results.
• The class should now discuss the different trains of thought they used to find their pair
and how this relates to factorising quadratic trinomials.

Chapter 7 • Quadratic expressions 261


number and algebra

Code puzzle

What historical event took


place in Ballarat in 1854?
Use an appropriate method to factorise the expressions
on the left side. The letters beside each question and the
numbers beside the answers will help solve the puzzle.

2 3 9 9 15 17 9 12 11 10 2 19 14 12 11 18 9

A x2 – 9 1 (x – 5)(x + 5 )
B x 2 – 2x 2 4(2x – 3)(2x +3)
C (x + 3) 2 – 2(x + 3) 3 (x – 3)(x – 2)
D x 2 + 10x + 25 4 (x – 4)(x + 4)
E (x –1) 2 – 16 5 x(x + 2)
F 2x 2 + 4x – 30 6 (x + 5)(x – 1)
G x 2 – 9x + 20 7 (2x + 5)(x – 3)
H x2 – 5x + 6 8 (3x + 1)(x + 2)
I 3x 2 + 7x + 2 9 (x – 5)(x + 3)
K 2x 2 + 8x + 6 10 (x + 5)(x – 3)
L x2 + 2x 11 (x + 3)(x – 3)
M x 2 + 4x – 5 12 2(x + 3)(x + 1)
N x2 – 25 13 (x – 5)(x – 4)
O 3x 2 + 4x + 1 14 (x + 3)(x + 1)
R 2x 2 + 6x – 20 15 (3x + 1)(x + 3)
S x 2 + 2x – 15 16 x(x –2)
T 16x 2 – 36 17 2(x + 5)(x – 2)
U 3x 2
+ 10x + 3 18 (x + 5) 2
V 2
x – 16 19 (3x + 1)(x + 1)
W 2x 2 – x – 15 20 2(x + 5)(x – 3)

11 16 11 2 2 5 9 16 9 2 7 9 9 1 2 3 9

11 15 2 3 19 17 8 2 8 9 10 11 1 18 2 3 9

13 19 5 18 18 8 13 13 9 17 10 19 4 9 17 2 3 9

8 10 10 15 8 1 13 11 1 18 9 1 20 19 17 14 9 6 9 1 2

19 20 6 8 1 9 17 10 5 8 14 9 1 14 9 10

262 Maths Quest 10 New South Wales Australian curriculum edition Stages 5.1 and 5.2
number and algebra

activities
Go to assessON
Chapter opener • Activity 7‐C‐2 (doc‐5051): More for questions to
video factorising expressions with two or four test your readiness
• The story of mathematics (eles‐1846) terms (page 251) FOR learning, your
• Activity 7‐C‐3 (doc‐5052): Advanced progress AS you learn
7A Expanding algebraic expressions
factorising expressions with two or four and your levels OF
Digital docs (page 243)
terms (page 251) achievement.
• Activity 7‐A‐1 (doc‐5044): Review of
• SkillSHEET (doc‐5246): Factorising by taking www.assesson.com.au
expansion
out the highest common factor (page 252)
• Activity 7‐A‐2 (doc‐5045): Expanding
• SkillSHEET (doc‐5247): Factorising by taking
algebraic expressions
out a common binomial factor (page 252)
• Activity 7‐A‐3 (doc‐5046): Expanding more
complex algebraic expressions 7D Mixed factorisation
• SkillSHEET (doc‐5244): Expanding brackets Digital docs
• SkillSHEET (doc‐5245): Expanding a pair of • Activity 7‐D‐1 (doc‐13723): Mixed
brackets factorisation (page 254)
• Activity 7‐D‐2 (doc‐13724): Harder mixed
7B Factorising expressions with three
factorisation (page 254)
terms
• Activity 7‐D‐3 (doc‐13725): Advanced mixed
Digital docs
factorisation (page 254)
• Activity 7‐B‐1 (doc‐5047): Introducing
• SkillSHEET (doc‐5252): Factorising by
quadratic factorisation (page 248)
grouping three and one (page 254)
• Activity 7‐B‐2 (doc‐5048): Practising
• SkillSHEET (doc‐5248): Simplifying algebraic
quadratic factorisation (page 248)
fractions (page 254)
• Activity 7‐B‐3 (doc‐5049): Tricky quadratic
• WorkSHEET 7.2 (doc‐13726): Mixed
factorisation (page 248)
factorisation (page 255)
• SkillSHEET (doc‐5246): Factorising by taking
out the highest common factor Chapter review
• SkillSHEET (doc‐5247): Factorising by taking Interactivities (page 256)
out a common binomial factor • Word search Chapter 7 (int‐2844)
• SkillSHEET (doc‐5250): Finding a factor pair • Crossword Chapter 7 (int‐2845)
that adds to a given number (page 248) • Sudoku (int‐3594)
• WorkSHEET 7-1 (doc‐5251): Factorising and
Digital docs
expanding (page 249)
Chapter summary (doc‐13728)
7C Factorising expressions with two of Concept map (doc‐13729)
four terms
To access eBookPLUS activities, log on to
Digital docs
www.jacplus.com.au
• Activity 7‐C‐1 (doc‐5050): Factorising
expressions with two or four
terms (page 251)

Chapter 7 • Quadratic expressions 263


number and algebra

rich task
Answers
Chapter 7 Quadratic expressions
Exercise 7A — Expanding algebraic expressions 14 (a + b) 2 = a2 + 2ab + b2
1    a 2x + 6 b 4x − 20 c 21 − 3x LHS:
d −x − 3 e x2 + 2x f 2x2 − 8x (5 + 3) 2
g 15x2 − 6x h 10x − 15x2 i 8x2 + 2x = 82
j 4x3 − 6x2 k 6x3 − 3x2 l 15x3 + 20x2 = 64
2    a x2 − x − 12 b x2 − 2x − 3 RHS:
c x2 − 5x − 14 d x2 − 6x + 5 52 + 2 × 5 × 3 + 32
e −x2 − x + 6 f x2 − 6x + 8 = 25 + 30 + 9
g 2x − 17x + 21
2
h 3x2 − x − 2 = 64
i 6x2 − 17x + 5 j 21 − 17x + 2x2 LHS = RHS ⇒ True
k 15 + 14x − 8x2 l 110 + 47x − 21x2 16 Answers will vary; examples are shown.
3    a 2x2 − 4x − 6 b 8x2 − 28x − 16
c −2x2 + 12x + 14 d 2x3 − 2x a e.g. (x + 4) (x + 3) = x2 + 7x + 12
e 3x − 75x
3 f 6x3 − 54x b e.g. (x + 4) 2 = x2 + 8x + 16
4    a x3 + 2x2 − x − 2 b x3 − 2x2 − 5x + 6 c e.g. (x + 4) (x − 4) = x2 − 16
c x − 5x − x + 5
3 2 d x3 − 6x2 + 11x − 6 d both a and b.
e 2x3 − 7x2 − 5x + 4 f 6x3 − 7x2 + 1 17 The square of a binomial is a trinomial; the difference of two
5    a x − x − 2
2 b −2x2 + 4x + 10 squares has two terms.
c 5x2 − 6x − 5 d −2x + 6 18 (a + b) (c + d) = (c + d) (a + b)
6    a A b C LHS:
7 B (a + b) (c + d) = ac + ad + bc + bd
8    a x2 − 2x + 1 b x2 + 4x + 4 RHS:
c x + 10x + 25
2
d 16 + 8x + x2 (c + d) (a + b) = ca + cb + da + db
e 49 − 14x + x2 f 144 − 24x + x2 LHS = RHS ⇒ True
g 9x − 6x + 1
2
h 144x2 − 72x + 9 Challenge 7.1
i 25x2 + 20x + 4 10, 11, 13, 18, 35
9    a 2x2 − 12x + 18 b 4x2 − 56x + 196
Exercise 7B — Factorising expressions with three terms
c 3x2 + 6x + 3 d −4x2 − 12x − 9
e −49x2 + 14x − 1 f 8x2 − 24x + 18 1    a (x + 2) (x + 1) b (x + 3) (x + 1)
10    a x − 49
2 b x − 81
2 c x2 − 25 c (x + 8) (x + 2) d (x + 4) 2
d x2 − 1 e 4x2 − 9 f 9x2 − 1 e (x − 3) (x + 1) f (x − 4) (x + 1)
g 49 − x 2 h 64 − x2 i 9 − 4x2 g (x − 12) (x + 1) h (x − 6) (x + 2)
11    a (x + 1) (x − 3) b x2 − 2x − 3 i (x + 4) (x − 1) j (x + 5) (x − 1)
k (x + 7) (x − 1) l (x + 5) (x − 2)
c 6 cm, 2 cm, 12 cm2
m (x − 3) (x − 1) n (x − 4) (x − 5)
12    a b
o (x + 14) (x − 5)
(x + 1) m 2    a −(x + 10) (x + 1) b −(x + 2) (x + 5)
c −(x + 12) (x + 1) d −(x + 3) (x + 4)
xm (x + 2) m 3    a (a − 7) (a + 1) b (t − 4) (t − 2)
c (x + 1) (x + 2) d x2 + 3x + 2 e 4 m2, 12 m2 c (b + 4) (b + 1) d (m + 5) (m − 3)
13    a Student C e (p − 16) (p + 3) f (c + 16) (c − 3)
b Student B: g (k + 19) (k + 3) h (s − 19) (s + 3)
(3x + 4) (2x + 5) i (g + 8) (g − 9) j (v − 25) (v − 3)
= 3x × 2x + 3x × 5 + 4 × 2x + 4 × 5 k (x + 16) (x − 2) l (x − 15) (x − 4)
= 6x2 + 23x + 20 4    a w2 + 5w − 6 b (w + 6) (w − 1) c (x + 5) (x − 2)
Student A: 5    a x(x + 5) b x(x + 5)
(3x + 4) (2x + 5) c (x − 1) 2 d (x + 9) (x + 5)
= 3x × 2x + 3x × 5 + 4 × 2x + 4 × 5 e (x − 15) (x − 6) f (x − 10) (x − 3)
6 (x − 0.5) (x + 1.5)
= 6x2 + 15x + 8x + 20
= 6x2 + 23x + 20 7    a (x − 5) (x + 1) b (x − 5) cm
14 (a − b) (a + b) = a2 − b2 c x = 15 cm d 160 cm2
LHS e 3000(x − 5) (x + 1) cm2 or (3000x2 − 12 000x − 15 000) cm2
(5 − 3) (5 + 3) 8    a x2 − 7x + 12 = (x − 3) (x − 4)
=2×8 b x2 + 7x − 12 = (x − 3) (x − 4)
= 16 c x2 − x + 2 = (x − 2) (x + 1)
RHS: d x2 − 4x − 21 = (x + 3) (x − 7)
52 − 32 e x2 + 4x − 21 = (x − 3) (x + 7)
f x2 − x − 30 = (x + 5) (x − 6)
= 25 − 9
g x2 + 7x − 8 = (x − 1) (x + 8)
= 16
h x2 − 11x + 30 = (x − 5) (x − 6)
LHS = RHS ⇒ True

264 Maths Quest 10 New South Wales Australian curriculum edition Stages 5.1 and 5.2
number and algebra

Exercise 7C — Factorising expressions with two or four terms (x + 5) (x − 2) (x + 2) (x + 2)


35    a ×
1    a x(x + 3) b x(x − 4) c 3x(x − 2) (x + 2) (x − 2) (x − 4) (x + 2)
d 4x(x + 4) e 3x(3x − 1) f 8x(1 − x) (x + 5) (x − 2) (x + 2) (x + 2) x+5
g 3x(4 − x) h 4x(2 − 3x) i x(8x − 11) b × c
(x + 2) (x − 2) (x − 4) (x + 2) x−4
2    a (x − 2) (3x + 2) b (x + 3) (5 − 2x)
c (x − 1) (x + 5) d (x + 1) (x − 1) x−1 18 x+2
36    a b c
e (x + 4) (x − 2) f (x − 3) (4 − x) x−6 x(x − 5) x+5
3    a (x + 1) (x − 1) b (x + 3) (x − 3) x−6 4(b + 2) p(p + 7)
d e f
c (x + 5) (x − 5) d (x + 10) (x − 10) x+3 5 (p + 3) (p − 2)
e (y + k) (y − k) f (2x + 3y) (2x − 3y) Challenge 7.2
g (4a + 7) (4a − 7) h (5p + 6q) (5p − 6q) a 10 609 b 3844 c 994 009
i (1 + 10d) (1 − 10d) d 1024 144 e 2809 f 9604
4    a 4(x + 1) (x − 1) b 5(x + 4) (x − 4)
c a(x + 3) (x − 3) d 2(b + 2d ) (b − 2d ) Chapter review
e 100(x + 4) (x − 4) f 3a(x + 7) (x − 7) Fluency
g 4p(x + 8) (x − 8) h 4(3x + 2) (3x − 2) 1 E 2 D 3 E
i 3(6 + x) (6 − x) 4 C 5 C 6 C
5    a C b B c B 7    a 3x2 − 12x b −21x2 − 7x
6    a C c x2 − 6x − 7 d 2x2 − 11x + 15
7    a (x − 3) (x + 1) b (x − 4) (x + 6) e 12x2 − 23x + 5 f 6x2 − 3x − 84
c (x − 5) (x + 1) d (x − 1) (x + 7) g 2x + 15x − 8x − 105
3 2 h 3x2 − 5x + 65
e (6 − x) (x + 8) f (10 − x) (x + 2) i 5x2 + 12x − 3
8    a (x − 2y) (1 + a) b (x + y) (2 + a) 8    a x2 − 14x + 49 b 4 − 4x + x2
c (x − y) (a + b) d (x + y) (4 + z) c 9x2 + 6x + 1 d −18x2 + 24x − 8
e (f − 2) (e + 3) f (n − 7) (m + 1) e −28x2 − 140x − 175 f −160x2 + 400x − 250
g 3(2r − s) (t + u) h 7(m − 3) (n + 5) g x2 − 81 h 9x2 − 1
i 2(8 − j) (4 + k) j a(3 − b) (a + c) i 25 − 4x2
k x(5 + y) (x + 2) l m(m + n) (2 − n) 9    a 2x(x − 4) b −4x(x − 3)
9    a (y + 7) (x − 2) b (m + 2) (n − 3) c ax(3 − 2x) d (x + 1) (x + 2)
c (q + 5) (p − 3) d (s + 3) (s − 4t) 10    a (x + 4) (x − 4) b (x + 5) (x − 5)
e (b + d) (a2 − c) f (1 + 5z) (xy − z) c 2(x + 6) (x − 6) d 3(x + 3y) (x − 3y)
10    a (a − b) (a + b + 4) b (p − q) (p + q − 3) e 4a(x + 2y) (x − 2y) f (x − 1) (x − 7)
c (m + n) (m − n + l) d (x + y) (7 + x − y) 11    a (x − y) (a + b) b (x + y) (7 + a)
e (1 − 2q) (5p + 1 + 2q) f (7g + 6h) (7g − 6h − 4) c (x + 2) (y + 5) d (1 + 2q) (mn − q)
11    a (x + 7 + y) (x + 7 − y) b (x + 10 + y) (x + 10 − y) e (5r + 1) (pq − r) f (v − 1) (u + 9)
c (a − 11 + b) (a − 11 − b) d (3a + 2 + b) (3a + 2 − b) g (a − b) (a + b + 5) h (d − 2c) (d + 2c − 3)
12    a E b A c D i (1 + m) (3 − m)
13 B 12    a (2x + 3 + y) (2x + 3 − y)
14    a (x − 5) (x + 5) b (x − 5) cm, (x + 5) cm b (7a − 2 + 2b) (7a − 2 − 2b)
c 2 cm, 12 cm d 24 cm2 c (8s − 1 + !3t) (8s − 1 − !3t)
e 120 cm2 or 6 times bigger 13    a (x + 9) (x + 1) b (x − 9) (x − 2) c (x − 7) (x + 3)
15    a Annie = (x + 3) (x + 2) m2, Bronwyn = 5(x + 2) m2 d (x + 7) (x − 4) e −(x − 3) 2 f 3(x + 13) (x − 2)
b (x + 3) (x + 2) − 5(x + 2) g −2(x − 5) (x + 1) h −3(x − 6) (x − 2)
c (x + 2) (x − 2) = x2– 4 14    a 3x(x − 4) b (2x + 5) (2x − 5) c (a + 2) (2x + 3)
d Width = 5 m 2(x + 4) (x − 2) (x − 1)
e Annie has 30 m2 and Bronwyn has 25 m2. 15    a b 78 c
5(x + 1) x(x − 4)
Exercise 7D — Mixed factorisation Problem solving
1 3(x + 3) 2 (x + 2 + 3y) (x + 2 − 3y) 1    a (x + 2) 2 b 32(x + 2) 2
3 (x + 6) (x − 6) 4 (x + 7) (x − 7) c 32x + 128x + 128
2 d 32 768 cm3
5 5(3x − 4y) 6 (c + e) (5 + d) 2    a 4r b 2r + 2 c 4πr2
7 5(x + 4) (x − 4) 8 −(x + 5) (x + 1) d (4πr + 8r + 4)π e 4π(2r + 1)
2 f 28π m2
9 (x + 4) (x − 3) 10 (m + 1) (n + 1) 3    a (x − 7) (x + 2) b x − 7 cm
11 (x + !7) (x − !7) 12 4x(4x − 1) c 35 d 1036 cm2
13 3(3 − y) (x + 2) 14 (x − 4 + y) (x − 4 − y) 4 Division by zero in Step 5
15 4(x2 + 2) 16 (g + h) (f + 2) 5 Check with your teacher.
17 (x + !5) (x − !5) 18 5(n + 1) (2m − 1) 6    a i x(x − 2) m2   ii x(2x) m2
19 (x + 5) (x + 1) 20 (x + 1) (x − 11) (x − 2) 2 50(x − 2)
b m c %
21 (x + 2) (x − 2) 22 (a + b) (c − 5) 2x ·
d i 25%      ii 27.8%      iii 29.17%
x
·
23 (y + 1) (x − 1) 24 7(x + 2) (x − 2)
e The third flag places the most importance on Australia.
25 (2 + r) (p − s) 26 3(x + 3) (x − 3)
27 (u + v) (t − 3) 28 (x + !11) (x − !11) Communicating — Rich task
29 (x + 1) (x − 3) 30 (x + 6) (x − 2) Teacher to check.
31 4(x − 1) (x + 4) 32 3(x + 2) (x + 8) Code puzzle
33 (3 + x) (7 − x) 34 4(3 − x + 2y) (3 − x − 2y) The Eureka stockade

Chapter 7 • Quadratic expressions 265


number and
NUMBER AND ALGEBRA
algebra

Chapter 8

Quadratic
equations
WHY LEARN THIS?
The Guggenheim Museum in Bilbao (Spain) is covered
with thin metal plates like the scales of a fish, each
one designed and shaped by a computer. This project
required the solving of thousands of non‐linear
equations. Parabolic shapes are widely used by
engineers and architects.

WHAT DO YOU KNOW?


1 Think List what you know about quadratic
equations. Use a thinking tool such as a concept
map to show your list.
2 pair Share what you know with a partner
and then with a small group.
3 share As a class, create a large concept map that
shows your class’s knowledge of quadratic equations.

LEARNING SEQUENCE
8A Solving quadratic equations algebraically
8B Solving quadratic equations graphically
Watch this video
The story of mathematics

Searchlight ID: eles-1847


number and algebra

8A  Solving quadratic equations algebraically


Quadratic equations
•• The general form of a quadratic equation is ax2 + bx + c = 0.
•• To solve an equation means to find the value of the pronumeral(s) or variables that
when substituted will make the equation a true statement.

The Null Factor Law


•• The Null Factor Law states that if the product of two numbers is zero, then one
(or both) of the numbers must equal zero.
•• In other words there are two solutions to the equation pq = 0. They are p = 0 and
q = 0.
•• When solving quadratic equations by applying the Null Factor law, it is best to write
the equations equal to zero.

WORKED EXAMPLE 1
Solve the equation (x − 7)(x + 11) = 0.

TH I N K W R I TE
1 Write the equation and check that the (x − 7)(x + 11) = 0
right‐hand side equals zero. ( The product
of the two numbers is zero.)
2 The left‐hand side is factorised, x − 7 = 0 or x + 11 = 0
so apply the Null Factor Law.
3 Solve for x. x=7 x = −11

WORKED EXAMPLE 2
Solve each of the following equations.
a x2 − 3x = 0 b 3x2 − 27 = 0 c x2 − 13x + 42 = 0

TH I N K W R I TE
a 1 Write the equation. Check that the a x2 − 3x = 0
right‐hand side equals zero.
2 Factorise by taking out the common x(x − 3) = 0
factor of x2 and 3x, which is x.
3 Apply the Null Factor Law. x = 0 or x − 3 = 0
4 Solve for x. x=0 x=3
b 1 Write the equation. Check that the b 3x2 − 27 = 0
right‐hand side equals zero.
2 Factorise by taking out the common 3(x2 − 9) = 0
factor of 3 and 27, which is 3.

268 Maths Quest 10 New South Wales Australian curriculum edition Stages 5.1 and 5.2
number and algebra

3 Factorise using the difference of two 3(x2 − 32) = 0


squares rule. 3(x + 3)(x − 3) = 0
4 Apply the Null Factor Law. x + 3 = 0 or x − 3 = 0
5 Solve for x.      x = −3 x=3
(Alternatively, x = ±3)
c 1 Write the equation. Check that the c x2 − 13x + 42 = 0
right‐hand side equals zero.
2 Factorise by finding a factor pair of
Factors of 42 Sum of factors
42 that adds to −13.
−6 × −7 −13
(x − 6)(x − 7) = 0
3 Apply the Null Factor Law. x − 6 = 0 or x − 7 = 0
4 Solve for x.      x = 6 x=7

Solving problems
•• There are many problems that can be modelled by a quadratic equation. You should
first form the quadratic equation that represents the situation before attempting to
solve such problems.
•• Recall that worded problems should always be answered with a sentence.

WORKED EXAMPLE 3
When two consecutive numbers are multiplied together, the result is 20.
Determine the numbers.

TH I N K W R I TE
1 Define the unknowns. First number = x
Second number = x + 1
2 Write an equation using the given    x(x + 1) = 20
information.
3 Transpose the equation so that the x(x + 1) − 20 = 0
right‐hand side equals zero.
4 Expand to remove the brackets.    x2 + x − 20 = 0
5 Factorise. (x + 5)(x − 4) = 0
6 Apply the Null Factor Law to solve x + 5 = 0 or x − 4 = 0
for x. x = −5 x=4
7 Use the answer to determine the If x = −5, x + 1 = −4.
second number. If x = 4, x + 1 = 5.
8 Check the solutions. Check: 4 × 5 = 20 −5 × −4 = 20
9 Answer the question in a sentence. The numbers are 4 and 5 or −5
and −4.

Chapter 8 • Quadratic equations 269


number and algebra

WORKED EXAMPLE 4
The height of a football after being kicked is determined by the
formula h = −0.1d2 + 3d, where d is the horizontal distance from the
player.
a How far away is the ball from the player when it hits the ground?
b What horizontal distance has the ball travelled when it first reaches
a height of 20 m?

TH I N K W R I TE
a 1 Write the formula. a h = −0.1d2 + 3d
2 The ball hits the ground when −0.1d2 + 3d = 0
h = 0. Substitute h = 0 into the
formula.
3 Factorise. −0.1d2 + 3d = 0
d(−0.1d + 3) = 0
4 Apply the Null Factor Law and d = 0 or −0.1d + 3 = 0
simplify. −0.1d = −3
−3
d=
−0.1
d = 30
5 Interpret the solutions. d = 0 is the origin of the kick.
d = 30 is the distance from the
origin that the ball has travelled
when it lands.
6 Answer the question in a sentence. The ball is 30 m from the player
when it hits the ground.
b 1 The height of the ball is 20 m b h = −0.1d2 + 3d
so substitute h = 20 into the 20 = −0.1d2 + 3d
formula.
2 Transpose the equation, so that 0.1d2 − 3d + 20 = 0
zero is on the right‐hand side.
3 Multiply both sides of the equation d2 − 30d + 200 = 0
by 10 to remove the decimal from
the coefficient.
4 Factorise. (d − 20)(d − 10) = 0
5 Apply the Null Factor Law. d − 20 = 0 or d − 10 = 0
6 Solve. d = 20
     d = 10
7 Interpret the solution. The ball The ball first reaches a height of
reaches a height of 20 m on the 20 m after it has travelled a distance
way up and the way down. The of 10 m.
first time the ball reaches a height
of 20 m is the smaller value of d.
Answer in a sentence.

270 Maths Quest 10 New South Wales Australian curriculum edition Stages 5.1 and 5.2
number and algebra

Exercise 8A Solving quadratic equations algebraically


INDIVIDUAL PATHWAYS
Questions: Questions: Questions:
1a–f, 2a–c, 3a–d, 4a–c, 5a–h, 1b–g, 2a–d, 3a–f, 4a–f, 5d–l, 1, 2, 3c–f, 4d–i, 5d–o, 6, 7, 9,
6, 7, 8, 10, 14 6, 7, 9, 11, 14–16 12–16
Activity 8‐A‐1 Activity 8‐A‐2 Activity 8‐A‐3
Solving simple quadratics Solving quadratic Solving more complex
doc‐5059 equations quadratics
doc‐5060 doc‐5061

FLUENCY
1 WE1 Solve each of the following equations.
a (x + 7)(x − 9) = 0 b (x − 3)(x + 2) = 0 c (x − 2)(x − 3) = 0
d x(x − 3) = 0 e x(x − 1) = 0 f x(x + 5) = 0 Digital docs
1 1 SkillSHEET
g 2x(x − 3) = 0 h 9x(x + 2) = 0 i 1x − 22 1x + 22 = 0 Factorising by taking
out the highest
2 Solve each of the following equations. common factor
a (2x − 1)(x − 1) = 0 b (3x + 2)(x + 2) = 0 c (4x − 1)(x − 7) = 0 doc‐5256
SkillSHEET
d (7x + 6)(2x − 3) = 0 e (5x − 3)(3x − 2) = 0 f (8x + 5)(3x − 2) = 0 Finding a factor pair
that adds to a given
3 WE2a Solve each of the following equations. number
doc‐5257
a x2 − 2x = 0 b x2 + 5x = 0 c x2 = 7x SkillSHEET
d 3x2 = −2x e 4x2 − 6x = 0 f 6x2 − 2x = 0 Simplifying surds
doc‐5258
4 WE2b Solve each of the following equations. SkillSHEET
Substituting into
a x2 − 4 = 0 b x2 − 25 = 0 c − 12 = 0
3x2 ­quadratic equations
d 4x2 − 196 = 0 e 9x2 − 16 = 0 f 4x − 25 = 0
2 doc‐5259
SkillSHEET
1
g 9x2 = 4 h 36x2 = 9 i x2 − 25 = 0 Equation of a vertical
line
5 WE2c Solve each of the following equations. doc‐5260

a x2 − x − 6 = 0 b x2 + 6x + 8 = 0 c x2 − 6x − 7 = 0
d x2 − 8x + 15 = 0 e x2 − 2x + 1 = 0 f x2 − 3x − 4 = 0
g x2 − 10x + 25 = 0 h x2 − 3x − 10 = 0 i x − 8x + 12 = 0
2

j x2 − 4x − 21 = 0 k x2 − x − 30 = 0 l x2 − 7x + 12 = 0
m x2 − 8x + 16 = 0 n x2 + 10x + 25 = 0 o x2 − 20x + 100 = 0
6 MC The solutions to the equation x2 + 9x − 10 = 0 are:
A x = 1 and x = 10 B x = 1 and x = −10 C x = −1 and x = 10
D x = −1 and x = −10 E x = 1 and x = 9
7 MC The solutions to the equation x2 − 100 = 0 are:
A x = 0 and x = 10 B x = 0 and x = −10 C x = −10 and x = 10
D x = 0 and x = 100 E x = −100 and x = 100

UNDERSTANDING
8 WE3 When two consecutive numbers are multiplied, the result is 72. Find the
numbers.
9 When two consecutive even numbers are multiplied, the result is 48. Find the
numbers.

Chapter 8 • Quadratic equations 271


number and algebra

10 When a number is added to its square the result is 90. Find the number.
11 Twice a number is added to three times its square. If the result is 16, find the
number.
12 Five times a number is added to two times its square. If the result is 168, find the
number.
13 WE4 A soccer ball is kicked. The height, h, in metres, of the soccer
ball t seconds after it is kicked can be represented by the equation
h = −t(t − 6). Find how long it takes for the soccer ball to hit the
ground again.
14 If the length of a paddock is 2 m more than its width and the area is
48 m2, find the length and width of the paddock.
15 Solve the following for x.
24 1
a x+5=6 b x= c x=
x x−5 x

REASONING
16 Henrietta is a pet rabbit who lives in
an enclosure that is 2 m wide and
4 m long. Her human family has
decided to purchase some more
rabbits to keep her company and so
the size of the enclosure must be
increased.
a Draw a diagram of Henrietta’s
enclosure, clearly marking the
lengths of the sides.
b If the length and width of the
enclosure are increased by x m,
find the new dimensions.
c If the new area is to be 24 m2,
write an equation relating the sides
and the area of the enclosure
(Area = length × width).
REFLECTION
Digital docs d Use the equation to find the value of x and, hence,
What does the Null Factor
WorkSHEET 8.1 the length of the sides of the new enclosure. Justify Law mean?
doc‐13733
your answer.

272 Maths Quest 10 New South Wales Australian curriculum edition Stages 5.1 and 5.2
number and algebra

8B  Solving quadratic equations graphically


•• The graph of y = ax2 + bx + c is in the shape of a parabola. y
•• The graph can be used to locate the solutions to quadratic y = ax2 + bx + c
equations such as ax2 + bx + c = 0.

(0, c)

0 x

Solutions to ax2 + bx + c = 0
WORKED EXAMPLE 5
Determine the solution’s of each of the following quadratic equations by inspecting their
corresponding graphs. Give answers to 1 decimal place where appropriate.
a x2 + x − 2 = 0 b 2x2 − 4x − 5 = 0

TH I N K W R I TE /DRAW
a 1 Examine the graph of y = x2 + x − 2 and locate a y
3
the points where y = 0, that is, where the graph 2
intersects the x‐axis. 1

–3 –2 –1–101 2 3x
–2 y = x + x – 2
2

–3

2 The graph cuts the x‐axis (y = 0) at x = 1 and x2 + x − 2 = 0


x = −2. Write the solutions. From the graph, the solutions are
x = 1 and x = −2.
b 1 Examine the graph of y = 2x2 − 4x − 5 and locate b y
6
the points where y = 0, that is, where the graph 4
intersects the x‐axis. 2

–3 –2 –1–20 1 2 3x
–4
–6
–8

2 The graph cuts the x‐axis (y = 0) at approximately 2x2 − 4x − 5 = 0


x ≈ −0.9 and x ≈ 2.9. Write the solutions. From the graph, the solutions are
x ≈ −0.9 and x ≈ 2.9.

Quadratic equations with only 1 solution


y
•• Some quadratic equations have only one solution. For example, the graph y = x2 – 4x + 4
of y = x2 − 4x + 4 has only one solution at x = 2. That is, the graph of 5
y = x2 − 4x + 4 touches the x‐axis only at x = 2.

–2 0 2 4 x

Chapter 8 • Quadratic equations 273


number and algebra

Quadratic equations with no solutions y

•• There are also quadratic equations that have no real solutions. 10


For example, the graph of y = 3x2 − 4x + 4 does not intersect
the x‐axis and so 3x2 − 4x + 4 = 0 has no real solutions (that 5
is, no solutions that are real numbers).
y = 3x2 – 4x + 4
0 x
–2 2

Confirming solutions
•• It is possible to confirm the solutions obtained by sight. This is achieved by
substituting the solution or solutions into the original quadratic equation and
determining whether they make a true statement.

WORKED EXAMPLE 6
Confirm, by substitution, the solutions obtained in Worked example 5a.
x2 + x − 2 = 0; solutions: x = 1 and x = −2

TH I N K W R I TE
1 Write the left‐hand side of the When x = 1,
equation and substitute x = 1 into LHS: x2 + x − 2 = 12 + 1 − 2
the expression. =0
2 Write the right‐hand side. RHS: 0
3 Confirm the solutions. LHS = RHS⇒ Solution is confirmed.
4 Write the left‐hand side and When x = −2,
substitute x = −2. LHS: x2 + x − 2 = (−2) 2 + −2 − 2
=4−2−2
=0
5 Write the right‐hand side. RHS = 0
6 Confirm the solutions. LHS = RHS⇒ Solution is confirmed.

WORKED EXAMPLE 7
A golf ball hit along a fairway follows the path shown in the graph. The
height, h metres after it has travelled x metres horizontally, follows the rule
1
h = −270 (x2 − 180x). Use the graph to find how far the ball landed from the
golfer. h 1
h = – 270 (x – 180x)
––– 2
30

20

10

0 90 180 x

274 Maths Quest 10 New South Wales Australian curriculum edition Stages 5.1 and 5.2
number and algebra

TH I N K W R I TE
On the graph, the ground is represented The golf ball lands 180 m from the golfer.
by the x‐axis since this is where h = 0.
The golf ball lands when the graph
intersects the x‐axis.

Exercise 8B Solving quadratic equations graphically


INDIVIDUAL PATHWAYS
Questions: Questions: Questions:
1a–d, 2–4 1c–h, 2–4 1e–j, 2–6
Activity 8‐B‐1 Activity 8‐B‐2 Activity 8‐B‐3
Finding solutions to Solving quadratic Harder solutions to
quadratic equations by equations by quadratic equations by
inspecting graphs inspecting graphs inspecting graphs
doc-13730 doc-13731 doc-13732

FLUENCY
1 WE5 Determine the solutions of each of the following quadratic equations by
inspecting the corresponding graphs. Give answers correct to 1 decimal place where
appropriate.
a x2 − x − 6 = 0 b x2 − 11x + 10 = 0
y y
12 8
8
–2 0 2 4 6 8 10 12 x
4 –8
–16
–6 –4 –2 0 2 4 6 x
–4 –24 y = x2 – 11x + 10
–8 y = x – x – 6
2

c −x2 + 25 = 0 d 2x2 − 8x + 8 = 0
y y
y = –x2 + 25 y = 2x2 – 8x + 8
30 20
20 10
10
–1 0 1 2 3 4 5 x
–10
–6 –4 –2 0 2 4 6 x
–10

e x2 − 3x − 4 = 0 f x2 − 3x − 6 = 0
y y
15 y = x – 3x – 4
2
15 y = x2 – 3x – 6
10 10
5 5
–1 0 1 2 3 4 5 6 x –1 0 1 2 3 4 5 6 x
–10 –10

Chapter 8 • Quadratic equations 275


number and algebra

g x2 + 15x − 250 = 0 h −x2 = 0


y
200
100 y
–30 –20 –10 0 10 x 5 y = –x2
–100 0
–200 –5 5 x
–300 –5
–400 –10
y = x2 + 15x – 250

i x2 + x − 3 = 0 j 2x2 + x − 3 = 0

5 5

–4 –2 0 2 4 x –2 –1 0 1 2 x

y = x2 + x – 3 y = 2x2 + x – 3
–5 –5

UNDERSTANDING
2 WE6 Confirm, by substitution, the solutions obtained in question 1.
3 WE7 A golf ball hit along a fairway follows the path shown in the graph.

h h = – –––
200 (x – 150x)
1 2
28

0 75 150 x

The height, h metres, after it has travelled x metres horizontally follows the
1
rule h = −200 (x2 − 150x). Use the graph to find how far the ball lands from
the golfer.
4 A ball is thrown upwards from a building and follows the path shown in the graph
until it lands on the ground.
h h = –x2 + 4x + 21
25
21

0 2 7 x

The ball is h metres above the ground when it is a horizontal distance of x metres
from the building. The path of the ball follows the rule h = −x2 + 4x + 21. Use the
graph to find how far from the building the ball lands.

276 Maths Quest 10 New South Wales Australian curriculum edition Stages 5.1 and 5.2
number and algebra

REASONING
The x‐intercepts of a particular equation are x = 2, 5. Suggest a possible equation.
5 a 
b If the y‐intercept in question 5a is (0, 4), give the exact equation in question 5a.
The x‐intercepts of a particular equation are x = p, q.
6 a 
Suggest a possible equation. REFLECTION
What does ‘the solution of Digital doc
b If the y‐intercept in question 6a is (0, r), give the a graph’ mean?
Worksheet 8.2
doc-13735
exact equation in question 6a.

CHALLENGE 8.2

Chapter 8 • Quadratic equations 277


number and algebra

CHAPTER REVIEW
LANGUAGE

coefficient factorise parabola solutions


curve linear equation product solve
expand null factor law quadratic equation substitution

int‐2847 doc‐13737
int‐2848 doc‐13738
int‐3595

FLUENCY
1 The solutions to the equation x2 + 10x − 11 = 0 are:
A x = 1 and x = 11 B x = 1 and x = −11 C x = −1 and x = 11
D x = −1 and x = −11 E x = 1 and x = 10
2 Solve each of the following quadratic equation by first factorising the left‐hand side
of the equation.
a x2 + 8x + 15 = 0 b x2 + 7x + 6 = 0
c x2 + 11x + 24 = 0 d x2 + 4x − 12 = 0
e x2 − 3x − 10 = 0 f x2 + 3x − 28 = 0
g x2 − 4x + 3 = 0 h x2 − 11x + 30 = 0
3 Solve each of the following quadratic equations.
a 2x2 + 16x + 24 = 0 b 3x2 + 9x + 6 = 0
c 5x2 + 25x − 70 = 0 d 2x2 − 6x + 4 = 0
4 The graph of y = x2 − 4x − 21 is shown.

y = x2 – 4x – 21
y

–4 –2 0 2 4 6 x
5
10

–21
(2, –25)

Use the graph to find the solutions to the quadratic equation x2 − 4x − 21 = 0.

278 Maths Quest 10 New South Wales Australian curriculum edition Stages 5.1 and 5.2
number and algebra

5 Determine the roots of the quadratic graph shown.

y = –2x2 – 4x + 6
y
10
5

–6 –4 –2 0 2 4 6 x
–5
–10

PROBLEM SOLVING
1 When a number is added to its square, the result is 56. Determine the number.
2 Leroy measure his bedroom and finds that its length is 3 metres more than its width.
If the area of the bedroom is 18 m2, calculate the length and width of the room.
3 A platform diver follows a path determined by the
equation h = −0.5d2 + 2d + 6, where h represents the
height of the diver above the water and d represents the
distance from the diving board. Both pronumerals are
measured in metres.
h
8
6
4
2

–4 –2–20 2 4 6 8 d

Use the graph to determine:


a how far the diver landed from the edge of the pool
b how high the diving board was above the water.
4 While it requires a minimum of 2 points to determine the graph of a line, it requires
a minimum of 3 points to determine the shape of a parabola. The general equation of
a parabola is y = ax2 + bx + c, where a, b and c are the constants to be determined.
a Determine the equation of the parabola that has a y‐intercept of (0, −2), and
passes though the points (1, −5) and (−2, 16).
b Determine the equation of a parabola that goes through the points (0, 0), (2, 2)
and (5, 5). Show full working to justify your answer.

Chapter 8 • Quadratic equations 279


number and algebra
Communicating

Rich task

Weaving

We need to understand the process of weaving. Block 1


Weaving machines have parts called warps. Each warp is
divided into a number of blocks. Consider a pattern that is Block 2
made up of a series of blocks, where the first block is all
one colour except for the last thread, which is a different Block n
colour. Let’s say our pattern is red and blue. The first
block contains all red threads, except for the last one, which is blue. The next block has all red
threads, except for the last two threads, which are blue. The pattern continues in this manner.
The last block has the first thread as red and the remainder as blue. The warp consists of a
particular number of threads, let’s say 42 threads. How many blocks and how many threads
per block would be necessary to create a pattern of this type?
To produce this pattern, we need to divide the warp into equally sized blocks, if possible.
What size block and how many threads per block would give us the 42‐thread warp? We will
need to look for a mathematical pattern. Look at the table on the opposite page, where we
consider the smallest block consisting of 2 threads through to a block consisting of 7 threads.

280 Maths Quest 10 New South Wales Australian curriculum edition Stages 5.1 and 5.2
number and algebra

1 Complete the entries in the table.


2 Consider a block consisting of n threads.
a How many blocks would be needed?
b What would be the total number of threads
in the warp?
The 42‐thread warp was chosen as a simple
example to show the procedure involved in
determining the number of blocks required
and the number of threads per block.
In this particular case, 6 blocks of 7 threads
per block would give us our design for a
42‐thread warp. In practice, you would not
approach the problem by drawing up a table
to determine the number of blocks and the
size of each block.
3 Take your expression in question 2b and let it equal 42. This should form a quadratic equation.
Solve this equation to verify that you would need 6 blocks with 7 threads per block to fulfil the
size of a 42‐thread warp.
4 In reality, the size of each block is not always clearly defined. Also, the thread warp sizes are
generally much larger, say, about 250. Let’s determine the number of threads per block and the
number of blocks required for a 250‐thread warp.
a Form your quadratic equation with the thread warp size equal to 250.
b A solution to this equation can be found using the quadratic formula. Use the quadratic
formula to determine a solution.
c The number of threads per block is represented by n and this obviously must be a whole
number. Round your solution down to the nearest whole number.
d How many whole blocks are needed?
e Use your solutions to c and d to determine the total number of threads used for the pattern.
f How many more threads do you need to make the warp size equal to 250 threads?
g Distribute these threads by including them at the beginning of the first block and the end of
the last block. Describe your overall pattern.

Chapter 8 • Quadratic equations 281


number and algebra

Code puzzle

Aussie fact
Factorise each of the following, and then solve for x to
discover the puzzle code.

A x 2 – 4x = 0 N 2x 2 – 2x – 4 = 0

C x 2 + 7x = 0 O 3 x 2 + 12x = 0

D x 2 + 6x + 5 = 0 R 3 x 2 – 12 = 0

E x 2 + 2x + 1 = 0 S x 2 + 7x + 10 = 0

F x 2 – 8x + 12 = 0 T 3 x 2 – 3x – 18 = 0

H x 2 – 81 = 0 U x 2 – 8x + 16 = 0

I x 2 – 16x + 64 = 0 W x 2 – 9x + 20 = 0

L x 2 – 2x – 3 = 0 X x 2 + 6x – 27 = 0

3, –2 9, –9 –1 0, 4 4 –2, –5 3, –2 2, –2 0, 4 3, –1 8 0, 4 2, –1 6, 2 –1

–5, –1 –1 2, –2 0, 4 3, –2 8 0, –4 2, –1 0, –7 0, –4 2, –1 –2, –5 8 –2, –5 3, –2

–2, –5 0, –4 6, 2 –2, –5 8 –9, 3 –2, –5 3, –2 0, 4 3, –2 –1 –2, –5 0, 4 2, –1 –5, –1

3, –2 4, 5 0, –4 3, –2 –1 2, –2 2, –2 8 3, –2 0, –4 2, –2 8 –1 –2, –5

282 Maths Quest 10 New South Wales Australian curriculum edition Stages 5.1 and 5.2
number and algebra

activities
Go to assessON
Chapter opener 8B Solving quadratic equations for questions to
Video graphically test your readiness
• The story of mathematics (eles‐1847) Digital docs FOR learning, your
• Activity 8‐B‐1 (doc‐13730): Finding solutions progress AS you learn
8A Solving quadratic equations
to quadratic equations by inspecting and your levels OF
algebraically
graphs (page 275) achievement.
Digital docs
• Activity 8‐B‐2 (doc‐13731): Solving quadratic www.assesson.com.au
• Activity 8‐A‐1 (doc‐5059): Solving simple
equations by inspecting graphs (page 275)
quadratics (page 271)
• Activity 8‐B‐3 (doc‐13732): Harder solutions
• Activity 8‐A‐2 (doc‐5060): Solving quadratic
to quadratic equations by inspecting
equations (page 271)
graphs (page 275)
• Activity 8‐A‐3 (doc‐5061): Solving more
• WorkSHEET 8.2 (doc-13735) (page 277)
complex quadratics (page 271)
• SkillSHEET (doc‐5256): Factorising by taking Chapter review
out the highest common factor (page 271) Interactivities  (page 278)
• SkillSHEET (doc‐5257): Finding a factor pair • Word search (int‐2847)
that adds to a given number (page 271) • Crossword (int‐2848)
• SkillSHEET (doc‐5258): Simplifying • Sudoku (int‐3595)
surds (page 271) Digital docs
• SkillSHEET (doc‐5259): Substituting into • Chapter summary (doc‐13737)
quadratic equations (page 271) • Concept map (doc‐13738)
• SkillSHEET (doc‐5260): Equation of
To access eBookPLUS activities, log on to
a vertical line (page 271)
www.jacplus.com.au
• WorkSHEET 8.1 (doc‐13733): Solving
quadratic equations (page 272)

Chapter 8 • Quadratic equations 283


number and algebra

Answers
Chapter 8 Quadratic equations
Exercise 8A — Solving quadratic equations algebraically Challenge 8.2
1    a −7, 9 b −2, 3 c 2, 3 7 years old
d 0, 3 e 0, 1 f −5, 0
Chapter review
g 0, 3 h −2, 0 i −12, 12
Fluency
1 1 B
2    a ,1 b −2, −23 c 1
,7
2 4 2    a −5, −3 b −6, −1 c −8, −3
d −67, 112 e 3 2
,
5 3
f −58, 23 d 2, −6 e 5, −2 f 4, −7
g 3, 1 h 5, 6
3    a 0, 2 b −5, 0 c 0, 7 3    a −2, −6 b −2, −1 c 2, −7 d 2, 1
d −23, 0 e 0, 112 f 0, 13 4 −3, 7
5 −3, 1
4    a −2, 2 b −5, 5 c −2, 2 Problem solving
d −7, 7 e −113, 113 f −212, 212 1 −8 and 7
2 Length = 6 m, width = 3 m
g −23, 23 h −12, 12 i −15, 15 3    a 6 m b 6m
5    a −2, 3 b −4, −2 c −1, 7 4    a y = 2x2 − 5x − 2
d 3, 5 e 1 f −1, 4 b No parabola is possible. The points are on the same
g 5 h −2, 5 i 2, 6 straight line.
j −3, 7 k −5, 6 l 3, 4 Communicating — Rich task
m −4 n −5 o 10 1
6 B Number Number Total
7 C of threads of threads
8 8 and 9 or −8 and −9 Pattern per block blocks in warp
9 6 and 8, −6 and −8 RB 2 1 2
10 9 or −10
11 2 or −223 RRB RRB 3 2 6
12 8 or −1012 RRRB RRBB 4 3 12
13 6 seconds RBBB
14 8 m, 6 m
15    a −6, 1 b 8, −3 c ±1 RRRRB RRRBB 5 4 20
16    a RRBBB RBBBB
2m
RRRRRB RRRRBB 6 5 30
4m
RRRBBB RRBBBB
b (2 + x) m, (4 + x) m RBBBBB
c (2 + x) (4 + x) = 24 RRRRRRB 7 6 42
d x = 2, 4 m wide, 6 m long RRRRRBB
Challenge 8.1 RRRRBBB
The width of the pathway is 1.5 m. RRRBBBB
RRBBBBB
Exercise 8B — Solving quadratic equations graphically RBBBBBB
1    a x = −2, x = 3 b x = 1, x = 10
c x = −5, x = 5 d x=2
e x = −1, x = 4 f x ≈ −1.4, x ≈ 4.4 2    a n − 1 b n2 − n
g x = −25, x = 10 h x=0 3 Teacher to check
!1001 + 1
i x ≈ −2.3, x ≈ 1.3 j x ≈ −1.5, x = 1 4    a n2 − n = 250 b n=
2
2 a–j Confirm by substitution of the above values into quadratic
equations. c n = 16 d 15
3 150 m e 240 f 10
4 7m g Answers will vary.
5    a y = a(x − 2) (x − 5)
Code puzzle
b y = 25 (x − 2) (x − 5) The Australian federation consists of six states and two territories.
6    a y = a(x − p) (x − q)
r
b y = (x − p) (x − q)
pq

284 Maths Quest 10 New South Wales Australian curriculum edition Stages 5.1 and 5.2
number and
NUMBER AND ALGEBRA
algebra

Chapter 9

Non-linear
relationships
WHY LEARN THIS?
The world around us, both natural and artificial, is full
of elegant curves and shapes that are not straight
lines. Achieving the brilliance of modern and ancient
architecture, or understanding the motion of planets
and tennis balls requires an understanding of non-
linear relationships.

WHAT DO YOU KNOW?


1 Think List what you know about
non-linear relationships. Use a thinking tool
such as a concept map to show your list.
2 pair Share what you know with a partner
and then with a small group.
3 share As a class, create a large concept
map that shows your class’s knowledge
of non-linear relationships.

LEARNING SEQUENCE
9A Plotting parabolas
9B Sketching parabolas
9C Exponential functions and graphs
9D The circle
Watch this video
The story of mathematics

Searchlight ID: eles-1848


number and algebra

9A  Plotting parabolas


•• The graphs of all quadratic relationships are called parabolas.
•• The graph of y = x2 shown below has been produced by generating a table
of values.
y y = x2
x −3 −2 −1 0 1 2 3 10

8
y 9 4 1 0 1 4 9
6

–6 –5 –4 –3 –2 –1 0 1 2 3 4 5 6
x
–2 (0, 0)

•• Parabolas are symmetrical; in other words, they have an axis of symmetry. In the
parabola above, the axis of symmetry is the y-axis, also called the line x = 0.
•• A parabola has a vertex or turning point. In this case the vertex is at the origin and is
called a ‘minimum turning point’.
•• Parabolas with the shape ∪ are said to be ‘concave up’ and have a minimum
turning point. Parabolas with the shape ∩ are said to be ‘concave down’ and have a
maximum turning point.

Parabolas in the world around us


•• Parabolas abound in the world around us. Here are some examples.

Satellite dishes Water droplets from a hose

Circle
Ellipse
Parabola

Hyperbola

The cables from a suspension bridge

A cone when sliced parallel to its edge


reveals a parabola.

288 Maths Quest 10 New South Wales Australian curriculum edition Stages 5.1 and 5.2
number and algebra

The trajectory of a football when it is


thrown or kicked

Reflector

Bulb

      
The reflectors in a car headlight

WORKED EXAMPLE 1
Plot the graph of each of the following equations. In each case, use the
values of x shown as the values in your table. State the equation of the axis
of symmetry and the coordinates of the turning point.
1
a y = 2x2 for −3 ≤ x ≤ 3 b y = 2x2 for −3 ≤ x ≤ 3

TH I N K W R I TE /DRAW
a 1 Write the equation. a y = 2x2
2 Produce a table of values x −3 −2 −1 0 1 2 3
using x-values from −3 y 18 8 2 0 2 8 18
to 3.

Chapter 9 • Non-linear relationships 289


number and algebra

3 Draw a set of clearly y


20
labelled axes, plot the
18
points and join them with
a smooth curve. The scale 16
would be from 20 to −2 on 14
the y-axis and −4 to 4 on 12
the x-axis. y = 2x2
10
8
4 Label the graph.
6
4
2

–4–3–2–10 1 2 3 4 x
–2

5 Write the equation of the The equation of the axis of symmetry is


axis of symmetry that x = 0.
divides the parabola exactly
in half.
6 Write the coordinates of The turning point is (0, 0).
the turning point.
1
b 1 Write the equation. b y = x2
2

2 Produce a table of values x −3 −2 −1 0 1 2 3


using x-values from −3
y 4.5 2 0.5 0 0.5 2 4.5
to 3.
3 Draw a set of clearly y
labelled axes, plot the 6
points and join them with 5
a smooth curve. The scale 4 y = 1–2 x2
would be from 6 to −2 on 3
the y-axis and −4 to 4 on 2
the x-axis. 1
4 Label the graph.
–4 –3 –2 –1 0 1 2 3 4 x
–1
–2

5 Write the equation of The equation of the axis of symmetry is


the line that divides the x = 0.
parabola exactly in half.
6 Write the coordinates of The turning point is (0, 0).
the turning point.

290 Maths Quest 10 New South Wales Australian curriculum edition Stages 5.1 and 5.2
number and algebra

WORKED EXAMPLE 2
Plot the graph of each of the following equations. In each case, use the values of x shown as
the values in your table. State the equation of the axis of symmetry, the coordinates of the
turning point and the y-intercept for each one.
a y = x2 + 2 for −3 ≤ x ≤ 3 b y = (x + 3) 2 for −6 ≤ x ≤ 0 c y = −x2 for −3 ≤ x ≤ 3

TH I N K W R I TE /DRAW
a 1 Write the equation. a y = x2 + 2
2 Produce a table of values. x −3 −2 −1 0 1 2 3
y 11 6 3 2 3 6 11

3 Draw a set of clearly labelled axes, plot the y


points and join them with a smooth curve. 12
The scale on the y-axis would be from 0 to 9
12 and −4 to 4 on the x-axis.
6
y = x2 + 2
3
(0, 2)
4 Label the graph.
–4 –2 0 2 4 x

5 Write the equation of the line that divides The equation of the axis of symmetry is
the parabola exactly in half. x = 0.
6 Write the coordinates of the turning point. The turning point is (0, 2).
7 Find the y-coordinate of the point where The y-intercept is 2.
the graph crosses the y-axis.
b 1 Write the equation. b y = (x + 3) 2
2 Produce a table of values. x −6 −5 −4 −3 −2 −1 0
y 9 4 1 0 1 4 9

3 Draw a set of clearly labelled axes, y


plot the points and join them with a 10
(0, 9)
smooth curve. The scale on the y-axis 8
would be from 0 to 10 and −7 to 1 on y = (x + 3)2 6
the x-axis. 4
(–3, 0)
4 Label the graph. 2

–7–6–5–4–3–2–1 0 1 x

5 Write the equation of the line that divides The equation of the axis of symmetry is
the parabola exactly in half. x = −3.
6 Write the coordinates of the turning The turning point is (−3, 0).
point.

Chapter 9 • Non-linear relationships 291


number and algebra

7 Find the y-coordinate of the point where The y-intercept is 9.


the graph crosses the y-axis.
c 1 Write the equation. c y = −x2
2 Produce a table of values. x −3 −2 −1 0 1 2 3
y −9 −4 −1 0 −1 −4 −9
3 Draw a set of clearly labelled axes, y
plot the points and join them with a –4 –2 0 2 4 x
smooth curve. The scale on the y-axis
–3
would be from −10 to 1 and from −4
to 4 on the x-axis. –6
4 Label the graph. y = –x2
–9

5 Write the equation of the line that divides The equation of the axis of symmetry is x = 0.
the parabola exactly in half.
6 Write the coordinates of the turning The turning point is (0, 0).
point.
7 Find the y-coordinate of the point where the The y-intercept is 0.
graph crosses the y-axis.

Exercise 9A Plotting parabolas


INDIVIDUAL PATHWAYS
Questions: Questions: Questions:
1, 2, 3a–b, 4a–b, 5, 7 1, 2, 3b–c, 4b–c, 5, 6, 7 1, 2, 3b–d, 4b–d, 5, 6, 7
Activity 9-A-1 Activity 9-A-2 Activity 9-A-3
Review of plotting parabolas Plotting parabolas Trends in plotting parabolas
doc-13739 doc-13740 doc-13741

Digital docs FLUENCY


SkillSHEET
Substitution into
1 WE1 Plot the graph of each of the following equations. In each case, use the values
­quadratic equations of x shown as the values in your table. State the equation of the axis of symmetry
doc-5266
SkillSHEET
and the coordinates of the turning point.
Equation of a a y = 3x2 for −3 ≤ x ≤ 3
­vertical line
doc-5267 1
b y = 4x2 for −3 ≤ x ≤ 3

2 WE2a Plot the graph of each of the following for values of x between −3 and 3.
State the equation of the axis of symmetry, the coordinates of the turning point and
the y-intercept for each one.
a y = x2 + 1 b y = x2 + 3 c y = x2 − 3 d y = x2 − 1

292 Maths Quest 10 New South Wales Australian curriculum edition Stages 5.1 and 5.2
number and algebra

3 WE2b Plot the graph of each of the following equations. In each case, use the
values of x shown as the values in your table. State the equation of the axis of
symmetry, the coordinates of the turning point and the y-intercept for each one.
a y = (x + 1) 2 −5 ≤ x ≤ 3 b y = (x + 2) 2 −6 ≤ x ≤ 2
c y = (x − 2) 2
−1 ≤ x ≤ 5 d y = (x − 1) 2
−2 ≤ x ≤ 4
4 WE2c Plot the graph of each of the following equations. In each case, use the
values of x shown as the values in your table. State the equation of the axis of
symmetry, the coordinates of the turning point and the y-intercept for each one.
a y = −x2 + 1 −3 ≤ x ≤ 3 b y = −(x − 1) 2 −2 ≤ x ≤ 4
c y = −(x + 2) 2
−5 ≤ x ≤ 1 d y = −x − 3
2
−3 ≤ x ≤ 3

UNDERSTANDING
5 A ball is thrown into the air. The height, h metres, of
the ball at any time, t seconds, can be found by using
the equation h = −(t − 4) 2 + 16.
a Plot the graph for values of t between 0 and 8.
b Use the graph to find:
i the maximum height of the ball
ii how long it takes for the ball to fall back to the
ground.

6 From a crouching position in a ditch, an archer wants


to fire an arrow over a horizontal tree branch, which is
15 metres above the ground. The height, in metres (h),
of the arrow t seconds after it has been fired
is given by the equation h = −8t(t − 3).
a Plot the graph for t = 0, 1, 1.5, 2, 3.
b From the graph find:
i the maximum height the arrow reaches
ii whether the arrow clears the branch and the distance
by which it clears or falls short of the branch
iii the time it takes to reach maximum height
iv how long it takes for the arrow to hit the ground.

REASONING
7 There are 0, 1, 2 and infinite possible points of
intersection for two parabolas.
a Illustrate these on separate graphs. REFLECTION
b Explain why infinite points of intersection are What x-values can a
possible. Give an example. parabola have? What
y-values can a parabola
c How many points of intersection are possible have?
for a parabola and a straight line? Illustrate these.

Chapter 9 • Non-linear relationships 293


number and algebra

9B  Sketching parabolas


Interactivity
Dilation of Sketching parabolas
y = x2
int-1148 •• A sketch graph of a parabola does not show a series of plotted points, but it does
accurately locate important features such as x- and y-intercepts and turning points.
•• The basic quadratic graph has the equation y = x2. Transformations or changes
in the features of the graph can be observed when the equation changes. These
transformations include:
–– dilation
–– translation y y = 2x2
–– reflection. 8
7

Dilation: The family of parabolas y = ax2 6


y = x2
5
•• The diagram at right shows the graphs of y = x 2
4
and y = 2x2.
3
•• The graphs are both parabolas and have the
same vertex. 2
•• The graph of y = 2x2 climbs twice as steeply as the 1
graph of y = x2.
–3 –2 –1 0 1 2 3 x
•• y = 2x2 has a dilation factor of 2.
•• A higher positive dilation factor produces a steeper y
graph, or one which is further away from the x-axis. y = 2x2
7
y = x2
•• Adding the graph of y = 12x2 to the diagram shows 6
that this graph is flatter than the graph of y = x2. 5
•• y = 12x2 has a dilation factor of 12. 4
•• The turning point of all these graphs is (0, 0). 3
2 y = —12 x2
1

–3 –2 –1 0 1 2 3 x

Reflection: The family of


parabolas y = ax2 where a is 5
y

a negative number 4 y = x2
•• The diagram at right shows the graphs 3
of y = x2 and y = −x2. 2
•• The graphs are both parabolas and have 1
the same vertex.
•• The graph of y = −x2 is a reflection about –5 –4 –3 –2 –1–10 1 2 3 4 5
x
the x-axis of the graph of y = x2.
•• The graph of y = x2 is said to be –2

‘concave up’ and has a minimum –3


y = ‒x2
turning point. –4
–5

294 Maths Quest 10 New South Wales Australian curriculum edition Stages 5.1 and 5.2
number and algebra

•• The graph of y = −x2 is said to be ‘concave y


5
down’ and has a maximum turning point.
4 y = 2x2
•• Similarly, the graph of y = −2x2 is a reflection
3
of the graph
2
of y = 2x2.
1 y = x2
•• In general, the graph of y = −ax2 is a
reflection about the
–5 –4 –3 –2 –1 0 1 2 3 4 5 x
x-axis of the graph of y = ax2. –1
•• The turning point of all these graphs is (0, 0). y = ‒2x2
–2
–3
Translation: The family of –4 y = ‒x2
parabolas y = x2 + c –5

•• The diagram at right shows the graphs of


y = x2 and y = x2 + 3. y = x2 + 3 y
10
•• The graphs have identical shape.
•• Although they appear to get closer to each 9

other, the graphs are constantly 3 units apart. 8 y = x2

•• When the graph of y = x2 is translated 7


3 units upward, it becomes the graph of 6
3 units
y = x2 + c. 5
•• ‘Translated’ means ‘moved without turning’. 4
3
3 units
2
1

x
–5 –4 –3 –2 –1–10 1 2 3 4 5
–2
•• Adding the graph of y = x2 − 1 to the
diagram shows that this graph is also the same
y y = x2 + 3
shape as y = x2 but has been translated 1 unit 10
downwards. 9
•• In general, the graph of y = x2 + c is the same 8
as y = x2 but translated vertically a distance of 7
c units. 6
5
4
3 y = x2
2 y = x2 ‒ 1
1

–5 –4 –3 –2 –1 0 1 2 3 4 5 x
–1

WORKED EXAMPLE 3
On the same axes, sketch the graphs of:
a y = x2 b y = x2 + 3
1
c y = 3x2 d y = − 3 x2

Chapter 9 • Non-linear relationships 295


number and algebra

TH I N K DRAW
a y = x2 is a parabola with vertex at the origin. Sketch y
the graph. 4
y = x2 + 3
b y = x + 3 is the same shape as y = x but is translated
2 2 y = 3x2 2
y = x2
3 units vertically up. Sketch the graph.
–4 –2 0 2 4 x
c y = 3x2 has the same vertex as y = x2 but is three times –2
as steep. Sketch the graph. –4
d y= −13x2 is concave down with the same vertex as
y = – —13 x2
y=x 2 but is only one third as steep. Sketch the graph.

Combining transformations 5
y

Interactivity
•• It is possible to combine translations, 4
y = x2
3
Vertical translation of dilations and reflections in the one graph. 2
1
y = x2+ c
int-1192
•• Consider the parabola given by y = −2x2 + 4.
•• Compared with the graph of y = x2, it has been –5 –3 –10 1 2 3 4 5 x
–2
reflected, dilated with a factor of 2 (it is steeper) –3
–4
and translated vertically 4 units upward. –5
y = −2x2+ 4

WORKED EXAMPLE 4
By considering transformations to the graph of y = x2, sketch the
graph of y = −12x2 + 2.

TH I N K W R I TE /DRAW
1 The dilation factor is 12. Comment on the The graph is less steep.
effect that this transformation has on the
graph of y = x2.
2 The coefficient of x2 is negative, so the The graph is inverted and
graph is reflected in the x-axis. Comment has a maximum turning
on the effect that this transformation has on point.
the graph of y = x2.
3 c = 2. Comment on the effect that this The graph has been
transformation has on the graph of y = x2. translated 2 units upwards.
4 Sketch the graphs of y = x2 and y = −12x2 + 2 y
on the same set of axes. 4 y = x2
2

–4 –2 0 2 4 x
–2
–4

y = − —12 x2 + 2

296 Maths Quest 10 New South Wales Australian curriculum edition Stages 5.1 and 5.2
number and algebra

Exercise 9B Sketching parabolas


INDIVIDUAL PATHWAYS
Questions: Questions: Questions:
1a, b, g, h, 2–5, 8, 9 1a, b, e, g, h, 2–6, 8–10 1–10
Activity 9-B-1 Activity 9-B-2 Activity 9-B-3
Review of sketching Sketching basic Trends in sketching
basic parabolas parabolas basic parabolas
doc-13742 doc-13743 doc-13744

FLUENCY
1 State whether each of the following graphs is wider or narrower than the graph of
y = x2 and state the coordinates of the turning point of each one.
1
a y = 5x2 b y = 3x2 c y = 7x2 d y = 10x2
2
e y = 5x2 f y = 0.25x2 g y = 1.3x2 h y = !3x2
2 State the vertical translation and the coordinates of the turning point for the graphs
of each of the following equations, when compared to the graph of y = x2.
1
a y = x2 + 3 b y = x2 − 1 c y = x2 − 7 d y = x2 + 4
1
e y = x2 − 2
f y = x2 − 0.14 g y = x2 + 2.37 h y = x2 + !3
3 In each of the following state whether the graph is wider or narrower than y = x2 and
whether it has a maximum or a minimum turning point.
1 1
a y = 3x2 b y = −3x2 c y = 2x2 d y = −5x2
4
e y = −3x2 f y = 0.25x2 g y = !3x2 h y = −0.16x2
4 Using a graphics calculator or other technology, sketch the following parabolas on
one set of axes.
y = x2, y = 2x2, y = 3x2, y = 13x2, y = 10
1 2
x
How does the value of a affect:
a the shape of the parabola y = ax2? b the vertex of each parabola?
5 Using a graphics calculator or other technology, sketch the following parabolas on
one set of axes.
y = x2, y = −x2, y = 2x2, y = −2x2, y = 14x2, y = −14x2
a What do the graphs have in common? b How do the graphs differ?
c What is the relationship between the graphs of y = ax2 and y = −ax2?
6 Using a graphics calculator or other technology, sketch the following parabolas on
one set of axes.
y = x2, y = x2 + 2, y = x2 + 4, y = x2 − 1, y = x2 − 3
Comment on:
a the shape of each parabola
b the vertex of each parabola.
7 WE3 Sketch each of the following sets of graphs on one set of axes.
a y = x2, y = x2 + 3, y = x2 − 1 b y = x2, y = 2x2, y = 4x2
1 1 1
c y = x2, y = −x2, y = −2x2 d y = x2, y = 2x2, y = −2x2, y = −10x2

Chapter 9 • Non-linear relationships 297


number and algebra

8 WE4 By considering transformations of the graph of y = x2, sketch the following


sets of graphs.
a y = x2, y = 2x2, y = −2x2, y = −2x2 + 1
b y = 3x2, y = −3x2, y = 3x2 − 3
1
c y = 2 − x2, y = 2 + x2, y = 2 + 2x2
d y = −x2 + 1, y = −2x2 + 1, y = 2x2 + 1

UNDERSTANDING
9 Match each graph with its correct label. y
a A y = x2
B y = x2 + 2
C y = 2x2 A
2 C
B

0 x

b A y = 3x2 y
A
B y = −3x2 B
C y = −x2
D y = 2x2 0 x

C D

c A y = x2 − 3 y
A B C
B y = 2x2 − 3
1
C y = 2x2 − 3

0 x

–3

d A y = −x2 − 2 y
B y = x2 + 2 A B

C y = −x2 + 2
D y = x2 − 2 0 x

C D

e A y = −x2 y
1
B y = −2x2 x
0
C y = −0.1x2 A
B
C

298 Maths Quest 10 New South Wales Australian curriculum edition Stages 5.1 and 5.2
number and algebra

REASONING
10 What is the name of each unlabelled graph? Explain your answers.
a y b y
y = x2 + 2 Digital doc
WorkSHEET 9.1
doc-5272

y = x2 + 2
y = x2

0 x
0 x

c y d y = x2
y
y = 2x2 y = x2

x
4 y = 1 x2
0 2
REFLECTION
What are the turning points
0 x
of the graphs
y = x 2 + k and
y = −x 2?

CHALLENGE 9.1

9C Exponential functions and graphs


•• Relationships of the form y = ax are called exponential functions with base a,
where a is a real number not equal to 1, and x is the index power or exponent.
•• The term ‘exponential’ is used, as x is an exponent (or index).

Chapter 9 • Non-linear relationships 299


number and algebra

•• For example, the graph of the exponential function y = 2x y


17
can be plotted by completing a table of values. 16
1
•• Remember that 2−3 = 3 15
2 14
1 13
= , and so on. 12
8
11
10
x −4 −3 −2 −1 0 1 2 3 4 9
8
1 1 1 1
y 1 2 4 8 16 7
16 8 4 2 6
5
4
•• The graph has many significant features. 3
–– The y-intercept is 1. 2
1
–– The value of y is always greater than zero.
–– As x decreases, y gets closer to but never –4 –3 –2 –1–10 x
1 2 3 4
reaches zero. So the graph gets closer to but
never reaches the x-axis. The x-axis (or the
line y = 0) is called an asymptote.
–– As x increases, y becomes very large.

Comparing exponential graphs 12


y
y = 3x
•• The diagram at right shows the graphs of y = 2x 11
y = 2x
and y = 3x. 10
•• The graphs both pass through the point (0, 1). 9
8
•• The graph of y = 3x climbs more steeply than
7
the graph of y = 2x. 6
•• y = 0 is an asymptote for both graphs. 5
4
3
2
1

–4 –3 –2 –1–10 x
1 2 3 4 5

y
11
Vertical translation y = 2x + 3 10
y = 2x
•• The diagram at right shows the graphs of y = 2x and 9
8
y = 2x + 3.
7
•• The graphs have identical shape. 6
•• Although they appear to get closer to each other, 3 units
5
the graphs are constantly 3 units apart. 4
•• As x becomes very small, the graph of y = 2x + 3 3
approaches but never reaches the line y = 3, 3 units 2
1
so y = 3 is the horizontal asymptote.
•• When the graph of y = 2x is translated –4 –3 –2 –1–10 1 2 3 4 5
x
3 units upward, it becomes the graph of y = 2x + 3.

300 Maths Quest 10 New South Wales Australian curriculum edition Stages 5.1 and 5.2
number and algebra

Reflection about the x-axis 8


y

•• The diagram at right shows the graphs of y = 2x y = 2x


7
and y = −2x. 6
•• The graphs have identical shape. 5
4
•• The graph of y = −2x is a reflection about the
3
x-axis of the graph of y = 2x. 2
•• The x-axis (y = 0) is an asymptote for both graphs. 1
•• In general, the graph of y = −ax is a reflection about x
–4 –3 –2 –1 0
the x-axis of the graph of y = ax. –1 1 2 3 4
–2
–3
–4 y = –2x
–5
–6
–7
–8

Reflection about the y-axis


•• The diagram at right shows the graphs of y = 2x y
9
and y = 2−x. 8
•• The graphs have identical shape. 7 y = 2x
•• The graph of y = 2−x is a reflection about the 6
y = 2 –x
y-axis of the graph of y = 2x. 5
•• Both graphs pass through the point (0, 1). 4
3
•• The x-axis (y = 0) is an asymptote for both graphs. 2
•• In general, the graph of y = a−x is a reflection about 1
the y-axis of the graph of y = ax.
–4 –3 –2 –1–10 1 2 3 4 x

WORKED EXAMPLE 5
Given the graph of y = 4x, sketch on the same axes the 6
y
graphs of: y = 4x
4
a y = 4x − 2
2
b y = −4x
c y = 4−x –4 –2 0 2 4x
–2

TH I N K DRAW
a The graph of y = 4x has already been drawn. It has a a 7
y
y-intercept of 1 and a horizontal asymptote at y = 0. 6
5
The graph of y = 4x − 2 has the same shape 4
3
as y = 4x but is translated 2 units vertically down. 2
y = 4 1 y = 4x – 2
x
It has a y-intercept of −1 and a horizontal
asymptote at y = −2. –3 –10 1 2 3 x
–2
–3 y = –2

Chapter 9 • Non-linear relationships 301


number and algebra

b y = −4x has the same shape as y = 4x but is b 6


y
reflected about the x-axis. It has a y-intercept of −1 5
and a horizontal asymptote at y = 0. 4
3
2
1 y = 4x

–3 –1 0 1 2 x
–2 y = – 4x
–3
–4
–5

c y = 4−x has the same shape as y = 4x but is c y


8
reflected about the y-axis. The graphs have the 7
same y-intercept and the same horizontal 6
5
asymptote (y = 0). 4
y = 4– x 3 y = 4x
2
1
0
–3 –1 1 2 3x

Combining transformations
•• It is possible to combine translations, dilations and reflections in one graph.
Interactivities
Exponential graphs WORKED EXAMPLE 6
int-1149
By considering transformations to the graph of y = 2x,
sketch the graph of y = −2x + 1.

TH I N K DRAW
Start by sketching y = 2x. 5
y
It has a y-intercept of 1 and a horizontal asymptote at y = 0. 4
Sketch y = −2x by reflecting y = 2x about the x-axis. 3
2 y = 2x
It has a y-intercept of −1 and a horizontal asymptote at y = 0. 1
Sketch y = −2x + 1 by translating y = −2x upwards by 1 unit.
–3 –10 1 2 3 x
The graph has a y-intercept of 0 and a horizontal asymptote –2 y = – 2x +1
at y = 1. –3
–4
y = – 2x

Exercise 9C Exponential functions and graphs


INDIVIDUAL PATHWAYS
Questions: Questions: Questions:
1–16 1–16 1–16
Activity 9-C-1 Activity 9-C-2 Activity 9-C-3
Exploring exponential graphs Features of exponential Tricky exponential graphs
doc-13745 graphs doc-13747
doc-13746

302 Maths Quest 10 New South Wales Australian curriculum edition Stages 5.1 and 5.2
number and algebra

FLUENCY
1 Complete the table below and use the table to plot the graph of y = 3x
for −3 ≤ x ≤ 3.
x −3 −2 −1 0 1 2 3
y
2 If x = 1, find the value of y when:
a y = 2x b y = 3x c y = 4x
d y = 10x e y = ax
3 Using a calculator or graphing program, sketch the graphs of y = 2x, y = 3x
and y = 4x on the same set of axes.
a What do the graphs have in common?
b How does the value of the base (2, 3, 4) affect the graph?
c Predict where the graph y = 8x would lie and sketch it in.
4 Using graphing technology, sketch the following graphs on one set of axes.
y = 3x, y = 3x + 2, y = 3x + 5, y = 3x − 3
a What remains the same in all of these graphs?
b What is changed?
c For the graph of y = 3x + 10, write down:
i the y-intercept
ii the equation of the horizontal asymptote.
5 a Using graphing technology, sketch the graphs of:
i y = 2x and y = −2x
ii y = 3x and y = −3x
iii y = 6x and y = −6x.
b What is the relationship between these pairs of graphs?
6 a Using graphing technology, sketch the graphs of:
i y = 2x and y = 2−x
ii y = 3x and y = 3−x
iii y = 6x and y = 6−x.
b What is the relationship between these pairs of graphs?
7 WE5 Given the graph of y = 2x, sketch on the same axes the graphs of:
a y = 2x − 6
y
b y = −2x 7
6
c y = 2−x 5
4
3
2 y = 2x
1
–3 –10 1 2 3 4 x
–2

Chapter 9 • Non-linear relationships 303


number and algebra

8 Given the graph of y = 3x, sketch on the same axes 7


y
the graphs of: 6
5
a y = 3x + 2 4
3 y = 3x
b y = −3x 2
1
–3 –10 1 2 3 4 x
–2

9 Given the graph of y = 4x, sketch on the same axes 7


y
y = 4x
the graphs of: 6
5
a y = 4x − 3 4
3
2
b y = 4−x 1

10 WE6 By considering transformations of the graph of y = 2x, –3 –10 1 2 3 4 x


–2
sketch the following graphs on the same set of axes.
a y = 2−x + 2
b y = −2x + 3
11 By considering transformations of the graph of y = 5x,
sketch the following graphs on the same set of axes.
a y = −5x + 10
b y = 5−x + 10

UNDERSTANDING
12 Match each graph with its correct label.
A y = 2x B y = 3x C y = −4x D y = 5−x
a y b y
10 2
9 1
8
7 –3 –10 1 2 3 4 x
6 –2
5 –3
4 –4
3 –5
2 –6
1 –7
–8
–3 –10 1 2 3 4 x –9
–2 –10

c y d y
10 10
9 9
8 8
7 7
6 6
5 5
4 4
3 3
2 2
1 1
–3 –10 1 2 3 4 x –3 –10 1 2 3 4 x
–2 –2

304 Maths Quest 10 New South Wales Australian curriculum edition Stages 5.1 and 5.2
number and algebra

13 Match each graph with its correct label. Explain your answer.
A y = 2x + 1 B y = 3x + 1 C y = −2x + 1 D y = 2−x + 1

a y b y
10 6
9 5
8 4
7 3
6 2
5 1
4
3 –3 –10 1 2 3 4 5 6 x
2 –2
1
–3 –10 1 2 3 4 x
–2

c y d y
9 2
8 1
7
6 –3 –10 1 2 3 4 x
5 –2
4 –3
3 –4
2 –5
1 –6
–7
–3 –10 1 2 3 4 x –8
–2 –9

REASONING
14 By considering transformations of the graph of y = 3x, sketch the graph of
y = −3−x − 3.
15 The graph of f(x) = 16x can be used to solve for x in the exponential
equation 16x = 32. Draw a graph of f(x) = 16x and use it to solve REFLECTION
16x = 32. Will the graph of an
exponential function
16 The graph of f(x) = 6x−1 can be used to solve for x in the exponential
always have a horizontal
equation 6x−1 = 36 !6. Draw a graph of f(x) = 6x−1 and use it to solve asymptote? Why?
6x−1 = 36 !6.

9D  The circle


•• Imagine a graph shaped like a circle with its centre at the origin.
•• If P (x, y) is any point on the circle, its distance from
(0, 0) is equal to r, the radius of the circle. y
•• Pythagoras’ theorem states that x2 + y2 = r2. P (x, y)
•• The graph with rule x2 + y2 = r2 is a circle of radius r
r
with its centre at the origin. y

0 x x

Chapter 9 • Non-linear relationships 305


number and algebra

WORKED EXAMPLE 7
Sketch the graph of x2 + y2 = 49, stating the centre and radius.

TH I N K W R I TE
1 From the equation we know that the graph is a x2 + y2 = 49
circle: x2 + y2 = r2.
2 The centre is at the origin. Centre (0, 0)
3 Find the value of the radius. r2 = 49
r = !49
r =7
Radius = 7 units
4 Sketch the graph by placing the centre at (0, 0) y
and showing a radius of 7. Label the graph. 7

–7 (0, 0) 7
x

–7 x2 + y2 = 49

WORKED EXAMPLE 8
Sketch the graph of 4x2 + 4y2 = 25, stating the centre and radius.

TH I N K W R I TE /DRAW
1 Express the equation in standard form by x2 + y2 = r2
dividing both sides by 4.
4x2 + 4y2 = 25
x2 + y2 = 25
4

2 State the coordinates of the centre. Centre (0, 0)


3 Find the length of the radius by taking the square r2 = 25
4
root of both sides. (Ignore the negative result.)
5
r =2
Radius = 2.5 units
4 Sketch the graph. y
2.5

–2.5 (0, 0) 2.5


x

–2.5

306 Maths Quest 10 New South Wales Australian curriculum edition Stages 5.1 and 5.2
number and algebra

Exercise 9D The circle


INDIVIDUAL PATHWAYS
Questions: Questions: Questions:
1–3 1–3 1–3
Activity 9-D-1 Activity 9-D-2 Activity 9-D-3
Circle graphs Exploring the circle More circle graphs
doc-13748 doc-13749 doc-13750

FLUENCY
1 WE 7, 8 Sketch the graphs of the following, stating the centre and radius of each.
a x2 + y2 = 49 b x2 + y2 = 42
c x2 + y2 = 36 d x2 + y2 = 81
2 a 2x2 + 2y2 = 50 b 9x2 + 9y2 = 100
c 5x2 + 5y2 = 3.2 d x2 = 16 − y2

UNDERSTANDING
3  MC For each of the following graphs, state whether the graph is:
A a straight line B a parabola
Digital doc
C an exponential graph D a circle. WorkSHEET 9.2
doc-13751
a y= 2x2 b y = 2x
REFLECTION
c y = 2x d x2 + y2 = 4 How could you write equations
representing a set of concentric
e +y=1
3x f 2x + y = 3
circles (circles with the same
g 2x2 = 16 − 2y2 h x2 + y = 3 centre, but different radii)?

CHALLENGE 9.2

Chapter 9 • Non-linear relationships 307


number and algebra

CHAPTER REVIEW
LANGUAGE

asymptote dilation factor origin symmetry


axes exponential parabola transformation
centre horizontal plot translation
circle horizontal asymptote radius turning point
concave down linear reflection vertex
concave up maximum sketch vertical
coordinates minimum substitute vertical asymptote
dilation non-linear symmetrical y-intercept

int-2850 doc-13753
int-2851 doc-13754
int-3596

FLUENCY
1 Construct a table of values and plot the graph of y = 2x2 − 5 for −3 ≤ x ≤ 3.
2 For the relation y = x2 − 5:
a find the values of y when x = –1 and when x = 4.
b find the values of x when y = –1 and when y = 4.
3 Which of these points lie on the parabola y = x2 + 2x? 10
y

(−1, 0), (1, 4), (0, 2), (2, 8), (−2, 0) 9


8
4 Given the graph of y = x2, sketch the following graphs and 7
6
explain the differences between them. 5
4
a y = x2 + 3 b y = 3x2 3
2 y = x2
1 1
c y = −x2 d y = −2x2
–3 –10 1 2 3 4 x
5 For the exponential function y = 5x: –2
–3
a complete the table of values below –4
–5
x y –6
–7
–8
−3 –9
–10
−2
−1
0
1
2
3
b plot the graph.

308 Maths Quest 10 New South Wales Australian curriculum edition Stages 5.1 and 5.2
number and algebra

6 Sketch the circle with equation shown. Clearly show the centre and the radius.
x2 + y2 = 16
y
7 Find the equation of this circle. 6

–6 0 6 x

–6

8 On the same set of axes, sketch the graphs of:


a y = 2x + 2 b y = 2−x
c y = −2x d y = −2x + 2
For each graph, give the equation of the asymptote.
9 On the same set of axes, sketch the graphs of:
a x2 + y2 = 16 b 4x2 + 4y2 = 16
10 For each of the following sets of graphs, match each equation with its correct label.
a y b y
10 10
9 9
8 A A 8
7 7
6 6
5 5
4 4
3 B 3
2 B 2 C
1 C 1
–3 –10 1 2 3 4 x –3 –10 1 2 3 4 x
–2 –2
–3 –3
–4 –4

x2 + y2 = 4, y = x2 + 4, y = 4x2 y = 2x, y = 2x + 1, y = x2 + 2
c y d y
10 B 10
A 9 C
9
8 8
7 7
6 6
5 5
4 4
3 3 A
2 2
1 1
–3 –10 1 2 3 4 x –3 –10 1 2 3 4 x
–2 –2 C
–3 –3
–4 –4 B

y = 3x, y = 3x, y = 3−x y = 4 − x, y = 4 − x2, y2 = 4 − x2

Problem Solving
1 The height, h, in metres of a golf ball t seconds after it is hit is given by the formula
h = 4t − t2.
a Plot the graph of the path of the ball for 0 ≤ x ≤ 4.
b What is the maximum height the golf ball reaches?
c How long does it take for the ball to reach the maximum height?
d How long is it before the ball lands on the ground after it has been hit?

Chapter 9 • Non-linear relationships 309


number and algebra
Communicating

Rich task

Parametric equations

310 Maths Quest 10 New South Wales Australian curriculum edition Stages 5.1 and 5.2
number and algebra

1 Complete the following table by calculating x- and y-values from the corresponding t-value.
t x y
0
1
2
3
4
5 Parametric equations
y
2 Graph the x-values and corresponding y-values on this Cartesian plane. 25
20
Join the points with a smooth curve and place an arrow on the curve 15
10
to indicate the direction of increasing t-values. 5
3 Is there any difference between this graph and that of y = x2? –5–4–3–2–10 1 2 3 4 5 x
Explain your answer.
4 Consider now the parametric equations x = 1 − t and y = (1 − t) 2. These clearly are also
equivalent to the equation y = x2. Complete the table and draw the graph of these two
equations for values of the parameter t ≥ 0. Draw an arrow on the curve in the direction of
increasing t-values.
t x y
Parametric equations
0 y
1 25
20
2 15
10
5
3
–5–4–3–2–10 1 2 3 4 5 x
4
5
Parametric equations
Describe the shape of your resulting graph. What values of the parameter x = t and y = –t 2

y
t would produce the same curve as that obtained in question 2?
0 x
The graph of y = −x2 is a reflection of y = x2 in the x-axis. Construct a –5–4–3–2–1
–5 1 2 3 4 5
–10
table and draw the graph of the parametric equations x = t and –15
y = −t2 for parameter values t ≥ 0. Remember to place an arrow –20
–25
on the curve in the direction of increasing t-values.
5 Without constructing a table, predict the shape of the graph of the parametric equations
x = 1 − t and y = −(1 − t) 2 for parameter values t ≥ 0. Draw a sketch of the shape.
6 This task requires you to produce the shape of the parabola y = x2 in the range −2 ≤ x ≤ 2
by considering two different parametric equations to those already considered. Answer this
question on a separate sheet of paper.
a State your two equations and the range of the parameter values.
b Construct a table showing calculated values.
c Draw a sketch of the graph.

Chapter 9 • Non-linear relationships 311


number and algebra

Code puzzle

Two important events


of 1973
Complete the table of values for the two exponential
functions and plot their graphs. The values of the functions
and the letters beside the x-values give the puzzle’s code.

x –3 A –2 D –1 E 0 G 1 H 2 I 3 Y 4 L
y = 2 x:
y

1
x –3 N –2 O –1 P 0 R 1 S 2 T 3 U 4 V
y = ( 2– ) x + 5:
y
y
20
18
16
14
12
10
8
6
4
2

–3 –2 –1 0 1 2 3 4 x
–2
–4

1 1 1 1 1 1 1 1

2 4 1 2 5 4– –
2

2 13 8 –
2

8 6 9 16 –
4 5 2–

1 1 1 1 1 1 1
1 —
4 516 – 13 5 4– 2 – — 9
5 16 5 4– –
2 2 2

1 1 1 1 1 1 1 1 1 1 1
52– 8 –
4 13 –
2 8 9 7 –
2 6 –
8 2 9 5 8– 5 2– –
2 9 7 –
2 13 –
2

4

312 Maths Quest 10 New South Wales Australian curriculum edition Stages 5.1 and 5.2
number and algebra

 ACTIVITIES
Go to assessON
Chapter opener Digital docs (page 302) for questions to
video • Activity 9-C-1 (doc-13745): Exploring test your readiness
• The story of mathematics (eles-1848) exponential graphs FOR learning, your
• Activity 9-C-2 (doc-13746): Features of progress AS you learn
9A Plotting parabolas
exponential graphs and your levels OF
Digital docs (page 292)
• Activity 9-C-3 (doc-13747): Tricky exponential achievement.
• Activity 9-A-1 (doc-13739): Review of plotting
graphs www.assesson.com.au
parabolas
• Activity 9-A-2 (doc-13740): Plotting parabolas 9D The circle
• Activity 9-A-3 (doc-13741): Trends in plotting Digital docs (page 307)
parabolas • Activity 9-D-1 (doc-13748): Circle graphs
• SkillSHEET (doc-5266): Substitution into • Activity 9-D-2 (doc-13749): Exploring the
quadratic equations circle
• SkillSHEET (doc-5267): Equation of a • Activity 9-D-3 (doc-13750): More circle
vertical line graphs
9B Sketching parabolas • WorkSHEET 9.2 (doc-13751): Non-linear
Interactivities relationships
• Dilation of y = x2 (int-1148) (page 294) Chapter review
• Vertical translation of y = x2 + c Interactivities (page 308)
(int-1192) (page 296) • Word search (int-2850)
Digital docs • Crossword (int-2851)
• Activity 9-B-1 (doc-13742): Review of • Sudoku (int-3596)
sketching basic parabolas (page 297) Digital docs
• Activity 9-B-2 (doc-13743): Sketching basic • Chapter summary (doc-13753)
parabolas (page 297) • Concept map (doc-13754)
• Activity 9-B-3 (doc-13744): Trends in
sketching basic parabolas (page 297) To access eBookPLUS activities, log on to
• WorkSHEET 9.1 (doc-5272): Quadratic www.jacplus.com.au
graphs (page 299)
9C Exponential functions and graphs
Interactivity
• Exponential graphs (int-1149) (page 302)

Chapter 9 • Non-linear relationships 313


number and algebra

Answers
Chapter 9 Non-linear relationships
Exercise 9A — Plotting parabolas 4    a y b y
y 1
1    a y
y = 3x2 b 1
y = –4 x2 0
30 x x
–3 –2 –1
0 1 2 3 4 –2 –1
–1 1 2 3 4
25 –1
20 2 –2 –2
15 –3 –3
10 1 –4 –4
5 –5 –5
x –6 –6
–3 –2 –1 0 1 2 3
x –3 –2 –1 0 1 2 3 –7
–7
–8 –8
x = 0, (0, 0) x = 0, (0, 0) y = –x2 + 1 –9
y = –(x – 1)2
x = 0, (0, 1), 1
x = 1, (1, 0), −1
2    a y b y
10 12
8 y = x2 + 1 10 y = x + 3
2 c y d y
6 8 0 0 1 2 3 4 x
x –2
4 6 –6 –4 –2 1 –4
2 4
–2 –6
x 2 (0, 3)
–3 –2 –1 0 1 2 3 –8
–3 –2 –1 0 1 2 3
x –4 –10
–12
x = 0, (0, 1), 1 y = –x2 –3
–6
x = 0, 1 0, 3 2 , 3 x = 0, (0, −3), −3
–8

c y d y y = –(x + 2)2
y = x2 –3 8 y = x2 –1
6 6
x = −2, (−2, 0), −4
4 4
2 2 5    a h
18
(0, –1) h = –(t – 4)2 + 16
x x 16
–3 –2 –1 0 1 2 3 –3 –2 –1 1 2 3
–2 –2 14
12
x = 0, (0, −3), −3 x = 0, (0, −1), −1 10
8
6
3    a y b y
4
2
y = (x + 2) 2
20 y = (x + 1)2 16
(–5, 16)
16 0 1 2 3 4 5 6 7 8 t
12
12
8 8
b        i 16 m ii 8 s
4 (1, 4) 4
0 1 2 x 6    a
–6 –5 –4 –3 –2 –1 x h
–6 –4 –2 0 2 18
16
x = −1, (−1, 0), 1 x = −2, (−2, 0), 4 14
12
10
c y d y
10 10 y = (x ‒ 1)2 8
y = (x ‒ 2)2 6
8 8
6 6 4
4 4 2
2 2
–1 0 1 2 3 4 t
x x
–1 0 1 2 3 4 5 –2 –1 0 1 2 3 4 5
x = 2, (2, 0), 4 x = 1, (1, 0), 1 b        i 18 m ii Yes, by 3 m
iii 1.5 s iv 3 s

314 Maths Quest 10 New South Wales Australian curriculum edition Stages 5.1 and 5.2
number and algebra

7    a y y

0 x 0 x

y
y

0 x
0 x

Exercise 9B — Sketching parabolas


y 1    a Narrower, TP (0, 0) b Wider, (0, 0)
c Narrower, TP (0, 0) d Narrower, TP (0, 0)
e Wider, TP (0, 0) f Wider, TP (0, 0)
g Narrower, TP (0, 0) h Narrower, TP (0, 0)
2    a Vertical 3 up, TP (0, 3)
b Vertical 1 down, TP (0, −1)
x
0 c Vertical 7 down, TP (0, −7)
d Vertical 14 up, TP (0, 14)
e Vertical 12 down, TP (0, −12)
f Vertical 0.14 down, TP (0, −0.14)
g Vertical 2.37 up, TP (0, 2.37)
y
h Vertical !3 up, TP (0, !3)
3    a Narrower, min b Narrower, max
c Wider, min d Wider, max
e Narrower, max f Wider, min
g Narrower, min h Wider, max
0 x 4 y
9
y = 3x2
8
7
6
b An infinite number of points of intersection occur when the
two equations represent the same parabola, with the effect that 5
the two parabolas superimpose. For example y = x2 + 4x + 3 4 y = —13 x2
and 2y = 2x2 + 8x + 6.
c It is possible to have 0, 1 or 2 points of intersection. 3
y 2
y = 2x2
1 y = x2
y = 10
—1 x2

x
–4 –3 –2 –1 0 1 2 3 4
–1
0 x
a As the value of a decreases, the graphs become flatter, or
closer to the x-axis.
b The graphs all have the same vertex, the origin.

Chapter 9 • Non-linear relationships 315


number and algebra

5 y b y
5 y = 2x2 y = 4x2 y = 2x2
9
y = x2
4 8 y = x2
3 7
2 6
y=—
1 x2
2
1 5
x 4
–4 –3 –2 –1 0 1 2 3 4
–1 3
y = –x2 –2
2
–3 1
1 x2 –4
y = –—
2 –4 –3 –2 –1 0 1 2 3 4 x
–5 –1
y = –2x2
a The graphs all have the same vertex (the origin). c y
b Where a is positive, the graphs are concave up, but where a is
negative, the graphs are concave down. The dilation of each 5
parabola depends on the size of a. 4 y = x2
c The graph of y = ax2 is a reflection of the graph ofy = −ax2.
6 y 3
8 2
y = x2 + 4
7 1
6 x
–4 –3 –2 –1 0 1 2 3 4
5 –1
4 –2
3 y = x2 + 2 –3
2 –4
y = x2 1 y = –x2 –5
y = –2x2
x
–4 –3 –2 –1 0 1 2 3 4 d y
–1 y = x2 – 1 4 y = x2
y = x2 – 3 –2 3
–3 2
1 y=—
1 x2
2
a The graphs have identical shape, each with a minimum
–4 –3 –2 –1 0 1 2 3 4 x
turning point. –1
b The graphs have different vertices. The vertex of each 1 2 –2
y = ‒— x
parabola lies on the y-axis. 10 –3
7    a y
–4 y = ‒ —
1 x2
9 y=x +3
2 2

8 8    a y
4 y = 2x2
7 3
6 2
y = x2
5 1

4 –3 –2 –1 0 1 2 3 x
–1
3 –2 y = –2x2
y = x2 –3 y = –2x2 + 1
2
–4
1 y = x2 ‒ 1

x
–4 –3 –2 –1 0 1 2 3 4
–1

316 Maths Quest 10 New South Wales Australian curriculum edition Stages 5.1 and 5.2
number and algebra

b y c y c y
16 y = 8x
3 5
14 y = 4x
2 y = 3x
2
4 y = 2 + x2 12 y = 3x
1 y = x2 3 10
2 8 y = 2x
–2 –1 0 1 2 x y = 2 + 12 x2 y = x2 6
–1 1 4
y = 3x2 – 3
–2 2
–3 y = –3x2 –2 –1 0 1 2 x
–1 y = 2 – x2
–5 –3 –1 0 1 2 3 4 5 6x
d –4
y
–6
3 –8
2 –10
1 –12
–14
x –16
–2 –1 0 1 2
–1
–2 4 y
11
9    a A: y = x2 + 2, B: y = 2x2, C: y = x2 10
9
b A: y = 3x2, B: y = 2x2, C: y = −3x2, D: y = −x2 8
c A: y = 2x2 − 3, B: y = x2 − 3, C: y = −12x2 − 3 7
d A: y = x2 − 2, B: y = x2 + 2, C: y = −x2 + 2, D: y = −x2 − 2 y = 3x + 5 6
e A: y = −0.1x2, B: y = −12x2, C: y = −x2 5
10    a y = −x2 + 2 b y = x2 + 1 4
c y = −2x 2 d y = −12x2 + 4 y = 3x + 2 3
2
Challenge 9.1 1
y = 3x
V = −2h2 + 40h
V is the vertical distance, –3 –1 0 1 2 3 4x
h is the horizontal distance. –2
–3
Exercise 9C — Exponential functions and graphs y = 3 – 3 –4
x
1 y
28 a The shape of each graph is the same.
26 b Each graph has a different y-intercept and a different
24 horizontal asymptote.
22
20 c         i (0, 11) ii y = 10
18 5    a i y ii y
16 8 9
14 7 8
12 6 7
10 5 6
8 4 5
6 3 4
4 y = 3x 2 3
2 y = 2x y = 3x
1 2
1
–3 –10 1 2 3 4 x –3 –10 1 2 3 4 x
–2 y = –2 x –3 –10 1 2 3 4 x
–3 –2 y = –3x
2    a 2 b 3 c 4 d 10 e a –4 –3
3 y y = 4x –5 –4
16 –6 –5
14 y = 3x –7 –6
12 –8 –7
10 –8
8 y = 2x –9
6
4 iii
2 y
7
–5 –3 –10 1 2 3 4 5 x 6
5
–4 4
–6 3
–8 2 y = 6x
–10 1
–12
–14 –3 –10 1 2 3 4 x
–16 –2 y = –6x
–3
a The graphs all pass through (0, 1). The graphs have the same –4
horizontal asymptote, y = 0. The graphs are all very steep. –5
b As the base is larger, so the graph is steeper when x > 0. b In each case the graphs are symmetric about the x-axis.

Chapter 9 • Non-linear relationships 317


number and algebra

6    a      i y ii y 9 a, b y
8
8 8 7
7 7 6 y = 4x
6 6 y = 4–x
5 5
5 4
4 4
3 3
y = 3−x 3 y = 3x
y=2 2
−x y = 2x 2 2
1 1 1
–3 –10 1 2 3 4 x –3 –10 1 2 3 4 x
–2 –3 –1 0 1 2 3 4x
–2 –2 y = 4x – 3
–3
iii y –4
8 –5
7 –6
−x 6 –7
y=6 5 y = 6x –8
4
3
2 10 y
1
8 y = 2x
–3 –10 1 2 3 4 x 7
–2 6
5
b In each case the graphs are symmetric about the y-axis. 4
3
7 a–c y = 2−x + 6 2 y = 2–x +2
y 1
8
7 –4 –3 –2 –1 0 1 2 3 4 x
6 –2 y = –2–x +3
5 –3
4 –4
3 –5
−x
y=2 2 y = 2x
1 –6
–7
–3 –10 1 2 3 4 x
–2 y =−2x
–3 y
–4 11
–5 16 y = 5x
–6 15
14
13
12
8 a, b y 11 y = 5–x + 10
8 10
7 9
6 8
5 y = 3x 7
4 6
y = 3x + 2 3 5
2 4
1 3
2
–3 –10 1 2 3 4x 1 y = –5x + 10
–2
y = −3x –4 –3 –2 –1 0 1 2 3 4 x
–3
–4 –2
–5
–6
12    a B b C
–7
–8 c D d A
–9 13    a B b D
c A d C

318 Maths Quest 10 New South Wales Australian curriculum edition Stages 5.1 and 5.2
number and algebra

14 y 3    a B b A c C d D
1 e C f A g D h B
–3 –10 1 2 3 4 5 6 7 x Challenge 9.2
–2 (−2, 0)
–3
–4 y = –3–x – 3 Chapter review
–5
–6 1
x −3 −2 −1 0 1 2 3
–7
–8 y 13 3 −3 −5 −3 3 13
–9
–10
–11 y
–12 16
15 y = x2 –5
15 y 14
y = 32 40 13
30
Point of
12
20
intersection
11
y = 16 x 10
(1.25, 32)
10
9
–3 –2 –1 0 1 2 3 x 8
–10
7
x = 1.25 6
5
4
16 Point of
3
y intersection
2
y = 36√6 100 (3.5, 88.181631)
1
80
60 –4 –3 –2 –1 0 1 2 3 4 x
40 y = 6x – 1 –2
20 –3
–4
–10 –5 0 5 10 x –5
–20
–6
x = 3.5
Exercise 9D — The circle 2    a −4, 11 b ±2, ±3
1    a y b y
3 (2, 8) and (−2, 0)
7 4 4 y
11
y = x2 10
–7 7 x –4 4 x
9
–7 –4 8
7
Centre (0, 0), radius 7 Centre (0, 0), radius 4 6
y = x2 + 3
c y d y 5
6 9 4
3
y = 3x2
–6 6 x 2
–9 9 x
1
–6 –9
–5 –4 –3 –2 –1–10 1 2 3 4 5 x
Centre (0, 0), radius 6 Centre (0, 0), radius 9 –2 y = – 1 x2
2    a y b y –3 2
5 1 –4
3–
3 –5
–5 5 x –6
1 –7
1
–3 – 3– x
–5 3 3 –8 y = –x2
1
–3 –
3
Centre (0, 0), radius 5 a Same as y = x2 but translated 3 units upwards
Centre (0, 0), radius 10
3 b Concave up, vertex at (0, 0), and steeper than y = x2 due to a
c y d 4
y dilation factor of 3
0.8
c Same as y = x2 but reflected about the x axis.
d Concave down, vertex at (0, 0), but wider than y = x2 due to a
–0.8 0.8x –4 4x dilation factor of 12
–0.8 –4

Centre (0, 0), radius = 0.8 Centre (0, 0), radius = 4

Chapter 9 • Non-linear relationships 319


number and algebra

5    a −3 −2 −1 0 1 2 3 Problem solving


x
1    a h (2, 4)
y 0.008 0.04 0.2 1 5 25 125
h = 4t – t2
b y
y = 5x
160
140 0 4 t
120
100
80
b 4m c 2s d 4s
60
40 Communicating — Rich task
(0, 1) 20 (1, 5) Parametric equations
1 2
t x y x = t and y = t2
–4 –3 –2 –1 0 1 2 3 4x
y
6 y 2 0 0 0 25
4 x + y = 16 20
2

1 1 1 15
10
2 2 4 5
–4 0 4 x
3 3 9 –5 –4 –3 –2 –1 0 1 2 3 4 5 x
–4
4 4 16
7 x2 + y2 = 36
8 y 5 5 25
10
y = 2–x x
9 y=2 +2
3 Answers will vary.
8
7 4 t x y
Parametric equations
6 x = 1 – t and y = (1 – t)2
5 y
0 1 1 25
4
20
3 1 0 0
15
2
1 2 −1 1 10
5
–6 –5 –4 –3 –2 –1 0 1 2 3 4 5 6 x 3 −2 4
–1 –5 –4 –3 –2 –1 0 1 2 3 4 5 x
–2 4 −3 9
–3
–4 5 −4 16
–5 y = –2x + 2
–6 Answers will vary. t = 1, 0, −1, −2, −3, −4
x
–7 y = –2
–8 5 Parametric equations
t x y x = t and y = – t 2
a y=2 b y=0 c y=0 d y=2 y
9 y 0 0 0
0
6
5 1 1 −1 –5 –4 –3 –2 –1–5 1 2 3 4 5x
x2 + y2 = 16 –10
4 2 2 −4
3 –15
2 3 3 −9 –20
1 –25
4 4 −16 –30
–6 –5 –4 –3 –2 –1 0 x
–1 1 2 3 4 5 6
–2 5 5 −25
–3
–4 6 Answers will vary.
4x2 +4y2 = 16
–5 Code puzzle
–6
Eighteen-year-olds given the vote.
Sydney Opera House opened
10 Match each graph with its correct label.
a A: y = x2 + 4, B: y = 4x2, C: x2 + y2 = 4
b A: y = x2 + 2, B: y = 2x + 1, C: y = 2x
c A: y = 3−x, B: y = 3x, C: y = 3x
d A: y = 4 − x, B: y = 4 − x2, C: y2 = 4 − x2

320 Maths Quest 10 New South Wales Australian curriculum edition Stages 5.1 and 5.2
measurement AND geometry

Chapter 10

Deductive
geometry
WHY LEARN THIS?
Learning about geometry includes being able to
reason deductively and to prove logically that certain
mathematical statements are true. It is important to be
able to prove theories meticulously and step by step in
order to show that the conclusions reached are soundly
based. Mathematicians spend most of their time trying
to prove new theories, and they rely heavily on all the
proofs that have gone before. Reasoning skills, and
hence the ability to prove theories, can be developed
and learned through practice and application.

WHAT DO YOU KNOW?


1 Think List what you know about geometry.
Use a thinking tool such as a concept
map to show your list.
2 pair Share what you know with a partner
and then with a small group.
3 share As a class, create a large concept map
that shows your class’s knowledge of geometry.

LEARNING SEQUENCE
10A Angles, triangles and congruence
10B Similar triangles
10C Quadrilaterals
10D Polygons
Watch this video
The story of mathematics

Searchlight ID: eles-1849


measurement AND geometry

10A  Angles, triangles and congruence


•• Euclid (c. 300 BC) was the mathematician who developed a systematic approach to
geometry, now referred to as Euclidean geometry, that relied on mathematical proofs.
•• A proof is a statement that is proven to be true.
•• A theorem is a statement that can be demonstrated to be true. To demonstrate that a
statement is proven, formal language needs to be used. It is conventional to use the
following structure when setting out a theorem.
–– Given: a summary of the information given
–– To prove: a statement that needs to be proven
–– Construction: a description of any additions to the diagram given
–– Proof: a sequence of steps that can be justified and form part of a formal
mathematical proof.

Angles
Angles at a point
•• The sum of the angles at a point is 360°.
a
a + b + c + d + e = 360° c b
d e

Supplementary angles
•• The sum of the angles on a straight line is 180°.
•• Angles a, b and c are supplementary angles.
c b a
a + b + c = 180°

Vertically opposite angles


•• Theorem 1: Vertically opposite angles are equal. D B

c
a b
O

A C

Given: Straight lines AB and CD intersect at O.


To prove: ∠AOD = ∠BOC and ∠BOD = ∠AOC
Construction: Label ∠AOD as a, ∠BOC as b and ∠BOD as c.
Proof: Let ∠AOD = a°, ∠BOC = b° and ∠BOD = c°.
a + c = 180° (supplementary angles)
b + c = 180° (supplementary angles)
∴a+c=b+c
∴a=b
So, ∠AOD = ∠BOC.
Similarly, ∠BOD = ∠AOC.

324 Maths Quest 10 New South Wales Australian curriculum edition Stages 5.1 and 5.2
measurement AND geometry

Parallel lines
•• If two lines are parallel and cut by a
d
transversal, then: c a
–– co-interior angles are equal.
For example, a = b. Transversal
–– corresponding angles are equal. b
For example, b = d.
–– alternate angles are equal. For example,
b = c.
–– opposite angles are equal, For example, c = d.

Angle properties of triangles


Theorem 2
•• Theorem 2: The sum of the interior angles of a triangle B
b
is 180°.

a c
A C

Given: ΔABC with interior angles a, b and c


To prove: a + b + c = 180°
Construction: Draw a line parallel to AC, passing through B and label it DE as
shown. Label ∠ABD as x and ∠CBE as y.

D B E
x b y

a c
A C

Proof: a = x (alternate angles)


c = y (alternate angles)
x + b + y = 180° (supplementary angles)
∴ a + b + c = 180°

Equilateral triangles
•• It follows from Theorem 2 that each interior angle of an equilateral triangle is 60°,
and, conversely, if the three angles of a triangle are equal, then the triangle is
equiangular.
a + a + a = 180°   (sum of interior angles in a triangle B
is 180°) a
3a = 180°
a = 60°
a a
A C

Chapter 10 • Deductive geometry 325


measurement AND geometry

•• Theorem 3: The exterior angle of a triangle is B


equal to the sum of the opposite interior angles. b

a c d
A C
Given: ΔABC with the exterior angle labelled d
To prove: d = a + b
Proof: c + d = 180° (supplementary angles)
a + b + c = 180° (sum of interior angles in a triangle is 180°)
∴d=a+b

Congruent triangles
•• Congruent triangles have the same size and the same shape; that is, they are identical
in all respects.
•• The symbol used for congruency is ≅.
•• For example, ΔABC in the diagram below is congruent to ΔPQR. This is written as
ΔABC ≅ ΔPQR.
C P Q

A B    R

•• Note that the vertices of the two triangles are written in corresponding order.
•• There are five tests to check whether triangles are congruent. The tests are
summarised in the table below.

Test Diagram Abbreviation


All three sides in one triangle are equal in SSS
length to the corresponding sides in the
other triangle.

Two corresponding sides and the included SAS


angle are the same in both triangles.

Two corresponding angles and a pair of ASA


corresponding sides are the same in both
triangles.

326 Maths Quest 10 New South Wales Australian curriculum edition Stages 5.1 and 5.2
measurement AND geometry

Test Diagram Abbreviation


A pair of corresponding angles and a non- AAS
included side are equal in both triangles.
• •

The hypotenuse and one pair of the other RHS


corresponding sides in two right-angled
triangles are the same.

WORKED EXAMPLE 1
Select a pair of congruent triangles from the diagrams below, giving a
reason for your choice.
A 18 cm N
Q L 35°
50° 15 cm
95°
C
95° 35° 95°
P R
B 15 cm M

TH I N K W R I TE
1 All three triangles have equal AC = PR = 15 cm, LN = 18 cm
angles, but the sides opposite the Since LN ≠ AC and LN ≠ PR, ΔLMN is
angle 95° are not equal. not congruent to ΔABC and ΔPQR.
2 To test whether ΔABC is ΔABC: ∠A = 50°, ∠B = 95°,
congruent to ΔPQR, ∠C = 180° − 50° − 95°
first find ∠C. = 35°

3 Apply a test for congruence to A pair of corresponding angles


Δ’s ABC and PQR. (∠B = ∠Q and ∠C = ∠R) and a
non-included side (AC = PR) are equal.
⇒ ΔABC ≅ ΔPQR (AAS)

Isosceles triangles
•• A triangle is isosceles if the lengths of two sides are equal.

Theorem 4 C

•• Theorem 4: The angles at the base of an isosceles


triangle are equal.

a b
A B

Chapter 10 • Deductive geometry 327


measurement AND geometry

Given: AC = CB
To prove: ∠BAC = ∠CBA
Construction: Draw a line from the vertex C to the midpoint of the base AB and
label the midpoint D. CD is the bisector of ∠ACB.
C

c c

a d b
A B
D

Proof: In ΔACD and ΔBCD,


CD = CD (common side)
AD = DB (construction, D is the midpoint of AB)
AC = CB (given)
⇒ ΔACD ≅ ΔBCD (SSS)
∴ ∠BAC = ∠CBA

•• Conversely, if two angles of a triangle are equal, then the sides opposite those angles
are equal.
•• It also follows that ∠ADC = ∠BDC = d
2d = 180°
and that ⇒ d = 90° (supplementary)

WORKED EXAMPLE 2
Given that ΔABD ≅ ΔCBD, find the values of the pronumerals in the figure
below.
B

40° z y
A x D C
3 cm

TH I N K W R I TE
1 In congruent triangles corresponding sides ΔABD ≅ ΔCBD
are equal in length. Side AD (marked x) AD = CD, AD = x, CD = 3
corresponds to side DC, so state the value of x. SO x = 3 cm.
2 Since the triangles are congruent, ∠BAD = ∠BCD
corresponding angles are equal. State the ∠BAD = 40°, ∠BCD = y
angles corresponding to y and z and hence So y = 40°.
find the values of these pronumerals. ∠BDA = ∠BDC
∠BDA = z, ∠BDC = 90°
So z = 90°.

328 Maths Quest 10 New South Wales Australian curriculum edition Stages 5.1 and 5.2
measurement AND geometry

WORKED EXAMPLE 3
Prove that ΔPQS is congruent to ΔRSQ.
P Q

S R

TH I N K W R I TE
1 Write the information given. Given: Rectangle PQRS with
diagonal QS.
2 Write what needs to be proved. Select Proof:
the appropriate congruency test for To prove: ΔPQS ≅ ΔRSQ
proof. (In this case it is RHS because the QP = SR (given)
triangles have an equal side, a right angle ∠SPQ = ∠SRQ = 90° (given)
and a common hypotenuse.) QS is common
so ΔPQS ≅ ΔRSQ 1 RHS 2 .

Exercise 10A Angles, triangles and congruence


INDIVIDUAL PATHWAYS
Questions: Questions: Questions:
1–5, 7, 9, 11 1–5, 6, 8–10, 12 1–12
Activity 10-A-1 Activity 10-A-2 Activity 10-A-3
Review of congruent shapes Practice with congruent Tricky congruent figures
doc-5095 figures doc-5097
doc-5096

FLUENCY
1 Determine the values of the unknown in each of the following.
a b A
120°
a 56°
30° b
B C

c A d B C e A
58° d e

44° B

62°
e
c
B C A C

Chapter 10 • Deductive geometry 329


measurement AND geometry

2 WE1 Select a pair of congruent triangles in each of the following, giving a reason
for your answer. All side lengths are in cm.
Digital docs
SkillSHEET
a        
65° 65°
Naming angles, lines
and figures
doc-5276 3 II
4 3 III
SkillSHEET 4
Angles and parallel I 4
70° 3
lines
doc-5280 65° 70° 45°
SkillSHEET
Corresponding sides
and angles of congruent
triangles
doc-5277 b 110°        
6 cm
I
40° 6 cm
III
II
110° 110° 40°
6 cm

40°

c     3    

5
4 II
3
4 III
I

d 3.5     2     3.5


2 I
III
4.8 II
3.5 4.8 2.5
4.8

UNDERSTANDING
3 Find the missing values of x and y in each of the following diagrams. Give reasons
for your answers.
a A b A

x
y
y O x
D
B C 6 D B
E 130°

330 Maths Quest 10 New South Wales Australian curriculum edition Stages 5.1 and 5.2
measurement AND geometry

c d A
C
32° y
A x
y

99° B

x 45°
B C

4 WE2 Find the value of the pronumeral in each of the following pairs of congruent
triangles. All side lengths are in cm.
a b c 80°
4 3

30°
85°

4 x x x y

d e
x
40°

y n m z
7 30° y
x

REASONING
5 WE3 Prove that each of the following pairs of triangles are congruent.

a P b P Q

S R

R S Q

Chapter 10 • Deductive geometry 331


measurement AND geometry

c P S Q d A B e Q

P R
D C
R

6  MC Note: There may be more than one correct answer.


Which of the following is congruent to the triangle
3 cm
shown at right? 5 cm
35°

A 3 cm B 5 cm

5 cm
3 cm

35°
35°

C D 3 cm

3 cm 35°
5 cm
35° 5 cm

7 Prove that ΔABC ≅ ΔADC and hence find the values of the pronumerals in each of
the following.
a A b B D c B C
30° y

30° 30°
w x 70° 65°
7 cm x
A C A D

B x y D

4 cm 40° 40° z
C

8 Explain why the triangles shown at


right are not necessarily congruent. 5 cm
40°
5 cm
7 cm
40°
7 cm

332 Maths Quest 10 New South Wales Australian curriculum edition Stages 5.1 and 5.2
measurement AND geometry

9 Explain why the triangles shown at


8 cm
right are not congruent.
8 cm
30° 30°
70° 70°

10 Show that ΔABO ≅ ΔACO if O is A B


the centre of the circle.

11 If DA = DB = DC, prove that ∠ABC C


is a right angle.
D

A B
12 If AC = CB and DC = CE in the diagram shown, prove that AB || DE.
D E

REFLECTION
C How can you be certain
that two figures are
congruent?
A B

10B Similar triangles


Similar figures
•• Two geometric shapes are similar figures when one is an enlargement or reduction
of the other shape.

Chapter 10 • Deductive geometry 333


measurement AND geometry

–– An enlargement increases the length of each side of a B'


figure in all directions by the same factor. For example,
in the diagram shown, triangle A′B′C′ is an enlargement
of triangle ABC by a factor of 3 from its A' C'
centre of enlargement at O. B

•• The symbol for similarity is ~ and is read as ‘is similar to’. A C


•• The image of the original object is the enlarged or
reduced shape. O
•• To create a similar shape, use a scale factor to enlarge or reduce the original shape.
•• The scale factor can be found using the formula below and the lengths of a pair of
corresponding sides.
image side length
Scale factor =
object side length
•• If the scale factor is less than 1, the image is a reduced version of the original shape. If
the scale factor is greater than 1, the image is an enlarged version of the original shape.

Similar triangles
•• Two triangles are similar if:
–– the angles are equal, or
–– the corresponding sides are proportional.
•• Consider the pair of similar triangles below.
U
A
10
5 6
3

B C V 8 W
4

•• The following statements are true for these triangles.


–– Triangle UVW is similar to triangle ABC that is, ΔUVW ∼ ΔABC.
–– The corresponding angles of the two triangles are equal
∠CAB = ∠WUV, ∠ABC = ∠UVW and ∠ACB = ∠UWV.
–– The corresponding sides of the two triangles are in the same ratio.
UV VW UW
= = = 2;
AB BC AC
–– The scale factor is 2.

Testing triangles for similarity


•• Triangles can be tested for similarity using one of the tests described in the table
below.
Test Diagram Abbreviation
Two angles of a triangle are equal to two AAA
angles of another triangle. This implies

that the third angles are equal, as the sum


of angles in a triangle is 180°.

334 Maths Quest 10 New South Wales Australian curriculum edition Stages 5.1 and 5.2
measurement AND geometry

Test Diagram Abbreviation


The three sides of a triangle are SSS
proportional to the three sides of another a
triangle. b ka kc
c

kb

Two sides of a triangle are proportional SAS


to two sides of another triangle, and the a
included angles are equal. ka kc
c

The hypotenuse and a second side of a RHS


right-angled triangle are proportional
to the hypotenuse and a second side of c kc

another right-angled triangle.


a
ka

WORKED EXAMPLE 4
Find a pair of similar triangles among those shown. Give a reason for your
answer.
a b c
3 cm 6 cm
3 cm 140° 5 cm
140° 140°
2 cm 4 cm

TH I N K W R I TE
1 In each triangle the lengths of two sides and For triangles a and b:
6
the included angle are known, so the SAS
3
= 42 = 2
test can be applied. Since all included angles For triangles a and c:
are equal (140°), we need to find the ratios 5
= 1.6, 32 = 1.5
of corresponding sides, taking two triangles 3
at a time. For triangles b and c:
5
6
= 0.83, 34 = 0.75
2 Only triangles a and b have corresponding Triangle a ~ triangle b
sides in the same ratio (and included angles (SAS)
are equal). State your conclusion, specifying
the similarity test that has been used.

Chapter 10 • Deductive geometry 335


measurement AND geometry

WORKED EXAMPLE 5
Prove that ΔABC is similar to ΔEDC.
A D
C

B
E

TH I N K W R I TE
1 Write the information given. 1 Given:
ΔABC and ΔCDE
AB || DE
C is common.

2 Write what is to be proved. 2 To Prove: ΔABC ∼ ΔEDC


3 Write the proof. 3 Proof:
∠ABC = ∠EDC (alternate angles)
∠BAC = ∠DEC (alternate angles)
∠BCA = ∠DCE (vertically opposite angles)
∴ ΔABC ∼ ΔEDC (equiangular, AAA)

Exercise 10B Similar triangles


INDIVIDUAL PATHWAYS
Questions: Questions: Questions:
1–10 1–11 1–12
Activity 10-B-1 Activity 10-B-2 Activity 10-B-3
Review of similar shapes Similarity practice Tricky similarity problems
doc-5098 doc-5099 doc-5100

FLUENCY
1 WE4 Find a pair of similar triangles among those shown in each part. Give a reason
for your answer.
a i ii iii

5 5 10

3 4 6

b i ii iii
4 2 8

20° 5 20° 2.5 20° 12

336 Maths Quest 10 New South Wales Australian curriculum edition Stages 5.1 and 5.2
measurement AND geometry

c i 2 ii 2 iii

4 5 6
3 4 4.5

d i ii iii

40° 60° 50° 60° 40 ° 60°

e i ii iii
4 8 7
3 6 5

2 4 4

UNDERSTANDING
2 Name two similar triangles in each of the following figures.
a Q b A c P Q

B D C R
A C
P R

S T
d A B e B
D

D E
A C E

AB BC
3 a Complete this statement: = = . 4
D
AD AE B
2
b Find the value of the pronumerals. A 3
4 f
C
g
E

4 Find the value of the pronumeral in the Q


diagram at right.
A
x
2
P
4 B 4 R

Chapter 10 • Deductive geometry 337


measurement AND geometry

5 The triangles shown at right are similar.


Find the value of x and y. 45°
4
45° 1
20°
9 x
y

6 a State why these two triangles are similar. S


P
b Find the values of x and y in the diagram at right. 1.5 8
3 R
y 6
Q x

T
7 Calculate the height of the top of the 8 A flag stick casts a shadow 2 m long. If
ladder using similar triangles. a 50 cm ruler is placed in the same
upright position and it casts a shadow
20 cm long, what is the height of the
flag stick?

1.6 m

0.7 m

0.9 m 2m

50 cm

20 cm
2m

338 Maths Quest 10 New South Wales Australian curriculum edition Stages 5.1 and 5.2
measurement AND geometry

REASONING
9 WE5 Prove that ΔABC is similar to ΔEDC in each of the following.
Digital docs
a C b D c E d D SkillSHEET
A A
Writing similarity
statements
E D C B doc-5278
C
B SkillSHEET
B D
A B Calculating unknown
E side lengths in a pair of
similar triangles
A C E
doc-5281

10 ΔABC is a right-angled triangle. A line is C


drawn from A to D as shown so that AD ⟂ BC.
Prove that: D
a ΔABD ~ ΔACB
b ΔACD ~ ΔACB
A B

11 Explain why the AAA test cannot be used to prove congruence but can be used to
prove similarity.
A
12 a Prove Pythagoras’ theorem, AC2 = AB2 + BC2,
using similar triangles.
D

B C
b Prove the converse of Pythagoras’ theorem; that is,
if the square on one side of a triangle equals the REFLECTION
How can you be certain
sum of the squares on the other two sides, then the that two figures are
Digital doc
WorkSHEET 10.1
angle between these other two sides is a right angle. similar? doc-5282

CHALLENGE 10.1

Chapter 10 • Deductive geometry 339


measurement AND geometry

10C Quadrilaterals
Interactivity •• Quadrilaterals are four-sided plane shapes whose interior angles sum to 360°.
Quadrilateral definitions
int-2786
Theorem 5
•• Theorem 5: The sum of the interior angles in a A B
quadrilateral is 360°.

Given: A quadrilateral ABCD


To prove: ∠ABC + ∠BCD + ∠ADC + ∠BAD = 360°
Construction: Draw a line joining vertex A to vertex C. Label the interior angles of
the triangles formed.
A B
a b
f

e
D
d c
C

Proof: a + b + c = 180° (sum of interior angles in a triangle is 180°)


   d + e + f = 180° (sum of interior angles in a triangle is 180°)
⇒ a + b + c + d + e + f = 360°
∴ ∠ABC + ∠BCD + ∠ADC + ∠BAD = 360°

Parallelograms
•• A parallelogram is a quadrilateral with two pairs of parallel sides.

Theorem 6
•• Theorem 6: Opposite angles of a parallelogram A B
are equal.

D C

Given: AB 7 DC and AD 7 BC
To prove: ∠ABC = ∠ADC
Construction: Draw a diagonal from B to D.
A B

D C

340 Maths Quest 10 New South Wales Australian curriculum edition Stages 5.1 and 5.2
measurement AND geometry

Proof: ∠ABD = ∠BDC (alternate angles)


∠ADB = ∠CBD (alternate angles)
∠ABC = ∠ABD + ∠CBD (by construction)
∠ADC = ∠BDC + ∠ADB (by construction)
∴ ∠ABC = ∠ADC

•• Conversely, if each pair of opposite angles of a quadrilateral is equal then it is a


parallelogram.

Theorem 7
•• Theorem 7: Opposite sides of a parallelogram are equal. A B

D C

Given: AB || DC and AD || BC
To prove: AB = DC
Construction: Draw a diagonal from B to D.

A B

D C

Proof: ∠ABD = ∠BDC (alternate angles)


∠ADB = ∠CBD (alternate angles)
BD is common to ΔABD and ΔBCD.
⇒ ΔABD ≅ ΔBCD (ASA)
∴ AB = DC

•• Conversely, if each pair of opposite sides of a quadrilateral is equal then it is a


parallelogram.

Theorem 8
•• Theorem 8: The diagonals of a parallelogram bisect A B
each other.
O

D C

Given: AB || DC and AD || BC with diagonals AC and BD


To prove: AO = OC and BO = OD

Chapter 10 • Deductive geometry 341


measurement AND geometry

Proof: In ΔAOB and ΔCOD,


∠OAB = ∠OCD (alternate angles)
∠OBA = ∠ODC (alternate angles)
AB = CD (opposite sides of a parallelogram)
⇒ ΔAOB ≅ ΔCOD (ASA)
⇒ AO = OC (corresponding sides in congruent triangles)
and BO = OD (corresponding sides in congruent triangles)

Rectangles
•• A rectangle is a parallelogram with four right-angles.

Theorem 9
•• Theorem 9: A parallelogram with a right-angle A B
is a rectangle.

D C

Given: Parallelogram ABCD with ∠BAD = 90°


To prove: ∠BAD = ∠ABC = ∠BCD = ∠ADC = 90°
Proof: AB || CD (properties of a parallelogram)
⇒ ∠BAD + ∠ADC = 180° (co-interior angles)
But ∠BAD = 90° (given)
⇒ ∠ADC = 90°
Similarly, ∠BCD = ∠ADC = 90°
∴ ∠BAD = ∠ABC = ∠BCD = ∠ADC = 90°

Theorem 10
•• Theorem 10: The diagonals of a rectangle are equal. A B

D C

Given: Rectangle ABCD with diagonals AC and BD


To prove: AC = BD
Proof: In ΔADC and ΔBCD,
AD = BC (opposite sides equal in a rectangle)
DC = CD (common)
∠ADC = ∠BCD = 90° (right angles in a rectangle)
⇒ ΔADC ≅ ΔBCD (SAS)
∴ AC = BD

342 Maths Quest 10 New South Wales Australian curriculum edition Stages 5.1 and 5.2
measurement AND geometry

Rhombuses
•• A rhombus is a parallelogram with four equal sides.

Theorem 11
•• Theorem 11: The diagonals of a rhombus are
A B
perpendicular.

D C

Given: Rhombus ABCD with diagonals AC and BD


To prove: AC ⟂ BD
Proof: In ΔAOB and ΔBOC,
AO = OC (property of parallelogram)
AB = BC (property of rhombus)
BO = OB (common)
⇒ ΔAOB ≅ ΔBOC (SSS)
⇒ ∠AOB ≅ ∠BOC
But ∠AOB + ∠BOC = 180° (supplementary angles)
⇒ ∠AOB = ∠BOC = 90°
Similarly, ⇒ ∠AOD = ∠DOC = 90°.
Hence, AC ⟂ BD

The mid-point theorem


•• Now that the properties of quadrilaterals have been explored, the mid-point theorem
can be tackled.

Theorem 12
•• Theorem 12: The interval joining the midpoints of A
two sides of a triangle is parallel to the third side and
half its length. E
D
C

Given: ΔABC in which AD = DB and AE = EC


To prove: DE || BC and DE = 12BC

Chapter 10 • Deductive geometry 343


measurement AND geometry

Construction: Draw a line through C parallel to AB. Extend DE to F on the


parallel line.
A F
E
D
C

B
Proof: In ΔADE and ΔCEF,
AE = EC (E is the midpoint of AC, given)
∠AED = ∠CEF (vertically opposite angles)
∠EAD = ∠ECF (alternate angles)
⇒ ΔADE ≅ ΔCEF (ASA)
∴ AD = CF and DE = EF  (corresponding sides in congruent
triangles)
So, AD = DB = CF.
We have AB || CF (by construction)
So BDFC is a parallelogram.
⇒ DE || BC
Also, BC = DF (opposite sides in parallelogram)
But DE = DF (sides in congruent triangles)
⇒ DE = BC1
2
Therefore, DE || BC and DE = 12BC.

•• Conversely, if a line interval is drawn parallel to a side of a triangle and half the length
of that side, then the line interval bisects each of the other two sides of the triangle.
•• A summary of the definitions and properties of quadrilaterals is shown in the table
below.
Shape Definition Properties
Trapezium A trapezium is a One pair of opposite sides is parallel
quadrilateral with one but not equal.
pair of opposite sides
parallel.

Parallelogram A parallelogram is a • Opposite angles are equal.


quadrilateral with both • Opposite sides are equal.
pairs of opposite sides • Diagonals bisect each other.
parallel.

Rhombus A rhombus is a • Diagonals bisect each other at right


parallelogram with four angles.
equal sides. • Diagonals bisect the angles at the
vertex through which they pass.

344 Maths Quest 10 New South Wales Australian curriculum edition Stages 5.1 and 5.2
measurement AND geometry

Shape Definition Properties


Rectangle A rectangle is a •• Diagonals are equal.
parallelogram whose • Diagonals bisect.
interior angles are right
angles.

Square A square is a • All angles are right angles.


parallelogram whose • All side lengths are equal.
interior angles are right • Diagonals are equal in length and
angles with four equal bisect each other at right angles.
sides. • Diagonals bisect the vertex through
which they pass (45°).

Relationships between quadrilaterals


•• The flowchart below shows the relationships between quadrilaterals.

Quadrilateral

Trapezium
Kite

Parallelogram

Rhombus Rectangle

Square

Exercise 10C Quadrilaterals


INDIVIDUAL PATHWAYS
Questions: Questions: Questions:
1–10 1–14 1–16
Activity 10-C-1 Activity 10-C-2 Activity 10-C-3
Quadrilaterals Harder quadrilaterals Tricky quadrilaterals
doc-13755 doc-13756 doc-13757

Chapter 10 • Deductive geometry 345


measurement AND geometry

FLUENCY
1 Use the definitions of quadrilaterals to decide if the following statements are true or
Digital doc
SkillSHEET false.
Identifying
quadrilaterals
a A square is a rectangle. b A rhombus is a parallelogram.
doc-5279 c A square is a rhombus. d A rhombus is a square.
e A square is a trapezium. f A parallelogram is a rectangle.
g A trapezium is a rhombus. h A rectangle is a square.
2 Determine the values of x and y in each of the following figures.
a b
(3x + 10)° y° x
3 cm
4 cm
(2x − 10)° y°

c d
9x° 11x°

3x°

2x°

UNDERSTANDING
3 Draw three different trapeziums. Using your ruler, compass and protractor, decide
which of the following properties are true in a trapezium.
a Opposite sides are equal. b All sides are equal.
c Opposite angles are equal. d All angles are equal.
e Diagonals are equal in length. f Diagonals bisect each other.
g Diagonals are perpendicular. h Diagonals bisect the angles they pass
through.
4 Draw three different parallelograms. Using your ruler, compass and protractor to
measure, decide which of the following properties are true in a parallelogram.
a Opposite sides are equal. b All sides are equal.
c Opposite angles are equal. d All angles are equal.
e Diagonals are equal in length. f Diagonals bisect each other.
g Diagonals are perpendicular. h Diagonals bisect the angles they pass
through.
5 Name four quadrilaterals that have at least one pair of opposite sides that are parallel
and equal.
6 Name a quadrilateral that has equal diagonals that bisect each other and bisect the
angles they pass through.

346 Maths Quest 10 New South Wales Australian curriculum edition Stages 5.1 and 5.2
measurement AND geometry

7 Pool is played on a rectangular table. Balls are hit with a cue and bounce
off the sides of the table until they land in one of the holes or pockets.
a Draw a rectangular pool table measuring 5 cm by 3 cm on graph paper.
Mark the four holes, one in each corner.

b A ball starts at A. It is hit so that it travels at a 45° diagonal


across the grid. When it hits the side of the table, it bounces off
at a 45° diagonal as well. How many sides does the ball bounce off
before it goes in a hole?
c A different size table is 7 cm by 2 cm. How many sides does
a ball bounce off before it goes in a hole when hit from A?
d Complete the following table. A

Table size Number of sides hit


5 cm × 3 cm
7 cm × 2 cm
4 cm × 3 cm
4 cm × 2 cm
6 cm × 3 cm
9 cm × 3 cm
12 cm × 4 cm

e Can you see a pattern? How many sides would a ball bounce
off before going in a hole when hit from A on an m × n table?
f The ball is now hit from B on a 5 cm × 3 cm pool table.
How many different paths can a ball take when hit along 45° diagonals?
Do these paths all hit the same number of sides before going
in a hole? Does the ball end up in the same hole each time?
Justify your answer. B
g The ball is now hit from C along the path shown.
What type of triangles and quadrilaterals are formed by
the path of the ball with itself and the sides of the table?
Are any of the triangles congruent?
h A ball is hit from C on a 6 cm by 3 cm table. What shapes are
formed by the path of the ball with itself and the sides of the table?
Is there only one path possible? C
i Challenge: A ball is hit from A along 45° diagonals.
The table is m × n. Can you find a formula to predict which hole the ball
will go in?
j Challenge: What would happen if the game
was played on a trapezoidal table?

Chapter 10 • Deductive geometry 347


measurement AND geometry

8 Prove that the diagonals of a rhombus bisect each other.


9 ABCD is a parallelogram. X is the midpoint of AB and Y A X B
is the midpoint of DC. Prove that AXYD is also
a parallelogram.

D Y C

10 ABCD is a parallelogram. P, Q, R and S are all midpoints A P B


of their respective sides of ABCD.
a Prove ΔPAS ≅ ΔRCQ. S Q

b Prove ΔSDR ≅ ΔPBQ.


D R C
c Hence, prove that PQRS is also a parallelogram.

11 AC and BD are diameters of a circle with centre O. A


Prove that ABCD is a rectangle.
D O
B

C
12 The diagonals of a parallelogram meet at right angles.
Prove that the parallelogram is a rhombus.

13 Two congruent right-angled triangles are arranged as P Q


shown. Show that PQRS is a parallelogram.

S R

14 Two circles, centred at M and N, have equal radii and P


intersect at P and Q. Prove that PNQM is a rhombus.
M N

Q
15 Give reasons why a square is a rhombus, but a
rhombus is not necessarily a square. REFLECTION
Digital doc
How do you know if a
WorkSHEET 10.2 16 Show that any result proven for a parallelogram quadrilateral is a rhombus?
doc-5283
would also hold for a rectangle.

CHALLENGE 10.2

348 Maths Quest 10 New South Wales Australian curriculum edition Stages 5.1 and 5.2
measurement AND geometry

10D Polygons
•• Polygons are closed shapes that have three or more straight sides.

Irregular Not a Not a


    polygon polygon polygon

•• Regular polygons are polygons with sides of the same length and interior angles of
the same size, like the pentagon shown in the centre of the photo above.
•• Convex polygons are polygons with no interior reflex angles.
•• Concave polygons are polygons with at least one reflex interior angle. For example,
the pentagon shown above is a concave polygon as well as a regular polygon.

Interior angles of a polygon


•• The interior angles of a polygon are the angles inside the polygon at each vertex.
•• The sum of the interior angles of a polygon is given by the formula:
Angle sum = 180° × (n–2)
where n = the number of sides of the polygon.

WORKED EXAMPLE 6
Calculate the value of the pronumerals in the figure below.

80°
110° a b
150°

TH I N K W R I TE
1 Angles a and 110° form a straight a + 110° = 180°
line and so are supplementary a + 110° − 110° = 180° − 110°
(add to 180°). a = 70°
2 The interior angles of a triangle b + a + 80° = 180°
sum to 180°.
3 Substitute 70° for a and solve b + 70° + 80° = 180°
for b. b + 150° = 180°
b = 30°

Chapter 10 • Deductive geometry 349


measurement AND geometry

Exterior angles of a polygon


•• The exterior angles of a polygon are formed by the side of the polygon and an
extension of its adjacent side. For example, x, y and z are external angles for the
polygon (triangle) below.
r

y q
b s
x a c
z t

•• The exterior angle and interior angle at that vertex are supplementary (add to 180°).
For example, x + a = 180°.
•• Exterior angles of polygons can be measured in a clockwise or anticlockwise
direction.
•• In a regular polygon, the size of the exterior angle can be found by dividing 360° by
the number of sides.
360
Exterior angle =
n
•• The sum of the exterior angles of a polygon equals 360°.
•• The exterior angle of a triangle is equal to the sum of the opposite interior angles.

Exercise 10D Polygons


INDIVIDUAL PATHWAYS
Questions: Questions: Questions:
1–7, 9 1–9 1–10
Activity 10-D-1 Activity 10-D-2 Activity 10-D-3
Classifying polygons Calculating angles Angle properties of
doc-13758 in polygons polygons
doc-13759 doc-13760

FLUENCY
1 How are the internal and external angles of a polygon related to the number of sides
Interactivities
Angle sum of a polygon in a polygon?
int-0818
Exterior angles of a
2 WE6 Calculate the values of the pronumerals in the diagrams below.
polygon
int-0819 a b
b
m
120° a

c d
(t – 10)° 160°

15° 10°

70° 5x

350 Maths Quest 10 New South Wales Australian curriculum edition Stages 5.1 and 5.2
measurement AND geometry

3 For the five triangles below, evaluate the pronumerals and determine the size of the
interior angles.
a y
b 15° 160° c n n

55° l 18°

d e 20°
4x
t
(3t + 10)°
105° x
92°
(2t – 2)°

4 For the five quadrilaterals below:


i label the quadrilaterals as regular or irregular
ii determine the value of the pronumeral for each shape.
a x 120° b c
70° t

80° 65° p 4t

20°

d y e
3m
2y 2m

60°

UNDERSTANDING y

5 The photograph at right shows a house built on the side of a hill.


Use your knowledge of angles to calculate the values of
the pronumerals.

133º

105º
x w

Chapter 10 • Deductive geometry 351


measurement AND geometry

6 Calculate the values of the four interior angles of the front face of the building in
the photograph below.

x + 15

7 Calculate the values of the pronumerals for the irregular polygons below.
a f b
b n
c m
120° o
a d e
350°

8 Calculate the size of the exterior angle of a regular hexagon (6 sides).

REASONING
9 A diagonal of a polygon joins two vertices.
a Calculate the number of diagonals in a regular polygon with:
i 4 sides ii 5 sides iii 6 sides iv 7 sides.
b Write a formula that relates the number of diagonals for an n-sided
polygon.
10 The external angle of a polygon can be calculated using the formula:
360
exterior angle =
Digital doc n REFLECTION
WorkSHEET 10.3
doc-5284 Use the relationship between internal and external How do the angles
angles of a polygon to write a formula for the internal associated with a polygon
angle of a regular polygon. relate to each other and
the polygon?

352 Maths Quest 10 New South Wales Australian curriculum edition Stages 5.1 and 5.2
measurement AND geometry

CHAPTER REVIEW
LANGUAGE

concave corresponding sides parallelogram rhombus


congruency diagonal quadrilateral similarity
convex exterior angle rectangle square
corresponding interior angle reflex trapezium
angles

int-2853 doc-13761
int-2854 doc-13762
int-3597

FLUENCY
1 Select a pair of congruent triangles in each of the following sets of triangles, giving a
reason for your answer. All angles are in degrees and side lengths in cm. (The figures
are not drawn to scale.)
a     4     4
75° 75°
40° III
II 6 65°
4 6 6
I

75°

b
I
6 8
10 6 6
II III

2 Find the value of the pronumeral in each pair of congruent triangles. All angles are
given in degrees and side lengths in cm.
a b c y z 60°
2

70° 30°
8
x

x
2

Chapter 10 • Deductive geometry 353


measurement AND geometry

Prove that the two triangles shown in the diagram at right


3 a  A B C
are congruent.

••
D
b Prove that ΔPQR is congruent to ΔQPS. S R

P Q
4 Test whether the following pairs of triangles are similar. For similar triangles find the
scale factor. All angles are in degrees and side lengths in cm.
a b c
47°
2
110°
5 3 2

50° 4
1

47°
3
110°
5
7.5

50°
2
5 Find the value of the pronumeral in each pair of similar triangles. All angles are
given in degrees and side lengths in cm.
a A 5 B b A
48°
1 50°
y
2 C z E
B 1.5 44° x
D x E

3 8

y
C
D
c      
P A
x

9 y 5
3

30°
Q z R C
4 B

354 Maths Quest 10 New South Wales Australian curriculum edition Stages 5.1 and 5.2
measurement AND geometry

6 Prove that ΔABC~ΔEDC. A D

C
B
E

7 Prove that ΔPST~ΔPRQ. Q


S

P R
T

8 Prove that the angles opposite the equal sides in an isosceles triangle
are equal.
9 Two corresponding sides in a pair of similar octagons have lengths of 4 cm and
60 mm. The respective scale factor in length is:
A 4 : 60
B 6 : 40
C 40 : 60
D 60 : 40
x + 1
10 A regular nonagon has side length x cm. Use a scale factor of x
to find the side
length of a similar nonagon.

PROBLEM SOLVING
1 ABC is a triangle. D is the midpoint of AB, E is the midpoint A

of AC and F is the midpoint of BC. DG ⟂ AB, EG ⟂ AC and


FG ⟂ BC. D E
a Prove that ΔGDA ≅ ΔGDB.
G
b Prove that ΔGAE ≅ ΔGCE.
B C
c Prove that ΔGBF ≅ ΔGCF. F

d What does this mean about AG, BG and CG?


e A circle centred at G is drawn through A. What other points
must it pass through?
2 PR is the perpendicular bisector of QS. Prove that ΔPQS is P
isosceles.

3 Name any quadrilaterals that have diagonals that bisect the


angles they pass through.
4 State three tests that can be used to show that a quadrilateral is Q R
S
a rhombus.

Chapter 10 • Deductive geometry 355


measurement AND geometry

5 Prove that WXYZ is a parallelogram.

W X
130° 50°

Z Y

6 Prove that the diagonals in a rhombus bisect the angles they pass through.
7 Explain why the triangles shown below are not congruent.

5 cm
80° 25°
25° 80° 5 cm

8 Prove that ΔEFO ~ ΔGHO.


E F

H G

9 State the definition of a rhombus.


10 Name any quadrilaterals that have equal diagonals.

356 Maths Quest 10 New South Wales Australian curriculum edition Stages 5.1 and 5.2
measurement AND geometry
Communicating

Rich task

Enlargement activity

The geometrical properties shared by a shape and its image under enlargement can be listed as:
• lines are enlarged as lines
• sides are enlarged to corresponding sides by the same factor
• matching angles on the two shapes are equal.
In this activity, we will start with a small cartoon character, and then ‘blow it up’ to almost life-size.
Equipment: ruler, pencil, cartoon print, butcher’s paper or some other large piece of paper.
1 Do some research on the internet and select a cartoon character or any character of your
choice.
2 Draw a grid of 2-cm squares over the small cartoon character.
A B C D E F G H I
Example: The small Casper is 9 squares wide and 7 squares tall. 1
2
3
4
5
6
7

358 Maths Quest 10 New South Wales Australian curriculum edition Stages 5.1 and 5.2
measurement AND geometry
MEASUREMENT GEOMETRY

3 Label the grids with letters across the top row and numbers down the first column, as shown in
the example.
4 Get a large piece of paper and draw the same number of squares. You will have to work out the
ratio of similitude (e.g. 2 cm : 8 cm).
5 If your small cartoon character stretches from one side of the ‘small’ paper (the paper the image
is printed on) to the other, your ‘large’ Casper must stretch from one side of the ‘big’ paper to
the other. Your large grid squares may have to be 8 cm by 8 cm or larger, depending on the
paper size.
6 Draw this enlarged grid on your large paper. Use a metre ruler or some other long
straight-edged tool. Be sure to keep all of your squares the same size.
• At this point, you are ready to DRAW. Remember, you do NOT have to be an artist to
produce an impressive enlargement.
• All you do is draw EXACTLY what you see in each small cell into its corresponding large
cell.
• For example, in cell B3 of the Casper enlargement, you see the tip of his finger, so draw
this in the big grid.
• If you take your time and are very careful, you will produce an extremely impressive
enlargement.
• What you have used is called a RATIO OF SIMILITUDE. This ratio controls how large the new
picture will be.
A 2 : 5 ratio will give you a smaller enlargement than a 2 : 7 ratio, because for every two units
on the original you are generating only 5 units of enlargement instead of 7.
If Casper’s ratio is 1 : 4, it produces a figure that has a linear measure that is four times bigger.
Big Casper’s overall AREA, however, will be 16 times larger than small Casper’s. This is
because Area is found by taking length times width.
The length is 4 times longer and the width is 4 times longer.
Thus the AREA is 4 × 4 = 16 times larger than the original Casper.
His overall VOLUME will be 4 × 4 × 4 or 64 times larger! This means that big Casper will
weigh 64 times more than small Casper.

Chapter 10 • Deductive geometry 359


measurement AND geometry

Code puzzle

Why was the


archaeologist upset?
The triangles in each pair are congruent. Find the unknown in each
to solve the puzzle.

A
8 2x B 60°
4

H 5 5
16 8 8 30°
20 3 3
2x
2x°
12
20 4
7 8x°
12
12

J
x 35
7
I
25° 25°
15 15 112°
12

9 8 9x 30°
8
60° 81

O N
4x
24
30°
60°

25
9
W 130° 9
30°
20° 4x°
9

26
36 36 S 2x
130°

x° U
5x
R 10

25

8 14 13 35 6 30 5 4 13 14 9 2 90 14 9 13

360 Maths Quest 10 New South Wales Australian curriculum edition Stages 5.1 and 5.2
measurement AND geometry

 ACTIVITIES
Go to assessON
Chapter opener Digital docs for questions to
Video • Activity 10-C-1 (doc-13755): Quadrilaterals test your readiness
• The story of mathematics (eles-1849) (page 345) FOR learning, your
• Activity 10-C-2 (doc-13756): Harder progress AS you learn
10A Angles, triangles and congruence quadrilaterals (page 345) and your levels OF
Digital docs
• Activity 10-C-3 (doc-13757): Tricky achievement.
• Activity 10-A-1 (doc-5095): Review of quadrilaterals (page 345) www.assesson.com.au
congruent shapes (page 329) • SkillSHEET (doc-5279): Identifying
• Activity 10-A-2 (doc-5096): Practice with quadrilaterals (page 346)
congruent figures (page 329) • WorkSHEET 10.2 (doc-5283): Deductive
• Activity 10-A-3 (doc-5097): Tricky congruent geometry II (page 348)
figures (page 329)
• SkillSHEET (doc-5276): Naming angles, lines 10D Polygons
and figures (page 330) Interactivities (page 350)
• SkillSHEET (doc-5277): Corresponding sides • Angle sum of a polygon (int‐0818)
and angles of congruent triangles • Exterior angles of a polygon (int‐0819)
(page 330) Digital docs
• SkillSHEET (doc-5280): Angles and parallel • Activity 10-D-1 (doc-13758): Classifying
lines (page 330) polygons (page 350)
• Activity 10-D-2 (doc-13759): Calculating
10B Similar triangles angles in polygons (page 350)
Digital docs
• Activity 10-D-3 (doc-13760): Angle properties
• Activity 10-B-1 (doc-5098): Review of similar of polygons (page 350)
shapes (page 336) • WorkSHEET 10.3 (doc-5284): Deductive
• Activity 10-B-2 (doc-5099): Similarity practice geometry III (page 352)
(page 336)
• Activity 10-B-3 (doc-5100): Tricky similarity Chapter review
problems (page 336) Interactivities (page 353)
• SkillSHEET (doc-5278): Writing similarity • Word search (int-2853)
statements (page 339) • Crossword (int-2854)
• SkillSHEET (doc-5281): Calculating unknown • Sudoku (int-3597)
side lengths in a pair of similar triangles Digital docs
(page 339) • Chapter summary (doc-13761)
• WorkSHEET 10.1 (doc-5282): Deductive • Concept map (doc-13762)
geometry I (page 339)
To access eBookPLUS activities, log on to
10C Quadrilaterals www.jacplus.com.au
Interactivity
• Quadrilateral definitions (int-2786)
(page 340)

Chapter 10 • Deductive geometry 361


measurement AND geometry

Answers
CHAPTER 10 Deductive geometry
Exercise 10A — Angles, triangles, and congruence d
1    a a = 56° b b = 30° c c = 60° Table size Number of sides hit
d d = 120° e e = 68°
2    a I and III, SAS b I and II, AAS c II and III, RHS 5 cm × 3 cm 6
d I and II, SSS 7 cm × 2 cm 7
3    a x = 6, y = 60° b x = 80°, y = 50°
c x = 32°, y = 47° d x = 45°, y = 45° 4 cm × 3 cm 5
4    a x = 3 cm b x = 85°
c x = 80°, y = 30°, z = 70° 4 cm × 2 cm 1
d x = 30°, y = 7 cm
6 cm × 3 cm 1
e x = 40°, y = 50°, z = 50°, m = 90°, n = 90°
5    a Use SAS b Use SAS. c Use ASA. 9 cm × 3 cm 2
d Use ASA. e Use SSS.
6 C, D 12 cm × 4 cm 2
7    a x = 110°, y = 110°, z = 4 cm, w = 7 cm
b x = 70° c x = 30°, y = 65° e If the ratio of the sides is written in simplest form then the
8 The third sides are not necessarily the same. pattern is m + n − 2.
9 Corresponding sides are not the same. f There are two routes for the ball when hit from B. Either 2 or
10 Use SSS. 3 sides are hit. The ball does not end up in the same hole
11, 12 Check with your teacher each time.
Exercise 10B — Similar triangles A suitable justification would be a diagram — student
1    a i and iii, RHS b i and ii, SAS c i and iii, SSS to draw.
d i and iii, AAA e i and ii, SSS g Isosceles triangles and parallelograms. The triangles are
2    a Triangles PQR and ABC b Triangles ADB and ADC congruent.
c Triangles PQR and TSR d Triangles ABC and DEC h The shapes formed are parallelograms. There is only one
e Triangles ABC and DEC possible path although the ball could be hit in either of two
AB BC AC b f = 9, g = 8 directions initially.
3    a = = i Given m : n is the ration length to width in simplest form.
AD DE AE When m is even and n is odd the destination pocket will be
4 x=4 the upper left. When m and n are both odd, the destination
5 x = 20°, y = 214 pocket will be the upper right. When m is odd and n is even
6    a AAA b x = 3, y = 4 the destination pocket will be the lower right.
7 2.1 m j Students to investigate.
8 5m 8 Use SAS to show ΔDAE ≅ ΔBAE. Hence, DE = EB.
9 Check with your teacher. (See previous question.)
10    a ∠BAD = ∠BAC (common angle) ∠ADB = ∠ABC = 90° 9 AX || DY because ABCD is a parallelogram
ΔABD ~ ΔACB (AAA) AX = DY (given)
b ∠BCD = ∠BCA (common angle) ∠BDC = ∠ABC = 90° ∴ AXYD is a parallelogram since opposite sides are equal and
ΔACD ~ ΔACB (AAA) parallel.
11 Congruent triangles must be identical, that is, the angles must 10    a Use SAS.
be equal and the side lengths must be equal. Therefore, it is not b Use SAS.
enough just to prove that the angles are equal. c Opposite sides are equal.
12 Check with your teacher. 11 AC = DB (diameters of the same circle are equal)
Challenge 10.1 AO = OC and OD = OB (radii of the same circle are equal)
3 ∴ ABCD is a rectangle. (Diagonals are equal and bisect each
Exercise 10C — Quadrilaterals other.)
1    a True b True c True d False 12 Check with your teacher.
13 PS = QR (corresponding sides in congruent triangles are equal)
e False f False g False h False
PS || QR (alternate angles are equal)
2    a x = 36°, y = 62° b x = 5 cm, y = 90°
∴ PQRS is a parallelogram since one pair of opposite sides are
c x = 10°, y = 70° d x = 40°, y = 60° parallel and equal.
3 None are true. 14 MP = MQ (radii of same circle)
4 a, c, f PN = QN (radii of same circle) and circles have equal radii.
5 Parallelogram, rhombus, rectangle, square ∴ All sides are equal.
6 Square ∴ PNQM is a rhombus.
7    a b 6 sides c 7 sides 15, 16 Check with your teacher.
Challenge 10.2
x = !10 cm

362 Maths Quest 10 New South Wales Australian curriculum edition Stages 5.1 and 5.2
measurement AND geometry

Exercise 10D — Polygons 9 C


1 The sum of the interior angles is based on the number of sides of 10 x + 1
the polygon. Problem solving
The size of the exterior angle can be found by dividing 360° by 1    a Use SAS. b Use SAS. c Use SAS.
the number of sides. d They are all the same length.
2    a m = 60° b a = 45°, b = 45° c t = 35° e B and C
d x = 10° 2 Use SAS.
3    a y = 35° b t = 5° c n = 81° PQ = PS (corresponding sides in congruent triangles are equal)
d x = 15° e t = 30° 3 Rhombus, square
4    a i Irregular ii x = 95° b i Irregular ii p = 135° 4 A quadrilateral is a rhombus if:
c i Irregular ii t = 36° d i Irregular ii y = 70° a all sides are equal
e i Irregular ii m = 36° b the diagonals bisect each other at right angles
5 w = 75°, x = 105°, y = 94°, z = 133° c the diagonals bisect the angles they pass through.
6 80°, 80°, 95°, 95° 5 WZ || XY (co-interior angles are supplementary) and WZ = XY
7    a a = 120°, b = 120°, c = 60°, d = 60°, e = 120°, f = 240° (given)
b m = 10°, n = 270°, o = 50° ∴ WXYZ is a parallelogram since one pair of sides is parallel
8 60° and equal.
9    a    i 2 ii 5 iii 9 iv 14 6 A B
b Number of diagonals = 12n(n − 3)
10 Internal angle = 180° − 360°
n
D C
Chapter review
Fluency ∠ABD = ∠ADB (angles opposite the equal sides in an isosceles
1    a I and III, ASA or SAS b I and II, RHS triangle are equal)
2    a x = 8 cm b x = 70° ∠ABD = ∠BDC (alternate angles equal as AB || DC)
c x = 30°, y = 60°, z = 90° ∴ ∠ADB = ∠BDC
3    a Use SAS. b Use ASA. ∴ Diagonals bisect the angles they pass through.
4    a Similar, scale factor = 1.5 b Not similar 7 Corresponding sides are not the same.
c Similar, scale factor = 2 8 ∠FEO = ∠OGH (alternate angles equal as EF || HG)
5    a x = 48°, y = 4.5 cm b x = 86°, y = 50°, z = 12 cm ∠EFO = ∠OHG (alternate angles equal as EF || HG)
c x = 60°, y = 15 cm, z = 12 cm ∠EOF = ∠HOG (vertically opposite angles equal)
6 Use the equiangular test. ∴ ΔEFO ~ ΔGHO (equiangular)
7 Use the equiangular test. 9 A rhombus is a parallelogram with two adjacent sides equal
8 A in length.
10 Rectangle, square
Communicating — Rich task
Teacher to check.
Code puzzle
B C His job was in ruins.
D
Bisect ∠BAC
AB = AC (given)
∠BAD = ∠DAC
AD is common.
∴ ΔABD ≅ ΔACD 1 SAS 2
∴ ∠ABD = ∠ACD 1 corresponding sides in congruent triangles are equal 2

Chapter 10 • Deductive geometry 363


ICT Activity
parents need them to pay back the $29 per month,
with either simple interest or compound interest
charged. What is the best option for the children?
Getting the budget in order How much do they need to work at their $9.75-per-
hour part-time jobs to cover the cost?
SEARCHLIGHT ID: PRO-0087 • Rent: The Thompson family lives in a rental house
within 50 km from a major city. How much rent would
Scenario they be paying?
Budgets are of great importance to everyone. From the • Taxes
federal budget down to the local council budget, they all • Utilities (electricity, water, gas, phone)
impact on our lives. On a smaller scale, but of no less • Food
importance, is the family budget. • Other expenses
As a financial planner, you have been asked to Within the Thompson family, Mr Thompson takes care
prepare a budget for the Thompson family. The of the household duties and Mrs Thompson works in
Thompson family lives in a major city in Australia and a salaried position, earning $85 000 per annum. Their
comprises of Mr and Mrs Thompson and their two children have part-time jobs to repay the cost of their
teenage children. They want to go on a holiday and they phone plans and to try and save for new clothes.
plan to save for a year. Taking their ongoing expenses
into account, they want to see if and how they can afford Task
this, following your recommendations. You will produce an oral presentation and a written
You are to form a financial-planning group of three budget plan with recommendations for the Thompsons’
people. Each group member is to produce a budget family holiday. The budget plan will include the preferred
plan that addresses the following holiday and ongoing method of interest charged to the children for the
expenses. cost of their phone plan and the hours per week the
• Holiday: They want to be able to spend $5000 on a children need to work at their part-time jobs to make
holiday within Australia. Where can they go and for this repayment. Your budget plan should also include
how long, taking flights, accommodation, meals and value-for-money rentals located within 50 km of a
sightseeing expenses into account? major city, which will help the family assess how much
• Phone plans: Their two children have phone plans they should be spending on rent. You also need to
that each cost $29 per month, currently paid by their recommend a package holiday that includes flights and
parents. However, to help fund the holiday, their accommodation. Your presentation should explain why

364 Maths Quest 10 New South Wales Australian curriculum edition Stages 5.1 and 5.2
you have made each of your recommendations and • Research. Make notes of
give evidence to back up you decisions, taking all the important information and SUGGESTED
expenses into account. ideas that you discovered SOFTWARE
during your research. Enter • ProjectsPLUS
your findings as articles • Microsoft Word
under your topics in the • PowerPoint, Prezi,
Research forum. You should Keynote or other
each find at least three presentation
sources of information software
(including off-line resources • Microsoft Excel
such as books and • CAS calculator
newspapers). You can view (optional)
and comment on other
group members’ articles
and rate the information they have entered. When
your research is complete, print your Research Report
to hand in to your teacher.
• Visit your Media Centre and download the budget
template and PowerPoint sample to help you prepare
your presentation. Your Media Centre also includes
Process images to help liven up your presentation.
• Open the ProjectsPLUS application for this chapter • Use the budget template to give a clear overview of all
in your eBookPLUS. Watch the introductory video the expenses taken into account and to give an overall
lesson, click the ‘Start Project’ button and then set summary of the whole family budget. Make sure
up your project group. You can complete this project you remember to address all the expenses that the
individually or invite other members of your class to Thompson family has requested you take into account.
form a group. Save your settings and the project will • Use the PowerPoint template to develop your
be launched. presentation. Remember that you are making
• Navigate to your Research Forum. Here you will find a recommendations that you believe are best for the
series of topics that will help you complete your task. Thompson family. Make sure you cover all the details
Select the expenses you are researching or add new they that have requested, and that your presentation
expenses you wish to include. will grab their attention.

ICT Activity — projectsplus 365


PROBLEM SOLVING

chapter 11

Problem solving I

WHY LEARN THIS?


Problem solving is something performed by many
people every day. Very good problem solvers think in
a very organised and systematic way, and they reason
logically and effectively. They can justify their decisions
because they can provide evidence for how they arrived
at their conclusions. Solving mathematical problems
will enable you to think laterally and provide you with
the opportunity to practise your skills and stretch your
imagination.
Watch this video
The story of mathematics

Searchlight ID: eles-1696


problem solving

1 A cuboid has dimensions 10 cm by 12 cm by 18 cm. Find the length of the longest


diagonal inside the cuboid.
2 Find the volume of a right circular cone with a base diameter of 14 cm and slant
height of 25 cm.
3 Consider a right‐angled triangle, such that the two shorter sides are 6.4 mm and
8.9 mm in length. Find the angle between the shortest side and the hypotenuse.
4 The perimeter of a rectangle is 20 cm and its area is 14 cm2. Calculate the
dimensions of the rectangle, correct to 1 decimal place.
5 Mary has baked a birthday cake in the shape of a trapezium.

16 cm

14 cm
18 cm

She has 60 cm of ribbon which she wants to wrap around the sides of the cake. Does
she have enough ribbon? Explain your answer.
6 Find the angle of elevation to the top of a 27.3-m high Norfolk Pine tree that is
83.6 m from the observer. Assume that the observer’s eye is 1.6 m above ground
level.
7 Marlon substituted numbers into the equation below until he had a true statement.
x(x − 3) = 10
Marlon’s answer for the problem was 5.
a Is Marlon’s answer right or wrong? Explain.
b A friend of Marlon’s showed him another way to solve the problem.
x(x − 3) = 10
x = 10 x − 3 = 10
x = 13
(10, 13)
Is Marlon’s friend correct? If the solution is correct explain why, and if the
solution is incorrect, provide a correct solution.
8 A piece of flat pastry is cut in the shape of a right‐angled triangle. The longest side is
6b cm and the shortest is 2b cm.
a Find the length of the third side. Give your answer in exact form.
b Find the sizes of the angles in the triangle.
c Prove that the area of the triangle is equal to 4 !2b2 cm2.
9 Cameron purchased 500 tickets for an AFL game so that all of the teachers and
students in the school would be able to attend the match.
Teachers AFL tickets: $15/ticket
Students AFL tickets: $5/ticket
The total cost for the game was $3500.
How many students attended the game?

368 Maths Quest 10 New South Wales Australian curriculum edition Stages 5.1 and 5.2
problem solving

10 Find the pairs of parallel lines from the following list and state the gradient and
y‐intercept for each.
a 3y + 6x = −36 b 4y = −4x + 20
c 3y + 1 = 9x d 12 = 2x + 2y
1
e 10y = − x − 8 f 12x = −6y + 12
2
g 2y − 6x − 7 = 0 h 20y = −x + 5
11 The perimeter, P, of a square lies in the range e to f ; that is, e ≤ P ≤ f . In terms of
e and f what is the range of values for its area, A?
12 Find the equation of the straight line going through (−1, 5), parallel to the line
which passes through (0, 4) and (5, −3).
13 Warwick was solving a pair of simultaneous equations using the elimination
method and reached the result that 0 = −5. Suggest a solution to the problem,
giving a reason for your answer.
14 A movie projector uses 35 mm film (35 mm wide and 24 mm high) with a light
source 60 mm from the film’s surface.
a How far away is the projector’s light source from the screen if the width of the
image on the screen is 16.5 m?
b If the distance between the film and the light source is halved, what happens to
the width of the image on the screen?
15 A landscape gardener wishes to put a fence around a rectangular lawn. The lawn’s
width is 3 m shorter than its length and there is to be allowance for a 2-m wide gate.
a Develop a formula for the total length of the fence in terms of the length of the lawn.
b The cost of the fence is $23 per metre plus a $100 additional fixed fee. Modify
your formula to provide the cost of the fence.
c Use your formula to determine the cost of a fence for a lawn whose width is 12 m.
16 A right‐angled triangle is drawn in a circle of diameter d cm as shown in the
diagram below. O is the centre of the circle.

8
O
d 4

a Show that d = 4 !6 cm.


!2
b Show that the proportion of the area of the triangle to the area of the circle is .

17 The cost of a return airline ticket to Perth from Sydney varies between airlines.
If a ticket travelling with Virgin Green Airline costs $458 and a ticket travelling
with Qintas costs $506, determine
the number of people who travelled
by air to Perth from Sydney if
there were 20% more passengers
that flew with Virgin Green and
the total price for all airline tickets
was $63  336.

Chapter 11 • Problem solving I 369


problem solving

18 The diagram below represents the safety ratio for placing ladders against vertical
structures.

4 units

1 unit

a Using the values shown in the diagram, determine the value of the angle, θ .
Write your answer correct to the nearest minute.
b A 3‐metre ladder is placed against a vertical wall. Determine the horizontal
distance, to the nearest centimetre, that the ladder should be placed so that it
satisfies the safety regulations.
19 Carol is celebrating her 16th birthday. In her
excitement she cuts the circular cake into 8 unequal
sections. Her brother takes the largest piece, which
is twice as much as her mother’s piece. Her sister
takes one of the smaller pieces, which is 13 the size
of her mother’s piece of cake. Carol and her father
each take a piece of cake that is 112 times as large
as Carol’s mother’s piece. If Carol had cut the cake
into 8 equal slices, each slice would have been the
same size as her mother’s piece of cake.
a What fraction of the cake was eaten by Carol and
her family?
b What fraction of the cake remains?
c If the exact amount of cake, in cm2, remaining
after Carol and her family have eaten their
one piece of cake is 160π, determine the exact
diameter of the cake.
20 Calculate the x‐coordinate of the intersection point(s), if any, of:
y = 2x2 − 5x − 3 and y = −x2 − 3x
Give your answer in exact form.
21 Shane is a coach driver who conducts tours in outback Australia. All tours are
based on twin share at a cost of $x per passenger. For passengers wishing to have
their own room, an additional cost of $385
is added to the overall tour price. On any
tour, an average of 75% of passengers
choose twin share.
a If there are n passengers, write down
an equation, in terms of n and x, that
can be used to determine the total
amount, A, in dollars, collected in
tour money.

370 Maths Quest 10 New South Wales Australian curriculum edition Stages 5.1 and 5.2
problem solving

b Shane conducts a tour with 50 passengers. On the next tour there are
45 passengers. The difference in the total amount between the first and second
tour is $17 981.25. Determine the value of x.
22 Jacques is test driving a new model Rocket Roadster. The speed of the car can
be modelled using the equation S 1 t 2 = −3t2 + 12t + 27 , where S is the speed in
metres per second 1 m/s 2 and t is time, in seconds.
a What was the initial speed, in km/h, of the car when the testing began?
b By converting S 1 t 2 into turning point form, determine the maximum speed,
in km/h correct to 1 decimal place, that the Rocket Roadster reached in this
road test and the time taken, in seconds, to reach that speed.
23 Find two consecutive natural numbers whose squares differ by 75.
24 Rebecca and Bethany are participating in a fund raising charity door knock.
In the first hour, each girl collected x number of gold coins.
In the second hour Rebecca collected (x − 1) gold coins and in the third hour she
collected (3x − 4) gold coins.
In the fourth hour, the number of gold coins each girl collected was the product of
the number of coins collected during the second and third hours.
Bethany’s number of gold coins collected in the fourth hour can be expressed as
2x2 + 5x − 7.
a Show that the number of coins Rebecca collected during the fourth hour was
3x2 − 7x + 4.
b Show that the expression, in terms of x, for the combined number of
coins both Rebecca and Bethany collected in the fourth hour is
(x − 1)(5x + 3).
25 In an election for Year 10 representatives on the school council, 25 of the votes went
to James, 14 to Jennifer, 16 to Raoul, 10
1
to Amy and the remaining 20 votes went to
Diana. If every student voted, how many students were there in year 10?
26 An Xbox games package comes in a box
that has a length 10 cm longer than its
width and a height that is 7 cm greater
than its width.
a Using the variable x to represent the
width of the box, write an equation for
the box’s volume.
b Find the volume of the box if its width
is 30 cm.
27 A cross brace (shown in red) has been placed to support the roof of a garage as
shown. Find the length (in mm) of this supporting beam.

1800 mm

5200 mm

3000 mm

Chapter 11 • Problem solving I 371


problem solving

28 When the movie The Fellowship of the Ring was shown in the cinema, every seat
1 550 2 was taken. The price of admission for adults was $9.50 and for children
$4.50. The takings for one night were $4275. How many adults and children were
present at the movie?
29 A rectangular coffee table cloth is to be decorated by sewing lace onto the edge of
the material. Its length is four times its width.
a If the width of the material is x cm, express the length of the cloth in terms of x.
b Give an expression for the perimeter and the area of the cloth in terms of x.
c Find the length and width of the cloth if its perimeter is 3 m.
d If the width of the lace is 6 cm, what is the outside perimeter of the cloth now,
and how much area does it cover?
e Given the original perimeter was 3 m, what increase in area of the cloth was
achieved by adding the lace?
30 This 8 cm by 12 cm rectangle is cut into two sections as shown.
6 cm 6 cm

8 cm 10 cm

12 cm

a Draw labelled diagrams to show how the two sections can be rearranged to
form:
i a parallelogram
ii a right‐angled triangle
iii a trapezium.
b Comment on the perimeters of the figures.
31 A yacht is anchored off an island. It is 2.3 km from the yacht club and 4.6 km from
a weather station. The three points form a right-angled triangle at the yacht club.

Weather station
Yacht club

2.3 km
4.6 km

Yacht

a Calculate the angle at the yacht between the yacht club and the weather station.
b Calculate the distance between the yacht club and the weather station.
The next day the yacht travels directly towards the yacht club, but is prevented
from reaching the club because of dense fog. The weather station notifies the yacht
that it is now 4.2 km from the station.
c Calculate the new angle at the yacht between the yacht club and the weather
station.
d Determine how far the yacht is now from the yacht club.

372 Maths Quest 10 New South Wales Australian curriculum edition Stages 5.1 and 5.2
problem solving

32 The minute hand in Penny’s watch is 1 cm long. Someone told her that the tip
of the hand travels more than 30 m in 8 hours. Is this true? Show full working to
justify your answer.
33 The small and large triangles in this figure are similar.

25
19

x
15
y

22

Determine the values of the pronumerals.


a−b
34 a Θ b is defined as .
a
b

1
2
Θ 13
What is the value of 1
in simplest terms?
3
Θ 12

35 Pulsars are rapidly spinning stars. They spin at


incredible rates when they are first formed —
about 30 times every second. As they age, they
slow down. Astronomers have represented
the spinning rates of two pulsars (Crab
Nebula and AP 2016 + 28) by two simple
equations.
Crab Nebula: P = 0.033 + 0.000 013T
AP 2016 + 28: P = 0.558 + 0.000 000 004 7T
P is the time (in seconds) it takes for the pulsar
to spin once on its axis, and T is the number of
years since the year 2000.
In approximately how many years will the
two pulsars be spinning at the same rate?
36 Draw two straight lines across the face of this clock, so that the sum of the numbers
in each region formed is the same.

11 12 1
10 2
9 3
8 4
7 6 5

Chapter 11 • Problem solving I 373


problem solving

37 Tennis balls are stacked in the shape of a triangular pyramid, with 5 balls on each
side of the base. How many balls are in the whole stack?

38 Prove that the set of numbers represented by 2n, (n2 − 1) and (n2 + 1), where
n represents a neutral number, produces Pythagorean triples for all values of
n greater than 1. Explain the restriction on the value of n.
39 Consider the number 234 written in words.
TWO HUNDRED AND THIRTY FOUR
The letters of each word are cycled separately as shown below, and placed in a
numbered vertical list.
1 TWO HUNDRED AND THIRTY FOUR
2 WOT UNDREDH NDA HIRTYT OURF
3 OTW NDREDHU DAN IRTYTH URFO
..
..
..
n TWO HUNDRED AND THIRTY FOUR
If n > 500, what is its smallest value?
40 Trains travel along two straight parallel tracks between Allensville and Bentley,
with the journey taking 4 h 15 min each way. The trains leave both towns on the
hour every hour.
If I leave Allensville at 12 noon and travel towards Bentley, how many trains will
pass by me coming in the opposite direction?
41 Expand (3x − 2y) 4.
42 Solve for x and y.
y = ax − 3b
dx + ey = f
43 When two algebraic fractions are equal, a method known as ‘cross‐multiplying’
makes finding the value of x a lot quicker.
a c
= so, a(x + d) = c(x + b)
x+b x+d
Expand and solve normally.
Use the above method to find x in each of the following.
3 5 3x + 4 5x − 4 2x + 1 4
a = b = c =
x−2 x+2 2 6 1 + 3x 5

374 Maths Quest 10 New South Wales Australian curriculum edition Stages 5.1 and 5.2
problem solving

44 Determine the length of the diagonal x.

x 8 cm

5 cm
7 cm

45 The formula that can be used to find the surface area, A cm2, of a solid cylinder
with radius r cm and height h cm is A = 2πr(r + h).
a Find an exact value for A when r = 4, h = 6
b Describe in words the change that will occur to A if r and h are both halved.
Justify your reasoning mathematically.
46 Translate each of these parabolas by the given amounts, then write the new
equation in the form y = ax2 + bx + c.
a y = x2 − 4x + 1 translated 3 units left, 2 units up.
b y = −4x2 + 6x − 2 translated 3 units down and 1 unit right.
c y = (x − 4) 2 − 5 translated 2 units left and 5 units up.
47 Develop a formula for the volume of a cone with a radius r and a slant height s.
48 A large rectangular advertising banner is to be placed on the side of a building. The
banner has a diagonal of length 4 !17 m and a height of 5 !5 m.
a Determine the exact width, in metres, of the banner.
b Determine the exact area, in m2, of the banner.
c To attach the banner to the side of the building, anchor points are attached
around the border. There is an anchor point attached to each corner and anchor
points across the width and the height.
i There are 7 anchor points across the width at the top and 7 at the bottom
of the banner. Determine the exact length, in metres, between these
points.
5 !5
ii Anchor points are placed 3 metres apart along both sides of the height
of the banner. Determine the total number of anchor points needed for the
banner.
49 A circular piece of paper has a 90° sector removed as shown in the diagram
below.

r
90°

The remaining area is carefully folded to make the shape of a cone.


a What is the slant height of this cone?
b Develop a formula for the radius of the base of this cone in terms of r.
3
c Show that the volume of the cone is 64 !7πr3.

Chapter 11 • Problem solving I 375


problem solving

50 Farmer Gillian has two rectangular paddocks ready for sowing.


One paddock has dimensions (2x − 1) metres by (3x + 1)
metres, and the second paddock has an area of
(36x2 − 6x − 6) m2.
a Show that the area ratio of paddock 1 to paddock 2 is 1:6.
b The area of paddock 2 is 97 026 m2. Show that the value of
x is 52. Hence find a set of possible dimensions for paddock
1 using your answer from part a.
51 Greg is building a garden shed. He measured out the width and
length of the shed to be 3 m by 4.5 m.
a The walls of the shed will be 2 metres high and the roof
will be a height of 3 metres from the base. The shed will
be constructed entirely from corrugated iron. The diagram
below shows the shed and its dimensions. The shed will
be built on a concrete slab. Using Greg’s measurements of
length of 4.5 m and width of 3 m, determine the minimum amount of iron, in m2,
required. Write your answer correct to 2 decimal places.

1m
2m

4.5 m
3m

b Corrugated iron is sold in lineal metres at $11.80 per metre. The effective width
of the corrugated iron sheet is 762 mm (this is allowing for overlapping of
sheets). Determine the minimum cost for the corrugated iron. Write your answer
to the nearest $10.
52 A certain type of carpet has
a width of (x + 2) metres.
Customers can purchase the carpet
in any length. Mr Barnes buys
(x + 5) m of this carpet for his
rumpus room and Mr Snowdon
buys a 4 m length for his family
room.
a Write an expression for the area
of carpet that each man buys.
b Write an expression for the
difference in area if Mr Barnes has the longer piece of carpet.
c Factorise and simplify this expression.
d If Mr Barnes has bought 6 m2 more than Mr Snowdon, find the width of the
carpet.
e What area of carpet did each man buy?

376 Maths Quest 10 New South Wales Australian curriculum edition Stages 5.1 and 5.2
problem solving

53 The shape of this vase is a truncated cone, as shown in the diagram below.
20 cm

30 cm

15 cm

Show that the total amount of water, in litres, that can be poured into the vase is 2.3π.
54 Martha decides to redesign the front cover of her diary which has an area equal to
(x2 − 3x − 10) cm2, where x is a positive integer.
a Factorise this expression to find the dimensions of the diary cover in terms of x .
b Write down the shorter length in terms of x.
c If the shorter sides of the diary cover are 12 cm in length, find the value of x.
d What is the area of the front cover of Martha’s diary?
55 A rectangular hallway rug is five times as long as it is wide. Its diagonal length is
410 cm. How wide and long is the rug?
56 A circular dining table made of cedar timber is inlaid with glass as shown in the
diagram below. The radius of the glass top is r cm with a 20 cm ring of cedar
around it.

a State the diameter of the glass (in terms of r).


b Give the radius of the glass and wood (the table top).
c Calculate the area of the glass.
d Determine the area of the top of the table (glass and wood)
e Write an expression to find the area of the wood section only and write it in
factorised form.
f If the radius of the glass is 40 cm, find the area of the wood needed to surround
the glass. Give your answer in m2.
g The manufacturers want to make a slightly larger table in the same design using
the same width cedar ring. If the area of the table top is to be 2 m2, find the size
of r (to the nearest cm).

Chapter 11 • Problem solving I 377


problem solving

57 A surveyor measures the angle of elevation to the top of a lighthouse from a point on
the ground 130 m from its base as 37°. When he looks further down the lighthouse,
he sees a large balcony. The angle of elevation to the balcony from the same point is
31°. What is the distance from the balcony to the top of the lighthouse?
58 Robyn keeps guinea pigs in a small square enclosure with sides measuring x m.
The number of guinea pigs is increasing so she wants to increase the size of the
enclosure by 1 m on one side and 3 m on the adjacent side.
a Draw a labelled diagram of the original square and show the additions to it.
b Write an expression for the area of the new enclosure.
c To satisfy animal safety requirements, the area of the enclosure will be 15 m2.
Find the dimensions of the original enclosure.

59 Two guy wires are used to support a flagpole as shown. One reaches the top of the
flagpole and the other is attached part way down the pole.

Wire

9.5 m
3m

6m

a What is the height of the flagpole (to the nearest metre)?


b What angle (to the nearest degree) does the longer guy wire make with the ground?
c The shorter wire is attached to the flagpole 1 m from the top. How long is this wire?
60 When a drop of water hits the flat surface
of a pool, circular ripples are made. One
ripple is represented by the equation
x2 + y2 = 9 and 5 seconds later, the
ripple is represented by the equation
x2 + y2 = 225, where the lengths of the
radii are in cm.
a State the radius of each of the ripples.
b Sketch these graphs.
c How fast is the ripple moving
outwards?
d If the ripple continues to move at the
same rate, when will it hit the edge of
the pool which is 2 m from its centre?

378 Maths Quest 10 New South Wales Australian curriculum edition Stages 5.1 and 5.2
problem solving

61 Stephie, a tennis player, serves the ball in


a tournament. She throws the ball in the
air and hits it over the net. Her arm length
is 60 cm and it is 40 cm from her grip on
the tennis racquet to the centre of the
racquet. How far does the centre of the
racquet travel if she swings through an
angle of 300°?

62 Bridgette is practising her golf drives. The


path the golf ball takes is defined by the
quadratic equation h = −16 (d − 6) 2 + 6,
where h is the height of the ball above the
ground for a horizontal distance of d. Both
h and d are in metres.
a Find the value of h when d = 0.

b State the turning point of the graph


h = −16 (d − 6) 2 + 6
c Sketch the graph of this relationship.
d What horizontal distance does the golf
ball cover in its flight?
e What is the maximum height the golf
ball reaches?
f At what horizontal distance was the
golf ball at its maximum height?
63 This cable drum has the measurements shown.

10 cm

25 cm
30 cm

10 cm

50 cm

a What volume of wood was used in its construction?


b Determine its surface area.

Chapter 11 • Problem solving I 379


problem solving

64 A skip bin for waste has been delivered to a building site. Its shape is in the form of
a trapezoidal prism with dimensions as shown in the diagram.
4m

2m

3.5 m
3m
a Calculate the volume of material this skip can hold. (Assume that it is not loaded
beyond the top rim.)
b A smaller skip has a volume one‐eighth the size of the larger one. If its shape is
similar to that of the larger one, what would its dimensions be?
65 The equation (x − 1) 2 + (y − 2) 2 = 4 describes a circle.
a State the centre of the circle.
b State the radius of the circle.
c Find the x‐ and y‐intercepts of this circle.
d Sketch the circle, clearly marking the centre.
66 Find the sum of the angles at the tips of this regular star.

67 There is a theorem which says:


If two distinct numbers are exactly divisible by the difference of the two numbers,
the difference is the HCF of the two numbers.
Explain what this means, illustrating with an example.
68 Explain why all perfect squares have an odd number of factors. Give an example to
support your explanation.
69 A cone is formed from a sector of a circle with a central angle of 72°. The radius
of the base of the cone is 3.18 cm. What is the radius of the circle from which the
sector was taken?
70 This tile pattern is made using congruent triangular tiles. There is 1 tile in row 1, 3
in row 2, and so on.

Develop a formula to determine the number of tiles needed to complete a pattern of


this type with r rows.

380 Maths Quest 10 New South Wales Australian curriculum edition Stages 5.1 and 5.2
problem solving

71 The school cafeteria sells apples and bananas at a different price. Six apples and
4 bananas cost $7, while 1 apple and 9 bananas cost $4.50.

a Show how you can determine how much more an apple costs, compared with a
banana, without actually finding the cost of each.
b What is this difference in price?
72 When children are sick, it’s important they’re given the correct dosage of medicine.
If adult medicines are the only ones available, you need to convert the adult dosage
into a safe dose for young children. One rule which can be used is:
Age in years + 1
child’s dose = × adult dose
24
a For an adult dose of 5 mL, how many mL would you give a 5‐year old?
Another rule commonly used is:
Age in years
child’s dose = × adult dose
age + 12
b How many mL of a 5-mL adult dose would you administer to a 5‐year old using
this formula?
c Your two answers should be different. Comment on this difference.
d Is there any age for which these two formulae give the same dosage?
73 A group of four people out bushwalking comes across a suspension bridge as the
last obstacle they need to cross to reach their campsite. They can’t all cross at once,
because the bridge can only support a maximum of 2 people at a time. Unfortunately
it is approaching dark, and they only have 1 torch among the 4 of them.

When walking alone, the four people would take 1, 2, 5 and 10 minutes to
cross one way. With 2 people walking together, because they need the flashlight for
safety reasons, they must travel at the speed of the slower person. One person must
then travel back across the bridge each time to bring the flashlight back.
How can the group arrange themselves for the bridge‐crossing to take the minimum
time? What is this minimum time?

Chapter 11 • Problem solving I 381


problem solving

74 Timber railings are manufactured to be 100 cm long, with a possible error of


4%. Dana’s deck is 55 m long, and she plans to place the railings end‐to‐end.
How many of these railings should she order to ensure she can cover the whole
length?
75 A palimage of a number is the number that has the same digits as the given number,
but in the reverse order. For example, the palimage of 476 is 674.
If the sum of a number and its palimage is 968, what could the original number
have been? There is more than one answer. See how many you can find.
76 A domino is a 2-dimensional figure formed by two congruent squares that share a
common side.

A tetromino is a 2-dimensional figure formed by four congruent squares that share


common sides.
a Draw the shapes of all of the different tetrominoes that are possible.
b Compare their perimeters.
−b ± "b2 − 4ac
77 The quadratic formula is x = .
2a
2c
An alternative form of the quadratic formula is x = .
−b ± "b2 − 4ac
Choose a quadratic equation and show that the two formulas give the same
answers.
78 You will be familiar with the following unit fraction additions.
1
2
= 13 + 16
1
3
= 14 + 12
1

1
5
= 16 + 30
1

Write a general equation involving n of the type


1
= .  . .
n
to represent unit fraction additions of this type.

382 Maths Quest 10 New South Wales Australian curriculum edition Stages 5.1 and 5.2
problem solving

79 It has been said that if you multiply the y‐coordinates for a particular x‐value of two
straight lines, then plot this product against the x‐value, a parabola will result. To
investigate this claim, consider the two straight lines y = −2x + 4 and y = x − 3.
Complete the following tables.
y1 = −2x + 4
x −2 −1 0 1 2 3 4 5 6
y1
y2 = x − 3
x −2 −1 0 1 2 3 4 5 6
y2

x −2 −1 0 1 2 3 4 5 6
y1 × y2

What is your conclusion regarding the claim?


80 A small company manufactures and sells muesli bars. The set‐up cost to make the
bars themselves was $5400, and the cost for ingredients is 45c per bar. The set‐up
cost to package them was $7500, and the cost of materials for each package is 35c.
The bars are sold in a multi‐pack at 5 for $6.
How many multi‐packs would the manufacturer need to sell in order to break even?

Chapter 11 • Problem solving I 383


problem solving

Answers
CHAPTER 11 Problem solving I
1 23.83 cm 29    a The cloth is x cm wide and 4x cm long.
2 1231.5 cm3 b P = 10x, A = 4x2
3 54.28° c Length = 120 cm, width = 30 cm.
4 8.3 cm by 1.7 cm d Perimeter = (10x + 48) cm, area = (4x2 + 60x + 144) cm2
5 No, Mary will need 64.5 cm of ribbon. e The area has increased by 1944 cm2.
6 17.05° 30    a i 6 cm
7    a This is a quadratic equation, which means that there is a
possibility of two different answers. Marlon has one of the
two parts of the answer correct. 10 cm 8 cm 10 cm
b No.
x(x − 3) = 10
x2 − 3x = 10 12 cm
x2 − 3x − 10 = 0 ii
(x − 5) (x − 2) = 0
(x − 5) = 0 or (x − 2) = 0 10 cm 8 cm
x=5 x=2
8    a 4!2b
b 19.5°, 70.5°, 90°. 10 cm 8 cm
c Area = 12 base × height
12 cm
= 12 × 2b × 4!2b
iii 6 cm
= 4!2b2 cm2.
9 400 students
10    a (y-intercept −12) and f (y-intercept = 2), gradient = –2;
b (y-intercept 5) and d (y-intercept = 6), gradient = –1; 10 cm 8 cm 10 cm

c A y-intercept – 13 B and g A y-intercept = 72 B , gradient = 3;


d (y-intercept – 45) and h A y-intercept = 14 B , gradient = −20
1
; 12 cm 6 cm

e 2 f 2 b Perimeter of rectangle = 40 cm.


11 a b ≤ A ≤ a b
4 4 Perimeter of parallelogram = 44 cm,
7 18 Perimeter of triangle = 48 cm,
12 y = − x + Perimeter of trapezium = 44 cm. The triangle has the largest
5 5
13 Any false statement that occurs during the solving of perimeter, while the rectangle has the smallest.
simultaneous equations indicates the lines are parallel, and have 31    a 60° b 3.98 km
no points of intersection. c 71° d 1.34 km
14    a 28.3 m 32 True: the tip travels 30.2 m.
b The image width doubles. 33 x = 13.75, y = 11.4
15    a Total length = 4l − 8, where l is the length of the lawn. 34 −49
b Cost = 23(4l − 8) + 100 35 Approximately 40 400 years from now
c $1296 36
16 Check with your teacher. 12
11 1
17 132 passengers — 72 Virgin Green passengers, 60 Qintas
10 2
passengers
18    a 75°58′ b 73 cm 9 3
19 5 8 4
19    a b c 32!3 cm
24 24 7 6 5
1 ± !10
20 x =
3
21    a A = nx + 96.25n b $3500 The total of the numbers here in each region is 26.
22    a 97.2 km/h b 140.4 km/h in 2 seconds 37 35 tennis balls
23 The square numbers are 1, 4, 9, 16, 25, 36, ………… The 38 Answers may vary. The value of n must be greater than 1 because
difference between these numbers is 3, 5, 7, 9, 11.  .  . If this the second number would be 0 if n were 1.
continues to 75, it is the 37th number, so 382 − 372 = 75. So, the 39 505
two natural numbers are 37 and 38. 40 9 trains
24 Check with your teacher. 41 81x4 − 216x3y + 216x2y2 − 96xy3 + 16y4
25 240 students f + 3be f + 3be
42 x = ; y = ac d − 3b
26    a V = x(x + 10) (x + 7) b 44 400 cm3 ae + d ae + d
27 5704 mm 43 a x = 8 b x = −4 c x = 12
28 360 adults and 190 children 44 11.75 cm

384 Maths Quest 10 New South Wales Australian curriculum edition Stages 5.1 and 5.2
problem solving

45 a A = 80π cm2 60 a First ripple’s radius is 3 cm, second ripple’s radius is 15 cm.
b If radius and height are both halved, the surface area is b y
15
one‐quarter its original value.
46 a y = x2 + 2x
10
b y = −4x2 + 14x − 15
c y = x2 − 4x + 4 5
47 V = 13πr2"s2 − r4
48 a 7!3 metres 0 x
–15 –10 –5 5 10 15
b 35!15 m2 –5
7!3
c i metres
6 –10
ii 18
–15
49 a r
b 34r c 2.4 cm/s
c Check with your teacher. d 1 minute 23 seconds after it is dropped
(2x − 1) (3x + 1) 1 61 The centre of the racket travels 5.24 m.
50 a = . 62 a 0 b 1 6, 6 2
6(2x − 1) (3x + 1) 6
b Solving 36x2 − 6x − 6 = 97 026 using any method gives c h

Height (metres)
8
x = −51.833 metres and x = 52. Ignore the negative solution 6
because x > 0 for measurement units. 4
Possible dimensions could be: 2
3(2x − 1) by 2(3x + 1) 0
0 2 4 6 8 10 12 d
2(2x − 1) by 3(3x + 1) Distance (metres)
6(2x − 1) by 1(3x + 1) d 12 m e 6m f 6m
(2x − 1) by 6(3x + 1). 63 a 56 941 cm3 b 11938 cm2
Or any possible combination for numbers whose product is 6, 64 a 24.5 m3
such as 1.5 and 4 b The dimensions of the smaller skip are half those of the
51    a 49.23 m2 b $760 larger one.
52 a Mr Barnes has (x + 2) (x + 5) m2; Mr Snowdon has 65 a Centre is (1, 2).
4(x + 2) m2. b Radius is 2.
b (x + 2) (x + 5) − 4(x + 2) c x‐intercept 1; y‐intercepts (−!3 + 2) and ( !3 + 2)
c (x + 2) (x + 1) d y
d The carpet has a width of 3 m.
5
e Mr Barnes bought 18 m2 and Mr Snowdon bought 12 m2.
53 Check with your teacher. 4
54 a (x − 5) (x + 2) 3
b (x − 5) is the shorter length. (1, 2)
c x = 17 2
d 228 cm2 1
55 The rug is 80 cm wide and 400 cm long.
56 a 4r cm 0 x
–2 –1 1 2 3 4 5
b (2r + 20) cm –1
c 4πr2 cm2 –2
d (4πr2 + 80πr + 400π) cm2
e 80π(r + 5) cm2
66 180°
f 1.131 m2
67 For example: take the two numbers 48 and 60.
g 30 cm
Their HCF is 12.
57 19.85 m
Difference = 60 − 48 = 12
58 a xm 1m 60 ÷ 12 = 5 and 48 ÷ 12 = 4
The two numbers are exactly divisible by 12.
xm This theorem says, then, that the HCF of 48 and 60 is 12, which
is the case.
68 The factors of a number are generally written in pairs, producing
an even number of factors. With a perfect square, one of these
factors will be paired with itself, producing an odd number of
3m factors. This occurs for all perfect squares.
69 15.9 cm
70 The total number of tiles needed for r rows is r2.
b (x + 1) (x + 3) 71    a Let the cost of an apple be a cents, and the cost of a banana be
c 5 m by 3 m b cents.
59 a 8m 6a + 4b = 700 [1]
b 57° 1a + 9b = 450 [2]
c 7.6 m Subtract [2] from [1].

Chapter 11 • Problem solving I 385


problem solving

5a − 5b = 250
Divide through by 5.
a − b = 50
b An apple costs 50c more than a banana.
72 a 1.25 mL b 1.47 mL
c Answers may vary. The two answers are only slightly
different. Because of an inability to measure to that degree
of accuracy in the home, they both provide a good guide to a
safe amount to administer.
d Approximately 1 and 10 years
73 Call the people A, B, C and D who take times of 1, 2, 5 and 10
minutes respectively.
A and B go over 2 min
B returns 2 min
C and D go over 10 min
A returns 1 min
A and B go over 2 min
Total time = 2 + 2 + 10 + 1 + 2 = 17 min
74 58 railings
75 The original number could have been 187, 781, 286, 682, 385,
583 or 484.
76    a There are 5 different shapes. b They all have a perimeter of 10 units, except for the last one
shown above, which has a perimeter of 8 units.
77 Check with your teacher.
1 1 1
78 = +
n n + 1 n(n + 1)
79 The claim appears to be true. Further investigation would need
to be conducted to determine if it worked in all cases — for
example, if the lines were parallel, at right angles, vertical or
horizontal.
80 6450 multi‐packs

386 Maths Quest 10 New South Wales Australian curriculum edition Stages 5.1 and 5.2
STATistics and probability

Chapter 12

Probability
WHY LEARN THIS?
Probability is the mathematics of chance. Our entire
lives are affected by events that are based on chance.
Learning about probability will help you understand
how chance is involved in everyday events and in many
decisions that you will make in your life.

WHAT DO YOU KNOW?


1 THINK List what you know about chance.
Create a concept map to show your list
2 pair Share what you know with a partner
and then with a small group
3 share As a class, create a large concept map
that shows your class’s knowledge of chance.

LEARNING SEQUENCE
12A Review of probability
12B  Tree diagrams
12C  Independent and dependent events
12D   Conditional probability
12E  Subjective probability
Watch this video
The story of mathematics

Searchlight ID: eles-1851


STATistics and probability

12A Review of probability


The language of probability
•• Probability measures the chance of an event taking place and ranges from zero (0)
for an impossible event to one (1) for a certain event.
Interactivity
Chances decrease
Random number
­generator
int-0089 Highly Unlikely Even Likely Highly
unlikely chance likely
Impossible Very Less than Better than Very Certain
unlikely even chance even chance likely

0 0.1 0.2 0.3 0.4 0.5 0.6 0.7 0.8 0.9 1


0% 50% 100%

Chances increase

•• The experimental probability of an event is based on the outcomes of experiments,


simulations or surveys.
•• A trial is a single experiment; for example, a single flip of a coin.
number of successful trials
Experimental probability =
total number of trials
•• The relative frequency of an event is the same as the experimental probability of
that event.
•• The list of all possible outcomes of an experiment is known as the event space or
sample space. For example, when flipping a coin there are two possible outcomes:
Heads or Tails. The event space can be written, using set notation, as 5 H, T 6 .

WORKED EXAMPLE 1
The spinner shown here is made up of 4 equal-sized
segments. It is known that the probability that the spinner II I
will land on any one of the four segments from one spin is 14 .
To test if the spinner shown here is fair, a student spun the III IV
spinner 20 times and each time recorded the segment in
which the spinner stopped. The spinner landed as follows.

Segment I II III IV
Tally 5 4 8 3

a List the event space.


b Given the experimental results, determine the relative frequency for
each segment.
c Compare the results from the experiment with the known probabilities
and suggest how the experiment could be changed to ensure that the
results give a better estimate of the true probability.

390 Maths Quest 10 New South Wales Australian curriculum edition Stages 5.1 and 5.2
STATistics and probability

T HIN K W RI T E
a The event space lists all a Event space = 5 I, II, III, IV 6
possible outcomes from
one spin of the spinner. There
are four possible outcomes.

b 1 For segment I there b number of successful trials


Relative frequencyI =
were 5 successful trials total number of trials
out of the 20. Substitute 5
=
these values into the 20
relative frequency = 0.25
formula.

2 Repeat for segments: 4


Relative frequencyII =
•• II (4 successes) 20
•• III (8 successes) = 0.2
•• IV (3 successes). 8
Relative frequencyIII =
20
= 0.4
3
Relative frequencyIV =
20
= 0.15

c Compare the relative c The relative frequency of segment I was the only segment that
frequency values with the mirrored the known value. To ensure that relative frequency
known value of 14 (0.25). gives a better estimate of the true probability, the spinner
Answer the question. should be spun many more times.

Two-way tables
•• The sample space can be displayed using a two-way table.
•• A two-way table represents two of the outcomes of events in a two-dimensional
table. A two-way table for the experiment of tossing a coin and rolling a die
simultaneously is shown below.

Die outcomes
Coin outcomes 1 2 3 4 5 6
H H, 1 H, 2 H, 3 H, 4 H, 5 H, 6
T T, 1 T, 2 T, 3 T, 4 T, 5 T, 6

Chapter 12 • Probability 391


STATistics and probability

WORKED EXAMPLE 2
Two dice are rolled, and the values on the two uppermost faces are
multiplied together. Draw a diagram to illustrate the sample space.
T HIN K W RI T E
The sample space for rolling 1 die is FIRST DIE
5 1, 2, 3, 4, 5, 6 6 . When two dice are
rolled and the two uppermost faces are × 1 2 3 4 5 6
multiplied, the sample space is made up of 1 1 2 3 4 5 6

SECOND DIE
36 products. This is best represented with 2 2 4 6 8 10 12
the use of a two-way table.
•• Draw a 7 × 7 grid. 3 3 6 9 12 15 18
•• In the first row and column list the 4 4 8 12 16 20 24
outcomes of each die. 5 5 10 15 20 25 30
•• At the intersection of a column and row
write the product of the relevant die 6 6 12 18 24 30 36
outcomes, as shown in green.

Theoretical probability
•• Theoretical probability is the probability of an event occurring, based on the
number of possible favourable outcomes, n(A), and the total number of possible
outcomes, n(ξ).
•• When all outcomes are equally likely, the theoretical probability of an event can be
calculated using the formula:
number of favourable outcomes n(E)
P(event) = or P(event) =
total number of possible outcomes n(ξ)
where n(E) is the number of favourable events and n(ξ) is the total number of
possible outcomes.
WORKED EXAMPLE 3
A fair die is rolled and the value of the uppermost side is recorded.
Calculate the theoretical probability that a 4 is uppermost.
T HIN K W RI T E
1 Write the number of favourable n(E) = 1
outcomes and the total number of n(ξ) = 6
possible outcomes. The number of
fours on a fair die is 1. There are 6
possible outcomes.
2 Substitute the values found in part 1 to n(E)
P(a four) =
calculate the probability of the event n(ξ)
that a four is uppermost when a die is 1
=
rolled. 6
3 Write the answer. The probability that a 4 is uppermost
when a fair die is rolled is 16.

392 Maths Quest 10 New South Wales Australian curriculum edition Stages 5.1 and 5.2
STATistics and probability

Complementary events
•• The complement of the set A is the set of all ξ
A
elements that belong to the universal set (ξ)
but which do not belong to A. For example, the A′
complement of {blue socks in a drawer} is
{all socks in the drawer that are not blue}.
•• The complement of A is written as A′ and is read
as ‘A dashed’ or ‘A prime’.
•• On a Venn diagram, complementary events appear as separate regions that together
occupy the whole universal set.
Since n(A) + n(A′) = n(ξ)
P(A) + P(A′) = 1

WORKED EXAMPLE 4
A player is chosen from a cricket team. Are the events ‘selecting a
batsman’ and ‘selecting a bowler’ complementary events if a player can
have more then one role? Give a reason for your answer.

T HIN K W RI T E
Explain the composition of a cricket No, the events ‘selecting a batsman’
team. Players who can bat and bowl and ‘selecting a bowler’ are not
are not necessarily the only players in a complementary events. These events
cricket team. There is a wicket-keeper as may have common elements; that is, the
well. Some players (all rounders) can bat all rounders in the team who can bat and
and bowl. bowl. The cricket team also includes a
wicket-keeper.

Mutually exclusive events (A or B)


•• Two events are mutually exclusive if one event happening excludes the other from
happening. These events may not encompass all possible events. For example, when
selecting a card from a deck of cards, selecting a black card excludes the possibility
that the card is a Heart.
•• For events that are mutually exclusive: P(A ∩ B) = 0.
•• On a Venn diagram, mutually exclusive events appear as ξ
disjointed sets within the universal set. A B
A and B are mutually exclusive events.
P(A ∪ B) = P(A) + P(B)

Chapter 12 • Probability 393


STATistics and probability

Probability of intersecting events (A or B or both)


•• When two events have outcomes in common, ξ
P(A ∪ B) ≠ P(A) + P(B), since this would A B
count the outcomes they have in common twice.
P(A ∪ B) = P(A) + P(B) − P(A ∩ B)
This is known as the Addition Law of probability.

WORKED EXAMPLE 5
A card is drawn from a pack of 52 playing cards. What is the probability
that the card is a heart or a club?
T HIN K W RI T E
1 Determine whether the given The two events are mutually exclusive as
events are mutually exclusive. they have no common elements.
13 13
2 Determine the probability of P(heart) = 52
P(club) = 52
drawing a heart and of drawing 1 1
= 4
= 4
a club.
3 Write the Addition Law for two P(A or B) = P(A) + P(B)
mutually exclusive events. where A = drawing a heart
and B = drawing a club

4 Substitute the known values into P(heart or club) = P(heart) + P(club)


the rule. = 1
+ 14
4
2
= 4

5 Evaluate and simplify. = 12

6 Write your answer. The probability of drawing a heart or a


club is 12.

Note: Alternatively, we can use the n(heart or club)


P(heart or club) =
formula for theoretical probability. n(ξ)
26
= 52
1
= 2

WORKED EXAMPLE 6
A die is rolled. Determine:
a P(an odd number)
b P(a number less than 4)
c P(an odd number or a number less than 4).

394 Maths Quest 10 New South Wales Australian curriculum edition Stages 5.1 and 5.2
STATistics and probability

T HIN K W RI T E
3
a 1 Determine the probability a P(odd) =
6
of obtaining an odd number; 1
that is, 5 1, 3, 5 6 . = 2

2 Write your answer. The probability of obtaining an odd


number is 12.
3
b 1 Determine the probability b P(less than 4) =
6
of obtaining a number less 1
than 4; that is, 5 1, 2, 3 6 . = 2

2 Write your answer. The probability of obtaining a


number less than 4 is 12.
c 1 Determine whether the c The two events are not mutually
given events are mutually exclusive as they have common
exclusive. elements; that is, 1 and 3.
2 Write the Addition Law for P(A or B) = P(A) + P 1 B 2 − P 1 A and B 2
two non-mutually exclusive where A = selecting an odd number and
events. B = selecting a number less than 4.
3 Substitute the known P[odd number ∪ (number < 4]
values into the rule. = P(odd number) + P[(number < 4]
Note: P(A and B) = 26 Q = 13 R − P[odd number ∩ (number < 4) ]
since the events have two = 12 + 12 − 13
elements in common.
4 Evaluate and simplify. = 23
5 Write your answer. The probability of obtaining an
odd number or a number less than 4
is 23.

WORKED EXAMPLE 7
Given P(A) = 0.6, P(B) = 0.4 and P(A ∪ B) = 0.9:
a use the Addition Law of probability to calculate the value of P(A ∩ B)
b draw a Venn diagram to represent the universal set
c calculate P(A ∩ B′).

T HIN K W RI T E
a 1 Write the Addition Law of a P(A ∪ B) = P(A) + P(B) − P(A ∩ B)
probability and substitute 0.9 = 0.6 + 0.4 − P(A ∩ B)
given values.

Chapter 12 • Probability 395


STATistics and probability

2 Collect like terms and 0.9 = 0.6 + 0.4 − P(A ∩ B)


rearrange to make 0.9 = 1.0 − P(A ∩ B)
P(A ∩ B) the subject. ∩
P(A B) = 1.0 − 0.9
Solve the equation. = 0.1
b 1 Draw intersecting sets b ξ
A and B within the A B
universal set and write 0.1
P(A ∩ B) = 0.1 inside the
overlapping section, as
shown in blue.
2 •• As P(A) = 0.6, 0.1 of this ξ
belongs in the overlap, A B
the remainder of set A is 0.5 0.1 0.3
0.5 (0.6 − 0.1).
•• Since P(B) = 0.4, 0.1
of this belongs in the
overlap, the remainder of
set B is 0.3 (0.4 − 0.1).
3 The total probability for ξ
events A and B is 1. That A B
means P(A ∪ B)′ = 0.1. 0.5 0.1 0.3
Write 0.1 outside sets
A and B to form the 0.1
remainder of the
universal set.
c P(A ∩ B′) is the overlapping c ξ
region of P(A) and P(B′). A B
Shade the region and write 0.5 0.1 0.3
down the corresponding
probability value for this area. 0.1

P(A ∩ B′) = 0.5

WORKED EXAMPLE 8
a Draw a Venn diagram representing the relationship between the
following sets. Show the position of all the elements in the Venn diagram.
ξ = {1, 2, 3, 4, 5, 6, 7, 8, 9, 10, 11, 12, 13, 14, 15, 16, 17, 18, 19, 20}
A = {3, 6, 9, 12, 15, 18}
B = {2, 4, 6, 8, 10, 12, 14, 16, 18, 20}
b Determine:
i P(A) ii P(B) iii P(A ∩ B)
iv P(A ∪ B) v P(A′ ∩ B′)

396 Maths Quest 10 New South Wales Australian curriculum edition Stages 5.1 and 5.2
STATistics and probability

T HIN K W RI T E /DR AW
a 1 Draw a rectangle with two partly a n(ξ ) = 20
intersecting circles labelled A and B.
A B
2 Analyse sets A and B and place any
common elements in the central overlap. 6 2 4
3 9
3 Place the remaining elements of set A 12 8 10
15
in circle A. 18 14 16
20
4 Place the remaining elements of set B
11 13
in circle B.
1 5 7 17 19
5 Place the remaining elements of the
universal set ξ in the rectangle.
b i 1 Write the number of elements b i n(A) = 6, n(ξ) = 20
that belong to set A and the total
number of elements.
n(A)
2 Write the rule for probability. P(A) =
n(ξ)
6
3 Substitute the known values into P(A) = 20
the rule.
3
4 Evaluate and simplify. = 10
i i 1 Write the number of elements i i n(B) = 10, n(ξ) = 20
that belong to set B and the total
number of elements.
n(B)
2 Repeat steps 2 to 4 of part b i. P(B) =
n(ξ)
10
P(B) = 20
1
= 2
i i i 1 Write the number of elements i i i n(A ∩ B) = 3, n(ξ) = 20
that belong to set A ∩ B and the
total number of elements.
n(A ∩ B)
2 Repeat steps 2 to 4 of part b i. P(A ∩ B) =
n(ξ)
P(A ∩ B) = 3
20
i v 1 Write the number of elements i v n(A ∪ B) = 13, n(ξ) = 20
that belong to set A ∪ B and the
total number of elements.
n(A ∪ B)
2 Repeat steps 2 to 4 of part b i. P(A ∪ B) =
n(ξ)
P(A ∪ B) = 13
20
v 1 Write the number of elements that v n(A′ ∩ B′) = 7, n(ξ) = 20
belong to set A′ ∩ B′ and the total
number of elements.
n(A′ ∩ B′)
2 Repeat steps 2 to 4 of part b i. P(A′ ∩ B′) =
n(ξ)
P(A′ ∩ B′) = 7
20

Chapter 12 • Probability 397


STATistics and probability

WORKED EXAMPLE 9
In a class of 35 students, 6 students like all three subjects: PE, Science and Music. Eight of
the students like PE and Science, 10 students like PE and Music, and 12 students like Science
and Music. Also, 22 students like PE only, 18 students like Science only and 17 like Music
only. Two students don’t like any of the subjects.
a Display this information on a Venn diagram.
b Determine the probability of selecting a student who:
i likes PE only
ii does not like Music.
c Find P[(Science ∪ Music) ∩ PE′] .

T HIN K W RI T E /DR AW
a 1 Draw a rectangle with three partly a n(ξ) = 35
intersecting circles, labelled PE, PE Science
Science and Music.

Music

2 Extract the information relating to n(ξ) = 35


students liking all three subjects. PE Science
Note: The central overlap is the
key to solving these problems. Six
students like all three subjects, so 6
place the number 6 into the section
corresponding to the intersection of
the three circles.
Music

3 Extract the relevant information n(ξ) = 35


from the second sentence and place PE Science
it into the appropriate position.
Note: Eight students like PE and 2
Science; however, 6 of these students 6
have already been accounted for 4 6
in step 2. Therefore, 2 will fill
the intersection of only PE and
Music
Science. Similarly, 4 of the 10
who like PE and Music will fill the
intersection of only PE and Music,
and 6 of the 12 students will fill
the intersection of only Science
and Music.

398 Maths Quest 10 New South Wales Australian curriculum edition Stages 5.1 and 5.2
STATistics and probability

4 Extract the relevant information from n(ξ) = 35


the third sentence and place it into PE Science
the appropriate position.
Note: Twenty-two students like 10 2 4
PE and 12 have already been 6
accounted for in the set. Therefore, 4 6
10 students are needed to fill the circle 1
corresponding to PE only. Similarly,
Music
4 students are needed to fill the circle
corresponding to Science only to
make a total of 18 for Science. One
student is needed to fill the circle
corresponding to Music only to make
a total of 17 for Music.
5 Extract the relevant information from n(ξ) = 35
the final sentence and place it into the PE Science
appropriate position.
Note: Two students do not like any of 10 2 4
the subjects, so they are placed in the 6
rectangle outside the three circles. 4 6
Check that the total number 1
6 in all positions is equal to the
Music 2
number in the universal set.
10 + 2 + 4 + 4 + 6 + 6 + 1 + 2 = 35
b i 1 Write the number of students who b i n(students who like PE only) = 10
like PE only and the total number of n(ξ) = 35
students in the class.
n(likes PE only)
2 Write the rule for probability. P(likes PE only) =
n(ξ)

3 Substitute the known values into the P(likes PE only) = 10


35
rule.
4 Evaluate and simplify. = 27
5 Write your answer. The probability of selecting a student who likes
PE only is 27.
i i 1 Write the number of students who do i i n(students who do not like Music) = 18
not like Music and the total number n(ξ) = 35
of students in the class.
Note: Add all the values that do not
appear in the Music circle as well
as the two that sit in the rectangle
outside the circles.
n(does not like Music)
2 Write the rule for probability. P(does not like Music) =
n(ξ)

Chapter 12 • Probability 399


STATistics and probability

3 Substitute the known values into P(does not like Music) = 18


35
the rule.
4 Write your answer. The probability of selecting a student
who does not like Music is 18
35
.
c 1 Write the number of students who c n[(Science ∪ Music) ∩ PE′] = 11
like Science and Music but not PE. n(ξ) = 35
Note: Add the values that appear in
the Science and Music circles but do
not overlap with the PE circle.
2 Repeat steps 2 to 4 of part b ii. P[(Science ∪ Music) ∩ PE′]
n[(Science ∪ Music) ∩ PE′]
=
n(ξ)
P[(Science ∪ Music) ∩ PE′] = 11 35
.
The probability of selecting a student who
likes Science or Music but not PE is 1135
.

Odds
•• Probabilities in gambling can be expressed as odds.
•• Odds are common in racing, where they are given as ratios; for example 5−1
(or 51 or 5 : 1).
•• In the odds of a–b,
a−b

a is the chance b is the chance for


against the event the event
•• If the odds for a horse to win are given as 5–1, then from 6 races the horse is
expected to lose 5 and win 1. The probability that the horse wins or loses can be
calculated from the odds given. These calculations are shown below.
n(expected wins) n(expected losses)
P(win) = P(lose) =
n(races) n(races)
1 5
=6 =6
•• If given odds of a–b, then:
b
–– P(win) =
a+b
a
–– P(loss) =
a+b

Payouts
•• The payout in races is calculated on the odds given.
a
•• If the odds are a–b, you can win $ for every $1 bet, or, $a for every $b bet. The
b
bookmaker will pay out your win plus the initial amount wagered.

400 Maths Quest 10 New South Wales Australian curriculum edition Stages 5.1 and 5.2
STATistics and probability

•• The TAB quotes a whole payout figure for a horse, made up of the winnings and the
initial wager.
For example:

Odds Bet Winnings Payout figure

5–1 $10 $5 for every $1 bet: 51 × $10 = $50 $60 ($50 + $10)

7–2 $14 $7 for every $2 bet: 72 × $14 = $49 $63 ($49 + $14)

WORKED EXAMPLE 10
The odds given for the horse Gunnawin to win the
Melbourne Cup are 9–4.
a Determine the probability of Gunnawin winning
the Melbourne Cup.
b Tony decides to bet $12 on Gunnawin to win. If the
horse does win, what is Tony’s payout?
c In the same race, the probability that the horse
5
‘Can’t Lose’ wins is given as 17 . What are the odds
that this horse will win?
T HIN K W RI T E
a 1 Write the number of ways Gunnawin a n(Gunnawin wins) = 4
can win (4) and the total number of n(ξ) = 13
outcomes (9 + 4 = 13).
n(Gunnawin wins)
2 Write the rule for probability. P(Gunnawin wins) =
n(ξ)
4
3 Substitute the known values into the rule. P(Gunnawin wins) = 13
4 Write your answer. The probability of Gunnawin winning the
4
Melbourne Cup is 13 .
9
b 1 Explain what the ratio means and relate b In the odds 9–4 the punter can win $4 for every
it to the bet. $1 that is bet (or for every $4 bet the punter will
win $9). Therefore, if Tony bets $12 he will win
9
4
× $12 = $27.
2 Add the original amount invested to the Payout = $27 + $12
amount returned. = $39
3 Write your answer. Tony’s payout will be $39.
c 1 Look at the given fraction. The c This horse has been given the chance of winning
5
numerator corresponds to the ‘win’ as 17 . Therefore its chance of losing is 12
17
.
component (second number) of the ratio.
2 The lose component of the ratio is Therefore the lose–win ratio is 12–5.
always the first number.
3 Write your answer. The odds of Can’t Lose winning the Melbourne
Cup are 12–5.

Chapter 12 • Probability 401


STATistics and probability

Exercise 12A Review of probability


INDIVIDUAL PATHWAYS
Questions: Questions: Questions:
1–8, 10, 12, 14, 16, 18, 20, 22 1, 3–6, 8, 9, 11, 13, 15, 17, 1, 2, 4, 5, 7, 9–13, 15, 17, 19–22
19, 21, 22
Activity 12-A-1 Activity 12-A-2 Activity 12-A-3
Review of probability General probability problems Tricky probability problems
doc-5110 doc-5111 doc-5112

FLUENCY
1 Explain the difference between experimental and theoretical probability.
2 WE1 The spinner shown at right was spun 50 times and the
II
Digital docs outcome each time was recorded in the table below. III I
SkillSHEET
Set notation
doc-5286 Segment I II III IV V VI IV VI
SkillSHEET V
Simplifying fractions Tally 10 6 8 7 12 7
doc-5287
SkillSHEET a List the event space.
Determining
complementary events b Given the experimental results, determine the relative frequency for each segment.
doc-5288
SkillSHEET c The theoretical probability of the spinner landing on any particular segment with
Addition and subtraction one spin is 16. How could the experiment be changed to give a better estimate of
of fractions
doc-5289 the true probabilities?
SkillSHEET
Working with Venn 3 A laptop company conducted a survey to see what were the most appealing colours
diagrams for laptop computers among 15–18-year-old students. The results were as follows:
doc-5291
SkillSHEET
Writing odds as Colour Black Sizzling Power Blazing Gooey Glamour
probabilities
doc-5292
Black Silver Pink Blue Green Gold
SkillSHEET
Writing probabilities
Number 102 80 52 140 56 70
as odds
doc-5293 a How many students were surveyed?
SkillSHEET
Distinguishing between b What is the relative frequency of students who found silver the most appealing
complementary and laptop colour?
mutually exclusive
events c What is the relative frequency of students who found black and green to be their
doc-5294 most appealing colours?
d Which colour was found to be most appealing?
4 WE2 Two dice are rolled and the values on the two uppermost faces are added
together.
a Construct a table to illustrate the sample space.
b What is the most likely outcome?
c What is the least likely outcome?
5 WE7 Given P(A) = 0.5, P(B) = 0.4 and P(A ∪ B) = 0.8:
a use the Addition Law of probability to calculate the value of P(A ∩ B)
b draw a Venn diagram to represent the universal set
c calculate P(A ∩ B′).

402 Maths Quest 10 New South Wales Australian curriculum edition Stages 5.1 and 5.2
STATistics and probability

6 Let P(A) = 0.25, P(B) = 0.65 and P(A ∩ B) = 0.05.


a Calculate:
i P(A ∪ B) ii P(A ∩ B)′

b MC Which Venn diagram below best illustrates P(A ∩ B)′?


A ξ B ξ
A B A B

C ξ D ξ
A B A B

E ξ
A B

7 WE4 A die is rolled. What is the probability that the outcome is an even number
or a 5?
8 WE5 A card is drawn from a well-shuffled pack of 52 playing cards. Calculate:
a P(a king is drawn)
b P(a heart is drawn)
c P(a king or a heart is drawn).
9 WE6 For each of the following pairs of events:
i state, giving justification, if the pair are complementary events
ii alter the statements, where applicable, so that the events become complementary
events.
a Having Weet Bix or having Strawberry Pops for breakfast
b Walking to a friend’s place or driving there
c Watching TV or reading as a leisure activity
d Rolling a number less than 5 or rolling a number greater than 5 with a
ten-sided die with faces numbered 1 to 10
e Passing a maths test or failing a maths test
10 a WE8 Draw a Venn diagram representing the relationship between the following
sets. Show the position of all the elements in the Venn diagram.
ξ = 5 1, 2, 3, 4, 5, 6, 7, 8, 9, 10, 11, 12, 13, 14, 15, 16, 17, 18, 19, 20 6
A = 5 1, 3, 5, 7, 9, 11, 13, 15, 17, 19 6
B = 5 1, 4, 9, 16 6
b Calculate:
i P1A2 ii P 1 B 2 iii P(A ∩ B) iv P(A ∪ B) v P(A′ ∩ B′).

Chapter 12 • Probability 403


STATistics and probability

11 Write the following odds as probabilities.


a 5–1 b 13–4 c 7–1
2
12 MC The probability of 9 written as odds is:
A 7–2 B 2–7 C 2–9 D 9–2 E 11–9

UNDERSTANDING
13 You and a friend are playing a dice game. You have an eight-sided die (with
faces numbered 1 to 8 inclusive) and your friend has a six-sided die (with faces
numbered 1 to 6 inclusive). You each roll your own die.
a The person who rolls the number 4 wins. Is this game fair?
b The person who rolls an odd number wins. Is this game fair?
14 A six-sided die has three faces numbered 5; the other faces are numbered 6. Are the
events ‘rolling a 5’ and ‘rolling a 6’ equally likely?
15 90 students were asked which lunchtime sports on offer, of basketball, netball
and soccer, they had participated in, on at least one occasion in the last week. The
results are shown below.

Sport Basketball Netball Soccer Basketball Basketball Netball All


and and soccer and three
netball soccer
Number of
35 25 39 5 12 8 3
students

a Copy and complete the Venn diagram shown below to illustrate the sample
space.

ξ B N

15
3
8

b How many students did not play basketball, netball or soccer at lunchtime?
c How many students played basketball and/or netball but not soccer?
d How many students are represented by the region
(basketball ∩ not netball ∩ soccer)?
e Calculate the relative frequency of the region described in part d above.
f Estimate the probability that a student will play 3 of the sports offered.
16 WE5 A card is drawn from a shuffled pack of 52 cards. Find the probability that
the card drawn is:
a an ace b a club c a red card
d not a jack e a green card f not a red card.

404 Maths Quest 10 New South Wales Australian curriculum edition Stages 5.1 and 5.2
STATistics and probability

17 A bag contains 4 blue marbles, 7 red marbles and 9 yellow marbles. All marbles are
of the same size. A marble is selected at random. What is the probability that the
marble is:
a blue b red c not yellow d black?
18 WE9 Thirty students were asked which lunchtime sports they enjoyed —
volleyball, soccer or tennis. Five students chose all three sports. Six students chose
volleyball and soccer, 7 students chose volleyball and tennis while 9 chose soccer
and tennis. Fifteen students chose volleyball, 14 students chose soccer and
18 students chose tennis.
a Copy the Venn diagram shown and enter the given information.
n(ξ) = 30
Volleyball Soccer

Tennis

b If a student is selected at random, determine the probability of selecting a


student who:
i chose volleyball
ii chose all three sports
iii chose both volleyball and soccer but not tennis
iv did not choose tennis
v chose soccer.
c Determine:
i P[(soccer ∪ tennis) ∩ volleyball′]
ii P[(volleyball ∪ tennis) ∩ soccer′] .
19 WE10 The odds given for the greyhound
‘Dog’s Breakfast’ to win its race are 7–3.
a Determine the probability of Dog’s Breakfast
winning its race.
b Maria decides to bet $15 on Dog’s Breakfast to
win the race. If Dog’s Breakfast wins, calculate
Maria’s payout.
c The dog ‘Zoom Top’ is also in the race. If the
4
probability of Zoom Top winning is 13 , what
odds should be given for Zoom Top?
20 A six-sided die has three faces numbered 1 and the
other three faces numbered 2. Are the events ‘rolling a REFLECTION
1’ and ‘rolling a 2’ equally likely? What basic formu la
21 Are the odds 10–6 the same as 5–3? Explain. must be remembered in
order to calculate simple Digital doc
22 With the use of diagrams, show that probabilities?
WorkSHEET 12.1

P(A′ ∩ B′) = P(A ∪ B)′.


doc-5295

Chapter 12 • Probability 405


STATistics and probability

CHALLENGE 12.1

12B  Tree diagrams


Two-step chance experiments
•• In two-step chance experiments the result is obtained after performing two trials.
Two-step chance experiments are often represented using tree diagrams.
eLesson Games at
Wimbledon •• Tree diagrams are used to list all possible outcomes of two or more events that are
eles-1032 not necessarily equally likely.
•• The probability of obtaining the result for a particular event is listed on the
branches.
•• The probability for each outcome in the sample space is the product of the
probabilities associated with the respective branches. For example, the tree diagram
shown here represents the sample space for flipping a coin, then choosing a marble
from a bag containing three red marbles and one black marble.

Coin Marble Outcomes Probability


toss pick

R HR P(HR) = 1–2 × 3–4 = 3–8


3–
4

H
1–
1– 4
2
B HB P(HB) = 1–2 × 1–4 = 1–8

1– R TR P(TR) = 1–2 × 3–4 = 3–8


2
3–
4
T
1–
4
B TB P(TB) = 1–2 × 1–4 = 1–8

4 possible
outcomes

406 Maths Quest 10 New South Wales Australian curriculum edition Stages 5.1 and 5.2
STATistics and probability

•• When added together, all the probabilities for the outcomes should sum to 1. They
are complementary events. For example,
P(HR) + P(HB) + P(TR) + P(TB) = 38 + 18 + 38 + 18
=1
•• Other probabilities can also be calculated from the tree diagram. For example,
the probability of getting an outcome that contains a red marble can be calculated
by summing the probabilities of each of the possible outcomes that include a
red marble. Outcomes that contain a red marble are HR and TR, therefore:
P(red marble) = P(HR) + P(TR).
= 38 + 38
= 68
= 34

WORKED EXAMPLE 11
A three-sided die is rolled and a name is picked out of a hat that contains 3 girls’ names and
7 boys’ names.
a Use a tree diagram to display the sample space.
b Calculate the probability of:
i rolling a 3, then choosing a boy’s name
ii choosing a boy’s name after rolling an odd number.

THINK WRITE
a 1 Draw 3 branches from the starting point to a Die Name Outcomes
show the 3 possible outcomes of rolling a
G 1G
three-sided die (shown in blue), and then 3
10
draw 2 branches off each of these to show 1 7
10
the 2 possible outcomes of choosing a 1– B 1B
3
name out of a hat (shown in red).
G 2G
2 Write probabilities on the branches to 3
1– 10
show the individual probabilities of 3 2 7
10
rolling a 1, 2 or 3 on a three-sided die. B 2B
As these are equally likely outcomes,
1–
P(1) = P(2) = P(3) = 13. 3
3
G 3G
10
3
3 Write probabilities on the branches to show 7
10
the individual probabilities of choosing a B 3B
name. Since there are 3 girls’ names and
3
7 boys’ names in the hat, P(G) = 10 and
7
P(B) = 10.

b i 1 Follow the pathway of rolling a 3 b i P(3B) = P(3) × P(B)


1
S P(3) = 3
T and choosing a boy’s = 1 1
× 10
3
7
name S P(B) = 10 T , and multiply the 7
=
probabilities. 30

Chapter 12 • Probability 407


STATistics and probability

2 Write the answer. The probability of rolling a 3, then choosing


7
a boy’s name is 10 .
ii 1 To roll an odd number (1 or 3) then i i P(odd B) = P(1B) + P(3B)
choose a boy’s name: = P(1) × P(B) + P(3) × P(B)
•• roll a 1, then choose a boy’s name or 1 7
•• roll a 3, then choose a boy’s name. = 3
× 10 + 13 × 7
10
7 7
Find the probability of each of these = 30
+ 30
and add them together to find the total 14
=
probability. 30
7
= 15
2 Write the answer. The probability of choosing a boy’s name
7
after rolling an odd number is 15 .

Three–step chance experiments


•• Outcomes are often made up of combinations of events. For example, when a coin is
flipped three times, three of the possible outcomes are HHT, HTH and THH. These
outcomes all contain 2 Heads and 1 Tail.
•• The probability of an outcome with a particular order is written such that the order
required is shown. For example, P(HHT) is the probability of H on the first die,
H on the second die and T on the third die.
•• The probability of an outcome with a particular combination of events in which the
order is not important is written describing the particular combination required. For
example, P(2 Heads and 1 Tail).

WORKED EXAMPLE 12
A coin is biased so that the chance of it falling as a Head when flipped is 0.75.
a Draw a tree diagram to represent the coin being flipped three times.
b Calculate the following probabilities:
i P(HTT) ii P(1H and 2T) iii P(at least 2 Tails).
T HIN K WRITE
a 1 Tossing a coin has two outcomes. a 1st 2nd 3rd Outcomes
Draw 2 branches from the starting toss toss toss
point to show the first toss, 2 0.75
H HHH
branches off each of these to show H
0.25
the second toss and then 2 branches 0.75 T HHT
H
off each of these to show the 0.25 0.75
H HTH
third toss. 0.75 T 0.25
2 Write probabilities on the branches T HTT
to show the individual probabilities H THH
of tossing a Head (0.75) and a 0.25 H
0.75
0.25
Tail. Because tossing a Head and 0.75 T THT
tossing a Tail are mutually exclusive, T
0.25 H TTH
P(T) = 1 − P(H) = 0.25. T
0.75
0.25
T TTT

408 Maths Quest 10 New South Wales Australian curriculum edition Stages 5.1 and 5.2
STATistics and probability

b i •• P(HTT) implies the order: H (0.75), b i P(HTT) = P(H) × P(T) × P(T)


T (0.25), T (0.25). = (0.75) × (0.25) 2
•• Multiply the probabilities. = 0.047
i i •• P(1H and 2T) implies: P(HTT), i i P(1H and 2T)
P(THT), P(TTH). = P(HTT) + P(THT) + P(TTH)
•• Add these probabilities. = 3(0.75 × 0.252)
= 0.141
i i i •• P(at least 2 Tails) implies: P(HTT), i i i P(at least 2T)
P(THT), P(TTH) and P(TTT). = P(HTT) + P(THT) + P(TTH) + P(TTT)
•• Add these probabilities. = 3(0.75 × 0.252) + 0.253
= 0.156

Exercise 12B Tree diagrams


INDIVIDUAL PATHWAYS
Questions: Questions: Questions:
1–7, 9 1, 3–6, 8, 9 1–10
Activity 12-B-1 Activity 12-B-2 Activity 12-B-3
Review of tree Practice with tree Tricky tree diagrams
diagrams diagrams doc-13765
doc-13763 doc-13764

FLUENCY
1 Explain how a tree diagram can be used to calculate probabilities of events that are
not equally likely.
2 Use this tree diagram to answer the following questions:
Fish
1
10
1–
Blue 5 Donkey
7
10
Elephant
1– Fish
4
1
10
1–
Red 5 Donkey
3 7
20 10
Elephant
Fish
1 1
10 10
Green 1–
5 Donkey
7
1– 10
2 Elephant
Fish
1
10
1–
Indigo 5 Donkey
7
10
Elephant

Chapter 12 • Probability 409


STATistics and probability

aHow many different outcomes are there?


bAre all outcomes equally likely? Explain.
cIs getting a red fish more, less or equally likely than getting a green elephant?
dWhat is the most likely outcome?
eCalculate the following probabilities.
i P(blue elephant)
ii P(indigo elephant)
iii P(donkey)
Copy the tree diagram shown here and complete the labelling for tossing
3 a 
a biased coin three times when the chance of tossing one Head in one toss
is 0.7.

1st 2nd 3rd Outcome P(outcome)


toss toss toss
H
H
T
H
H
T
T
H
H
T
T
H
T
T

b What is the probability of tossing three Heads?


c What is the probability of getting at least one Tail?
d What is the probability of getting exactly two Tails?
4 The questions below relate to rolling a fair die.
a What is the probability of each of the following outcomes from one roll of a die?
i P(rolling number < 4)
ii P(rolling a 4)
iii P(rolling a number other than a 6)
1st roll 2nd roll
b The tree diagram shown at right has been condensed to
6
depict rolling a die twice, noting the number relative to 4
<4
on the first roll and 6 on the second. Complete a labelled
6′
tree diagram, showing probabilities on the branches and
all outcomes, similar to that shown. 6
c What is the probability of rolling the following with 4
2 rolls of the die? 6′

i P(a 4 then a 6) 6
ii P(a number less than 4 then a 6) >4
6′
iii P(a 4 then 6′)
iv P(a number > 4 and then a number < 6)

410 Maths Quest 10 New South Wales Australian curriculum edition Stages 5.1 and 5.2
STATistics and probability

5 WE11 The spinner shown at right is divided into 3 equal-sized


wedges labelled 1, 2 and 3. It is spun three times, and it is noted 3 1
whether the spinner lands on a prime number, P = {2, 3} = ‘prime’
or not a prime number, P = {1} = ‘not prime’. 2
a Construct a labelled tree diagram for 3 spins of the spinner,
showing probabilities on the branches and all possible outcomes.
b Find the following probabilities.
i P(3 prime numbers)
ii P(PPP′ in this order)
iii P(PPP′ in any order)

UNDERSTANDING
6 WE12 A coin is biased so that the chance of it falling as a Tail when
tossed is 0.2.
a Draw a tree diagram to represent the coin being tossed three times.
b What is the probability of getting the same outcome on each toss?
7 A die is tossed twice and each time it is recorded whether or not the number is a
multiple of 3. If M = the event of getting a multiple of 3 on any one toss and
M′ = the event of not getting a multiple of 3 on any one toss:
a draw a tree diagram to represent the 2 tosses
b what is the probability of getting two multiples of 3?
8 The biased spinner illustrated is spun three times.
a Draw a completely labelled tree diagram for 3 spins of the 1 2
spinner, showing probabilities on the branches and all possible
outcomes and associated probabilities. 3 1

b What is the probability of getting exactly 2 ones?


c What is the probability of getting at most 2 ones?

REASONING
9 A restaurant offers its customers a three-course dinner, where they choose between
two entrées, three main meals and two desserts. The managers find that 30%
choose soup and 70% choose prawn cocktail for the entrée, 20% choose vegetarian,
50% chicken and the rest have beef for their main meal, and 75% have sticky date
pudding while the rest have apple crumble for dessert.
a Draw a fully labelled tree diagram showing all possible choices.
b What is the probability that a customer will choose the soup, chicken and sticky
date pudding?
c If there are 210 people booked for the following week at the restaurant, how
many would you expect to have the meal combination referred to in question 2?
10 A bag contains 7 red and 3 white balls. A ball is taken at random, its colour noted
and it is then placed back in the bag before a second ball is chosen at random and
its colour noted.
a i Show the possible outcomes with a fully labelled tree diagram.
ii As the first ball was chosen, how many balls were in the bag?
iii As the second ball was chosen, how many balls were in the bag?

Chapter 12 • Probability 411


STATistics and probability

iv Does the probability of choosing a red or white ball change from the first
selection to the second? Explain.
v Calculate the probability of choosing a red ball twice.
b Suppose that after the first ball had been chosen it was not placed back in
the bag.
i As the second ball is chosen, how many balls are in the bag?
ii Does the probability of choosing a red or white
ball change from the first selection to the
second? Explain. REFLECTION
iii Construct a fully labelled tree diagram to show WHOLE CLASS
all possible outcomes. What strategies would you
Digital doc
use to remember how to
WorkSHEET 12.2 iv Calculate the probability of choosing two red construct tree diagrams?
doc-13775
balls.

12C  Independent and dependent events


Independent events
•• If a coin is tossed the outcome is a Head or a Tail. The outcome of the first toss does
not affect the outcome of the next toss of the coin. The second toss will still yield a
Interactivity Head or a Tail irrespective of the outcome of the first toss. Similarly, the outcome on
Independent and
dependent events the roll of a die will not affect the outcome of the next roll.
int-2787 •• If successive events have no effect on each other, they are called independent
events.
•• If events A and B are independent then the Multiplication Law of probability
states that:
P(A and B) = P(A) × P(B) or P(A ∩ B) = P(A) × P(B)
•• The reverse is also true. If:
P(A and B) = P(A) × P(B) or P(A ∩ B) = P(A) × P(B)
is true then event A and event B are independent events.

WORKED EXAMPLE 13
Adam is one of the 10 young golfers to represent his state. Paz is one of the 12 netball players
to represent her state. All the players in their respective teams have an equal chance of being
nominated as captains.
a Are the events ‘Adam is nominated as captain’ and ‘Paz is nominated as captain’
independent?
b Determine:
i P(Adam is nominated as captain)
ii P(Paz is nominated as captain).
c What is the probability that both Adam and Paz are nominated as captains of their
respective teams?

412 Maths Quest 10 New South Wales Australian curriculum edition Stages 5.1 and 5.2
STATistics and probability

T HIN K W RI T E
a Determine whether the given events a Adam’s nomination has nothing to do with Paz’s
are independent and write your nomination and vice versa. Therefore, the events are
answer. independent.
b i 1 Determine the probability b i P(Adam is nominated) = P(A)
of Adam being nominated n(Adam is nominated)
=
as captain. He is one of n(ξ)
1
10 players. P(Adam is nominated) = 10

2 Write your answer. The probability that Adam is nominated as


1
captain is 10 .

i i 1 Determine the probability i i P(Paz is nominated) = P(A)


of Paz being nominated n(Paz is nominated)
=
as captain. She is one of n(ξ)
1
12 players. P(Paz is nominated) = 12

2 Write your answer. The probability that Paz is nominated as


1
captain is 12 .

c 1 Write the Multiplication Law c P(A and P)


of probability for independent = P(A ∩ P)
events. = P(A) × P(P)

2 Substitute the known values P(Adam and Paz are nominated)


into the rule. = P(Adam is nominated) × P(Paz is nominated)
1 1
= 10 × 12

3 Evaluate. 1
= 120

4 Write your answer. The probability that both Adam and Paz are nominated
1
as captains is 120 .

Dependent events
•• Sometimes one event affects the outcome of another. For example, if a card is drawn
from a pack of playing cards, the probability that its suit is hearts, P(hearts), is 13 52
(or 14). If this card is not replaced, then this will affect the probability of subsequent
draws. The probability that the second card drawn is a heart will be 12 51
, while the
probability that the second card is not a heart will be 39 51
.
•• When one event affects the occurrence of another, the events are called dependent
events.
•• If two events are dependent, then the probability of occurrence of one event affects
that of the subsequent event.

Chapter 12 • Probability 413


STATistics and probability

WORKED EXAMPLE 14
A bag contains 5 blue, 6 green and 4 yellow marbles. The marbles are
identical in all respects except in their colours. Two marbles are picked
in succession without replacement. Determine the probability of picking
2 blue marbles.
T HIN K W RI T E /DR AW
n(B)
1 Determine the probability of P(picking a blue marble) =
picking the first blue marble. n(ξ)
5
P(picking a blue marble) = 15
1
= 3

n(B)
2 Determine the probability P(picking second blue marble) =
of picking the second blue n(ξ)
4
marble. P(picking second blue marble) = 14
Note: The two events are = 2
dependent since marbles are 7

not being replaced. Since we


have picked a blue marble
this leaves 4 blue marbles
remaining out of a total of
14 marbles.

3 Calculate the probability of P(2 blue marbles) = P(1st blue) × P(2nd blue)
obtaining 2 blue marbles. = 1
× 27
3
2
= 21

4 Write your answer. The probability of obtaining 2 blue


2
marbles is 21 .

Note: Alternatively, a tree —4


14
Blue
diagram could be used to solve
Blue
this question. —5
15
10
The probability of selecting —
14 Not blue
2 blue marbles successively can —5 Blue
14
be read directly from the first 10

15 Not blue
branch of the tree diagram. 9

14 Not blue

5 4
P(2 blue marbles) = 15
× 14
1
= 3
× 27
2
= 21

414 Maths Quest 10 New South Wales Australian curriculum edition Stages 5.1 and 5.2
STATistics and probability

Exercise 12C Independent and dependent events


INDIVIDUAL PATHWAYS
Questions: Questions: Questions:
1–7, 9, 11, 13 1, 3–6, 8, 9, 11, 12, 14 1–14
Activity 12-C-1 Activity 12-C-2 Activity 12-C-3
Simple independent and Independent and Tricky independent and
dependent events dependent events dependent events
doc-13766 doc-13767 doc-13768

FLUENCY
1 If A and B are independent events and P(A) = 0.7 and P(B) = 0.4, calculate:
a P(A and B)
Interactivity
b P(A′ and B) where A′ is the complement of A Random numbers
c P(A and B′) where B′ is the complement of B int-0085

d P(A′ and B′).


UNDERSTANDING
2 WE13 A die is rolled and a coin is tossed.
a Are the outcomes independent?
b Determine:
i P(Head) on the coin
ii P(6) on the die.
c Determine P(6 on the die and Head on the coin).
3 A tetrahedron (4-faced) die and a 10-sided die are rolled simultaneously. What is the
probability of getting a 3 on the tetrahedral die and an 8 on the 10-sided die?
4 A blue die and a green die are rolled. What is the probability of getting a 5 on the
blue die and not a 5 on the green die?
4
5 Dean is an archer. The experimental probability that Dean will hit the target is 5.

a What is the probability that Dean will hit the target on two successive attempts?
b What is the probability that Dean will hit the target on three successive attempts?
c What is the probability that Dean will not hit the target on two successive attempts?
d What is the probability that Dean will hit the target on the first attempt but miss
on the second attempt?

Chapter 12 • Probability 415


STATistics and probability

6 MC A bag contains 20 apples, of which 5 are bruised.


Peter picks an apple and realises that it is bruised. He puts
the apple back in the bag and picks another one.
a The probability that Peter picks 2 bruised apples is:
1 1 15
A 4
C 16
E 16
1 3
B 2
D 4
b The probability that Peter picks a bruised apple first
but a good one on his second attempt is:
1 1 3 3 1
A 4
B 2
C 4
D 16
E 16
1
7 The probability that John will be late for a meeting is 7 and the probability that Phil
3
will be late for a meeting is What is the probability that:
11
.
a John and Phil are both late
b neither of them is late
c John is late but Phil is not late
d Phil is late but John is not late?
8 On the roulette wheel at the casino there are 37 numbers, 0 to 36 inclusive. Bidesi
puts his chip on number 8 in game 20 and on number 13 in game 21.

a What is the probability that he will win in game 20?


b What is the probability that he will win in both games?
c What is the probability that he wins at least one of the games?
9 Based on her progress through the year, Karen was given a probability of 0.8 of
passing the Physics exam. If the probability of passing both Maths and Physics is
0.72, what is her probability of passing the Maths exam?

416 Maths Quest 10 New South Wales Australian curriculum edition Stages 5.1 and 5.2
STATistics and probability

10 Suresh found that, on average, he is delayed 2 times out of 7 at Melbourne airport.


Rakesh made similar observations at Brisbane airport, but found he was delayed
1 out of every 4 times. Find the probability that both Suresh and Rakesh will be
delayed if they are flying out of their respective airports.

11 Bronwyn has 3 pairs of Reebok and 2 pairs of Adidas running shoes. She has
2 pairs of Reebok, 3 pairs of Rio and a pair of Red Robin socks. Preparing for an
early morning run, she grabs at random for a pair of socks and a pair of shoes.
What is the probability that she chooses:
a Reebok shoes and Reebok socks
b Rio socks and Adidas shoes
c Reebok shoes and Red Robin socks
d Adidas shoes and socks that are not Red Robin?
12 WE14 Two cards are drawn successively and without replacement from a pack of
playing cards. Determine the probability of drawing:
a 2 hearts
b 2 kings
c 2 red cards.
13 In a class of 30 students there are 17 girls. Two students are picked randomly
to represent the class in the Student Representative Council. Determine the
probability that:
a both students are boys
b both students are girls
c one of the students is a boy.
REFLECTION
REASONING How are dependen t events,
14 Greg has tossed a tail on each of 9 successive coin independent events and
the multiplication law of
tosses. He believes that his chances of tossing a
probability reflected on a tree
Head on his next toss must be very high. Is Greg diagram?
correct? Justify your answer.

Chapter 12 • Probability 417


STATistics and probability

12D  Conditional probability


•• Conditional probability is when the probability of an event is conditional (depends)
on another event occurring first.
•• The effect of conditional probability is to reduce the event space and thus increase
the probability of the desired outcome.
•• For two events, A and B, the conditional probability of event B, given that event A
occurs, is denoted by P(B|A) and can be calculated using the formula:
P(A ∩ B)
P(B|A) = , P(A) ≠ 0
P(A)
WORKED EXAMPLE 15
A group of students was asked to nominate their favourite food, spaghetti (S) or lasagne (L).
The results are illustrated in the Venn diagram at right. Use the Venn diagram to calculate
the following probabilities relating to a student’s favourite food. ξ
a What is the probability that a randomly selected student prefers S L
spaghetti? 11 9 15
b What is the probability that a randomly selected student likes
lasagne given that they also like spaghetti? 5
T H IN K W RI T E /DR AW
a 1 From 40 students surveyed, shown a ξ
in blue, 20 nominated their favourite S L
food as ‘spaghetti’ or ‘spaghetti and 11 9 15
lasagne’ as shown in red.
5

number of favourable outcomes


2 The probability that a randomly P(event) =
selected student prefers spaghetti is total number of possible outcomes
20
found by substituting these values into P(spaghetti) =
the probability formula. 40
1
=
2
b 1 The condition imposed ‘given they b ξ
also like spaghetti’ alters the sample S L
space to the 20 students described 11 9 15
in part a , as shaded in blue. Of these
20 students, 9 stated their favourite 5
foods as lasagne and spaghetti, as
shown in red.
P(A ∩ B)
2 The probability that a randomly P(B|A) =
selected student likes lasagne, given P(A)
that they like spaghetti, is found by 9
40
substituting these values into the P(L|S) =
1
probability formula for conditional 2
probability. 9
= 20

418 Maths Quest 10 New South Wales Australian curriculum edition Stages 5.1 and 5.2
STATistics and probability

WORKED EXAMPLE 16
If P(A) = 0.3, P(B) = 0.5 and P(A ∪ B) = 0.6, calculate:
a P(A ∩ B) b P(B | A)
T H IN K W RI T E
a 1 State the addition law for probability to a P(A ∪ B) = P(A) + P(B) − P(A ∩ B)
determine P(A ∪ B).
2 Substitute the values given in the question into 0.6 = 0.3 + 0.5 − P(A ∩ B)
this formula and simplify. P(A ∩ B) = 0.3 + 0.5 − 0.6
= 0.2
P(A ∩ B)
b 1 State the formula for conditional probability. b P(B|A) = , P(A) ≠ 0
P(A)
2 Substitute the values given in the question into P(B|A) = 0.2
0.3
this formula and simplify.
= 23

•• It is possible to transpose the formula for conditional probability to calculate P(A ∩ B):

P(A ∩ B)
P(B|A) = , P(A) ≠ 0
P(A)
P(A ∩ B) = P(A) × P(B|A)

This is called the multiplication rule for probability.

Exercise 12D Conditional probability


INDIVIDUAL PATHWAYS
Questions: Questions: Questions:
1–7, 9, 11 1, 3–6, 8, 9, 11, 12 1–13
Activity 12‐D‐1 Activity 12‐D‐2 Activity 12‐D‐3
Introducing conditional Practice with conditional Tricky conditional probability
probability probability problems
doc‐13769 doc‐13770 doc‐13771

FLUENCY
1 WE15 A group of students was asked to nominate
their favourite form of dance, hip hop (H) or jazz (J).
The results are illustrated in the Venn diagram below.
Use the Venn diagram given to calculate the following
probabilities relating to a student’s favourite form of
dance.
ξ
H J

35 12 29

14

Chapter 12 • Probability 419


STATistics and probability

a What is the probability that a randomly selected student prefers jazz?


b What is the probability that a randomly selected student prefers hip hop, given
that they prefer jazz?
2 A group of students was asked which seats they found most comfortable, the seats
in the computer lab or the science lab. The results are illustrated in the Venn diagram
below. Use the Venn diagram given to calculate the following probabilities relating
to the most comfortable seats.
ξ
C S

15 8 5

a What is the probability that a randomly selected student prefers the science lab?
b What is the probability that a randomly selected student prefers the science lab,
given that they might prefer the computer lab or the science lab?
3 WE16 If P(A) = 0.7, P(B) = 0.5 and P(A ∪ B) = 0.9, calculate:
a P(A ∩ B) b P(B|A).
4 If P(A) = 0.65, P(B) = 0.75 and P(A ∩ B) = 0.45, calculate:
a P 1 B|A 2 b P 1 A|B 2 .

UNDERSTANDING
5 A medical degree requires applicants to participate in two tests, an aptitude test and
an emotional maturity test. This year 52% passed the aptitude test and 30% passed
both tests. Use the conditional probability formula to calculate the probability that a
student who passed the aptitude test also passed the emotional maturity test.
6 At a school classified as a ‘Music
school for excellence’ the prob-
ability that a student elects to
study Music and Physics is 0.2.
The probability that a student
takes Music is 0.92. What is the
probability that a student takes
Physics, given that the student is
taking Music?
7 The probability that a student is
well and misses a work shift the
night before an exam is 0.045,
and the probability that a student misses a work shift is 0.05. What is the probability
that a student is well, given they miss a work shift the night before an exam?
8 Two marbles are chosen, without replacement, from a jar containing only red and
green marbles. The probability of selecting a green marble and then a red marble
is 0.67. The probability of selecting a green marble on the first draw is 0.8. What is
the probability of selecting a red marble on the second draw, given the first marble
drawn was green?

420 Maths Quest 10 New South Wales Australian curriculum edition Stages 5.1 and 5.2
STATistics and probability

9 Consider rolling a red and a black die and the probabilities of the following events:
Event A the red die lands on 5
Event B the black die lands on 2
Event C the sum of the dice is 10.
a MC  The initial probability of each event described is:
1 5 5
A P(A) = B P(A) = 6 C P(A) = 6
6
P(B) = 16 P(B) = 26 P(B) = 2
6
7 5
P(C) = 16 P(C) = 36 P(C) = 18
1 1
D P(A) = 6
E P(A) = 6
1 2
P(B) = P(B) = 6
6
1 1
P(C) = P(C) = 12
12
b Calculate the following probabilities.
i P 1 A|B 2 ii P 1 B|A 2 iii P 1 C|A 2 iv P 1 C|B 2
10 MC A group of 80 schoolgirls consists of 54 dancers and 35 singers. Each
member of the group is either a dancer or a singer, or both. The probability that a
randomly selected student is a singer given that she is a dancer is:
A 0.17 B 0.44 C 0.68 D 0.11 E 0.78

REASONING
11 Explain how imposing a condition alters probability calculations.
12 At your neighbouring school, 65% of the students are male and 35% are female. Of
the male students, 10% report that dancing is their favourite activity; of the
female students, 25% report that dancing is their favourite activity. Find the
probability that:
a a student selected at random prefers dancing and is female
b a student selected at random prefers dancing and is male.
13 Using the information presented in question 12 above,
construct a tree diagram. From your diagram, calculate: REFLECTION
a the probability that a student is male and does not How does imposin g
a condition alter the
prefer dancing
probability of an event?
b the overall percentage of students who prefer dancing.

CHALLENGE 12.2

Chapter 12 • Probability 421


STATistics and probability

12E Subjective probability


•• Consider the following claims:
‘I feel the Australian cricket team will win this year’s Test cricket series because, in
my opinion, they have a stronger side than the opposition.’ Claims like this are often
made by people who may not have all the facts, and may also be biased.
  ‘I think this summer will be a cold one.’ A statement like this will have merit if
it comes from an individual with relevant knowledge, such as a meteorologist or a
scientist. However, often people make these remarks with limited observation.
•• Subjective probability is usually based on limited mathematical evidence and may
involve one or more of the following: judgements, opinions, assessments, estimations
and conjectures by individuals. It can also involve beliefs, sentiments and emotions
that may result in a certain amount of bias.

WORKED EXAMPLE 17
On Anzac Day Peter plays two‐up, which involves tossing two coins. Heads win if both coins
land Heads, while Tails win if both coins land Tails. If the coins land with one Head and one
Tail they are called ‘odd’, and the coins are tossed again until either Heads or Tails wins.
After observing for a while, Peter notices that the last five tosses had either Tails w
­ inning
or were odd. This leads Peter to believe that Heads will win the next game, so he places $50
on Heads and loses. Peter questions the fairness of the game and states that the game is
­biased and favours Tails. Discuss the accuracy of Peter’s statement.
T H IN K W RI T E
Discuss the statement made and Each game is independent, so five Tails or odd outcomes
comment on the probability of obtaining in the previous games have no effect on the outcome of
Heads or Tails in this particular game. the current game. The game is not biased. Peter took a risk
and paid for it. He is wrong in suggesting that the game is
not fair.

Exercise 12E Subjective probability


INDIVIDUAL PATHWAYS
Questions: Questions: Questions:
1–4 1–4 1–4
Activity 12‐E‐1 Activity 12‐E‐2 Activity 12‐E‐3
Subjective probability Harder subjective In‐depth subjective
doc‐13772 probability probability
doc‐13773 doc‐13774

UNDERSTANDING
1 WE17 Discuss the accuracy of these statements.
a The team batting last can never win a cricket match at the MCG.
b The Australian cricket team is so good that not even bad weather can stop it from
winning.
c Two children in John’s family are girls so the third one will be a girl, too.

422 Maths Quest 10 New South Wales Australian curriculum edition Stages 5.1 and 5.2
STATistics and probability

d The Wallabies defeated the All Blacks three times last year so they will win the
first game this year.
e It rained heavily on the last three consecutive Fridays so do not organise sport on
a Friday.
f According to the weather report only three in every twenty houses were damaged
by the cyclone, so my house will not be damaged.
g New Zealand lost its cricket match against Australia because their team uniform
looked boring.
h This coin is biased because we obtained six Heads in a row.
i The USA topped the medal tally in the last Olympics so they will do the same
again in the next Olympics.
j Australian Rules football is the best sport in the world.
2 Comment on the accuracy of these statements.
a I have bought only one ticket for the raffle, therefore I cannot win.
b This particular horse has odds of 1–2. It is certain to win.
c If you keep on betting on Heads, you cannot lose.
d If you want to win at all times, bet on the favourites.
e It is no use betting on the favourites as you cannot win a great deal of money,
therefore you should bet on the outsiders.
3 Assign a probability to each of the following,
based on your experience or judgement.
a The probability that you will be late for a
class this week
b The probability that your favourite sporting
team will win its next match
c The probability that two traffic lights in
a row will be red when you approach
successive intersections
d The probability that you will see a dog some
time today
REASONING
4 Comment on the contradictions involved in the following statements.
a That job was hers but she did not do well in the interview.
b The team had won the match but they became a little
complacent towards the end. REFLECTION
Digital doc
Is subjective probability
c ‘Makybe Diva’ was certain to win. I cannot believe reliable?
WorkSHEET 12.3
doc‐5297
she lost the race.

Chapter 12 • Probability 423


STATistics and probability

CHAPTER REVIEW
LANGUAGE

array impossible relative frequency


certain independent sample space
chance intersection scale
chance experiment likely subjective probability
complementary events mutually exclusive theoretical probability
conditional odds tree diagram
dependent outcome trial
equally likely payout two‐way table
event probability unlikely
favourable outcome random Venn diagrams

int‐2856 doc‐13777
int‐2857 doc‐13778
int‐3598

FLUENCY
1 Which of the following is always true for an event, M, and its complementary
event, M′?
A P(M) + P(M′) = 1 B P(M) − P(M′) = 1 C P 1 M 2 + P(M′) = 0
D P(M) − P(M′) = 0 E P(M) × P(M′) = 1
2 A number is chosen from the set 5 0, 1, 2, 3, 4, 5, 6, 7, 8, 9, 10 6 . Which of the
following pairs of events is mutually exclusive?
A 5 2, 4, 6 6 and 5 1, 2, 3 6
B 5 1, 2, 3, 5 6 and 5 4, 6, 7, 8 6
C 5 0, 1, 2, 3 6 and 5 3, 4, 5, 6 6
D 5 multiples of 2 6 and 5 factors of 8 6
E 5 even numbers 6 and 5 multiples of 3 6
3 Which of the following states the Multiplication Law of probability correctly?
A P(A ∩ B) = P(A) + P 1 B 2 B P(A ∩ B) = P(A) × P 1 B 2
C P(A ∪ B) = P(A) × P 1 B 2 D P(A ∪ B) = P(A) + P 1 B 2
E P(A) = P(A ∪ B) + P 1 B 2
4 The odds 3‐2 expressed as a probability are:
1 3 1 2 1
A 5
B 5
C 2
D 5
E 3
The following information relates to questions 5 and 6.
ξ = 5 1, 2, 3, 4, 5, 6, 7, 8, 9, 10 6 and A = 5 2, 3, 4 6 and B = 5 3, 4, 5, 8 6

424 Maths Quest 10 New South Wales Australian curriculum edition Stages 5.1 and 5.2
STATistics and probability

5 A ∩ B equals:
A 5 2, 3, 3, 4, 4, 5, 8 6 B 5 3, 4 6 C 5 2, 3, 4 6
D 5 2, 3, 4, 5, 8 6 E 5 2, 5, 8 6
6 A ∩ B′ equals:
A 5 3, 4 6 B 526 C 5 2, 3, 4, 5, 8 6
D 5 2, 3, 4 6 E 5 1, 2, 6, 7, 9, 10 6
7 Shade the region stated for each of the following Venn diagrams.
a A′ ∪ B    ξ
A B

b A′ ∩ B′      A B ξ

c A′ ∩ B′ ∩ C ξ
A B

8 Convert the following odds to probabilities.


a 3–7 b 5–2 c 12–5
9 Convert the following probabilities to odds.
7 6 25
a 11 b 7 c 33

PROBLEM SOLVING
1 From past experience, it is concluded that there is a 99% probability that July will be
a wet month in Launceston (it has an average rainfall of approximately 80 mm). The
probability that July will not be a wet month next year in Launceston is:
1
A 99% B 0.99 C 100 D 1 E 0
2 A card is drawn from a well‐shuffled deck of 52 cards. What is the theoretical prob-
ability of not selecting a red card?
3 1 1 1
A 4 B 4 C 13 D 2 E 0
3 Which of the following events is not equally likely?
A Obtaining a 5 or obtaining a 1 when a die is rolled
B Obtaining a club or obtaining a diamond when a card is drawn from a
pack of cards
C Obtaining 2 Heads or obtaining 2 Tails when a coin is tossed
D Obtaining 2 Heads or obtaining 1 Head when a coin is tossed twice
E Obtaining a 3 or obtaining a 6 when a die is rolled

Chapter 12 • Probability 425


STATistics and probability

4 The Australian cricket team has won 12 of the last 15 Test matches. What is the
experimental probability of Australia losing its next Test match?
4 1 1 3
A 5
B 5
C 4
D 4
E 1
5 A card is drawn from a well‐shuffled pack of 52 cards. What is the theoretical
­probability of drawing:
a an ace b a spade c a queen or a king d not a heart?
6 The odds for a horse to win a race are 4–3.
a What is the probability that this horse will win the race?
b What is the probability that this horse will not win the race?
c Charlie bets $12 that this horse will win. If the horse wins, what is Charlie’s payout?
7 A die is rolled five times.
a What is the probability of rolling five 6s?
b What is the probability of not rolling five 6s?
8 Alan and Mary own 3 of the 8 dogs in a race. What is the probability that:
a one of Alan’s or Mary’s dogs will win?
b none of Alan’s or Mary’s dogs will win?
9 A die is rolled. Event A is obtaining an even number. Event B is obtaining a 3.
a Are events A and B mutually exclusive?
b Calculate P(A) and P 1 B 2 .
c Calculate P(A ∪ B).
10 A card is drawn from a shuffled pack of 52 playing cards. Event A is drawing a
club and event B is drawing an ace.
a Are events A and B mutually exclusive?
b Calculate P(A), P(B) and P(A ∩ B).
c Calculate P(A ∪ B).
11 Discuss the accuracy of the following statements.
a It did not rain on Monday, Tuesday or Wednesday, so it will not rain on Thursday.
b A cricket team lost because two of its batsmen scored ducks.
c The Rams family had a boy, then a girl and then another boy. They must have a
girl next.
12 Comment on the contradictions involved in these statements.
a I was defeated by a loser.
b The slowest motocross racer in the competition won the race.
c The most popular person did not get elected.
13 The Venn diagram below shows the results of a survey completed by a Chinese
restaurateur to find out the food preferences of his regular customers.
Fried ξ
rice
7 5 12
3
10 5
Chicken
wings
8
Dim sims

426 Maths Quest 10 New South Wales Australian curriculum edition Stages 5.1 and 5.2
STATistics and probability

a Determine the number of customers:


i surveyed
ii showing a preference for fried rice only
iii showing a preference for fried rice
iv showing a preference for chicken wings and dim sims.
b A customer from this group won the draw for a lucky door prize. Determine the
probability that this customer:
i likes fried rice
ii likes all three — fried rice, chicken wings and dim sims
iii prefers chicken wings only.
c A similar survey was conducted a month later with another group of
50 customers. This survey yielded the following results: 2 customers liked
all three foods; 6 preferred fried rice and chicken wings; 7 preferred chicken
wings and dim sims; 8 preferred fried rice and dim sims; 22 preferred fried rice;
23 preferred chicken wings; and 24 preferred dim sims.
i Display this information on a Venn diagram.
ii What is the probability of selecting a customer who prefers all three foods, if
a random selection is made?
14 A pair of dice is rolled and the sum of the numbers shown is noted.
a Show the sample space in a two‐way table.
b In how many different ways can the sum of 7 be obtained?
c Are all outcomes equally likely?
d Complete the given table.
Sum 2 3 4 5 6 7 8 9 11 12
Frequency
e What are the relative frequencies of the following sums?
i 2 ii 7 iii 11
f What is the probability of obtaining the following sums?
i 2 ii 7 iii 11
g If a pair of dice is rolled 300 times, how many times do you expect to obtain the
sum of 7?
15 A tetrahedral die is numbered 0, 1, 2 and 3. Two of these dice are rolled and the
sum of the numbers (the number on the face that the die sits on) is taken.
a Show the possible outcomes in a two‐way table.
b Are all the outcomes equally likely?
c Which total has the least chance of being rolled?
d Which total has the best chance of being rolled?
e Which sums have the same chance of being rolled?
16 An eight‐sided die is rolled three times to see whether 5 occurs.
a Draw a tree diagram to show the sample space.
b Calculate:
i P(three 5s) ii P(no 5s)
iii P(two 5s) iv P(at least two 5s).

Chapter 12 • Probability 427


STATistics and probability

17 A tetrahedral die (four faces labelled 1, 2, 3 and 4) is rolled and a coin is tossed
simultaneously.
a Show all the outcomes on a two‐way table.
b Draw a tree diagram and list all outcomes and their respective probabilities.
c Calculate the probability of getting a Head on the coin and an even number on
the die.
18 A bag contains 20 pears, of which 5 are bad. Cathy randomly selects 2 pears
(without replacement) from the bag. What is the probability that:
a both pears are bad?
b both pears are good?
c one of the two pears is good?
19 Determine the probability of drawing 2 aces from a pack of cards if:
a the first card is replaced before the second one is drawn
b the first card drawn is not replaced.
20 On grandparents day at a school a group of grandparents was asked where they
most like to take their grandchildren — the beach (B) or shopping (S). The results
are illustrated in the Venn diagram below. Use the Venn diagram given to calculate
the following probabilities relating to the place grandparents most like to take their
grandchildren.
ξ
B S

5 8 2

10

a What is the probability that a randomly selected grandparent preferred to take


their grandchildren to the beach or shopping?
b What is the probability that a randomly selected grandparent preferred to
take their grandchildren to the beach, given that they preferred to take their
grandchildren shopping?
21 Two marbles are chosen, without replacement, from a jar containing only red and
green marbles. The probability of selecting a green marble and then a red marble
is 0.72. The probability of selecting a green marble on the first draw is 0.85. What
is the probability of selecting a red marble on the second draw if the first marble
drawn was green?

428 Maths Quest 10 New South Wales Australian curriculum edition Stages 5.1 and 5.2
STATistics and probability
Communicating

Rich task

Tricky dice

Die 1
1 Complete the grid at right to show the sample space on 1 2 3 1 2 3
rolling these two dice.
4
2 How many different outcomes are there? Compare this with
the number of different outcomes using two regular dice. 5

6
Die 2

4
5
6

3 What is the chance of rolling a double using these dice? Die 1

4 The numbers on the two dice are added after rolling. 1 2 3 1 2 3


Complete the table at right to show the totals possible. 4

6
Die 2

4
5
6

430 Maths Quest 10 New South Wales Australian curriculum edition Stages 5.1 and 5.2
STATistics and
STATISTICS AND PROBABILITY
probability

5 How many different totals are possible? What are they?


6 Which total do you have the greatest chance of rolling? Which total do you have the least
chance of rolling?
7 If you played a game in which you had to bet on rolling a total of less than 7, equal to 7 or
greater than 7, which option would you be best to take? Explain why.
8 If you had to bet on an even‐number outcome or an odd‐number outcome, which would be
the better option? Explain your answer.
9 The rules are changed to subtracting the numbers on the two dice instead of adding them.
Complete the following table to show the outcomes possible.
Die 1

1 2 3 1 2 3

6
Die 2

4
5
6

10 How many different outcomes are possible in this case? What are they?
11 What is the most frequently occurring outcome? How many times does it occur?
12 Devise a game of your own using these dice. On a separate sheet of paper, write out rules for
your game and provide a solution, indicating the best options for winning.

Chapter 12 • Probability 431


STATistics AND
STATISTICS and PROBABILITY
probability

Code puzzle

The study of .  .  .


Each probability and the letter beside it give the puzzle’s answer
code. Discover the matching word for each definition.

A card is drawn from a standard deck of playing cards.


Y P(drawing an ace) =
O P(drawing a diamond) =
M P(drawing a 4 of clubs) =
A P(drawing a numbered card) =
R P(drawing a card numbered less than 4) =
E P(drawing a black card numbered less than 7) =
H P(drawing a red numbered card that is a multiple of 3) =
G P(drawing a black 9) =
T P(drawing a picture card) =
L P(drawing a red card) =
I P(drawing an even numbered card) =
1 1 25 1 1 1 1 1
— — — — — — — —
The study of fungi: 52 13 51 4 2 4 26 13

1 5 26 1 1 1 1 1
— — — — — — — — : The study of Chinese language,
17 13 51 4 2 4 26 13
history, customs, etc.
1 1 1 1 1 1 1 1
— — — — — — — —
The science of fruit growing: 49 4 52 4 2 4 26 13

26 5 1 3 1 1 1 1 1

51

26

49

26

4

2

4

26

13 : The study of clouds

S P(drawing a queen if the queen of hearts has been drawn and not replaced) =
C P(drawing a black card if the ace of spades has been drawn and not replaced) =
N P(drawing a red card if the king of clubs has been drawn and not replaced) =
P P(drawing the ace of diamonds if 3 black cards have been drawn previously and not replaced) =

The study of extinct animals and plants:


1 9 1 9 5 1 26 3 1 1 1 1 1
— — — — — — — — — — — — —
49 13 2 13 26 4 51 13 4 2 4 26 13

The branch of medicine dealing with diseases of the ear, nose and throat:
1 3 1 2 3 5 26 1 1 9 2 1 26 1 1 1 1 1 1
— — — — — — — — — — — — — — — — — — —
4 13 4 13 26 13 51 4 2 13 13 13 51 26 4 2 4 26 13

432 Maths Quest 10 New South Wales Australian curriculum edition Stages 5.1 and 5.2
STATISTICS AND PROBABILITY

ACTIVITIES
Go to assessON
Chapter opener 12C Independent and dependent events for questions to
Digital doc Interactivities test your readiness
• The story of mathematics (eles-1851) • Independent and dependent events FOR learning, your
12A Review of probability (int‐2787) (page 412) progress AS you learn
Interactivity • Random numbers (int‐0085) (page 415) and your levels OF
• Random number generator (int‐0089) Digital docs (page 415) achievement.
(page 390) • Activity 12‐C‐1 (doc‐13766): Simple www.assesson.com.au
Digital docs independent and dependent events
• Activity 12‐A‐1 (doc‐5110): Review of • Activity 12‐C‐2 (doc‐13767): Independent and
probability (page 402) dependent events
• Activity 12‐A‐2 (doc‐5111): General probability • Activity 12‐C‐3 (doc‐13768): Tricky
problems (page 402) independent and dependent events
• Activity 12‐A‐3 (doc‐5112): Tricky probability 12D Conditional probability
problems (page 402) Digital docs (page 419)
• SkillSHEET (doc‐5286): Set notation • Activity 12‐D‐1 (doc‐13769): Introducing
(page 402)
• SkillSHEET (doc‐5287): Simplifying conditional probability
fractions (page 402) • Activity 12‐D‐2 (doc‐13770): Practice with
• SkillSHEET (doc‐5288): Determining conditional probability
complementary events (page 402) • Activity 12‐D‐3 (doc‐13771): Tricky conditional
• SkillSHEET (doc‐5289): Addition and probability problems
subtraction of fractions (page 402) 12E Subjective probability
• SkillSHEET (doc‐5291): Working with Digital docs
Venn diagrams (page 402) • Activity 12‐E‐1 (doc‐13772): Subjective
• SkillSHEET (doc‐5292): Writing odds as probability (page 422)
probabilities (page 402) • Activity 12‐E‐2 (doc‐13773): Harder subjective
• SkillSHEET (doc‐5293): Writing probabilities probability (page 422)
as odds (page 402) • Activity 12‐E‐3 (doc‐13774): In‐depth
• SkillSHEET (doc‐5294): Distinguishing subjective probability (page 422)
between complementary and mutually • WorkSHEET 12.3 (doc‐5297): Subjective
exclusive events (page 402) probability (page 423)
• WorkSHEET 12.1 (doc‐5295): Introducing Chapter review
probability (page 405) Interactivities (page 424)
12B Tree diagrams • Word search (int‐2856)
• Activity 12‐B‐1 (doc-13763): Review of • Crossword (int‐2857)
tree diagrams (page 409) • Sudoku (int‐3598)
• Activity 12‐B‐2 (doc-13764): Practice with Digital docs
tree diagrams (page 409) • Chapter summary (doc‐13777)
• Activity 12‐B‐3 (doc-13765): Tricky • Concept map (doc‐13778)
tree diagrams (page 409) To access eBookPLUS activities, log on to
• WorkSHEET 12.2 (doc-13775): Tree www.jacplus.com.au
diagrams (page 412)

Chapter 12 • Probability 433


STATistics and probability

Answers
CHAPTER 12 Probability
Exercise 12A — Review of probability 10    a ξ
A B
1 Experimental probability is based on the outcomes of experi- 5
3
ments, simulations or surveys. Theoretical probability is based 11 1 20
7 13 4
on the number of possible favourable outcomes and the total 15 9
16
possible outcomes. 17
19 14
2    a {I, II, III, IV, V, VI} 2 8
12
6 10 18
b Frequency for I = 0.2
Frequency for II = 0.12 b   i  10 = 12 ii 4
= 15 iii 2 1
= 10
Frequency for III = 0.16 20 20 20
Frequency for IV = 0.14 iv 12
= 35 v 8
= 25
Frequency for V = 0.24 20 20
1 4 1
Frequency for VI = 0.14 11    a 6
b 17
c 8
c The spinner should be spun a larger number of times.
3    a 500 students 12    A
b Frequency for silver = 0.16 13    a No. For a 6‐sided die, P(4) = 16; for an 8‐sided die, P(4) = 18.
c Frequency for Black and Green = 0.316 b Yes; P(odd) = 12.
d Blazing Blue
4    a 14 Yes; P(5) = 12, P(6) = 12.
+ 1 2 3 4 5 6
15    a ξ
1 2 3 4 5 6 7 B N

2 3 4 5 6 7 8 2
15 15
3 4 5 6 7 8 9 3
15 5
4 5 6 7 8 9 10
5 6 7 8 9 10 11 16
S 19
6 7 8 9 10 11 12
b 19 students
b The sum of 7 c 32 students
c The sum of 2 or 12 d 15 students
5    a P(A ∩ B) = 0.1 e Frequency = 0.1667
b ξ 3
A B f Probability = 90
1 1 1
16    a 13
b 4
c 2
0.4 0.1 0.3
12 1
d 13
e 0 f 2
1 7
0.2 17    a b
5 20

c P(A ∩ B′) = 0.4


11
c 20
d 0
6    a i   P(A ∪ B) = 0.85 ii P(A ∩ B)′ = 0.95
18    a ξ=30
b C Volleyball Soccer
2
7 3 1
7 4
1
8    a 13 b 14 c 134
5
2 4
9 Answers may vary; check with your teacher.
a i  No. There are many other foods one could have. 7
ii Having Weet Bix and not having Weet Bix Tennis
b i  No. There are other means of transport; for example, 1 1 1 2 7
catching a bus. b   i 2
ii 6
iii 30
iv 5
v 15
ii Walking to a friend’s place and not walking to a friend’s place 1 8
c   i ii
c i  No. There are other possible leisure activities. 2 15
ii Watching TV and not watching TV 19    a 3
b $50 c 9−4
10
d i  No. The number 5 can be rolled too.
1
ii  Rolling a number less than 5 and rolling a number 5 or 20 Yes. Both have a probability of 2
.
greater 6
e     Yes. There are only two possible outcomes; passing or 21 Yes, equivalent fractions; 16 = 38
failing.

434 Maths Quest 10 New South Wales Australian curriculum edition Stages 5.1 and 5.2
STATistics and probability

22 5    a 1
P′ P′P′P′ 1
A B A B 3 27
1 P′ 2
3 2 P P′P′P 27
P′ 1 3 2
1 3 P′ P′PP′ 27
3 2 P
3 4
2 P P′PP 27
1 3 2
P′ PP′P′
Overlaying A ʹ and B ʹ shows The union of A and B is shown 3 27
A ʹ ∩ B ʹ as the area surrounding in brown, leaving the surrounding 2 1 P′ 4
3 3 2 P PP′P 27
A and B area as (A ∪ B) ʹ P 1 3 4
3 P′ PPP′ 27
2 P
3 8
Challenge 12.1 2 P PPP 27
3
11 red and 4 purple
8 4 12
Exercise 12B — Tree diagrams b   i 27
  ii 27
  iii 27
1 If the probabilities of 2 events are different, the first column of 6    a 0.8 T′ T′T′T′ 0.512
branches indicates the probabilities for the first event and the 0.8 T′
second column of branches indicates the probabilities for the 0.2 T T′T′T 0.128
T′
second event. The product of each branch gives the probability. 0.8 T′ T′TT′ 0.128
0.8 0.2
All probabilities add to 1. T
0.2 T T′TT 0.032
2    a 12 different outcomes
0.8 T′ TT′T′ 0.128
b No. Each branch is a product of different probabilities.
0.8 T′
c Less likely 0.2 0.2 T TT′T 0.032
d Indigo elephant T
0.8 T′ TTT′ 0.032
7
e   i P(Blue elephant) = 40 0.2 T
0.2 T TTT 0.008
7
  ii P(Indigo elephant) = 20
b 0.520
7    a 2
4
  iii P(Donkey) = 15 3 M′ M′M′ 9
2 M′
3 1 2
M M′M 9
3    a 1st 2nd 3rd Outcome P(outcome) 3 2
2
toss toss toss 3 M′ MM′ 9
1
0.7 3 M
H HHH 0.343 1 1
3 M MM 9
0.7 H
1
0.3 T HHT 0.147 b 9
H
0.7 H HTH 0.147 1
0.7 0.3 8    a 2 1
T
1
0.3 T HTT 0.063 1 4 2
1 1 3
0.7 H THH 0.147 2 4
1
H 1 2 1
0.3 0.7 4 1
0.3 T THT 0.063 1 2 4
2
T 1 3
0.7 H TTH 0.063 4
1 1
0.3 T 4 2 1
T TTT 0.027 1 2
0.3 3 4
1 3
4
b P(HHH) = 0.343 1
2
1
2 1
c P(at least 1 Tail) = 0.657 1
1 2
d P(exactly 2 Tails) = 0.189 1
4
2
1 3
4
1 1 5 1
4    a   i 2
  ii 6
  iii 6
1 1 2 1
4 4 1
2 2 4
2
1 3
4
b 1
6 6
1 1 1
12 4 2 1
<4 1 2
3 4
1 5
2 5 6′ 1 1 3
12 4
6 4
1 1
1 6 36
1
6 6 2 1
4 1
5 5 1 4 2
6 6′ 1
36
2
1 3
1 1 4
1 1
6 6 18 1 1
3 2
>4 4 1
3 2 4
2
5 1
5 6′ 18
3
4
6 1 1
4 2 1
1 2
3 4
1 1 5 5
c   i 36
  ii 12
  iii 36
iv 18
1 3
4

Chapter 12 • Probability 435


STATistics and probability

b 3
c 7 9 0.9
8 8
1
9    a 0.75 Pudding 10 14
Vegetarian 1 1 1 1
11    a 5
b 5
c 10
d 3
0.25 Apple
0.2
1 1 25
0.75 Pudding 12    a 17
b 221
c 102
0.5
Soup Chicken
26 136 221
0.25 Apple 13    a 145
b 435
c 435

0.3
0.3 0.75 Pudding 14 No. Coin tosses are independent events. No one toss affects the
Beef outcome of the next. The probability of a Head or Tail on a fair
0.25 Apple coin is always 0.5. Greg has a 50% chance of tossing a Head
on the next coin toss as was the chance in each of the previous
0.75 Pudding
9 tosses.
Vegetarian Exercise 12D — Conditional probability
0.7 Apple
0.2 0.25 1    a P(J) = 41
90
0.75 Pudding
Prawn
0.5
Chicken b P(H | J) = 12
41
0.25 Apple
2    a P(S) = 13
30
0.3 0.75 Pudding
Beef b P(S | (C ∪ S)) = 13
28
0.25 Apple 3    a 0.3
3
b 0.1125 b 7
c 24 people
9
10    a i 0.7 Red 4    a 13
Red 3
0.7 0.3 White b 5
15
5 0.58 or 26
0.7 Red
0.3 5
White 6 0.22 or 23
0.3 White
7 0.9
ii 10 balls iii 10 balls 8 0.8375
iv No; the events are independent. 9    a D
v P(RR) = 0.49 b i P(A | B) = 16
b i  9 balls
ii Yes. One ball has been removed from the bag. ii P(B | A) = 16
iii 0.67 Red
Red iii P(C | A) = 16
0.7 0.33 White
iv P(C | B) = 0
10 A
0.78 Red
0.3 11 Conditional probability is when the probability of one event
White
depends on the outcome of another event.
0.22 White
12    a 0.0875
iv P(RR) = 7
or 0.469 (using the rounded values from iii) b 0.065
15
13    a 0.585
Exercise 12C — Independent and dependent events b 0.1525 or 15.25%.
1    a 0.28 b 0.12 c 0.42 d 0.18 Challenge 12.2
1 1 1 1
2    a Yes b i 2
ii 6
c 12 2
1
3 40
Exercise 12E — Subjective probability
1    a The outcome depends upon whether it is a Test match or a
5
4 36
one‐day game and how effective the bowlers and batsmen
are; not forgetting the pitch usually favours spin bowling.
16 64 1 4
5    a 25
b 125
c 25
d 25
b The outcome depends on which team is better on the day
and which team can adjust to the conditions.
6    a C b D c No. The third one has an equal chance of being a girl or
3 48 8 18
a boy.
7    a 77
b 77
c 77
d 77 d This is not necessarily true. Current position and form of
1 1 73 both teams should be used as a gauge.
8    a 37
b 1369
c 1369 e It does not mean it will rain again on Friday.

436 Maths Quest 10 New South Wales Australian curriculum edition Stages 5.1 and 5.2
STATistics and probability

f There is no certainty about that. It depends upon the condition 8    a 3


8
and location of your house. 5
g Cricket games are not won or lost by the attractiveness of the b 8
uniform. 9    a Yes
h It is possible to get 6 Heads in a row on a normal coin.
i They will have a good chance but there is no certainty. The b P(A) = 12 and P(B) = 16
country with the best competitors on the day of each event c 2
3
will win.
j This is dependent on the person’s own interests. 10    a No
2    a You still have a chance. b P(A) = 14, P(B) = 13
1
, P(A ∩ B) = 52
1

b No horse is certain to win. Lots of problems can occur on the 4


track. c 13
c This is not true. Even though Heads and Tails have equal 11    a Whether it rains or not on Thursday is not determined by what
chances, it does not mean half the results will show Heads. happened on Monday, Tuesday or Wednesday. It can still rain
d Favourites do not always win. on Thursday.
e Sometimes outsiders pay well, if you back the right one! You b The team’s win or loss depends upon how other players bat
can lose more money than you win. and bowl or how the other team plays.
3 Answers will vary. Class discussion is required as there are many c There is an equal chance of having a boy or a girl.
factors to consider. 12    a If you were defeated, the opponent was the winner.
4    a There is a contradiction. The job was never hers. She had to b The slowest motocross rider could not win the race if he/she
do well to win the position. crossed the finish line first.
b The team may have had a lead but a match is only won when c The person elected was the most popular choice for the
finished. position.
c No horse is certain to win. 13    a i 50 ii 7 iii 25 iv 8
Chapter review 3 6
b i 1 ii iii
Fluency 2 50 25
1 A 2 B 3 B 4 D ii 1
c i 25
5 B 6 B n(ξ) = 50
7    a A B ξ Fried Chicken
rice wings
10 4 12
2
6 5

11
Dim sims

b A B ξ
14    a Die 2 outcomes
1 2 3 4 5 6

1 (1, 1) (1, 2) (1, 3) (1, 4) (1, 5) (1, 6)

c 2 (2, 1) (2, 2) (2, 3) (2, 4) (2, 5) (2, 6)


Die 1 outcomes

A
ξ
B
3 (3, 1) (3, 2) (3, 3) (3, 4) (3, 5) (3, 6)

4 (4, 1) (4, 2) (4, 3) (4, 4) (4, 5) (4, 6)

5 (5, 1) (5, 2) (5, 3) (5, 4) (5, 5) (5, 6)

C 6 (6, 1) (6, 2) (6, 3) (6, 4) (6, 5) (6, 6)

7 2 5
8    a 10
b 7
c 17 b 6
9    a 4–7 c No. The frequency of the numbers is different.
b 1–6 d
c 8–25 Sum 2 3 4 5 6 7 8 9 10 11 12
Problem solving Frequency 1 2 3 4 5 6 5 4 3 2 1
1 C 2 D 3 D 4 B
1 1 2 3 1 1
5    a 13
b 4
c 13
d 4 e i 1 ii iii
36 6 18
3 4
6    a b c $28 1 1
7 7
f i 1 ii 6
iii 18
36
1 7775
7    a 7776
b 7776 g 50

Chapter 12 • Probability 437


STATistics and probability

15    a Die 2 outcomes Communicating — Rich task


1 Die 1
0 1 2 3
1 2 3 1 2 3
0 (0, 0) (0, 1) (0, 2) (0, 3)
Die 1 outcomes

4 (1, 4) (2, 4) (3, 4) (1, 4) (2, 4) (3, 4)


1 (1, 0) (1, 1) (1, 2) (1, 3)
5 (1, 5) (2, 5) (3, 5) (1, 5) (2, 5) (3, 5)
2 (2, 0) (2, 1) (2, 2) (2, 3)
6 (1, 6) (2, 6) (3, 6) (1, 6) (2, 6) (3, 6)

Die 2
3 (3, 0) (3, 1) (3, 2) (3, 3)
4 (1, 4) (2, 4) (3, 4) (1, 4) (2, 4) (3, 4)
b No
c 0 and 6 5 (1, 5) (2, 5) (3, 5) (1, 5) (2, 5) (3, 5)
d 3
6 (1, 6) (2, 6) (3, 6) (1, 6) (2, 6) (3, 6)
e 0 and 6, 1 and 5, 2 and 4
16    a 1 2 3 Outcomes Probability 2 9 3 0
1– 1
8 f fff —–
512 4 Die 1
1– f 7– fʹ fffʹ —–7
8 512

1– f
8 1–
8 f ffʹf —–7 1 2 3 1 2 3
512
8 7– fʹ 49
8 7– fʹ ffʹfʹ —–
8 1– 512
4 5 6 7 5 6 7
1–
8 f fʹff —–7
512
7– 8 f fʹ fʹffʹ 49
8 fʹ 7–
8 1–
—–
512 5 6 7 8 6 7 8
49
7– 8 f fʹfʹf —–
512
8 fʹ
7– fʹ fʹfʹfʹ 343
—– 6 7 8 9 7 8 9
Die 2
8 512

4 5 6 7 5 6 7
f = outcome of 5
b i 1
ii 343 5 6 7 8 6 7 8
512 512

21 11 6 7 8 9 7 8 9
iii 512
iv 256

17    a 5 5; 5, 6, 7, 8, 9 6 7; 5, 9
Die outcomes 7 Equal to 7; probability is the highest.
1 2 3 4 8 Odd‐number outcome; probability is higher.
Die
outcomes

H (H, 1) (H, 2) (H, 3) (H, 4)


Coin

1 2 3 4 5 6 7 8
T (T, 1) (T, 2) (T, 3) (T, 4)
Head (H, 1) (H, 2) (H, 3) (H, 4) (H, 5) (H, 6) (H, 7) (H, 8)
Coin

Tail (T, 1) (T, 2) (T, 3) (T, 4) (T, 5) (T, 6) (T, 7) (T, 8)


b Outcomes Probability
1– 𝖷 1– = 1–
1 H1 9
1–
4 1–
2 4 8 Die 1
4 2 H2 1– 𝖷 1– = 1–
2 4 8
H
1–
4
3 H3 1– 𝖷 1– = 1– 1 2 3 1 2 3
2 4 8
1– 1–
2 4 4 H4 1– 𝖷 1– =
2 4
1–
8 4 3 2 1 3 2 1
1– 𝖷 1– = 1–
1–
1 T1 2 4 8 5 4 3 2 4 3 2
1– 4 1–
4 1– 𝖷 1– = 1–
2 2 T2 2 4 8
T 1– 6 5 4 3 5 4 3
Die 2

4 1– 𝖷 1– = 1–
3 T3 2 4 8
1–
4 4 T4 1– 𝖷 1– =
2 4
1–
8
4 3 2 1 3 2 1

1
5 4 3 2 4 3 2
1
c 6 5 4 3 5 4 3
4
1 21 15
18    a 19
b 38
c 38
10 5; 1, 2, 3, 4, 5,
1
11 3; 12
19    a 169 12 Answers will vary.
b 1 Code puzzle
221
Mycology
20    a 15
= 35 Sinology
25
Pomology
b 8
= 45 Nephology
10 Palaeontology
21 0.847 Otorhinolaryngology

438 Maths Quest 10 New South Wales Australian curriculum edition Stages 5.1 and 5.2
STATistics and probability

chapter 13

Univariate data
WHY LEARN THIS?
According to the novelist Mark Twain, ‘There are three
kinds of lies: lies, damned lies and statistics.’ There
is so much information in our lives, increasingly so
with the world wide web, smart phones and social
media tracking our every move and accumulating vast
amounts of data about us. This data is used to gather
information about our likes and dislikes, our buying
habits, our voting preferences and so on. Statistics can
easily be used to manipulate people unless they have
an understanding of the basic concepts involved.

WHAT DO YOU KNOW?


1 Think List what you know about data.
Use a thinking tool such as a concept
map to show your list.
2 pair Share what you know with a partner
and then with a small group.
3 share As a class, create a large concept map
that shows your class’s knowledge of data.

LEARNING SEQUENCE
13A Measures of central tendency
13B Measures of spread
13C Box‐and‐whisker plots
13D Comparing data sets
Watch this video
The story of mathematics

Searchlight ID: eles-1852


STATistics and probability

13A Measures of central tendency


Univariate data
•• In this chapter you will learn how to measure and analyse Univariate data.
Univariate data is data with one variable; for example, the heights of year 10 students.
•• Measures of central tendency are summary statistics that measure the middle
(or centre) of the data. These are known as the mean, median and mode.
–– The mean is the average of all observations in a set of data.
–– The median is the middle observation in an ordered set of data.
–– The mode is the most frequent observation in a data set.

Mean, median and mode of ungrouped data


Mean
•• To obtain the mean of a set of ungrouped data, all numbers (scores) in the set are
added together and then the total is divided by the number of scores in that set.
sum of all scores
Mean =
number of scores
ox
•• Symbolically the mean is written x = .
n
Median
•• The median is the middle value of any set of data arranged in ascending numerical order.
n+1
In the set of n numbers, the median is located at the th score. The median is:
2
–– the middle score for an odd number of scores arranged in numerical order
–– the average of the two middle scores for an even number of scores arranged in
numerical order.
Mode
•• The mode is the score that occurs most often in a set of data.
•• Sets of data may contain:
1. no mode; that is, each score occurs once only
2. one mode
3. more than one mode.

WORKED EXAMPLE 1
For the data 6, 2, 4, 3, 4, 5, 4, 5, find the:
a mean b median c mode.

T HIN K W RI T E
a 1 Calculate the sum of the scores; that is, Σx. a ox = 6 + 2 + 4 + 3 + 4 + 5 + 4 + 5
= 33
2 Count the number of scores; that is, n. n=8
ox
3 Write the rule for the mean. x=
n

442 Maths Quest 10 New South Wales Australian curriculum edition Stages 5.1 and 5.2
STATistics and probability

33
4 Substitute the known values into the rule. =
8
5 Evaluate. = 4.125
6 Answer the question. The mean is 4.125.
b 1 To calculate the median arrange in ascending order. b 23444556
n+1
2 Locate the position of the median using the rule Median = th score
n+1 2
, where n = 8. This places the median as the 8+1
2 = th score
4.5th score; that is, between the 4th and 5th score. 2
= 4.5th score
23444556
4+4
3 Obtain the average of the two middle scores. Median =
2
8
=
2
= 4
4 Answer the question. The median is 4.
↓↓
c 1 Systematically work through the set and make note c 23444556
of any repeated values (scores). ↑↑ ↑
2 Answer the question. The mode is 4.

Calculating mean, median and mode from a frequency


distribution table
•• If data are presented in a frequency distribution table, the formula used to calculate
o (f × x)
the mean is x = .
n
•• Here, each value (score) in the table is multiplied by its corresponding frequency;
then all the f × x products are added together and the total sum is divided by the
number of observations in the set.
•• To find the median, find the position of each score from the cumulative frequency
column.
•• The mode is the score with the highest frequency.

WORKED EXAMPLE 2
For the table at right find the:
a mean Score (x) Frequency ( f )
b median 4 1
c mode. 5 2
6 5
7 4
8 3
Total 15

Chapter 13 • Univariate data 443


STATistics and probability

T HIN K W RI T E
1 Rule up a table with four columns titled
Frequency Cumulative
Score (x), Frequency ( f ), Frequency ×
Score Frequency × score frequency
score ( f × x) and Cumulative
(x) (f) (f × x) (cf )
frequency (cf ).
2 Enter the data and complete both 4 1 4 1
the f × x and cumulative frequency 5 2 10 1+2=3
columns.
6 5 30 3+5=8
7 4 28 8 + 4 = 12
8 3 24 12 + 3 = 15
n = 15 g( f × x) = 96

g( f × x)
a 1 Write the rule for the mean. a x=
n
96
2 Substitute the known values into the x=
rule and evaluate. 15
= 6.4
3 Answer the question. The mean of the data set is 6.4.

Locate the position of the median using 15 + 1


b 1 b The median is the th or 8th score.
2
n+1
the rule , where n = 15. This
2
places the median as the 8th score.
2 Use the cumulative frequency column The median of the data set is 6.
to find the 8th score and answer the
question.
c 1 The mode is the score with the highest c The score with the highest frequency is 6.
frequency.
2 Answer the question. The mode of the data set is 6.

Mean, median and mode of grouped data


Mean
•• When the data are grouped into class intervals, the actual values (or data) are lost. In
such cases we have to approximate the real values with the midpoints of the intervals
into which these values fall. For example, when measuring heights of students
in a class, if we found that 4 students had a height between 180 and 185 cm, we
have to assume that each of those 4 students is 182.5 cm tall. The formula used for
calculating the mean is:

a ( f × x)
x=
n

444 Maths Quest 10 New South Wales Australian curriculum edition Stages 5.1 and 5.2
STATistics and probability

  Here x represents the midpoint (or class centre) of each class interval, f is the
corresponding frequency and n is the total number of observations in a set.
Median
•• The median is found by drawing a cumulative frequency curve (ogive) of the data
and estimating the median from the 50th percentile.

Modal class
•• The modal class is the class interval that has the highest frequency.

Cumulative frequency curves (ogives)


Ogives
•• Data from a cumulative frequency table can be plotted to form a cumulative
frequency curve (sometimes referred to as cumulative frequency polygons), which is
also called an ogive (pronounced ‘oh‐jive’).
•• To plot an ogive for data that is in class intervals, the maximum value for the class
interval is used as the value against which the cumulative frequency is plotted.
For example, the following table and graph show the mass of cartons of eggs ranging
from 55 g to 65 g.

Cumulative Percentage cumulative


Mass (g) Frequency ( f ) frequency (cf ) frequency (%cf )
55–<57 2 2   6%
57–<59 6 2+6=8 22%
59–<61 12 8 + 12 = 20 56%
61–<63 11 20 + 11 = 31 86%
63–<65 5 31 + 5 = 36 100%
Percentage cumulative frequency

100
90
80
70
60
50
40
30
20
10
0
55 56 57 58 59 60 61 62 63 64 65 66
Mass (g)

Quantiles
•• An ogive can be used to divide the data into any given number of equal parts called
quantiles.
•• Quantiles are named after the number of parts that the data is divided into.
–– Percentiles divide the data into 100 equal‐sized parts.
–– Quartiles divide the data into 4 equal‐sized parts. For example, 25% of the data
values lie at or below the first quartile.

Chapter 13 • Univariate data 445


STATistics and probability

Percentile Quartile and symbol Common name


25th percentile First quartile, Q1 Lower quartile
50th percentile Second quartile, Q2 Median
75th percentile Third quartile, Q3 Upper quartile
100th percentile Fourth quartile, Q4 Maximum

•• A percentile is named after the percentage of data that lies at or below that value. For
example, 60% of the data values lie at or below the 60th percentile.
•• Percentiles can be read off a percentage cumulative frequency curve.
•• A percentage cumulative frequency curve is created by:
–– writing the cumulative frequencies as a percentage of the total number of data
values
–– plotting the percentage cumulative frequencies against the maximum value for
each interval.

WORKED EXAMPLE 3
For the given data:
Class interval Frequency
a estimate the mean
b estimate the median 60–<70 5
c find the modal class.
70–<80 7
80–<90 10
90–<100 12
100–<110 8
110–<120 3
Total 45

T HIN K W RI T E
1 Draw up a table with 5 columns Class Fre­ Frequency Cumulative
headed Class interval, Class centre Class Centre quency × Class Centre fre­quency
(x), Frequency ( f ), Frequency × interval (x) ( f) ( f × x) (cf )
class centre ( f × x) and
60–<70 65 5 325 5
Cumulative frequency (cf ).
70–<80 75 7 525 12
2 Complete the x, f × x and cf 80–<90 85 10 850 22
columns. 90–<100 95 12 1140 34
100–<110 105 8 840 42

n = 45 g( f × x) = 4025
110–<120 115 3 345 45

446 Maths Quest 10 New South Wales Australian curriculum edition Stages 5.1 and 5.2
STATistics and probability

g( f × x)
a 1 Write the rule for the mean. a x=
n
2 Substitute the known values 4025
x=
into the rule and evaluate. 45
≈ 89.4
3 Answer the question. The mean for the given data is approximately 89.4.
b 1 Draw a combined cumulative b
45
frequency histogram and 40

Cumulative frequency
ogive, labelling class centres 35
on the horizontal axis and 30
25
cumulative frequency on the 20
vertical axis. Join the end‐ 15
points of each class interval 10
5
with a straight line to form 0
65 75 85 95105115
the ogive. Data

2 Locate the middle of the


45
cumulative frequency axis,
Cumulative frequency

40
which is 22.5. 35
30
3 Draw a horizontal line from 25
this point to the ogive and 20
15
then a vertical line to the 10
horizontal axis. 5
0
65 75 85 95105115
Data

4 Read off the value of the The median for the given data is approximately 90.
median from the x‐axis and
answer the question.
c 1 The modal class is the class c The class interval 90–<100 occurs 12 times, which is
interval with the highest the most frequent.
frequency.
2 Answer the question. The modal class is the 90–<100 class interval.

Exercise 13A Measures of central tendency


INDIVIDUAL PATHWAYS
Questions: Questions: Questions:
1–10, 12, 15, 16 1–10, 12, 14, 16 1–17
Activity 13‐A‐1 Activity 13‐A‐2 Activity 13‐A‐3
Mean, median and mode Practice with mean, Mean, median
doc‐5128 median and mode and mode in depth
doc‐5129 doc‐5130

Chapter 13 • Univariate data 447


STATistics and probability

FLUENCY
Digital docs
1 WE1 For each of the following sets of data find the:
SkillSHEET i mean ii median iii mode.
Finding the mean of a
small data set a 3, 5, 6, 8, 8, 9, 10
doc‐5299
SkillSHEET
b 4, 6, 7, 4, 8, 9, 7, 10
Finding the median of a c 17, 15, 48, 23, 41, 56, 61, 52
small data set
doc‐5300 d 4.5, 4.7, 4.8, 4.8, 4.9, 5.0, 5.3
7 12, 10 14, 12, 12 14, 13, 13 12, 13 12, 14
SkillSHEET
Finding the mode of a e
small data set
doc‐5301 2 The back‐to‐back stem‐and‐leaf plot below shows
SkillSHEET the test results of 25 Year 10 students in Mathematics
Finding the mean,
median and mode from and Science. Find the mean, median and mode for
a stem‐and‐leaf plot each of the two subjects.
doc‐5302
SkillSHEET Key: 3 ∣ 2 = 32
Presenting data in a
frequency distribution Leaf Stem Leaf
table Science Mathematics
doc‐5303
SkillSHEET 873 3 29
Drawing statistical
graphs 96221 4 068
doc‐5304
876110 5 135
97432 6 2679
8510 7 3678
73 8 044689
9 258
3 WE2 For the data shown in each of the following frequency distribution tables, find the:
i mean ii median iii mode.

a Score (x) Frequency ( f ) b Score (x) Frequency ( f )


4 3 12 4
5 6 13 5
6 9 14 10
7 4 15 12
8 2 16 9
Total 24 Total 40
4 The following data show the
number of bedrooms in each of
the 10 houses in a particular
neighbourhood: 2, 1, 3, 4, 2,
3, 2, 2, 3, 3.
a Calculate the mean and median
number of bedrooms.
b A local motel contains 20 rooms.
Add this observation to the set
of data and recalculate the values
of the mean and median.

448 Maths Quest 10 New South Wales Australian curriculum edition Stages 5.1 and 5.2
STATistics and probability

c Compare the answers obtained in parts a and b and complete the following
statement: When the set of data contains an unusually large value(s), called an
outlier, the ________ (mean/median) is the better measure of central tendency, as
it is less affected by this extreme value.
5 WE3 For the given data:
a estimate the mean b estimate the median c find the modal class.

Class interval Frequency


40–<50 2
50–<60 4
60–<70 6
70–<80 9
80–<90 5
90–<100 4
Total 30

6 Calculate the mean of the grouped data shown in the table below.

Class interval Frequency


100–109 3
110–119 7
120–129 10
130–139 6
140–149 4
Total 30

7 Find the modal class of the data shown in the table below.

Class interval Frequency


50–<55 1
55–<60 3
60–<65 4
65–<70 5
70–<75 3
75–<80 2
Total 18

Chapter 13 • Univariate data 449


STATistics and probability

8 MC The number of textbooks sold by various bookshops during the second week
of December was recorded. The results are summarised in the table below.

Number of books sold Frequency


220–229 2
230–239 2
240–249 3
250–259 5
260–269 4
270–279 4
Total 20

a The modal class of the data is given by the class interval(s):


A 220–229 and 230–239 B 250–259
C 260–269 and 270–279 D of both A and C
b The class centre of the first class interval is:
A 224 B 224.5 C 224.75 D 225
c The median of the data is in the interval:
A 230–239 B 240–249 C 250–259 D 260–269
d The estimated mean of the data is:
A 251 B 252 C 253 D 254

UNDERSTANDING
9 A random sample was taken, composed of 30 people shopping at a Coles
supermarket on a Tuesday night. The amount of money (to the nearest dollar) spent
by each person was recorded as follows:
6, 32, 66, 17, 45, 1, 19, 52, 36, 23, 28, 20, 7, 47, 39
6, 68, 28, 54, 9, 10, 58, 40, 12, 25, 49, 74, 63, 41, 13
a Find the mean and median amount of money spent at the checkout by the people
in this sample.
b Group the data into class intervals of 10 and complete the frequency distribution
table. Use this table to estimate the mean amount of money spent.
c Add the cumulative frequency column to your table and fill it in. Hence,
construct the ogive. Use the ogive to estimate the median.
d Compare the mean and the median of the original data from part a with the mean
and the median obtained for grouped data in parts b and c. Were the estimates
obtained in parts b and c good enough? Explain your answer.
10 a Add one more number to the set of data 3, 4, 4, 6 so that the mean of a new set is
equal to its median.
b Design a set of five numbers so that mean = median = mode = 5.
c In the set of numbers 2, 5, 8, 10, 15, change one number so that the median
remains unchanged while the mean increases by 1.

450 Maths Quest 10 New South Wales Australian curriculum edition Stages 5.1 and 5.2
STATistics and probability

11 Thirty men were asked to reveal the number of hours they spent doing housework
each week. The results are detailed below.
1 5 2 12 2 6 2 8 14 18
0 1 1 8 20 25 3 0 1 2
7 10 12 1 5 1 18 0 2 2
a Present the data in a frequency distribution table. (Use class intervals of 0–4,
5–9 etc.)
b Use your table to estimate the mean number of hours that the men spent doing
housework.
c Find the median class for hours spent by the men at housework.
d Find the modal class for hours spent by the men at housework.
REASONING
18 16 6 75 24
12 The data at right give the age of 25 patients admitted to the emergency
ward of a hospital. 23 82 75 25 21
a Present the data in a frequency distribution table. (Use class intervals of
43 19 84 76 31
0–<15, 15–<30 and so on.) 78 24 20 63 79
b Draw a histogram of the data. 80 20 23 17 19
c What word could you use to describe the pattern of the data in this distribution?
d Use your table to estimate the mean age of patients admitted.
e Find the median class for age of patients admitted.
f Find the modal class for age of patients admitted.
g Draw an ogive of the data.
h Use the ogive to determine the median age.
i Do any of your statistics (mean, median or mode) give a
clear representation of the typical age of an emergency
ward patient?
j Give some reasons which could explain the pattern of the
distribution of data in this question.
13 The batting scores for two cricket players over 6 innings are as follows:
Player A 31, 34, 42, 28, 30, 41
Player B 0, 0, 1, 0, 250, 0
a Find the mean score for each player.
b Which player appears to be better, based upon mean result?
c Find the median score for each player.
d Which player appears to be better when the decision is based on the
median result?
e Which player do you think would be the most useful to have in a cricket
team and why? How can the mean result sometimes lead to a misleading
conclusion?
14 The resting pulse rate of 20 female athletes was measured. The results are
detailed below.
50 52 48 52 71 61 30 45 42 48
43 47 51 62 34 61 44 54 38 40

Chapter 13 • Univariate data 451


STATistics and probability

a Construct a frequency distribution table. (Use class sizes of 1–<10, 10–<20 etc.)
b Use your table to estimate the mean of the data.
c Find the median class of the data.
d Find the modal class of the data.
e Draw an ogive of the data. (You may like to use a graphics calculator for this.)
f Use the ogive to determine the median pulse rate.
15 MC In a set of data there is one score that is extremely small when compared to
all the others. This outlying value is most likely to:
A have greatest effect upon the mean of the data
B have greatest effect upon the median of the data
C have greatest effect upon the mode of the data
D have very little effect on any of the statistics as we are told that the number is
extremely small.
16 The following frequency table gives the number of employees in different salary
brackets for a small manufacturing plant.
Number of
Position Salary ($) employees
Machine operator 18 000 50
Machine mechanic 20 000 15
Floor steward 24 000 10
Manager 62 000 4
Chief executive officer 80 000 1

a Workers are arguing for a pay rise but the management of the factory claims that
workers are well paid because the mean salary of the factory is $22 100. Are
they being honest?
b Suppose that you were representing the factory workers and had to write a short
submission in support of the pay rise. How could you explain the management’s
claim? Quote some other statistics in favour of your case.
17 Design a set of five numbers with:
a mean = median = mode REFLECTION
Under what circumstances might
b mean > median > mode the median be a more reliable
c mean < median = mode. measure of centre than the mean?

CHALLENGE 13.1

452 Maths Quest 10 New South Wales Australian curriculum edition Stages 5.1 and 5.2
STATistics and probability

13B  Measures of spread


•• Measures of spread describe how far data values are spread from the centre or from
each other.
•• A music store proprietor has stores in Newcastle and Wollongong. The number of
CDs sold each day over one week is recorded below.

Newcastle: 45, 60, 50, 55, 48, 40, 52


Wollongong: 20, 85, 50, 15, 30, 60, 90

In each of these data sets consider the measures of central tendency.


Newcastle: Mean = 50 Wollongong: Mean = 50
Median = 50 Median = 50
No mode No mode
With these measures being the same for both data sets we could come to the
conclusion that both data sets are very similar; however, if we look at the data sets,
they are very different. We can see that the data for Newcastle are between 45 and 60,
whereas the Wollongong data are from 15 to 90.
•• Range and interquartile range (IQR) are both measures of spread.

Range
•• The most basic measure of spread is the range. It is defined as the difference
between the highest and the lowest values in the set of data.
Range = highest score − lowest score
⇒ Range = Xmax − Xmin

WORKED EXAMPLE 4
Find the range of the given data set: 2.1, 3.5, 3.9, 4.0, 4.7, 4.8, 5.2.
T HIN K W RI T E
1 Identify the lowest score of the data set. Lowest score Xmin = 2.1
2 Identify the highest score of the data set. Highest score Xmax = 5.2
3 Write the rule for the range. Range = Xmax − Xmin
4 Substitute the known values into the rule. = 5.2 − 2.1
5 Evaluate. = 3.1

Interquartile range
•• The interquartile range (IQR) is the range of the middle 50% of all the scores in an
ordered set.
When calculating the interquartile range, the data is first organised into quartiles,
each containing 25% of the data. The word ‘quartile’ comes from the word
‘quarter’.

Chapter 13 • Univariate data 453


STATistics and probability

25% 25% 25% 25%

Minimum Q1 Q2 Q3 Maximum
Lower Median Upper
quartile quartile
Interquartile range = upper quartile – lower quartile
This can be written as:

= Qupper − Qlower
IQR     
or
= Q3 − Q1
IQR     

•• The IQR is not affected by extremely large or extremely small data values (outliers),
so in some circumstances the IQR is a better indicator of the spread of data than
the range.

WORKED EXAMPLE 5
Calculate the interquartile range (IQR) of the following set of data: 3, 2, 8,
6, 1, 5, 3, 7, 6.
T HIN K W RI T E
1 Arrange the scores in order. 123356678
2 Locate the median and use it to divide the data 1 2 3 3    5   6 6 7 8
into two halves. Note: The median is the 5th score
in this data set and should not be included in the
lower or upper end of the data.
3 Find Q1, the median of the lower half of the data. 2+3
Q1 =
2
5
=
2
= 2.5
4 Find Q3, the median of the upper half of the data. 6+7
Q3 =
2
13
=
2
= 6.5
5 Calculate the interquartile range. IQR = Q3 − Q1
= 6.5 − 2.5
=4

Determining the IQR from a graph


•• When data are presented in a frequency distribution table, either ungrouped or
grouped the interquartile range is find by drawing are ogive.

454 Maths Quest 10 New South Wales Australian curriculum edition Stages 5.1 and 5.2
STATistics and probability

WORKED EXAMPLE 6
The following frequency distribution table gives the number of customers who order
different volumes of concrete from a readymix concrete company during the course of a day.
Find the interquartile range of the data.
Volume (m3) Frequency Volume (m3) Frequency
0.0–<0.5 15 1.5–<2.0 8
0.5–1.0 12 2.0–<2.5 2
1.0–<1.5 10 2.5–<3.0 4

T HIN K W RI T E
1 To find the 25th and 75th percentiles from the ogive, first
Class
add a class centre column and a cumulative frequency
Volume centre f cf
column to the frequency distribution table and fill them in.
0.0–<0.5 0.25 15 15
0.5–1.0 0.75 12 27
1.0–<1.5 1.25 10 37
1.5–<2.0 1.75 8 45
2.0–<2.5 2.25 2 47
2.5–<3.0 2.75 4 51

2 Draw the ogive. A percentage axis will be useful.

Cumulative frequency (%)


Cumulative frequency

50 100%
40 75%
30
50%
20
10 25%

0 5 75 25 5 5 75
0.2 0. 1. 1.7 2.2 2.
Volume (m3)

3 Find the upper quartile (75th percentile) and lower quartile Q3 = 1.6 m3
(25th percentile) from the ogive. Q1 = 0.4 m3
4 The interquartile range is the difference between the upper IQR = Q3 − Q1
and lower quartiles. = 1.6 − 0.4
= 1.2 m3

Exercise 13B Measures of spread


INDIVIDUAL PATHWAYS
Questions: Questions: Questions:
1–7, 10 1–8, 10, 11 1–11
Activity 13–B‐1 Activity 13–B‐2 Activity 13–B‐3
Range and quartiles Practice with range Range and quartiles
doc‐5131 and quartiles in depth
doc‐5132 doc‐5133

Chapter 13 • Univariate data 455


STATistics and probability

FLUENCY
1 WE4 Find the range for each of the following sets of data.
a 4, 3, 9, 12, 8, 17, 2, 16
b 49.5, 13.7, 12.3, 36.5, 89.4, 27.8, 53.4, 66.8
1 3 1 2 1 3
c 7 2, 12 4, 5 4, 8 3, 9 6, 3 4
2 WE5 Calculate the interquartile range (IQR) for the following sets of data.
a 3, 5, 8, 9, 12, 14
b 7, 10, 11, 14, 17, 23
c 66, 68, 68, 70, 71, 74, 79, 80
d 19, 25, 72, 44, 68, 24, 51, 59, 36
3 The following stem‐and‐leaf plot shows the mass of newborn babies (rounded to the
nearest 100 g). Find the:
a range of the data
b IQR of the data.
Key: 1* | 9 = 1.9 kg
Stem Leaf
1* 9
2 24
2* 6789
3 001234
3* 55678889
4 01344
4* 56689
5 0122

Cumulative frequency (%)


4 Use the ogive at right to determine the
Cumulative frequency

50 100%
interquartile range of the data.
40
30
50%
20
10

100 120 140 160 180


Height (cm)
5 WE6 The following frequency distribution table gives the amount of time spent by
50 people on shopping for Christmas presents. Estimate the IQR of the data.

Time (h) 0–<0.5 0.5–<1 1–<1.5 1.5–<2 2–<2.5 2.5–<3 3–<3.5 3.5–<4
Frequency 1 2 7 15 13 8 2 2

6 MC  Calculate the interquartile range of the following data:


17, 18, 18, 19, 20, 21, 21, 23, 25
A 8 B 18 C 4 D 20

456 Maths Quest 10 New South Wales Australian curriculum edition Stages 5.1 and 5.2
STATistics and probability

UNDERSTANDING
7 The following frequency distribution table shows the life expectancy of
40 household batteries.

Life (hours) 50–<55 55–<60 60–<65 65–<70 70–<75 75–<80


Frequency 4 10 12 8 5 1
a Draw an ogive curve that represents the data in the table above.
b Use the ogive to answer the following questions.
i What is the median score?
ii What are the upper and lower quartiles?
iii What is the interquartile range?
iv How many batteries lasted less than 60 hours?
v How many batteries lasted 70 hours or more?
8 Calculate the IQR for the following data.
Class interval Frequency
120–<130 2
130–<140 3
140–<150 9
150–<160 14
160–<170 10
170–<180 8
180–<190 6
190–<200 3
9 For each of the following sets of data, state: i the range and ii the IQR of each set.
a 6, 9, 12, 13, 20, 22, 26, 29
b 7, 15, 2, 26, 47, 19, 9, 33, 38
c 120, 99, 101, 136, 119, 87, 123, 115, 107, 100

REASONING
10 As newly appointed coach of Terrorolo’s Meteors netball team, Kate decided to
record each player’s statistics for the previous season. The number of goals scored
by the leading goal shooter was:
1, 3, 8, 18, 19, 23, 25, 25, 25, 26, 27, 28,
28, 28, 28, 29, 29, 30, 30, 33, 35, 36, 37, 40.
a Find the mean of the data.
b Find the median of the data.
c Find the range of the data.
d Find the interquartile range of the data.
e There are three scores that are much lower than most. Explain the effect these
scores have on the summary statistics.

Chapter 13 • Univariate data 457


STATistics and probability

11 The following back‐to‐back stem‐and‐leaf plot shows the ages of 30 pairs of men
and women when entering their first marriage.

Key: 1 ∣ 6 = 16 years old


Leaf: Men Stem Leaf: Women
998 1 67789
99887644320 2 001234567789
9888655432 3 01223479
6300 4 1248
60 5 2

a Find the mean, median, range and interquartile REFLECTION


What do measures of
range of each set.
Digital doc spread tell us about a set
WorkSHEET 13.1 b Write a short paragraph comparing the two of data?
doc-5311
distributions.

13C  Box‐and‐whisker plots


Five‐number summary
•• A five‐number summary is a list consisting of the lowest score, lower quartile,
median, upper quartile and greatest score of a set of data.

X min Q1 Median Q3 X max

WORKED EXAMPLE 7
From the following five‐number summary:

X min Q1 Median Q3 X max


29 37 39 44 48

find:
a the interquartile range b the range.

T HIN K W RI T E
a The interquartile range is the a IQR = Q3 − Q1
difference between the upper and lower = 44 − 37
quartiles. =7

b The range is the difference between the b Range = Xmax − Xmin


greatest score and the lowest score. = 48 − 29
= 19

458 Maths Quest 10 New South Wales Australian curriculum edition Stages 5.1 and 5.2
STATistics and probability

Box‐and‐whisker plots
•• A box‐and‐whisker plot (or boxplot) is a graph of the five‐number summary.
•• Box‐and‐whisker plots consist of a central divided box with attached whiskers.
•• The box spans the interquartile range.
•• The median is marked by a vertical line drawn inside the box.
•• The whiskers indicate the range of scores:

The lowest The lower The The upper The greatest


score quartile median quartile score
Xmin Q1 Med. Q3 Xmax
(lower extreme) (upper extreme)

•• Box‐and‐whisker plots are always drawn to scale. 4 15 21 23 28


•• They are presented either with the five‐number
summary figures attached as labels (diagram at right) or with a scale presented alongside
the box‐and‐whisker plot like the diagram below. They can also be drawn vertically.
0 5 10 15 20 25 30 Scale

Identification of extreme values


•• If an extreme value or outlier occurs in a set of data, it can be denoted by a small
cross on the box‐and‐whisker plot. The whisker is then shortened to the next largest
(or smallest) figure.
The box‐and‐whisker plot below shows that the lowest score was 5. This was an
extreme value as the rest of the scores were located within the range 15 to 42.
0 5 10 15 20 25 30 35 40 45 Scale

Describing distributions
Symmetry and skewness
•• A symmetrical plot has data that is evenly spaced around a central point. Examples
of a stem‐and‐leaf plot and a symmetrical boxplot are shown below.
Stem Leaf
26* 6
27 013
27* 5 6 8 9 x
20 22 24 26 28 30

28 011124
28* 5 7 8 8
29 222
29* 5

Chapter 13 • Univariate data 459


STATistics and probability

•• A negatively skewed plot has larger amounts of data as the values of the data
increase. This is illustrated by the stem‐and‐leaf plot below where the leaves increase
in length as the data increases in value. It is illustrated on the boxplot when the
median is to the right within the box.
Stem Leaf
5 1
6 29
7 1122 x
0 2 4 6 8 10
8 144566
9 534456777

•• A positively skewed plot has larger amounts of data as the values of the data
decrease. This is illustrated on the stem‐and‐leaf plot below where the leaves
increase in length as the data decreases in value. It is illustrated on the boxplot
when the median is to the left within the box.
Stem Leaf
5 134456777
6 244566
7 1122 x
0 2 4 6 8 10
8 16
9 5

WORKED EXAMPLE 8
The following stem‐and‐leaf plot gives the speed of 25 cars caught by a
roadside speed camera.

Key: 8 | 2 = 82 km/h, 8* | 6 = 86 km/h


Stem Leaf
8 224444
8* 5 5 6 6 7 9 9 9
9 01124
9* 5 6 9
10 02
10*
11 4

a Prepare a five‐number summary of the data.


b Draw a box‐and‐whisker plot of the data. (Identify any extreme values.)
c Describe the distribution of the data.

460 Maths Quest 10 New South Wales Australian curriculum edition Stages 5.1 and 5.2
STATistics and probability

T HIN K W RI T E
1 First identify the positions of The median is the 25+1 th score — that is,
2
the median and upper and lower the13th score.
quartiles. There are 25 data values.
Q1 is the 12+1 th score in the lower half —
n+1 2
The median is the th score. that is, the 6.5th score. That is, halfway
2
The lower quartile is the median of between the 6th and 7th scores.
the lower half of the data. The upper Q3 is halfway between the 6th and 7th
quartile is the median of the upper scores in the upper half of the data.
half of the data (each half contains
12 scores).
2 Mark the positions of the median 8 | 2 = 82 km/h
Key:
and upper and lower quartiles on 8* | 6 = 86 km/h
the stem‐and‐leaf plot. Q1
Stem Leaf
8 2 2 4 4 4 4| Median
8* 556679 9 9
9 0 1 1 2 4|
9* 569
Q3
10 02
10*
11 4

a Write the five‐number summary: a Five‐number summary:


The lowest score is 82.
The lower quartile is between Xmin Q1 Median Q3 Xmax
84 and 85; that is, 84.5. 82 84.5 89 94.5 114
The median is 89.
The upper quartile is between
94 and 95; that is, 94.5.
The greatest score is 114.
b Draw a labelled axis using an b Speed (km/h)
appropriate scale. Plot the points 80 90 100 110
from the five‐number summary.

c Describe the distribution. c The data are skewed (positively).

Shapes of graphs
Boxplots and dot plots
•• Boxplots are a concise summary of data. A boxplot can be directly related to
a dot plot.
•• Dot plots display each data value represented by a dot placed on a number line.

Chapter 13 • Univariate data 461


STATistics and probability

The following data is the amount of money (in $) that a group of 27 five‐year‐olds
had with them on a day visiting the zoo with their parents.

0 1.65 0 2.60 3 8.45 4 0.55 4.10 3.35 3.25


2 2.85 2.90 1.70 3.65 1 0 0 2.25 2.05 3
3.80 2.65 4.75 3.90 2.95

•• The dot plot below and its comparative boxplot show the distribution of this data.

0 2 4 6 8 10
Amount of money ($)

Both graphs indicate that the data is positively skewed. The dot plot clearly
shows the presence of the outlier. This is less obvious with the box plot. However,
the boxplot provides an excellent summary of the centre and spread of the
distribution.

Boxplots and histograms


•• Histograms are graphs that display continuous numerical variables and do not retain
all original data.
•• The following data are the number of minutes, rounded to the nearest minute, that
forty Year 10 students take to travel to their school on a particular day.

15 22 14 12 21 34 19 11 13 0 16
4 23 8 12 18 24 17 14 3 10 12
9 15 20 5 19 13 17 11 16 19 24
12 7 14 17 10 14 23

The data is displayed in the histogram and boxplot shown.

16
14
Frequency

12
10
8
6
4
2
0
0 5 10 15 20 25 30 35

0 5 10 15 20 25 30 35
Number of minutes

Both graphs indicate that the data is slightly positively skewed. The histogram
clearly shows the frequencies of each class interval. Neither graph displays the
original values. The histogram does not give precise information about the centre,
but the distribution of the data is visible. However, the boxplot provides an excellent
summary of the centre and spread of the distribution.

462 Maths Quest 10 New South Wales Australian curriculum edition Stages 5.1 and 5.2
STATistics and probability

Parallel boxplots
•• A major reason for developing statistical skills is to be able to make comparisons
between sets of data.
•• Consider the following scenario.
–– Each member of a class was given a jelly snake to stretch.

They each measured the initial length of their snake to the nearest centimetre and
then slowly stretched the snake to make it as long as possible. They then measured
the maximum length of the snake by recording how far it had stretched at the time
it broke. The results were recorded in the following table.

Initial length Stretched length Initial length Stretched length


(cm) (cm) (cm) (cm)
13 29 14 27
14 28 13 27
17 36 15 36
10 24 16 36
14 35 15 36
16 36 16 34
15 37 17 35
16 37 12 27
14 30 9 17
16 33 16 41
17 36 17 38
16 38 16 36
17 38 17 41
14 31 16 33
17 40 11 21

Chapter 13 • Univariate data 463


STATistics and probability

–– The data was then displayed on parallel boxplots, with the axis displaying in
millimetres.
–– By drawing the two boxplots on a single axis it is easy to compare them.
Stretched

Initial

100 200 300 400 500


Length of snake in mm

The change in the length of the snake when stretched is evidenced by the increased
median and spread shown on the boxplots. The median snake length before being
stretched was 150 mm, but the median snake length after being stretched was 350 mm.
The range increased after stretching, as did the IQR.

Exercise 13C Box‐and‐whisker plots


INDIVIDUAL PATHWAYS
Questions: Questions: Questions:
1–7, 10, 13 1–8, 10–12, 14 1–18
Activity 13‐C‐1 Activity 13‐C‐2 Activity 13‐C‐3
Constructing boxplots Boxplots and outliers Boxplots with decimals
doc‐5134 doc‐5135 doc‐5136

FLUENCY
1 WE7 From the following five‐number summary, find:
Xmin Q1 Median Q3 Xmax
6 11 13 16 32

a the interquartile range


b the range.
2 From the following five‐number summary, find:

Xmin Q1 Median Q3 Xmax


101 119 122 125 128

a the interquartile range


b the range.
3 From the following five‐number summary, find:

Xmin Q1 Median Q3 Xmax


39.2 46.5 49.0 52.3 57.8
a the interquartile range
b the range.

464 Maths Quest 10 New South Wales Australian curriculum edition Stages 5.1 and 5.2
STATistics and probability

4 The box‐and‐whisker plot at 50 70 90 110 130 150 Points


right shows the distribution of
final points scored by a football
team over a season’s roster.
a What was the team’s greatest points score?
b What was the team’s least points score?
c What was the team’s median points score?
d What was the range of points scored?
e What was the interquartile range of points scored?
5 The box‐and‐whisker plot at right shows Number of
30 35 40 45 50 55 60 gummy bears
the distribution of data formed by counting
the number of gummy bears in each of a
large sample of packs.

a What was the largest number of gummy bears in any pack?


b What was the smallest number of gummy bears in any pack?
c What was the median number of gummy bears in any pack?
d What was the range of numbers of gummy bears per pack?
e What was the interquartile range of gummy bears per pack?
Questions 6 to 8 refer to the following box‐and‐whisker plot.
5 10 15 20 25 30 Score

6 MC The median of the data is:


A 20 B 23 C 25 D 31
7 MC The interquartile range of the data is:
A 23 B 26 C 5 D 20 to 25
8 MC Which of the following is not true of the data represented by the box‐and‐
whisker plot?
A One‐quarter of the scores are between 5 and 20.
B Half of the scores are between 20 and 25.
C The lowest quarter of the data is spread over a wide range.
D Most of the data are contained between the scores of 5 and 20.

Chapter 13 • Univariate data 465


STATistics and probability

UNDERSTANDING
9 The number of sales made each day by a salesperson is
recorded over a 2‐week period:
25, 31, 28, 43, 37, 43, 22, 45, 48, 33
a Prepare a five‐number summary of the data. (There is no
need to draw a stem‐and‐leaf plot of the data. Just arrange
them in order of size.)
b Draw a box‐and‐whisker plot of the data.

10 The data below show monthly rainfall in millimetres.

J F M A M J J A S O N D
10 12 21 23 39 22 15 11 22 37 45 30

a Prepare a five‐number summary of the data.


b Draw a box‐and‐whisker plot of the data.
11 WE 8 The stem‐and‐leaf plot at right details Key: 1 | 8 = 18 years
the age of 25 offenders who were caught during
random breath testing. Stem Leaf
a Prepare a five‐number summary of the data. 1 8 8 9 9 9
b Draw a box‐and‐whisker plot of the data. 2 0 0 0 1 1 3 4 6 9
c Describe the distribution of the data. 3 0 1 2 7
4 2 5
5 3 6 8
6 6
7 4

12 The following stem‐and‐leaf plot details the price at which 30 blocks of land in a
particular suburb sold for.
Key: 12 | 4 = $12 4000
Stem Leaf
12 4 7 9
13 0 0 2 5 5
14 0 0 2 3 5 5 7 9 9
15 0 0 2 3 7 7 8
16 0 2 2 5 8
17 5
a Prepare a five‐number summary
of the data.
b Draw a box‐and‐whisker plot of
the data.

466 Maths Quest 10 New South Wales Australian curriculum edition Stages 5.1 and 5.2
STATistics and probability

13 Prepare comparative boxplots for the following dot plots (using the same axis) and
describe what each plot reveals about the data.
a Number of sick days taken by workers last year at factory A

0 1 2 3 4 5 6 7

b Number of sick days taken by workers last year at factory B

0 2 4 6 8 10 12 14

14 An investigation into the transport needs of an outer suburb community


recorded the number of passengers boarding a bus during each of its
journeys, as follows.
12, 43, 76, 24, 46, 24, 21, 46, 54, 109, 87, 23, 78, 37, 22, 139, 65, 78,
89, 52, 23, 30, 54, 56, 32, 66, 49
Display the data by constructing a histogram using class intervals of
20 and a comparative boxplot on the same axis.
15 At a weight‐loss clinic, the following weights (in kilograms) were
recorded before and after treatment.

Before 75 80 75 140 77 89 97 123 128 95 152 92


After 69 66 72 118 74 83 89 117 105 81 134 85

Before 85 90 95 132 87 109 87 129 135 85 137 102


After 79 84 90 124 83 102 84 115 125 81 123 94

a Prepare a five‐number summary for weight before and after treatment.


b Draw parallel boxplots for weight before and after treatment.
c Comment on the comparison of weights before and after treatment.

REASONING
16 The following data detail the number of hamburgers sold by a fast food
outlet every day over a 4‐week period.

M T W T F S S
125 144 132 148 187 172 181
134 157 152 126 155 183 188
131 121 165 129 143 182 181
152 163 150 148 152 179 181

a Prepare a stem‐and‐leaf plot of the data. (Use a class size of 10.)


b Draw a box‐and‐whisker plot of the data.
c What do these graphs tell you about hamburger sales?

Chapter 13 • Univariate data 467


STATistics and probability

17 The following data show the ages of 30 mothers upon the birth of their first baby.
22 21 18 33 17 23 22 24 24 20
25 29 32 18 19 22 23 24 28 20
31 22 19 17 23 48 25 18 23 20
a Prepare a stem‐and‐leaf plot of the data. (Use a class size of 5.)
b Draw a box‐and‐whisker plot of the data. Indicate any extreme values
appropriately.
c Describe the distribution in words. What does the distribution say about the age
that mothers have their first baby?
18 MC Match the box‐and‐whisker plot with its most likely histogram.

A f B f

Size Size
REFLECTION
C f D f What advantages
and disadvantages
do box‐and‐whisker
plots have as a
visual form of
representing data?
Size Size

13D  Comparing data sets


•• Back‐to‐back stem‐and‐leaf plots and multiple or parallel box‐and‐whisker plots
can be used to help compare statistics such as the median, range and interquartile
Interactivity range for sets of data about the same information.
Parallel boxplots
int‐2788

WORKED EXAMPLE 9
A shop compares the number
Key: 1 ∣ 2 = 12 customers
of customers on weekdays and
weekends. Twenty weekdays Leaf: Weekday Stem Leaf: Weekend
and twenty weekend days are 7 0 788
chosen for the sample. The 86311 1 1124456667
back‐to‐back stem‐and‐leaf 9666554331 2 2558
plot at right shows the result.
952 3 16
a Find the median number of
customers on weekdays and 5 4
weekends. 5 7
b Calculate the range of customer numbers on weekdays and
weekends.
c What conclusions can be made from the display about the average
number of customers on weekdays and weekends?

468 Maths Quest 10 New South Wales Australian curriculum edition Stages 5.1 and 5.2
STATistics and probability

T HIN K W RI T E
a There are 20 scores in each set a Weekdays: Median = 24 + 25
and so the median will be the 2
average of the 10th and 11th = 24.5
16 + 16
scores. Weekends: Median =
2
= 16

b For each data set, subtract the b Weekdays: Range = 45 − 7


lowest score from the highest = 38
score. Weekends: Range = 57 − 7
= 50

c Write your conclusion from c There are generally fewer customers on


observing that there are generally weekends. There is one outlier in the
fewer customers on weekends weekend scores, causing the range to be
and the results are more larger. However, apart from this outlier,
consistent except for one outlier. the weekend scores are less spread out.

•• In Worked example 9, the interquartile range could have been calculated as further
evidence that the scores on weekends are generally less spread out.
•• The most common method for comparing data sets is to compare the summary
statistics for both data sets.
–– The measures of centre such as mean and median are used to compare the typical
score in a data set.
–– Measures of spread such as range and interquartile range are used to make
assessments about the consistency of scores in the data set.

WORKED EXAMPLE 10
Below are the scores for two students in eight mathematics tests throughout
the year.
John: 45, 62, 64, 55, 58, 51, 59, 62
Penny: 84, 37, 45, 80, 74, 44, 46, 50
a Use the statistics function on a calculator to find the mean for each
student.
b Which student had the better overall performance on the eight tests?
c Which student was more consistent over the eight tests?

T HIN K W RI T E
a Enter the statistics into a a John: x = 57
calculator and use the x function Penny: x = 57.5
for the mean. Write the means.
b Compare the mean for each b Penny performed slightly better overall as
student. The student with the her mean mark was higher than John’s.
higher mean performed better Penny
overall. Range = 47
IQR = 22.5

Chapter 13 • Univariate data 469


STATistics and probability

John
Range = 19
IQR = 9
c Compare the range and c John was the more consistent student
interquartile range for each because his range and interquartile range
student. The student with the were much lower than Penny’s.
lower range and IQR performed
more consistently.

Exercise 13D Comparing data sets


INDIVIDUAL PATHWAYS
Questions: Questions: Questions:
1–7, 9, 10, 12, 14 1–12, 15 1–16
Activity 13‐D‐1 Activity 13‐D‐2 Activity 13‐D‐3
Comparing data 1 Comparing data 2 Comparing data 3
doc‐13779 doc‐13780 doc‐13781

UNDERSTANDING
1 WE9 The back‐to‐back stem‐and‐leaf plot drawn below shows the number of days
that both a group of boys and girls were absent from school over a two‐year period.

Key:2 ∣ 1 = 21 days
Leaf: Boys Stem Leaf: Girls
0 17
7410 1 24799
9976653110 2 133466
87752 3 4448
2 4 36
5 4
a Calculate the median number of days absent for both boys and girls.
b Calculate the range for both boys and girls.
c Comment on the distribution of days absent for each group.
2 A bank surveys the average morning and afternoon waiting times for customers.
The figures were taken each Monday to Friday in the morning and afternoon for one
month. The stem‐and‐leaf plot below shows the results.

Key: 1 ∣ 2 = 1.2 minutes


Leaf: Morning Stem Leaf: Afternoon
7 0 788
86311 1 1124456667
9666554331 2 2558
952 3 16
5 4
5 7

470 Maths Quest 10 New South Wales Australian curriculum edition Stages 5.1 and 5.2
STATistics and probability

a Find the median morning waiting time and the median afternoon waiting time.
b Calculate the range for morning waiting times and the range for afternoon waiting
times.
c What conclusions can be made from the display about the average waiting time at
the bank in the morning compared with the afternoon?
3 In a class of 30 students there are 15 boys and 15 girls. Their heights are measured
(in metres) and are listed below.

Boys: 1.65, 1.71, 1.59, 1.74, 1.66, 1.69, 1.72, 1.66,


1.65, 1.64, 1.68, 1.74, 1.57, 1.59, 1.60
Girls: 1.66, 1.69, 1.58, 1.55, 1.51, 1.56, 1.64, 1.69,
1.70, 1.57, 1.52, 1.58, 1.64, 1.68, 1.67

Display this information in a back‐to‐back stem‐and‐leaf plot.


4 The stem‐and‐leaf plot at right is used to display the number of vehicles sold by
the Ford and Holden dealerships in a Sydney suburb each week for a three‐month
period.
a State the median of both distributions. Key: 1 ∣ 5 = 15 vehicles
b Calculate the range of both distributions. Leaf: Ford Stem Leaf: Holden
c Calculate the interquartile range of both 74 0 39
distributions. 952210 1 111668
d Show both distributions on a box‐and‐ 8544 2 2279
whisker plot. 0 3 5

Chapter 13 • Univariate data 471


STATistics and probability

5 The box‐and‐whisker plot drawn below displays statistical data for two AFL teams
over a season.
Sydney Swans

Brisbane Lions

50 60 70 80 90 100 110 120 130 140 150 Points

a Which team had the higher median score?


b What was the range of scores for each team?
c For each team calculate the interquartile range.
6 Tanya measures the heights (in m) of a group of Year 10 boys and girls and produces
the following five‐point summaries for each data set.
Boys: 1.45, 1.56, 1.62, 1.70, 1.81
Girls: 1.50, 1.55, 1.62, 1.66, 1.73
a Draw a box‐and‐whisker plot for both sets of data and display them on the
same scale.
b What is the median of each distribution?
c What is the range of each distribution?
d What is the interquartile range for each distribution?
e Comment on the spread of the heights among the boys and the girls.
7 The box‐and‐whisker plots at right show the average daily sales of cold drinks at
the school canteen in summer and winter.
Summer

Winter

0 5 10 15 20 25 30 35 40 Daily sales
of cold
drinks

a Calculate the range of sales in both summer and winter.


b Calculate the interquartile range of the sales in both summer and winter.
c Comment on the relationship between the two data sets, both in terms of measures
of centre and measures of spread.
8 MC Andrea surveys the age of people at two movies being shown at a local cinema.
The box‐and‐whisker plot at right shows the results.
Movie A

Movie B

0 10 20 30 40 50 60 70 80 Age

Which of the following conclusions could be drawn based on the above information?
A Movie A attracts an older audience than Movie B.
B Movie B attracts an older audience than Movie A.
C Movie A appeals to a wider age group than Movie B.
D Movie B appeals to a wider age group than Movie A.

472 Maths Quest 10 New South Wales Australian curriculum edition Stages 5.1 and 5.2
STATistics and probability

9 MC Note: There may be more than one correct answer.


The figures below show the age of the first 10 men and women to finish a
marathon.

Men: 28, 34, 25, 36, 25, 35, 22, 23, 40, 24
Women: 19, 27, 20, 26, 30, 18, 28, 25, 28, 22

Which of the following statements is correct?


A The mean age of the men is greater than the mean age of the women.
B The range is greater among the men than among the women.
C The interquartile range is greater among the men than among the women.
D The median is greater among the men than among the women.

REASONING
10 WE10 Cory recorded his marks for each test that he did in English and Science
throughout the year.

English: 55, 64, 59, 56, 62, 54, 65, 50


Science: 35, 75, 81, 32, 37, 62, 77, 75

a In which subject did Cory achieve the better average mark?


b In which subject was Cory more consistent? Explain your answer.
11 The police set up two radar speed checks on a back street of Sydney and on a main
road. In both places the speed limit is 60 km/h. The results of the first 10 cars that
have their speed checked are given below.

Back street: 60, 62, 58, 55, 59, 56, 65, 70, 61, 64
Main road: 55, 58, 59, 50, 40, 90, 54, 62, 60, 60

a Calculate the mean of the readings taken at each point.


b On which road are drivers generally driving faster?
c On which road is the spread of the reading taken greater? Justify your
answer.
12 Nathan and Timana are wingers in their local rugby league team. The number of
tries they have scored in each season are listed below.

Nathan: 25, 23, 13, 36, 1, 8, 0, 9, 16, 20


Timana: 5, 10, 12, 14, 18, 11, 8, 14, 12, 19

a Calculate the mean number of tries scored by each player.


b What is the range of tries scored by each player?
c What is the interquartile range of tries scored by each player?
d Which player would you consider to be the more consistent player? Justify your
answer.

Chapter 13 • Univariate data 473


STATistics and probability

13 In boxes of Smarties it is advertised that


there are 50 Smarties in each box. Two
machines are used to distribute the Smarties
into the boxes. The results from a sample
taken from each machine are shown in the
stem‐and‐leaf plot below.

Key: 5 | 1 = 51    5* | 6 = 56
Leaf: Machine A Stem Leaf: Machine B
4 4
99877665 4* 57899999999
43222111000000 5 0000011111223
55 5* 9

a Display the data from both machines on parallel box‐and‐whisker


plots.
b Calculate the mean of the number of matches distributed from both
machines.
c Which machine is the more dependable? Justify your answer.
14 Year 10 students at Merrigong High School sit exams in Science and Maths. The
results are shown in the table below.

Number of students Number of students


Mark Science Maths
51–60 7 6
61–70 10 7
71–80 8 12
81–90 8 9
91–100 2 6

a Is either distribution symmetrical?


b If either distribution is not symmetrical, state whether it is positively or
negatively skewed.
c Discuss the possible reasons for any skewness.
d State the modal class of each distribution.
15 Draw an example of a graph that is:
a symmetrical
b positively skewed with one mode
c negatively skewed with two modes.

474 Maths Quest 10 New South Wales Australian curriculum edition Stages 5.1 and 5.2
STATistics and probability

16 A new drug for the relief of cold


symptoms has been developed. To
test the drug, 40 people were exposed
to a cold virus. Twenty patients were
then given a dose of the drug while
another 20 patients were given a
placebo. (In medical tests a control
group is often given a placebo drug.
The subjects in this group believe that
they have been given the real drug
but in fact their dose contains no drug
at all.) All participants were then asked to indicate the time when they first felt
relief of symptoms. The number of hours from the time the dose was administered
to the time when the patients first felt relief of symptoms are detailed below.

Group A (drug)
25 29 32 45 18 21 37 42 62 13
42 38 44 42 35 47 62 17 34 32

Group B (placebo) Digital doc


WorkSHEET 13.2
doc-13782
25 17 35 42 35 28 20 32 38 35
34 32 25 18 22 28 21 24 32 36
a Detail the data on a back‐to‐back stem‐and‐leaf plot.
b Display the data for both groups on a box‐and‐whisker plot.
c Make comparisons of the data. Use statistics in your
answer.
d Does the drug work? Justify your answer. REFLECTION
What statistical values
e What other considerations should be taken into
should be used when
account when trying to draw conclusions from an comparing data sets?
experiment of this type?

CHALLENGE 13.2

Chapter 13 • Univariate data 475


STATistics and probability

CHAPTER REVIEW
LANGUAGE

box‐and‐whiskers frequency distribution outlier


box‐and‐whiskers table percentile
plot grouped data positively skewed
cumulative frequency interquartile range quartile
curve interval range
data mean score
data sets measures of central skewed
extreme values tendency
skewness
five‐number median
summary spread
mode
frequency stem‐and‐leaf plot
negatively skewed
frequency distribution symmetrical
ogive

int‐2859 doc‐13784
int‐2860 doc‐13785
int‐3599

FLUENCY
1 Find the mean, median and mode for each of the following sets of data:
a 7, 15, 8, 8, 20, 14, 8, 10, 12, 6, 19
b Key: 1 | 2 = 12
Stem Leaf
1 26
2 178
3 033468
4 01159
5 136

c Score (x) Frequency ( f )


70 2
71 6
72 9
73 7
74 4

476 Maths Quest 10 New South Wales Australian curriculum edition Stages 5.1 and 5.2
STATistics and probability

2 For each of the following data sets, find the range.


a 4, 3, 6, 7, 2, 5, 8, 4, 3
b x 13 14 15 16 17 18 19
f 3 6 7 12 6 7 8
c Key: 1 | 8 = 18
Stem Leaf
1 7889
2 12445777899
3 0001347
3 For each of the following data sets, find the interquartile range.
a 18, 14, 15, 19, 20, 11, 16, 19, 18, 19
b Key: 9 | 8 = 9.8
Stem Leaf
8 7889
9 02445777899
10 01113
4 The following back‐to‐back stem‐and‐leaf
plot shows the typing speed in words per
minute (wpm) of 30 Year 8 and Year 10
students.

Key: 2 | 6 = 26 wpm
Leaf: Year 8 Stem Leaf: Year 10
99 0
9865420 1 79
988642100 2 23689
9776410 3 02455788
86520 4 1258899
5 03578
6 003
a Using a calculator or otherwise, construct a pair of parallel box‐and‐whisker plots
to represent the two sets of data.
b Find the mean, median, range and interquartile range of each set.
c Compare the two distributions, using your answers to parts a and b.
5 Consider the box‐and‐whisker plot drawn below.

0 5 10 15 20 25 30 35 40 Score
a Find the median.
b Find the range.
c Find the interquartile range.

Chapter 13 • Univariate data 477


STATistics and probability

6 The following data give the amount of cut meat (in kg) obtained from each of
20 lambs.
4.5 6.2 5.8 4.7 4.0 3.9 6.2 6.8 5.5 6.1
5.9 5.8 5.0 4.3 4.0 4.6 4.8 5.3 4.2 4.8
a Detail the data on a stem‐and‐leaf plot. (Use a class size of 0.5 kg.)
b Prepare a five‐point summary of the data.
c Draw a box‐and‐whisker plot of the data.

PROBLEM SOLVING
7 A sample of 30 people was selected at random from those attending a local
swimming pool. Their ages (in years) were recorded as follows:
19 7 58 41 17 23 62 55
40 37 32 29 21 18 16 10
40 36 33 59 65 68 15 9
20 29 38 24 10 30
a Find the mean and the median age of the people in this sample.
b Group the data into class intervals of 10 (0–9 etc) and complete the frequency
distribution table.
c Use the frequency distribution table to estimate the mean age.
d Calculate the cumulative frequency and, hence, plot the ogive.
e Estimate the median age from the ogive.
f Compare the mean and median of the original data in part a with the estimates of
the mean and the median obtained for the grouped data in parts c and e.
g Were the estimates good enough? Explain your answer.
8 Consider the box‐and‐whisker plot below which shows the number of weekly sales
of houses by two real estate agencies.
HJ Looker

Hane & Roarne

0 1 2 3 4 5 6 7 8 9 10 Number of
weekly sales

a What is the median number of weekly sales for each real estate agency?
b Which agency had the greatest range of sales?
c Which agency had the greatest interquartile range of sales?
d Which agency performed better? Explain your answer.
9 Kloe compares her English and Maths marks. The results of eight tests in each
subject are shown below.
English: 76, 64, 90, 67, 83, 60, 85, 37
Maths: 80, 56, 92, 84, 65, 58, 55, 62
a Calculate Kloe’s mean mark in each subject.
b Calculate the range of marks in each subject.
c Based on the above data, in which subject would you say that Kloe has performed
more consistently?

478 Maths Quest 10 New South Wales Australian curriculum edition Stages 5.1 and 5.2
STATistics and probability

10 Consider the data set represented by the frequency histogram below.


10
9
8
7
6

Frequency
5
4
3
2
1
0
1 2 3 4 5
Score
a Are the data symmetrical?
b Can the mean and median of the data be seen? If so, what are their values?
c What is the mode of the data?
11 The table below shows the number of cars that are garaged at each house in a
certain street each night.

Number of cars Frequency


1 9
2 6
3 2
4 1
5 1

a Show these data in a frequency histogram.


b Are the data positively or negatively skewed? Justify your answer.
12 Find the mean, median and mode of this data set: 2, 5, 6, 2, 5, 7, 8. Comment on
the shape of the distribution.

Chapter 13 • Univariate data 479


STATistics and probability
Communicating

Rich task

Cricket scores

Batting averages
The following table shows the runs scored by four cricketers who are vying for selection to the
state team.

Player Runs in the last 25 matches Mean Median Range IQR


Allan 13, 18, 23, 21, 9, 12, 31, 21, 20, 18, 14, 16, 28,
17, 10, 14, 9, 23, 12, 24, 0, 18, 14, 14, 20
Shane 2, 0, 112, 11, 0, 0, 8, 0, 10, 0, 56, 4, 8, 164, 6,
12, 2, 0, 5, 0, 0, 0, 8, 18, 0
Glenn 12, 0, 45, 23, 0, 8, 21, 32, 6, 0, 8, 14, 1, 27, 23,
43, 7, 45, 2, 32, 0, 6, 11, 21, 32
Rod 2, 0, 3, 12, 0, 2, 5, 8, 42, 0, 12, 8, 9, 17, 31, 28,
21, 42, 31, 24, 30, 22, 18, 20, 31

480 Maths Quest 10 New South Wales Australian curriculum edition Stages 5.1 and 5.2
STATistics and
STATISTICS AND PROBABILITY
probability

1 Find the mean, median, range and IQR scored for each cricketer.
2 You need to recommend the selection of two of the four cricketers. For each player, write two
points as to why you would or would not select them. Use statistics in your comments.
a Allan __________________________________________________________________________
______________________________________________________________________________
b Shane _________________________________________________________________________
  ______________________________________________________________________________
c Glenn _________________________________________________________________________
    ______________________________________________________________________________
d Rod ___________________________________________________________________________
    ______________________________________________________________________________

Bowling averages
The bowling average is the number of runs per wicket taken.
no. of runs scored
Bowling average =
no. of wickets taken
The smaller the average, the better the bowler has performed.
Brad and Dennis were competing for three bowling awards:
• Best in semifinal
• Best in final
• Best overall
The following table gives their scores.

Semifinal Final
Runs scored Wickets taken Runs scored Wickets taken
Brad 12 5 28 6
Dennis 10 4 15 3

3 Calculate the bowling averages for the following and fill in the table below.
a Semifinal
b Final
c Overall

Semifinal average Final average Overall average


Brad
Dennis

4 Explain how Dennis can have the better overall average when Brad has the better
average in both the semifinal and final.

Chapter 13 • Univariate data 481


STATistics AND
STATISTICS and PROBABILITY
probability

Code puzzle

Medical discovery
of 1928
Using technology where appropriate, calculate the required
measures of central tendency for each of the following data
sets to find the puzzle’s code.

x 24 25 26 27 28 9 9 9 10
10 15 16 18
f 10 9 8 1 0
C = mean =
A = mean = B = mode =
D = median =
E = mode =
Stem Leaf
F = mean =
1 5 3 0 1 x f
2 3 0 3 1 G = median =
3 1 0 H = mode = 7 4
4 5 8 3
9 2
10 1
11 0

I = mean = L = mode =
7 8 9 10 11 12 13 14 15 16 17 18

Stem Leaf
M = mean = N = median = O = mode =
1 5 5 5 6 7 8
2 0 0 1 3
3
12 16 19
4 0
20 17 15 P = mean =
13 32 16 T = mean =
R = median =
18 17 40 V = median =
S = mode =
19 15 16 X = mode =

25 7 9 15 25 14 10 9 17 22 7 9 13 8 14 21


10 8 16 12 11 18 9 17 16 19 9 14 8 12 8 7 7 8 14

20 23 9 22 8 17 16 20 25 14 20 8 24 8 11 20 8 12

482 Maths Quest 10 New South Wales Australian curriculum edition Stages 5.1 and 5.2
STATistics and probability

Activities
Go to assessON
Chapter opener • WorkSHEET 13.1 (doc‐5311): Univariate for questions to
video dataI (page 458) test your readiness
• The story of mathematics (eles-1852) FOR learning, your
13C Box‐and‐whisker plots
13A Measures of central tendency Digital docs (page 464) progress AS you learn
Digital docs • Activity 13‐C‐1 (doc‐5134): Constructing and your levels OF
• Activity 13‐A‐1 (doc‐5128): Mean, median boxplots achievement.
and mode (page 447) • Activity 13‐C‐2 (doc‐5135): Boxplots and www.assesson.com.au
• Activity 13‐A‐2 (doc‐5129): Practice with outliers
mean, median and mode (page 447) • Activity 13‐C‐3 (doc‐5136): Boxplots with
• Activity 13‐A‐3 (doc‐5130): Mean, median decimals
and mode in depth (page 447)
13D Comparing data sets
• SkillSHEET (doc‐5299): Finding the mean of a
Interactivity (page 468)
small data set (page 448)
• Parallel boxplots (int‐2788)
• SkillSHEET (doc‐5300): Finding the median of
Digital docs
a small data set (page 448)
• Activity 13‐D‐1 (doc‐13779): Comparing data 1
• SkillSHEET (doc‐5301): Finding the mode of a
(page 470)
small data set (page 448)
• Activity 13‐D‐2 (doc‐13780): Comparing
• SkillSHEET (doc‐5302): Finding the mean,
data 2 (page 470)
median and mode from a stem‐and‐leaf
• Activity 13‐D‐3 (doc‐13781): Comparing
plot (page 448)
data 3 (page 470)
• SkillSHEET (doc‐5303): Presenting data
• WorkSHEET 13.2 (doc‐13782): Univariate
in a frequency distribution table
data II (page 475)
(page 448)
• SkillSHEET (doc‐5304): Drawing statistical Chapter review
graphs (page 448) Interactivities (page 476)
• Word search (int‐2859)
13B Measures of spread
• Crossword (int‐2860)
Digital docs
• Sudoku (int‐3599)
• Activity 13‐B‐1 (doc‐5131): Range and
Digital docs
quartiles (page 455)
• Chapter summary (doc‐13784)
• Activity 13‐B‐2 (doc‐5132): Practice with
• Concept map (doc‐13785)
range and quartiles (page 455)
• Activity 13‐B‐3 (doc‐5133): Range and To access eBookPLUS activities, log on to
quartiles in depth (page 455) www.jacplus.com.au

Chapter 13 • Univariate data 483


STATistics and probability

Answers
CHAPTER 13 Univariate data
Exercise 13A — Measures of central tendency 11    a Frequency column: 16, 6, 4, 2, 1, 1
1    a    i 7 ii 8 iii 8 b 6.8
    b    i 6.875 ii 7 iii 4, 7 c 0–4 hours
    c    i 39.125 ii 44.5 iii No mode d 0–4 hours
    d    i 4.857 ii 4.8 iii 4.8 12    a Frequency column: 1, 13, 2, 0, 1, 8
    e    i 12 ii 12.625 iii 13.5 b
Age of emergency
2 Science: mean = 57.6, median = 57, mode = 42, 51 Maths: 15 ward patients
mean = 69.12, median = 73, mode = 84

Frequency
10
3    a    i 5.83 ii 6 iii 6
    b    i 14.425 ii 15 iii 15 5
4    a Mean = 2.5, median = 2.5
0
b Mean = 4.09, median = 3 7.5 22.5 37.5 52.5 67.5 82.5
Age
c Median
5    a 72 23
c Asymmetrical or bimodal (as if the data come from two
b 73
separate graphs).
c 70–<80
6 124.83 d 44.1
7 65–<70 e 15–<30
8    a B b B c C d D f 15–<30
9    a Mean = $32.93, median = $30 g
26 100%

Cumulative frequency (%)


24
Cumulative frequency
22
20
b Cumulative
18
16
14
Class interval Frequency frequency 12 50%
10
8
0–9 5 5 6
4
2
10–19 5 10 0 15 30 45 60 75 90
Age
20–29 5 15
h 28
30–39 3 18 i No
j Class discussion
40–49 5 23 13    a Player A mean = 34.33, Player B mean = 41.83
b Player B
50–59 3 26
c Player A median = 32.5, Player B median = 0
60–69 3 29 d Player A
e Player A is more consistent. One large score can distort the
70–79 1 30 mean.
14    a Frequency column: 3, 8, 5, 3, 1
Total 30
b 50.5
c 40 –<50
d 40 –<50
   Mean = $32.50 Median = $30
e Ogive of pulse rate
c
Cumulative frequency (%)

30 of female athletes
Cumulative frequency

Cumulative frequency

25 100%
20
20
15
15
10 50%
10
5
5
0 30 50 70
10 20 30 40 50 60 70 80 Beats per minute
Amount spent ($)
f Approximately 48 beats/min
d The mean is slightly underestimated; the median is exact. 15 A
The estimate is good enough as it provides a guide only to the 16 Check with your teacher.
amount that may be spent by future customers. 17 Answers will very. Examples given.
10    a 3 b 3, 4, 5, 5, 8
b 4, 5, 5, 5, 6 (one possible solution) c 4, 4, 5, 10, 16
c One possible solution is to exchange 15 with 20. d 2, 3, 6, 6, 12

484 Maths Quest 10 New South Wales Australian curriculum edition Stages 5.1 and 5.2
STATistics and probability

Challenge 13.1 12    a (124 000, 135 000, 148 000, 157 000, 175 000)
13, 31, 31, 47, 53, 59     b 120 140 160 180 ($×1000)
Exercise 13B — Measures of spread
1   a 15 b 77.1 c 9 13    a
2    a 7 b 7 c 8.5 d 39
3   a 3.3 kg b 1.5 kg 0 1 2 3 4 5 6 7
4 22 cm Number of sick days taken
5 0.8 by workers at factory A
6 C
Both graphs indicate that the data is slightly positively
7    a
40 skewed. However, the boxplot provides an excellent summary
Cumulative frequency

35 of the centre and spread of the distribution.


30
25 b
20
15
10 0 2 4 6 8 10 12 14
5
0
Number of sick days taken
50 55 60 65 70 75 80 by workers at factory B
Battery life (h)
Both graphs indicate that the data is slightly negatively
    b    i 62.5 ii Q1 = 58, Q3 = 67
skewed. However, the boxplot provides an excellent summary
iii 9 iv 14
of the centre and spread of the distribution.
iv 6
8 14 Histogram of number of
55
50
10 passenges on bus journeys
9

Number of journeys
45
8
Cumulative frequency

40 7
35 6
30 5
25 4
20 3
15 2
10 1
5 0
0 0 20 40 60 80 100 120 140
120130 140 150160170180190200
Class interval
IQR = 27
9    a    i Range = 23 ii IQR = 13.5
    b    i Range = 45 ii IQR = 27.5
0 20 40 60 80 100 120 140
    c    i Range = 49 ii IQR = 20
10    a 25.5 b 28 c 39 d 6 Number of passengers
e The three lower scores affect the mean but not the median or
mode. 15    a Xmin Q1 Median Q3 Xmax
11    a Men: mean = 32.3; median = 32.5; range = 38;
IQR = 14 Before 65 85 95 129 152
   Women: mean = 29.13; median = 27.5; After 66 81 87 117 134
range = 36; IQR = 13
b Typically, women marry younger than men, although the b After
spread of ages is similar.
Exercise 13C — Box‐and‐whisker plots Before
1    a 5 b 26
2    a 6 b 27 65 85 95 129 152
3    a 5.8 b 18.6 c The median weight was lower after treatment, and the range of
4    a 140 b 56 c 90 d 84 e 26 weights was reduced. The IQR was lowered from 44 to 36 kg.
5    a 58 b 31 c 43 d 27 e 7 This indicates that the weight loss program was effective.
6 B 7 C 8 D 16    a Key: 12 | 1 = 121
9    a (22, 28, 35, 43, 48)
    b 20 30 40 50 Sales Stem Leaf
12 1 5 6 9
10    a (10, 13.5, 22, 33.5, 45) 13 1 2 4
    b 0 10 20 30 40 50 Rainfall (mm)
14 3 4 8 8
11    a (18, 20, 26, 43.5, 74) 15 0 2 2 2 5 7
    b 10 30 50 70 Age 16 3 5
17 2 9
c The distribution is positively skewed, with most of the
18 1 1 1 2 3 7 8
offenders being young drivers.

Chapter 13 • Univariate data 485


STATistics and probability

b 120 140 160 180 Number sold c There are generally more cold drinks sold in summer as
shown by the higher median. The spread of data is similar as
c On most days the hamburger sales are less than 160. Over the shown by the IQR although the range in winter is greater.
weekend the sales figures spike beyond this. 8 A
9 A, B, C, D
17    a Key: 1* ∣ 7 = 17 years 10    a Cory achieved a better average mark in Science (59.25) than
Stem Leaf he did in English (58.125).
1* 7 7 8 8 8 9 9 b Cory was more consistent in English (lower IQR) than he was
2 0 0 0 1 2 2 2 2 3 3 3 3 4 4 4 in Science (higher IQR).
11    a Back street: x = 61;
2* 5 5 8 9 main road: x = 58.8
3 1 2 3 b The drivers are generally driving faster on the back street.
3* c The spread of speeds is greater on the main road as indicated
4 by the higher IQR.
12    a Nathan: mean = 15.1; Timana: mean = 12.3
4* 8 b Nathan: range = 36; Timana: range = 14
c Nathan: IQR = 15; Timana: IQR = 4
b 15 25 35 45 Age
d Timana’s lower range and IQR shows that he is the more
×
consistent player.
c The distribution is positively skewed, with first‐time mothers 13    a Machine A
being under the age of 30. There is one outlier (48) in this group.
Machine B
18 C
Exercise 13D — Comparing data sets 40 42 44 46 47 48 50 52 54 56 58 60
1    a Boys: median = 26; girls: median = 23.5 Number of Smarties in a box
b Boys: range = 32; girls: range = 53 b Machine A: mean = 49.88,
c Both sets have similar medians but the girls have a far greater Machine B: mean = 50.12
range of absenteeism than boys. c Machine B is more reliable, as shown by the lower IQR. The
2    a Morning: median = 2.45; afternoon: median = 1.6 range is greater on machine B only because of a single outlier.
b Morning: range = 3.8; afternoon: range = 5 14    a No
c The waiting time is generally shorter in the afternoon. One b Science: positively skewed, Maths: negatively skewed
outlier in the afternoon data causes the range to be larger. c The science test may have been more difficult.
Otherwise the afternoon data are far less spread out. d Science: 61–70, Maths: 71–80
3    Key: 16 ∣ 1 = 1.61 m 15 Answers will vary. Check with your teacher.
16    a Key: 2 | 3 = 2.3 hours
Leaf Stem Leaf Leaf Stem Leaf
Boys Girls Group A Group B
997 15 1256788 8 7 3 1 7 8
98665540 16 4467899 9 5 1 2 0 1 2 4 5 5 8 8
4421 17 0 8 7 5 4 2 2 3 2 2 2 4 5 5 5 6 8
4    a Ford: median = 15; Holden: median = 16 7 5 4 2 2 2 4 2
b Ford: range = 26; Holden: range = 32 5
c Ford: IQR = 14; Holden: IQR = 13.5 2 2 6
d Ford
Holden
b Five‐point summary
Group A: 13 27 36 43 62
0 5 10 15 2025303540
Group B: 17 23 30 35 42
5    a Brisbane Lions Group B
b Brisbane Lions: range = 63; Group A
Hours
Sydney Swans: range = 55
c Brisbane Lions: IQR = 40; 10 20 30 40 50 60 70
Nouns
Sydney Swans: IQR = 35
c Student comparison
6    a Girls
Boys Statistics Group A Group B
1.4 1.5 1.6 1.7 1.8 1.9 Five‐point summary 13 27 36 43 62 17 23 30 35 42
Height
x 35.85 hours 28.95 hours
b Boys: median = 1.62; girls: median = 1.62
c Boys: range = 0.36; girls: range = 0.23 Range 49 hours 25 hours
d Boys: IQR = 0.14; girls: IQR = 0.11
e Although boys and girls have the same median height, the IQR 16 hours 12 hours
spread of heights is greater among boys as shown by the
greater range and interquartile range. d Student decision, justifying answer
7    a Summer: range = 23; winter: range = 31 e Class discussion
b Summer: IQR = 14; winter: IQR = 11

486 Maths Quest 10 New South Wales Australian curriculum edition Stages 5.1 and 5.2
STATistics and probability

Challenge 13.2 e Median = 30


32% f Estimates from parts c and e were fairly accurate.
g Yes, they were fairly close to the mean and median of the raw
Chapter review data.
Fluency 8    a HJ Looker: median = 5;
1    a Mean = 11.55; median = 10; mode = 8 b Hane and Roarne: median = 6
b Mean = 36; median = 36; mode = 33, 41 c HJ Looker
c Mean = 72.18; median = 72; mode = 72 d Hane and Roarne had a higher median and a lower spread and
2    a 6 b 6 c 20 so they appear to have performed better.
3    a 4 b 8.5 9    a English: mean = 70.25; Maths: mean = 69
4    a Year 10 b English: range = 53; Maths: range = 37
c Kloe has performed more consistently in Maths as the range
Year 8 for her Maths marks is lower.
10    a Yes
0 10 20 30 40 50 60 70
b Yes. Both are 3.
b Year 8: mean = 26.83, median = 27, range = 39, c 3
IQR = 19
11    a
Year 10: mean = 40.7, median = 39.5, range = 46, 9
IQR = 20 8
c The typing speed of Year 10 students is about 13 to 14 wpm 7

Frequency
faster than that of Year 8 students. The spread of data in Year 8 6
5
is slightly less than in Year 10.
4
5    a 20 b 24 c 8 3
6    a Key: 3* ∣ 9 = 3.9 kg 2
1
Stem Leaf
1 2 3 4 5
3* 9 Number of cars
4 0 0 2 3
4* 5 6 7 8 8 b Positively skewed — a greater number of scores is distributed
5 0 3 at the lower end of the distribution.
5* 5 8 8 9 12 Mean = 5, median = 5, mode = 2 and 5. The distribution is
6 1 2 2 positively skewed and bimodal.
6* 8
Communicating — Rich task
b (3.9, 4.4, 4.9, 5.85, 6.8) 1  
c 3.5 4.5 5.5 6.5 kg
Runs in the last 25
Problem solving Player matches Mean Median Range IQR
7    a Mean = 32.03; median = 29.5
b Allan 13, 18, 23, 21, 9, 12, 16.76 17 31 8.5
Class interval Frequency 31, 21, 20, 18, 14,
0–9 2 16, 28, 17, 10, 14, 9,
7 23, 12, 24, 0, 18, 14,
10–19
14, 20
20–29 6
30–39 6 Shane 2, 0, 112, 11, 0, 0, 8, 17.04 4 164 10.5
40–49 3 0, 10, 0, 56, 4, 8, 164,
6, 12, 2, 0, 5, 0, 0, 0,
50–59 3 8, 18, 0
60–69 3
Total 30 Glenn 12, 0, 45, 23, 0, 8, 21, 16.76 12 45 25.5
32, 6, 0, 8, 14, 1, 27,
c Mean = 31.83 23, 43, 7, 45, 2, 32, 0,
d 6, 11, 21, 32
30
Cumulative frequency

25
20 Rod 2, 0, 3, 12, 0, 2, 5, 8, 16.72 17 42 25
15 42, 0, 12, 8, 9, 17, 31,
10 28, 21, 42, 31, 24, 30,
5 22, 18, 20, 31
0
10 20 30 40 50 60 70
Age

Chapter 13 • Univariate data 487


STATistics and probability

2    a Allan: has a similar mean and median, which shows he was Would recommend Rod if the team needs someone who is fairly
fairly consistent. The range and IQR values are low indicating consistent but can score quite well at times and the rest of the time
that his scores remain at the lower end with not much has a better median than Glenn.
deviation for the middle 50%.
b Shane: has the best average but a very low median indicating 3
Semifinal Final Overall
his scores are not consistent. The range is extremely high and average average average
the IQR very low in comparison showing he can score very
well at times but is not a consistent scorer. Brad 2.4 4.67 3.64
c Glenn: has a similar mean to Allan and Rod but a lower
median, indicating his scores are sometimes high but Dennis 2.5 5 3.57
generally are lower than the average. The range and IQR show
a consistent batting average and spread with only a few higher Brad was best in the semifinal.
scores and some lower ones. Brad was best in the final.
d Rod: Has a similar mean and median which shows he was Dennis was best overall.
a consistent player. The range and IQR show a consistent 4 In the final, wickets were more costly than in the semifinal. Brad
batting average and spread. therefore conceded many runs in getting his six wickets. This
Players to be selected: affected the overall mean. In reality Brad was the most valuable
Would recommend Allan if the team needs someone with very player overall, but this method of combining the data of the two
consistent batting scores every game but no outstanding runs. matches led to this unexpected result.
Would recommend Shane if the team needs someone who might Code puzzle
score very high occasionally but in general fails and doesn’t score
Alexander Fleming discovers penicillin — the first antibiotic.
many runs at all.
Would recommend Glenn if the team needs someone who is fairly
consistent but can score quite well at times and the rest of the time
does OK.

488 Maths Quest 10 New South Wales Australian curriculum edition Stages 5.1 and 5.2
STATistics and probability

Chapter 14

Bivariate data
WHY LEARN THIS?
Observations of two or more variables are often
recorded, for example, the heights and weights of
individuals. Studying the data allows us to investigate
whether there is any relationship between the variables,
how strong the relationship is, and whether one variable
can be effectively predicted from information about
another variable. Statistics can be applied to medical
research, sport, agriculture, sustainability, weather
forecasting and fashion trends, to name but a few fields.
The capacity to analyse data and draw conclusions is
an essential skill in a world where information is readily
available and often manipulated.

WHAT DO YOU KNOW?


1 Think List what you know about data that shows
relationships between two variables (such as height
and mass data for a group of people). Use a thinking
tool such as a concept map to show your list.
2 pair Share what you know with a partner
and then with a small group.
3 share As a class, create a large concept map that
shows your class’s knowledge of data of this type.

LEARNING SEQUENCE
14A Variables
14B Graphing bivariate data
14C Scatterplots and linear modelling
Watch this video
The story of mathematics

Searchlight ID: eles-1853


STATistics and probability

14A Variables
Univariate and bivariate data
•• Univariate data is data with one variable; for example, the heights of Year 10
students.
•• Bivariate data is data with two variables; for example, the weights of babies over a
period of time.

Variables
•• A particular characteristic of a population or sample that is being studied will usually
vary from one member of the population or sample to another. Such a characteristic
is called a variable if it can assume two or more different values.
•• Data may be classified in the following way:

Data

Qualitative Quantitative
Data can be placed in Data is numerical and
categories, e.g. favourite can be counted or measured,
songs and types of songs. e.g. weights of footballers.

Nominal Ordinal Discrete Continuous


Data is categorised Data can be placed Data can have specific Data can have any
without any order, in ordered groups, numerical values only, numerical value,
e.g. hair colour. e.g. school year level. e.g. number of goals e.g. weight and
scored in a basketball height.
match.

WORKED EXAMPLE 1
Classify the following types of data.
a Time b Shoe size c Eye colour
T HIN K W RI T E
a Time is measured in various units, such a Time measurements give
as years, days, minutes and seconds. It is quantitative, continuous data.
recorded in numbers; that is, how many there
are of these units (and any of their fractions,
so it is continuous).
b Shoes come only in set sizes. You cannot get b Shoe sizes are quantitative,
sizes in between these. discrete data.
c Eye colour is a description of the colour of c Eye colour is qualitative,
the eye. nominal data.

492 Maths Quest 10 New South Wales Australian curriculum edition Stages 5.1 and 5.2
STATistics and probability

Dependent and independent variables


•• One variable in bivariate data is generally the dependent variable, and the other
the independent variable. The dependent variable, as the name suggests, is the one
whose value depends on the other variable. The independent variable takes on values
that do not depend on the value of the other variable.
–– When data are expressed in the form of a table, generally the independent variable
is written in the first row or the first column.
–– The independent variable is placed on the x­axis and the dependent variable on
the y­axis.

WORKED EXAMPLE 2
State the dependent and the independent variable in each of the following
pairs of data.
a Age and height of a child
b Cost of bus fare and distance travelled in the bus
c Number of people at a football match and the number of drinks sold there.
T HIN K W RI T E
a Generally, a child grows taller with a Height is the dependent variable,
increasing age (height depends on age). and age the independent variable.
b The fare is more when you travel b Cost is the dependent variable, and
further (fare depends on distance). distance the independent variable.
c More drinks will be sold if more c Number of drinks sold is the
people are there (number of drinks dependent variable, while number
sold depends on number of people). of people at the match is the
independent variable.

Identifying a relationship
•• The options we have when considering two variables are as follows.
–– There is a relationship between them.
–– There is no relationship between them.
•• If there is a relationship, it can be classified as being strong, moderate or weak. The
relationship can also be regarded as positive or negative.
–– If one variable increases as the other increases, the relationship is positive.
–– If one variable increases as the other decreases, the relationship is negative.

WORKED EXAMPLE 3
For each of the following pairs of variables, indicate:
i whether you think there is a relationship between them, and if so
ii whether the relationship is positive or negative, and
iii the strength of the relationship.

Chapter 14 • Bivariate data 493


STATistics and probability

a A person’s height (h) and mass (m)


b The length of a song (l) and its position on a CD ( p)
c The speed of travel (s) and the time taken to reach a destination (t).
T HIN K W RI T E
a i Generally as a person grows taller, a There is a relationship between h
their mass changes. and m.
ii With increasing height, generally The relationship is positive.
mass also increases.
iii This change tends to be quite The relationship is quite strong.
consistent.
b There is no particular order to the b There is no relationship between the
songs. l and p.
c i As speed changes, the time taken c There is a relationship between s
to reach a destination also changes. and t.
ii With a faster speed it takes less The relationship is negative.
time to reach a destination.
iii The change tends to be consistent There is a strong negative
since travelling at twice the speed relationship.
will halve the time.

Exercise 14A Variables


INDIVIDUAL PATHWAYS
Questions: Questions: Questions:
1a–c, 2a–c, 3a–c, 4–6 1b–d, 2b–d, 3b–d, 4–6 1e–h, 2c–e, 3c–f, 4–6
Activity 14‐A‐1 Activity 14-A-2 Activity 14-A-3
Pairs of variables More pairs of variables Advanced pairs of variables
doc‐5146 doc-5147 doc-5148

Digital docs
SkillSHEET
Distinguishing FLUENCY
­qualitative from
­quantitative data 1 WE1Classify the types of data below, using two words from the following:
doc‐5327 qualitative, quantitative, nominal, ordinal, discrete and continuous.
SkillSHEET
Distinguishing discrete a The number of children in your school
from continuous data b The types of bicycles students ride to school
doc‐5328
SkillSHEET c The heights of students in your class
Determining
­independent and
d The mass of your textbooks
dependent variables e The languages you speak at home
doc‐5329
SkillSHEET f The time it takes you to travel to school
Reading scales
(how much is each
g The number of cousins each person in your
interval worth?) class has
doc‐5330
h The time it takes you to run 100 m

494 Maths Quest 10 New South Wales Australian curriculum edition Stages 5.1 and 5.2
STATistics and probability

2 WE2 State the dependent and the independent


variable in each of the following pairs of data.
a Time spent travelling to school and distance
to school
b The heart rate of a runner and the running speed
c The value of a music collection and the
number of songs in the collection
d The amount of computer memory used by a
file and the length of the file
e The cost of a second‐hand car and the age of
the car
3 WE3 For each of the following pairs of
variables, indicate:
i whether you think there is a relationship
between them and, if so,
ii whether the relationship is positive or negative, and
iii the strength of the relationship.
a The length of your foot (f) and the length
of your hair (h)
b The cost of a TV (c) and the size of its
screen (s)
c The length of a race (l) and the time taken (t)
d The size of an animal’s paw (p) and the
size of the animal (a)
e The height of a person (h) and their arm span (a)
f The time it takes a train to travel from A to B (t) and the time it takes a bus to
travel from A to B (b)
UNDERSTANDING
4 It was a hot day at the beach. The
lifesavers noticed there was an increase
of 20% in the number of people seeking
help with sunburn. On the same day,
the café also recorded an increase of
20% in their ice‐cream sales. Since
both of these variables increased, explain
whether there is a relationship between
the two, and if so, the strength of the
relationship.
REASONING
5 Explain the difference between discrete and
continuous data. REFLECTION
6 Explain whether data can be: How can we know if there
is a relationship between
a nominal and discrete
two variables?
b ordinal and continuous.

Chapter 14 • Bivariate data 495


STATistics and probability

14B  Graphing bivariate data


•• When a set of bivariate data is collected, it is generally first organised
into a table.
•• A graph is then drawn to display the trends in the data.
•• This visual representation shows at a glance whether there is a relationship
between the two variables and, if so, the strength of the relationship.

Drawing column graphs from data tables


•• If both variables are quantitative, it is appropriate to draw a column graph.
•• The independent variable should be shown on the x­axis and the dependent variable
on the y­axis.

WORKED EXAMPLE 4
The table shows data collected by measuring the length of the shadow of a
stick every 12 hour from noon until 4.00 pm.
Time(t) Length of shadow (l) (cm)
Noon 20
12.30 pm 32
1.00 pm 48
1.30 pm 60
2.00 pm 76
2.30 pm 88
3.00 pm 100
3.30 pm 112
4.00 pm 128
Draw a column graph to display the data.

T HIN K W RI T E /DR AW
1 The length of the shadow 130
Shadow length over time
depends on the time 120
of day, so time is the 110
Length of shadow (cm)

independent variable, 100


90
so it must be placed on 80
the x­axis. Draw up axes 70
with time on the x­axis 60
50
and length of shadow 40
on the y­axis. Choose a 30
suitable scale. 20
10
2 Draw columns for each 0
reading. Noon 12.30 1.00 1.30 2.00 2.30 3.00 3.30 4.00
pm pm pm pm pm pm pm pm
Time

496 Maths Quest 10 New South Wales Australian curriculum edition Stages 5.1 and 5.2
STATistics and probability

Using a column graph to create a scatterplot


•• Once a column graph has been drawn, the height of each column can be used to plot
points of the bivariate data.
•• Graphs that display the data as a set of points are called scatterplots.

WORKED EXAMPLE 5
a Use the column graph showing length of a shadow over time from
Worked example 4 to create a scatterplot of the bivariate data.
b Comment on the trend shown by the scatterplot.

T HIN K W RI T E /DR AW
a 1 Place dots at the a 130 Shadow length over time
tops of the columns 120
to represent the 110
Length of shadow (cm)

length of the 100


90
shadow at that time. 80
70
60
50
40
30
20
10
0
Noon 12.30 1.00 1.30 2.00 2.30 3.00 3.30 4.00
pm pm pm pm pm pm pm pm
Time

2 Erase the columns, 130 Shadow length over time


leaving the dots on 120
the graph. 110
Length of shadow (cm)

100
90
80
70
60
50
40
30
20
10
0
Noon 12.30 1.00 1.30 2.00 2.30 3.00 3.30 4.00
pm pm pm pm pm pm pm pm
Time

b Examine the points b The scatterplot shows a strong, positive relationship


on the graph, looking between the time of the reading and the length of the
for trends. As time shadow.
increases, the length of
the shadow increases.

Chapter 14 • Bivariate data 497


STATistics and probability

•• Note that this trend applies only to the data shown. It is obvious that the trend in
this case will not continue much longer, as the sun will soon set and there will be
no shadow. Would the trend be the same if the experiment was conducted in the
morning? It is important to understand that the trends of data collected from an
experiment may change under different circumstances.
•• Even when we do not know what the particular variables represent, we can still
determine the type of relationship between them.

WORKED EXAMPLE 6
The column graph shows the relationship between two variables, p and m.
a Produce a scatterplot from the column graph.
b Comment on the trend shown by the scatterplot.
m

0
p

T HIN K W RI T E /DR AW
a Place dots at the tops a
of the columns then
erase the columns,
leaving the dots on
the graph.
m

0
p

b Examine the points b The scatterplot shows a negative relationship. As


on the graph, the value of p increases, the value of m generally
looking for trends. decreases. The points do not all lie on a straight line.
As the value of p The negative relationship is fairly strong.
increases, the value
of m generally
decreases.

498 Maths Quest 10 New South Wales Australian curriculum edition Stages 5.1 and 5.2
STATistics and probability

WORKED EXAMPLE 7
Consider this scatterplot, which shows the
number of sick days taken in a year by 10

Number of sick days


12
employees and the number of children they
have. 8
Write two questions that an employer may be
4
hoping to answer from this data.
0
T HIN K W RI T E 0 2 4 6
Number of children
Think of questions that an Do those employees
employer would like to know with more children
about the number of sick days have more sick days?
taken by her staff. If a staff member has more than two children,
are they likely to exceed their allocated number
of sick days?

Exercise 14B Graphing bivariate data


INDIVIDUAL PATHWAYS
Questions: Questions: Questions:
1–7 1–8, 10 1–10
Activity 14‐B‐1 Activity 14‐B‐2 Activity 14‐B‐3
Scatterplots More scatterplots Advanced scatterplots
doc‐5149 doc‐5150 doc‐5151

FLUENCY
1 WE4 This table shows the number of goals scored in a ball game. Digital docs
SkillSHEET
Time after start (min) 0 10 20 30 40 50 60 70 80 90 100 Reading a column
graph
Number of goals 0 5 12 17 19 25 28 30 35 38 42 doc‐5331
SkillSHEET
Draw a column graph to display the data. Reading line graphs
doc‐5332
2 WE5 a Use the column graph you
SkillSHEET
created in Question 1 to create a 70 Plotting coordinate
scatterplot. points
doc‐5333
60
b Comment on the trend shown by
the scatterplot. 50
3 WE6 This column graph shows the
relationship between two variables r 40
b

and b.
30
a Produce a scatterplot from the
column graph. 20
b Comment on the trend shown by
10
the scatterplot.
0
1 2 3 4 5 6 7 8 9 10 11 12
r

Chapter 14 • Bivariate data 499


STATistics and probability

4 WE7 Write one question that could be asked for each of the following sets of
bivariate data collected.
a Length of newborn baby and height of father
b Length of formal education and income in a job
c Age and exercise performed.

UNDERSTANDING
5 Each point on the scatterplot at right shows the time
(in weeks) spent by a person on a healthy diet and the

Loss in mass (kg)


corresponding mass lost (in kg).
Study the scatterplot and state whether each of the
following is true or false.
a The number of weeks that the person stays on a diet
is the independent variable. Number of weeks
b The y‐coordinates of the points represent the time
spent by a person on a diet.
c There is evidence to suggest that the longer the person stays on a diet, the greater
the loss in mass.
d The time spent on a diet is the only factor that contributes to the loss in mass.
e The relationship between the number
of weeks on the diet and the number of
kilograms lost is positive.
6 MC The scatterplot that best represents
the relationship between the amount
of water consumed daily by a certain
household for a number of days in summer
and the daily temperature is:
A
Temperature (°C)

Water usage (L)

B
Water usage (L)

Temperature (°C)

C D E
Temperature (°C)

Water usage (L)


Water usage (L)

Temperature (°C) Water usage (L) Temperature (°C)

500 Maths Quest 10 New South Wales Australian curriculum edition Stages 5.1 and 5.2
STATistics and probability

7 MC The scatterplot at right shows the number 1300


of sides and the sum of interior angles for a 1200
number of polygons. 1100
1000

Sum of angles (°)


Which of the following statements is not 900
true? 800
700
A The relationship between the number of 600
sides and the angle sum of the polygon is 500
400
perfectly linear. 300
B The increase in the number of sides causes 200
the increase in the size of the angle sum. 0
3 4 5 6 7 8 9 10
C The number of sides depends on the sum Number of sides
of the angles.
D The angle sum is the dependent variable.
E The relationship between the two variables is positive.
8 MC After studying a non‐linear scatterplot, it was concluded that there was
evidence that the greater the level of one variable, the smaller the level of the other
variable. The scatterplot must have shown a:
A strong, positive relationship
B strong, negative relationship
C moderate, positive relationship
D moderate, negative relationship
E weak, positive relationship.
9 MC In which of the following is no relationship evident between the variables?
A y B y C y

x
x x

D y E y

x x

REASONING
10 Give an example of a situation where the scatterplot may look like the ones below. Digital doc
WorkSHEET 14.1
a y b y
doc‐5334

REFLECTION
Why is a graph an important
tool for analysing data?
0 x 0 x

Chapter 14 • Bivariate data 501


STATistics and probability

Challenge 14.1

14C  Scatterplots and linear modelling


•• A scatterplot is a graph that shows whether there is a relationship between two
variables.
Interactivity •• Correlation is a descriptive measure of the relationship between two variables.
Scatterplots
int‐2789 •• Correlation describes the strength, the direction and the form of the relationship
between the two variables.
•• The closer the points are to a straight line (form), the stronger the correlation
between the two variables. The strength is described as weak, moderate or strong.
•• If the points on a scatterplot have a generally positive slope, the relationship has a
positive direction. If the slope is negative, the direction is negative.

Strong positive linear Weak positive linear No correlation


correlation correlation

Weak negative linear Strong negative linear


correlation correlation

502 Maths Quest 10 New South Wales Australian curriculum edition Stages 5.1 and 5.2
STATistics and probability

WORKED EXAMPLE 8
Data were collected to investigate whether the outside
temperature is related to the number of people preferring to
spend time in a recreation room at an island resort.
Temperature (°C) 25 33 30 22 15 18 27 22 28 20
People in room 26 18 19 31 46 40 20 36 31 42

a Propose a question that this set of data might be able to


answer.
b Draw a scatterplot of the data.
c Describe the trend shown by the scatterplot.
d What advice would you give the management regarding the use of the room?

T HIN K W RI T E /DR AW
a Examine the data and consider a a Does the outside temperature have an influence on the
question that could be answered. number of people spending time in the recreation room?
b The temperature outside is the b Outside temperature and
independent variable, so it should people in room
50
be placed on the x­axis. Choose
People in room

a suitable scale for both axes and 40


plot the points. 30

20

10

0
10 20 30 40 50
Temperature outside (°C)

c Look at the direction and strength c The general trend is a negative one. As the temperature
of the plotted points. increases, fewer people spend time in the recreation room.
The relationship between the two variables is moderate to
strong.
d Provide useful tips for the d It seems that as the temperature increases, more people
management. prefer to spend their time outside. A suggestion could then
be to provide more activities outside for the guests to enjoy
at these times.

Making predictions
•• If the points on a scatterplot appear to lie fairly closely distributed in a linear pattern,
a straight line can be drawn through the data. The line can then be used to make
predictions about the data.
•• A line of best fit is a line on a scatterplot which is positioned so that it is as close as
possible to all the data points.
•• A line of best fit is used to generalise the relationship between two variables.

Chapter 14 • Bivariate data 503


STATistics and probability

Lines of best fit by eye


•• A line of best fit can be drawn on a scatterplot by eye. This means that a line is
positioned so that there is an equal number of points above and below the line.

WORKED EXAMPLE 9
Consider the scatterplot from Worked example 8. Outside temperature and
a Draw the line of best fit. 50
people in room
b Use the line of best fit to predict:
i the temperature outside when 22 people are in the room

People in room
40
ii the number of people in the room when the outside
30
temperature is 16 °C
iii the number of people in the room when the outside 20
temperature is 50 °C.
10
c Comment on your answers from part b.
0
10 20 30 40 50
Temperature outside (°C)
T HIN K W RI T E /DR AW
a Align a ruler to draw a line with roughly the a Outside temperature and
same number of points above and below the people in room
50
line. The line does not necessarily have to pass
People in room

through any of the points. 40


Line of best fit
30

20

10

0
10 20 30 40 50
Temperature outside (°C)

b Note that these answers are only estimates. b Outside temperature and
people in room
50
42
People in room

40
Line of best fit
30
(i) 22
20
(ii) 16 °C
10
30 °C
0
10 20 30 40 50
Temperature outside (°C)

i Rule a horizontal line from 22 on the y­axis i When 22 people are in the room, the outside
to meet the line of best fit. From that point, temperature is about 30 °C.
draw a line vertically to the x­axis and read
this value.

504 Maths Quest 10 New South Wales Australian curriculum edition Stages 5.1 and 5.2
STATistics and probability

ii Rule a vertical line from 16 °C on the ii When the temperature is 16 °C outside,


x‐axis to meet the line of best fit. From that about 42 people are in the room.
point, draw a line horizontally to the y­axis
and read this value. Note that because this
variable represents discrete data, the answer
must be rounded to the nearest whole
number, if necessary.
iii It seems not possible to draw a line from iii A temperature of 50 °C is probably not
50 °C to the line of best fit (even if the line possible. Even if a line could be drawn
was extended). from 50 °C to the line of best fit, the answer
would not be appropriate.
c Examine the answers for appropriateness. c The answers to parts i and ii are estimates only.
Part iii sends a warning to always check for the
appropriateness of an answer.

•• Worked example 4 provided data about the length of the shadow of a stick during
part of the day. Both variables, length and time, are quantitative and continuous.
In this case we can therefore use the line of best fit to make predictions from one
variable to the other for a continuous set of values. After all, the shadow length
doesn’t just jump from one recorded value to the next; it grows continuously in
length, even while recording is not taking place.

WORKED EXAMPLE 10
This scatterplot from Worked example 5 shows the recorded shadow length of a stick over a
time period from noon to 4 pm.
130 Shadow length over time
120
110
Length of shadow (cm)

100
90
80
70
60
50
40
30
20
10
0
Noon 12.30 1.00 1.30 2.00 2.30 3.00 3.30 4.00
pm pm pm pm pm pm pm pm
Time

a Draw the line of best fit.


b Use this line to predict:
i the length of the shadow at 2.15 pm
ii the time when the shadow length would be 50 cm
iii the time when there is no shadow.
c Comment on your answers to part b.

Chapter 14 • Bivariate data 505


STATistics and probability

T HIN K W RI T E /DR AW
a Draw a line that is as close as possible a 130 Shadow length over time
to all the points. 120
110

Length of shadow (cm)


100
90
80
70 Line of
best fit
60
50
40
30
20
10
0
Noon 12.30 1.00 1.30 2.00 2.30 3.00 3.30 4.00
pm pm pm pm pm pm pm pm
Time

b i Locate the position of 2.15 pm on b 130


Shadow length over time
the x­axis. (It is half way between 120
2 pm and 2.30 pm.) From here, 110
Length of shadow (cm)

draw a vertical line until it meets 100


90 81 cm Line of
the line of best fit. From this point, 80 best fit
draw a horizontal line to the y­axis. 70
Read the shadow length on the 60
(ii) 50 cm
50
y­axis. (Note that since length 40 (i) 2.15 pm
is a continuous data variable, the 30
answer does not have to be a whole 20 1.06 pm
number.) 10
0
Noon 12.30 1.00 1.30 2.00 2.30 3.00 3.30 4.00
pm pm pm pm pm pm pm pm
Time

ii Locate 50 cm on the y­axis. From i The shadow is about 81 cm long at 2.15 pm.
here, draw a horizontal line until it ii The shadow length would be 50 cm at about
meets the line of best fit. From this 1.06 pm.
point, draw a vertical line to the
x­axis. Read the time on the x­axis.
(Note that since time is a continuous
data variable, the answer does not
have to be a whole number.)
iii Look at the graph to see when the iii At no time within the range of the data is there no
shadow length is 0. shadow.
c Examine the answers to part b and c The answers to parts i and ii are estimated answers. You
make a comment. would think that there would be no shadow at noon.
However, this is only the case when the sun is directly
overhead. This depends on the season and on latitude.

506 Maths Quest 10 New South Wales Australian curriculum edition Stages 5.1 and 5.2
STATistics and probability

•• In providing answers for questions posed about bivariate data, it is important to be


able to justify why a particular conclusion is reached. It is not advisable to simply
state an answer without also providing a reason.

Exercise 14C Scatterplots and linear modelling


INDIVIDUAL PATHWAYS
Questions: Questions: Questions:
1–3, 5–7, 9 1–7, 9, 10, 12 1–9, 11, 12
Activity 14‐C‐1 Activity 14‐C‐2 Activity 14‐C‐3
Types of relationships More relationships Advanced relationships
doc‐5152 doc‐5153 doc‐5154

FLUENCY
1 WE8 The table below shows the marks obtained by nine students in English and
History.

English 55 20 27 33 73 18 37 51 79
History 72 37 53 74 73 44 59 55 84

a Draw a scatterplot of the data.


b Describe the trend shown by the scatterplot.
2 The table below shows the daily temperature and the number of hot pies sold at the
school canteen.

Temperature (°C) 24 32 28 23 16 14 26 20 29 21
No. of pies sold 56 20 24 60 84 120 70 95 36 63

a Draw a scatterplot of the data.


b Describe the trend shown by the scatterplot.
c What advice could you give the managers of the school canteen?
3 Draw a line of best fit for the following scatterplots, which show relationships
between the variables x and y. Remember to try to place an approximately equal
number of points above and below the line.
a y b y c y

x x x

d y e y

x x

Chapter 14 • Bivariate data 507


STATistics and probability

4 WE9 Use the given scatterplot and line of y


best fit to predict: 70
60
a the value of y when x = 45 50
40
b the value of x when y = 15. 30
20
10
0 10 20 30 40 50 60 70 80 x

5 Analyse the graph at right and use the line y

of best fit to predict: 600


500
a the value of y when the value of x is: 400
300
i 7 ii 22 iii 36 200
100
b the value of x when the value of y is: 0 5 10 15 20 25 30 35 40 45 x
i 120 ii 260 iii 480

6 WE10 A random sample of ten Year 10 students who have part‐time jobs was
selected. Each student was asked to state his/her average number of hours worked
per week and average weekly earnings (to the nearest dollar). The results are
summarised in the table below.
Hours worked 4 8 15 18 10 5 12 16 14 6
Weekly earnings ($) 23 47 93 122 56 33 74 110 78 35

a Draw a scatterplot of the data.


b Draw the line of best fit.
c Use your line of best fit to
determine:
i how many hours per week would
return $100
ii how much would be earned by
working 9 hours per week
iii the average rate of pay for the
students.
7 The data in this table show the
distance travelled by 10 cars, and the
amount of fuel (to the nearest litre)
used for their journeys.
Distance travelled (km) 52 36 83 12 44 67 74 23 56 95
Petrol used (L) 7 5 9 2 7 9 12 3 8 14
a Construct a scatterplot of the data.
b Draw the line of best fit.
c Use your line of best fit to determine:
i how much petrol would be used on a 60 ­km journey
ii how far a car could travel on 10 L of petrol
iii the average petrol consumption in km/L.

508 Maths Quest 10 New South Wales Australian curriculum edition Stages 5.1 and 5.2
STATistics and probability

UNDERSTANDING
8 As a part of her project Rachel is growing a crystal. Every day she measures
the crystal’s mass using special laboratory scales and notes it in her book.
The table below shows the results of her experiment.
Day number 1 2 3 4 5 8 9 10 11 12 15 16
Mass (g) 2.5 3.7 4.2 5.0 6.1 8.4 8.9 11.2 11.6 12.8 16.1 17.3
Measurements on days 6, 7,
13 and 14 are missing, since
these were 2 consecutive
weekends and, hence, R ­ achel
did not have a chance to
measure her crystal, which is
kept in the school laboratory.
a Construct a scatterplot of
the data, and draw in the
line of best fit.
b For her report, Rachel
would like to fill in the
missing measurements for days 6, 7, 13 and 14. Give an estimate of these
measurements.
c Rachel fell sick and couldn’t record the mass of the crystal for the last two days
of the experiment (days 17 and 18). What would you predict these masses to be?
d What is the average daily increase in mass of the crystal?

REASONING
9 Consider this scatterplot and line of best fit.
y

x
10 20 30 40 50 60 70

The line of best fit is used to predict values of y when x = 15, x = 40 and when
x = 60. Comment on these predictions.
10 This scatterplot is used to predict the value of y when x = 300.
y
500
400
300
200
100

0 100 200 300 400 500 600 700 x

Comment on the prediction.

Chapter 14 • Bivariate data 509


STATistics and probability

11 A class of Year 10 students was


asked to record the amount of
time in hours that they spent on a
History assignment and the mark
out of 100 that they received for
the assignment.
a Draw a scatterplot and a line of
best fit.
b Find the equation of the line
of best fit and use it to predict
the mark received if the
students spent 10 hours on the
assignment.
c Comment on the reasonable­
ness of the answer to part b.
Does the amount of time a
­student spends on an assignment
impact directly on the grade for that task?
Time spent 2 0.5 1.5 2.5 0.25 2 2.5 2.5 2 0.5 0.75 1.5 1 2 3 3.5 1 3 2.5 3
Mark 72 52 76 82 36 73 84 80 74 48 58 69 62 78 90 94 70 92 88 97
12 A researcher who is investigating the proposition that ‘tall mothers have tall sons’
measures the heights of twelve mothers and the heights of their adult sons. The
results are shown below.

Height of mother (cm) 185 155 171 169 170 175 158 156 168 169 179 173
Height of son (cm) 188 157 172 173 174 180 159 150 172 175 180 190
a Draw a scatterplot of this data.
b Determine the equation of the line of best fit. REFLECTION
Digital doc Since lines of best fit are not
WorkSHEET 14.2 c Predict the height of a son whose mother is really accurate, why should we
doc‐5335
172 cm. use them at all?

Challenge 14.2

510 Maths Quest 10 New South Wales Australian curriculum edition Stages 5.1 and 5.2
STATistics and probability

Chapter review
LANGUAGE

bivariate discrete predictions time series


continuous form relationship trend
correlation independent scatterplot univariate
dependent line of best fit strength variable
direction moderate strong weak

int-2862 doc-13786
int-2863 doc-13787
int-3600

FLUENCY
1 The classification of data describing the number of iPods sold during the year is:
A quantitative, discrete. B qualitative, nominal.
C qualitative, ordinal. D quantitative, continuous.
E none of these.
2 The classification of data which describe a person’s mass is:
A quantitative, discrete. B qualitative, nominal.
C qualitative, ordinal. D quantitative, continuous.
E none of these.
3 The data which describe sandwich
types at a takeaway outlet are:
A qualitative, nominal.
B quantitative, discrete.
C qualitative, ordinal.
D quantitative, continuous.
E none of these.
4 For each of the following pairs,
decide which of the variables
is independent and which is
dependent.
a Number of hours spent studying for a Mathematics test and the score on that test.
b Daily amount of rainfall (in mm) and daily attendance at the Botanical Gardens.
c Number of hours per week spent in a gym and the annual number of visits to
the doctor.
d Amount of computer memory taken by an essay and the length of the essay
(in words).

Chapter 14 • Bivariate data 511


STATistics and probability

e The cost of care in a childcare centre and attendance in the childcare centre.
f The cost of the property (real estate) and the age of the property.
g The cut‐off ENTER score for a certain course and the number of applications for
that course.
h The heart rate of a runner and the running speed.
5 This table shows the maximum and minimum daily temperatures in a city over a
one‐week period.
Day 1 2 3 4 5 6 7
Maximum (°C) 12 13 10 11 9 10 8
Minimum (°C) 3 3 2 1 0 4 2
Use the table to answer the following questions.
a What was the maximum temperature on day 3?
b Which day had the lowest minimum temperature?
c Which day was the coldest?
d Which day had the warmest overnight temperature?
e What was the temperature range (variation) on day 2?
f Which day had the smallest range of temperatures?
6 Consider this table showing the age and height of a Age (years) Height (cm)
child over 6 years.
1 80
a Which variable is the dependent variable?
b Which variable should be graphed on the x­axis? 2 85
c Draw a column graph to display the data. 3 88
d Describe the relationship between the two 4 93
variables. 5 99
7 Look at the following graph, showing the change in 6 104
size of an iceblock over time.
Size of iceblock over a period of 11 minutes
1000
Size (mm3)

800
600
400
200

0 1 2 3 4 5 6 7 8 9 10 11
Time (min)

a What are the units on the x­axis?


b What are the units on the y­axis?
c Which variable is the independent variable?
d What is the scale on the y­axis?
e What type of data is shown on:
i the x­axis ii the y­axis?
f How long did it take for half the iceblock to melt?
g Describe the relationship between the two variables.

512 Maths Quest 10 New South Wales Australian curriculum edition Stages 5.1 and 5.2
STATistics and probability

8 A researcher administers different amounts of fertiliser to a number of trial plots of


potato crop. She then measures the total mass of potatoes harvested from each plot.
When drawing the scatterplot, the researcher should graph:
A mass of harvest on the x­axis because it is the independent variable, and amount
of fertiliser on the y­axis because it is the dependent variable.
B mass of harvest on the y­axis because it is the independent variable, and amount
of fertiliser on the x­axis because it is the dependent variable.
C mass of harvest on the x­axis because it is the dependent variable, and amount
of fertiliser on the y­axis because it is the independent variable.
D mass of harvest on the y­axis because it is the dependent variable, and amount
of fertiliser on the x­axis because it is the independent variable.
E none of the above.
9 Which of the following graphs best depicts a strong negative relationship between
the two variables?
A y B y C y

x x x

D y E y

x x
10 What type of relationship is shown by the graph on the right?
A Strong positive relationship y
B Moderate positive relationship
C Moderate negative relationship
D Strong negative relationship x
E None of the above
11 State the type of relationship between x and y for each of the following scatterplots.
a y b y c y

x x x
d y e y f y

x x x
g y h y i y

x x x

Chapter 14 • Bivariate data 513


STATistics and probability

j y k y l y

x x x

my n y o y

x x x

PROBLEM SOLVING
1 Eugene is selling handbags at the local
market. During the day he keeps records
of his sales. The table below shows the
number of bags sold over one weekend
and their corresponding prices (to the
nearest dollar).

Price ($) of a bag 30 35 40 45 50 55 60 65 70 75 80


Number of bags sold 10 12 8 6 4 3 4 2 2 1 1

a Which is the dependent variable?


b What would be a relevant question that could be answered upon examination of
the data?
c Draw a scatterplot of the data.
d What type of relationship exists between the variables?
e What advice would you give Eugene after examining his records of sales?
2 The relationship between two variables x and y is y
shown in this scatterplot. 50
a Use the line of best fit to predict the value of y 45
when the value of x is: 40
i 10 35
30
ii 35.
25
b Use the line of best fit to predict the value of x 20
when the value of y is: 15
i 15 10
ii 30. 5

5 10 15 20 25 30 35 40 x

514 Maths Quest 10 New South Wales Australian curriculum edition Stages 5.1 and 5.2
STATistics and probability

3 This table shows the gestation time and the birth mass of 10 babies.
Gestation time (weeks) 31 32 33 34 35 36 37 38 39 40
Birth mass (kg) 1.1 1.5 1.8 2.1 2.2 2.5 2.8 3.1 3.1 3.4
a What type of data is this?
b Which is the dependent variable?
c Suggest a question that could be answered by this data.
d Construct a scatterplot of the data and draw in the line of best fit.
e Full term of gestation is considered to be 40 weeks, although some pregnancies
last longer. Use your line of best fit to predict the mass of a baby born after:
i 41 weeks ii 42 weeks.
f Many babies are born prematurely. Use your line of best fit to predict the mass of
a baby born at 30 weeks.
g What was the gestation time (to the nearest week) of a baby born with a birth
mass of 2.4 kg?
h Consider your question from part c. Supply an answer, indicating how you came
to this conclusion.
4 As preparation for a Mathematics test, a group of 22 students was given a revision
sheet containing 60 questions. The table below shows the number of questions from
the revision sheet successfully completed by each student and the mark, out of 100,
of that student on the test.
Number of questions 9 12 37 60 55 40 10 25 50 48 60
Test result 18 21 52 95 100 67 15 50 97 85 89
Number of questions 50 48 35 29 19 44 49 20 16 58 52
Test result 97 85 62 54 30 70 82 37 28 99 80
a Which of these variables is the dependent variable and which is the independent
variable?
b Suggest a question that could be answered by this set of data.
c Draw a scatterplot of the data.
d What type of relationship exists between the two variables?
e What answer would you give to the question you suggested in
part b? Provide evidence to support your answer.
5 For his birthday, Ari was given a small white rabbit. To monitor
the rabbit’s development, Ari decided to measure it once a week.
The table below shows the length of the rabbit for various weeks.
Week number 1 2 3 4 6 8 10 13 14 17 20
Length (cm) 20 21 23 24 25 30 32 35 36 37 39
a Construct a scatterplot of the data, and draw the line of best fit.
b As can be seen from the table, Ari did not measure his rabbit on
weeks 5, 7, 9, 11, 12, 15, 16, 18 and 19. Use your line of best fit
to predict the length of the rabbit for those weeks.
c Predict the length of the rabbit for the next 3 weeks.
d Comment on your predictions for parts b and c.

Chapter 14 • Bivariate data 515


STATistics and probability
Communicating

Rich task

Collecting, recording and analysing


data over time

Time series are often used for forecasting, that is, making predictions about the future. The
predictions made with the help of time series are always based on the assumption that the
observed trend will continue in the future.
1 Choose a subject that is of interest to you and that can be observed and measured during one
day or over the period of a week or more. (Suitable subjects are shown in the list on the next
page.)
2 Prepare a table for recording your results. Select appropriate regular time intervals. An example
is shown below.

516 Maths Quest 10 New South Wales Australian curriculum edition Stages 5.1 and 5.2
STATistics and
STATISTICS AND PROBABILITY
probability

3 Take your measurements at the selected time intervals and


record them in the table.
4 Use your data to plot the time series. You can use software
such as Excel or draw the scatterplot by hand.
5 Describe the graph and comment on its trend.
6 If appropriate, draw a line of best fit and predict the next few data
values.
7 Take the actual measurements during the hours you have
made predictions for. Compare the predictions with the actual
measurements. Were your predictions good? Give reasons.
Here are some suitable subjects for data observation and recording:
• Minimum and maximum temperatures each day for 2 weeks (use
the TV news or newspaper as resources)
• The value of a stock on the share market (e.g. Telstra, Wesfarmers and Rio Tinto)
• Your pulse over 12 hours (Ask your teacher how to do this or check on the internet.)
• The value of sales each day at the school canteen

• The number of students absent each day


• The position of a song in the Top 40 over a number of weeks

• Petrol prices each day for 2 weeks


• Other measurements (check with your teacher)
• World population statistics over time.

Chapter 14 • Bivariate data 517


STATistics AND
STATISTICS and PROBABILITY
probability

Code puzzle

What did the chewing


gum say to the shoe?
Use the two graphs to find the value of each
letter and discover the puzzle’s answer code.

Baby’s length in first year


y
110

100

90
Length (cm)

80

70

60

2 4 6 8 10 12 x
Time (months)
(4, I) (8, N) (12, Y)

Petrol price each day for a week


y
Day of Petrol Code
150 the week price letter
1 O
145
Petrol price (cents)

2 U

140 3 M
4 K
135 5 T
6 C
130
7 S
1 2 3 4 5 6 7 x
Day

65 , 142 138 140 132 135 145 137 80 100 137 132

518 Maths Quest 10 New South Wales Australian curriculum edition Stages 5.1 and 5.2
STATistics and probability

ACTIVITIES
Go to assessON
Chapter opener • SkillSHEET (doc‐5331): Reading a column for questions to
Video graph (page 499) test your readiness
• The story of mathematics (eles‐1853) • SkillSHEET (doc‐5332): Reading line FOR learning, your
graphs (page 499) progress AS you learn
14A Variables • SkillSHEET (doc‐5333): Plotting coordinate and your levels OF
Digital docs  (page 494)
points (page 499) achievement.
• Activity 14‐A‐1 (doc‐5146): Pairs of variables www.assesson.com.au
• Activity 14‐A‐2 (doc‐5147): More pairs of 14C Scatterplots and linear modelling
variables Digital docs
• Activity 14‐A‐3 (doc‐5148): Advanced pairs of • Activity 14‐C‐1 (doc‐5152): Types of
variables relationships (page 507)
• SkillSHEET (doc‐5327): Distinguishing • Activity 14‐C‐2 (doc‐5153): More
qualitative from quantitative data relationships (page 507)
• SkillSHEET (doc‐5328): Distinguishing • Activity 14‐C‐3 (doc‐5154): Advanced
discrete from continuous data relationships (page 507)
• SkillSHEET (doc‐5329): Determining • WorkSHEET 14.2 (doc‐5335): Bivariate data II
independent and dependent variables (page 510)
• SkillSHEET (doc‐5330): Reading scales (how Interactivity
much is each interval worth?) • Scatterplots (int‐2789) (page 502)
Chapter review
14B Graphing bivariate data
Interactivities (page 511)
Digital docs
• Word search (int‐2862)
• Activity 14‐B‐1 (doc‐5149): Scatterplots
• Crossword (int‐2863)
(page 499)
• Sudoku (int‐3600
• Activity 14‐B‐2 (doc‐5150): More scatterplots
Digital docs
(page 499)
• Chapter summary (doc‐13786)
• Activity 14‐B‐3 (doc‐5151): Advanced
• Concept map (doc‐13787)
scatterplots (page 499)
• WorkSHEET 14.1 (doc‐5334): Bivariate data I To access eBookPLUS activities, log on to
(page 501) www.jacplus.com.au

Chapter 14 • Bivariate data 519


STATistics and probability

Answers
Chapter 14 Bivariate data
Exercise 14A — Variables 3    a Relationship between r and b
70
1    a Quantitative, discrete b Qualitative, nominal 60
c Quantitative, continuous d Quantitative, continuous 50
e Qualitative, nominal f Quantitative, continuous 40

b
g Quantitative, discrete h Quantitative, continuous 30
20
2    a Dependent: time spent travelling to school; independent:
10
distance to school
b Dependent: heart rate of a runner; independent: running speed 0
1 2 3 4 5 6 7 8 9 10 11 12
c Dependent: value of music collection; independent: number of r
songs in collection
d Dependent: amount of computer memory used by file; b The trend is positive, but only moderate.
independent: length of file 4    a Are long babies born to tall fathers?
e Dependent: cost of second‐hand car; independent: age of car b Do people with more formal education earn more money?
3    a No relationship c Does the amount of exercise performed decrease with age?
b i There is a relationship between c and s. 5    a True b False c True
  ii Positive iii Strong d False e True
c i There is a relationship between l and t. 6 B
  ii Positive iii Strong 7 C
d i There is a relationship between p and a. 8 D
  ii Positive iii Moderate 9 A
e i There is a relationship between h and a. 10    aNumber of tickets sold and the total money raised for a
  ii Positive iii Strong number of different charity concerts.
f No relationship b Number of items sold and the price of the item.
4 There is no relationship between sunburn and ice‐cream sales. Challenge 14.1
The increase in both is influenced by the weather. No, he would have to get 108%. That would be impossible on a test.
5 Discrete data can be counted in exact values; continuous data can
be measured in a continuous scale. Exercise 14C — Scatterplots and linear modelling
6    a If data is nominal, it is qualitative in nature, so it cannot also 1    a
80
be discrete.
History

b If data is ordinal, this implies an order, which is a qualitative 40


classification. This means that it cannot also be continuous.
Exercise 14B — Graphing bivariate data 0
0 40 80
1 Goals scored in a match
50 English

b The direction of the relationship is positive; the greater


Total number of goals

40
the English mark, the greater the history mark, generally.
30 However, as the points on the scatterplot vary quite a bit from
a straight line, the relationship is only moderate.
20
2    a
Number of
pies sold

10 80

0
0 0 20 40
10 20 30 40 50 60 70 80 90 100
Temperature (°C)
Time after start of match (min)
2    a b The direction of the relationship is negative; the greater
50 Goals scored in a match
the temperature, the fewer the pies sold. The points on the
scatterplot lie close to a straight line, so the relationship is
Total number of goals

40
strong.
c The school canteen should stock more pies during cooler
30
weather, and fewer pies during hot weather.
20 Note: Best fit lines are indicated as a guide only.
3    a y b y
10

0
10 20 30 40 50 60 70 80 90 100
Time after start of match (min)
b The trend is positive and strong, indicating that as the match x x
progressed goals were scored at a steady rate.

520 Maths Quest 10 New South Wales Australian curriculum edition Stages 5.1 and 5.2
STATistics and probability

c y d y b 7.1 g, 8.1 g, 13.9 g, 14.9 g


c 17.8g, 18.8g
d About 1 g
9 Prediction of y­ values when x = 15 and x = 60 would be
considered unreliable as these x­ values are beyond the range of
x the data. For an x­ value of 40, the predicted y­ value would be
x
considered reliable as this is within the range of the data.
e y 10 This prediction would be considered unreliable, as the scattering
of the points indicates that there is no relationship between
x and y.
11    a
100
80

Marks
x 60
4    a 38    b 18 40
20
5    a i 460 ii 290   iii 130
0
b i 37    ii 24    iii 6 1 2 3 4
Time spent
6 a and b
Note: Answers may vary depending on the line of best fit drawn. b y = 16x + 43.7
mark = 16 × time spent + 43.7
130
120 10 hours = 203.7 marks
110 c This result is not reasonable as the highest mark would
100
90 be 100. The amount of hours spent should be limited to
Earnings ($)

80 under 4 hours.
70
60 12    a y
50 190
40
30
20 180

Son’s height
10
0 170
2 4 6 8 10 12 14 16 18
Hours worked
160
c i Approximately 16 hours
  ii About $55 150
iii About $6.25
7 a and b 0 150 160 170 180 190 x
Note: Answers may vary depending on the line of best fit drawn. Mother’s height
14 b Son’s height = 1.22 × mother’s height − 33.48
13
12
c 176.35 cm
11
10
Challenge 14.2
74
Petrol used (L)

9
8
7 Chapter review
6
5 Fluency
4 1 A
3
2 2 D
1 3 A
0
10 20 30 40 50 60 70 80 90 100 4 Independent Dependent
Distance travelled (km)
c i About 8 litres a Number of hours Test results
  ii About 70 km b Rainfall Attendance
iii About 7 km/L c Hours in gym Visits in the doctor
8 Note: Answers may vary depending on the line of best fit drawn. d Lengths of essay Memory taken
    a 18
17 e Attendance Cost of care
16
15
f Age of property Cost of property
14 g Number of applicants Cut‐off ENTER score
13
12 h Running speed Heart rate
Mass (g)

11
10 5    a 10°C
9
8
b Day 5
7 c Day 7
6
5
d Day 6
4 e 10°C
3 f Days 6 and 7
2
1
0
1 2 3 4 5 6 7 8 9 10111213141516
Day

Chapter 14 • Bivariate data 521


STATistics and probability

6    a Height 2    a   i 12.5 ii 49


b Age b   i 12 ii 22.5
c Age and height of child 3    a The two sets of data are quantitative and continuous.
120
b Birth mass
100 c What influence on the birth mass does the gestation period have?
d 3.6
3.4
Height (cm)

80
3.2
3.0
60 2.8
2.6

Mass (kg)
2.4
40 2.2
2.0
20 1.8
1.6
1.4
1.2
0
1 2 3 4 5 6 1.0
Age
31 32 33 34 35 36 37 38 39 40
d The relationship is positive and strong. As the child grows Weeks
older his/her height also increases. e   i 3.7 kg ii 4 kg
7    a Minutes b mm3 f 1 kg
g 36 weeks
c Time d 1 unit = 100 mm3
h During weeks 36 to 40 of the gestation period, the birth
e   i Quantitative, continuous
mass increased about 0.3 kg per week. This is supported by
ii Quantitative, continuous
readings from the line of best fit.
f 3 minutes
4    a The test result is the dependent variable, while the number of
g The relationship is negative; as time increases the size
questions is the independent variable.
of the ice block decreases. It is moderately strong, but not
b Does completing more of these revision questions contribute
linear.
to a higher test mark?
8 D
c
9 D 100
10 C 90
80
11    a Positive, perfectly linear 70
b No relationship
Test result

60
c Negative, moderate 50
d Positive, strong 40
30
e No relationship 20
f Positive, moderate 10
g Negative, perfectly linear 0
5 10 15 20 25 30 35 40 45 50 55 60
h Negative, moderate Number of questions
i Negative, weak d The relationship is positive, and moderately strong.
j Positive, moderate e There is evidence to suggest that completing more revision
k Positive, moderate questions is beneficial, since the relationship is positive and
l Negative, moderate moderately strong.
m Negative, strong 5    a L
n Positive, weak 39
38
o Positive, moderate 37
36
Problem solving 35
34
1    a Number of bags sold 33
32
Length (cm)

b Does the number of bags sold depend on the price? 31


30
c 29
28
12 27
26
11 25
10 24
Number of bags sold

23
9 22
8 21
7 20
6 0 1 2 3 4 5 6 7 8 9 1011121314151617181920 n
5 Week
4
3 b 25 cm, 27 cm, 29 cm, 31 cm, 33 cm, 36 cm, 37 cm, 39 cm, 40 cm
2 c 42 cm, 43 cm, 44 cm
1
d The predictions for part b are quite reliable, as they have been
30 35 40 45 50 55 60 65 70 75 80 made within the limits of the data. The predictions in part c
Cost ($)
for the 3 weeks beyond the upper limit of the data would not
d As the price increases, the number of bags sold decreases. be considered reliable.
This means that the relationship is negative. The points
Communicating — Rich task
vary quite a bit from a straight line, so this indicates that the
relationship is moderately strong. Answers will vary; teacher to check.
e Since the cheaper bags sell better, have a greater stock of Code puzzle
them than the more‐expensive bags. I’m stuck on you.

522 Maths Quest 10 New South Wales Australian curriculum edition Stages 5.1 and 5.2
STATistics and probability

Chapter 15

Statistics in
the media
WHY LEARN THIS?
Statistics, when used properly, can be an invaluable aid
to good decision making. However, deliberate distortion
of the data or meaningless pictures can be used to
support almost any claim or point of view. Whenever
you read an advertisement, hear a news report or are
given some data by a friend, you need to have a healthy
degree of scepticism about the reliability of the source
and nature of the data presented.

WHAT DO YOU KNOW?


1 Think List what you know about how the
media reports data. Use a thinking tool such
as a concept map to show your list.
2 pair Share what you know with a partner
and then with a small group.
3 share As a class, create a thinking tool such
as a large concept map that shows your class’s
knowledge of how the media reports data.

LEARNING SEQUENCE
15A Populations and samples
15B Primary and secondary data
15C Evaluating inquiry methods and statistical reports
15D Statistical investigations
Watch this video
The story of mathematics

Searchlight ID: eles-1854


STATistics and probability

15A Populations and samples


Populations
•• The term population refers to a complete set of individuals, objects or events
belonging to some category.
•• When data are collected from a whole population, the process is known as a census.
–– It is often not possible, nor cost‐effective, to conduct a census.
–– For this reason, samples have to be selected carefully from the population.
A sample is a subset of its population.

WORKED EXAMPLE 1
List some of the problems you might encounter in trying to collect data on
the following populations.
a The life of a mobile phone battery
b The number of possums in a local area
c The number of males in Australia
d The average cost of a loaf of white bread

T HIN K W RI T E
For each of these scenarios,
consider how the data
might be collected, and the
problems in obtaining this
data.
a The life of a mobile a The life of a mobile phone battery cannot be
phone battery measured until it is dead. The battery life also
depends on how the phone is used, and how many
times it has been recharged.
b The number of possums b It would be almost impossible to find all the
in a local area possums in a local area in order to count them.
The possums also may stray into other areas.
c The number of males c The number of males in Australia is constantly
in Australia changing. There are births and deaths every second.
d The average cost of a d The price of one particular loaf of white bread
loaf of white bread varies widely from one location to another.
Sometimes the bread is on ‘Special’ and this
would affect the calculations.

Samples
•• Surveys are conducted using samples. Ideally the sample should reveal
generalisations about the population.
•• A random sample is generally accepted as being an ideal representation of the
population from which it was drawn. However, it must be remembered that different
random samples from the same population can produce different results. This means

526 Maths Quest 10 New South Wales Australian curriculum edition Stages 5.1 and 5.2
STATistics and probability

that we must be cautious about making predictions about a population, as results


from surveys conducted using samples may vary.
•• A sample size must be sufficiently large. As a general rule, the sample size should be
about!N, where N is the size of the population. It is a misconception that a larger
sample will produce a more reliable prediction of the characteristics of its population.

WORKED EXAMPLE 2
A die was rolled 50 times and the following results were obtained.
6 5 3 1 6 2 3 6 2 5 3 4 1 3 2 6 4 5 5 4 3 1 2 1 6 4 5 2
3 6 1 5 3 3 2 4 1 4 2 3 2 6 3 4 6 2 1 2 4 2.
a Determine the mean of the population (to 1 decimal place).
b A suitable sample size for this population would be 7(!50 ≈ 7.1).
i Select a random sample of 7 scores, and determine the mean of these scores.
ii Select a second random sample of 7 scores, and determine the mean of these.
iii Select a third random sample of 20 scores, and determine the mean of these.
c Comment on your answers to parts a and b.

T HIN K W RI T E
a Calculate the mean by first finding the sum of a Population mean
all the scores, then dividing by the number of Σx
=
scores (50). n
169
=
50
= 3.4
b i Use a calculator to randomly generate b The 7 scores randomly selected are numbers
7 scores from 1 to 50 17, 50, 11, 40, 48, 12, 19 in the set of 50 scores.
Relate these numbers back to the scores, These correspond to the scores 4, 2, 3, 3,
then calculate the mean. 2, 4, 5.
23
The mean of these scores = = 3.3.
7
ii Repeat b i to obtain a second set of Ignore the second and third attempts to select 7
7 randomly selected scores. random numbers because of repeated numbers.
This second set of random numbers The second set of 7 scores randomly selected
produced the number 1 twice. Try again. is numbers 16, 49, 2, 42, 31, 11, 50 of the set
Another attempt produced the number of 50. These correspond to the scores 6, 4, 5,
14 twice. Try again. 6, 1, 3, 2.
A third attempt produced 7 different 27
The mean of these scores = = 3.9.
numbers. This set of 7 random numbers 7
will then be used to, again, calculate the
mean of the scores.
iii Repeat for a randomly selected 20 scores. The set of 20 randomly selected numbers
produced a total of 68.
68
Mean of 20 random scores = = 3.4
20

Chapter 15 • Statistics in the media 527


STATistics and probability

c Comment on the results. c The population mean is 3.4.


The means of the two samples of 7 are 3.3 and
3.9. This shows that, even though the samples
are randomly selected, their calculated means
may be different.
The mean of the sample of 20 scores is 3.4. This
indicates that by using a bigger sample the result
is more accurate than those obtained with the
smaller samples.

To sample or to conduct a census?


•• The particular circumstances determine whether data are collected from a population,
or from a sample of the population. For example, suppose you collected data on
the height of every Year 10 student in your class. If your class was the only Year
10 class in the school, your class would be the population. If, however, there were
several Year 10 classes in your school, your class would be a sample of the Year 10
population.
•• Worked example 2 showed that different random samples can produce different
results. For this reason, it is important to acknowledge that there could be some
uncertainty when using sample results to make predictions about the population.

WORKED EXAMPLE 3
For each of the following situations, state whether the information was obtained by census
or survey. Justify why that particular method was used.
a A roll call is conducted each morning at school to determine which students are absent.
b TV ratings are collected from a selection of viewers to discover the popular TV shows.
c Every hundredth light bulb off an assembly production line is tested to determine the life
of that type of light bulb.
d A teacher records the examination results of her class.

T HIN K W RI T E
a Every student is recorded a This is a census. If the roll call only applied to a sample of the
as being present or absent students, there would not be an accurate record of attendance at
at the roll call. school. A census is essential in this case.
b Only a selection of the b This is a survey. To collect data from the whole viewer population
TV audience contributed to would be time‐consuming and expensive. For this reason, it is
this data. appropriate to select a sample to conduct the survey.
c Only 1 bulb in every c This is a survey. Light bulbs are tested to destruction (burn‐out)
100 is tested. to determine their life. If every bulb was tested in this way, there
would be none left to sell! A survey on a sample is essential.
d Every student’s result is d This is a census. It is essential to record the result of every student.
recorded.

528 Maths Quest 10 New South Wales Australian curriculum edition Stages 5.1 and 5.2
STATistics and probability

Exercise 15A Populations and samples


INDIVIDUAL PATHWAYS
Questions: Questions: Questions:
1–3, 5, 7, 10 1–4, 6, 8, 11 1–3, 5, 7, 9–11
Activity 15‐A‐1 Activity 15‐A‐2 Activity 15‐A‐3
Populations and More populations and In depth populations and
samples samples samples
doc‐5155 doc‐5156 doc‐5157

FLUENCY
1 WE1 List some of the problems you might encounter in trying to collect data from
Digital docs
the following populations. SkillSHEET
Determining suitability
a The life of a laptop computer battery. of questions for a
b The number of dogs in your neighbourhood survey
doc‐5337
c The number of fish for sale at the fish markets SkillSHEET
d The average number of pieces of popcorn Finding proportions
doc‐5338
in a bag of popcorn SkillSHEET
Distinguishing between
2 WE2 The data below show the results of the rolled die from Worked example 2. types of data
doc‐5339
6 5 3 1 6 2 3 6 2 5 3 4 1 3 2 6 4 5 5 4 3 1 2 1 6 4 5 SkillSHEET
2 3 6 1 5 3 3 2 4 1 4 2 3 2 6 3 4 6 2 1 2 4 2 Reading bar graphs
doc‐5340
The mean of the population is 3.4. Select your own samples for the following SkillSHEET
Determining
questions. independent and
a Select a random sample of 7 scores, and determine the mean of these scores. dependent variables
doc‐5341
b Select a second random sample of 7 scores, and determine the mean of these.
c Select a third random sample of 20 scores, and determine the mean of these.
d Comment on your answers to parts a, b and c.
3 WE3 In each of the following scenarios, state whether the information was obtained
by census or survey. Justify why that particular method was used.
a Seating for all passengers is recorded for each aeroplane flight.
b Movie ratings are collected from a selection of viewers to discover the best
movies for the week.
c Every hundredth soft drink bottle off an assembly production line is measured to
determine the volume of its contents.
d A car driving instructor records the number of hours each learner driver has spent
driving.
4 For each of the following, state whether a census or a survey has been used.
a Two hundred people in a shopping centre are asked to nominate the supermarket
where they do most of their grocery shopping.
b To find the most popular new car on the road, 500 new car buyers are asked what
make and model they purchased.
c To find the most popular new car on the road, data are obtained from the transport
department.
d Your Year 10 Maths class completed a series of questions on the amount of maths
homework for Year 10 students.

Chapter 15 • Statistics in the media 529


STATistics and probability

UNDERSTANDING
5 To conduct a statistical investigation, Gloria needs to obtain information from
630 students.
a What size sample would be appropriate?
b Describe a method of generating a set of random numbers for this sample.
6 A local council wants the opinions of its residents regarding its endeavours to
establish a new sporting facility for the community. It has specifically requested all
residents over 10 years of age to respond to a set of on‐line questions.
a Is this a census or a survey?
b What problems could you encounter collecting data this way?

7 A poll was conducted at a school a few days before the election for Head Boy and
Head Girl. After the election, it was discovered that the polls were completely
misleading. Explain how this could have happened.
8 A sampling error is said to occur when results of a sample are different from those
of the population from which the sample was drawn. Discuss some factors which
could introduce sampling errors.
REASONING
9 Since 1961, a census has been conducted in Australia every 5 years. Some people
object to the census on the basis that their privacy is being invaded. Others say that
the expense involved could be directed to a better cause. Others say that a sample
could obtain statistics which are just as accurate. What are your views on this?
Justify your statements.
10 Australia has a very small population compared with other countries like China and
India. These are the world’s most populous nations, so the problems we encounter
in conducting a census in Australia would be insignificant compared with those
encountered in those countries. What different problems would authorities come
across when conducting a census there?
11 The game of Lotto involves picking the same 6 numbers in the range 1 to 45 as has
been randomly selected by a machine containing 45 numbered balls. The balls are
mixed thoroughly, then 8 balls are selected representing the 6 main numbers, plus 2
extra numbers, called supplementary numbers.

530 Maths Quest 10 New South Wales Australian curriculum edition Stages 5.1 and 5.2
STATistics and probability

Here is a list of the number of times each number had been drawn over a period
of time, and also the number of weeks since each particular number has been
drawn.

NUMBER OF WEEKS SINCE NUMBER OF TIMES EACH NUMBER


EACH NUMBER DRAWN DRAWN SINCE DRAW 413
1 2 3 4 5 6 7 8 1 2 3 4 5 6 7 8

1 5 2 1 1 7 - 4 246 238 244 227 249 241 253 266


9 10 11 12 13 14 15 16 9 10 11 12 13 14 15 16

3 3 1 5 5 7 - 4 228 213 250 233 224 221 240 223


17 18 19 20 21 22 23 24 17 18 19 20 21 22 23 24

9 - 9 2 2 12 10 8 217 233 240 226 238 240 253 228


25 26 27 28 29 30 31 32 25 26 27 28 29 30 31 32

5 11 17 2 3 3 - 22 252 239 198 229 227 204 230 226


33 34 35 36 37 38 39 40 33 34 35 36 37 38 39 40

4 3 - 1 12 - 6 - 246 233 232 251 222 221 219 259


41 42 43 44 45 41 42 43 44 45

6 1 7 - 31 245 242 237 221 224

If these numbers are randomly chosen, explain the REFLECTION


differences shown in the tables. A well‐known saying about Digital doc
statistics is: Statistics means WorkSHEET 15.1
never having to say you’re doc‐5342

certain. What does this


saying mean?

15B  Primary and secondary data


Primary data
•• Primary data collection involves collecting data yourself.
•• This means that you have ownership of the data, and no one else has access to the
data until it is released or published.
•• A variety of methods for collecting primary data are used. These include observation,
measurement, survey, experiment or simulation.

Observation
•• Observation involves recording the behavioural
patterns of people, objects and events in a
systematic manner.
•• The data are collected as a disguised observation
(respondents are unaware they are being
observed) or undisguised observation (the
respondent is aware). Video surveillance
cameras are an example of people knowing
that their movements are being recorded, but
are not always aware of where the recording
takes place.

Chapter 15 • Statistics in the media 531


STATistics and probability

•• Observations can be in a natural environment (for example, in a food hall), or a


contrived environment (a food‐tasting session for a food company).
•• Mechanical devices (video cameras, closed circuit television, counting devices across
a road) can also be used.

Measurement
•• Measurement involves using some
measuring device to collect data.
•• This generally involves conducting an
experiment of some type.
–– The height of everyone in your class
can be measured.
–– The mass of all new‐born babies can
be collected.
–– A pedometer measures the number of
steps the wearer takes.

Surveys
•• Surveys involve designing a questionnaire to interview people. Often the
questionnaire requires many rewrites to obtain one which is clear and unbiased.
•• The interview can be in person — face to face or by telephone. The advantage of
interviews is that you are able to gauge the reactions of those you are interviewing,
and explain particular questions, if necessary.
•• Most frequently these days, email is used to survey participants; however, there are
advantages and disadvantages to using this type of survey.
•• Advantages
–– Can cover a large number of people
or organisations
–– A wide geographic coverage is
possible
–– It avoids embarrassment on the part
of the respondent
–– There is no interviewer bias
–– The respondent has time to consider
responses
–– It’s relatively cheap
•• Disadvantages
–– The questions have to be relatively simple
–– The response rate is often quite low (inducements often given as an incentive to
return)
–– The reliability of the answers is questionable
–– No control over who actually completes the questionnaire
–– Problems with incomplete questionnaires

Experiment
•• Generally, when conducting an experiment the data collected are quantitative.
•• Particular care should be taken to ensure that the experiment is conducted in a
manner which would produce similar results if repeated.

532 Maths Quest 10 New South Wales Australian curriculum edition Stages 5.1 and 5.2
STATistics and probability

•• Care must be taken with the recording of


results.
•• The results must be in a form which
can readily be analysed.
•• All results need to be recorded, including
the weird or unexpected outcomes

Simulation
•• Experiments such as rolling a die, tossing a coin or
drawing a card from a deck may
be conducted to model some real‐life situation.
•• Simulations occur in areas such as business,
engineering, medical and scientific research.
•• They are often used to imitate real‐life situations
which may be dangerous, impractical or too expensive
to explore by other means.

WORKED EXAMPLE 4
It is widely believed that there is equal chance of having a boy or girl with each birth.
Genetics and the history of births in a family sometimes have a great influence on the sex of
the child as well. Ignore those factors in this question.
a Design an experiment to simulate the chance of giving birth to a boy or a girl.
b Describe how your experiment could be conducted to determine the number of children a
couple should have, on average, to ensure they have offspring of both sexes.
T HIN K W RI T E
a Use a device that can simulate two a A fair coin will be tossed with a Head representing a boy
outcomes which are equally likely. (B), and a Tail representing a girl (G).
This could be a random number
generator to generate two integers,
say a 0 (representing a boy) and
1 (representing a girl). A fair
coin could be tossed, such that, a
Head represents a boy, and a Tail
represents a girl.
b 1 Describe how the experiment b The experiment will be conducted 50 times, and a record
will be conducted. kept of each experiment. For each experiment, the coin
will be tossed until both sexes result. This may mean
that there could be for example 7 trials in an experiment
(GGGGGGB) before both sexes are represented.

Chapter 15 • Statistics in the media 533


STATistics and probability

2 Display the table of results. The table below shows the results of the 50 experiments.

Exp. No. of Exp. No. of


no. Results trials no. Results trials
1 BG 2 26 GGGB 4
2 GGB 3 27 GGGGB 5
3 BG 2 28 GGGB 4
4 GGGGB 5 29 BG 2
5 BBBBBBG 7 30 BBBG 4
6 GGGB 4 31 BG 2
7 BBG 3 32 GB 2
8 BBG 3 33 GGGB 4
9 BBBBG 5 34 BG 2
10 GB 2 35 GGGGGGB 7
11 BG 2 36 BBBBBBG 7
12 GGGB 4 37 GB 2
13 BBG 3 38 BG 2
14 BBG 3 39 GGB 3
15 GB 2 40 GGGGB 5
16 BG 2 41 BBG 3
17 GGB 3 42 BBBBBG 6
18 GB 2 43 GGB 3
19 GGB 3 44 GGB 3
20 BBBG 4 45 BBBG 4
21 BG 2 46 BBG 3
22 GB 2 47 GGGGGGB 7
23 GGGGB 5 48 BG 2
24 BG 2 49 BBG 3
25 GGGGB 5 50 GGGGGB 6
Total 175

This table shows that 175 trials were undertaken in 50


experiments where each experiment resulted in both sexes.
175
3 Determine the average number Average number of children = = 3.5
of children required to produce 50
offspring of both sexes.
4 Write a conclusion. The average number of children a couple should have to
reach the goal of having both sexes is 4.

•• Before collecting any primary data, it must be clear what data are to be collected.
•• A decision must be made as to the method of collection.
•• The advantages and disadvantages of the collection method must be
acknowledged.
•• The reason for the data collection should be clear from the outset.

534 Maths Quest 10 New South Wales Australian curriculum edition Stages 5.1 and 5.2
STATistics and probability

WORKED EXAMPLE 5
You have been asked to obtain primary data to determine the methods of transport used to
travel to school by the students at your school. The data collected are to provide support for
the Student Council’s proposal for a school bus.
a What data should be collected?
b Outline possible methods which could be used to collect this data.
c Decide which method you consider to be the best option, and discuss its advantages and
disadvantages.
T HIN K W RI T E
a Outline the various forms of a The modes of transport available to students at the school are:
transport available to the students. car, bus, train, bicycle and walking
b Consider all the alternatives for b Several methods could be used to collect the data.
collecting the data. •• Could stand at the school gate one morning and ask students
as they arrive
•• A questionnaire could be designed
•• Students could be asked to write their mode of transport on a
piece of paper and place in a collection tin.
c 1 Decide on best option. c The first option of standing at the school gate is very time‐
consuming, and students could arrive at the back gate.
The third option does not seem reliable, as some students may
not comply, and other students may place multiple pieces of
paper in the collection tin.
The second option seems the best of the three.
2 Discuss advantages and The advantages of a questionnaire include:
disadvantages. •• There is a permanent record on paper.
•• It is not time‐consuming to distribute or collect.
•• Students can complete it at their leisure.
Disadvantages include:
•• Students may not return it.
•• Expense involved in producing copies.

Note: This example does not represent the views of all those collecting such data.
It merely serves to challenge students to explore and discuss available options.

Secondary data
•• Secondary data are data that have already been collected by someone else.
•• The data can come from a variety of sources:
–– Paper — books, journals, magazines, company reports
–– Electronic — online databases, internet, broadcasts, DVDs
–– Government sources — ABS provides a wealth of statistical data
–– General business sources — academic institutions, stockbroking firms, sporting clubs
–– Media — newspapers, TV reports.

Chapter 15 • Statistics in the media 535


STATistics and probability

•• Secondary data sources often provide data that would not be possible for an
individual to collect.
•• The data can be qualitative or quantitative.
•• The accuracy and reliability of the data sometimes needs to be questioned, depending
on its source.
•• The age of the data should always be considered.
•• Often the data that surrounds us passes by unnoticed.
•• It is important to learn the skills to be able to critically analyse secondary data.

WORKED EXAMPLE 6
Bigbite advertise the energy and fat
content of some of their rollaways on
their placemats.
a What information can you gain
from this data? Bigbite fresh as rollaways
Roasted vegetable
Energy (kJ)
900
Fat (g)
3.0
Sat. fat (g)
1.0

b Bigbite advertise that they have a HAM


TURKEY
1100
1140
6.0
4.8
1.4
1.7

range of rollaways with less than BBQ BEEF


Bigbite ribbon
1150
1130
5.0
4.8
1.5
1.3

6 grams of fat. Comment on this TURKEY AND HAM


BBQ CHICKEN
1250
1460
4.5
4.7
1.5
1.2

claim. CHICKEN Tandoori 1110 4.0 1.0


Fresh as Dessert
c This could be the starting point of a Fruit Slices 200 <1 <1

statistical investigation. How could Bigbite rollaways


Regular rollaways include white and/or wholemeal bread,

you proceed from here?


salads and meat.

Nutritional value is changed by adding cheese or sauces.


d Investigations are not conducted
simply for the sake of investigating.
Suggest some aims for investigating
further.

T HIN K W RI T E
a Look at the data on the a The placemat reveals the following information:
placemat to gain as much • A higher energy content of a rollaway does not necessarily
information as possible. mean that its fat content is higher.
• As the fat content of a rollaway increases, generally the
saturated fat content also increases.
• The addition of some types of protein (ham, turkey, beef,
chicken) increases the energy content of the rollaway.
• These data are only for those rollaways on white or
wholemeal bread with salads and meat.
• The addition of condiments (sauces) or cheese will alter
these figures.
• A fruit slice has much less energy and fat than a rollaway.
b Examine the data to discover if b All the rollaways displayed have less than 6 grams of fat, so
there is evidence to support the Bigbite’s claim is true.
claim. Make further comment. It must be remembered that the addition of cheese and sauce
to these rollaways would increase their fat content. Also, if
the rollaway was on any bread other than white or wholemeal,
the fat content could go beyond 6 grams.

536 Maths Quest 10 New South Wales Australian curriculum edition Stages 5.1 and 5.2
STATistics and probability

c What would be the next step in c The placemat displays a toll‐free phone number for further
the investigation? information.
Their website also contains additional detailed information.
d What are some interesting facts d Suggested aims for investigating further could be:
which could be revealed through • H ow much extra fat is added to a rollaway by the addition
a deeper investigation? of cheese and/or sauce?
• What difference does a different type of bread make to the
fat content of the rollaway?
• Which rollaway contains the highest fat content?
• What is the sugar content of the rollaways?

Exercise 15B Primary and secondary data


INDIVIDUAL PATHWAYS
Questions: Questions: Questions:
1–8, 11 1–9, 11 1, 3, 6, 7, 10, 11
Activity 15‐B‐1 Activity 15‐B‐2 Activity 15‐B‐3
Data collection Further data collection Advanced data collection
doc‐5158 doc‐5159 doc‐5160

FLUENCY
1 WE4 Devise an experiment to simulate each of the following situations and specify
the device used to represent the outcomes.
a A true/false test in which answers are
randomly distributed.
b A casino game with outcomes grouped in
colours of either red or black.
c Breakfast cereal boxes containing 4 different
types of plastic toys.
d In a group of six people, one person is to be
chosen as the leader.
e A choice of three main meals on a
restaurant’s menu, all of which are equally popular.
f Five possible holiday destinations offered by a travel agent; such that all
destinations are equally available and equally priced.
g Five types of takeaway fast foods available in one area, where one pizza is twice
as popular as each of the others types of takeaway food (the other 4 are equally
popular).
2 WE5 You have been asked to obtain primary data from students at your school to
determine internet access students have at home. The data collected are to provide
support for opening the computer room for student use at night.
a What data should be collected?
b Outline possible methods which could be used to collect this data.
c Decide which method you consider to be the best option, and discuss its
advantages and disadvantages.

Chapter 15 • Statistics in the media 537


STATistics and probability

3 WE6 This label shows the nutritional information of Brand X rolled oats.

Nutrition Information
Servings Per Package: 25 Serving Size 30g
Per Serving 30g %Dl* Per Serving Per 100g
Energy 486kJ 6% 1620kJ
Protein 4.3g 9% 14.3g
Fat - Total 2.8g 4% 9.3g
- Saturated 0.5g 2% 1.7g
- Trans Less than 0.1g - Less than 0.1g
- Polyunsaturated 1.0g - 3.2g
- Monounsaturated 1.3g - 4.4g
Carbohydrate 16.8g 5% 56g
- Sugars 0.9g 1% 3.0g
Dietary Fibre 3.1g 10% 10.4g
Sodium 0.7mg 0.1% 2mg
* % DI = Percentage daily intake

a What information can you gain from this data?


b This could be the starting point of a statistical investigation. How could you
proceed from here?
c Suggest some aims for investigating further.
4 a Provide a list of methods you could use to collect primary data.
b Describe which method you would use to collect the following primary
data.
i Heights of trees along the footpaths of a tree‐lined street
ii Number of buses that transport students to your school in the morning
iii Sunrise times during summer
iv Student opinion regarding length of lessons

UNDERSTANDING
For questions 5 and 6, design an experiment to simulate the situation, carry out the
experiment and give the results of the experiment.
5 A mouse in a maze can make left
or right turns at each junction.
Assuming each turn is equally
likely, how many junctions on
average must the mouse go through
before each type of turn will have
been made?
6 A restaurant menu features
4 desserts which are assumed to
be equally popular. How many
dessert orders must be filled
(on average) before the owner can
be sure all types will have
been ordered?

538 Maths Quest 10 New South Wales Australian curriculum edition Stages 5.1 and 5.2
STATistics and probability

7 This label shows the nutritional information of Brand Y rolled oats.

nutrition information
Servings per package: 30
Serving size: 30g
Avg. Quantity Avg. Quantity
Per serving 30g Per 100g
Energy 480kJ (115Cal) 1600kJ (383Cal)
Protein 3.2g 10.5g
Fat, total 2.4g 8.0g
- saturated LESS THAN 1g 1.5g
Carbohydrate 18.3g 61.0g
- sugars 0.0g 0.0g
Dietary Fibre, total 3.3g 10.0g
Sodium LESS THAN 5mg LESS THAN 5mg

ingredients
Oats (100%)
attention

THIS PRODUCT CONTAINS


GLUTEN.
storage
Store in a cool, dry place.

Compare the nutritional information with that on the Brand X label on page 538.
8 Comment on this claim.
We did a survey on 100 people regarding eating chocolate.
60 of these people said they regularly ate chocolate.
We then measured the heights of all 100 people.
**** The result ****
Eating chocolate makes you taller!!
9 Russel operates a computer software sales outlet. He keeps a log of all complaints
from customers. Suggest how he could organise his log.
10 The following claim has been made regarding secondary data.
There’s a lot more secondary data than primary data, it’s a lot cheaper and it’s
easier to acquire.
Comment on this statement.
REASONING
11 The local Bed Barn was having a sale on selected beds by Sealy and Sleepmaker.
Four of the beds on sale were:
Sealy Posturepremier on sale for $1499 a saving of $1000
Sealy Posturepedic on sale for $2299 a saving of $1600 REFLECTION
Sleepmaker Casablanca on sale for $1199 a saving of $800 When using secondary
Sleepmaker Umbria on sale for $2499 a saving of $1800 data from other countries,
what different unit
The store claimed that all these beds had been discounted by
classifications could you
40%. Comment on whether this statement is true, supporting encounter?
your comments with sound mathematical reasoning.

Chapter 15 • Statistics in the media 539


STATistics and probability

CHALLENGE 15.1

15C  Evaluating inquiry methods and


statistical reports
•• Statistical investigations involve collecting data, recording the data, analysing the
data then reporting the results.

Interactivity
Compare statistical
Data collection methods
reports •• Collection methods involve gathering primary data, or using secondary data from
int‐2790
stored records.
•• Primary data can be collected by observation, measurement, survey, experiment or
simulation as discussed earlier.
•• Secondary data can be collected electronically or via a hard copy.
•• It is important to be able to justify the particular method chosen for each of these
processes.
•• Sometimes alternative methods are just as appropriate.

WORKED EXAMPLE 7
You have been given an assignment to investigate which year level uses
the school library, after school, the most.
a Explain whether it is more appropriate to use primary or secondary
data in this case. Justify your choice.
b Describe how the data could be collected. Discuss any problems
which might be encountered.
c Explain whether an alternative method would be just as
appropriate.

540 Maths Quest 10 New South Wales Australian curriculum edition Stages 5.1 and 5.2
STATistics and probability

T HIN K W RI T E
a No records have been a Since records are not kept on the library use,
kept on library use. secondary data is not an option.
Primary data collection could be either sampling
or census. A sufficiently large sample size could be
chosen; this would take less time than conducting
a census, although it would not be as accurate.
Sampling would be considered appropriate in
this case.

b The data can be b A questionnaire could be designed and


collected via a distributed to a randomly‐chosen sample.
questionnaire or in The problem here would be the non‐return of
person. the forms.
Observation could be used to personally interview
students as they entered the library. This would
take more time, but random interview times could
be selected.

c A census is the other c A census could be conducted, either by


option. questionnaire or observation. This should yield a
more accurate outcome.

WORKED EXAMPLE 8
Which method would be the most appropriate to collect the following data?
Suggest an alternative method in each case.
a The number of cars parked in the staff car park each day.
b The mass of books students carry to school each day.
c The length a spring stretches when weights are added to it.
d The cost of mobile phone plans with various network providers.

T HIN K W RI T E
a Observation a The best way would probably be observation by
visiting the staff car park to count the number of
cars there.
An alternative method would be to conduct a
census of all workers to ask if they parked in the
staff car park. This is probably not as good.

b Measurement b The mass of the books could be measured by


weighing each student’s pack on scales.
A random sample would probably yield a
reasonably accurate result.

Chapter 15 • Statistics in the media 541


STATistics and probability

c Experiment c Conduct an experiment and measure the extension


of the spring with various weights.
There is probably no alternative to this method.

d Internet search d An internet search would enable data to be


collected.
Alternatively, a visit to mobile phone outlets would
yield similar results.

Analysing the data


•• Once the data have been collected and collated, a decision must be made with regard
to the best methods for analysing the data. Choose:
–– a measure of central tendency — mean, median or mode
–– a measure of spread — range, interquartile range or standard deviation.
–– an appropriate graph.
Statistical graphs
•• Data can be graphed in a variety of ways — line graphs, bar graphs, histograms, stem
plots, box plots, etc. These have all been discussed in detail previously.
•• In media reports it is common to see line and bar graphs.
•• Because graphs give a quick visual impression, the temptation is to not look at them
in great detail. Often graphs can be quite misleading.
•• It is easy to manipulate a graph to give an impression which is supported by the
creator of the graph. This is achieved by careful choice of scale on the horizontal and
vertical axes.
–– Shortening the horizontal axis tends to highlight the increasing/decreasing nature of
the trend of the graph. Lengthening the vertical axis tends to have the same effect.
–– Lengthening the horizontal and shortening the vertical axes tends to level out the
trends.
WORKED EXAMPLE 9
This report shows the annual change in HOUSES
median house prices in the local government Suburb/locality Median house price Annual
areas (LGA) of Queensland from 2009–10 2008–09 change
2008–09 to 2009–10. Brisbane (LGA) $530,000 $475,000 11.6%
Ipswich City (LGA) $323,000 $310,000 4.2%
a Draw a bar graph which would give the
Redland City (LGA) $467,500 $435,000 7.5%
impression that the percentage annual Logan City (LGA) $360,000 $340,000 5.9%
change was much the same throughout the Moreton Bay (LGA) $399,000 $372,000 7.3%
whole state. Gold Coast City (LGA) $505,000 $465,000 8.6%
Toowoomba (LGA) $289,500 $269,000 7.6%
b Construct a bar graph to give the impression Sunshine Coast (LGA) $470,000 $445,000 5.6%
that the percentage annual change in Fraser Coast (LGA) $307,400 $297,750 3.2%
Brisbane was far greater than that in the Bundaberg (LGA) $282,000 $275,000 2.5%
Gladstone (LGA) $370,000 $370,000 0.0%
other local government areas. Rockhampton (LGA) $315,250 $300,000 5.1%
Mackay (LGA) $398,000 $383,000 3.9%
Townsville City (LGA) $375,000 $359,000 4.5%
Cairns (LGA) $365,000 $355,000 2.8%

542 Maths Quest 10 New South Wales Australian curriculum edition Stages 5.1 and 5.2
STATistics and probability

T HIN K W RI T E /DR AW
a To flatten out trends, a % house price changes in QLD 2008–9 to 2009–10

Annual % change
lengthen the horizontal axis 10
and shorten the vertical
axis. 5

Brisbane
Ipswich
Redland
Logan
Moreton Bay
Gold Coast
Toowoomba
Sunshine Coast
Fraser Coast
Bundaberg
Gladstone
Rockhampton
Mackay
Townsville

Cairns
Area

b To accentuate trends, b % house price changes in QLD


12
shorten the horizontal axis 11 2008–9 to 2009–10
and lengthen the vertical 10
Annual % change

axis. 9
8
7
6
5
4
3
2
0
Logan
Brisbane
Ipswich
Redland

Moreton Bay
Gold Coast
Toowoomba
Sunshine Coast
Fraser Coast
Bundaberg
Gladstone
Rockhampton
Mackay
Townsville
Cairns

Area

WORKED EXAMPLE 10
Consider the data displayed in the table of Worked example 9. Use the data collected for the
median house prices in 2009–10.
a Explain whether this data would be classed as primary or secondary data.
b Why does this data show median house prices rather than the mean or modal house
price?
c Calculate a measure of central tendency for the data. Explain the reason for this choice.
d Give a measure of spread of the data, giving a reason for the particular choice.
e Display the data in a graphical form, explaining why this particular form was chosen.

T HIN K W RI T E
a This is data which has been a This is secondary data because it has been collected by someone
collected by someone else. else.

Chapter 15 • Statistics in the media 543


STATistics and probability

b Median is the middle price, b The median price is the middle one. It is not affected by outliers as
mean is the average price, the mean is. The modal house price may only occur for two house
and mode is the most sales with the same value. On the other hand, there may not be
frequently‐occurring price. any mode.
The median price is the most appropriate in this case.
c Which measure of central c The measures of central tendency are the mean, median and mode.
tendency is the most The mean is affected by high values (i.e. $530  000) and low values
appropriate one? (i.e. $282  000). These are not typical values, so the mean would not
be appropriate.
There is no modal value, as all the house prices are different.
The median house price is the most suitable measure of central
tendency to represent the house prices in the Queensland local
government areas. The median value is $370  000.
d Consider the range and d The five‐number summary values are:
the interquartile range as Lowest score = $282  000
measures of spread. Lowest quartile = $315  250
Median = $370  000
Upper quartile = $467  500
Highest score = $530  000
Range = $530 000 − $282 000
= $248 000
Interquartile range = $467 500 − $315 250
= $152 250
The interquartile range is a better measure for the range as the
house prices form a cluster in this region.
e Consider the graphing e Of all the graphing options, the box plot seems the most
options. appropriate as it shows the spread of the prices as well as how they
are grouped around the median price.

280 000 340 000 400 000 460 000 520 000
280 000 340 000house
Median 400price
000 2009−10
460 000($) 520 000
Median house price 2009−10 ($)

WORKED EXAMPLE 11
The Australian women’s national basketball team, the Opals, competed at the 2008 Olympic
Games in Beijing, winning a silver medal. These are the heights (in metres) of the 12 team
members:
1.73, 1.65, 1.8, 1.83, 1.96, 1.88, 1.63, 1.88, 1.83, 1.88, 1.8, 1.96
Provide calculations and explanations as evidence to verify or refute the following statements.
a The mean height of the team is greater than their median height.
b The range of the heights of the 12 players is almost 3 times their interquartile range.
c Only 5 players are on the court at any one time. A team of 5 players can be chosen such
that their mean, median and modal heights are all the same.

544 Maths Quest 10 New South Wales Australian curriculum edition Stages 5.1 and 5.2
STATistics and probability

T HIN K W RI T E
Σ x 21.83
a 1 Calculate the mean a Mean = = = 1.82 m
n 12
height of the 12 players.
2 Order the heights to The heights of the players, in order, is:
determine the median. 1.63, 1.65, 1.73, 1.8, 1.8, 1.83, 1.83, 1.88, 1.88, 1.88, 1.96, 1.96
There are 12 scores, so the median is the average of the 6th and
7th scores.
1.83 + 1.83
Median = = 1.83 m
2
3 Comment on the The mean is 1.82 m, while the median is 1.83 m. This means
statement. that the mean is less than the median, so the statement is not
true.
b 1 Determine the range and b Range = 1.96 − 1.63 = 0.33 m
the interquartile range of Lower quartile is the average of 3rd and 4th scores.
the 12 heights. 1.73 + 1.8
Lower quartile = = 1.765 m
2
Upper quartile is average of 3rd and 4th scores from the end.
1.88 + 1.88
Upper quartile = = 1.88 m
2
Interquartile range = 1.88 − 1.765 = 0.115 m
2 Compare the two values. Range = 0.33 m
Interquartile range = 0.115 m
Range 0.33
= = 2.9
Interquartile range 0.115
3 Comment on the Range = 2.9 × interquartile range
statement. This is almost 3 times, so the statement is true.
c 1 Choose 5 players whose c Three players have a height of 1.88 m. If a player shorter and
mean, median and modal one taller are chosen both the same measurement from 1.88 m,
heights are all equal. this would make the mean, median and mode all the same.
Trial and error is Choose players with heights:
appropriate here. There 1.8, 1.88, 1.88, 1.88, 1.96
may be more than one 9.4
answer. Mean = = 1.88 m
5
Median = 3rd score = 1.88 m
Mode = Most frequent score = 1.88 m

2 Comment on the The 5 players with heights 1.8 m, 1.88 m, 1.88 m, 1.88 m,
statement. 1.96 m have a mean, median and modal height of 1.88 m.
It is true that a team of 5 such players can be chosen.

Statistical reports
•• Reported data must not be simply taken at face value; all reports should be examined
with a critical eye.

Chapter 15 • Statistics in the media 545


STATistics and probability

WORKED EXAMPLE 12
This is an excerpt from an article that appeared in a newspaper on Father’s Day. It was
reported to be a national survey findings of a Gallup Poll of data from 1255 fathers of
children aged 17 and under.

THE GREAT AUSSIE DADS SURVEY


Thinking about all aspects of your life, how %
happy would you say you are? We share the cooking and cleaning.................. 42
% My partner does most of it............................... 41
I am very happy............................................... 26 I do nothing/my partner does
I am fairly happy.............................................. 49 everything........................................................... 4
Totally happy.................................................... 75 None of the above.............................................. 1
Some days I’m happy and some days Which of these aspects of your children’s
future do you have concerns about?
I’m not.............................................................. 21
Their safety...................................................... 70
I am fairly unhappy............................................ 3
Being exposed to drugs.................................... 67
I am very unhappy............................................. 1
Their health...................................................... 54
Total unhappy..................................................... 4
Bullying or cyber‐bullying............................... 50
How often, if ever, do you regret having
children? Teenage violence.............................................. 50
Every day........................................................... 1 Their ability to afford a home.......................... 50
Most days........................................................... 2 Alcohol consumption and binge
drinking............................................................ 47
Some days........................................................ 18
Achieving academic success............................ 47
Never................................................................ 79
Feeling pressured into sex................................ 41
Which one of these best describes the impact
of having children on your relationship with Being able to afford the lifestyle they
your partner? expect to have................................................... 38
We’re closer than ever...................................... 29 Climate change................................................. 23
We don’t spend as much time together Having them living with you in their
as we should..................................................... 40 mid 20s............................................................. 14
We’re more like friends now than lovers......... 21 None of the above.............................................. 3
We have drifted apart......................................... 6 What is the best thing about being a dad?
None of the above.............................................. 4 The simple pleasures of family life.................. 61
Which one of these best describes the Enjoying the successes of your kids................ 24
allocation of cooking and cleaning duties in The unpredictability it brings............................. 9
your household? The comfort of knowing that you
My partner does nothing/I do everything........... 1 will be looked after in later life.......................... 3
I do most of it................................................... 11 None of the above.............................................. 3

546 Maths Quest 10 New South Wales Australian curriculum edition Stages 5.1 and 5.2
STATistics and probability

Key findings “Work‐life balance is definitely an


issue for dads in 2010.”
75% of Aussie dads are totally happy David Briggs
79% have never regretted having children Galaxy principal

are worried about their children


67%
being exposed to drugs
would like more intimacy with their
57%
partner

Source: The Sunday Mail, 5 Sept. 2010, pp. 14–15.

a Comment on the sample chosen.


b Discuss the percentages displayed.
c Comment on the claim that 57% of dads would like more intimacy with
their partner.

T HIN K W RI T E
a How is the sample chosen? a The results of a national survey such as this should reveal the
Is it truly representative of outlook of the whole nation’s dads. There is no indication of
the population of Australian how the sample was chosen, so without further knowledge
dads? we tend to accept that it is representative of the population.
A sample of 1255 is probably large enough.

b Look at the percentages in b For the first question regarding happiness, the percentages total
each of the categories. more than 100%. It seems logical that, in a question such as this,
the respondents would tick only one box, but obviously this has
not been the case.
In the question regarding aspects of concern of ‘your children’s
future’, these percentages also total more than 100%. It seems
appropriate here that dads would have more than one concerning
area, so it is possible for the percentages to total more
than 100%.
In each of the other three questions, the percentages total 100%,
which is appropriate.

c Look at the tables to try to c Examining the reported percentages in the question regarding
find the source of this figure. ‘relationship with your partner’, there is no indication how a
figure of 57% was determined.

Note: Frequently media reports make claims where the reader has no hope of confirming their
truth.

Chapter 15 • Statistics in the media 547


STATistics and probability

WORKED EXAMPLE 13
This article appeared in a newspaper. Read the article, then answer the following questions.

SPONGES ARE TOXIC


Washing dishes can pose a serious health risk, Microbiologist Craig Andrew‐Kabilafkas
with more than half of all kitchen sponges of Australian Food Microbiology said the
containing high levels of dangerous bacteria, Westinghouse study of more than 1000
research shows. households revealed germs can spread easily
A new survey dishing the dirt on washing to freshly washed dishes.
up shows more than 50 per cent of kitchen The only way to safeguard homes from
sponges have high levels of E coli, which sickness was to wash utensils at very high
can cause severe cramps and diarrhoea, and temperatures in a dishwasher.
staphylococcus aureus, which releases toxins
that can lead to food poisoning or toxic
shock syndrome.

Source: The Sunday Mail, 5 Sept. 2010, p. 36.

a Comment on the sample used in this survey.


b Comment on the claims of the survey.
c Is the heading of the article appropriate?

T HIN K W RI T E
a Look at sample size and a The report claims that the sample size was more than 1000.
selection of sample. There is no indication how the sample was selected.
The point to keep in mind is whether this sample is truly
representative of the population consisting of all households. We
have no way of knowing.
b What are the results of the b The survey claims that 50% of kitchen sponges have high levels
survey? of E. coli which can cause severe medical problems.
The study was conducted by Westinghouse, so it is not surprising
they recommend using a dishwasher.
c Examine the heading in the c The heading is sensational, designed to catch the attention of
light of the contents the article. readers.

Exercise 15C Evaluating inquiry methods and


statistical reports
INDIVIDUAL PATHWAYS
Questions: Questions: Questions:
1–4, 6, 8, 9, 10 1–4, 7–10 1–10
Activity 15‐C‐1 Activity 15‐C‐2 Activity 15‐C‐3
Collecting and More collecting Detailed collecting
analysing data and analysing data and analysing data
doc‐5161 doc‐5162 doc‐5163

548 Maths Quest 10 New South Wales Australian curriculum edition Stages 5.1 and 5.2
STATistics and probability

FLUENCY
1 WE7, 8 You have been given an assignment to investigate which Year level has the
greatest number of students who are driven to school each day by car.
a Explain whether it is more appropriate to use primary or secondary data in this
case. Justify your choice.
b Describe how the data could be collected. Discuss any problems which might be
encountered.
c Explain whether an alternative method would be just as appropriate.
2 WE9 You run a small company that is listed on the Australian Stock Exchange
(ASX). During the past year you have given substantial rises in salary to all your
staff. However, profits have not been as spectacular as in the year before. This table
gives the figures for the salary and profits for each quarter.

1st quarter 2nd quarter 3rd quarter 4th quarter


Profits $’000  000 6 5.9 6 6.5
Salaries $’000  000 4 5 6 7

Draw two graphs, one showing profits, the other showing salaries, which will show
you in the best possible light to your shareholders.
3 WE10 The data below were collected from a real estate agent and show the sale
prices of ten blocks of land in a new estate.
$150  000, $190  000, $175  000, $150  000, $650  000, $150  000, $165  000,
$180  000, $160  000, $180  000
a Calculate a measure of central
tendency for the data. Explain the
reason for this choice.
b Give a measure of spread of
the data, giving a reason for the
particular choice.
c Display the data in a graphical
form, explaining why this
particular form was chosen.
d The real estate agent advertises
the new estate land as:
Own one of these amazing blocks of land for only $150  000 (average)!
Comment on the agent’s claims.
4 WE11 Use the data for the heights of the Opal players in Worked example 11
(page 544) to answer the following question.
Provide calculations and explanations as evidence to verify or refute the following
statements.
a The mean height of the team is closer to the lower quartile than it is to the
median.
b Half the players have a height within the interquartile range.
c Which 5 players could be chosen to have the minimum range in heights?

Chapter 15 • Statistics in the media 549


STATistics and probability

5 This table below shows the number of shoes of each size that were sold over a week
at a shoe store.
Size Number sold
4 5
5 7
6 19
7 24
8 16
9 8
10 7
a Calculate the mean shoe size sold.
b Determine the median shoe size sold.
c Determine the modal shoe size sold.
d Explain which measure of central tendency has the most meaning to the store
proprietor.
6 The resting pulse of 20 female athletes was measured and is shown below.
50 62 48 52 71 61 30 45 42 48 43 47 51 52 34 61 44 54 38 40
a Represent the data in a distribution table using appropriate groupings.
b Find the mean, median and mode of the data.
c Comment on the similarities and differences between the three values.

UNDERSTANDING
7 The batting scores for two cricket players over six innings were recorded as follows.
Player A 31, 34, 42, 28, 30, 41
Player B 0, 0, 1, 0, 250, 0
Player B was hailed as a hero for his score of 250.
Comment on the performance of the two players.
8 A small manufacturing plant employs 80 workers. This table below shows the
structure of the plant.

Position Salary ($) Number of employees


Machine operator 18  000 50
Machine mechanic 20  000 15
Floor steward 24  000 10
Manager 62  000 4
Chief Executive Officer 80  000 1

a Workers are arguing for a pay rise, but the management of the factory claims that
workers are well paid because the mean salary of the factory is $22  100. Explain
whether this is a sound argument.
b Suppose that you were representing the factory workers and had to write a short
submission in support of the pay rise. How could you explain the management’s
claim? Provide some other statistics to support your case.

550 Maths Quest 10 New South Wales Australian curriculum edition Stages 5.1 and 5.2
STATistics and probability

9 WE12, 13 This report from Woolworths appeared in a newspaper.

IT’S A RECORD
• Woolworths posted 10.1% gain • Wants to increase its share of the
in annual profit to $2.02b fresh food market
• 11th consecutive year of double‐ • Announced $700m off‐market
digit growth share buyback
• Flags 8% to 11% growth in the • Final fully franked dividend 62¢
current financial year a share
• Sales rose 4.8% to $51.2b

SHARES REBOUND NET PROFIT


$
28.40
28.10 2.4% $b +$2.02b
27.70 Yesterday +10.1%
2 +12.8%
27.40 +25.7%
27.10 1.5 +27.5%
20.80
+24.3%
26.50 1
26.20
0.5
25.90
25.60
Aug 26 0
May 26 2006 2007 2008 2009 2010
Source: IRESS

Source: The Courier Mail, 27 Aug. 2010, pp. 40–1.


Comment on the report.
REASONING
10 The graph at right shows the fluctuation in the Australian dollar in terms of
the US dollar during the period 13 July to 13 September 2010. The higher the
Australian dollar, the cheaper it is for Australian companies to import goods
from overseas, and the cheaper they should be able to sell their goods to the
Australian public. AUSSIE
The manager of Company XYZ produced a graph to US¢ US 93.29¢
support his claim that, because there hasn’t been much 92.8
change in the Aussie dollar over that period, there hasn’t 90.9
been any change in the price he sells his imported goods
88.8
to the Australian public. Draw a graph that would support
his claim. Explain how you were able to achieve this 86.8

effect. 84.8
82.8

REFLECTION 80.8
Jul 13 Sep 13
What is the point of Source: IRESS
drawing a misleading
graph in a report? Source: The Courier Mail,
14 Sept. 2010, p. 25.

Chapter 15 • Statistics in the media 551


STATistics and probability

15D  Statistical investigations


Using primary data
•• This section deals with the steps involved in carrying out a statistical investigation
with primary data collection.
•• For this exercise we will assume you have
been given this task.

Which pizza on the market is the


best value for money?

•• This is a very broad investigation, and each


stage of the investigation must be carefully
planned.
–– Collecting the data
–– Organising the data
–– Performing calculations
–– Analysing the data
–– Reporting the results

Collecting the data


•• At this initial stage, questions should be posed with regard to the data.
•• What data should be collected?
–– Best value for money involves the price and size of the pizza. Data on both of
these need to be collected.
–– Stores have different prices for different sizes.
–– Would size best be measured as area or mass?
–– Not all pizzas are round; some are rectangular.
–– What about the variety of toppings? A standard one should be chosen.
–– Should frozen pizzas be included?
•• How should the data be collected?
–– It is not possible to buy every pizza on the market, so what alternatives are there?
–– A store is probably not willing to allow their pizzas to be weighed, so mass is
most likely out of the question.
–– Will the store allow their pizzas to be measured?
•• What problems are likely to be encountered?
–– How many different companies market pizza?

Organising the data Value for


•• The data should be organised Price Measurement Area money
into some sort of table format.
–– What format is appropriate
for this investigation?
–– A table with column headings
Price and Measurements will
organise this data.

552 Maths Quest 10 New South Wales Australian curriculum edition Stages 5.1 and 5.2
STATistics and probability

•• Take time to design the table so figures required for calculation are readily visible.
–– What calculations are required at this stage?
–– Measurements are required to calculate the area of each pizza.
•• Think forward and add extra columns for future calculations.
–– What further calculations are needed?
–– Area and Value for money need to be calculated. Provide two extra columns for these.

Performing calculations
•• What calculations need to be performed?
•• The Area of the pizza and Value for money are required in this case. How should
there be calculated? Dividing price by area gives ($/cm2), while dividing area by
price gives (cm2/$).

Analysing the data


•• Are there any anomalies, or obvious calculation errors?
•• Do the calculated results ‘make sense’?
•• In this case, if Value for money is calculated in units of $/cm2, the pizza with the
smallest of these values is the best value for money. Using units of cm2/$, the pizza
with the highest of these values gives the best value for money.
•• Would the inclusion of graphs be appropriate?

Reporting the results


•• The results should be reported in a clear, concise manner.
•• Justify any conclusions.
•• Are there any anomalies or exceptions to mention?

Using secondary data


•• The procedure for undertaking a statistical investigation using secondary data is
similar to that for primary data, the difference being that you sometimes have to
search for data in several areas before you find the appropriate source.
Suppose you were given this assignment.

There have been Prime Ministers of Australia since 1901 until


this day.
There have been elections.
Prime Ministers have been defeated at a general election.
There have been changes of Prime Minister without an
election.
The average length these Prime Ministers served in office is .
Undertake a statistical investigation to complete the details.

Collecting the data


•• What data should be collected?
•• Where can this data be found? The internet is probably a good starting point, but not
all sites are reliable.
•• If there are multiple sources for the data, are they all in agreement?
•• How many of these statements require calculations?

Chapter 15 • Statistics in the media 553


STATistics and probability

Organising the data


•• Design a table to record all the data.
•• Consider how many columns are necessary.
•• Leave columns for calculations.

Performing calculations
•• There is at least one calculation here — to determine the average length of time
served in office. Are there any more?

Analysing the data


•• Do all the calculated values make sense?
•• Would a graph be appropriate?

Reporting the results


•• Complete the details.
•• Acknowledge the source of the secondary data.

Investigating media reports


•• Frequently reports in the media provide a good starting point for an interesting
investigation. Here are a few suggestions.

Media report 1
Here is an article on an analysis of the speech habits of two high‐profile
parliamentarians — Julia Gillard (who was Prime Minister when the article was
written) and Tony Abbott (who was then the Leader of the Opposition). It was written
by Roly Sussex, a professor of English.

Word Limit
Roly Sussex her, she speaks in long complete sentences.
sussex@uq.edu.au But unlike Rudd, her sentences contain a
We all have individual features in the way fair number of pre‐programmed mantras and
we speak. Our tone of voice, an intonation, phrases: “happy to be judged”, “enhanced
a rhythm, a favourite word or phrase — the the capacity”, “regional neighbours”. Her
things that make us quickly recognised even pronunciation is also distinctive. She grew
on bad telephone connections. But if you up in South Australia, and so says W instead
are a public figure, and especially a political of L at the end of a syllable. Her “milk” is
leader, your speech habits will be picked MIWK, and her “football” is FOOTBAW. And
up, criticised, satirised and caricatured. As she is our first Prime Minister to have high
I discussed in previous weeks, our current rising tone, the rising intonation at the end of a
political leaders show a wide variety of habits clause (rising pitch).
and idiosyncrasies. Her favourite word is “obviously” and she
Our new PM, Julia Gillard is a very has also quickly assumed the leader’s “I”.
consecutive speaker. Like Kevin Rudd before The closer you get to the top job, the more the

554 Maths Quest 10 New South Wales Australian curriculum edition Stages 5.1 and 5.2
STATistics and probability

ego asserts itself in grammar. Especially in “Spin .  .  . contradiction .  .  .


answer to a probing question: “I’m not going incompetent .  .  . disarray .  .  .”. It’s like
to be .  .  .”. Former PM John Howard perfected swearing — the more you use these words,
this technique, and it is piously observed by the less meaning they convey. The second
our current leaders. Treasurer Wayne Swan is shared feature is the pre‐programmed
acquiring it. Aha. response. A trigger in the question presses a
Compared to the PM, Opposition Leader specific answer‐button. “Asylum seekers”,
Tony Abbott is a less fluent speaker. He is an “deficit”, “mining super‐tax”, “health” and
“um”, “look” and “ah” man. His sentences similar key issues prompt the automatic
contain pauses, sometimes for reflection, rehearsed rejoinder. You know it’s pre‐
sometimes for emphasis, sometimes both. programmed because you’ll always hear
He is somewhat less given to mantra, and the same words, whenever the trigger is
greetings‐wise he is more a man of the people: pressed.
he says “G’day”. But Gillard and Abbott share The third thing they share is that they won’t
three features which are now so ingrained say “yes” or “no”. Both respond to a “can
under the fingernails of our pollies that they you tell us, yes or no?” with streams of verbal
won’t scrub off. flimflam. Interviewers should give up trying
One feature is repetition. “As I said in my to prise a clear yea/nay out of either of them.
speech .  .  .” says the PM, “.  .  . as I said in my But the public does have a right to know, yes
speech”. Well, yes, we know that. We heard or no, where they stand on issues, and we
the speech. Abbott, on the other hand, repeats aren’t getting what we crave.
repeated negatives about the Government:

Source: The Courier Mail, 14–15 Aug. 2010, p. 25.


There is no doubt that these comments are true. How could you find evidence of this?
Media report 2

Single Women Earn More


WASHINGTON: The income of one group The report says that one reason for the
of US women is catching up to and even finding is that girls are “going to college in
overtaking men, a study shows. droves”.
They are single women in their 20s without Nearly three‐quarters of girls who complete
children, who live in large cities and work high school go on to university, compared with
full‐time, according to a study of census data two‐thirds of boys.
1
by Reach Advisors, a New York–based strategy Women are 1 2 times more likely than men
and research firm focused on emerging shifts to graduate from university and to obtain a
in the consumer landscape. masters degree or higher.
These young women earn on average 8 per Census data released in April showed that
cent more than men in their age group, but in 58 per cent of all US masters degrees or
some cities, such as Atlanta in Georgia and PhDs were awarded to women. As women
Memphis, Tennessee, women earn about one‐ go further in their education, they are
fifth more than men. On average, American also delaying getting married and starting
women who work full‐time earn about 80 per a family.
cent of what men earn.

Source: The Weekend Australian, 4–5 Sept. 2010, p. 20.

Chapter 15 • Statistics in the media 555


STATistics and probability

Is this report really true? Is it perhaps only true in America? (The heading seems to
suggest that it is universally true.) What is the status of women in other parts of the
world? Further investigation could reveal interesting comparisons.
Media report 3

Egg shortage is no yolk


Producers lay plan to meet need number of eggs recommended for a healthy
Peddy Hintz diet from two a week to six.
But, Mr O’Hara said, cooking shows such as
Blame MasterChef or the Heart Foundation, MasterChef had also led to a rise in demand.
but it’s getting harder to find the right kind of Current estimates have Australians eating
eggs at the supermarket — and it’s likely to 203–205 eggs per person a year, compared
stay that way until Christmas. with 195 last year, 156 the year before and a
Queensland egg producers are struggling low of 132  10 years ago.
to keep up with demand but the boom in sales
has also been matched by the recent interest in UNSCRAMBLING EGGS
keeping backyard chickens. Annual egg consumption (per person)
The winter shortage of eggs on Australian Australia: 205
supermarket shelves will mean that instead Japan: 320
of sitting in a coolroom for a week, eggs are US: 230
being transferred to shelves almost straight UK: 230
from the supplier. NZ: 214
“It’s currently taking only about 48 hours Egg sales
from being laid to getting onto the shelves so 65% caged
the eggs that people do buy will be a really 24–25% free range
good, fresh product,” chief executive of Sunny 10% cage free
Queen, one of the country’s biggest suppliers, Fresh is best
John O’Hara said. The most effective way to test if an egg
The Australian Egg Corporation has put the is fresh is to put it in water. The more it
increased demand for eggs down to revised sinks, the fresher the egg. If it floats, it’s
Heart Foundation guidelines raising the nearly off.

Source: The Courier Mail, 28–9 Aug. 2010, p. 13.

Note the catchy heading on this article. Does the advice from the Heart Foundation
or cooking shows like Master Chef really have that much effect on egg sales?
How does egg consumption in Australia compare with that in the other countries
mentioned? This is worthy of further investigation.

Exercise 15D Statistical investigations


INDIVIDUAL PATHWAYS
Questions: Questions: Questions:
1–7 1–7 1–7
Activity 15‐D‐1 Activity 15‐D‐2 Activity 15‐D‐3
Analysing reports Analysing reports in depth Analysing reports in greater depth
doc‐5164 doc‐5165 doc‐5166

556 Maths Quest 10 New South Wales Australian curriculum edition Stages 5.1 and 5.2
STATistics and probability

UNDERSTANDING
1 a   Write a plan detailing how you would collect primary
data to undertake an investigation to determine which
pizza on the market is the best value for money.
b Undertake your investigation.
c Report on the results of your findings.
2 Undertake the investigation on the history of Prime
Ministers in Australia. Report your findings.
3 Find evidence from speeches of Julia Gillard and
Tony Abbott to support Roly Sussex’s report on the
speech habits of these two politicians.
4 Do single women really earn more? Investigate.
5 What’s the story on egg consumption in Australia?
6 Search for a media article you would like to investigate.
Provide a full report on your findings.

REASONING
7 Below are a few statistics on Facebook users. These figures are those reported
in the Year 2010.
• There are more than 400 million active users.
• 70% of Facebook users are outside the US.
• 50% of active users log on to Facebook in any given day.
• More than 60 million updates are posted each day.
• More than 3 billion photos are uploaded to the site each month.
• The average user has 130 friends on the site.
• The average user spends more than 55 minutes per day on Facebook.
The top 10 countries on Facebook represent just a little over half of the Facebook
users. China (population 1.3 billion) and India (1.2 billion) do not appear in
the top 10 list. Write a report summarising the usage of Facebook throughout
the world.
These are the top 10 countries on Facebook.

Country Population (millions) % of world population Users (millions)


1 USA 310.3 4.5 111.2
2 UK 62 0.9 23.5
3 Indonesia 237.6 3.5 19.5
4 Turkey 72.6 1.1 18.7
5 France 65.4 1.0 15.9
6 Italy 60.4 0.9 14.9
7 Canada 34.3 0.5 13.4
(continued)

Chapter 15 • Statistics in the media 557


STATistics and probability

Country Population (millions) % of world population Users (millions)


8 Philippines 94 1.4 10.6
Digital doc
WorkSHEET 15.2 9 Spain 46.1 0.7   8.9
doc‐5343
10 Mexico 108.4 1.6   8.2

REFLECTION
What would you consider
to be the most important
factor in reporting the
results of a statistical
investigation?

CHALLENGE 15.2

558 Maths Quest 10 New South Wales Australian curriculum edition Stages 5.1 and 5.2
STATistics and probability

Chapter review
LANGUAGE

census observation secondary statistical


data collection population data investigations
methods primary data simulation statistical reports
experiment sample statistical data surveys

int‐2865 doc‐13788
int‐2866 doc‐13789
int‐3601

FLUENCY
1 List some problems you might encounter in trying to collect data from the following
populations.
a The average number of mL in a can of soft drink.
b The number of fish in a dam.
c The number of workers who catch public transport to work each weekday morning.
2 a Calculate the mean of the integers 1 to 100.
b i Randomly select 10 numbers in the range 1 to 100.
ii Calculate the mean of these numbers.
c i Randomly select 20 numbers in the range 1 to 100.
ii Calculate the mean of these numbers.
d Comment on the similarities/differences between your means calculated in
parts a, b and c.
3 For each of the following investigations, state whether a census or a survey has been used.
a The average price of petrol in Canberra was estimated by averaging the price at
30 petrol stations in the area.
b The performance of a cricketer is measured by looking at his performance in
every match he has played.
c Public opinion on an issue is sought by a telephone poll of 2000 homes.
4 Traffic lights (red, amber, green) are set so that each colour shows for a set
amount of time. Describe how you could use a spinner to simulate the situation
so that you could determine (on average) how many sets of lights you must
encounter in order to get two green lights in succession.
5 John and Bill play squash each week. In any given game they are evenly
matched. A device that could not be used to represent the outcomes of the
situation is:
A a die
B a coin

Chapter 15 • Statistics in the media 559


STATistics and probability

C a circular spinner divided into 2 equal sectors


D a circular spinner divided into 5 equal sectors
E a circular spinner divided into 4 equal sectors
6 The table below shows the number of students in each year level from Years 7 to 12.

Year Number of students


7 230
8 200
9 189
10 175
11 133
12 124
Draw two separate graphs to illustrate the following.
a The principal of the school claims a high retention rate in Years 11 and 12 (that is,
most of the students from Year 10 continue on to complete Years 11 and 12).
b The parents claim that the retention rate of students in Years 11 and 12 is low (that
is, a large number of students leave at the end of Year 10).
7 Records from a school were examined to determine the number of absent days of
both boys and girls over the two years of Year 9 and Year 10. The result is shown in
this stem‐and‐leaf plot.
Key: 2 | 1 = 21 days
Leaf Stem Leaf
Boys Girls
0 17
7410 1 24799
9976653110 2 133466
87752 3 4448
2 4 36
5 4
a Calculate the median number of days absent for both boys and girls.
b Calculate the range for both boys and girls.
c Comment on the distribution of days absent for each group.
8 15 boys and 15 girls were randomly chosen from a group
of 900 students. Their heights (in metres) were measured as
shown below.
Boys: 1.65, 1.71, 1.59, 1.74, 1.66, 1.69, 1.72, 1.66, 1.65, 1.64,
1.68, 1.74, 1.57, 1.59, 1.60
Girls: 1.66, 1.69, 1.58, 1.55, 1.51, 1.56, 1.64, 1.69, 1.70, 1.57,
1.52, 1.58, 1.64, 1.68, 1.67
a Comment on the size of the sample.
b Display the data as a back‐to‐back stem plot.
c Compare the heights of the boys and girls.
9 The stem plot below is used to display the number of vehicles
sold by the Ford and Holden dealerships in a Sydney suburb
each week for a three‐month period.

560 Maths Quest 10 New South Wales Australian curriculum edition Stages 5.1 and 5.2
STATistics and probability

Key: 1 | 5 = 15 vehicles
Leaf Stem Leaf
Ford Holden
74 0 39
952210 1 111668
8544 2 2279
0 3 5
a State the median of both distributions.
b Calculate the range of both distributions.
c Calculate the interquartile range of both distributions.
d Show both distributions on a parallel box plot.
10 The box plots drawn below display statistical data for two AFL teams over a season.
Sydney Swans

Brisbane Lions

50 60 70 80 90 100110120 130140150 Points


a Which team had the higher median score?
b What was the range of scores for each team?
c For each team calculate the interquartile range.
11 Tanya measures the heights (in m) of a group of Year 10 boys and girls and
produces the following five‐point summaries for each data set.
Boys: 1.45, 1.56, 1.62, 1.70, 1.81
Girls: 1.50, 1.55, 1.62, 1.66, 1.73
a Draw a box plot for both sets of data and display them on the same scale.
b What is the median of each distribution?
c What is the range of each distribution?
d What is the interquartile range for each distribution?
e Comment on the spread of the heights among the boys and the girls.
12 The box plots below show the average daily sales of cold drinks at the school
canteen in summer and winter.
Summer

Winter

0 5 10 15 20 25 30 35 40 Daily sales
of cold
drinks
a Calculate the range of sales in both summer and winter.
b Calculate the interquartile range of the sales in both summer and winter.
c Comment on the relationship between the two data sets, both in terms of
measures of location and measures of spread.
13 A movie theatre has taken a survey of the ages of people at a showing of two of
their movies. The results are shown in these box plots.
Movie A

Movie B

0 10 20 30 40 50 60 70 80 Age

Chapter 15 • Statistics in the media 561


STATistics and probability

Which of the following conclusions


could be drawn based on the preceding
information?
A Movie A attracts an older audience
than Movie B.
B Movie B attracts an older audience
than Movie A.
C Movie A appeals to a wider age
group than Movie B.
D Movie B appeals to a wider age
group than Movie A.
E More people went to Movie A.

PROBLEM SOLVING
1 A sample of 30 people was selected at
random from those attending a local
swimming pool. Their ages (in years)
were recorded as follows:
19, 7, 58, 41, 17, 23, 62, 55, 40, 37,
32, 29, 21, 18, 16, 10, 40, 36, 33, 59,
65, 68, 15, 9, 20, 29, 38, 24, 10, 30.
a Find the mean and the median age
of the people in this sample.
b Group the data into class intervals
of 10 (0–9 etc) and complete the
frequency distribution table.
c Use the frequency distribution table to estimate the mean age.
d Calculate the cumulative frequency and, hence, plot the ogive.
e Estimate the median age from the ogive.
f Compare the mean and median of the original data in part a with the estimates of
the mean and the median obtained for the grouped data in parts c and e.
g Were the estimates good enough? Explain your answer.
2 The typing speed (words per minute) was recorded for a group of Year 8 and Year 10
students. The results are displayed in this back‐to‐back stem plot.
Key: 2 | 6 = 26 wpm
Leaf Stem Leaf
Year 8 Year 10
99 0
9865420 1 79
988642100 2 23689
9776410 3 02455788
86520 4 1258899
5 03578
6 003
Write a report comparing the typing speeds of the two groups.

562 Maths Quest 10 New South Wales Australian curriculum edition Stages 5.1 and 5.2
STATistics and probability

3 These parallel box plots show the number of weekly house sales by two real estate
agencies over a 3‐month period.
HJ Looker

Hane & Roarne

0 1 2 3 4 5 6 7 8 9 10 Number of
weekly sales

Prepare a report to compare the performance of the two agencies.


4 This taste test on corn chips appeared in a newspaper.

TASTE TEST: CORN CHIPS


Byron Bay Chilli Co. Corn chips IGA Black and Gold Plain Corn chips
230g $3.40 1 $1.48 per 100g 2 230g $1.99 1 87 cents per 100g 2
• Made in Australia • Made in Australia
• Fat 24.9g/100g • Fat 24.9g/100g
• Saturated Fat 11.8g/100g • Saturated Fat 11.8g/100g
• Sodium 44mg/100g • Sodium 415mg/100g
Verdict: Crisp, thick chips with fresh Verdict: Cheap in comparison and quite nice.
corn flavour and low sodium content. A good budget option as they are as good as
No preservatives, no GM corn. Put Byron the popular branded ones.
Bay in the title and things cost more! Doritos Corn Chips
CCs Corn Chips 200g $2.99 1 $1.50 per 100g 2
200g $2.69 1 $1.34 per 100g 2 • Made in Australia
• Made in Australia • Fat 23.2g/100g
• Fat 24.4g/100g • Saturated Fat 11.2g/100g
• Saturated Fat 10.6g/100g • Sodium 435mg/100g
• Sodium 550 mg/100g Verdict: Very crisp and fresh, but really
Verdict: Chips are quite thin and very salty no better than the home brand which is
to taste. Very high sodium content. $1 cheaper.
Source: The Sunday Mail, 4 Apr. 2010, p. 26.

Comment on the information displayed.


5 There has been a rise in supermarket‐own brands in Australia. These are commonly
available in supermarkets like Woolworths, Coles and Aldi. It has been said
that these brands account for almost one‐quarter of all grocery sales. It has also
been claimed that the quality of supermarket‐own brands is comparable with the
equivalent market‐leading brand, at a much reduced cost.
Assume you are planning undertake a study of a particular grocery line
(e.g., baked beans or breakfast cereal). Write a plan of how you would undertake
this study.

Chapter 15 • Statistics in the media 563


STATistics and probability
Communicating

Rich task

Populations and samples

Use digital technology to work with the large set of data on the opposite
page. This task will only use the first column of literacy rates for the
country. Other conclusions may be drawn from the male and female
literacy rates and possibly compared to the overall literacy rates in this
investigation.
1 Find the five‐figure summary for the 100 countries’ literacy rates and draw
a boxplot.
2 Find the mean and standard deviation.
3 Take a sample of 20 from the set of 100 countries using the random
number generator and make a list of the country and the literacy rates.
4 Find the five‐number summary for the sample.
5 Find the mean and standard deviation for the sample.
6 Draw a boxplot that compares your sample with the whole
population.
7 Describe the sample data obtained in terms of the
population as a whole; in particular, concentrate on the
middle 50 per cent of the data.
8 Now take a further two random samples of 20 countries
and draw boxplots of all three on the same scale. Comment
on these three samples.
9 Find the mean and standard deviation for the three
samples. How do they compare?
10 From the results you have obtained, how would you
describe the reliability of a sample compared with using the
whole population?

564 Maths Quest 10 New South Wales Australian curriculum edition Stages 5.1 and 5.2
STATistics and
STATISTICS AND PROBABILITY
probability

Chapter 15 • Statistics in the media 565


STATistics AND
STATISTICS and PROBABILITY
probability

Code puzzle

Animals that originated


in Australia include .  .  .
Calculate the mean, median, mode and range for each data set.
The answer and corresponding letter gives the puzzles’s answer code.

9, 3, 5, 13, 5, 6, 11, 6, 5
A Mean =

B Median =

E Mode =

G Range =

9, 10, 15, 11, 18, 13, 9, 15, 11, 18, 9, 14, 12


16, 9, 10, 15, 10, 5, 11
K Mean =

L Median =

M Mode =

N Range =

79, 85, 64, 80, 56, 74, 61, 61


O Mean =

R Median =

U Mode =

D Range =

12 70 7 11 7 5 9 61 12 7 13 10 7 69 70 70

7 13 29 12 70 70 12 7 6 61 69 69 7

566 Maths Quest 10 New South Wales Australian curriculum edition Stages 5.1 and 5.2
STATistics and probability

ACTIVITIES
Go to assessON
Chapter opener Digital docs (page 548) for questions to
Video • Activity 15‐C‐1 (doc‐5161): Collecting and test your readiness
• The story of mathematics (eles-1854) analysing data FOR learning, your
• Activity 15‐C‐2 (doc‐5162): More collecting progress AS you learn
15A Populations and samples
and analysing data and your levels OF
Digital docs
• Activity 15‐C‐3 (doc‐5163): Detailed collecting achievement.
• Activity 15‐A‐1 (doc‐5155): Populations and
and analysing data www.assesson.com.au
samples (page 529)
• Activity 15‐A‐2 (doc‐5156): More populations 15D Statistical investigations
and samples (page 529) Digital docs
• Activity 15‐A‐3 (doc‐5157): In depth • Activity 15‐D‐1 (doc‐5164): Analysing reports
populations and samples (page 529) (page 556)
• SkillSHEET (doc‐5337): Determining suitability • Activity 15‐D‐2 (doc‐5165): Analysing reports
of questions for a survey (page 529) in depth (page 556)
• SkillSHEET (doc‐5338): Finding • Activity 15‐D‐3 (doc‐5166): Analysing reports
proportions (page 529) in greater depth (page 556)
• SkillSHEET (doc‐5339): Distinguishing • WorkSHEET 15.2 (doc‐5343): Statistics in the
between types of data (page 529) media (page 558)
• SkillSHEET (doc‐5340): Reading bar
Chapter review
graphs (page 529)
INTERACTIVITIES (page 559)
• SkillSHEET (doc‐5341): Determining
• Word search (int-2865)
independent and dependent
• Crossword (int-2866)
variables (page 529)
• Sudoku (int-3601)
• WorkSHEET 15.1 (doc‐5342): Populations and
Digital docs
samples (page 531)
• Chapter summary (doc‐13788)
15B Primary and secondary data • Concept map (doc‐13789)
Digital docs (page 537) To access eBookPLUS activities, log on to
• Activity 15‐B‐1 (doc‐5158): Data collection www.jacplus.com.au
• Activity 15‐B‐2 (doc‐5159): Further data
collection
• Activity 15‐B‐3 (doc‐5160): Advanced data
collection
15C Evaluating inquiry methods and
statistical reports
Interactivity
• Compare statistical reports (int‐2790)
(page 540)

Chapter 15 • Statistics in the media 567


STATistics and probability

ANSWERS
CHAPTER 15 Statistics in the media
Exercise 15A — Populations and samples f Spinner with 5 equal sectors (each one representing a
1    a When was it first put into the machine? How old was the particular destination)
battery before being purchased? How frequently has the g Spinner with 5 sectors, one which will have an angle size of
computer been used on battery? 120°, while the other 4 each have an angle size of 60° (each
b Can’t always see if a residence has a dog; A census is very one representing a particular fast food)
time‐consuming; Perhaps could approach council for dog 2 Answers will vary, however some possible suggestions
registrations. include:
c This number is never constant with ongoing purchases, and Which students have internet access at home?
continuously replenishing stock. Do the students need access at night?
d Would have to sample in this case as a census would involve What hours would be suitable?
opening every packet. How many would make use of this facility?
2 These answers will vary with the samples chosen. 3 Answers will vary. Check with your teacher.
3    a Census. The airline must have a record of every passenger on 4 Answers will vary, however some possible suggestions
every flight. include:
b Survey. It would be impossible to interview everyone. a Census, survey, questionnaire, interview, observation,
c Survey. A census would involve opening every bottle. experiment, on‐line response, …
d Census. The instructor must have an accurate record of each
b i Measurement
learner driver’s progress.
ii Observation
4    a Survey
iii Newspaper recordings
b Survey
iv Survey
c Census
d Survey 5–7 Student’s own response
5    a About 25 8 The claim is false. It is not a logical deduction.
b Drawing numbers from a hat, using a calculator, ….. 9 Student’s own response
6    a The council is probably hoping it is a census, but it will 10 Student’s own response
1000
probably be a survey because not all those over 10 will 11 Sealy Posturepremier 40% off ( × 100%),
2499
respond. 1600
b Residents may not all have internet access. Only those who Sealy Posturepedic 41% off (3899 × 100%) ,
are highly motivated are likely to respond. 800
Sleepmaker Casablanca 40% off (1999 × 100%),
7 The sample could have been biased. The questionnaire may have
been unclear. Sleepmaker Umbria 42% off (1800
4299
× 100%).
8 Sample size, randomness of sample
There is at least 40% off these beds.
9 Answers will vary. Check with your teacher.
10 Populations growing very rapidly, large number of expatriate Challenge 15.1
workers in China have a different background and forms need 1250 trout
to be modified for them, people from Hong Kong working on
Exercise 15C — Evaluating inquiry methods and statistical
mainland China, large migrant population in New Delhi, often
migrants don’t have residency permits (so the truth of their reports
answers is questionable), many people live in inaccessible 1    a Primary. There is probably no secondary data available.
areas, some families in China have more than 1 child and do b, c Answers will vary. Check with your teacher.
not disclose this. 2 Company profits
11 There is quite a variation in the frequency of particular Company profits
numbers drawn. For example, the number 45 has not been
6.5
drawn for 31 weeks, while most have been drawn within the last
Profits ($’000 000)

6.4
10 weeks. In the long term, one should find the frequency of
6.3
drawing each number is roughly the same. It may take a 6.2
long time for this to happen, as only 8 numbers are drawn each 6.1
week. 6.0
5.9
Exercise 15B — Primary and secondary data 0
1 These are simply examples of simulations which could be 1 2 3 4 5
­conducted. Quarter
a Coin could be flipped (Heads represents ‘True’, while Tails Mean salaries
represents ‘False’)
b Coin could be flipped (Heads represents ‘red’, while Tails Company salaries
represents ‘black’)
Mean salaries

15
($’000 000)

c Spinner with 4 equal sectors (each sector representing a 10


different toy) 5
d Roll a die (each face represents a particular person) 0
e Spinner with 3 equal sectors (each one representing a 1 2 3 4
particular meal) Quarter

568 Maths Quest 10 New South Wales Australian curriculum edition Stages 5.1 and 5.2
STATistics and probability

3    a Mean = $215 000, median = $170 000, mode = $150 000. 2    a 50.5
The median best represents these land prices. The mean is b, c, d Answers will vary.
inflated by one large score, and the mode is the lowest price. 3    a Survey
b Range = $500 000, interquartile range = $30 000. The b Census
interquartile range is the better measure of spread. c Survey
c 4 Use a spinner of 3 equal sectors, each sector having an angle size
of 120° and representing a particular colour. Twirl the spinner
150 000 300 000 450 000 600 000 until a green/green combination has been obtained. This is
Price
defined as one experiment. Count the number of trials required
This dot plot shows how 9 of the scores are grouped close for this experiment. Repeat this procedure a number of times and
together, while the score of $650  000 is an outlier. determine an average.
d The agent is quoting the modal price, which is the lowest 5 D
price. This is not a true reflection of the average price of these 6 Check with your teacher.
blocks of land. a This graph should look relatively flat, with little decline in the
4    a True. Mean = 1.82 m, lower quartile = 1.765 m, Years 11 and 12 region.
median = 1.83 m b This graph should show a sharp decline in the Years 11 and 12
b True. This is the definition of interquartile range. region.
c Players with heights 1.83 m, 1.83 m, 1.88 m, 1.88 m, 1.88 m 7    a Boys: median = 26; girls: median = 23.5
5    a 7.1 b Boys: range = 32; girls: range = 53
b 7 c Both sets have similar medians, but the girls have a greater
c 7 range of absenteeism than the boys.
d The mode has the most meaning as this size sells the most. 8    a The sample is an appropriate size as !900 = 30.
6 Check with your teacher. Answers depend on groupings used. b Key: 16 | 1 = 1.61
7   Player B appears to be the better player if the mean result is used.
However, Player A is the more‐consistent player. Leaf Stem Leaf
8    a The statement is true, but misleading as most of the Boys Girls
employees earn $18  000. 997 15 1256788
b The median and modal salary is $18  000 and only 15 out of 80 98665540 16 4467899
(less than 20%) earn more than the mean. 4421 17 0
9 Points which could be mentioned.
• 10.1% is only just ‘double digit’ growth. c The boys are generally better than the girls, with
• 2006–08 showed mid to low 20% growth. Growth has been the mean of the boys being 1.66 m and that of the
declining since 2008. girls being 1.62 m. The five‐number summaries are:
• Share price has rebounded, but not to its previous high. Boys: 1.57 m, 1.6 m, 1.66 m, 1.71 m, 1.74 m
• Share price scale is not consistent. Most increments are 30c, Girls: 1.51 m, 1.56 m, 1.64 m, 1.68 m, 1.7 m
except for $27.70 to $28.10 (40c increment). Note also the
figure of 20.80 — probably a typo instead of 26.80. 9    a Ford: median = 15; Holden: median = 16
10 Shorten the y-axis and expand the x-axis. b Ford: range = 26; Holden: range = 32
c Ford: IQR = 14; Holden: IQR = 13.5
US c
Aussie dollar d Ford
Holden
90 c
0 5 10 15 20 2530 35 40
Number of vehicles sold
80 c
10    a Brisbane Lions
b Brisbane Lions: range = 65;
0 13 July 13 September Sydney Swans: range = 55
Time c Brisbane Lions: IQR = 40;
Exercise 15D — Statistical investigations Sydney Swans: IQR = 35
The questions in this exercise relate to student investigations, so there 11    a Girls
will be a variety of answers. Boys
2 There have been 27 Prime Ministers of Australia since 1901 until 1.4 1.5 1.6 1.7 1.8 1.9
this day. Height (m)
There have been 42 elections. b Boys: median = 1.62 m; girls: median = 1.62 m
10 Prime Ministers have been defeated at a general election. c Boys: range = 0.36 m; girls: range = 0.23 m
There have been 21 changes of Prime Minister without an election. d Boys: IQR = 0.14 m; girls: IQR = 0.11 m
The average length these Prime Ministers served in office is (This e Although boys and girls have the same median height, the
changes daily). spread of heights is greater among boys as shown by the
Challenge 15.2 greater range and interquartile range.
12    a Summer: range = 23; winter: range = 32
2, 4, 10, 10, 10; 6 solutions
b Summer: IQR = 13; winter: IQR = 11
Chapter review c There are generally more cold drinks sold in summer
Fluency as shown by the higher median. The spread of data is
1    a You would need to open every can to determine this. similar as shown by the IQR although the range in winter is
b Fish are continuously dying, being born, being caught. greater.
c Approaching work places and public transport offices 13   A

Chapter 15 • Statistics in the media 569


STATistics and probability

Problem solving 4 They are all made in Australia and have comparable fat and
1    a Mean = 32.03; median = 29.5 saturated fat contents. The Byron Bay Chilli corn chips have a
b much lower salt content than the other three varieties. The verdict
Class interval Frequency comments require a mention.
0–9 2 5 Student’s plan for an investigation.
10–19 7 Communicating — Rich task
20–29 6 1 Data for the whole population
30–39 6 Min 24
40–49 3 Q1 61.75
50–59 3 Median 87
Q3 98
60–69 3
Max 100
Total 30
y
c Mean = 31.83
d 1
30
Cumulative frequency

25
1.8
20
15
1.6
10
5 1.4
0
10 20 30 40 50 60 70
Age 1.2
e Median = 30
f Estimates from parts c and e were fairly accurate. 0 20 40 60 80 100 x
g Yes, they were fairly close to the mean and median of the raw 2 Mean = 77.85; Standard deviation = 23
data.
2 Year 8: mean = 26.83, median = 27, range = 39, IQR = 19 3–10 Answers will vary.
Year 10: mean = 40.7, median = 39.5, range = 46, IQR = 20 Code puzzle
The typing speed of Year 10 students is about 13 to 14 wpm Koala, emu, kangaroo and kookaburra
faster than that of Year 8 students. The spread of data in Year 8 is
slightly less than the spread in Year 10.
3 Hane and Roarne had a higher median and a lower spread, so
they appear to have performed better.

570 Maths Quest 10 New South Wales Australian curriculum edition Stages 5.1 and 5.2
ICT activity
islands, such as Tuvalu Island in the Pacific Ocean, to
announce that they are abandoning their homeland
due to rising sea levels. In Sydney, many well known
Climate change suburbs could be threatened by rising sea levels in the
future, including Caringbah, Kurnell, Cromer, Manly
SEARCHLIGHT ID: PRO‐0100
Vale, Newcastle, the central coast, Homebush Bay,
Newington Silverwater, Arncliffe, Marrickville and Sydney
Scenario
Airport. Climate change could cause the extinction of
Climate change is upon us and has become one of the many species as ecosystems are damaged by rising
great challenges facing humanity. Our fossil fuel driven temperatures.
economies are producing large volumes of greenhouse In order to address these apocalyptic issues, we
gases (water vapour, carbon dioxide, methane and need to understand the role of human activity in climate
ozone) that are warming the planet. As our planet heats change. It will be your job to investigate and understand
up, the ice sheets at the poles slowly melt, causing sea the relationships that underpin global warming. You can
levels to rise. Islands in the Pacific Ocean are already then make recommendations to our political leaders and
being overcome by water, leading the inhabitants of take action yourself to help save our planet.

572 Maths Quest 10 New South Wales Australian curriculum edition Stages 5.1 and 5.2
Task • You must have a gmail
You will need to analyse real data sets to develop account and internet access suggested
a mathematical understanding of climate change to use the Google data software
issues. The analysis will involve the use of scatter plots, tools. • Microsoft Excel
box‐and‐whisker plots and five number summaries. • You will need Microsoft Excel • Geogebra
Scatter plots will be used to investigate and comment and GeoGebra installed • Internet
on relationships between two climate change variables. on your computer. Go to connection
Data sets will be compared using box‐and‐whisker projectsPLUS on your eBook, • Internet browser
plots, dot plots and histograms. Environmental data will set up a group and then with Adobe
be graphed, such that the independent variable is time. open the Media Centre to Flash player
At the end of your project, your improved mathematical locate everything you need. installed.
understanding of climate change will allow you to • Open the Word documents • Use the World
make key recommendations on how we can meet the titled Lesson 1, Lesson 2 etc. Bank weblink in
environmental challenges of the future. Follow the instructions in your eBookPLUS
each document to complete to locate banks
your project. of data in Excel
Process • At various stages of your form.
You will use Microsoft Excel, Google Fusion Tables project, you will need to
and Google Public Data Explorer to investigate global access data sets in
environmental data. Microsoft Excel files.

ICT activity — projectsplus 573


number and algebra

Chapter 16

Financial
mathematics
WHY LEARN THIS?
Everyone requires food, housing, clothing and transport,
and a fulfilling social life. Money allows us to purchase
the things we need and desire. The ability to manage
money is key to a financially secure future and a
reasonable retirement with some fun along the way.
Each individual is responsible for managing his or her
own finances; therefore, it is imperative that everyone is
financially literate.

WHAT DO YOU KNOW?


1 Think List what you know about financial
mathematics. Use a thinking tool such
as a concept map to show your list.
2 pair Share what you know with a partner
and then with a small group.
3 share As a class, create a thinking tool such
as a large concept map that shows your class’s
knowledge of financial mathematics.

LEARNING SEQUENCE
16A Purchasing goods
16B Buying on terms
16C Successive discounts
16D Compound interest
16E Depreciation
16F Loan repayments
Watch this video
The story of mathematics

Searchlight ID: eles-1855


number and algebra

16A Purchasing goods


•• There are many different payment options when purchasing major goods, such as flat
screen televisions and computers. Payment options include:
–– cash –– credit card –– lay‐by
–– deferred payment –– buying on terms –– loan.
•• The cost of purchasing an item can vary depending on the method of payment used.
•• Some methods of payment involve borrowing money and, as such, mean that interest
is charged on the money borrowed.
•• The simple interest formula can be used to calculate the interest charged on borrowed
money,
P×r×T
I=
100
where: I is the simple interest ($)
P is the principal or amount borrowed or invested ($)
r is the rate of interest per time period
T is the time for which the money is invested or borrowed.
If T is in years, then r is the rate of interest per annum (% p.a.).
WORKED EXAMPLE 1
Find the simple interest on $4000 invested at 4.75% p.a. for 4 years.
THINK WRITE
1 Write the formula and the known values of P×r×T
I= , where
the variables. 100
P = $4000, r = 4.75%, T = 4
2 Substitute known values to find I. $4000 × 4.75 × 4
I=
100
3 Calculate the value of I. = $760

•• What are the ways of purchasing the item shown in the advertisement below?

120 cm HD TV
5 year
ty
warran

■ High definition
■ HDMI ports
■ 16 : 9 aspect ratio
■ 1080i

$1200

576 Maths Quest 10 New South Wales Australian curriculum edition Stages 5.1 and 5.2
number and algebra

Payment options
Cash
•• With cash, the marked price is paid on the day of purchase with nothing more to pay.
•• A cash‐paying customer can often negotiate, with the retailer, to obtain a lower price
for the item.

Lay‐by
•• With lay-by, the item is held by the retailer while the customer makes regular
payments towards paying off the marked price.
•• In some cases a small administration fee may be charged.

Credit cards
•• With a credit card, the retailer is paid by the credit card provider, generally a
financial lender.
•• The customer takes immediate possession of the goods.
•• The financial lender later bills the customer — collating all purchases over a monthly
period and billing the customer accordingly. The entire balance shown on the bill can
often be paid with no extra charge, but if the balance is not paid in full, interest is
charged on the outstanding amount, generally at a very high rate.

WORKED EXAMPLE 2
The ticketed price of a mobile phone is $600. Andrew decides to purchase
the phone using his credit card. After 1 month the credit card company
charges interest at a rate of 15% p.a. Calculate the amount of interest that
Andrew must pay on his credit card after 1 month.
THINK WRITE
1 Write the formula and the known values P×r×T
I=
of the variables. Remember that 100
1
1 month = 12 year. 1
P = $600, r = 15%, T = 12

2 Substitute known values to find I. 600 × 15 × 1


I=
100 × 12
3 Calculate the value of I. = $7.50

Exercise 16A Purchasing goods


INDIVIDUAL PATHWAYS
Questions: Questions: Questions:
1–3, 5, 7, 10 1–4, 6, 8, 10 1–3, 5, 7, 9, 10
Activity 16‐A‐1 Activity 16‐A‐2 Activity 16‐A‐3
Simple interest Harder simple interest Tricky simple interest
doc‐8488 doc‐8489 doc‐8490

Chapter 16 • Financial mathematics 577


number and algebra

FLUENCY
Digital docs
1 WE1 Find the simple interest payable on a loan of $8000 at 6% p.a. for 5 years.
SkillSHEET 2 Find the simple interest on each of the following loans.
Converting a
­Percentage to a decimal a $5000 at 9% p.a. for 4 years b $4000 at 7.5% p.a. for 3 years
doc‐5345 1 1
SkillSHEET c $12  000 at 6.4% p.a. for 2 2 years d $6000 at 8% p.a. for 1 2 years
Finding simple interest 3 Find the simple interest on each of the following investments.
doc‐5346
a $50  000 at 6% p.a. for 6 months b $12  500 at 12% p.a. for 1 month
c $7500 at 15% p.a. for 3 months d $4000 at 18% p.a. for 18 months
4 Calculate the monthly interest charged on each of the following outstanding credit
card balances.
a $1500 at 15% p.a. b $4000 at 16.5% p.a.
c $2750 at 18% p.a. d $8594 at 17.5% p.a.
e $5690 at 21% p.a.

UNDERSTANDING
5 WE2 The ticketed price of a mobile phone is $800. Elena decides to purchase the
phone using her credit card. After 1 month the credit card company charges interest
at a rate of 15% p.a. Calculate the amount of interest that Elena must pay on her
credit card after 1 month.
6 Reece decides to purchase a new sound system using her credit card. The ticketed
price of the sound system is $900. When Reece’s credit card statement arrives, it
shows that she will pay no interest if she pays the full amount by the due date.

a If Reece pays $200 by the due date, what is the balance owing?
b If the interest rate on the credit card is 18% p.a., how much interest will Reece be
charged in the month?
c What will be the balance that Reece owes at the end of the month?
d At this time Reece pays another $500 off her credit card. How much interest is
Reece then charged for the next month?
e Reece then pays off the entire remaining balance of her card. What was the true
cost of the sound system including all the interest payments?

578 Maths Quest 10 New South Wales Australian curriculum edition Stages 5.1 and 5.2
number and algebra

7 Carly has an outstanding balance of $3000 on her credit card for June and is
charged interest at a rate of 21% p.a.
a Calculate the amount of interest that Carly is charged for June.
b Carly makes the minimum repayment of $150 and makes no other purchases
using the credit card in the next month. Calculate the amount of interest that
Carly will be charged for July.
c If Carly had made a repayment of $1000 at the end of June, calculate the amount
of interest that Carly would then have been charged for July.
d How much would Carly save in July had she made the higher repayment at the
end of June?
8 Arup buys a new home theatre system using his credit card. The ticketed price
of the bundle is $7500. The interest rate that Arup is charged on his credit card is
18% p.a. Arup pays off the credit card at a rate of $1000 each month.
a Complete the table below.
Month Balance owing Interest Payment Closing balance
January $7500.00 $112.50 $1000.00 $6612.50
February $6612.50 $99.19 $1000.00
March $1000.00
April $1000.00
May $1000.00
June $1000.00
July $1000.00
August $1015.86 $0
b What is the total amount of interest that Arup pays?
c What is the total cost of purchasing the home theatre system using his credit card?

REASONING
9 Design a table that compares the features of each method of payment: cash, lay‐by
and credit card.
10 Choose the most appropriate method of payment for each of the described
scenarios below. Explain your choice.
Scenario 1: Andy has no savings and will not be paid for another two weeks. Andy
would like to purchase an HD television
and watch tomorrow’s football final. REFLECTION
Scenario 2: In September Lena spots on special a What can you do to
remember the simple
home theatre system which she would like interest formula?
to purchase for her family for Christmas.

16B Buying on terms


•• When buying an item on terms:
–– a deposit is paid
–– the balance is paid off over an agreed period of time with set payments
–– the set payments may be calculated as a stated arbitrary amount or interest rate
–– total monies paid will exceed the initial cash price.

Chapter 16 • Financial mathematics 579


number and algebra

WORKED EXAMPLE 3
The cash price of a computer is $2400. It can also be purchased on the
following terms: 25% deposit and payments of $16.73 per week for 3 years.
Calculate the total cost of the computer purchased on terms as described.
THINK WRITE
1 Calculate the deposit. Deposit = 25% of $2400
= 0.25 × $2400
= $600
2 Calculate the total of the weekly Total repayment = $16.73 × 52 × 3
repayments. = $2609.88
3 Add these two amounts together to Total cost = $600 + $2609.88
find the total cost. = $3209.88

WORKED EXAMPLE 4
A diamond engagement ring has a purchase
price of $2500. Michael buys the ring on the
following terms: 10% deposit with the balance
plus simple interest paid monthly at 12% p.a.
over 3 years.
a Calculate the amount of the deposit.
b What is the balance owing after the initial
deposit?
c Calculate the interest payable.
d What is the total amount to be repaid?
e Find the amount of each monthly repayment.

THINK WRITE
a Calculate the deposit by finding 10% a Deposit = 10% of $2500
of $2500. = 0.1 × $2500
= $250
b Find the balance owing by subtracting b Balance = $2500 − $250
the deposit from the purchase price. = $2250
P×r×T
c Find the simple interest on $2250 at c I= , where P = $2250,
12% p.a. for 3 years. 100
r = 12%, T = 3
= $2250 × 0.12 × 3
= $810
d Find the total repayment by adding the d Total repayment = $2250 + $810
balance owing with the interest payable. = $3060
e Find the monthly repayment by e Monthly repayment = $3060 ÷ 36
dividing the total repayment by the = $85
number of months over which the
ring is to be repaid.

580 Maths Quest 10 New South Wales Australian curriculum edition Stages 5.1 and 5.2
number and algebra

Loans
•• Money can be borrowed from a bank or other financial institution.
•• Interest is charged on the amount of money borrowed.
•• Both the money borrowed and the interest charged must be paid back.
•• The interest rate on a loan is generally lower than the interest rate offered on a credit
card or when buying on terms.
•• The calculation of loan payments is done in the same way as for buying on terms;
that is, calculate the interest and add it to the principal before dividing into equal
monthly repayments.

Exercise 16B Buying on terms


INDIVIDUAL PATHWAYS
Questions: Questions: Questions:
1–4, 8, 10, 13, 14 1–4, 6, 8, 10, 12–14 1–5, 8–18
Activity 16‐B‐1 Activity 16‐B‐2 Activity 16‐B‐3
Buying on terms Buying on difficult terms Buying on tricky terms
doc‐8491 doc‐8492 doc‐8493

FLUENCY
1 Calculate the total cost of a $3000 purchase given the terms described below.
a i 12% deposit and monthly payments of $60 over 5 years
Digital doc
ii 20% deposit and weekly payments of $20 over 3 years SkillSHEET
iii 15% deposit and annual payments of $700 over 5 years Finding a percentage of
a quantity (money)
b Which of these options is the best deal for a purchaser? doc‐5347

2 Calculate the amount of each repayment for a $5000 purchase given the terms
described below.
a 10% deposit with the balance plus simple interest paid monthly at 15% p.a. over
5 years
b 10% deposit with the balance plus simple interest paid fortnightly at 12% over 5 years
c 20% deposit with the balance plus simple interest paid monthly at 10% over 3 years
3 Calculate the total repayment and the amount of each monthly repayment for each of
the following loans.
a $10  000 at 9% p.a. repaid over 4 years
b $25  000 at 12% p.a. repaid over 5 years
c $4500 at 7.5% p.a. repaid over 18 months
d $50  000 at 6% p.a. repaid over 10 years
e $200  000 at 7.2% p.a. repaid over 20 years

UNDERSTANDING
4 WE3 The cash price of a bedroom suite is $4200.
The bedroom suite can be purchased on the following
terms: 20% deposit and weekly repayments of $43.94
for 2 years. Calculate the total cost of the bedroom
suite if you bought it on terms.

Chapter 16 • Financial mathematics 581


number and algebra

5 Guy purchases a computer that has a cash price of $3750 on the following terms:
$500 deposit with the balance plus interest paid over 2 years at $167.92 per month.
What is the total amount that Guy pays for the computer?
6 Raj wants to buy a used car with a cash price of $12  600. The dealer offers terms of
10% deposit and monthly repayments of $812.70 for 2 years.
a Calculate the amount of the deposit.
b Calculate the total amount to be paid in monthly repayments.
c What is the total amount Raj pays for the car?
d How much more than the cash price of the car does Raj pay? (This is the interest
charged by the dealer.)
7 Alja wants to purchase an entertainment system that has a cash price of $5800. She
purchases the entertainment system on terms of no deposit and monthly repayments
of $233.61 for 3 years.
a Calculate the total amount that Alja pays for the entertainment system.
b Calculate the amount that Alja pays in interest.
c Calculate the amount of interest that Alja pays each year.
d Calculate this amount as a percentage of the cash price of the entertainment
system.
8 WE4 A used car has a purchase price of $9500. Dayna buys the car on the
following terms: 25% deposit with balance plus interest paid at 12% p.a. interest
over 3 years.
a Calculate the amount of the deposit.
b What is the balance owing?
c Calculate the interest payable.
d What is the total amount to be repaid?
e Find the amount of each monthly repayment.
9 A department store offers the following terms: one‐third deposit with the balance
plus interest paid in equal, monthly instalments over 18 months. The interest rate
charged is 9% p.a. Ming buys a lounge suite with a ticketed price of $6000.
a Calculate the amount of the deposit.
b What is the balance owing?
c Calculate the interest payable.
d What is the total amount to be repaid?
e Find the amount of each monthly repayment.
10 Calculate the monthly payment on
each of the following items bought
on terms.
(Hint: Use the steps shown in
question 8.)
a Dining suite: cash price $2700,
deposit 10%, interest rate
12% p.a., term 1 year

582 Maths Quest 10 New South Wales Australian curriculum edition Stages 5.1 and 5.2
number and algebra

b Video camera: cash price $990, deposit 20%, interest rate 15% p.a., term
6 months

c Car: cash price $16  500, deposit 25%, interest rate 15% p.a., term 5 years

d Mountain bike: cash price $3200, one‐third deposit, interest rate 9% p.a., term
2 12 years

e Watch: cash price $675, no deposit, interest rate


18% p.a., term 9 months

Chapter 16 • Financial mathematics 583


number and algebra

11 Samir wants to purchase his first car. He has saved $1000 as a deposit but the cost
of the car is $5000. Samir takes out a loan from the bank to cover the balance of the
car plus $600 worth of on‐road costs.
a How much will Samir need to borrow from the bank?
b Samir takes the loan out over 4 years at 9% p.a. interest. How much interest will
Samir need to pay?
c What will be the amount of each monthly payment that Samir makes?
d What is the total cost of the car after paying off the loan, including the on‐road
costs? Give your answer to the nearest $.
12 MC Kelly wants to borrow $12  000 for some home improvements. Which of the
following loans will lead to Kelly making the lowest total repayment?
A Interest rate 6% p.a. over 4 years B Interest rate 7% p.a. over 3 years
1
C Interest rate 5.5% p.a. over 32 years D Interest rate 6.5% p.a. over 5 years
E Interest rate 7.5% p.a. over 3 years

REASONING
13 MC Without completing any calculations explain which of the following loans
will be the best value for the borrower.
A Interest rate 8.2% p.a. over 5 years
B Interest rate 8.2% over 4 years
C Interest rate 8% over 6 years
D Interest rate 8% over 5 years REFLECTION
E Interest rate 8% over 4 years When buying on terms,
what arrangements are
Digital doc 14 Explain how, when purchasing an item, making a
the most beneficial to
WorkSHEET 16.1 deposit using existing savings and taking out a loan for the buyer?
doc‐5350
the balance can be an advantage.

584 Maths Quest 10 New South Wales Australian curriculum edition Stages 5.1 and 5.2
number and algebra

16C Successive discounts


•• Consider the case of Ziggy who is a mechanic.
Ziggy purchases his hardware from Tradeways
hardware store, which is having a 10%‐off sale.
Tradeways also offers a 5% discount to
tradespeople. Ziggy purchases hardware that
has a total value of $800. What price does
Ziggy pay for these supplies?
After the 10% discount, the price of the
supplies is
90% of $800 = 0.90 × $800
= $720
The 5% trades discount is then applied.
95% of $720 = 0.95 × $720
= $684
So the price ziggy pays is $684.
Now let us consider what single discount Ziggy has actually received.
Amount of discount = $800 − $684
= $116
$116
Percentage discount = × 100%
$800
= 14.50%
So we can conclude that the successive discounts of 10% followed by a further 5% is
equivalent to receiving a single discount of 14.50%.
•• When two discounts are applied one after the other, the total discount is not the same
as a single discount found by adding the two percentages together.
•• The order of calculating successive discounts does not affect the final answer.

WORKED EXAMPLE 5
A furniture store offers a discount of 15% during a sale. A further 5%
discount is then offered to customers who pay cash.
a Find the price paid by Lily, who pays cash for a bedroom suite priced
at $2500.
b What single percentage discount does Lily receive on the price of the
bedroom suite?
THINK WRITE
a 1 Subtract 15% from 100% to a 100% − 15% = 85%
find the percentage paid.
2 Calculate 85% of the price. 85% of $2500 = 0.85 × $2500
= $2125
3 Subtract 5% from 100% to 100% − 5% = 95%
find the next percentage paid.
4 Calculate 95% of $2125. 95% of $2125 = 0.95 × $2125
= $2018.75

Chapter 16 • Financial mathematics 585


number and algebra

b 1 Calculate the amount of b Discount = $2500 − $2018.75


discount received. = $481.25
2 Express the discount as a $481.25
Percentage discount = × 100%
percentage of the original $2500
marked price. = 19.25%

•• The single discount that is equivalent to successive discounts can also be worked out
by working out a percentage of a percentage, as shown in the worked example below.

WORKED EXAMPLE 6
Find the single percentage discount that is equivalent to successive
discounts of 15% and 5%.
THINK WRITE
1 Subtract 15% from 100% to find the 100% − 15% = 85%
percentage paid after the first discount.
2 Subtract 5% from 100% to find the percentage 100% − 5% = 95%
paid after the second discount.
3 Find 95% of 85%. This is actually the 95% of 85% = 0.95 × 0.85
percentage of the marked price that the = 0.8075
customer pays. = 80.75%
4 Subtract the percentage from 100% to find Discount = 100% − 80.75%
the single percentage discount. This answer = 19.25%
should be less than 15% + 5%.

Exercise 16C Successive discounts


INDIVIDUAL PATHWAYS
Questions: Questions: Questions:
1–4, 6, 8, 10, 11 2–7, 9–11 2–6, 8–12
Activity 16‐C‐1 Activity 16‐C‐2 Activity 16‐C‐3
Successive discounts Difficult successive discounts Tricky successive discounts
doc‐8494 doc‐8495 doc‐8496

Digital docs
FLUENCY
SkillSHEET 1 In each of the following, an item is reduced in price.
Finding percentage
discount
Calculate the percentage discount, correct to
doc‐5348 1 decimal place.
SkillSHEET
Decreasing a quantity
a A jumper, usually $29.95, is reduced to $24.95.
by a percentage b A video game, usually $60, is reduced to $53.90.
doc‐5349
SkillSHEET c A child’s bike, usually $158, is reduced to $89.
Expressing one ­quantity
as a percentage of
d A new car, usually $29  500, is reduced to $24  950.
another e A plot of land, priced at $192  000, is reduced to
doc‐5351
$177  500 for a quick sale.

586 Maths Quest 10 New South Wales Australian curriculum edition Stages 5.1 and 5.2
number and algebra

2 WE6 Calculate the single percentage discount that is equivalent to successive


discounts of 15% and 10%.
3 MC The single percentage discount that is equivalent to successive discounts of
10% and 20% is:
A 10% B 18%
C 28% D 30%
E 35%
4 Find the single percentage discount that is equivalent to each of the following
successive discounts.
a 15% and 20% b 12% and 8%
c 10% and 7.5% d 50% and 15%
5 Calculate the single percentage discount that is equivalent to two successive 10%
discounts.

UNDERSTANDING
6 WE5 A supplier of electrical parts offers tradespeople a 20% trade discount. If
accounts are settled within 7 days, a further 5% discount is given.
a Calculate the price paid by an electrician for parts to the value of $4000 if the
account is settled within 7 days.
b What single percentage discount does the electrician receive on the price of the
electrical parts?
7 At a confectionary wholesaler, customers
have their accounts reduced by 10% if they
are paid within 7 days.
a Jacinta pays her $100 account within
7 days. How much does she actually pay?
b If customers pay cash, they receive a
further 5% discount. How much would
Jacinta pay if she pays cash?
c By how much in total has her account been
reduced?
d What is the single percentage discount
equivalent to these successive discounts?
8 A fabric supplier offers discounts to fashion stores and
a further discount if the store’s account is paid with
14 days. ‘David’s Fashion Stores’ have ordered fabric
to the value of $2000 from the fabric supplier.
a If fashion stores receive a reduction of 8%, how
much does ‘David’s Fashion Stores’ owe on its
account?
b This amount is reduced by a further 5% for payment
within 14 days. How much needs to be paid now?
c What has been the total reduction in the cost?
d What do the successive discounts of 8% and 5% equal
as a single percentage discount?

Chapter 16 • Financial mathematics 587


number and algebra

9 Tony is a mechanic who wants to buy equipment worth $250 at a hardware store.
Tony receives 15% off the marked price of all items and then a further 5% trade
discount.
a Calculate the amount that is due after Tony is given the first 15% discount.
b From this amount, apply the trade discount of 5% to find the amount due.
c How much is the cash discount that Tony receives?
d Calculate the amount that would have been due had Tony received a single
discount of 20%. Is this the same answer?
e Calculate the amount of cash discount that Tony receives as a percentage of the
original bill.
f Would the discount have been the same had the 5% discount been applied before
the 15% discount?
g Calculate the single percentage discount that is equivalent to successive
discounts of 10% and 20%.
10 A car has a marked price of $25  000.
a Find the price paid for the car after successive discounts of 15%, 10% and 5%.
b What single percentage discount is equivalent to successive discounts of 15%,
10% and 5%?

REASONING
11 Is a 12.5% discount followed by a 2.5% discount the same single discount as a
2.5% discount followed by a 12.5% discount? Investigate and explain your answer
giving mathematical evidence.
12 Derive a mathematical formula to calculate the REFLECTION
In what situations might
single discount (expressed as a decimal) generated
a successive discount be
by two successive discounts, a and b (expressed as applied?
decimals).

16D Compound interest


•• Interest on the principal in a savings account, or short or long term deposit, is
generally calculated using compound interest rather than simple interest.
Interactivity •• When interest is added to the principal at regular intervals, increasing the balance of
Compound interest
int‐2791 the account, and each successive interest payment is calculated on the new balance, it
is called compound interest.
•• Compound interest can be calculated by calculating simple interest one period at a
time.
•• The amount to which the initial investment grows is called the compounded value
or future value.

WORKED EXAMPLE 7
Kyna invests $8000 at 8% p.a. for 3 years with interest paid at the end of
each year. Find the compounded value of the investment by calculating the
simple interest on each year separately.

588 Maths Quest 10 New South Wales Australian curriculum edition Stages 5.1 and 5.2
number and algebra

THINK WRITE
1 Write the initial (first year) Initial principal = $8000
principal.
2 Calculate the interest for the first Interest for year 1 = 8% of $8000
year. = $640
3 Calculate the principal for the Principal for year 2 = $8000 + $640
second year by adding the first = $8640
year’s interest to the initial
principal.
4 Calculate the interest for the Interest for year 2 = 8% of $8640
second year. = $691.20
5 Calculate the principal for the Principal for year 3 = $8640 + $691.20
third year by adding the second = $9331.20
year’s interest to the second
year’s principal.
6 Calculate the interest for the Interest for year 3 = 8% of $9331.20
third year. = $746.50
7 Calculate the future value of the Compounded value after 3 years
investment by adding the third = $9331.20 + $746.50
year’s interest to the third year’s = $10 077.70
principal.

•• To calculate the actual amount of interest received, we subtract the initial principal
from the future value.
•• In the example above, compound interest = $10 077.70 − $8000
= $2077.70
We can compare this with the simple interest earned at the same rate.
P×r×T
I=
100
8000 × 8 × 3
=
100
= $1920
•• The table below shows a comparison between the interest earned on an investment of
$8000 earning 8% p.a. at both simple interest (I) and compound interest (CI) over an
eight year period.

Year 1 2 3 4 5 6 7 8
Total (I ) $640.00 $1280.00 $1920.00 $2560.00 $3200.00 $3840.00 $4480.00 $5120.00
Total (CI ) $640.00 $1331.20 $2077.70 $2883.91 $3754.62 $4694.99 $5710.59 $6807.44

•• We can develop a formula for the future value of an investment rather than do each
example by repeated use of simple interest. Consider Worked example 7. Let the
compounded value after each year, n, be An.

Chapter 16 • Financial mathematics 589


number and algebra

After 1 year, A1 = 8000 × 1.08 (increasing $8000 by 8%)


After 2 years, A2 = A1 × (1.08)
= 8000 × 1.08 × 1.08 (substituting the value of A1)
= 8000 × 1.082
After 3 years, A3 = A2 × 1.08
= 8000 × 1.082 × 1.08 (substituting the value of A2)
= 8000 × 1.083
The pattern then continues such that the value of the investment after n years equals:
$8000 × 1.08n.
•• This can be generalised for any investment:
│ A = P(1 + R) n│
where A = amount (or future value) of the investment
P = principal (or present value)
R = interest rate per compounding period expressed as a decimal
n = number of compounding periods.
•• To calculate the amount of compound interest (CI) we then use the formula
│ CI = A − P │

Using technology
•• Digital technologies such as spreadsheets can be used to draw graphs in order to
compare interest accrued through simple interest and compound interest.
Comparison of $8000 invested at 8% p.a. simple and compound
interest
16 000.00

14 000.00

12 000.00

10 000.00
Amount ($)

Amount after simple


interest ($)
8000.00
Amount after compound
6000.00 interest ($)

4000.00

2000.00

0.00
0 1 2 3 4 5 6 7 8
Year

WORKED EXAMPLE 8
William has $14  000 to invest. He invests the money at 9% p.a. for 5 years
with interest compounded annually.
a Use the formula A = P(1 + R) n to calculate the amount to which this
investment will grow.
b Calculate the compound interest earned on the investment.

590 Maths Quest 10 New South Wales Australian curriculum edition Stages 5.1 and 5.2
number and algebra

THINK WRITE
a 1 Write the compound a A = P(1 + R) n
interest formula.
2 Write down the values of P = $14 000, R = 0.09, n = 5
P, R and n.
3 Substitute the values into A = $14000 × 1.095
the formula.
4 Calculate. = $21 540.74
The investment will grow to $21 540.74.
b Calculate the compound interest b CI = A − P
earned. = $21 540.74 − $14 000
= $7540.74
The compound interest earned is $7540.74.

Comparison of fixed principal at various interest


rates over a period of time
•• It is often helpful to compare the future value ($A) of the principal at different
compounding interest rates over a fixed period of time.
•• Spreadsheets are very useful tools for making comparisons. The graph below,
generated from a spreadsheet, shows the comparisons for $14  000 invested for
5 years at 7%, 8%, 9% and 10% compounding annually.
•• There is a significant difference in the future value depending on which interest rate
is applied.
$14 000 invested at 7%, 8%, 9% and 10% p.a. over five years
23 000
22 000 Amount after
21 000 CI @ 7% p.a. ($)

20 000 Amount after


Amount ($)

CI @ 8% p.a. ($)
19 000
Amount after
18 000 CI @ 9% p.a. ($)
17 000 Amount after
16 000 CI @ 10% p.a. ($)

15 000
14 000
0 1 2 3 4 5
Year

Compounding period
•• In Worked example 8 interest is paid annually.
•• Interest can be paid more regularly — it may be paid six‐monthly (twice a year),
quarterly (4 times a year), monthly or even daily. This is called the compounding
period.
•• The time and interest rate on an investment must reflect the compounding period. For
example, an investment over 5 years at 6% p.a. compounding quarterly will have:
n = 20 (5 × 4) and R = 0.015 (6% ÷ 4).

Chapter 16 • Financial mathematics 591


number and algebra

•• To find n:
n = number of years × compounding periods per year
•• To find R:
R = interest rate per annum ÷ compounding periods per year

WORKED EXAMPLE 9
Calculate the future value of an investment of $4000 at 6% p.a. for 2 years
with interest compounded quarterly.
THINK WRITE
1 Write the compound interest A = P(1 + R) n
formula.
2 Write the values of P, R and n. P = $4000, R = 0.015, n = 8
3 Substitute the values into the A = $4000 × 1.0158
formula.
4 Calculate. = $4505.97
The future value of the investment is $4505.97.

Guess and refine


•• Sometimes it is useful to know approximately how long it will take to reach a
particular future value once an investment has been made. Mathematical formulas
can be applied to determine when a particular future value will be reached. In this
section, a ‘guess and refine’ method will be shown.
•• For example, to determine the number of years required for an investment of $1800
at 9% compounded quarterly to reach a future value of $2500, the following method
can be used.
Let n = the number of compounding periods (quarters) and A = the future value in $.

n A = P(1 + R/4) n Comment


1 $1840.50 It is useful to know how the principal is growing
after 1 year, but the amount is quite far from $2500.
3 $1924.25 The amount is closer to $2500 but still a long way
off, so jump to a higher value for n.
10 $2248.57 The amount is much closer to $2500.
12 $2350.89 The amount is much closer to $2500.
14 $2457.87 The amount is just below $2500.
15 $2513.17 The amount is just over $2500.

Therefore, it will take approximately 15 quarters, or 3 years and 9 months, to reach


the desired amount.

592 Maths Quest 10 New South Wales Australian curriculum edition Stages 5.1 and 5.2
number and algebra

Exercise 16D Compound interest


INDIVIDUAL PATHWAYS
Questions: Questions: Questions:
1–3, 5, 6, 11, 13, 15 1–6, 7, 9, 10, 12–16 1–8, 11, 12, 14–17
Activity 16‐D‐1 Activity 16‐D‐2 Activity 16‐D‐3
Compound interest puzzle 1 Compound interest puzzle 2 Compound interest puzzle 3
doc‐8497 doc‐8498 doc‐8499

FLUENCY
1 Use the formula A = P(1 + R) n to calculate the amount to which each of the
following investments will grow with interest compounded annually.
a $3000 at 4% p.a. for 2 years b $9000 at 5% p.a. for 4 years
c $16  000 at 9% p.a. for 5 years d $12  500 at 5.5% p.a. for 3 years
e $9750 at 7.25% p.a. for 6 years f $100  000 at 3.75% p.a. for 7 years
2 Calculate the compounded value of each of the following investments.
a $870 for 2 years at 3.50% p.a. with interest compounded six‐monthly
1
b $9500 for 2 2 years at 4.6% p.a. with interest compounded quarterly
1
c $148  000 for 3 2 years at 9.2% p.a. with interest compounded six‐monthly
d $16  000 for 6 years at 8% p.a. with interest compounded monthly
e $130  000 for 25 years at 12.95% p.a. with interest compounded quarterly

UNDERSTANDING
3 WE7 Danielle invests $6000 at 10% p.a. for 4 years with interest paid at the end of
each year. Find the compounded value of the investment by calculating the simple
interest on each year separately.
4 Ben is to invest $13  000 for 3 years at 8% p.a. with interest paid annually. Find the
amount of interest earned by calculating the simple interest for each year separately.
5 WE8 Simon has $2000 to invest. He invests the money at 6% p.a. for 6 years with
interest compounded annually.
a Use the formula A = P(1 + R) n to calculate the amount to which this investment
will grow.
b Calculate the compound interest earned on the investment.
6 WE9 Calculate the future value of an investment of $14  000 at 7% p.a. for 3 years
with interest compounded quarterly.
7 A passbook savings account pays interest of 0.3% p.a. Jill has $600 in such
an account. Calculate the amount in Jill’s account after 3 years, if interest is
compounded quarterly.
8 Damien is to invest $35  000 at 7.2% p.a. for 6 years with interest compounded six‐
monthly. Calculate the compound interest earned on the investment.
9 Sam invests $40  000 in a one‐year fixed deposit at an interest rate of 7% p.a. with
interest compounding monthly.
a Convert the interest rate of 7% p.a. to a rate per month.
b Calculate the value of the investment upon maturity.

Chapter 16 • Financial mathematics 593


number and algebra

10 MC A sum of $7000 is invested for 3 years at the rate of 5.75% p.a., compounded
quarterly. The interest paid on this investment, to the nearest dollar, is:
A $1208 B $1308 C $8208 D $8308 E $8508

11 MC After selling their house and paying off their mortgage, Mr and Mrs Feng
have $73  600. They plan to invest it at 7% p.a. with interest compounded annually.
The value of their investment will first exceed $110  000 after:

A 5 years B 6 years C 8 years D 10 years E 15 years


12 MC Maureen wishes to invest $15  000 for a period of 7 years. The following
investment alternatives are suggested to her. The best investment would be:
A simple interest at 8% p.a.
B compound interest at 6.7% p.a. with interest compounded annually
C compound interest at 6.6% p.a. with interest compounded six‐monthly
D compound interest at 6.5% p.a. with interest compounded quarterly
E compound interest at 6.4% p.a. with interest compounded monthly
13 MC An amount is to be invested for 5 years and compounded semi‐annually
at 7% p.a. Which of the following investments will have a future value closest
to $10  000?
A $700 B $6500 C $7400 D $9000 E $9900
14 Jake invests $120  000 at 9% p.a. for a 1‐year term. For such large investments
interest is compounded daily.
a Calculate the daily percentage interest rate, correct to 4 decimal places.
Use 1 year = 365 days.
b Calculate the compounded value of Jake’s investment on maturity.
c Calculate the amount of interest paid on this investment.
d Calculate the extra amount of interest earned compared with the case where the
interest is calculated only at the end of the year.

594 Maths Quest 10 New South Wales Australian curriculum edition Stages 5.1 and 5.2
number and algebra

REASONING
15 Daniel has $15  500 to invest. An investment over a 2‐year term will pay interest of
7% p.a.
a Calculate the compounded value of Daniel’s investment if the compounding
period is:
i 1 year ii 6 months iii 3 months iv monthly.
b Explain why it is advantageous to have interest compounded on a more frequent
basis.
16 Jasmine invests $6000 for 4 years at 8% p.a. simple interest. David also invests
$6000 for 4 years, but his interest rate is 7.6% p.a. with interest compounded
quarterly.
a Calculate the value of Jasmine’s investment on maturity.
b Show that the compounded value of David’s investment is greater than Jasmine’s
investment.
c Explain why David’s investment is worth more than Jasmine’s investment
despite receiving a lower rate of interest.
17 Quan has $20  000 to invest over the next 3 years. He has the choice of investing his
money at 6.25% p.a. simple interest or 6% p.a. compound interest.
a Calculate the amount of interest that Quan will earn if he selects the simple
interest option.
b Calculate the amount of interest that Quan will earn if the interest is compounded:
i annually ii six monthly iii quarterly.
c Clearly Quan’s decision will depend on the compounding period. Under what
conditions should Quan accept the lower interest rate on the compound interest
investment?
d Consider an investment of $10  000 at 8% p.a. simple interest over 5 years. Use a
trial‐and‐error method to find an equivalent rate of compound interest over the
same period. Digital doc

e Will this equivalent rate be the same if we change: REFLECTION WorkSHEET 16.2
doc‐5352
How is compound interest
i the amount of the investment? calculated differently to
ii the period of the investment? simple interest?

Chapter 16 • Financial mathematics 595


number and algebra

16E Depreciation
•• Depreciation is the reduction in the value of an item as it ages over a period of time.
For example, a car that is purchased new for $45  000 will be worth less than that
eLesson amount 1 year later and less again each year.
What is depreciation?
eles‐0182 •• Depreciation is usually calculated as a percentage of the yearly value of the item.
•• To calculate the depreciated value of an item use the formula
A = P(1 − R) n
where A is the depreciated value of the item, P is the initial value of the item, R is
the percentage that the item depreciates each year expressed as a decimal and n is the
number of years that the item has been depreciating for.
•• This formula is almost the same as the compound interest formula except that it
subtracts a percentage of the value each year instead of adding.

WORKED EXAMPLE 10
A farmer purchases a tractor for $115  000. The value of the tractor
depreciates by 12% p.a. Find the value of the tractor after 5 years.
THINK WRITE
1 Write the depreciation formula. A = P(1 − R) n
2 Write the values of P, R and n. P = $115 000, R = 0.12, n = 5
3 Substitute the values into the formula. A = $115 000 × (0.88) 5
4 Calculate. = $60 689.17
The value of the tractor after 5 years
is $60 689.17.

•• In many cases, depreciation can be a tax deduction.


•• When the value of an item falls below a certain value it is said to be written off. That
Interactivity is to say that, for tax purposes, the item is considered to be worthless.
Different rates of
depreciation •• Trial‐and‐error methods can be used to calculate the length of time that the item will
int‐1155 take to reduce to this value.

WORKED EXAMPLE 11
A truck driver buys a new prime mover for $500  000. The prime mover
depreciates at the rate of 15% p.a. and is written off when its value falls
below $100  000. How long will it take for the prime mover to be written off?
THINK WRITE
1 Make an estimate of, say, n = 5. Use Consider n = 5.
the depreciation formula to find the A = P(1 − R) n
value of the prime mover after 5 years. = $500 000 × (0.85) 5
= $221 852.66
2 Because the value will still be greater Consider n = 10.
than $100  000, try a larger estimate, A = P(1 − R) n
say, n = 10. = $500 000 × (0.85) 10
= $98 437.20

596 Maths Quest 10 New South Wales Australian curriculum edition Stages 5.1 and 5.2
number and algebra

3 As the value is below $100  000, check Consider n = 9.


n = 9. A = P(1 − R) n
= $500 000 × (0.85) 9
= $115 808.47
4 Because n = 10 is the first time that the The prime mover will be written off in
value falls below $100  000, conclude 10 years.
that it takes 10 years to be written off.

Exercise 16E Depreciation


INDIVIDUAL PATHWAYS
Questions: Questions: Questions:
1, 2, 7, 9, 12, 14 1–3, 5, 7, 9, 11, 12, 14 1–8, 11, 10–15
Activity 16‐E‐1 Activity 16‐E‐2 Activity 16‐E‐3
Depreciation Harder depreciation Tricky depreciation
doc‐8500 doc‐8501 doc‐8502

FLUENCY
1 Calculate the depreciated value of an item for the initial value, depreciation rate and
time, in years, given below.
a Initial value of $30  000 depreciating at 16% p.a. over 4 years
b Initial value of $5000 depreciating at 10.5% p.a. over 3 years
c Initial value of $12  500 depreciating at 12% p.a. over 5 years

UNDERSTANDING
2 WE10 A laundromat installs washing machines and clothes dryers to the value of
$54  000. If the value of the equipment depreciates at a rate of 20% p.a., find the
value of the equipment after 5 years.
3 A drycleaner purchases a new machine for $38  400. The machine depreciates at
16% p.a.
a Calculate the value of the machine after 4 years.
b Find the amount by which the machine has depreciated over this period of time.
4 A tradesman values his new tools at $10  200. For tax purposes, their value
depreciates at a rate of 15% p.a.
a Calculate the value of the tools after 6 years.
b Find the amount by which the value of the tools has depreciated over these
6 years.
c Calculate the percentage of the initial value that the tools are worth after 6 years.
5 A taxi is purchased for $52  500 with its value depreciating at 18% p.a.
a Find the value of the taxi after 10 years.
b Calculate the accumulated depreciation over this period.

Chapter 16 • Financial mathematics 597


number and algebra

6 A printer depreciates the value of its printing presses by 25% p.a. Printing presses
are purchased new for $2.4 million. What is the value of the printing presses after:

a 1 year b 5 years c 10 years?


7 MC A new computer workstation costs $5490. With 26% p.a. reducing‐value
depreciation, the workstation’s value at the end of the third year will be close to:
A $1684 B $2225 C $2811 D $3082 E $3213

8 MC The value of a new photocopier is $8894. Its value depreciates by 26% in


the first year, 21% in the second year and 16% reducing balance in the remaining
7 years. The value of the photocopier after this time, to the nearest dollar, is:
A $1534 B $1851 C $2624 D $3000 E $3504
9 MC A company was purchased 8 years ago for $2.6 million. With a depreciation rate
of 12% p.a., the total amount by which the company has depreciated is closest to:
A $0.6 million B $1.0 million C $1.7 million
D $2.0 million E $2.3 million
10 MC Equipment is purchased by a company and is depreciated at the rate of
14% p.a. The number of years that it will take for the equipment to reduce to half
of its initial value is:
A 4 years B 5 years C 6 years D 7 years E 8 years
11 MC An asset, bought for $12  300, has a value of $6920 after 5 years. The
depreciation rate is close to:
A 10.87% B 16.76% C 18.67% D 21.33% E 27.34%
12 WE11 A farmer buys a light aeroplane for crop‐dusting. The aeroplane costs $900  000.
The aeroplane depreciates at the rate of 18% p.a. and is written off when its value falls
below $150  000. How long will it take for the aeroplane to be written off?

598 Maths Quest 10 New South Wales Australian curriculum edition Stages 5.1 and 5.2
number and algebra

13 A commercial airline buys a jumbo


jet for $750 million. The value of this
aircraft depreciates at a rate of 12.5% p.a.
a Find the value of the plane after
5 years, correct to the nearest
million dollars.
b How many years will it take for
the value of the jumbo jet to fall
below $100 million?
REASONING
14 A machine purchased for $48  000 will
have a value of $3000 in 9 years.
a Use a trial‐and‐error method to find the
rate at which the machine is depreciating per annum.
n
b Consider the equation x = an, a = "x. Verify your
answer to part a using this relationship.
15 Camera equipment purchased for $150  000 will have a REFLECTION
value of $9000 in 5 years. How and why is
the formula for
Find the rate of annual depreciation using trial and error depreciation different
first and then algebraically with the relationship, ‘if x = an to compound interest?
n
then a = "x’. Compare and contrast each method.

16F Loan repayments


•• The simple interest formula is used to calculate the interest on a flat rate loan.

WORKED EXAMPLE 12
Calculate the interest payable on a loan of $5000 to be repaid at 12% p.a.
flat interest over 4 years.
THINK WRITE
1 Write the simple interest formula. P×r×T
I=
100
2 List the known values. P = $5000, r = 12%, T = 4
3 Substitute the values into the formula. 5000 × 12 × 4
I=
100
4 Calculate the interest. = $2400
The interest payable is $2400.

•• The total amount that would have to be repaid under the loan in Worked example 12
is $7400, and this could be made in 4 equal payments of $1850. With a flat‐rate loan,
the interest is calculated on the initial amount borrowed regardless of the amount of
any repayments made.
•• In contrast, taking a reducible‐interest‐rate loan means that each annual amount of
interest is based on the amount owing at the time.

Chapter 16 • Financial mathematics 599


number and algebra

•• Consider the same loan of $5000, this time at 12% p.a. reducible interest and an
agreed annual repayment of $1850. At the end of each year, the outstanding balance
is found by adding the amount of interest payable and then subtracting the amount of
each repayment.
Interest for year 1 = 12% of $5000
= 0.12 × $5000
= $600
Balance for year 2 = $5000 + $600 − $1850
= $3750
Interest for year 2 = 12% × $3750
= 0.12 × $3750
= $450
Balance for year 3 = $3750 + $450 − $1850
= $2350
Interest for year 3 = 12% of $2350
= 0.12 × $2350
= $282
Balance for year 4 = $2350 + $282 − $1850
= $782
Interest for year 4 = 12% of $782
= 0.12 × $782
= $93.84
In the fourth year, a payment of only $875.84 is required to fully repay the loan. The
total amount of interest charged on this loan is $1425.84, which is $974.16 less than
the same loan calculated using flat‐rate interest.

WORKED EXAMPLE 13
Calculate the amount of interest paid on a loan of $10  000 that is charged
at 9% p.a. reducible interest over 3 years. The loan is repaid in two annual
instalments of $4200 and the balance at the end of the third year.

THINK WRITE
1 Calculate the interest for the Interest for year 1 = 9% of $10 000
first year. = 0.09 × $10 000
= $900
2 Calculate the balance at the Balance for year 2 = $10 000 + $900 − $4200
start of the second year. = $6700
3 Calculate the interest for the Interest for year 2 = 9% of $6700
second year. = 0.09 × $6700
= $603
4 Calculate the balance at the Balance for year 3 = $6700 + $603 − $4200
start of the third year. = $3103

600 Maths Quest 10 New South Wales Australian curriculum edition Stages 5.1 and 5.2
number and algebra

5 Calculate the interest for the Interest for year 3 = 9% of $3103


third year. = 0.09 × $3103
= $279.27
6 Calculate the amount of the Balance remaining at end of year
final repayment and ensure = $3103 + $279.27
that the loan is fully repaid. = $3382.27
7 Find the total amount of Interest charged = $900 + $603 + $279.27
interest paid by adding each = $1782.27
year’s amount.

Exercise 16F Loan repayments


INDIVIDUAL PATHWAYS
Questions: Questions: Questions:
1, 2, 4, 7, 10 1–4, 6, 8, 9, 11 1, 2c–e, 3–5, 8–11
Activity 16‐F‐1 Activity 16‐F‐2 Activity 16‐F‐3
Repaying a loan Harder loan repayments Difficult loan repayments
doc‐8503 doc‐8504 doc‐8505

FLUENCY
1 Calculate the interest payable on a loan of $10  000 to be repaid at 15% p.a.
WE12
flat‐rate interest over 3 years.
2 Calculate the interest payable on each of the following loans.
a $20  000 at 8% p.a. flat‐rate interest over 5 years
b $15  000 at 11% p.a. flat‐rate interest over 3 years
c $7500 at 12.5% p.a. flat‐rate interest over 2 years
d $6000 at 9.6% p.a. flat‐rate interest over 18 months
e $4000 at 21% p.a. flat‐rate interest over 6 months

UNDERSTANDING
3 Larry borrows $12  000 to be repaid at 12% p.a. flat rate of interest over 4 years.
a Calculate the interest that Larry must pay.
b What is the total amount that Larry must repay?
c If Larry repays the loan in equal annual instalments, calculate the amount of each
repayment.
4 WE13 Calculate the amount of interest paid on a loan of $12  000 that is charged at
10% p.a. reducible interest over 3 years. The loan is repaid in two annual instalments
of $5000 and the balance at the end of the third year.
5 Calculate the total amount that is to be repaid on a loan of $7500 at 12% p.a.
reducible interest over 3 years with two annual repayments of $3400 and the balance
repaid at the end of the third year.

Chapter 16 • Financial mathematics 601


number and algebra

6 Brian needs to borrow $20  000. He finds a loan that charges 15% p.a. flat‐rate
interest over 4 years.
a Calculate the amount of interest that Brian must pay on this loan.
b Calculate the total amount that Brian must repay on this loan.
c Brian repays the loan in 4 equal annual instalments. Calculate the amount of
each instalment.
d Brian can borrow the $20  000 at 15% p.a. reducible interest instead of flat‐rate
interest. If Brian makes the same annual repayment at the end of the first three
years and the balance in the fourth, calculate the amount of money that Brian
will save.
7 Farrah borrows $12  000 at 10% p.a. reducible interest over 3 years. Farrah repays
the loan in two equal annual payments of $4900 and the balance at the end of the
third year.
a Calculate the amount of interest that Farrah must pay on this loan.
b Farrah finds that she can afford to repay $5200 each year. How much does
Farrah save by making this higher repayment?
8 Aamir borrows $25  000 at 12% p.a. reducible interest over 3 years with two annual
repayments of $11  000 and the balance repaid at the end of the third year.
a Find the total amount of interest that Aamir pays on this loan.
b What is the average amount of interest charged on this loan per year?
c By writing your answer to part b as a percentage of the initial amount borrowed,
find the equivalent flat rate of interest on the loan.
9 Felicity borrows $8000 at 8% p.a. reducible interest over 3 years, repaying the loan
in two annual payments of $3200 and the balance repaid at the end of the third
year.
a Using the method described in question 8, find the equivalent flat rate of interest.
b Find the equivalent flat rate of interest charged if Felicity increases the amount
of each annual repayment to $4000.
REASONING
10 Natalie has the choice of two loans of $15  000. Each loan is to be taken over a
three‐year term with annual repayments of $6350. Loan A is charged at 9% flat‐rate
interest; Loan B is charged at 10% reducible interest. As Natalie’s financial planner,
construct a detailed report to advise Natalie which loan would be better for her
to take.
11 Barack borrows $13  500 at 10% p.a. reducible
interest over 2 years, making an annual repayment
Digital doc
of $7800 and the balance repaid at the end of the REFLECTION
WorkSHEET 16.3 How does a loan at reducible
doc‐5353 second year. Show that if interest is added every
interest compare with the
six months, at which time a repayment of $3900 same loan at flat‐rate interest?
is made, a saving of approximately $350 is made.

602 Maths Quest 10 New South Wales Australian curriculum edition Stages 5.1 and 5.2
number and algebra

CHAPTER REVIEW
LANGUAGE

compound interest future value principal


compounding period interest repayments
deposit interest rate simple interest
depreciation lay‐by successive discounts
discount loan terms

int‐2868 doc‐13790
int‐2869 doc‐13791
int‐3602

FLUENCY
1 Calculate the simple interest that is earned on $5000 at 5% p.a. for 4 years.
2 Jim invests a sum of money at 9% p.a. Which one of the following statements
is true?
A Simple interest will earn Jim more money than if compound interest is paid
annually.
B Jim will earn more money if interest is compounded annually rather than
monthly.
C Jim will earn more money if interest is compounded quarterly rather than
six‐monthly.
D Jim will earn more money if interest is compounded annually rather than
six‐monthly.
E It does not matter whether simple interest or compound interest is used to
calculate the growth of Jim’s investment.
3 Find the single discount that is equivalent to successive discounts of 12.5% and 5%.
4 Which one of the following statements is correct?
A Successive discounts of 10% and 15% are less than a single discount of 25%.
B Successive discounts of 10% and 15% are equal to a single discount of 25%.
C Successive discounts of 10% and 15% are greater than a single discount of 25%.
D Successive discounts of 10% and 15% are equal to successive discounts of 12%
and 13%.
E Successive discounts of 10% and 15% are equal to successive discounts of 13%
and 12%.
5 Benito has a credit card with an outstanding balance of $3600. The interest rate
charged on the loan is 18% p.a. Calculate the amount of interest that Benito will be
charged on the credit card for the next month.

Chapter 16 • Financial mathematics 603


number and algebra

6 An LCD television has a cash price of $5750. It can be purchased on terms of 20%
deposit plus weekly repayments of $42.75 for 3 years. Calculate the total cost of
the television if it is purchased on terms.
7 Erin purchases a new entertainment unit that has a cash price of $6400. Erin buys the
unit on the following terms: 10% deposit with the balance plus interest to be repaid in
equal monthly repayments over 4 years. The simple interest rate charged is 12% p.a.
a Calculate the amount of the deposit.
b Calculate the balance owing after the deposit has been paid.
c Calculate the interest that will be charged.
d What is the total amount that Erin has to repay?
e Calculate the amount of each monthly repayment.
8 A new car has a marked price of $40  000. The car can be purchased on terms of
10% deposit and monthly repayments of $1050 for 5 years.
a Find the total cost of the car if it is purchased on terms.
b Calculate the amount of interest paid.
c Calculate the amount of interest paid per year.
d Calculate the interest rate charged.
9 The single discount that is equivalent to successive discounts of 15% and 20% is:
A 10% B 18% C 28% D 30% E 32%
10 A car dealership offers a 10% discount on the price of service of a car purchased at
the dealership.
a Calculate the price paid for a service valued at $290.00 by a person who
purchased their car at the dealership.
b During November, the dealership offers an extra 15% discount on all services
and mechanical repairs. Calculate the price Callum, who purchased his car at the
dealership, pays for a service in November.
c What is the total discount given on this service?
d Determine the single percentage discount that would be equivalent to the
successive discounts of 10% and 15% that Callum receives.
11 Ryan invests $12  500 for 3 years at 8% p.a. with interest paid annually. By
calculating the amount of simple interest earned each year separately, determine the
amount to which the investment will grow.
12 Calculate the compound interest earned on $45  000 at 12% p.a. over 4 years if
interest is compounded:
a annually b six‐monthly
c quarterly d monthly.
13 A new computer server costs $7290. With 22% p.a. reducing‐value depreciation,
the server’s value at the end of the third year will be close to:
A $1486 B $2257 C $2721 D $3023 E $3460
14 An asset, bought for $34  100, has a value of $13  430 after 5 years. The depreciation
rate is close to:
A 11% B 17% C 18% D 21% E 22%
15 The value of a new car depreciates by 15% p.a. Find the value of the car after
5 years if it was purchased for $55  000.

604 Maths Quest 10 New South Wales Australian curriculum edition Stages 5.1 and 5.2
number and algebra

PROBLEM SOLVING
1 The value of a new tractor is $175  000. The value of the tractor depreciates by
22.5% p.a.
a Find the value of the tractor after 8 years.
b What percentage of its initial value is the tractor worth after 8 years?

2 Gavin borrows $18  000 over 5 years from the bank. The loan is charged at 8.4% p.a.
flat‐rate interest. The loan is to be repaid in equal monthly instalments. Calculate the
amount of each monthly repayment.
3 A building society advertises investment accounts at the following rates:
a 3.875% p.a. compounding daily
b 3.895% p.a. compounding monthly
c 3.9% p.a. compounding quarterly.
Peter thinks the first account is the best one because the interest is calculated more
frequently. Paul thinks the last account is the best one because it has the highest
interest rate. Explain whether either is correct.

Chapter 16 • Financial mathematics 605


number and algebra
Communicating

Rich task

Consumer Price Index

Weight (% of total)
CPI group
19
Food
7
Clothing
8.2
Tobacco/alcohol
14.1
Housing
5.6
Health/personal care
18.3
Household equipment
17
Transportation
10.8
Recreation/education

606 Maths Quest 10 New South Wales Australian curriculum edition Stages 5.1 and 5.2
number and algebra

Consider a simplified example showing how this CPI is calculated and how we are able to
compare prices between one period and another. Take three items with prices as follows:
a pair of jeans costing $75, a hamburger costing $3.90 and a CD costing $25. Let us say
that during the next period of time, the jeans sell for $76, the hamburger for $4.20 and the
CD for $29. This can be summarised in the following table.

1 Complete the table to determine the total weighted price for Period 2.
In order to calculate the CPI for Period 2, we regard the first period as the base and
allocate it an index number of 100 (it is classed as 100%). We compare the second period
with the first by expressing it as a percentage of the first period.
weighted expenditure for Period 2
CPI = × 100%
weighted expenditure for Period 1
2 a Calculate the CPI for the above example, correct to 1 decimal place.
b This figure is over 100%. The amount over 100% is known as the inflation factor. What is the
inflation factor in this case?
3 Now apply this procedure to a more varied basket of goods. Complete the following table then
calculate the CPI and inflation factor for the second period.

Chapter 16 • Financial mathematics 607


number and algebra

Code puzzle

The longest and shortest


gestation periods in mammals
Calculate the interest paid on the following purchase plans.
The amount, along with the letter, will help to solve the puzzle.

Longest (660 days)

$228 $520 $151 $5400 $432 $228 $780 $160 $1272 $160 $0 $2790 $228 $780 $446

A television with a A computer has a An entertainment system A used car has a


marked price of $2100 marked price of $1500. has a marked price of purchase price of $9000.
is purchased on a plan A purchase plan requires $3600 for cash purchase A plan involves payment
requiring $48.50 per $37.60 per fortnight or 36 equal monthly of $250 per month
month for 4 years. for 2 years. instalments of $112. for 5 years.

A B C D
A telephone system has A television is priced at A computer system can An office can purchase
a cost price of $800. $1400 for cash sale. A be purchased for $2900. furniture for a total
Arrangements for a purchase plan involves A purchase plan involves of $10 000 or $600 per
purchase plan involve a deposit of $300 plus $45 10% deposit and $150 month for two years
24 equal instalments a month for 3 years. per month for 3 years. (including a 10%
deposit).
E F H I
of $40.

A sports club purchased An entertainment system A used car is purchased Glassware is purchased
some equipment valued with a marked price of on a plan involving 48 on a plan involving 24
at $5000 on the following $3900 is purchased for monthly payments of monthly payments of $45.
terms. $500 deposit and weekly payments of $45 $225. The advertised price The glassware is marked
$74 per fortnight for for 2 years. for the car was $8000. at $1080.
3 years.
L N O P
An iPod is advertised for A lounge suite valued at An industrial microwave has
$499 or 52 weekly $3800 can be purchased a value of $2450. A hotel
payments of $12.50. on a plan involving 10% purchases the microwave on
deposit and 48 monthly a plan involving $400 deposit
payments of $80. plus $24 per week for 2 years.
R S T
Shortest (12 days)


$420 $2790 $2800 $151 $446 $780 $2800 $420 $160 $6000 $455.20 $228 $780 $6000 $5400 $432 $2800 $2800 $446

608 Maths Quest 10 New South Wales Australian curriculum edition Stages 5.1 and 5.2
number and algebra

ACTIVITIES
Go to assessON
Chapter opener Digital docs
for questions to
Video • Activity 16‐D‐1 (doc‐8497): Compound
interest puzzle 1 (page 593) test your readiness
• The story of mathematics (eles-1855)
• Activity 16‐D‐2 (doc‐8498): Compound FOR learning, your
16A Purchasing goods interest puzzle 2 (page 593) progress AS you learn
Digital docs and your levels OF
• Activity 16‐D‐3 (doc‐8499): Compound
• Activity 16‐A‐1 (doc‐8488): Simple interest puzzle 3 (page 593) achievement.
interest (page 577) • WorkSHEET 16.2 (doc‐5352): Compound www.assesson.com.au
• Activity 16‐A‐2 (doc‐8489): Harder simple interest (page 595)
interest (page 577)
• Activity 16‐A‐3 (doc‐8490): Tricky simple 16E Depreciation
interest (page 577) eLesson
• SkillSHEET (doc‐5345): Converting a • What is depreciation? (eles‐0182)
percentage to a decimal (page 578) (page 596)
• SkillSHEET (doc‐5346): Finding simple Interactivity
interest (page 578) • Different rates of depreciation
(int‐1155) (page 596)
16B Buying on terms Digital docs (page 597)
Digital docs
• Activity 16‐E‐1 (doc‐8500): Depreciation
• Activity 16‐B‐1 (doc‐8491): Buying on • Activity 16‐E‐2 (doc‐8501): Harder
terms (page 581) depreciation
• Activity 16‐B‐2 (doc‐8492): Buying on difficult • Activity 16‐E‐3 (doc‐8502): Tricky
terms (page 581) depreciation
• Activity 16‐B‐3 (doc‐8493): Buying on tricky
terms (page 581) 16F Loan repayments
• SkillSHEET (doc‐5347): Finding a percentage Digital docs
of a quantity (money) (page 581) • Activity 16‐F‐1 (doc‐8503): Repaying a
• WorkSHEET 16.1 (doc‐5350): Buying on loan (page 601)
terms (page 584) • Activity 16‐F‐2 (doc‐8504): Harder loan
repayments (page 601)
16C Successive discounts • Activity 16‐F‐3 (doc‐8505): Difficult loan
Digital docs
repayments (page 601)
• Activity 16‐C‐1 (doc‐8494): Successive • WorkSHEET 16.3 (doc‐5353): Loan
discounts (page 586) repayments (page 602)
• Activity 16‐C‐2 (doc‐8495): Difficult
successive discounts (page 586) Chapter review
• Activity 16‐C‐3 (doc‐8496): Tricky successive Interactivities (page 603)
discounts (page 586) • Word search (int‐2868)
• SkillSHEET (doc‐5348): Finding percentage • Crossword (int‐2869)
discount (page 586) • Sudoku (int‐3602)
• SkillSHEET (doc‐5349): Decreasing a quantity Digital docs
by a percentage (page 586) • Chapter summary (doc‐13790)
• SkillSHEET (doc‐5351): Expressing one • Concept map (doc‐13791)
quantity as a percentage of another To access eBookPLUS activities, log on to
(page 586) www.jacplus.com.au
16D Compound interest
Interactivity
• Compound interest (int‐2791) (page 588)

Chapter 16 • Financial mathematics 609


number and algebra

Answers
CHAPTER 16 Financial maths
Exercise 16A — Purchasing goods Exercise 16B — Buying on terms
1 $2400    1    a i $3960
2    a $1800 b $900 c $1920 ii $3720
d $720 iii $3950
3    a $1500 b $125 c $281.25 b The best deal is the one with the lowest cost — 20% deposit
d $1080 and weekly payments of $20 over 3 years.
4    a $18.75 b $55.00 c $41.25 2    a $131.25 b $55.38 c $144.44
d $125.33 e $99.58 3    a $13  600, $283.33 b $40  000, $666.67
5 $10 c $5006.25, $278.13 d $80  000, $666.67
6    a $700 b $10.50 c $710.50 e $488  000, $2033.33
d $3.16 e $913.66 4 $5409.76 5 $4530.08
7    a $52.50 b $50.79 c $35.92 6    a $1260 b $19  504.80 c $20  764.80
d $14.87 d $8164.80
7    a $8409.96 b $2609.96 c $869.99
8    a
Balance Closing d 15%
Month owing Interest Payment balance 8    a $2375 b $7125 c $2565
d $9690 e $269.17
January $7500.00 $112.50 $1000.00 $6612.50 9    a $2000 b $4000 c $540
d $4540 e $252.22
February $6612.50 $99.19 $1000.00 $5711.69 10    a $226.80 b $141.90 c $360.94
March $5711.69 $85.68 $1000.00 $4797.37 d $87.11 e $85.13
11    a $4600 b $1656 c $130.33
April $4797.37 $71.96 $1000.00 $3869.33 d $7256
12 C
May $3869.33 $58.04 $1000.00 $2927.37 13 E. This option has the lowest interest rate and time frame when
compared to all others.
June $2927.37 $43.91 $1000.00 $1971.28 14 The larger the deposit the smaller the loan and hence the interest
charged. Loans generally offer a lower rate than buying on terms.
July $1971.28 $29.57 $1000.00 $1000.85
Challenge 16.1
August $1000.85 $15.01 $1015.86 $0 20 days
Exercise 16C — Successive discounts
b $515.86 1    a 16.7% b 10.2% c 43.7%
c $8015.86 d 15.4% e 7.6%
9 2 23.5%
3 C
Payment Immediate Immediate Possible Possible price 4    a 32% b 19.04% c 16.75%
option payment possession extra cost negotiation d 57.5%
5 19%
Cash ✓ ✓ ✓ 6    a $3040 b 24%
7    a $90 b $85.50 c $14.50
Lay‐by ✓ d 14.5%
8    a $1840 b $1748 c $252
Credit ✓ ✓ d 12.6%
card 9    a $212.50 b $201.88 c $48.12
d $200, no e 19.25% f Yes
Payment Extra g g 28%
option Payment Possession cost Price 10    a $18  168.75 b 27.325%
Cash immediate immediate nil negotiable 11 Yes. Both lead to a single discount of 14.69%
12 Single discount = 1 − (1 − a) × (1 − b), where a and b are
Lay‐by intervals delayed limited – successive discounts (as decimals).
Exercise 16D — Compound interest
Credit delayed immediate possible –
1    a $3244.80 b $10  939.56 c $24  617.98
card
d $14  678.02 e $14  838.45 f $129  394.77
2    a $932.52 b $10  650.81 c $202  760.57
10 S1: Credit card — payment is delayed, but possession is d $25  816.04 e $3  145  511.41
immediate 3 $8784.60
S2: Lay‐by, or cash if she has savings, would like to negotiate a 4 $3376.26
lower price and has somewhere to store it. 5    a $2837.04 b $837.04
6 $17  240.15 7 $605.42 8 $18  503.86

610 Maths Quest 10 New South Wales Australian curriculum edition Stages 5.1 and 5.2
number and algebra

9    a 0.5833% b $42  891.60 Chapter review


10 B 11 B 12 C 13 C Fluency
14    a 0.0247% b $131  319.80 1 $1000 2 C 3 16.875% 4 A
c $11  319.80 d $519.80 5 $54 6 $7819
 15    a   i $17  745.95 ii $17  786.61 7    a $640 b $5760 c $2764.80 d $8524.80
iii $17  807.67 iv $17  821.99 e $177.60
b The interest added to the principal also earns interest. 8    a $67  000 b $27  000 c $5400
16    a $7920 d 15% p.a.
b David’s investment = $8108.46 9 E
c Because David’s interest is compounded, the interest is added 10    a $261 b $221.85 c $68.15 d $23.5%
to the principal each quarter and earns interest itself. 11 $15  746.40
17    a $3750 interest 12    a $25  808.37 b $26  723.16 c $27  211.79
     b     i $3820.32 interest d $27  550.17
ii $3881.05 13 E
iii $3912.36 14 B
c Compound quarterly gives the best return. 15 $24  403.80
d If we assume that interest is compounded annually, an Problem solving
equivalent return of R = 7% would be achieved. 1    a $22  774.65 b 13%
     e i Yes ii No 2 $426
Challenge 16.2 3    a 3.95% p.a. flat rate
4 years and 8 months b 3.97% p.a. flat rate
c 3.96% p.a. flat rate
Exercise 16E — Depreciation
Neither is correct. The best option is to choose 3.895% p.a.
1    a $14  936.14 b $3584.59 c $6596.65
compounding monthly.
2 $17  694.72
3    a $19  118.26 b $19  281.74 Communicating — Rich task
4    a $3846.93 b $6353.07 c 38% 1
5    a $7216.02 b $45  283.98 Period 1 Period 2
6    a $1.8 million b $569  531.25 c $135  152.44 Weight Price Price
7 B Item (W) (P) W×P (P) W×P
8 A Jeans 7 $75 525 $76 532
9 C
10 B Hamburger 19 $3.90 74.1 $4.20 79.8
11 A CD 10.8 $25 270 $29 313.2
12 10 years
Total 36.8 $103.90 869.1 $109.20 925
13    a $385 million b 16 years
14    a 27% 2    a 106.4%
b A = P(1 − R) n b 6.4%
A 3 Period 1 Period 2
= (1 − R) n
P Weight Price Price
n A
= (1 − R) Item (W) (P) W×P (P) W×P
ÅP
n A
Bus fare 17 $4.80 81.6 $4.95 84.15
R=1−
ÅP Rent 14.1 $220 3102 $240 3384
15    a Approx 43% Movie ticket 10.8 $10.50 113.4 $10.80 116.64
b Trial and error: can be time consuming, answer is often Air 18.3 $1200 21  960 $1240 22  692
an estimate; algebraic solution: correct answer calculated conditioner
immediately from equation
Haircut 5.6 $18.50 103.6 $21.40 119.84
Exercise 16F — Loan repayments
Bread 19 $2.95 56.05 $3.20 60.8
1 $4500
2    a $8000 b $4950 c $1875 Shirt 7 $32.40 226.8 $35 245
d $864 e $420 Bottle of 8.2 $19.95 163.59 $21 172.2
3    a $5760 b $17  760 c $4440 Scotch
4 $2422
Total 100 $1509.10 25  807.04 $1576.35 26  874.63
5 $9264
6    a $12  000 b $32  000 104%; 4%
c $8000 d $4966.87 Code puzzle
7    a $2453 b $93 African elephant
8    a $6004.80 b $2001.60 c 8% Short‐nosed bandicoot
9    a 5.4% b 4.6%
10 Loan B better (total savings $1053.50)
11 Actual savings $355.15

Chapter 16 • Financial mathematics 611


Problem solving

Chapter 17

Problem solving II
WHY LEARN THIS?
Problem solving is something performed by many
people every day. Very good problem solvers think in
a very organised and systematic way, and they reason
logically and effectively. They can justify their decisions
because they can provide evidence for how they arrived
at their conclusions. Solving mathematical problems
will enable you to think laterally and provide you with
the opportunity to practise your skills and stretch your
imagination.

612 Maths Quest 10 New South Wales Australian curriculum edition Stages 5.1 and 5.2
Watch this video
The story of mathematics

Searchlight ID: eles-1696

Chapter 17 • Problem solving II 613


problem solving

1 2
1 −
1 What is the value of c A4 B 2 d ?

2 Rachel is flying from Sydney to Perth. She left Sydney at 11:00 pm on January 6 and
arrived in Perth five hours later.
a What is the time in Sydney when Rachel’s plane landed in Perth?
b What is the time in Perth when Rachel’s plane landed?
3 The numbers 1 through 10 were written on pieces of paper and placed into a hat.
Greg chose one piece of paper from the hat, and without replacing that number, then
chose a second piece of paper from the hat.
a Is Greg’s first choice dependent upon his second choice? Explain.
b Is Greg second choice dependent upon his first choice? Explain.
4 Asuka sells musical instruments at discount prices. She had a drum kit on sale for
15% off the retail price of $5000. After two months the drum kit did not sell, and
Asuka decided to apply an extra 10% discount to the existing sale price.
a What is the total amount saved by the customer?
b What is the final price of the drum kit?
c Explain how a 25% discount on the retail price would compare with the
successive discounts.

5 a Graph the equation y = 4−x using the following table.

x −2.5 −2.0 −1.5 −1.0 −0.5 0 0.5 1.0 1.5 2.0 2.5
y

b Describe what happens to y as x becomes larger.


c Describe what happens to y as x becomes smaller.

614 Maths Quest 10 New South Wales Australian curriculum edition Stages 5.1 and 5.2
problem solving

6 What is the probability of choosing a red card or the Ace of Clubs from a standard
pack of cards?
7 What is the cost of buying 2000 shares in the mining company DIGGER
at $10.47 each if there is a transaction fee of 0.1% OR $20, whichever is the
larger?
8 A TV cost $700 cash. I buy it on terms that are $200 deposit plus $30 per month
for 20 months.
a How much more do I actually pay?
b Express this amount as a percentage of the cash price.
9 What is the probability of rolling two even numbers in one throw of two unbiased,
6‐sided dice?
10 Annie’s and Barbara’s ages add to 25, and have a difference of 11. If Annie is the
older of the two, how old are both Annie and Barbara?
11 Palmo is in his third year as an apprentice cabinet maker. He earns $855 gross
salary per week.
a Calculate his income for the year, if he receives a 17.5% four week holiday
loading.
b Palmo purchased cutting tools for $5000. In the first three years they
depreciated at a constant rate to $3635. How much did the tools depreciate
each year?
12 The difference between two numbers is 3. If six times the larger number minus
twice the smaller number is 46, determine the two numbers.
13 The height of each student in a Year 10 class was measured and it was found
that the mean height was 160 cm. Two students were absent. When their
heights were included in the data for the class, the mean height did
not change.
Suggest a pair of heights that are possible for the two absent students. What
reasoning could be used to find a pair of possible heights?
14 The IT department for a school can buy mini‐laptops from an educational supplier
and receive a discount. If the IT manager orders 150 mini‐laptops (for all the
Year 10 students), the cost is $30  000 and if she orders 80 mini‐laptops the
cost is $16  000. If she only had $20  000 to spend how many mini‐laptops could
she buy?
15 The data below shows the number of times 25 Year 10 students have used a
computer in the last week.
10, 19, 7, 0, 1, 6, 22, 3, 9, 15, 3, 6, 13, 2, 16, 8, 5, 4, 11, 10, 16, 4, 8, 5, 13
a Group the data into a frequency table in class intervals of size 5.
b Represent the grouped data as a histogram.
c Add a frequency polygon to the histogram.
d Construct a cumulative frequency polygon.
e From the cumulative frequency polygon, estimate how many students used a
computer more than 10 times in the week.
f Add a percentage cumulative frequency axis to the graph.
g Estimate the 50th percentile and the 30th percentile. Interpret these results.

Chapter 17 • Problem solving II 615


problem solving

16 The following data show the ages of a Male Female


group of 30 males and 30 females as
they enter hospital for the first time. 9  8 0 5
a Construct a pair of parallel box 9  9 8  8 8  6 3  2 1 1 7  7 8  9 9
plots to represent the two sets of 8  7 7  6 4  3 2  0 2 0  0 1  2 4  5 5  6 7  9
data, showing working out for the 8  6 3  1 0 3 0  1 3  3 5  8
median and 1st and 3rd quartiles.
7  5 2 4 2  3 6  8
b Calculate the mean, range and IQR
for both sets of data. 5  3 5 1  3 4
c Determine any outliers if they 6 2
exist. 8 7
d Write a short paragraph comparing
the data.
17 Fiona invests $8000 for 4 years at 6% p.a. simple interest. Nathan also
invests $8000 for 4 years; however, his interest rate is 5.6% p.a. compounded
quarterly.
a Calculate the value of Fiona’s investment at maturity.
b Show that the compounded value of Nathan’s investment is greater than that of
Fiona’s.
c Explain why Nathan’s investment after 4 years is greater than Fiona’s even
though she is receiving a higher rate of interest.
18 Catherine invests $2000 in a term deposit account which pays interest at a rate of
4.5% per annum on the balance at the beginning of each year.
a After one year what is her investment worth?
b What percentage of her original investment is her bank balance at the end of the
year?
c What would Catherine’s investment be worth after two years?
d Plot the graph of this exponential function.
e Use the graph to predict when her investment will be worth $10  000.
19 In preparation for the Christmas
office party, Fred is put in charge of
providing coffee. He determines that
each cup of coffee requires 3.2 grams
of coffee ($19 per kg), 6.4 grams of
sugar ($0.98 per kg) and 10.5 grams of
milk ($1.40 per kg) plus a plastic cup
which costs $2.00 for 24 cups.
a Determine the cost of providing a
single cup of coffee.
b If Fred has $25 to spend, how many cups of coffee can he make, assuming that
he charges 10 cents a cup, and half the people pay?
20 Virgin Blue buys a new plane so that extra flights can be arranged between Sydney,
Australia and Wellington, New Zealand. The plane costs $1  200  000. It depreciates
at a rate of 16.5% p.a. and is written off when its value falls below $150  000. How
long can Virgin Blue use this plane before it is written off?

616 Maths Quest 10 New South Wales Australian curriculum edition Stages 5.1 and 5.2
problem solving

21 The following data show the number of


pets in each of the 12 houses in Coral
Avenue, Rosebud.
2, 3, 3, 2, 2, 3, 2, 4, 3, 1, 1, 0
a Calculate the mean and median
number of pets.
b The empty block of land at the end of
the street was bought by a Cattery and
now houses 20 cats. Recalculate the
mean and median.
c Explain why the answers are so different, and which measure of central tendency
is best used for certain data.
22 Erin borrows $12  000 for a new car at 9% p.a. over 4 years.
a Calculate the total amount to be repaid if the interest
is compounded monthly.
b How much will be paid in interest for this
loan?
c How much would each repayment be in order
to repay the loan in equal monthly instalments?
23 Based on her performance throughout the year, Mary
was given a probability of 0.7 of winning her first
tennis tournament. If the probability of winning
both her first and second tennis tournaments is 0.56,
what is her probability of winning the second tennis
tournament?
24 The number of Year 10 students in all the 40 schools in
the Northern District of the Education Department was
recorded as follows:
56, 134, 93, 67, 123, 107, 167, 124, 108, 78,
89, 99, 103, 107, 110, 45, 112, 127, 106, 111,
127, 145, 87, 75, 90, 123, 100, 87, 116, 128,
131, 106, 123, 87, 105, 112, 145, 115, 126, 92
a Using a interval of 10, produce a table showing the frequency for each
interval.
b Use the table to estimate the mean.
c Calculate the mean of the ungrouped data.
d Compare the results from parts b and c and explain any differences.
25 An electronics store is having trouble selling the latest mp3 player. The original
price was $99 but on October 1 it was reduced 10%. On October 8 it was reduced a
further 10%. On October 12 the regional manager decided to increase all prices by
5%. On October 15 the local manager decided to reduce the price by another 10%
anyway.
a Calculate the prices on all 4 dates after the discounts/increases have been
applied.
b What is the ‘final’ percentage discount after Oct 15?

Chapter 17 • Problem solving II 617


problem solving

26 Complete this table regarding compound interest.

Annual Compounding Length of Value of


Principal interest rate period investment investment
$1000 6.4% Monthly 11 months
$2000 7.82% Daily 77 days
$4000 8.08% Quarterly 8 years

27 A survey recorded the number of years of education of the parents of a class of


Year 10 students. The results were as follows
12, 15, 12, 11, 13, 17, 10, 12, 14, 16,
12, 13, 11, 9, 11, 12, 12, 16, 12, 14,
12, 13, 11, 11, 14, 12, 11, 10, 15, 12,
13, 12, 12, 11, 10, 10, 12, 12, 14, 13,
11, 9, 12, 15, 16, 13, 12, 14, 15, 12
a Produce a table showing frequency and cumulative frequency for each number
of years.
b Produce a cumulative frequency polygon.
c From the results of part a and part b, produce a box‐plot.
28 The annual rate of inflation is very similar to the annual interest rate on a
compound interest investment. Determine the annual rate of inflation in the
following cases. Give your answers as a percentage rounded to 2 decimal places.
a In 2005 the price of a family‐sized pizza was $11. The exact same pizza in 2010
is $17.
b In 2000 the price of a litre of petrol was $0.98. In 2009 the price was $1.45.
c In 1972 the price of a can of soft drink was $0.25 while in 2010 the price is $2.10.
29 a A Year 10 boy is talking with a Year 10 girl and asks her if she has any brothers or
sisters. She says, ‘Yes, I have one’. What is the probability that she has a sister?
b A Year 10 boy is talking with a Year 10 girl and asks her if she has any brothers
or sisters. She says, ‘Yes, I have an older one’. What is the probability that she
has a sister?
30 A recent survey of the heights (in cm) of a group of Year 10 boys resulted in the
following data.
172, 178, 159, 168, 167, 172, 177, 171, 169, 172,
170, 189, 173, 177, 169, 168, 171, 180, 174, 160,
175, 171, 173, 168, 170, 171, 172, 174, 168, 170
a Calculate the mean.
b Calculate the median.
c Examine the data and identify any possible outliers. If they exist, discuss the
effect of the outliers on the mean and the median.
31 The times, in seconds, of the duration of 20 TV advertisements shown in the
6–8 pm time slot are recorded below.
16 60 35 23 45 15 25 55 33 20 22 30 28 38 40 18 29 19 35 75

618 Maths Quest 10 New South Wales Australian curriculum edition Stages 5.1 and 5.2
problem solving

a From the data, determine the:


i mode
ii median
iii mean, write your answer correct to 2 decimal places
iv range
v lower quartile
vi upper quartile
vii interquartile range.
b Using your results from part a, construct a box plot for the time, in seconds, for
the 20 TV advertisements in the 6–8 pm time slot.
c From your box plot, determine:
i the percentage of advertisements that are more than 39 seconds in length
ii the percentage of advertisements that last between 21 and 39 seconds
iii the percentage of advertisements that are more than 21 seconds in length.
The types of TV advertisements during the 6−8 pm time slot were categorised as
Fast Food, Supermarkets, Program information, Retail (clothing, sporting goods,
furniture). A frequency table for the frequency of these advertisements being shown
during this time slot is shown below.
Type Frequency
Fast food 7
Supermarkets 5
Program information 3
Retail 5
d What type of data has been collected in the table?
e What percentage of advertisements are advertisements for fast food outlets?
f What would be good options for a graphical representation of this type of data?
32 Calculate the total interest paid on a 6 month loan of $14  000 with 10% p.a. reducible
interest. Each month $2000 is paid with the balance at the end of the 6 months.
33 In any leap year, the calendar for January is the same as the calendar for another
month in the year. Which month is this?
34 I have a combination of $10 notes and $5 notes in my wallet. If I have 27 notes
altogether and their total value is $190, how many of each type of note do I have?
35 Mariah the Mathematics teacher wanted to give her students a chance to win a
reward at the end of the term. She placed 20 cards into a box, and wrote the word
ON on 16 cards, and OFF on 4 cards. After a student chooses a card, that card is
replaced into the box for the next student to draw. If a student chooses an OFF card,
then they do not have to attend school on a specified day. If they choose an ON
card, then they do not receive a day off.
a Mick, a student, chose a random card from the box. What is the probability he
received a day off?
b Juanita, a student, chose a random card from the box after Mick. What is the
probability that she did not receive a day off?
c What is the probability that Mick and Juanita both received a day off?

Chapter 17 • Problem solving II 619


problem solving

36 Thomas went to an electronics store to buy a flat screen HD TV together with some
accessories. The store offered him two different loans to buy the television and
equipment.
The following agreement was struck with the store.
• Thomas will not be penalised for paying off the loans early.
• Thomas does not have to pay the principal and interest until the end of the loan
period.
Loan 1 $7000 for 3 years at 10.5% p.a. compounding yearly
Loan 2 $7000 for 5 years at 8% p.a. compounding yearly
a Explain which loan Thomas should
choose if he decides to pay off the
loan at the end of the first, second
or third year.
b Explain which loan Thomas should
choose for these two options.
Paying off Loan 1 at term
Paying off Loan 2 at the end of
four years
c Thomas considers the option to
pay off the loans at the end of
their terms. Explain how you can determine the better option without further
calculations.
d Why would Thomas decide to choose Loan 2 instead of Loan 1 (paying over its
full term), even if it cost him more money?
#
37 Express 4.27 as an improper fraction.
n
38 The mean mass of a group of n people is m kg. If another people whose mean
2
5m
mass is kg join the group, the mean mass changes to (m + 6) kg. Evaluate m.
4
39 A and B are complementary events and P(A) = a. Determine:
a P(B)
b P(A ∪ B)
c P(A ∩ B)
40 Lance lives in a small town with a population of 350 people. The town was
predicted to grow at rate of 10% per year.
a Write an equation to model the growth of this town over n years.
b How many years will it be until the population is over 1000 people? (Whole
years only)
c How many years will it be until the population is over 2000 people? (Whole
years only)
d Lance, using the data from part c, assumed that the town should have
approximately 10  000 people 95 years from now. Explain why his reasoning
is incorrect.
e Explain how you can help Lance see how the population will change over the
next 100 years.

620 Maths Quest 10 New South Wales Australian curriculum edition Stages 5.1 and 5.2
problem solving

41 Write the following expression in its simplest form.

18x 15x + 30 15
× ÷
(3x − 12) 2 x(x + 2) (x − 4) 2
42 A calculator company took a poll of 80 Year 10 students to find out what types of
calculators students had used during their academic careers:
Scientific calculators: 70
Graphing calculators: 50
Graphing and Scientific calculators: 40
What is the probability that a student had used a graphing calculator given that they
had also used a scientific calculator?
43 Jan bought a computer for her business at a cost of $2500. She elected to use the
diminishing value method (compound depreciation), instead of the straight‐line
method of depreciation. Her accountant told her that she was entitled to depreciate
the cost of the computer over 5 years at 40% per year.
a How much was the computer worth at the end of the first year?
b By how much could Jan reduce her taxable income at the end of the first year?
(The amount Jan can reduce her taxable income is equal to how much value the
asset lost from one year to the next.)
c Explain whether the amount she can deduct from her taxable income will
increase or decrease at the end of the second year.
44 A Shend is a type of tropical pumpkin grown by
the people of Outer Thrashia.
The diameter (D m) of a Shend increases over
a number of months (m) according to the rule
D = 0.25 × (10) 0.01m.
a Determine the diameter of the pumpkin after
4 months.
b If the Shend is not harvested it will explode
when it reaches a critical diameter of
0.5 metres. Show that it takes approximately
30 months for an unharvested Shend to explode.
45 There are 6 black discs and 9 red discs in a jar. Two discs are withdrawn
simultaneously. What is the probability that the discs are the same colour?
46 Natalina is going to build a movie theatre with n screens. At each screen, there
will be 250 seats for the audience to watch that movie. In addition to audience
members, there are 15 employees on the premises at any given time (selling tickets
and popcorn and so on). According to building regulations, she must have one toilet
for every 100 people in the building.
a Write an equation relating the number of screens (n) to the total number of
people who can possibly be in the building (p) any one time. Which variable is
dependent? Which variable is independent? Explain.
b Write an equation relating the total number of people who can possibly be in the
building (p) to the number of toilets (t).
c What kind of numbers do p, t and n have to be?

Chapter 17 • Problem solving II 621


problem solving

d If Natalina builds 5 theatre screens, what is the minimum number of toilets she
must also build?
e If Natalina can only supply eight toilets, what is the maximum number of
screens she can build?
47 Jason and Paul go to the shopping car park on weekends to see if they can find any
loose change that people have dropped. Over the past year they have kept track of
how much money they have found. They found twelve $2 coins, three fifty‐cent
pieces, thirty‐nine 20c coins, thirty‐eight 10c coins, and one hundred and fifty‐two
5c coins. What is the probability that the next coin they find will be worth more
than ten cents? State your answer as a percent to the nearest percent.
48 A set of examination results is displayed in the following cumulative frequency
histogram and ogive.
a Stephanie knows that her result is in the
85th percentile. Explain whether Stephanie 100

Cumulative frequency
can work out her exact result. 80
b What is the class median? 60
c Stephanie’s twin, Betty, knows that she got 40
70% for her exam. Can she compare her 20
exam result to Stephanie’s result?
0 30 40 50 60 70 80 90
d Approximately in which percentile would
Examination mark
Betty’s result lie?
49 Dimitri constructed a back‐to‐back stem plot to compare the ages of the students in
his dance class.
Ages of students attending at the Ballroom Dancing Studio
Females Males
9 1 1  2 3
7 2 0  2 2  2 4  5
5 3 0  0 1  7
5  2 4 6  7
3  2 0 5 2
4  4 2  1 6 4  4
a Compare the range of distribution of the ages of males and females from this set
of data.
b What is the mode for this set of data?
c One of Dimitri’s students, Anthea, used a grouped frequency distribution table of
all the students to calculate the mean age of the students at Dimitri’s Ballroom
Dancing Studio. Create the grouped frequency distribution table using class
intervals of 0–9, 10–19 etc.
d Anthea correctly calculated the mean from the grouped frequency distribution
table to be 39.5. Elena, another student, correctly used the original data in the
back‐to‐back stem plot and calculated the mean to be 38.2. Show how they both
got their answers.
e What is the reason for the difference in the two answers?

622 Maths Quest 10 New South Wales Australian curriculum edition Stages 5.1 and 5.2
problem solving

50 Tylar is offered a new Beta Brava, a sleek 4 cylinder sports car for $56  000. He
borrows the money and repays it at $12  500 per year for 5 years.
All new cars depreciate in value; the Beta Brava is no different. It loses 10% in
the first year and then 5% of the previous year’s value each year thereafter.
a What is the car worth after 5 years ?
b Taking the depreciation into account and considering how much he paid for his
car, how much has Tylar lost, on average, each year over the 5 years?
51 The equation 2x + 3y = 6 is changed according to the following rules.
i The whole equation is doubled. ii Only the x‐value is doubled.
iii Only the y‐value is doubled. iv The y‐intercept is doubled.
a Write the new equation in each case.
b Draw the graph of each new equation on the same axes as the original equation.
c Describe the effect of each transformation on the graph of the original equation.
52 Andrew does not know the answer to two questions on a multiple choice exam. The
first question has four choices and the second question he does not know has five
choices.
a What is the probability that he will get both questions wrong?
b If he is certain that one of the choices cannot be the answer in the first question,
how will this change the probability that he will get both questions wrong?
53 Lara is employed as a salesperson. She is offered two methods of calculating her
income.

Method 1: Commission only of 13% on all sales


Method 2: $350 per week plus a commission of 4.5% on all sales

Lara’s research shows that the average sales total per employee per month
is $14  382.
a If Lara were to choose her method based on the average employee sales total,
from which method of payment would she receive the most income, in dollars
per annum?
b Compare the difference, in dollars, between the two methods of payment based
on the average sales total. Justify your answer with calculations.
54 Rosetta and Theo have been married for 16 years but have kept separate investment
accounts during that time. The graph below shows the value of their investments
over time where interest has been paid annually.
Value of investments
A Theo’s
Value of investment ($)

25000 account
Rosetta’s
20000
account
15000
10000
5000

0 n
2 4 6 8 10 12 14 16
Time (years)

Chapter 17 • Problem solving II 623


problem solving

a What were the values of Rosetta’s and Theo’s initial investments?


b How much did Rosetta’s investment change yearly over time? What does this
represent in real terms?
c Give the equation to describe the value of Rosetta’s investment over time (A).
R n
d Theo’s investment grew according to the general formula A = Pa1 + b
100
compounded annually. Determine the rate at which Theo’s investment is growing.
e After initially investing the same amount of money, the graph shows that after
approximately 13 years their investments had grown to the same amount. Use
your equations to show that this is so.
f Compare the two investments over the 16 year period.
55 The marks for two Mathematics tests, A and B, for a class are presented in the box
plots below.
a Compare the five point summary Mathematics test A
for each.
Mathematics test B
b Which statistic appears to be the
same for both tests? What does 0 10 20 30 40 50 60 70 80 90 100
this indicate?
c In which test did the students appear to perform better?
d What can the teacher deduce about the tests from these results?
e What can the teacher deduce about her students’ learning from these results?
56 Your big brother has buried your mp3 player in the backyard to tease you. He’s
given you the following Cartesian co‐ordinate clues to help you find it, where x and
y are the horizontal and vertical distances from the back door in metres.
The back door is at (0, 0)
The tree is at (0, 3)
The shrub is at (8, 7)
The stump is at (8, 3)
The player is on the line y = −54x + 10 which bisects the line connecting the tree
and shrub and it is also 3 metres from the stump.
Draw a diagram to represent this information and mark the possible locations of
your player on the Cartesian plane.
57 In the scientific area of genetics, probability is used to assist in determining the
likelihood of inherited characteristics.
For example, a widow’s peak hairline is dominant; a straight hairline is recessive.
Consider a mother who is heterozygous dominant (Ww) for the widow’s peak and
a father who is homozygous recessive (ww).
W w
a Complete this table.
b Use your table to determine the probability that w
their offspring will have a widow’s peak. w
58 Penny is a softball player. The number of runs she
scored in her first eight games were 2, 3, 4, 5, 6, 6, 8, 9.
a Calculate the mean, median and mode for the number of runs to 1 decimal
place. Discuss which statistic is the best indicator of the measure of centre
in this case.

624 Maths Quest 10 New South Wales Australian curriculum edition Stages 5.1 and 5.2
problem solving

b Based on the above figures:


i what is the probability that in future games, Penny will score at least six runs?
ii in how many of her next five games would you expect her to score at least
six runs?
c Penny’s coach claims that it is possible for her to achieve a median of 7 if she
keeps up her practice.
i What is the minimum number of games she must play to have a median run
score of 7? At least how many runs will she need in each game?
ii Give a possible set of scores for these games that would allow Penny to
achieve this.
d Given your answer to part b above, how likely is this?
59 The surface area of a lake is evaporating at a rate of 5% per year due to climate
change. To model this situation, a relationship between the surface area of the lake
(S km2) over time is given by S = 20 000 × 0.95x, where x is the time in years.
a Explain whether this is an exponential relationship.
b What is the surface area of the lake initially?
c What will the surface area be in 10 years’ time?
d Plot a graph for this relationship.
e What will the surface area be in 100 years’ time?
f Explain whether this is a realistic model.
60 A bakery employs experienced bakers at $25 per hour
and apprentices at $16 per hour. The manager has a
budget that will allow her to employ four experienced
bakers to work a 40 hour week.
a How many apprentices who will only work for
30 hours per week can be employed for the same
amount of money?
b If the manager want to employ a combination of experienced bakers and
apprentices, how many of each could she employ for $4000 per week?
61 Bailey has built a wooden ramp to practise his skateboard tricks.
A

E D 2.2 m

3.6 m

F 12 m C

a Find the distance travelled up the ramp (i.e. distance FB).


b What angle does the ramp make with the ground?
c After construction, Bailey checks that the ramp is ‘square’ by measuring the
diagonals. Find the length of AF.
d Two such ramps are made so that they can be placed 10 m apart. If Bailey can cover
the distance from the top of one ramp, along the ground, then to the top of the other,
on his skateboard, in 12 seconds, how fast is he going? (Answer in km/h).

Chapter 17 • Problem solving II 625


problem solving

62 Briana’s normal rate of pay is $15.25 per hour. Last week she was paid for
11 hours, at time‐and‐a‐half.
a If Briana was paid at double the hourly rate, how many hours would she need to
work the next week, to earn the same amount of money?
b Briana’s boss is offering her the choice for two shifts, the rate paid depending on
the work days offered.
Shift 1: 15 hours at the normal rate plus 5 hours at time‐and‐a‐half.
Or
Shift 2: 10 hours at the normal rate plus 8 hours at double time.
Which shift offers a better wage and by how much?
63 At the entrance to a car rally, 35 people
were surveyed and asked which of three
models of 4WD rally cars they preferred —
Toyota, Nissan or Subaru. Six of the group
liked all three types of 4WD vehicles.
Eight of the group liked Toyota and Nissan,
10 liked Toyota and Subaru and 12 liked
Nissan and Subaru. Also 22 of the group
liked Toyota, 18 liked Nissan and 17 liked
Subaru. Two people didn’t like any of the
models of 4WD rally cars.
a Display this information on a Venn diagram.
b Determine the probability of selecting a person who:
i liked Toyota only
ii does not like the Subaru.
c Find the probability that a person likes Subaru or Nissan but not Toyota.
64 Bronwyn decides to buy a new laptop. The ticketed price is $1200. When
Bronwyn’s credit card statement arrives, it shows that she will not pay any interest
if she pays the full amount by the due date.
a If Bronwyn pays $300 by the due date, what is the balance owing?
b If the interest rate on the credit card is 22% p.a., how much interest will
Bronwyn be charged on the balance owing in the next month?
c How much will Bronwyn owe at the end of that month?
d Bronwyn now pays $600 off her credit card. How much interest is she charged
the following month?
e Bronwyn then pays off the remaining balance on her credit card. How much
does she pay?
f How much has the laptop cost her, including all the interest payments?
65 A pencil case contains 3 black pens and 2 red pens.
The pencil case is shaken, one falls out and is put
back in the case. This is repeated twice more. Each
pen has an equal chance of falling out.
a Represent this information on a tree diagram.
b Find the probability of getting three black pens.
c Find the probability of getting at least two red pens.

626 Maths Quest 10 New South Wales Australian curriculum edition Stages 5.1 and 5.2
problem solving

66 A well‐known problem in mathematics is called


the ‘handshake’ problem.
a 3 people are in a room. How many handshakes are
required so that everyone shakes hands with everyone
else once?
b Repeat the above exercise for 4 people.
c Develop a general rule for the number of handshakes
for n people.
67 The cost of a mobile phone from Company A is $40 per month plus $0.25 per
SMS, while Company B offers a plan for $30 a month but $0.30 per SMS. How
many SMSs would make the plans the same monthly cost?
68 At a baby shower, we started discussing baby
statistics. One of the women told us she had heard
a report that for every 100 babies born, there were
6 more boys than girls. If we were to randomly
pick a child from a representative group, what is
the probability of picking a girl?
69 The numbat is an Australian animal which is in
danger of becoming extinct because of habitat loss
and foxes. Since it only eats white ants, its source of food is also diminishing. The
number of numbats, t weeks after their habitat has been lost is modelled by the
500
function N = 60 + numbats per hectare.
t+2
a How many numbats were there before their habitat was lost?
b How many numbats are there 10 weeks after their habitat loss?
c How long after habitat loss are there only 80 numbats per hectare?
d According to this model, will the numbats die out completely?

70 The amount of money in an investment plan (V) grows exponentially with


compound interest according to the rule V = PAn, where n = the time (in years),
P = the initial investment and A is the compound interest term.
After 1 year (n = 1), the amount of money in the investment is $2662.50, while
after 2 years the amount is $2835.56.
a Determine the values of P and A.
b Determine the annual rate of interest as a percentage.

Chapter 17 • Problem solving II 627


problem solving

71 In the game of draw poker, a player is dealt


5 cards from a deck of 52. To obtain a flush,
all 5 cards must be of the same suit.
a Determine the probability of getting a
diamond flush.
b Determine the probability of getting
any flush.
72 You use this spinner to play a game. Explain
whether it is a fair game and whether you
would play if the scoring was:
a if the pointer lands in A, you win $8, if the pointer lands in B,
you win $3 B
b if the pointer lands in A, you win $18, if the pointer lands in B,
you win $5. A

73 Moore’s Law states that the capacity of computer hard drives


doubles every 18 months. Assume that on 1 January 2010 the
capacity of a hard drive was 512 GB (1 GB = 220 bytes).
a Sketch a graph showing hard drive capacity going back at least 10 years.
b What was the capacity on 1 January 2000?
c Predict the capacity on 1 January 2016.
74 In the game of blackjack, players are initially given 2 cards from a deck of 52.
Face cards (jack, queen, king) are worth 10 points, an ace is either 1 or 11 (player’s
choice) and other cards (2–10) are worth their numerical value. The value of the
2 cards dealt is added together; for example; 2 + king = 2 + 10 = 12.
a What is the probability of getting a total of 10 with the first 2 cards?
b What is the probability of getting a total of 20 with the first 2 cards?
75 A small car yard has 60 cars, half of which are Toyotas. Since running a car on LPG
rather than petrol has become one of the most popular features, 25 cars are LPG
based, including 20 of the Toyotas. There are some 4WD vehicles in the car yard, 5 of
which are Toyota and 6 of which have LPG systems. Unfortunately only one of the
Toyota 4WDs is also LPG based. 18 of the vehicles have none of the characteristics
already mentioned. Draw a Venn diagram and then answer the following questions. If
a customer looks at a vehicle, find the probability that the vehicle is:
a a 4WD vehicle
b an LPG Toyota
c not LPG based
d not a Toyota
e either a Toyota or a 4WD
f neither LPG based nor 4WD
g an LPG system but is not a Toyota
h not an LPG based Toyota 4WD.
76 Patrick and Trisha were told that they needed a deposit of $34  000 before they
could borrow enough money to buy their new home. At that time Trisha had saved
$6200 and Patrick had saved $7400. They both worked and although they paid rent
of $1280 per month they are able to save $260 per week.

628 Maths Quest 10 New South Wales Australian curriculum edition Stages 5.1 and 5.2
problem solving

a When they made enquiries about the loan, how much less than the deposit did
they have?
b At their current rate of savings, how long will it take them to save the rest of the
deposit?
c If the deposit is 10% of the cost of the home, what was the cost of the house?
d How much money will they have to borrow once they get their deposit?
e How much interest is charged for the first month, if the interest rate is 9% p.a.
reducible?
f If Trisha and Patrick use their savings every week and the amount they would
have spent on rent to repay the loan, how much can they afford to pay every
month?
g How much would they owe at the beginning of the second month?
h What will happen if interest rates rise to 9.5% p.a. at the beginning of the second
month?
i What would you advise Trisha and Patrick?
77 100 people seated at different tables in a Mexican restaurant were asked if their
party had ordered any of the following items: burritos, fajitas, or tacos. The
following information was found.
23 people had ordered none of these
items.
11 people had ordered all three of
these items.
29 people had ordered fajitas or tacos
but did not order burritos.
41 people had ordered tacos.
46 people had ordered at least two of
these items.
13 people had ordered burritos and
tacos but had not ordered fajitas.
26 people had ordered burritos and fajitas.
a Place this information in a Venn diagram.
b How many people ordered Burritos only?
c How many people ordered Fajitas?
78 To determine whether a game is fair the expected value is calculated. In its simplest
form:
expected value = P(win) × (prize) − P(loss) × (cost of game)
If there is more than one prize, with an associated probability, then add up the
various P(win) × prize terms.
a A game is fair if expected value = ______
b Consider the following game based on the total of 2 dice.

2 3 4 5 6 7 8 9 10 11 12
W1 L2 W3 L4 W5 L5 W4 L3 W2 L1 W1

Chapter 17 • Problem solving II 629


problem solving

Where, for example, W5 means you win $5, L2 means you lose $2 and so on.
Determine the expected value and comment on your result.
79 Ethan wants to borrow $30  000 to buy a new Toyota corolla. He finds a bank that
will give him the loan at 12% flat‐rate interest over 5 years.
a Calculate the amount of interest that Ethan will pay on this loan.
b Calculate the total amount that Ethan must repay on his loan.
c Ethan decides to repay the loan in 5 equal yearly instalments. Calculate the
amount of each instalment.
d Ethan could have borrowed $30  000 at 12% reducible interest instead of flat‐rate
interest. If he makes the same repayments for 4 years, how much money will
Ethan have saved?
80 An electronics store offers its customers 2 choices.
Option 1: 20% down then make equal payments (based on the money owing at the
beginning of the loan) every 6 months for 2 years. The interest rate (simple) is 8%
(annual) of the amount owing at the time of the payment.
Option 2: No money down, with 10 equal payments (based on the money owing at
the beginning of the loan), 1 per month. The interest rate (simple) is 12% (annual)
of the amount owing at the time of the payment.
In both cases, the balance owing at the end of the time period is to be paid in full.
Calculate the better deal on a $200 TV. Justify your conclusions with appropriate
calculations.
81 In the game of major league baseball, a very rare event
is what is called a ‘perfect game’. This occurs when the
pitcher gets all men out. Since there are 9 innings and
3 outs/inning, this means that the pitcher must get 27 men in
a row out.
The most recent perfect game was by Mark Buehrle,
July 23, 2009.
a Given that the probability a batter who faces Mark Buehrle
does not get out is 0.313, determine the probability that he
throws a perfect game.
b Given that there are about 2000 games per year, about how
often is a perfect game thrown.
82 The test scores, out of a total score of 50, for two classes A
and B are shown in the stem plot below.
Class A Class B
5 0 1  2 4
9  7 5  3 1 1  4 5
9  7 7  5 4 2 0  0 5
8  8 6  5 5  1 3 1  5 5
3  2 0 4 1  5 7  7 8  9
0 5 0  0

630 Maths Quest 10 New South Wales Australian curriculum edition Stages 5.1 and 5.2
problem solving

a Ms Vinculum teaches both classes and made the statement that ‘Class A’s
performance on the test showed that the students’ ability was more closely
matched than the students’ ability in Class B’. By finding the measure of centre,
first and third quartiles, and the measure of spread for the test scores for each
class, explain if Ms Vinculum’s statement was correct.
b Would it be correct to say that Class A performed better on the test than Class B?
Justify your answer by comparing the quartiles and median for each class.
83 There are two major methods of calculating depreciation; the usual reducing value
depreciation and what is called straight‐line depreciation. In the latter you are
allowed to claim the same, fixed amount each year until there is no value left. For
example, over 5 years one can claim 15 of the item’s initial value each year.
Compare the two methods of depreciation in the following cases.
a Original price = $1000
Reducing value depreciation of 30% for 3 years
Straight‐line depreciation of 20% of original value each year for 5 years
b Original price = $5000
Reducing value depreciation of 20% for 5 years
Straight‐line depreciation of 16.7% of original value each year for 6 years
84 Helena is the owner of the web store ‘Warm as Toast’ which makes and
sells sheepskin slippers. It costs Helena $1000 for machinery and materials
plus a cost of $15 for each slipper made.
a Write an equation that determines the cost, C in dollars, of each slipper,
s, made.
b Helena is able to sell each pair of slippers for $25. Write an equation that
determines the revenue, R, in dollars, Helena makes on each slipper sold, s.
c Using your equations from parts a and b, how many slippers does
Helena need to sell to meet her costs?
d Helena makes a profit of $2500. By writing an equation that determines the
profit, P, in dollars, Helena makes on each pair of slippers sold, determine the
number of slippers Helena sold to make this profit.
85 Fred purchases a plasma TV on a payment plan. He pays 15% deposit and then
equal monthly repayments of $105.50 for 24 months. The TV is priced at $2650.
a Determine the amount Fred pays in deposit. Write your answer to the nearest cent.
b After the deposit is paid, determine the amount owing on the TV.
c How much does Fred pay for the TV after he has made 24 equal repayments?
d Determine the amount Fred pays extra for the TV.
e Using your value from part d determine the percentage interest Fred pays on the
money he borrowed for the TV. Base your calculations on simple interest. Write
your answer correct to 1 decimal place.
86 A sheet of paper is torn in half, and the pieces placed on top of one another.
These pieces together are torn in half, and the new pieces are placed on top of one
another. This process continues through 20 such tears, with the pieces being placed
on top of each other each time. If the original piece of paper was 0.01 mm thick,
what is the height of the final pile of pieces of paper? Give your answer in metres,
correct to 1 decimal place.

Chapter 17 • Problem solving II 631


problem solving

87 The amount of energy the sun delivers to the


surface of the Earth each second is known as
the Solar Constant. It is measured to be
1.35 × 106 ergs/cm2 each second. If 1 watt is
equal to 1 × 107 ergs/s, what is the Solar
Constant each second, expressed in watts/m2.
88 Theresa’s home office equipment is
depreciated based on reducing balance
method at a rate of x% per year until the scrap
value is less than $300. The office equipment had an initial value of $3700. After
7 years the office equipment will, for the first time, have a value below the scrap
value of $300. Determine the rate of depreciation, x, correct to 2 decimal places.
89 Gloria is investing $25  000 in a bank account. She is offered two different interest
options:
Option A: 4.5% p.a. compounded monthly
Option B: 4.75% p.a. compounded half yearly
Gloria decides to invest using Option B for a period of five years. Show, by finding
the amount Gloria’s investment has grown after 5 years, that she receives $319.08
more using Option B than Option A.
90 Tracy places her money in an investment account that earns x% interest per year
for five years. At the end of each year the amount of interest for the 12 months is
added to the sum in the account. At the end of the first year her $2500, with interest
added, has grown to $2575. At the end of the second year the amount of money in
her account was $2652.25.
a If the rate of interest Tracy’s money earned over the first two years was constant,
determine the value of x.
b At the end of the second year, the rate of interest was increased. This rate did
not change for the next two years. At the end of the fourth year, the amount of
money in Tracy’s account was $2896.32. Determine the new rate of interest
Tracy’s account earned during the third and fourth years. Write your answer
correct to 1 decimal place.
c After interest had been paid into Tracy’s account at the end of the fourth year,
she was able to deposit another $1000 into the account. The interest rate earned
on the account was increased by 1.5%. Determine the amount of money Tracy
will have in her account at the end of the fifth year. Write your answer to the
nearest cent.
91 Andy, Bill and Cam each have a mobile phone with a different payment plan.
Andy’s plan
40c flag fall plus 45c for every 30 sec or part thereof of a call
Bill’s plan
35c flag fall plus 92c per minute or part thereof of a call
Cam’s plan
36c flag fall plus 46c per 30 second or part thereof of a call
The boys call each other regularly, with calls lasting up to 3 min. Give an analysis
of call costs for each of them for calls lasting up to 3 mins.

632 Maths Quest 10 New South Wales Australian curriculum edition Stages 5.1 and 5.2
problem solving

92 In financial circles, there is a formula that is commonly used to determine the


length of time it takes to double a sum of money invested at a compound interest
rate of R% p.a.
n
Number of years to double money = .
R
Using an investment of $1000 and an interest rate of 10% p.a., determine a value
for n.
93 A draw for a tennis tournament looks like a tree diagram. This could be the draw
for eight players.
Round 1 Round 2 Round 3
X X
X
X

X X
X
Champion
X X
X
X
X X
X

In this type of draw, a player is out of the tournament after one loss. With an odd
number of players, one of the competitors is given a ‘bye’, which means the player
does not compete in that round. No competitor can be given more than one bye in a
tournament.
Consider a tournament of this type with nine tennis players.
a Draw a diagram showing how this draw could be played.
b What is the maximum number of games the champion will have to play?
c What would be the minimum number of games the champion could play?
94 Sandy’s savings account pays a simple interest rate of 3.5% on daily balances (the
interest is calculated daily on the balance in the account). Her statement for July
looks like this.

Date Deposit Withdrawal Balance


1/7 $4200
3/7 $100
7/7 $500
21/7 $725
28/7 $85
31/7

Chapter 17 • Problem solving II 633


problem solving

a Complete the Balance column for the month of July.


b In order to calculate the interest she has earned for the month, complete the
following table.
Date Number of days Balance Interest calculation Interest earned
2
4200 × 3.5 ×
2/7 2 $4200 365 $0.81
100
6/7 4
20/7
27/7
31/7

What is Sandy’s total interest for the month?


95 In a school election for House Captain there are six candidates — let’s call them
A, B, C, D, E and F. After the election, the following facts were released.
• A won with 50 votes.
• B was second.
• F was last with 5 votes.
• No two candidates received the same number of votes.
• 100 students voted.
What is the greatest number of votes B could have received?
96 In my drawer I have socks of 5 different colours, and 10 of each colour. They are
not in pairs, and randomly distributed in the drawer. In the dark I go to my drawer
to get 4 socks of the same colour. How many would I need to select to be certain of
getting 4 of the same colour?
97 A die in the shape of a dodecahedron has regular pentagonal faces numbered
1 to 12. The sum of the numbers on each pair of opposite faces equals 13.
With its face numbered 1 uppermost, it is adjacent to faces numbered 2, 4, 6, 5
and 3, reading clockwise.

3 2
1
5 4
6

If the 6 was sitting uppermost, draw a diagram to show the numbers adjacent to
its face.

634 Maths Quest 10 New South Wales Australian curriculum edition Stages 5.1 and 5.2
problem solving

Answers
CHAPTER 17 Problem solving II
1 4 d y
2    a 4 am, 7 January 25
b 1 am, 7 January (Perth is 3 hours behind Sydney during

Cumulative frequency
daylight saving time.) 20
3    a No. Greg’s first choice of a number is independent of his second
choice of a number. The ten numbers will always be in the hat on 15
the first draw, thus he has an equal chance to pick any of them.
b Greg’s second choice is dependent upon his first choice. 10
When Greg chooses a number in the first draw and does not
5
replace that number, then he changes the sample space and the
probability for the second draw.
0 x
4    a $1175 b $3825 5 10 15 20 25
Number of computers used
c 25% discount gives a final price of $3750. The customer
would be $75 better off. e 10 students
5    a y f y

Percentage cumulative
2 100
80

frequency
60
1 40
y = 4–x
0.5 20
0 x
0 x 5 10 15 20 25
1 2 3 Number of computers
used

b y becomes smaller and approaches 0, but never actually g 50th percentile is about 8.5 computers used, 30th percentile is
reaches 0. 5 computers used. This means that 50% of the data lies below
c y approaches infinity as x becomes smaller. 8.5 and 30% of the data lies below 5.
27
16    a Males
6 52 Females
7 $20  960.94
8    a $100 b 14.29%
x
9 14 0 10 20 30 40 50 60 70 80
Age
10 Annie is 18 and Barbara is 7. b Males Females
11   a $45  058.50 b $455 per year
12 The numbers are 7 and 10. Mean 28.2 31.1
13 149 cm and 171 cm. The average height of the two students must Range 70 57
equal 160 cm. IQR 18 22
14 100 mini-laptops
c There is one outlier — a male aged 78.
15    a Class interval Frequency d Typically males seem to enter hospital for the first time at a
0−4 7 younger age than females.
17    a $9920
5−9 8
b Nathan will have $9993.03 at maturity.
10−14 5 c Because Nathan’s interest is compounded, the interest is
15−19 4 added to the principal each quarter and earns interest itself.
20−24 1 18    a $2090 b 104.5% c $2184.05
d A
Total 25
12000
b, c y
Investment value ($)

10000
8 8000

6 6000
Frequency

4000
4
2000
2
0 x
10 20 30 40
0 x Time (years)
0 5 10 15 20 25
Number of computers used e Approximately 36.5 years

Chapter 17 • Problem solving II 635


problem solving

19    a $0.165 b 50
b 217 cups
20 12 years
40
21    a Mean = 2.17, median = 2

Cumulative frequency
b Mean = 3.54, median = 2
30
c The median relies on the middle value of the data and won’t
change much if an extra value is added. The mean however
has increased because this large value will change the average 20
of the numbers. The mean is used as a measure of central
tendency if there are no outliers or if the data are symmetrical. 10
The median is used as a measure of central tendency if there
are outliers or the data are skewed.
0
22    a $17  176.86 b $5176.86 8 10 12 14 16 18
Years of education
c $357.85
c Parents of year 10 students
23 0.8
24    a
Interval Frequency ( f ) Midpoint × ( f )
40–49 1 44.5 × 1 = 44.5 8 10 12 14 16 18
x
Years of education
50–59 1 54.5 × 1 = 54.5
60–69 1 64.5 × 1 = 64.5 28    a 9.10% b 4.45%
c 5.76%
70–79 2 74.5 × 2 = 149
29    a 13 b 1
80–89 4 84.5 × 4 = 338 2
30    a 171.6 cm
90–99 4 94.5 × 4 = 378
b 171 cm
100–109 8 104.5 × 8 = 836 c At the top end of the heights, there is a possible outlier of
110–119 6 114.5 × 6 = 687 189 cm. Removal of this value results in a mean of 171 cm
and a median of 171 cm. The mean is reduced slightly, while
120–129 8 124.5 × 8 = 996 the median is unchanged.
130–139 2 134.5 × 2 = 269 At the lower end of the heights, there are possible outliers
140–149 2 144.5 × 2 = 289 of 159 cm and 160 cm. Removal of these values results in a
mean of 172.5 cm and a median of 171.5 cm. The mean and
150–159 0 154.5 × 0 = 0 median are increased slightly.
160–169 1 164.5 × 1 = 164.5 Removal of both the upper and lower outliers results in
a mean of 171.9 cm and a median of 171 cm. The mean is
TOTAL 40 4270 increased slightly, while the median is unchanged.
b 106.75 c 107.15 31    a i   35 s
d The differences in this case were minimal; however, the ii   29.5 s
grouped data mean is not based on the actual data but on the iii   33.05 s
frequency in each interval and the interval midpoint. It is
iv   60 s
unlikely to yield an identical value to the actual mean. The
spread of the scores within the class interval has a great effect v   21 s
on the grouped data mean. vi   39 s
25    a October 1: $89.10, October 8: $80.19, October 12: $84.20, vii   18 s
October 15: $75.78 b 21 29.5 39
b 23.5%
26 $1060.26, $2003.30, $7585.60 t
15 20 25 30 35 40 45 50 55 60 65 70 75
27    a Years of Cumulative c i   25%
education Frequency frequency ii   50%
9 2 2 iii   75%
d Categorical
10 4 6 e 35%
11 8 14 f Pictogram, pie chart or bar chart
12 17 31 32 $461.96
33 July
13 6 37 34 16 × $5 notes and 11 × $10 notes
14 5 42 35 a 15 b 45 1
c 25
15 4 46 36 a Since the interest rate is lower for Loan 2 than for Loan 1,
Thomas should choose Loan 2 if he decides to pay the loan
16 3 49
off at the end of the first, second or third year.
17 1 50 b Loan 1 at term amounts to $9444.63. Loan 2 at the end of 4
Total 50 years amounts to $9523.42. Thomas should choose Loan 1.

636 Maths Quest 10 New South Wales Australian curriculum edition Stages 5.1 and 5.2
problem solving

c Thomas should choose Loan 1. At the end of its term (3 d


years), it amounts to less than Loan 2 at 4 years, 1 year before Midpoint Frequency
its term is finished. Class interval x f fx
d Thomas may not have the money to pay off Loan 1 in 3 years.
He may need the extra 2 years to accumulate his funds. 0–9 4.5 0 0
37 77
18 10–19 14.5 4 58
38 72
39 a 1 − a b 1 c 0 20–29 24.5 7 171.5
40 a Future population in n years = 350(1 + 0.1) n.
b 12 years 30–39 34.5 5 172.5
c 19 years
d Lance has assumed that every 19 years there will be 40–49 44.5 4 178
approximately 2140 additional people.
e Lance has assumed that the growth is linear, whereas it is 50–59 54.5 4 218
actually exponential. Drawing a graph would help him see the
growth. 60–69 64.5 6 387
41 2
42 47
43 a $1500 a fx 1185
af
b $1000 Anthea: Mean = = = 39.5
30
c Since the depreciation of 40% is on a lower value each year,

a x 1146
the amount Jan can deduct from her taxable income decreases
every year.
44 a 0.27 m Elena: Mean = = = 38.2
n 30
b Teacher to check.
45 17 e Anthea used the midpoints of the class intervals, whereas
35 Elena used the exact values.
46 a p = 250n + 15, n is independent variable and p dependent
variable — number of people depend on number of screens. 50 a $41  051 b Approx $4300
p 51    a i   4x + 6y = 12
b t=
100 ii   4x + 3y = 6
c Integers
iii   2x + 6y = 6
d At least 13
e 3 iv   2x + 3y = 12
b i    y
47 22%
4
48 a Stephanie can only work out which class interval her test
result is in — the 65% to 74% interval.
4x 2 2x +
b The median is the 50th percentile, which corresponds to the +
6y 3y
= =
55% to 64% interval. 12 6
c No.
0 x
d 85th — the same as Stephanie. –4 –2 2 4

49 a Range of females = 45 years; –2


range of males = 53 years
b Females: 64 years; males: 22 years
–4
c
Class interval Class mid-point Frequency
ii    y
0–9 4.5 0 4

10–19 14.5 4
2 2x
+
20–29 24.5 7 6y
=
6
0 x
30–39 34.5 5 –4 –2 2 4
4x
+

40–49 44.5 4 –2
3y
=
6

50–59 54.5 4
–4

60–69 64.5 6

Chapter 17 • Problem solving II 637


problem solving

iii   y 56 y
2x 4 8 Shrub
+
3y
=
6 6
2x +
6y =
6 2 4
Tree Stump
2

–4 –2 0 2 4 x 0 x
2 4 6 8 10 12 14

–2
57    a
W w
–4 w Ww ww
w Ww ww
iv    y
2x
+ 4 b 50%
2x
3y
= +
3y
58    a Mean = 5.4; median = 5.5; mode = 6
6 =
12 The median is best because it allows for the range of values
2
and is also between the mean and mode.
b i   0.5 ii   2 or 3
x c i   4 more games scoring at least 8 runs in each
–4 –2 0 2 4
ii   2 3 4 5 6 6 8 8 8 8 8 9
–2
d Not very likely
59    a Yes, because the relationship involves a variable as an
exponent.
–4 b 20 000 km2
c S = 11 975 km2
c i  The two equations are the same, so the graphs lie on top of d y

each other. 20 000

  ii  The gradient is doubled, the y-intercept is unchanged, the


Surface area (km2)

x-intercept is halved. 15 000


  iii  The gradient is halved, the y-intercept is halved, the
x-intercept is unchanged. 10 000
  iv  The gradient is unchanged, the y-intercept is doubled, the
x-intercept is doubled. These are parallel lines. 5000
3
52    a 5
8
b 15 0 x
20 40 60 80 100
53    a Method 2 Years
b Difference of $3530 in favour of Method 2. e In 100 years, S = 118 km2
54    a $5000 f No this is not a realistic model as is it does not take into
b $1000, interest earned per annum. account changes to climate, rain, runoff from mountains,
c A = 5000 + 1000n glaciers etc.
d R = 10.5%, This is the percentage interest rate per annum. So, 60    a About 8 apprentices can work for the same amount of money.
R = 10.41% p.a. b
e Check with your teacher. Experienced Apprentices
f Rosetta had a higher return in the first 13 years then Theo had bakers $1000 each 30 h × 416 = 4480 Total cost
the higher return after that. 4 ($4000) 0 $4000
55    a 3 ($3000) 2 1 4480 2 = 4960 $3960
Xmin QL Median QU Xmax
2 ($2000) 4 1 4480 2 = 41920 $3920
Test A 20 40 50 70 80 1 ($1000) 6 1 4480 2 = 42880 $3880
Test B 10 50 60 80 90 0 8 1 4480 2 = 43840 $3840
b The interquartile range is the same for both tests (IQR = 30).
This indicates that the spread of the results across the middle 61    a 12.2 m
group of the class for both tests is the same. b 10.4°
c Based on the median score, the students appear to have done c 12.72 m
better in Test B. The middle groups stayed the same, the d 10.3 km/h
strong maths students did better, but the struggling students 62    a 8.25 hours
did worse. b Shift 2 offers $53.38 more.
d Test B may have been an easier test than Test A.
e Students may have found topic B easier to understand.

638 Maths Quest 10 New South Wales Australian curriculum edition Stages 5.1 and 5.2
problem solving

63    a Toyota Nissan b Approx. 5 GB


(22) (18) c 8192 GB
10 2 4 74    a 0.053
b 0.103
6 75 Toyota LPG System
4 6

1 6 19 0
2
(17) 1
Subaru 4 5

7
b i   27 ii   18
35
11 4WD 18
c 35 17 1
64    a $900 c $916.50 e $322.30 a 60
b 3
b $16.50 d $5.80 f $1222.30 c 7
d 1
12 2
65    a 3 7 2
3 5 Black
e 10
f 5
5 Black
Red 1 59
Black
2 g 12
h 60
3 3 5
5 5 Black
2
5
Red 76    a $20  400
Red
3
2
5
b 79 weeks (112 years)
3
Black
5 Black c $340  000
5
2 2 Red d $306  000
5 Red 5
3
5
e $2295
Black
2 Red f $2406.67
5 2 Red
5 g $305  888.33
h Payments will be more than they are able to afford.
b 0.216 i Find a less-expensive house, save more money then pay more
c 0.352 off the principal each month, find a loan with a lower interest
66    a 3 rate, save for a larger deposit.
b 6 77    a
n(n − 1) Burritos Fajitas
c
2 15
67 200 9 12
68 0.47 11
13 7
69    a 310
b 101 10
c 23 weeks
d No Tacos 23
70    a P = 2500, A = 1.065 b 9
b 6.5% c 45
71    a 0.000  495 78    a $0
b 0.001  981 b This game has a total expected loss of $36 2
, so the game is not
72    a It is 3 times more likely that the spinner will land on A. fair and is biased against the player.
Therefore I would not play this game as I should be winning 79    a $18  000
$9 not $8. b $48  000
b I would play this game as a fair game would only give me c $9600
$15. To get $18 would be in my favour. d $8117.09
73    a y 80 Option 2 is less expensive, despite having a higher annual
500 interest rate, because it is paid off in only 10 months instead of
2 years. The TV would cost $217 with option 1 and $212 with
option 2.
400
81    a 0.000  039  6
b About once in 12.6 years
Capacity (GB)

300 82 a Based on the comparison between Class A’s IQR (16.5) and Class
B’s IQR (32.5), Ms Vinculum was correct in her statement.
b No
200
83 a Reducing value allows you to claim $300, $210, $147 over
the 3 years for a total of $657.
100 Straight line allows you to claim $200 each year over 5 years
for a total of $1000.
Although the reducing value depreciation is greater in the
0 x years 1 and 2, the sum over the life of the item is greater in
1/1/95 1/1/01 1/1/07 1/1/10
Date the straight line case.

Chapter 17 • Problem solving II 639


problem solving

b Reducing value: $1000, $800, $640, $512, $409.60 for a total 94 a


of $3361.60 Date Deposit Withdrawal Balance
Straight line allows $835 per year over 6 years for a total of
$5000 (actually $5010). 1/7 $4200
In this case the reducing value method is only better in the 1st
year. 3/7 $100 $4300
84 a C = 1000 + 15s b R = 25s
c 100 d 350 slippers 7/7 $500 $4800
85 a $397.50 b $2252.50 c $2929.50
d $279.50 e 6.2% p.a. 21/7 $725 $4075
86 10.5 m
87 1350 watts/m2 28/7 $85 $4160
88 30.16%
89 Student’s own work 31/7 $4160
90 a 3 b 4.5% c $4130.10
91 For each 30-sec block, the lowest-cost calls are shown in blue, b Total interest earned is $13.24.
while the highest-cost calls are shown in red.
Number Interest Interest
Call Date of days Balance calculation earned
length Andy (cents) Bill (cents) Cam (cents)
2
4200 × 3.5 ×
30 sec 40 + 45 35 + 92 36 + 46 365
= 85 = 127 = 82 2/7 2 $4200 $0.81
100
1 min 40 + 45 × 2 35 + 92 36 + 46 × 2 4
= 130 = 127 = 128 4300 × 3.5 ×
365
6/7 4 $4300 $1.65
1 min 100
40 + 45 × 3 35 + 92 × 2 36 + 46 × 3
30 sec
= 175 = 219 = 174 14
4800 × 3.5 ×
2 min 40 + 45 × 4 35 + 92 × 2 36 + 46 × 4 20/7 14 $4800
365
$6.44
= 220 = 219 = 220 100
2 min 7
40 + 45 × 5 35 + 92 × 3 36 + 46 × 5 4075 × 3.5 ×
30 sec 365
= 265 = 311 = 266 27/7 7 $4075 $2.74
100
3 min 40 + 45 × 6 35 + 92 × 3 36 + 46 × 6
= 310 = 311 = 312 4
4160 × 3.5 ×
365
31/7 4 $4160 $1.60
If calls last a full number of minutes, as opposed to half-minute 100
calls, Bill’s plan seems to be a good one. The 30-sec plans seem
to be best for calls lasting just over the minute.
92 70 95 24 votes
96 16
93 a The draw for 9 players could look like this. 97 The faces adjacent to face number 6 would be 1, 4, 10, 11 and 5.
Round 1 Round 2 Round 3 Round 4
1
X X
5 4
X 6
X Bye
X 11 10
X X
X
Champion
X X Bye
X
X
X
X X
X X
Bye
X X

b 4
c 3

640 Maths Quest 10 New South Wales Australian curriculum edition Stages 5.1 and 5.2
Glossary
2-dimensional: a description of a plane shape. The Arc (of a circle): a portion of the circumference of a circle
dimensions are given in two directions, such as length and Area: the amount of flat surface enclosed by the shape. It
width or length and height. is measured in square units, such as square metres, m2, or
3-dimensional: a shape that occupies space (a solid). That is, square kilometres, km2.
one that has dimensions in three directions — length, width Area of triangle (using sine): if the perpendicular
and height. height of a triangle is not known, but two sides
Addition Law of probability: if A and B are mutually and the included angle are known, the area of any
exclusive events, then P(A or B) = P(A) + P(B) or triangle ABC can be calculated using the rule:
P(A ∪ B) = P(A) + P(B). Area = 12 ab sin C, Area = 12 ac sin B or Area = 12 bc sin A
Adjacent angles: angles at a point that share a common ray B
and a common vertex
A c
a
C A
b
O B C

Algebraic expression: an expression formed by numbers Associative Law: a method of combining two numbers or
and algebraic symbols using arithmetic operations. For algebraic expressions is associative if the result of the
example, 4x + 3y − 2 is an algebraic expression. combination of these objects does not depend on the way in
which the objects are grouped. Addition and multiplication
Algebraic fractions: fractions that contain pronumerals (letters)
obey the Associative Law, but subtraction and division are
Algebraic term: an algebraic expression that forms a
not associative.
‘separable’ part of some other algebraic expression. For
Asymptote: a line that a graph approaches but never meets
example, in the expression 4x + 2y − 3, 4x and 2y are
algebraic terms, while −3 is a constant term. Average: see Mean
Alternate angles: angles on alternate sides of a transversal. Average speed: the total distance travelled during a journey
On parallel lines, alternate angles are equal. divided by the total time taken. It is given by the formula:
total distance travelled
average speed = .
total time taken
Back-to-back stem-and-leaf plot: a method for comparing
two data distributions by attaching two sets of ‘leaves’
to the same ‘stem’ in a stem-and-leaf plot; for example,
Amplitude: half the distance between the maximum and
comparing the pulse rate before and after exercise
minimum values of a function
Angle of depression: the angle measured down from the Pulse rate
horizontal line (through the observation point) to the line Before After
of vision
9888 6
Horizontal
Angle of depression 8664110 7
8862 8 6788
Line of sight 60 9 02245899
4 10 0 4 4
Object 0 11 8
Angle of elevation: the angle measured up from the horizontal 12 4 4
line (through the observation point) to the line of vision
13
Object
14 6
Line of sight
Bar graph: a graph drawn in a similar way to a column
graph, with horizontal bars instead of vertical columns.
Angle of elevation
Categories are graphed on the vertical axis and the
Horizontal frequencies (numbers) on the horizontal axis.

Glossary 641
Base: the digit at the bottom of numbers written in index Circumference: distance around the outside of a circle. It is
form. For example, in 64, the base is 6. This tells us that given by the rule 2πr or πD, where r is the radius and D is
6 is multiplied by itself four times. the diameter of the circle.
Bi-modal: describes data whose distribution has two modes Class interval: a subdivision of a set of data. For example,
Bisect: cut into two equal parts students’ heights may be grouped into class intervals of
Bivariate data: sets of data where each piece is represented 150 cm − 154 cm, 155 cm − 159 cm.
by two variables Closure Law: when an operation is performed on an element
Boxplots (box-and-whisker plots): a graphical (or elements) of a set, the result produced must also be an
representation of the 5-number summary; that is, the lowest element of that set.
score, lower quartile, median, upper quartile and highest Coefficient: the number part of a term, generally written in
score, for a particular set of data front of the pronumeral
Coincident: lines that lie on top of each other
Co-interior angles: angles that lie on the same side of a
transversal that cuts across a pair of lines. For a pair of
65 70 75 80 85 90 95 100 105 110 parallel lines, co-interior angles are supplementary (add
Pulse rate to 180°).
Brackets: also called grouping symbols Q
By eye: by carefully looking at the data and drawing a line
so that there is an equal number of points above and below
the line G D
β
Capacity: the maximum amount of fluid that can be
C α B
contained in an object. It is usually applied to the
measurement of liquids and is measured in units such as F
A
millilitres (mL), litres (L) and kilolitres (kL).
Cartesian coordinate system: the position of any point in P
the Cartesian plane can be represented by an ordered pair of Collinear points: points that all lie on the same straight line
numbers (x, y). These are called the coordinates of the point.
Column graph: a graph in which equal width columns are used
y
5
to represent the frequencies (numbers) of different categories
4 x-coordinate Common factor (common divisor): a factor that is common
3 to each element of the set; for example, 3x is a common
(4, 2)
2 factor of the elements 9x2 and 12x
1 y-coordinate Commutative Law: a method of combining two numbers
–5 –4 –3 –2 –1–10 1 2 3 4 5x or algebraic expressions is commutative if the result of the
–2 combination does not depend on the order in which the
–3 objects are given. For example, the addition of 2 and 3 is
–4 commutative, since 2 + 3 = 3 + 2. However, subtraction is
–5 not commutative, since 2 − 3 ≠ 3 − 2.
Compass bearings: directions measured in degrees from
Cartesian plane: the area formed by a horizontal line with a the north–south line in either a clockwise or anticlockwise
scale (x-axis) joined to a vertical line with a scale (y-axis). direction. To write the compass bearing we need to state
The point of intersection of the lines is called the origin. whether the angle is measured from the north or south, the
Categorical (data): data that cannot be measured or size of the angle and whether the angle is measured in the
counted but can be categorised; for example, eye colour or direction of east or west; for example, N27°W, S32°E.
television programs Complement (of a set): the complement of a set, A, written
Census: collection of data from a population (e.g. all Year 10 A′, is the set of elements that are in ξ but not in A
students) rather than a sample Complementary angles: two angles that add to 90°; for
Centre (of circle): middle point of a circle, equidistant (equal example, 24° and 66° are complementary angles
in distance) from all points on its circumference Complementary events: events that have no common
Centre of enlargement: the point from which the elements and together make up the sample space. If A and
enlargement of an image is measured A′ are complementary events, then P(A) + P(A′) = 1.
Circle (equation): the general equation of a circle with Composite number: a number that has more than two
centre (0, 0) and radius r is: factors. For example, 6 is a composite number because it
x2 + y2 = r2. has factors 1, 2, 3 and 6.

642 Glossary
Composite figure: a figure made up of more than one Congruent triangles: there are five standard congruence tests
basic shape for triangles: SSS (side, side, side), SAS (side, included angle,
Compound graphs: column and bar graphs that display two or side), ASA (two angles and one side), AAS (two angles and a
more sets of data simultaneously. They are drawn with each non-included side) and RHS (right angle, hypotenuse, side)
column or bar representing combined sets of data. Individual Constant: a term or expression whose value does not vary
columns or bars are multicoloured, one colour for each set. Constant of proportionality (or variation) k: used to prove
Compound interest: the interest earned by investing a sum that a proportionality relationship (direct or inverse) exists
of money (the principal) when each successive interest between 2 or more variables (or quantities)
payment is added to the principal for the purpose of Continuous (data): numerical data that can take any value
calculating the next interest payment. The formula used within a certain range. They are generally associated with
for compound interest is: A = P(1 + R) n, where A is the measuring; for example, the heights of students.
amount to which the investment grows, P is the principal Convex polygons: polygons with no interior reflex angles
or initial amount invested, R is the interest rate per Coordinates (x, y): two numbers that give the position of
compounding period (as a decimal) and n is the number of a point on the Cartesian plane. The first number is the
compounding periods. The compound interest is calculated x-coordinate and the second number is the y-coordinate.
by subtracting the principal from the amount: CI = A − P. Correlation: a measure of the relationship between two
Compounded value: the value of the investment with variables. Correlation can be classified as linear, non-linear,
accrued interest included positive, negative, weak, moderate or strong.
Compounding period: the period of time over which interest Correlation coefficient r: the value of r indicates the
is calculated strength of the relationship between two variables. Its range
Concave polygons: polygons with at least one reflex interior is −1 ≤ r ≤ + 1, −1 being a strong negative relationship
angle and +1 being a strong positive relationship. The closer the
Concentration: a measure of the strength of a solution. The value of r is to 0, the less strong the relationship between
measured units can be, for example, g/mL. the variables.
Conditional probability: where the probability of an event is Corresponding angles: angles that are in corresponding
conditional (depends) on another event occurring first. For positions with respect to a transversal. On parallel lines,
two events A and B, the conditional probability of event B, corresponding angles are equal.
given that event A occurs, is denoted by P(B | A) and can be Q
calculated using the formula:
P(A ∩ B)
P(B | A) = , P(A) ≠ 0. G
P(A) D
Cone: A solid formed by taking a circular base and a point
not in the plane of the circle, called the vertex, which lies C F B
above or below the circle, and joining the vertex to each
point on the circumference of the circle. A

Radius r P
Cosine (cos) ratio: the ratio of the adjacent side to the
hypotenuse in a right-angled triangle.
adjacent
So, cos θ = .
Height h Slant height l hypotenuse

Hypotenuse
Opposite
Vertex θ
Adjacent
Congruent figures: figures that are identical; that is, they
have exactly the same shape and size Cross-section: the shape (plane section) produced when a
A D Dʹ Aʹ solid is cut through by a plane parallel to the base. For
example, the cross-section of a cone is a circle.
Cube: a polyhedron with 6 faces. All faces are squares of the
same size.
Cubic functions: the basic form of a cubic function is
B C Cʹ Bʹ y = ax3. These functions can have 1, 2 or 3 roots.

Glossary 643
Cumulative frequency: the total of all frequencies up to and Direct variation: describes a particular relationship between
including the frequency for a particular score in a frequency two variables (or quantities); that is, as one variable
distribution increases so does the other variable. The graph of the
Cumulative frequency curve: a line graph that is formed when relationship is a straight line, passing through the origin and
the cumulative frequencies of a set of data are plotted against the rule used to relate the two variables is y = kx.
the end points of their respective class intervals and then Direction: if the points in scatterplot show an upward pattern,
joined up by straight-line segments. It is also called an ogive. the directions is said to be positive, and if the pattern is
Cyclic quadrilateral: a quadrilateral that has all four vertices downward, the direction is negative.
on the circumference of a circle. That is, the quadrilateral is Discrete data: numerical data in which the information can
inscribed in the circle. take only certain exact values, usually whole numbers.
Cylinder: a solid that has parallel circular discs of equal They are associated with counting.
radius at the ends. The centres of the discs form the axis of Disjoint sets: these sets have no elements in common with
the cylinder. each other.
Axis Distance formula: the distance between two points
A (x1, y1) and B (x2, y2) is given by the formula
"(x2 − x1) 2 + (y2 − y1) 2.
Cross-section Distributive Law: the product of one number with the
is a circle sum of two others equals the sum of the products of
the first number with each of the others; for example
4(6 + 2) = 4 × 6 + 4 × 2. It is also applicable to algebra;
for example, 3x(x + 4) = 3x2 + 12x.
Dodecahedron: a regular polyhedron (platonic solid) with
Cylinder
12 faces, all of which are regular pentagons.
Data: various forms of information Dot plot: this graphical representation uses one dot to
Decimal number system: the base 10 place-value system represent a single observation. Dots are placed in columns
most commonly used for representing real numbers or rows, so that each column or row corresponds to a single
Degree (angle): a unit used to measure the size of an angle category or observation.
Degree (of a polynomial): the degree of a polynomial in x is
the highest power of x in the expression.
Denominator: the lower number of a fraction that represents the
number of equal fractional parts a whole has been divided into
Density: the ratio of mass to volume of a substance. It is
given by the formula: 0 1 2 3 4 5 6 7
mass
density = . Passengers
volume
It is measured in units such as g/cm3. Edges: straight lines where pairs of faces of a polyhedron meet
Dependent events: successive events in which one event Eighth Index Law: terms with fractional indices can be
1 n m n
affects the occurrence of the next written as surds. For example, an = "a and a n = "am.
Dependent variable: this variable is graphed on the y-axis. Element: an element of a set is a member of that set; for
Depreciation: the reduction in the value of an item as it ages example; 5 is an element of the set of counting numbers.
over a period of time. The formula used is A = P(1 − R) n, Elimination method: a method used to solve simultaneous
where A is the depreciated value of the item, P is its initial equations. This method combines the two equations into a
value, R is the percentage the item depreciates each year third equation involving only one of the variables.
(expressed as a decimal) and n is the number of years the Ellipse: a plane figure in the shape of an oval
item has depreciated. Enlargement (dilation): a scaled-up (or down) version of a
Diameter: the straight line from one point on the figure in which the transformed figure is in proportion to
circumference of a circle to another on the circumference, the original figure; that is, the two figures are similar
passing through the centre. Equally-likely outcomes: outcomes in a probability
Dilated (Quadratics): occurs when graphs are made thinner experiment that have the same chance of occurring
or wider Equating: the process of writing one expression as equal to
Dilation (Geometry): occurs when figures are made larger another
(enlarged) or smaller (reduced) in proportion Equation: a statement that asserts that two expressions are
Dilation factor: the scale factor by which a figure is enlarged equal in value. An equation must have an equal sign. For
or reduced example, x + 4 = 12.

644 Glossary
Equilateral triangle: a triangle with all sides equal in length, Flat rate loan: a loan in which the interest charged is simple
and all angles equal to 60° interest
Equivalent fractions: fractions that can be reduced to the FOIL: a diagrammatic method of expanding a pair of
same basic fraction; that is, fractions that have the same brackets. The letters in FOIL represent the order of the
value, for example, 13 = 26 = 39 = 12
4
expansion: First, Outer, Inner and Last.
Estimate: information about a population extrapolated from a Form: if the points in a scatterplot show an approximate
sample of the population linear pattern, it is described as having linear form.
Evaluate: determine a value for an expression Fourth Index Law: to remove brackets, multiply the indices
Event: a set of favourable outcomes in each trial of a inside the brackets by the index outside the brackets. Where
probability experiment no index is shown, assume that it is 1. So, (am) n = amn.
Event space: the set of all possible outcomes of an event a
Fraction: numbers represented in the form , where a and b
Expanding (algebra): this is the process of multiplying b
are whole numbers and b is not equal to zero.
everything inside the brackets by what is directly
Frequency: the number of times a particular score appears
outside the brackets. Expanding is the opposite of
factorising. Frequency polygon: a special type of line graph, which uses
the same scaled axes as the histogram. The midpoints of the
Expected frequency: the number of times a particular event
tops of the histogram columns are joined by straight line
is expected to occur when a chance experiment is repeated
a number of times intervals. The polygon is closed by drawing lines at each
end down to the score- or x-axis.
Experimental probability: the probability of an event based
on the outcomes of experiments, simulations or surveys Frequency table: a means of organising a large set of data.
Exponent: see Index It shows the number of scores (frequencies) that belong to
each group or class interval.
Exponential decay: a quantity that decreases by a constant
percentage in each fixed period of time. This growth can Function: a process that takes a set of x-values and
be modelled by exponential functions of the type y = kax, produces a related set of y-values. For each distinct
where 0 < a < 1. x-value, there is only one related y-value. They are usually
Exponential functions: relationships of the form y = ax, defined by a formula for f(x) in terms of x; for example,
where a ≠ 1, are called exponential functions with base a. f(x) = x2.
Exponential growth: a quantity that grows by a constant Future value: the future value of a loan or investment
percentage in each fixed period of time. This growth can Gradient (slope) m: this is a measure of the steepness of a line
be modelled by exponential functions of the type y = kax, or plane. The gradient of a line is given by
where a > 1. rise y2 − y1
m= =
Expression: this is a collection of two or more numbers run x2 − x1
or variables, connected by operations. For example, and is constant anywhere along that line.
12 − 2, 2a + 3b. Expressions do not contain an equal sign.
Faces: 2-dimensional, closed shapes that form the surfaces of y
a polyhedron B(x2, y2)
Factor: a factor of a given number is a whole number that
divides it exactly. For example, 1, 2, 3, 4, 6 and 12 are the y2 – y 1
factors of 12. (Rise)
Factorising: breaking down a number or expression into A(x1, y1)
smaller factors that can be numeric or algebraic. The process x 2 – x1
of factorising an algebraic expression involves changing it 0 (Run)
x
from a sum (or difference) into a product of factors.
Fifth Index Law: to remove brackets containing a product, Heron’s formula: this formula is used to find the area of
raise every part of the product to the index outside the a triangle when all three sides are known. The formula
brackets. So, (ab) m = ambm. is A = !s(s − a)(s − b)(s − c), where a, b and c are
Finite: a fixed number or amount. For example, the decimal the lengths of the sides and s is the semi-perimeter or
0.25 has a fixed number of decimal places. a+b+c
s= .
First Index Law: when terms with the same base are 2
multiplied, the indices are added. So, am × an = am+n. Histogram: a special type of column graph, in which no gaps
Five-number-summary: a method for summarising a data are left between columns and each column straddles an x-axis
set using five statistics: the minimum value, the lower score. The x-axis scale is continuous and usually a half-
quartile, the median, the upper quartile and the maximum interval is left before the first column and after the last
value column.

Glossary 645
y Intersection (of sets): region that represents the common
10 elements of two or more sets. A ∩ B denotes the intersection
8 of sets A and B.
Frequency

Inverse Law: when the additive inverse of a number or


6
pronumeral is added to itself, the sum is equal to 0. When
4 the multiplicative inverse of a number or pronumeral
is multiplied by itself, the product is equal to 1. So,
2 1
x + (−x) = 0 and x × = 1.
x
0 155 160 165 170 175 180 185 190 195 200 x Inverse operation: the operation that reverses the effect
Height of the original operation. Addition and subtraction are
inverse operations; multiplication and division are inverse
Horizontal: a line is said to be horizontal if it is parallel to operations.
the horizon of the Earth. Horizontal lines have a gradient of Inverse variation: describes a particular relationship between
zero and are parallel to the x-axis. two variables (or quantities); that is, as one variable
1
Hyperbola: The graph of y = is a rectangular hyperbola increases, the other decreases. The rule used to relate the
x k
with asymptotes on the x- and y-axes. two variables is y = .
x
Hypotenuse: the longest side of a right-angled triangle. It is Irrational numbers (I): numbers that cannot be written as
the side opposite the right angle. fractions. Examples of irrational numbers include surds,
Icosahedron: a regular polyhedron (platonic solid) with π and non-terminating, non-recurring decimals.
20 faces, all of which are equilateral triangles
Isometric drawing: a 2-dimensional representation of a
Identity Law: when 0 is added to an expression or the 3-dimensional shape in which vertical lines remain vertical,
expression is multiplied by 1, the value of the variable does horizontal lines are drawn at an angle and parallel edges
not change. For example, x + 0 = x and x × 1 = x.
remain parallel.
Image (similar figures): the enlarged (or reduced) figure
Isosceles triangle: a triangle with two sides equal in length
produced
Iterations: repeated calculations
Improper fraction: a fraction in which the numerator is
greater than the denominator Kite: a quadrilateral with two pairs of adjacent sides equal.
A kite may be convex or non-convex.
Independent events: successive events that have no effect on
each other
Independent variable: this is the x-axis (or horizontal)
variable
Index (power or exponent): the number expressing the power
to which a number or pronumeral is raised. For example, in
the expression 32, the index is 2. Plural: indices.
Inequality: a statement that one algebraic expression or one
number is greater than or less than another Lay-by: a method used to purchase an item whereby the
Inequality signs: signs used in inequations. They are purchaser makes regular payments to the retailer who
< (less than), > (greater than), ≤ (less than or equal to) retains the item until the complete price is paid off
and ≥ (greater than or equal to). Like terms: terms that contain exactly the same pronumeral
Inequations: similar to equations, but contain an inequality (letter) part; for example, 3ab and 7ab are like terms but 5a
sign instead of an equal sign. For example, x = 3 is an is not.
equation, but x < 3 is an inequation. Line of best fit: a straight line that best fits the data points
Infinite: never-ending; for example, the decimal 0.3 is non- of a scatterplot that appear to follow a linear trend. It is
terminating and therefore its number of decimal places positioned on the scatterplot so that there is approximately
cannot be counted. an equal number of data points on either side of the line,
Integers (Z): These include the positive and negative whole and so that all the points are as close to the line as
numbers, as well as zero; that is, .  .  ., −3, −2, −1, 0, 1, 2, .  .  . possible.
Intercepts: points where a curve crosses the x- or y-axis Line of vision: the straight line from an observation point to
Interquartile range: the difference between the upper the object being viewed
(or third) quartile, QU (or Q3), and the lower (or first) Line segment: a line segment or interval is a part of a line
quartile, QL (or Q1); that is, IQR = QU − QL = Q3 − Q1. with end points.
It is the range of approximately the middle half of A B
the data.

646 Glossary
Line symmetry: a figure has line symmetry if one or more Mixed number (numeral): a number that consists of a whole
lines (‘line of symmetry’ or ‘axis of symmetry’) can be number part and a fractional part, for example, 2 13
drawn that divide the figure into two mirror images. Minimum turning point: the lowest point of a parabola
Linear equation: an equation involving pronumerals of which is upright
degree 1. The general form of a linear equation in one Minutes (angle): units of angular measurement, where
variable is ax + b = 0. 1 degree (1°) = 60 minutes (60′)
Linear graphs: consist of an infinite number of points that Modal class: the term used when analysing grouped data. It
can be joined to form a straight line is the class interval with the highest frequency.
Linear modelling: applies the principle of linear equations to Mode: one measure of the centre of a set of data. It is the
represent practical situations score that occurs most often. There may be no mode (all
Lower (or first) quartile: the score that marks the end of the first scores occur once), one mode or more than one mode (two
quarter in an ordered set of data. It is denoted by QL or Q1. It is or more scores occur equally frequently).
calculated by finding the median of the lower half of the data. Moderate: describes the relationship if the points on the
Lowest common denominator (LCD): the lowest number scatterplot are reasonably close together and approximate a
that denominators of all fractions considered in a problem linear pattern
will divide equally into Monic: a monic pronumeral or monic expression is one
Lowest common multiple (LCM): the lowest multiple that in which the coefficient of the leading term is 1. For
two or more numbers have in common example, x2 + 4x − 3 is monic, while 6x2 + 4x − 3 is not.
Many-to-one correspondence: a function or mapping that takes Multiple: a number that is the product of a given number
the same value for at least two different elements of its domain and any whole number greater than zero. For example, the
Maximum turning point: the highest point of a parabola that multiples of 3 are 3, 6, 9, 12, 15, .  .  .
is inverted Multiplication Law of probability: if events A and B are
Mean: one measure of the centre of a set of data. It is given independent, then:
by mean =
sum of all scores Σx
or x = . When data are P(A and B) = P(A) × P(B) or P(A ∩ B) = P(A) × P(B).

a (f × x)
number of scores n Mutually exclusive events: events that cannot occur together.
On a Venn diagram, two mutually exclusive events will
presented in a frequency distribution table, x = .
n appear as disjoint sets.
Measures of central tendency: mean, median and mode Natural numbers: the set of positive integers, or counting
Measures of spread: range, interquartile range, standard numbers; that is, the set 1, 2, 3, .  .  .
deviation Negatively skewed: showing larger amounts of data as the
Median: one measure of the centre of a set of data. It is values of the data increase
the middle score for an odd number of scores arranged in Net: a 2-dimensional plan of a solid that can be cut out
numerical order. If there is an even number of scores, the and folded to form that solid. Below is the net of a cube.
median is the mean of the two middle scores when they are
n+1
ordered. Its location is determined by the rule . For
2
example, the median value of the set 1 3 3 4 5 6 8 9 9 is 5,
while the median value for the set 1 3 3 4 5 6 8 9 9 10 is
the mean of 5 and 6 (5.5).
Midpoint: the midpoint of a line segment is the point that
divides the segment into two equal parts. The coordinates
of the midpoint M between the two points P (x1, y1) and Nominal (data): a type of categorical data in which the
x + x2 y1 + y2 information is divided into subgroups; for example, eye
Q (x2, y2) is given by the formula a 1 , b. colour (hazel, blue, green)
2 2
y Non-recurring decimals: decimals that have no repeating
y2 Q (x2, y2)
digits or pattern; for example 5.482 786 2.  .  .
Non-terminating decimals: decimals that have an infinite
M (x, y) number of decimal places
y
T Null Factor Law: if a × b = 0, then either a = 0 or b = 0
or both a = 0 and b = 0; used when solving quadratic
y1 equations
P (x1, y1) S
Numerator: the upper number of a fraction that represents
the number of equal fractional parts
0 x1 x x2 x Numerical (data): data that can be measured or counted

Glossary 647
Object (similar figures): the original figure is classed as the the data sets. These boxplots compare the pulse rates of the
object same group of people before and after exercise.
Octahedron: a regular polyhedron (platonic solid) with

After
8 faces, all of which are equilateral triangles
Odds: relates to probabilities in gambling. They are given as

Time
ratios, such as 5−1, 51 or 5 : 1.
Ogive (cumulative frequency polygon): a graph formed

Before
by joining the top right-hand corners of the columns of a
cumulative frequency histogram
One-to-one correspondence: refers to the relationship 70 80 90 100 110 120 130 140 150
between two sets such that every element of the first set Pulse rate
corresponds to one and only one element of the second set Parallelogram: a quadrilateral with both pairs of opposite
Ordinal (data): a type of categorical data in which the sides parallel
information is in some type of ranked order; for example, A D
first, second, third, .  .  .
Order of rotational symmetry: the number of times a figure
coincides with its original position in turning through
one full rotation. For example, an equilateral triangle B C
has rotational symmetry of order three and a square has Payout: winnings paid, for example, on races. A payout
rotational symmetry of order four. is made on the ratio given by the odds, with the initial
Orthogonal drawing: a drawing that consists of the investment being returned plus the winning amount.
front view, the top view and the side views of an object Percentage: a fraction whose denominator is 100; for
28
Outcome: the result obtained when a probability experiment example, 28% = 100
is conducted Percentage cumulative frequency curve: a cumulative
Outlier: a piece of data that is considerably different from the frequency polygon expressed as a percentage of the total.
rest of the values in a set of data; for example, 24 is the outlier Most often this simply means adding a vertical percentage
in the set of ages {12, 12, 13, 13, 13, 13, 13, 14, 14, 24}. axis on the right-hand side of a cumulative frequency graph.
Percentile: the value below which a given percentage of all
scores lie. For example, the 20th percentile is the value
below which 20% of the scores in the set of data lie.
11 12 13 14 15 16 17 18 19 20 21 22 23 24 25
Perimeter: the distance around the boundary of a
Age
2-dimensional shape
Palindromic numbers: numbers that are the same if read Perpendicular lines: perpendicular lines are at right angles to
forwards or backwards, for example 33, 16  561 each other. The product of the gradients of two perpendicular
Parabola: the graph of a quadratic function has the shape lines is −1.
of a parabola. For example, the typical shape is that of the Pi (𝞹): the Greek letter π represents the ratio of the
graph of y = x2. circumference of any circle to its diameter. The number π
y is irrational, with an approximate value of 227
and a decimal
9 value of π = 3.141 59 . . ..
8
Pie chart (graph): see Sector graph
7
6 Platonic solids: five regular polyhedra, that is, five polyhedra
5 whose faces are regular congruent polygons: tetrahedron
4 (4 faces); cube (6 faces); octahedron (8 faces);
3 dodecahedron (12 faces); icosahedron (20 faces)
2 Plotting: placing points on a Cartesian plane, using their
1 coordinates
–4 –3 –2 –1–10 1 2 3 4x Polygon: a plane figure bounded by line segments
–2

Parallel: parallel lines in a plane never meet, no matter


how far they are extended
Parallel lines: lines that have the same gradient.
Parallel boxplots: two or more boxplots drawn on the same
scale to visually compare the five-number summaries of

648 Glossary
Polyhedron: a solid in which each face is a polygon; plural: squares on the other two sides. This is often expressed as
polyhedra c2 = a2 + b2.
Population: the whole group from which a sample is drawn A
Positively skewed: showing larger amounts of data as the
values of the data decrease
Power: see Index c b
Primary data: data collected by the user
Prime factor: a prime number that divides a given number
exactly; for example, the prime factors of 42 are 2, 3 and 7 B a C
Prime number: a number that has only two different
factors — itself and one. For example, 3 is a prime number Quadrant: a sector with an arc equal to a quarter of a circle
because its only factors are 1 and 3. Note that 1 is not a (and therefore centre angle of 90°)
prime number because its two factors 1 × 1 are the same. Quadrant 1: the quarter of the unit circle where the value of
Principal: an amount of money borrowed or invested the angle being considered is between 0° and 90°. That is,
Prism: A solid comprising two congruent parallel faces the x- and y-coordinates are both positive.
(bases) and the (lateral) faces that connect them. The lateral Quadrants of a Cartesian plane: four regions of the
faces are parallelograms. If they are all right-angled Cartesian plane produced by the intersection of the x- and
(i.e. rectangles) the prism is a right prism; if they are not all y-axes
right-angled, then the prism is an oblique prism. Quadratic equation: the general form of the quadratic
equation is ax2 + bx + c = 0.
Quadratic formula: gives the roots of the quadratic equation
−b ± "b2 − 4ac
ax2 + bx + c = 0. It is expressed as: x = .
2a
Quadratic trinomial: an algebraic expression that contains
Right Oblique Right three terms, in which the highest power of the pronumeral
rectangular rectangular triangular is a squared term; for example, 4x2 − 3x + 7
prism prism prism
Quantiles: percentiles expressed as decimals. For
Probability: the likelihood or chance of a particular event example, the 95th percentile is the same as the
(result) occurring. 0.95 quantile.
number of favourable outcomes Quantitative data: data that can be counted (discrete
P(event) = . data) or measured (continuous data), for example, the
number of possible outcomes
The probability of an event occurring ranges from 0 number of students enrolled in a school (discrete),
(impossible — will not occur) to 1 (certainty — will the heights in centimetres of the students in a class
definitely occur). (continuous)
Product: the result of a multiplication Quartiles: Values that divide an ordered set into
four (approximately) equal parts. There are three
Pronumerals: letters used in place of numbers
quartiles — the first (or lower) quartile Q1, the
Proof: an argument that shows why a statement is true second quartile (or median) Q2 and the third
Proportion: corresponding elements are in proportion if there (or upper) quartile Q3.
circumference Quotient: the result of dividing one number or algebraic
is a constant ratio; for example, = π for all
circles diameter expression by another
Pyramid: a group of solids with any polygon as the base. Radius: the straight line from a circle’s centre to any point on
Its other faces are triangles that meet at a common its circumference
vertex. Pyramids are named according to their base. Random number: a number whose value is governed by
For example, a pyramid with a square base is a square chance, and cannot be predicted in advance
pyramid. Range: the difference between the highest and lowest
scores in a set of data; that is,
range = highest score − lowest score
Rate: a particular kind of ratio where the two quantities
are measured in different units; for example, km/h, $/g
Square-based Triangular-based Hexagonal-based
pyramid pyramid pyramid
Ratio: the comparison of two or more quantities of the same
kind. A ratio has no units.
Pythagoras’ theorem: in any right-angled triangle, the Rational numbers (Q): numbers that can be written as
square on the hypotenuse is equal to the sum of the fractions, where the denominator is not zero

Glossary 649
Rationalising the denominator: a method used to Rhombus: a parallelogram with all sides equal
express the denominator as a rational number. Both the
numerator and denominator of a fraction are multiplied
by the surd (or conjugate surd) contained in the
denominator.
Real numbers (R): the set of all rational and irrational
numbers
Reciprocal: a number by which a given number is multiplied Right angle: the size of a right angle is 90°.
to result in 1
1 X
Rectangular hyperbola: the graph of y = is called
x
a rectangular hyperbola. The x- and y-axes are asymptotes.
A
y
O

1
y= x B

Rotation: turning a figure about a fixed point, called the


centre of rotation
0 x
Sample: part of a population chosen so as to give information
about the population as a whole
Sample space: a list of all the possible outcomes obtained
from a probability experiment. It is written as ξ or S, and
the list is enclosed in a pair of curled brackets 56 .
Sampling: obtaining data from a small group of subjects
Rectangular prism (cuboid): a 3-dimensional solid object (often people) within a larger population. This smaller
which has six rectangular faces and uniform cross-section group should be representative of the larger population.
Recurring decimals: These decimals have one or more digits Scale factor: the ratio of the corresponding sides in similar
repeated continuously; for example, 0.999.  .  . . They can be figures, where the enlarged (or reduced) figure is referred
expressed exactly by placing a dot or horizontal line over to as the image and the original figure is called the object.
the repeating digits; for example,
··
image length
scale factor =
8.343 434 = 8.3 4 or 8.34. object length
Reducible-interest-rate loan: a loan in which the Scalene triangle: a triangle with no two sides equal in
interest charged is compound interest, but the amount length
of the loan and the interest are repaid with regular Scatterplot: a graphical representation of bivariate data that
repayments displays the degree of correlation between two variables.
Reflected (Quadratics): one parabola is a mirror image of Each piece of data on a scatterplot is shown by a point. The
the other. x-coordinate of this point is the value of the independent
Reflection (Geometry): the image is a mirror image of the variable and the y-coordinate is the corresponding value of
object. the dependent variable.
Regular polygons: polygons with sides of the same length Scientific notation: a method of expressing a number as
and interior angles of the same size the product of a power of 10, and a decimal that has just
Relative frequency: represents the frequency of a particular one digit to the left of the decimal point; for example
score divided by the total sum of the frequencies. 54 267 would be written as 5.4267 × 104 and 0.005 426 7
It is given by the rule: as 5.4267 × 10−3.
frequency of the score Second Index Law: when terms with the same base are
relative frequency of a score = . divided, the indices are subtracted. So, am ÷ an = am−n.
total sum of frequencies
Secondary data: data collected by others
Revolution (angle): the size of a revolution is 360°.
Sector: part of a circle bounded by two radii and an arc
Sector graph: a type of graph mostly used to represent
categorical data. A circle is used to represent all the
360° data, with each category being represented by a sector of
the circle, whose size is proportional to the size of that
category compared to the total.

650 Glossary
Segment: a region of a circle between a chord and Slant height (of cone): the distance from any point on the
the circumference. The smaller segment is called the circumference of the circular base of the cone to the vertex
minor segment and the larger one is called the major of the cone
segment. Solid: a 3-dimensional object
Semicircle: part (half) of a circle bounded by a diameter and Speed: the rate that describes how quickly distance
an arc joining the ends of the diameter changes over a period of time. It is given by the formula
Set: a collection of similar elements distance
speed = .
Seventh Index Law: a term with a negative index can be time
1 Sphere: a sphere is a 3-dimensional object shaped like a ball
expressed with a positive index using this law. So, a−n = n
1 a Stem-and-leaf plot: a display that provides simultaneously
and −n = an. a rank order of individual scores and the shape of the
a
Similar figures: figures that have identical shape but different distribution. The stem is used to group the scores and the
size. The corresponding angles in similar figures are equal leaves indicate the individual scores within each group. The
in size, and the corresponding sides are in the same ratio, stem-and-leaf plot for the pulse rates of a group of students
called a scale factor. is shown.
Similar triangles: triangles that have similar shape but Pulse rate
different size. There are four standard tests to determine 6 8889
whether two triangles are similar: AAA (angle, angle, 7 0114668
angle), SAS (side, angle, side), SSS (side, side, side) and
RHS (right angle, hypotenuse side). 8 2688
Simple interest: the interest accumulated when the interest 9 06
payment in each period is a fixed fraction of the principal. 10 4
P×r×T
The formula used is I = , where I is the interest 11 0
100
earned (in $) when a principal of $P is invested at an A back-to-back stem-and-leaf plot has two sets of data
interest rate of r% p.a. for a period of T years. displayed — one on either side of the common stem.
Simple random sampling: a survey that ensures all Stratified random sampling: sampling of a population that
subjects have an equal chance of inclusion in the consists of identifiable groups or strata, so that each group
sample receives fair representation. Each group should make up
Simplify: to write an expression in its simplest form by the the same proportion of the sample as it does of the full
use of algebraic or arithmetical techniques population.
Simultaneous equations (linear): two (or more) linear Strength: the strength of a linear relationship is an indication
graphs that have the same solution of how closely the points on a scatterplot fit a straight line.
Simultaneous system of equations: a set of equations that Strong: describes the relationship if the points on the
are all satisfied by the same values of the variables scatterplot are close together and show a definite linear
Sine (sin) ratio: the ratio of the opposite side to the hypotenuse pattern
opposite Subjective probability: probability that is based on one or
in a right-angled triangle. So, sin θ = .
hypotenuse more of the following: judgements, opinions, assessments,
Sixth Index Law: to remove brackets containing a fraction, estimations and conjectures by individuals. It may also
multiply the indices of both the numerator and denominator involve beliefs, emotions and bias.
a m am Subset: a smaller set within another set. It is denoted by the
by the index outside the brackets. So, a b = m.
b b symbol ⊂.
Skewed: if a distribution's shape is not symmetric, it can Substitution: the replacement of a variable by a number.
be described as being positively skewed (tailing off to the For example, substituting x = 2 in the expression 5x gives
upper end of the distribution) or negatively skewed (tailing 5 × 2 = 10.
off to the lower end of the distribution).
Substitution method: a method used to solve simultaneous
Negative skew equations. It is useful when one (or both) of the equations
has one of the variables as the subject.
Shape

Positive skew Subtended (angle): an angle standing on an arc of a circle,


with its vertex on the circumference or at the centre of
the circle
Symmetric
Successive discount: a discount applied after an initial
70 90 110 130 150 discount has been applied
Values Supplementary (angles): angles that add to 180°

Glossary 651
Summary statistics: measures such as mean, mode, median Travel graphs: graphs that represent the relationship
and range, used in analysing a set of data between distance and time. The distance covered is shown
Surds: roots of numbers that do not have an exact answer, on the vertical axis and the time taken on the horizontal axis.
so they are irrational numbers. Surds themselves are exact Tree diagrams: branching diagrams that list all the possible
3
numbers; for example, !6 or " 5. outcomes of a probability experiment. This diagram shows
Symmetrical: the identical size, shape and arrangement the outcomes when a coin is tossed twice.
of parts of an object on opposite sides of a line or plane Head, Head
Tangent (tan) ratio: the ratio of the opposite side Head
to the adjacent side in a right-angled triangle. So,
opposite
tan θ = . Head Tail
adjacent Head, Tail
Terminating decimals: decimals that have a fixed number of
Tail, Head
places; for example, 0.6 and 2.54 Tail Head
Tetrahedron: a regular polyhedron (platonic solid) with
4 faces, all of which are equilateral triangles
Theorems: rules or laws Tail
Tail, Tail
Theoretical probability: given by the rule
number of favourable outcomes Trend line: the line of best fit that is drawn on a time series
P(event = or
number of possible outcomes graph, which is used to forecast future values
n(E) Trial: the number of times a probability experiment is
P(E) = , conducted
n(S)
Triangle: a 3-sided polygon
where n(E) = number of times or ways an event, E, can
Trigonometric ratios: three different ratios of one side of a
occur and n(S) = number of elements in the sample space
triangle to another. The three ratios are the sine, cosine and
or number of ways all outcomes can occur, given all the tangent ratios.
outcomes are equally likely
Trinomial: an expression consisting of three terms; for
Third Index Law: any term (excluding 0) with an index of example, x2 + 3x − 5
0 is equal to 1. So, a0 = 1. True bearings: directions that are written as the number
Total surface area (TSA): the area of the outside surface of of degrees (3 digits) from north in a clockwise direction,
a 3-dimensional figure followed by the word true or T; for example, due east
Transformations (Quadratics): changes that occur to the would be 090° true or 090°T
basic parabola y = x2 in order to obtain another graph. Truncated cone: a cone with its top cut off
Examples of transformations are translations, reflections or Turning point: the point at which the graph of a
dilations. quadratic function (parabola) changes direction (either
Translated (Quadratics): moving a parabola horizontally up or down)
(left/right) or vertically (up/down) Two-step chance experiment: a probability experiment that
Translation (Geometry): occurs when an object moves involves two trials
up, down, left or right without flipping, turning or Two-way tables: a table that lists all the possible outcomes
changing size of a probability experiment in a logical manner
Transposing: changing the order of terms in an equation
Transversal: a line that meets two or more other lines in Hair colour Hair type Total
a plane Red 1 1 2

Brown 8 4 12
Transversal
Blonde 1 3 4

Black 7 2 9

Trapezium: a quadrilateral with only one pair of parallel Total 17 10 27


sides
Uniform cross-section: a solid has a uniform cross-section if
cross-sections taken parallel to its base are always the same
size and shape. Cross-sections parallel to the base of prisms
are uniform, whereas cross-sections parallel to the base of
pyramids are not.

652 Glossary
Union (of sets): represents the combination of elements of Vertical: vertical lines are parallel to the y-axis and have an
two or more sets. A ∪ B denotes the union of sets A and B. undefined (infinite) gradient.
Unit circle: a circle with its centre at the origin and having a Vertically opposite angles: when two lines intersect, four
radius of 1 unit angles are formed at the point of intersection, and two pairs
Unit fraction: a fraction that has a numerator of one; for of vertically opposite angles result. Vertically opposite
example, 12, 15, 10
1
angles are equal.
Univariate data: data relating to a single variable Y B
Universal set (𝝃): the largest set that contains all possible
elements of the data considered O
Upper quartile: the score that marks the end of the third quarter
in an ordered set of data. It is denoted by QU or Q3. It is
calculated by finding the median of the upper half of the data. A X
Venn diagrams: a series of circles, representing sets, within Volume: the amount of space a 3-dimensional object
a rectangle, which represents the universal set. They show occupies. The units used are cubic units, such as cubic
the relationships between the sets. centimetres (cm3) and cubic metres (m3).
Weak: describes the relationship if the points on the
scatterplot are far apart
A B Wedge: a piece of wood, metal etc., thick at one end,
tapering to a thin edge.
x-intercept: the point where a graph intersects the x-axis
y-intercept: the point where a graph intersects the
Vertex: plural: vertices; a point where two or more sides of y-axis. In the equation of a straight line, y = mx + c,
a polygon or edges of a solid meet. For example, a square the constant term, c, represents the y-intercept of
has 4 vertices and a cube has 8 vertices. that line.

Glossary 653
Index
Addition Law of probability 394 Closure Law 29–30
algebra and equations coincident lines 111
adding and subtracting algebraic fractions 33–6 collinear points 84
multiplying and dividing algebraic fractions 36–9 column graphs
solving simple equations 40–5 drawing from data tables 496
substitution 28–33 using to create scatterplots 497–9
algebraic fractions Commutative Law 29
adding 33–6 compass bearings 177
dividing 37–8 complementary events 393
equations involving 47–8 composite figures, area 202–3
multiplying 36–7 composite solids
multiplying by the reciprocal 37 total surface area 212–14
pronumerals in the denominator 34 volume 221–2
simplifying 36 compound interest 588–95
subtracting 33–6 calculating 589–90
angle of depression 172–6 compounded value 588
angle of elevation 172–6, 190–1 compounding period 591–2
angles fixed principal at various interest rates
at a point 324 over time 591
parallel lines and transversals 325 future value 588
supplementary angles 324 guess and refine 592–5
vertically opposite angles 324 using technology 590–1
area see surface area; total surface area compounded value 588
Associative Law 29 compounding period 591–2
computer symbols 96–7
back-to-back stem-and-leaf plots 468–75 concave polygons 349
base (index form) 4 conditional probability 418–21
base two (binary) system 20 cone
bearings 176–82 total surface area 210–1
binary system, difference to decimal system 20–1 volume 220
binomial expansion, quadratic expressions 240, 241–2 congruent triangles, tests of congruency
bits 20 326–7
bivariate data continuous data 492
definition 492 convex polygons 349
drawing column graphs from data tables 496 coordinate geometry
graphing 496–502 determining linear equations 71–6
scatterplots and linear modelling 502–10 distance between two points 77–9
using column graphs to create scatterplots 497–9 midpoint of line segment 80–3
variables 492–5 parallel and perpendicular lines 83–91
box-and-whisker plots sketching linear graphs 64–70
boxplots and dot plots 461–2 correlation 502
boxplots and histograms 462 cosine ratio 158
describing distributions 459–61 credit card purchases 577
description of 459 cricket scores
five-number summary 458 batting averages 480–1
identification of extreme values 459 bowling averages 481
negatively skewed plots 460 cube
parallel boxplots 463–4 total surface area 209
positively skewed plots 460 volume 217
shapes of graphs 461–2 cuboid (rectangular prism)
symmetrical plots 459 total surface area 209
symmetry and skewness 459–61 volume 218
business expenses, documenting 138–9 cumulative frequency curves (ogives) 445
buying on terms 579–84 cylinder
bytes 20 total surface area 209
volume 218
capacity (volume) 222–3
cash payments 577 data
celebrity squares and doubles 260–1 classification of 492
census 526, 528 collecting, recording and analysing over time
centre of enlargement 334 516–17
circles primary data 531–5
area 200 secondary data 535–7
non-linear relationships 305–7 see also bivariate data; univariate data

654 Index
data analysis First Index Law 4
choice of methods 542 FOIL method, expanding quadratic expressions
statistical graphs 542–5 240–1
statistical investigations 553, 554 Fourth Index Law 5
statistical reports 545–8 frequency distribution tables, calculating mean,
data collection methods 540–2 median and mode 443–4
primary data 531–5, 540 future value 588
secondary data 535–7, 540
data entry errors, checking for 54–5 geometry see coordinate geometry; deductive
data organisation, in statistical investigations geometry
552–3, 554 gigabytes (GB) 20
data sets, comparing 468–75 gradient 64–8, 190–1
decimal system, difference to binary system 20–1 gradient–intercept method 66–7
deductive geometry
angles, triangles and congruence 324–33 histograms 462
polygons 349–52 horizontal lines 87–8
quadrilaterals 340–8 hypotenuse 147
similar triangles 333–9
dependent variables 492 Identity Law 29
depreciation 596–9 independent variables 492
depression, angle of 172–6 index form 4
dice game 430–1 index laws
discrete data 492 combining 13–17
dot plots 461–2 Fifth Index Law 5
First Index Law 4
elevation, angle of 172–6, 190–1 Fourth Index Law 5
elimination method, solving simultaneous linear review 4–8
equations 117–20 Second Index Law 4
ellipse, area 201 Seventh Index Law 9
enlargements 358–9 Sixth Index Law 5
similar figures 334 Third Index Law 5
equations indices
and inequalities 127 combining index laws 13–17
nature of 40 index laws 4–8
system of equations 108 negative indices 9–13
see also linear equations; multi-step equations; inequalities
simple equations between two expressions 127
equilateral triangles, angle properties 325 solving 128–32
Euclidean geometry 324 inequations 127
event space 390 interquartile range (IQR) 453–4
expanding algebraic expressions determining from a graph 454–5
binomial expansion 240 Inverse Law 29
difference of two squares 242 isosceles triangles, angle properties 327–9
FOIL method 240–1
square of binomial 241–2 kilobytes (KB) 20
experimental probability 390 kite, area 201
experiments, primary data collection
532–3 lay-by payments 577
exponential functions line of best fit
combining transformations 302 drawing by eye 504–7
comparing graphs 300 and relationship between variables 503
form 299 line segments, midpoint 80–3
non-linear relationships 299–305 linear equations
plotting 300 determining 71–6
reflection about x-axis 301 finding given two points on line 71–3
reflection about y-axis 301 finding using gradient and one
vertical translation 300 point 73–4
see also simultaneous linear equations
Fifth Index Law 5 linear graphs
financial mathematics plotting 64
buying on terms 579–84 sketching 64–70
compound interest 588–95 sketching straight lines 64–7
consumer price index 606–7 where y = b and x = a 68–9
depreciation 596–9 where y = mx 67–8
loan repayments 599–602 linear inequalities
purchasing goods 576–9 between two expressions 127
successive discounts 585–8 solving 128–32

Index 655
loan repayments 599–602 translation: y = x² + c 295–6
flat rate loans 599 in world around us 288–9
reducible-interest-rate loans 599–601 parallel boxplots 463–4
loans 581 parallel lines 83
determining equation of line 86
mean 442 and simultaneous linear equations 111
in frequency distribution tables 443–4 and transversals 325
of grouped data 444–5 parallelograms
measurement angle properties 340–2, 344
primary data collection 532 area 200
significant figures 233 parametric equations 310–11
measurement errors 232–3 patterns, repetitive patterns 280–1
in calculations 233 percentiles 445, 446
and rounding 233 perpendicular lines 84–5
measures of central tendency 442–52 determining equation of line 86
cumulative frequency curves (ogives) 445 and simultaneous linear equations 112
frequency distribution tables 443–4 polygons
grouped data 444–5 concave polygons 349
mean 442 convex polygons 349
median 442 exterior angles 350
mode 442 interior angles 349
quantiles 445 regular polygons 349
measures of spread 453–8 populations 526, 564–5
determining IQR from graph 454–5 primary data 531
interquartile range (IQR) 453–4 in statistical investigations 552–3
range 453 primary data collection
media reports, investigating 554–6 experiments 532–3
median 442–3 measurement 532
in frequency distribution tables 443–4 methods 531
of grouped data 445 observation 531–2
megabytes (MB) 20 simulation 533–5
mid-point theorem, quadrilaterals 343–4 in statistical investigations 552
midpoint of line segment 80–3 surveys 532
modal class, of grouped data 445 prisms, volume 217–19
mode 442–3 probability
in frequency distribution tables 443–4 Addition Law of probability 394
monic quadratic trinomials, factorising 245–9 complementary events 393
multi-step equations conditional probability 418–21
involving algebraic fractions 47–8 definition 390
with multiple brackets 46 dependent events 413–14
solving 46–50 dice game 430–1
Multiplication Law of probability 412 event or sample space 390
mutually exclusive events 393 experimental probability 390
independent events 412–13
negative indices 9–13 of intersecting events 394–400
nominal data 492 Multiplication Law of probability 412
non-linear relationships mutually exclusive events 393
exponential functions and graphs 299–305 odds 400
parametric equations 310–11 payouts 400–1
plotting parabolas 288–93 relative frequency 390
sketching parabolas 294–9 review 390–406
Null Factor Law 268–9 subjective probability 422–3
number laws 28 terminology 390–1
theoretical probability 392
observation, primary data collection 531–2 three-step chance experiments 408–9
odds 400 tree diagrams 406–12
ogives 445 two-step chance experiments 406–8
one-step equations 40–1 two-way tables 391–2
ordinal data 492 proofs 324
purchasing goods
parabolas buying on terms 579–84
combining transformations 296 payment options 576–9
dilation: y = ax² 294 pyramids, volume 220–1
plotting 288–93 Pythagoras’ theorem
reflection: y = ax² and a is negative number finding distance between two points 77
294–5 finding the hypotenuse 147
sketching 294–9 finding shorter side 148–9

656 Index
Pythagoras’ theorem (continued) and linear modelling 502–10
review 147 making predictions 503
similar right-angled triangles 146–7 Second Index Law 4
in three dimensions 153–7 secondary data 535
collection 553
quadratic equations organisation 554
confirming solutions 274–5 sources 535–7
form 268 in statistical investigations 553–4
with no solutions 274 sector, area 201
Null Factor Law 268–9 Seventh Index Law 9
with only one solution 273 similar figures
solving algebraically 268–72 enlargements 334, 358–9
solving graphically 273–7 image 334
solving problems 269–70 nature of 333–4
quadratic expressions similar triangles, tests for similarity 334–6
binomial expansion 240 simple equations
difference of two squares 242 one-step equations 40–1
expanding algebraic expressions 240–5 with pronumeral on both sides 41–2
factorising with four terms 250–1 solving 40–5
factorising monic quadratic trinomials two-step equations 41
245–9 simple interest 599
factorising with three terms 245–9 simulation, primary data collection 533–5
factorising with two terms 249–50 simultaneous linear equations
FOIL method of expanding 240–1 applications 123–6
mixed factorisation 253–5 coincident lines 111
square of binomial 241–2 graphical solution 108–14
quadrilaterals with multiple solutions 111
angle properties 340–8 nature of 108
mid-point theorem 343–4 with no solutions 111
relationships between 345 parallel lines 111
sum of interior angles 340 perpendicular lines 112
qualitative data 492 solving linear inequalities 127–32
quantiles 445 solving using elimination 117–22
quantitative data 492 solving using substitution 115–17
quartiles 445–6 sine ratio 158
Sixth Index Law 5
range 453 sphere
reciprocals, multiplying by 37 total surface area 209
rectangles volume 220
angle properties 342, 345 square
area 200 angle properties 345
rectangular prism area 200
total surface area 209 statistical graphs, analysis 542–5
volume 218 statistical investigations
reducible-interest-rate loans 599–601 reports 553, 554
relative frequency 390 using primary data 552–3
repetitive patterns 280–1 using secondary data 553–4
reports, statistical investigations 553, 554 statistical reports, analysis 545–8
rhombus statistics in the media
angle properties 343, 344 evaluating inquiry methods and statistical reports 540–51
surface area 201 populations and samples 526–31, 564–5
right-angled triangles primary and secondary data 531–40
calculating angle size 167–72 statistical investigations 552–8
calculating side lengths 163–6 straight lines
finding the hypotenuse 147 sketching using gradient–intercept method 66–7
finding shorter side 148–9 sketching using x- and y-intercepts 65–6
names of sides 158 subjective probability 422–3
and Pythagoras’ theorem 146–7 substitution
rounding, and measurement errors 233 Associative Law 29
Closure Law 29–30
sample space 390 Identity Law 29
samples 526–31, 564–5 Inverse Law 29
scale factor 334 number laws 28
scatterplots substitution method, solving simultaneous linear
created from column graphs 497–9 equations 115–17
described 502 successive discounts 585–8
line of best fit 503–7 supplementary angles 324

Index 657
surface area sine ratio 158
circle 200 tangent ratio 158
composite figures 202–3 trigonometry
definition 200 angles of elevation and depression 172–6
ellipse 201 applications 182–5
formulas 200–1 bearings 176–82
Heron’s formula 201 calculating angle size 167–72
kite (including rhombus) 201 calculating side lengths 163–6
measurement errors 233 Pythagoras’ theorem 146–53
parallelogram 200 Pythagoras’ theorem in three dimensions 153–7
rectangle 200 trigonometric ratios 158–62
sector 201 true bearings 177
square 200 two-step chance experiments 406–8
trapezium 200 two-step equations 41
triangle 200, 201 two-way tables 391–2
see also total surface area (TSA)
surveys, primary data collection 532 univariate data
box-and-whisker plots 458–68
tangent ratio 158 comparing data sets 468–75
terabytes (TB) 20 definition 492
terms, buying on 579–84 measures of central tendency 442–52
theorems, definition 324 measures of spread 453–8, 542–57
theoretical probability 392
Third Index Law 5 variables
three-step chance experiments 408–9 correlation 502
time series 516–17 definition 492
total surface area (TSA) dependent variables 492
composite solids 212–14 identifying a relationship 493–4
cone 210 independent variables 492
cylinder 209 vertical lines 87–8
measurement errors 233 vertically opposite angles 324
other solids 211–12 volume
rectangular prisms or cuboids 209 capacity 222–3
sphere 209 composite solids 221–2
transposition errors 54–5 cone 220
transversals 325 cube 218
trapezium cylinder 218
angle properties 344 definition 217
area 200 measurement errors 233
triangles prisms 217–19
angle properties 325–9 pyramid 220–1
area 200, 201 rectangular prism 218
sum of interior angles 325–6 sphere 220
triangular prism, volume 218 triangular prism 218
trigonometric ratios
cosine ratio 158 weaving, repetitive patterns 280–1
finding values 158–60
naming sides in right-angled triangle 158 x- and y-intercept method 65–6

658 Index

You might also like